You are on page 1of 1334

Ultra Practice Bundle PDF

Reasoning Questions for SBI Clerk/ RBI Assist. Mains

Page 1 of 1334

Get a Special Paid Bundle PDF on High Level Puzzles & Seating for All Bank Clerk/ PO Mains

If there are any suggestions/ errors in our PDFs Feel Free to contact us via this email: admin@exampundit.in
Ultra Practice Bundle PDF
Reasoning Questions for SBI Clerk/ RBI Assist. Mains
S. No Title Page No
1 Seating Linear 2 Variable 04

2 Seating Circle 2 Variable 23


3 Seating Parallel Row 2 Variable 47
4 Seating Square 2 Variable 74
5 Seating Rectangle 2 Variable 103
6 Seating Triangle 2 Variable 130
7 Seating Linear 3 Variable 147
8 Seating Circle 3 Variable 189
9 Seating Square 3 Variable 219
10 Seating Rectangle / Square New Pattern 254
11 Puzzle Box 280
12 Puzzle Floor 312
13 Puzzle Floor with Flat 348
14 Puzzle Month 380
15 Puzzle Month and Date 409
16 Puzzle Days 447
17 Puzzle Year 486
18 Puzzle Random 521
19 Puzzle Profession 560
20 Puzzle Timing Based 591
21 Puzzle Scheduling 621
22 Puzzle Age 652
23 Seating with Ages 689
24 Seating Linear with Blood Relation 720
25 Seating Circle with Blood Relation 743
26 Seating Linear with Distance 768

Page 2 of 1334
Get a Special Paid Bundle PDF on High Level Puzzles & Seating for All Bank Clerk/ PO Mains
If there are any suggestions/ errors in our PDFs Feel Free to contact us via this email: admin@exampundit.in
Ultra Practice Bundle PDF
Reasoning Questions for SBI Clerk/ RBI Assist. Mains
27 Seating Unknown Number of Person 793
28 Seating 3 Parallel Row 826
29 Seating Concentric Diagrams 854
30 Seating Movement Based 882
31 Puzzle Income Based 928
32 Inequality 945
33 Syllogism 959
34 Coded Direction Part I 973
35 Coded Blood Relation Part 985
36 Alpha Numeric NP 1001
37 Alphabet Series 1019
38 Coding Decoding Number Symbol Based Condition 1033
39 Coding Decoding NP 1055
40 Coding Decoding Number Condition Based 1068
41 Coding Decoding Symbol Alphabet Based 1093
42 Coding Decoding Box Type 1125
43 Input Output Part 1 1138
44 Input Output Part 2 1154
45 Input Output Box Type 1174
46 Critical Reasoning 1201
47 Data Sufficiency 1233
48 Coding Decoding with Input Output 1277
49 Coding Decoding New Pattern 2 1302
50 Coded Direction 1313

Page 3 of 1334
Get a Special Paid Bundle PDF on High Level Puzzles & Seating for All Bank Clerk/ PO Mains
If there are any suggestions/ errors in our PDFs Feel Free to contact us via this email: admin@exampundit.in
Ultra Practice Bundle PDF
Reasoning Questions for SBI Clerk/ RBI Assist. Mains
Seating Linear 2 Variable
Directions (1-5): Answer the questions based on the (b) Immediate left
information given below: (c) Third to right
There are eight friends S, T, U, V, W, X, Y and Z are (d) Second to left
sitting in linear table and all are facing towards the south. (e) Immediate right
All friends have cleared different examinations i.e.RRB, 3. Who sits immediate left of X?
LIC, RBI, IBPS, SEBI, SSC, UPSC and SBI but not (a) S
necessarily in the same order. (b) Y
There are three friends sit between S and V. Z has cleared (c) U
RRB. The friends who have cleared RBI and SEBI sit (d) Z
together. S has cleared SEBI. Y sits immediate left of one (e) W
who cleared IBPS. U and W sit together but none of them 4. Who sits third to the left of the one, who has
cleared RBI. The one who cleared UPSC is immediate cleared LIC?
right of the one who cleared SBI.V does not cleared SBI. (a) W
The one, who sits at extreme end, has cleared RRB. V sits (b) Z
third to the right of the one who cleared RRB, who is an (c) S
immediate neighbor of Y. W has not cleared UPSC. X (d) X
does not sit near to U. The immediate right of the one who (e) T
sits at extreme end has cleared SSC. 5. Usits second to the left of the one who has cleared
1. What is the position of U with respect to the one, which among the following exam?
who has cleared RBI? (a) RBI
(a) Third to right (b) SBI
(b) Second to left (c) IBPS
(c) Third to right (d) SEBI
(d) Immediate left (e) None of these
(e) Cannot be determined Directions (6-10) : Answer the questions based on the
2. What is the position of T with respect to W? information given below:
(a) Second to right

Page 4 of 1334
Get a Special Paid Bundle PDF on High Level Puzzles & Seating for All Bank Clerk/ PO Mains
If there are any suggestions/ errors in our PDFs Feel Free to contact us via this email: admin@exampundit.in
Ultra Practice Bundle PDF
Reasoning Questions for SBI Clerk/ RBI Assist. Mains
There are eight Friends P, Q, R, S, T, U, V and Ware (e) Immediate right
sitting in linear table. Five of them are facing north and 8. Who sits second to left of W?
three are facing south. Their ages are in years i.e. 35, 38, (a) Q
39, 42, 44, 46, 48 and 50 but not necessarily in the same (b) T
order. (c) R
Q faces south. V who is 46 years old is the immediate (d) V
neighbor of the persons, who are facing north. There are (e) Cannot be determined
four people between P and R. None of the people who face 9. Who is sitting third to the left of the one, whose
north is 46, or 42 or 39 years old. W is 50 years old. T sits age is 50 years old?
fifth to the right of V. P and S are sitting at the extreme (a) S
ends and neither of them is 48 or 38 years old. T faces the (b) Q
direction opposite to that of W. V sits third to the right of (c) T
U. There are as many people sit between T and W is same (d) U
as the number of person sits to the right of R.The person (e) P
who is 35 years old is sitting third to the left of the one 10. P sits immediate right to the one whose age is?
who is 48 years old. Q who is 42 years old is sitting second (a) 48
to the left of W. V is not an immediate neighbor of P. (b)50
6. What is the position of S with respect to U? (c) 39
(a) Third to right (d) 46
(b) Second to left (e) 44
(c) Third to right Directions (11-15) : Answer the questions based on the
(d) Fourth to right information given below:
(e) Cannot be determined Nine persons P, Q, R, S, T, U, V, W and X are sitting in a
7. What is the position of Q with respect to the one, linear table and facing the north direction but not
whose is 39 years old? necessarily in the same order. Each one of them has
(a) Second to right different brands watches i.e. Titan, Fastrack, Fossil,
(b) Immediate left Diesel, Giordano, Rado, Maxima, Armani and Tissot but
(c) Third to right not necessarily in the same order.
(d) Second to left
Page 5 of 1334
Get a Special Paid Bundle PDF on High Level Puzzles & Seating for All Bank Clerk/ PO Mains
If there are any suggestions/ errors in our PDFs Feel Free to contact us via this email: admin@exampundit.in
Ultra Practice Bundle PDF
Reasoning Questions for SBI Clerk/ RBI Assist. Mains
Two persons sit between X and V who does not have (d) Maxima
Fossil. W sits just to the right of R. One person sits (e) None of these
between W and P. R have Fossil. Two persons sit between 14. Who is sitting second to the left of the one, who
the one who have Giordano and the one who have Armani. have Rado?
Q have Giordano but he does not sit just to the right of (a) O
W.Three persons sit between X and the one who have (b) J
Diesel. X sits on an even position from the left end. The (c) Q
one who, have Fossil sits just to the left of the one who (d) L
have Diesel. The one who have the Titan’s watch sits just (e) None of these
to the right of T. V does not have Titan’s watch. S has 15. How many persons sit between P and U?
Fastrack and T has Tissot’s watch. The one who have (a) One
Maxima sits immediate left to the V. (b) Four
11. Who sits to the immediate left of X? (c)Three
(a) R (d) Six
(b) T (e) None of these
(c) Q
(d) V Directions (16-20) : Answer the questions based on the
(e) None of these information given below:
12. Who sits second to the left of the one, who have There are seven persons P, Q, R, S, T, U and V sit ina
Tissot? linear table. Only two persons are facing south direction
(a) U and others are facing north direction. They were born in
(b) P different years i.e. 1995, 1996 and 1997 but not
(c) Q necessarily in the same order. Two of them born in 1995,
(d) S two of them born in 1996 and three of them born in 1997.
(e) None of these The one who have born in same year do not sit together.
13. R is sitting third to the left the one, who have? Q faces north direction and sits third to right of S. There
(a) Armani are three persons sit between P and R. V was does not born
(b) Diesel in 1996 and sits third to the right of Q. U sits immediate
(c) Tissot right of R and does not born in 1996. Q does not born in
Page 6 of 1334
Get a Special Paid Bundle PDF on High Level Puzzles & Seating for All Bank Clerk/ PO Mains
If there are any suggestions/ errors in our PDFs Feel Free to contact us via this email: admin@exampundit.in
Ultra Practice Bundle PDF
Reasoning Questions for SBI Clerk/ RBI Assist. Mains
1995. P and R face the same direction. U and V facing 20. Who among the following are the second oldest
same direction as S. Pdoes not born in 1996.P sits people?
immediate left of T who is an immediate neighbor of Q. T (a) P and Q
does not born in 1995. S borns in 1995 sits at one of the (b)Q and V
extreme end of the table. (c) T and R
16. What is the position of R with respect to T? (d) U and P
(a) Third to right (e) None of these
(b) Second to left Directions (21-25): Answer the questions based on the
(c) Third to right information given below:
(d) Fourth to right There are seven persons J, K, L, M, N, O and P are sitting
(e) Cannot be determined in a linear table. Some of them face north and some of
17. What is the position of S with respect to P? them face south direction. All of them like different
(a) Second to right colours i.e Red, Blue, Black, Yellow, Green, White and
(b) Immediate left Pink but not necessarily in the same order.
(c) Third to right O sits immediate right of P who does not sit immediate to
(d) Second to left K. The one who likes Blue and Pink are immediate
(e) Immediate right neighbors of N. The one who likes Yellow sits immediate
18. Who sits third to right of Q? to the left of who likes Black. The persons who sitting at
(a) R extreme end facing opposite direction to each other.O
(b) V does not facing south direction. M is an immediate
(c) U neighbor of J and likes Red. There are more than two
(d) P persons sit between J and L. L does not like Green and
(e) T Pink. J and K are facing opposite direction to M. N does
19. Who sits second to the right of T? not face south direction. J sits second from extreme end
(a) V and likes white. Three persons sit between J and the
(b) R person who likes Green. K sits second to the right of L.
(c) S 21. Who sits immediate left of who likes Pink?
(d) U (a) M
(e) P (b) P
Page 7 of 1334
Get a Special Paid Bundle PDF on High Level Puzzles & Seating for All Bank Clerk/ PO Mains
If there are any suggestions/ errors in our PDFs Feel Free to contact us via this email: admin@exampundit.in
Ultra Practice Bundle PDF
Reasoning Questions for SBI Clerk/ RBI Assist. Mains
(c) O Directions (26-30) : Answer the questions based on the
(d) J information given below:
(e) N There are eight people A, B, C, D, E, F, G and H are sitting
22. Who sits second to the left of O? in a linear table facing north direction. They all belong to
(a) N different cities i.e. Noida, Patna, Jhansi, Indore, Lucknow,
(b) P Madurai, Jaipur and Bhopal but not necessarily in the
(c) J same order.
(d) L Two people are sitting between C and the one who
(e) None of these belongs to Bhopal. The one who belongs to Jhansi is
23. Who is sitting third to the left of the one, who likes sitting second to the left of C. B is sitting to the immediate
Green? left of the one who belongs to Bhopal. One person is
(a) P sitting between H and the one who belongs to Bhopal. H
(b) L is sitting at the extreme left end. The one who belongs to
(c) K Patna is sitting second to the left of D. D is not sitting at
(d) M the extreme end. There is one people sitting between F and
(e) None of these the one who belongs to Jaipur. A is an immediate neighbor
24. Who is sitting second to the right of L? of the one who belongs to Jaipur. E and the one who
(a) J belongs to Indore are immediate neighbors. H belongs to
(b) M Patna. There are two persons sit between the one who
(c) K belongs to Indore and the one who belongs to Jhansi. B
(d) P neither belongs to Noida nor Jaipur. G sits immediate left
(e) None of these to the one who belongs to Madurai. F does not belong to
25. Who sits fifth to left of one who likes Green? Indore.
(a) P 26. What is the position of G with respect to the one
(b) M who belong to Jhansi?
(c) J (a) Third to left
(d) L (b) Second to left
(e) None of these (c) Third to right
(d) Fourth to right
Page 8 of 1334
Get a Special Paid Bundle PDF on High Level Puzzles & Seating for All Bank Clerk/ PO Mains
If there are any suggestions/ errors in our PDFs Feel Free to contact us via this email: admin@exampundit.in
Ultra Practice Bundle PDF
Reasoning Questions for SBI Clerk/ RBI Assist. Mains
(e) Cannot be determined Directions (31-35) : Answer the questions based on the
27. What is the position of H with respect to the one information given below:
who belong to Lucknow? There are nine persons P, Q, R, S, T, U, V, W and X is
(a) Second to right sitting in a linear table and all of them are facing north
(b) Third to right direction. Each of them goes to different countries i.e.
(c) Immediate left Canada, Brazil, Russia, Japan, India, England, Cuba,
(d) Second to left Thailand and USA but not necessarily in the same order.
(e) Immediate right Q, who goes to India sits second to the left of X. U who
28. Who sits third to right of the one who belongs to goes to Brazil, is a neighbour of both P and Q. X who goes
Noida? to England is not sitting at any of the extreme ends. P and
(a) H R are sitting together. W sits to the immediate left of V
(b) G who neither goes to Thailand nor USA. The person who
(c) B goes to India sits second to the right of the person who
(d) D goes to USA. Only two persons are sitting between Q and
(e) E S who is sitting at the extreme right end. The person who
29. Who sits immediate right of the one who belongs goes to Russia sits third to the right of the person who goes
to Indore? to USA. Only one person is sitting between the persons
(a) Who belongs to Noida who go to Japan and Russia. The person who goes to Cuba
(b) Who belongs to Madurai is sitting at one of the extreme ends, but he is not a
(c) Who belongs to Lucknow neighbour of the person who goes to England.
(d) Who belongs to Jhansi 31. Who sits to the immediate right of the one who goes
(e) Who belongs to Jaipur to Cuba?
30. How many person sit between A and E? (a) V
(a) Six (b) Q
(b) Four (c) R
(c) None (d) T
(d) Two (e) P
(e) Three 32. Who among the following goes to Thailand?
(a) S
Page 9 of 1334
Get a Special Paid Bundle PDF on High Level Puzzles & Seating for All Bank Clerk/ PO Mains
If there are any suggestions/ errors in our PDFs Feel Free to contact us via this email: admin@exampundit.in
Ultra Practice Bundle PDF
Reasoning Questions for SBI Clerk/ RBI Assist. Mains
(b) Q profesions i.e HR, Architect, Engineer, Doctor, Builder,
(c) R Teacher, Banker and Pilot but not necessarily in the same
(d) T order.
(e) U L sits second to the left of O whois an architect. Q faces
33. Who sits third to the left of the one who goes to same direction as facing O. There are two persons sit
Brazil? between K and who is Doctor. N is not an immediate
(a) R neighbor of P who is Engineer. The persons who sit at
(b) W extreme ends are facing opposite direction to each other.
(c) T Only two persons are sitting to the right of the one who is
(d) S Builder. K sits exactly between L and N. L is only
(e) V neighbor of Q and he is Teacher. J and P are immediate
34. W goes to which among the following country? neighbors of each other and facing same directions as Q.
(a) Canada O sits at the immediate left of the one who is Builder. K
(b) Cuba faces same direction as P. L and M are facing opposite
(c) Thailand direction to each other. K is facing in north direction. Q
(d) India is neither Banker nor Pilot. L is an immediate neighbor of
(e) England person who is Builder. J is neither Pilot nor HR.
35. How many persons sit between the one, who goes 36. Who sits to the immediate left of the one who is
to Japan and the who, goes to India? Engineer?
(a) One (a) K
(b) Four (b) O
(c) Six (c) J
(d) Two (d) M
(e) Three (e) L
Directions (36-40) : Answer the questions based on the 37. Four of the following five are alike in a certain way
information given below: based on the given arrangement and hence form a
There are eight friends J, K, L, M, N, O, P and Q are sitting group. Which is the one that doesn’t belong to that
in a linear table. Some of them face north and some of group?
them face south direction. They belong to different (a) Q
Page 10 of 1334
Get a Special Paid Bundle PDF on High Level Puzzles & Seating for All Bank Clerk/ PO Mains
If there are any suggestions/ errors in our PDFs Feel Free to contact us via this email: admin@exampundit.in
Ultra Practice Bundle PDF
Reasoning Questions for SBI Clerk/ RBI Assist. Mains
(b) O Volleyball, Handball, Football, Tennis, Badminton
(c) M andHockey but not necessarily in the same order.
(d) L N sits immediate right of the person who plays
(e) J Handball.Football player does not sit immediate right to
38. Who sits second to the right of P? the Badminton player.L is sitting second from one of the
(a) Builder extreme ends. Badminton player is sitting in the middle of
(b) Doctor the table but not the neighbour of O. K who plays
(c) Teacher Volleyball is sitting immediate left of O who plays
(d) Architect Cricket. P who is playing Tennis is sitting to left of Q who
(e) Pilot neither plays Handball nor Football. The person who plays
39. Who is second to the left of the one who is Hockey is a neighbour of Handball player. K who is
Architect? neither Tennis player nor Hockey player is sitting second
(a) K to left of P. Q is sitting second to left of N. M who plays
(b) M Badminton sits to right of Cricket player. M and Tennis
(c) J player are neighbours.
(d) L 41. Who is sitting third to the right of K?
(e) P (a) L
40. How many persons sit between P and the one who (b) P
is Teacher? (c) Q
(a) One (d) M
(b) Four (e) None of these
(c) Six 42. What is the position of L with respect to the one
(d) Two who is player of Hockey?
(e) Three (a) Immediate right
Directions (41-45) : Answer the questions based on the (b) Immediate left
information given below: (c) Third to the right
There are seven players K, L, M, N, O, P, Q are sitting in (d) Third to the left
a linear table and all of them facing in south direction. (e) None of these
Each of them is player of different sports i.e.Cricket,
Page 11 of 1334
Get a Special Paid Bundle PDF on High Level Puzzles & Seating for All Bank Clerk/ PO Mains
If there are any suggestions/ errors in our PDFs Feel Free to contact us via this email: admin@exampundit.in
Ultra Practice Bundle PDF
Reasoning Questions for SBI Clerk/ RBI Assist. Mains
43. What is O’s position with respect to the one who is Chemistry, Computer, History, Geography, and
player of Volleyball? Economics.
(a)Third to the right I is facing north and is sits third to left of the person who
(b)Second to the right likes English.The person who likes Reasoning sits
(c) Immediate right immediate right to one of the extreme ends. The person
(d)Second to the left who likes Quant and Reasoning never sits next to each
(e) Third to the left other. Person who likes Geography and Quant sits at
44. Who is sitting third to the right of the one who is extreme ends. The person who likes History sits
player of Tennis? immediate left to one of the extreme ends. H is facing
(a) L north direction and sits second to left of the one who likes
(b) M Quant. C is facing north direction and sits fourth to the
(c) K right of the one who likes Physics.F and G sits next to each
(d) P other.The person who likes English sits immediate left of
(e) Q E. I is not an immediate neighbor of the person who likes
45. Which one of the following statements is true Geography. The person who likes Chemistry is not an
according to the below mentioned arrangement? immediate neighbor of G. G and D facing same direction
(a) K is player of Hockey as A facing.Only two person sits between A and E. A sits
(b) Q is player of Hockey one of the extreme ends. The person who likes Economics
(c) L is player of Cricket sits at fifth to the left of A. G sits third right to the person
(d) N is player of Volleyball. who is sitting at one of the extreme ends. B sit at one of
(e) None of these the extreme ends and the person who likes Physics sits
Directions (46-50): Answer the questions based on the sixth to right of B.
information given below: 46. Who is the immediate left of the one who likes
There are nine people A, B, C, D, E, F, G, H, and I are Reasoning?
sitting in a linear table. Some of them facing north and (a) B
some of them facing south direction but not necessarily in (b) C
the same order. Each one of them has likes different (c) D
subjects i.e. Quant, Reasoning, English, Physics, (d) G
(e) None of these
Page 12 of 1334
Get a Special Paid Bundle PDF on High Level Puzzles & Seating for All Bank Clerk/ PO Mains
If there are any suggestions/ errors in our PDFs Feel Free to contact us via this email: admin@exampundit.in
Ultra Practice Bundle PDF
Reasoning Questions for SBI Clerk/ RBI Assist. Mains
47. Who likes Computer subject? (a) G and D
(a) H (b) C and D
(b) E (c) G and H
(c) G (d) B and A
(d) C (e) None of these
(e) None of these 50. Who is sitting fourth to the right of the one who
48. Who is sitting fourth to the left of F? likes History?
(a) A (a) E
(b) G (b) D
(c) D (c) F
(d) H (d) B
(e) None of these (e) G
49. Who are the immediate neighbors of the one who
likes Computer?

Seating Linear 2 Variable - Answer and Explanation


SOLUTION (1-5): 3. V sits third to the right of the one who cleared
Explanation in detail: RRB.
1. The one who sits at extreme end, has cleared RRB. 4. Y is an immediate neighbor of who cleared RRB.
2. Z has cleared RRB. 5. There are three friends sit between S and V.

Page 13 of 1334
Get a Special Paid Bundle PDF on High Level Puzzles & Seating for All Bank Clerk/ PO Mains
If there are any suggestions/ errors in our PDFs Feel Free to contact us via this email: admin@exampundit.in
Ultra Practice Bundle PDF
Reasoning Questions for SBI Clerk/ RBI Assist. Mains
4.b
5.d
SOLUTION(6-10):
6. The friends who have cleared RBI and SEBI sit
Explanation in detail:
together.
1. The one who sit at the extreme ends do not faces
7. S has cleared SEBI.
south.
8. Y sits immediate left of one who cleared IBPS.
2. P and S are sitting at the extreme ends.
3. V is not an immediate neighbor of P.
4. T sits fifth to the right of V.
9. U and W sit together but none of them cleared
RBI.
10. The one who cleared UPSC is immediate right of
the one who cleared SBI.
11. V does not cleared SBI.
12. W has not cleared UPSC.

5. V who is 46 years old is the immediate neighbor


of who facing north.
13. X does not sit near to U.
6. There are four people between P and R.
14. The immediate right of the one who sits at extreme
7. There are as many people sit between T and W as
end has cleared SSC.
the number of person sits to the right of R.

1.d
2.a
3.b
Page 14 of 1334
Get a Special Paid Bundle PDF on High Level Puzzles & Seating for All Bank Clerk/ PO Mains
If there are any suggestions/ errors in our PDFs Feel Free to contact us via this email: admin@exampundit.in
Ultra Practice Bundle PDF
Reasoning Questions for SBI Clerk/ RBI Assist. Mains
There is no such space for W in CASE2 so CASE2 10.c
is cancelled out. SOLUTION(11-15):
8. V sits third to the right of U. Explanation in detail:
9. W is 50 years old. 1. Three persons sit between X and the one who have
10. Q who is 42 years old is sitting second to the left Diesel.
of W. 2. X sits on sixth from the left end.
11. Q faces south. 3. The one who have Fossil sits just to the left of the
one who have Diesel.
4. Two persons sit between X and V who does not
have Fossil.

12. P and S isnot of 48 or 38 years old.


13. T faces the direction opposite to that of W.
14. None of the people who facing north is 46, or 42
or 39 years old.
15. The person who is 35 years old is sitting third to
the left of the one who is 48 years old.
5. The one who have Maxima sits immediateleft to
the V.
6. R have Fossil.

6.d
7.b
8.a
9.c

Page 15 of 1334
Get a Special Paid Bundle PDF on High Level Puzzles & Seating for All Bank Clerk/ PO Mains
If there are any suggestions/ errors in our PDFs Feel Free to contact us via this email: admin@exampundit.in
Ultra Practice Bundle PDF
Reasoning Questions for SBI Clerk/ RBI Assist. Mains
There is no such space for maxima in CASE1 so 15.c
CASE1 is cancelled out. SOLUTION(16-20):
7. Two persons sit between the one who have Explanation in detail:
Giordano and the one who have Armani. 1. S borns in 1995 sits at one of the extreme end of
8. Q have Giordano but he does not sit just to the the table.
right of W. 2. Q faces north direction and sits third to right of S.
9. W sits just to the right of R.
10. One person sits between W and P.

11. The one who have the Titan’s watch sits just to the
right of T.
12. V does not have Titan’s watch. 3. V does not born in 1996 and sits third to the right
13. S have Fastrack’s watch and T have Tissot’s of Q.
watch. There is no such space for V in CASE2 so CASE2 is
cancelled out.

4. P sits immediate left of T who is an immediate


neighbor of Q.
5. There are three persons sit between P and R.
11.b 6. U sits immediate right of R and does not born in
12.d 1996.
13.a
14.c

Page 16 of 1334
Get a Special Paid Bundle PDF on High Level Puzzles & Seating for All Bank Clerk/ PO Mains
If there are any suggestions/ errors in our PDFs Feel Free to contact us via this email: admin@exampundit.in
Ultra Practice Bundle PDF
Reasoning Questions for SBI Clerk/ RBI Assist. Mains

16.a
17. e
18. b
19. d
20. c
7. P and R facing the same direction.
SOLUTION(21-25):
8. U and V facing same direction as S.
Explanation in detail:

1. J sits second from extreme end and likes white.


2. Three persons sit between J and the person who
likes Green.
3. There are more than two persons sit between J and
L.
In CASE1a there is only one person facing south
direction SO CASE1a is cancelled out.
9. P does not born in 1996.
10. T does not born in 1995.
11. Q does not born in 1995.
12. V does not born in 1996.
13. U does not born in 1996 4. K sits second to the right of L.
5. The persons who sitting at extreme end facing
opposite direction to each other.
6. O sits immediate right of P who does not sit
immediate to K.
7. O does not facing south direction.
Page 17 of 1334
Get a Special Paid Bundle PDF on High Level Puzzles & Seating for All Bank Clerk/ PO Mains
If there are any suggestions/ errors in our PDFs Feel Free to contact us via this email: admin@exampundit.in
Ultra Practice Bundle PDF
Reasoning Questions for SBI Clerk/ RBI Assist. Mains

21. e
8. M is an immediate neighbor of J and likes Red.
22. c
9. N does not face south direction.
23. a
24. C
25. b
SOLUTION(26-30):
Explanation in detail:
1. H is sitting at the extreme left end. H belongs to
10. The one who likes Blue and Pink are immediate
Patna.
neighbors of N.
2. B is sitting to the immediate left of the one who
11. L does not like Green and Pink.
belongs to Bhopal.
12. J and K are facing opposite direction to M.
3. One person is sitting between H and the one who
belongs to Bhopal.
4. Two people are sitting between C and the one who
belongs to Bhopal.

13. The one who likes Yellow sits immediate to the


5. The one who belongs to Jhansi is sitting second to
left of who likes Black.
the left of C.
There is no such space for Yellow and Black in CASE2
6. The one who belongs to Patna is sitting second to
so CASE2 is cancelled out.
the left of D.

Page 18 of 1334
Get a Special Paid Bundle PDF on High Level Puzzles & Seating for All Bank Clerk/ PO Mains
If there are any suggestions/ errors in our PDFs Feel Free to contact us via this email: admin@exampundit.in
Ultra Practice Bundle PDF
Reasoning Questions for SBI Clerk/ RBI Assist. Mains
7. There are two persons sit between the one who Explanation in detail:
belongs to Indore and the one who belongs to 1. S is sitting at the extreme right end.
Jhansi. 2. Only two persons are sitting between Q and S.
3. B, who goes to India sits second to the left of X.
4. X who goes to England is not sitting at any of the
extreme ends
8. E and the one who belongs to Indore are
immediate neighbors.
9. There is one people sitting between F and the one
who belongs to Jaipur. 5. U who goes to Brazil is a neighbour of both P and
10. B neither belongs to Noida nor Jaipur. Q.
11. F does not belongs Indore. 6. P and R are sitting together.

12. G sits immediate left to the one who belongs to


7. Wsit to the immediate left of V who neither goes
Madurai.
to Thailand nor USA.
13. A is an immediate neighbor of the one who
8. The person who goes to India sits second to the
belongs to Jaipur.
right of the person who goes to USA.
9. The person who goes to Russia sits third to the
right of the person who goes to USA.
10. Only one person is sitting between the persons
who goes to Japan and Russia.

26. C
27. c
11. The person who goes to Cuba is sitting at one of
28. e
the extreme ends.
29. b
12. Vneither goes to Thailand nor USA.
30. d
SOLUTION(31-35):
Page 19 of 1334
Get a Special Paid Bundle PDF on High Level Puzzles & Seating for All Bank Clerk/ PO Mains
If there are any suggestions/ errors in our PDFs Feel Free to contact us via this email: admin@exampundit.in
Ultra Practice Bundle PDF
Reasoning Questions for SBI Clerk/ RBI Assist. Mains
8. Only two persons are sitting to the right of the one
who is Builder.

31. a
32. C
33. e
9. N is not an immediate neighbor of P who is
34. b
Engineer.
35. d
10. J and P are immediate neighbors of each other and
SOLUTION(36-40):
facing same directions as Q.
Explanation in detail:
11. K faces same direction as P.
1. L is only neighbor of Q and he is Teacher.
12. L and M are facing opposite direction to each
2. L sits second to the left of O who is an architect.
other.
3. Q faces same direction as facing O.

13. K is facing in north direction.


4. The persons who sits at extreme ends are facing So CASE2 cancelled out.
opposite direction to each other. 14. Q is neither Banker nor Pilot.
5. K sits exactly between L and N. 15. J is neither Pilot nor HR.
6. There are two persons sit between K and who is
Doctor.
7. O sits at the immediate left of the one who is
Builder.
Page 20 of 1334
Get a Special Paid Bundle PDF on High Level Puzzles & Seating for All Bank Clerk/ PO Mains
If there are any suggestions/ errors in our PDFs Feel Free to contact us via this email: admin@exampundit.in
Ultra Practice Bundle PDF
Reasoning Questions for SBI Clerk/ RBI Assist. Mains

36. a
37. c 8. Football player does not sit immediate right to the

38. b Badminton player.

39. d 9. The person who plays Hockey is a neighbour of

40. e Handball player.

SOLUTION(41-45): 10. K who is neither Tennis player nor Hockey player

Explanation in detail: is sitting second to left of P.

1. L is sitting second from one of the extreme ends. 11. N sits immediate right of the person who plays

2. Badminton player is sitting in the middle of the Handball.

row but not the neighbour of O.


3. M who plays Badminton sits to right of Cricket
player.
4. M and Tennis player are neighbours.
5. P who is playing Tennis is sitting to left of Q who
neither plays Handball nor Football.
6. Q is sitting second to left of N.
41.d
42.a
43.c
44.a
45.b
SOLUTION(46-50):

7. K who plays Volleyball is sitting immediate left of Explanation in detail:

O who plays Cricket. 1. Only two person sits between A and E.

So there is no such space for K and O in CASE2 so 2. A sits one of the extreme end.

CASE2 is cancelled out.


Page 21 of 1334
Get a Special Paid Bundle PDF on High Level Puzzles & Seating for All Bank Clerk/ PO Mains
If there are any suggestions/ errors in our PDFs Feel Free to contact us via this email: admin@exampundit.in
Ultra Practice Bundle PDF
Reasoning Questions for SBI Clerk/ RBI Assist. Mains
3. The person who likes Economics sits at fifth to the 11. C is facing north direction and sits fourth to the
left of A. right of the one who likes Physics.
4. G sits third to the right of the person who is sitting There is no such space for C in CASE2 so CASE2 is
at one of the extreme ends. cancelled out.

12. F and G sits next to each other.


13. The person who likes English sits immediate left
of E.
5. B sit at one of the extreme ends and the person who
14. I is facing north and is sits third to left of the
likes Physics sits sixth to right of B.
person who likes English.
6. The person who likes History sits immediate left
15. I is not an immediate neighbor of the person who
to one of the extreme ends.
likes Geography.
7. The person who likes Reasoning sits immediate
16. The person who likes Chemistry is not an
right to one of the extreme ends.
immediate neighbor of to G.
8. Person who likes Geography and Quant sits at
17. G and D facing same direction as A facing.
extreme ends.
9. The person who likes Quant and Reasoning never
sits next to each other.

46.A
47.d
10. H is facing north direction and sits second to left 48.a
of the one who likes Quant. 49.a
50.e

Download Seating Arrangement Practice Questions PDF

Page 22 of 1334
Get a Special Paid Bundle PDF on High Level Puzzles & Seating for All Bank Clerk/ PO Mains
If there are any suggestions/ errors in our PDFs Feel Free to contact us via this email: admin@exampundit.in
Ultra Practice Bundle PDF
Reasoning Questions for SBI Clerk/ RBI Assist. Mains

Seating Circle 2 Variable


Directions (1-5): Answer the questions based on the (b) second to left
information given below: (c) third to left
There are eight persons A, B, C, D, E, F, G and H are (d) Immediate left
sitting around a circlular table facing to the centre. They (e) Cannot be determined
all purchased different items i.e. shoes, belt, wallet, jeans, 3. What is the position of F with respect to E?
trousers, shirts, cap and tie but not necessarily in the same (a) second to right
order. (b) immediate left
F sits third to right of A. C sits second to the left of D. D (c) third to right
is not an immediate neighbor of A and F. D is neighbor of (d) second to left
the one who purchased Shoes. There are three persons sit (e) immediate right
between A and the one who purchased Cap. A, F and also 4. Who sits immediate left of H?
their neighbors do not purchased Belt. Only one person sit (a) D
between the one who purchased Shoes and B. A purchased (b) C
Tie. The one who purchased Wallet and Jeans are (c) G
immediate neighbors. F is neither purchased Wallet nor (d) B
Jeans. Only one person sit between H and the one who (e) E
purchased Trouser. H do not purchased Belt and Wallet. 5. Who sits third to the left of one who buys wallet?
The one who purchased Trouser sit opposite to F. G sits to (a) F
immediate right of E and purchased Wallet. (b) B
2. What is the position of C with respect to one who (c) D
buys shoes? (d) A
(a) third to right (e) E

Page 23 of 1334
Get a Special Paid Bundle PDF on High Level Puzzles & Seating for All Bank Clerk/ PO Mains
If there are any suggestions/ errors in our PDFs Feel Free to contact us via this email: admin@exampundit.in
Ultra Practice Bundle PDF
Reasoning Questions for SBI Clerk/ RBI Assist. Mains
6. E purchased Jeans seated second to the left of (b) second to left
whom? (c) third to left
(a) Trouser (d) fourth to right
(b) Wallet (e) Cannot be determined
(c) Shirt 8. What is the position of D with respect to one who
(d) Shoes belongs from sector 18?
(e) None of these (a) second to right
Directions (6-10) : Answer the questions based on the (b) immediate left
information given below: (c) third to right
There are eight friends A, B, C, D, E, F, G and H sitting (d) second to left
around a circular table and all are facing towards the (e) immediate right
centre. All friends belongs to different sector in Noida i.e. 9. Who sits opposite to A?
sector 11, sector 12, sector 13, sector 14, sector 15, sector (a) C
16, sector 17 and sector 18 but not necessarily in the same (b) G
order. (c) B
E is sitting third to the left of G. The person who is from (d) D
sector 12 is to the immediate right of E and E is not from (e) Cannot be determined
sector 11. B is sitting fourth to the right of H. H is not an 10. Who is sitting third to the left of one who belongs
immediate neighbor of G. Neither B nor H is an immediate to sector 12?
neighbor of E. F is from sector 14 and is sitting third to (a) D
the right of the person who from sector 12. The person (b) A
from sector 16 is sitting second to the left of the person (c) H
from sector 14. The person from sector 17 is sitting second (d) G
to the left of E. A, who is from sector 13 is sitting exactly (e) B
between F and H. The person from sector 15 is sitting 11. B sits opposite to the one belongs from which
second to the right of the person from sector 13. C is sector?
sitting third to the left of F. (a) Sector 12
7. What is the position of B with respect to H? (b) Sector 15
(a) third to right (c) Sector 17
Page 24 of 1334
Get a Special Paid Bundle PDF on High Level Puzzles & Seating for All Bank Clerk/ PO Mains
If there are any suggestions/ errors in our PDFs Feel Free to contact us via this email: admin@exampundit.in
Ultra Practice Bundle PDF
Reasoning Questions for SBI Clerk/ RBI Assist. Mains
(d) Sector 11 (b) O
(e) Sector 14 (c) K
Directions (11-15): Answer the questions based on the (d) J
information given below: (e) None of these
There are eight persons J, K, L, M, N, O, P and Q sitting 12. Who sits second to the left of one who have MBA
around a circular table and all are facing away from the degree?
centre. All friends have different degrees i.e. B.Tech, (a) O
MBA, Biotech, B.Pharma, LLB, B.Sc, MCA and BBA (b) L
but not necessarily in the same order. (c) Q
J and L, who has Biotech degree, can never sit together. (d) M
There are two persons sits between the persons who have (e) None of these
MCA and LLB degree. The person who has LLB degree 13. N is sitting third to the left of whom?
sits second to the right of O. Q has not MBA or B.Tech (a) Biotech
degree and he is also an immediate neighbor of one who (b) MBA
has LLB degree. The person who has B.Tech degree (c) MCA
cannot sit with the person who has B.Pharma degree. (d) LLB
There are two persons sitting between L and the person (e) None of these
who has B.Sc degree. K and N are immediate neighbors 14. Who is sitting second to the right of the one who
of each other. The person who has B.Tech degree sits on has BBA degree?
the immediate left of the person who has BBA degree. J (a) O
sits second to the right the person who has MCA degree. (b) J
M has neither B.Sc nor MCA degree. N and P are not (c) M
immediate neighbors. P who has not LLB degree sits on (d) L
the immediate right of Q. There is only one person sitting (e) None of these
between those who has B.Sc and BBA degree. There is 15. How many persons sits between Q and J?
only one person sitting between P and the person who has (a) One
B.Sc degree and that person can never be O. (b) Four
11. Who sits to the immediate left of L? (c)Three
(a) M (d) Six
Page 25 of 1334
Get a Special Paid Bundle PDF on High Level Puzzles & Seating for All Bank Clerk/ PO Mains
If there are any suggestions/ errors in our PDFs Feel Free to contact us via this email: admin@exampundit.in
Ultra Practice Bundle PDF
Reasoning Questions for SBI Clerk/ RBI Assist. Mains
(e) None of these (b) immediate left
Directions (16-20) : Answer the questions based on the (c) third to right
information given below: (d) second to left
There are eight friends A, B, C, D, E, F, G and H sitting (e) immediate right
around a circular table and Four of them are facing the 23. Who likes HRX brand shoes?
centre and other four are facing away from the centre. (a) A
Each of them likes different brands of shoes i.e. Redchief, (b) D
HRX, Puma, Redtape, Adidas, Nike, rebook and UCB but (c) G
not necessarily in the same order. (d) B
E faces towards the centre and likes Redchief. Both the (e) E
immediate neighbors of E faces away from the centre and 24. Who sits second to the right of one who likes UCB
likes Puma and Redtape. D faces away from the centre. brand shoes?
Both the immediate neighbors of D do not faces away (a) F
from the centre. E sits third to the right of F, who likes (b) A
Adidas. C sits third to the left of F. The one who likes (c) H
Puma sits opposite to F. The one who likes HRX is not the (d) E
immediate neighbor of F and faces away from the centre. (e) B
A sits second to the left of C and does not like Rebook and 25. Who sits opposite to the one who likes Nike brand
UCB. The one who likes Rebook sits between F and H. B shoes?
faces away from the centre and does not like HRX. (a) F
21. What is the position of A with respect to B? (b) C
(a) third to right (c) E
(b) second to left (d) B
(c) third to left (e) G
(d) fourth to right Directions (21-25) : Answer the questions based on the
(e) Cannot be determined information given below:
22. What is the position of H with respect to one who There are eight persons J, K, L, M, N, O, P and Q sitting
likes Redchief? around a circular table and all are facing towards the
(a) second to right centre but not necessarily in the same order. Each one of
Page 26 of 1334
Get a Special Paid Bundle PDF on High Level Puzzles & Seating for All Bank Clerk/ PO Mains
If there are any suggestions/ errors in our PDFs Feel Free to contact us via this email: admin@exampundit.in
Ultra Practice Bundle PDF
Reasoning Questions for SBI Clerk/ RBI Assist. Mains
them likes different subjects i.e. History, Geography, (d) L
English, Quant, Reasoning, GA, Politics and Economics (e) None of these
but not necessarily in the same order. 23. Who is sitting third to the left of one who likes
J sits third to the right of who likes Economics. Only two Geography?
people sit between the person who likes economics and Q. (a) O
The person who likes Reasoning and the geography are (b) L
immediate neighbors of each other. Neither J nor Q likes (c) P
Reasoning and Geography. The one who likes Reasoning (d) M
is not an immediate neighbor of the person who likes (e) None of these
economics. The person who likes History sits second to 24. Who is sitting second to the right of L?
the left of N. N is not an immediate neighbor Q. The (a) J
person who likes History is an immediate neighbor of both (b) M
the persons who likes GA and Politics. The person who (c) K
likes Politics sits third to the right of K. K does not like (d) P
Reasoning. L sits to the immediate right of the person (e) None of these
who likes English. J does not like English. O is not an 25. Who sits opposite to one who likes Politics?
immediate neighbor of J. P is not an immediate neighbor (a) P
of the person who likes History. (b) L
21. Who is to the immediate left of one who likes (c) J
Economics? (d) M
(a) M (e) None of these
(b) P Directions (26-30) : Answer the questions based on the
(c) O information given below:
(d) J There are eight persons A, B, C, D, E, F, G and H are
(e) N sitting around a circular table and all of them facing
22. Who sits second to the left of Q? outside the centre. They belongs to different places i.e.
(a) N Noida, Saket, Gurugram, Mayur vihar, Ashok nagar,
(b) P Laxmi nagar, GTB nagar and Rajiv chowk.
(c) J
Page 27 of 1334
Get a Special Paid Bundle PDF on High Level Puzzles & Seating for All Bank Clerk/ PO Mains
If there are any suggestions/ errors in our PDFs Feel Free to contact us via this email: admin@exampundit.in
Ultra Practice Bundle PDF
Reasoning Questions for SBI Clerk/ RBI Assist. Mains
E belongs to Mayur vihar and sits opposite to D. A sits 29.Who sits immediate right of one who belongs to
second to left of D. B belongs to Ashok nagar and sits third Saket?
to left of A. F sits second to left of B. C is not an (a) Who belongs to Noida
immediate neighbor of A. H belongs to Rajiv chowk and (b) Who belongs to Gurugram
sits second to the left of G. The one who belongs to Noida (c) Who belongs to Ashok nagar
sits opposite to the one who belongs to GTB nagar. The (d) Who belongs to Mayur Vihar
one who belongs to Noida sits second to the left of the one (e) Who belongs to Rajiv chowk
who belongs to Saket. F does not belong to Laxmi nagar. 30.How many persons sit between G and E from
26. What is the position of D with respect to one who right of E?
belongs to Rajiv chowk? (a) Six
(a) third to right (b) Four
(b) second to left (c) None
(c) third to left (d) Two
(d) fourth to right (e) Three
(e) Cannot be determined Directions (31-35) : Answer the questions based on the
27.What is the position of G with respect to one from information given below:
Saket? There are eight persons P, Q, R, S, T, U, V and W are
(a) second to right sitting around a circular table and all of them facing
(b) third to right outside the centre. They likes different engineering
(c) immediate left branches i.e. Civil, Mechanical, Chemical, Electrical,
(d) second to left Optical, Power, Computer and Geological but not
(e) immediate right necessarily in the same order.
28. Who sits third to the right of one who is from P likes Electrical and R sits second to the left of P. Only
GTB nagar? one person is sitting between R and U. Q likes Chemical
(a) E and is sitting second to the left of U. S sits third to right of
(b) G Q. T likes Power and sits second to right of S. Only one
(c) B person is sitting between T and V. The one who likes
(d) D Mechanical is sitting second to the right of the one who
(e) H
Page 28 of 1334
Get a Special Paid Bundle PDF on High Level Puzzles & Seating for All Bank Clerk/ PO Mains
If there are any suggestions/ errors in our PDFs Feel Free to contact us via this email: admin@exampundit.in
Ultra Practice Bundle PDF
Reasoning Questions for SBI Clerk/ RBI Assist. Mains
likes Geological. S does not like Mechanical. R likes (a) One
Computer and U does not like Optical. (b) Four
31. Who sits to the immediate right of one who likes (c) Six
Electrical? (d) Two
(a) V (e) Three
(b) Q Directions (36-40) : Answer the questions based on the
(c) R information given below:
(d) T Eight people – P, Q, R, S, T, U, V and W are sitting around
(e) P a circular table facing the centre, not necessarily in the
32. Who among the following likes Civil? same order. All of them have different companies car i.e.
(a) S Ford, TATA, Renault, Maruti, Kia, Toyota, BMW and
(b) Q Audi.
(c) R Only one person is sitting between T and the one who has
(d) T Maruti car. W does not have Toyota. The one who has a
(e) U Ford car is sitting exactly opposite to the one who has a
33. Who sits third to the left of one who likes Optical? BMW car. Two persons are sitting between V and the one
(a) R who has a Ford car. V does not have a Renault car. Three
(b) U persons are sitting between Q and the one who has a Audi
(c) T car. Q neither has Renault nor BMW car. R neither has
(d) S Maruti nor Audi car. P is sitting third to the left of Q. The
(e) Q one who has TATA car is sitting fifth to the left of W. The
34. Which engineering branch likes V? one who has a Toyota car is not an immediate neighbour
(a) Chemical of Q. One person is sitting between R and the one who has
(b) Geological a BMW car. The person who has Renault car is sitting
(c) Electrical third to the right of R. U is sitting fourth to the left of the
(d) Computer one who has Renault car.
(e) Power 36. Who sits to the immediate left of who has Kia’s
35. How many persons sit between those who likes car?
Civil and Power from left of Power? (a) R
Page 29 of 1334
Get a Special Paid Bundle PDF on High Level Puzzles & Seating for All Bank Clerk/ PO Mains
If there are any suggestions/ errors in our PDFs Feel Free to contact us via this email: admin@exampundit.in
Ultra Practice Bundle PDF
Reasoning Questions for SBI Clerk/ RBI Assist. Mains
(b) W (b) Four
(c) T (c) Six
(d) S (d) Two
(e) U (e) Three
37. Four of the following five are alike in a certain way Directions (41-45) : Answer the questions based on the
based on the given arrangement and hence form a information given below:
group. Which is the one that doesn’t belong to that There are eight persons P, Q, R, S, T, U, V and W are
group? sitting around a circular table four of them are facing the
(a) WR centre and other four are facing away from the centre.
(b) QV They are working in different banks i.e. BOB, Canara
(c) TR bank, Axis bank, Hdfc bank, SBI, ICICI bank, Syndicate
(d) SU bank and PNB bank but not necessarily in the same order.
(e) PT The person who works in Hdfc and SBI banks are facing
38. Who sits second to the right of P? same direction. T is sitting between the person who works
(a) Who has Toyota’s car in Axis bank and the one who works in PNB bank. U faces
(b) Who has Maruti’s car the centre and sits third to the right of R. S works in Axis
(c) Who has Audi’s car bank and faces the person who works in ICICI bank. V
(d) Who has Ford’s car sits third to the right of Q, who works in BOB bank. P
(e) Who has Kia’s car works in SBI and R works in Canara bank. The person
39. Who is second to the left of one who has Maruti’s who works in Syndicate bank is facing outside the centre
car? and an immediate neighbor of the one who works in PNB
(a) Q bank. W is an immediate neighbor of the person who
(b) U works in SBI and Syndicate bank. T sits to immediate left
(c) W of S.
(d) P 41. Who is sitting third to the right of R?
(e) S (a) W
40. How many persons sit between V and who has (b) S
Kia’s car? (c) P
(a) One (d) U
Page 30 of 1334
Get a Special Paid Bundle PDF on High Level Puzzles & Seating for All Bank Clerk/ PO Mains
If there are any suggestions/ errors in our PDFs Feel Free to contact us via this email: admin@exampundit.in
Ultra Practice Bundle PDF
Reasoning Questions for SBI Clerk/ RBI Assist. Mains
(e) None of these (d) T sits fourth to the right of one who works in
42. What is the position of W with the one who works ICICI.
in Syndicate bank? (e) None of these
(a) Immediate right Directions (46-50) : Answer the questions based on the
(b) Immediate left information given below:
(c) Third to the right In a wrestling championship, there are eight wrestler i.e.
(d) Third to the left A, B, C, D, E, F, G and H representing their countries i.e.
(e) None of these India, USA, UK, China, Japan, Brazil, Russia and Iraq are
43. What is T’s position with respect to one who works sitting around a circular table facing outside the centre but
in PNB ? not necessarily in the same order.
(a) Third to the right C and H and also their immediate neighbors do not
(b) Second to the right represents UK. Only one person sits between G and the
(c) Immediate right person who represents UK. The person who represents
(d) Second to the left India and China are immediate neighbors of each other. C
(e) Third to the left represents neither Japan nor China. Only one person sits
44. Who is sitting third to the right of one who works between A and who represents Brazil. A does not
in HDFC bank? represents UK and India. C sits third to the right of H. H
(a) Q represents Iraq. E is not an immediate neighbor of either
(b) P C or H. The one who represents USA is an immediate
(c) R neighbor of E. Three people sit between H and the one
(d) W who represents Russia. B is not representing either UK or
(e) T USA. F sits third to the left of E and does not represent
45. Which one of the following statements is true Japan or China.
according to the above mentioned arrangement? 46. Who is to the immediate left of one who represents
(a) P is to the immediate left of one who works in Russia?
Axis. (a) B
(b) W sits second to the right of one who works in (b) C
BOB. (c) D
(c) V sits second to right of one who works in SBI. (d) G
Page 31 of 1334
Get a Special Paid Bundle PDF on High Level Puzzles & Seating for All Bank Clerk/ PO Mains
If there are any suggestions/ errors in our PDFs Feel Free to contact us via this email: admin@exampundit.in
Ultra Practice Bundle PDF
Reasoning Questions for SBI Clerk/ RBI Assist. Mains
(e) None of these (a) B and D
47. Who represents China in wrestling championship? (b) C and D
(a) C (c) G and H
(b) E (d) B and A
(c) G (e) None of these
(d) A 50. Who is sitting fourth to the right of the one who
(e) None of these represents Japan in wrestling championship?
48. Who is sitting third to the left of B? (a) E
(a) A (b) D
(b) G (c) F
(c) D (d) B
(d) F (e) C
(e) None of these
49. Who is immediate neighbors of those who
represents UK?

Seating Circle 2 Variable – Answer and Explanation


SOLUTION(1-5): 1. F sits third to right of A.
1. d 2. A purchased Tie.
2. a 3. C sits second to the left of D.
3. b 4. D is not an immediate neighbor of A and F.
4. b
5. c
Explanation in detail:

Page 32 of 1334
Get a Special Paid Bundle PDF on High Level Puzzles & Seating for All Bank Clerk/ PO Mains
If there are any suggestions/ errors in our PDFs Feel Free to contact us via this email: admin@exampundit.in
Ultra Practice Bundle PDF
Reasoning Questions for SBI Clerk/ RBI Assist. Mains
11. A, F and also their neighbors do not purchased
Belt.

5. The one who purchased Trouser sit opposite to F.


6. G sits to immediate right of E and purchased
Wallet.
7. The one who purchased Wallet and Jeans are
12. Only one person sit between the one who
immediate neighbors.
purchased Shoes and B.
8. F is neither purchased Wallet nor Jeans.
13. Only one person sit between H and the one who
purchased Trouser.
14. H do not purchased Belt and Wallet.

9. D is neighbor of the one who purchased Shoes.


10. There are three persons sit between A and the one
who purchased Cap.

Page 33 of 1334
Get a Special Paid Bundle PDF on High Level Puzzles & Seating for All Bank Clerk/ PO Mains
If there are any suggestions/ errors in our PDFs Feel Free to contact us via this email: admin@exampundit.in
Ultra Practice Bundle PDF
Reasoning Questions for SBI Clerk/ RBI Assist. Mains

SOLUTION(6-10):
6 F is from sector 14 and is sitting third to the right
6. d
of the person from sector 12.
7. b
6. The person from sector 16 is sitting second to the
8. a
left of the person from sector 14.
9. c
7. The person from sector 17 is sitting second to the
10. c
left of E.
Explanation in detail:
8. A, who is from sector 13 is sitting exactly between
1. E is sitting third to the left of G.
F and H.
2. The person who is from sector 12 is to the
9. The person from sector 15 is sitting second to the
immediate right of E and E is not from sector 11.
right of the person from sector 13.
3. B is sitting fourth to the right of H.
10. C is sitting third to the left of F.
4. H is not an immediate neighbor of G.
5. Neither B nor H is an immediate neighbor of E.

Page 34 of 1334
Get a Special Paid Bundle PDF on High Level Puzzles & Seating for All Bank Clerk/ PO Mains
If there are any suggestions/ errors in our PDFs Feel Free to contact us via this email: admin@exampundit.in
Ultra Practice Bundle PDF
Reasoning Questions for SBI Clerk/ RBI Assist. Mains
Final arrangement:

SOLUTION(11-15):
11. b
12. d 5. There are two persons sits between the persons
13. a who have MCA and LLB degree.
14. c 6. J sits second to the right the person who has MCA
15. c degree.
Explanation in detail: 7. There are two persons sitting between L and the
1. The person who has LLB degree sits second to the person who has B.Sc degree.
right of O. 8. J and L, who has Biotech degree, can never sit
2. Q is an immediate neighbor of one who has LLB together.
degree.
3. P who has not LLB degree sits on the immediate
right of Q.
4. There is only one person sitting between P and the
person who has B.Sc degree and that person can
never be O.

Page 35 of 1334
Get a Special Paid Bundle PDF on High Level Puzzles & Seating for All Bank Clerk/ PO Mains
If there are any suggestions/ errors in our PDFs Feel Free to contact us via this email: admin@exampundit.in
Ultra Practice Bundle PDF
Reasoning Questions for SBI Clerk/ RBI Assist. Mains
9. K and N are immediate neighbors of each other. SOLUTION(16-20):
10. The person who has B.Tech degree sits on the 16. a
immediate left of the person who has BBA degree. 17. e
11. M has neither B.Sc nor MCA degree. 18. b
12. N and P are not immediate neighbors. 19. d
13. There is only one person sitting between those two 20. c
persons who has B.Sc and BBA degree. Explanation in detail:
14. The person who has B.Tech degree can not sit with 1. E faces towards the centre and likes Redchief.
the person who has B.Pharma degree. 2. E sits third to the right of F, who likes Adidas.
15. Q has not MBA or B.Tech degree. 3. Both the immediate neighbors of E faces away
from the centre and likes Puma and Redtape.
4. The one who likes Puma sits opposite to F.
5. C sits third to the left of F.

Page 36 of 1334
Get a Special Paid Bundle PDF on High Level Puzzles & Seating for All Bank Clerk/ PO Mains
If there are any suggestions/ errors in our PDFs Feel Free to contact us via this email: admin@exampundit.in
Ultra Practice Bundle PDF
Reasoning Questions for SBI Clerk/ RBI Assist. Mains
6. A sits second to the left of C and does not like
Rebook and UCB.
7. The one who likes Rebook sits between F and H.

8. D faces away from the centre.


9. Both the immediate neighbors of D do not faces
away from the centre.

Page 37 of 1334
Get a Special Paid Bundle PDF on High Level Puzzles & Seating for All Bank Clerk/ PO Mains
If there are any suggestions/ errors in our PDFs Feel Free to contact us via this email: admin@exampundit.in
Ultra Practice Bundle PDF
Reasoning Questions for SBI Clerk/ RBI Assist. Mains
There is no such space for D in CASE1 So CASE1 is 21. e
cancelled out 22. c
1. The one who likes HRX is not the immediate 23. a
neighbor of F and faces away from the centre. 24. d
2. B faces away from the centre and does not like 25. b
HRX. Explanation in detail:
1. J sits third to the right of who likes Economics.
2. Only two people sit between the person who likes
economics and Q.
3. The person who likes Reasoning and the
geography are immediate neighbors of each other.
4. Neither J nor Q likes Reasoning and Geography.
5. The one who likes Reasoning is not an immediate
neighbor of the person who likes economics.

SOLUTION(21-25):

Page 38 of 1334
Get a Special Paid Bundle PDF on High Level Puzzles & Seating for All Bank Clerk/ PO Mains
If there are any suggestions/ errors in our PDFs Feel Free to contact us via this email: admin@exampundit.in
Ultra Practice Bundle PDF
Reasoning Questions for SBI Clerk/ RBI Assist. Mains

There is no such space for GA and Politics. So


6. The person who likes History sits second to the left
CASE2 is cancelled out
of N.
11. L sits to the immediate right of the person who
7. N is not an immediate neighbor Q.
likes English.
8. The person who likes History is an immediate
12. J doesnot like English.
neighbor of both the persons who likes GA and
13. O is not an immediate neighbor of J.
Politics.
14. P is not an immediate neighbor of the person who
9. The person who likes Politics sits third to the right
likes History.
of K.
10. K does not likes Reasoning.

final arrangement:

Page 39 of 1334
Get a Special Paid Bundle PDF on High Level Puzzles & Seating for All Bank Clerk/ PO Mains
If there are any suggestions/ errors in our PDFs Feel Free to contact us via this email: admin@exampundit.in
Ultra Practice Bundle PDF
Reasoning Questions for SBI Clerk/ RBI Assist. Mains
4. F sits second to left of B.
5. C is not an immediate neighbor of A.

SOLUTION(26-30): 6. H belongs to Rajiv chowk and sits second to the


26. a left of G.
27. c 7. The one who belongs to Noida sits opposite to the
28. e one who belongs to GTB nagar.
29. b 8. The one who belongs to Noida sits second to the
30. d left of the one who belongs to Saket.
Explanation in detail: 9. F does not belong to Laxmi nagar.
1. E belongs to Mayur vihar and sits opposite to D.
2. A sits second to left of D.

3. B belongs to Ashok nagar and sits third to left of


A.
Page 40 of 1334
Get a Special Paid Bundle PDF on High Level Puzzles & Seating for All Bank Clerk/ PO Mains
If there are any suggestions/ errors in our PDFs Feel Free to contact us via this email: admin@exampundit.in
Ultra Practice Bundle PDF
Reasoning Questions for SBI Clerk/ RBI Assist. Mains
6. Only one person is sitting between T and V.

SOLUTION(31-35):
31. a
7. The one who likes Mechanical is sitting second to
32. e
the right of the one who likes Geological.
33. e
8. S does not like Mechanical.
34. b
9. R likes Computer.
35. d
10. U does not like Optical.
Explanation in detail:
1. P likes Electrical and R sits second to the left of P.
2. Only one person is sitting between R and U.
3. Q likes Chemical and is sitting second to the left
of U.

4. S sits third to right of Q.


5. T likes Power and sits second to right of S.

Page 41 of 1334
Get a Special Paid Bundle PDF on High Level Puzzles & Seating for All Bank Clerk/ PO Mains
If there are any suggestions/ errors in our PDFs Feel Free to contact us via this email: admin@exampundit.in
Ultra Practice Bundle PDF
Reasoning Questions for SBI Clerk/ RBI Assist. Mains

SOLUTION(36-40):
36. b
37. c
38. b
39. d
40. e
Explanation in detail:
1. The person who has Renault car is sitting third to
the right of R. 5. Two persons are sitting between V and the one
2. U is sitting fourth to the left of the one who has who has a Ford car.
Renault car. 6. V does not have a Renault car.
3. One person is sitting between R and the one who
has a BMW car.
4. The one who has a Ford car is sitting exactly
opposite to the one who has a BMW car.

Page 42 of 1334
Get a Special Paid Bundle PDF on High Level Puzzles & Seating for All Bank Clerk/ PO Mains
If there are any suggestions/ errors in our PDFs Feel Free to contact us via this email: admin@exampundit.in
Ultra Practice Bundle PDF
Reasoning Questions for SBI Clerk/ RBI Assist. Mains

7. Three persons are sitting between Q and the one


who has a Audi car.
8. Q neither has Renault nor BMW car.

There is no such space for Q and audi in CASE1 so


case1 is cancelled out.
9. R neither has Maruti nor Audi car.
10. P is sitting third to the left of Q.

Page 43 of 1334
Get a Special Paid Bundle PDF on High Level Puzzles & Seating for All Bank Clerk/ PO Mains
If there are any suggestions/ errors in our PDFs Feel Free to contact us via this email: admin@exampundit.in
Ultra Practice Bundle PDF
Reasoning Questions for SBI Clerk/ RBI Assist. Mains
11. The one who has TATA car is sitting fifth to the 45. b
left of W. Explanation in detail:
12. Only one person is sitting between T and the one 1. The person who works in Hdfc and SBI banks are
who has Maruti car. facing same direction.
13. W neither has Toyota nor Maruti car. 2. T sits to immediate left of S.
14. The one who has a Toyota car is not an immediate 3. T is sitting between the person who works in Axis
neighbour of Q. bank and the one who works in PNB bank.
4. S works in Axis bank and faces the person who
works in ICICI bank.

SOLUTION(41-45):
41. d
42. a
43. c
44. d

Page 44 of 1334
Get a Special Paid Bundle PDF on High Level Puzzles & Seating for All Bank Clerk/ PO Mains
If there are any suggestions/ errors in our PDFs Feel Free to contact us via this email: admin@exampundit.in
Ultra Practice Bundle PDF
Reasoning Questions for SBI Clerk/ RBI Assist. Mains
In CASE1 S does not face who works in ICICI bank so
CASE1 is cancelled out.
5. U faces the centre and sits third to the right of R.
6. V sits third to the right of Q, who works in BOB
bank.
7. P works in in SBI.
8. R works in Canara bank.
9. The person who works in Syndicate bank is facing
outside the centre and an immediate neighbor of
who works in PNB bank.
10. W is an immediate neighbor of the person who SOLUTION(46-50):
works in SBI and Syndicate bank. 46. b
47. d
48. a
49. a
50. e
Explanation in detail:
1. C sits third to the right of H.
2. H represents Iraq.
3. Three people sit between H and the student of
Russia.
4. F sits third to the left of E.
5. E is not an immediate neighbor of either C or H.

Page 45 of 1334
Get a Special Paid Bundle PDF on High Level Puzzles & Seating for All Bank Clerk/ PO Mains
If there are any suggestions/ errors in our PDFs Feel Free to contact us via this email: admin@exampundit.in
Ultra Practice Bundle PDF
Reasoning Questions for SBI Clerk/ RBI Assist. Mains
9. The person who represents India and China are
immediate neighbors of each other.
10. C represents neither Japan nor China.
11. A does not represents UK and India.
12. Only one person sits between A and who
represents Brazil.
13. B is not representing either UK or USA.

6. The one who represents USA is an immediate


neighbor of E.
7. C and H and also their immediate neighbors do not
represents UK.
8. Only one person sits between G and the person
who represents UK.

Download Seating Arrangement Practice Questions PDF


Get More Reasoning Practice Questions PDF
Page 46 of 1334
Get a Special Paid Bundle PDF on High Level Puzzles & Seating for All Bank Clerk/ PO Mains
If there are any suggestions/ errors in our PDFs Feel Free to contact us via this email: admin@exampundit.in
Ultra Practice Bundle PDF
Reasoning Questions for SBI Clerk/ RBI Assist. Mains
Seating Parallel Row 2 Variable
Directions-1-5: Study the following information a. O and U
carefully and answer the questions given below: b. M and Y
Eighteen persons I through Z are sitting in three c. P and T
horizontally parallel rows such that each of them are d. O and N
facing north. Six persons sit equidistant to each other in e. Cannot be determined
each row. If a person sits opposite to another person then 3) How many persons sit to the left of V?
both of them must be on the adjacent row. Row 1 is in a. Three
north or row 2 and row 2 is in north of row 3. The left end b. Four
of the three rows lie in the same line and the right end of c. Six
the three rows lie in the same line. d. One
S sits third to the left of U, who sits opposite to N. W sits e. None of these
in row 2 and to the immediate left of V but neither sit at 4) Who among the following sits to the immediate left
any end. N sits opposite to U and M. K sits second to the of M if Q sits adjacent to S?
left of M and K sits in the 3rd row. One person sits a. K
between V and I, who does not sit at any end. S does not b. J
sit at any end. W and K are not opposite to each other. O c. N
sits opposite to R and Y. Neither Y nor R is adjacent to M. d. Y
X sits to the immediate left of Z but not adjacent to S. P e. None of these
and T are adjacent to each other. P is not opposite to L. L 5) Who sits third to the left of N?
is in 2nd row. P is not adjacent to U. a. O
1) Who among the following persons sits opposite to S? b. I
a. L c. V
b. V d. W
c. X e. None of these
d. W Directions 6-10: Study the following information
e. None of these carefully and answer the questions given below:
2) Who sits at extreme end of row 2?

Page 47 of 1334
Get a Special Paid Bundle PDF on High Level Puzzles & Seating for All Bank Clerk/ PO Mains
If there are any suggestions/ errors in our PDFs Feel Free to contact us via this email: admin@exampundit.in
Ultra Practice Bundle PDF
Reasoning Questions for SBI Clerk/ RBI Assist. Mains
Eighteen persons I through Z are sitting in three parallel c. N
rows. There are six persons in each row. The gap between d. W
each seat in each row is same. The persons in row 1 are e. S
facing south. The persons in row 3 are facing north. In row 8) Who sits to the immediate left of K?
2, first three persons from left end face north while rests a. S
of the persons are facing south. Row 1 is in north of row b. L
2 while row 2 is in north of row 3. The persons in Row 1 c. X
sit opposite to the persons in Row 2 and the persons in d. O
Row 2 sit opposite to the persons in Row 3. e. None of these
Note: If A is facing B then A and B must be on adjacent 9) Who sits second to the left of M?
rows and are facing each other. a. N
P sits third to the right of X.S and M sit to the immediate b. V
right of each other. P faces K, who sits to the immediate c. W
left of S. N faces W, who sits to the immediate right of V, d. Z
who does not sit at any end. L faces Q, who sits second to e. None of these
the right of R. Z sits to the immediate left of T, who does 10) Who sits second to the right of P?
not sit adjacent to V. Z and I do not sit at extreme end. a. R
One person sits between O and I. J faces south and does b. I
not sit adjacent to T or Z. c. X
6) Who among the following sits to the immediate right d. O
of J? e. None of these
a. S Directions-11-15: Study the following information
b. M carefully and answer the questions given below:
c. N Sixteen persons are sitting in three rows namely Row 1, 2,
d. K and 3 such that 4 persons sit in each Row 1 and Row 3.
e. None of these And 8 persons sit in Row 2. Persons sitting in the Row 1
7) Find odd one out. are facing towards the south direction and the people
a. Q sitting in the Row 3 are facing towards the north direction.
b. O The first four persons from the left end of the Row 2 are
Page 48 of 1334
Get a Special Paid Bundle PDF on High Level Puzzles & Seating for All Bank Clerk/ PO Mains
If there are any suggestions/ errors in our PDFs Feel Free to contact us via this email: admin@exampundit.in
Ultra Practice Bundle PDF
Reasoning Questions for SBI Clerk/ RBI Assist. Mains
facing north direction and remaining four are facing south e. K and S are immediate neighbors
direction (considering all of them are facing the north). 13) Four of the following are related to each other in
Thus, the first four persons of Row 2 are facing the some way and thus formed a group. Choose the one
persons sitting in Row 1 and the person sitting in Row 3 which does not belong to that group.
are facing the remaining persons of the Row 2 who are a. Q
facing towards the south. The distance between adjacent b. S
person in each row is same. c. T
Y sits third to the left of T. W faces the one who sits d. Z
second to the right of K. Only two persons sitting between e. J
K and T. Z sits third to the right of V, who does not face 14) Who sits second to the right of I?
the north. Z is not adjacent to Y. H is sitting second to the a. The one who faces K
right of the one who faces Z. No one sits to the right of U. b. The one who faces R
Neither G nor H sits at the extreme end of the row also c. The one who faces U
one of them faces towards the south. S faces the one who d. The one who faces Y
is on the immediate left of G. R sits third from an extreme e. The one who faces Q
end but does not face south. E sits second to the left of J. 15) Who sits opposite to the one, who sits immediate
Three persons sit between Q and N, who faces R. One of right of Y?
the persons is I. a. J
11) Who sits second to the left of N? b. H
a. V c. G
b. R d. U
c. Q e. None of these
d. K Directions 16-20: Study the following information
e. G carefully and answer the questions given below:
12) Which of the following is correct about S? There are three horizontally parallel rows i.e. row 1, row
a. The one, who sits immediate right of T faces S 2 and row 3. Row 1 is north of row 2, which is in north of
b. The one, who faces R sits second to the right of S row 3. Four persons sit on row 1 facing south, four persons
c. S and H are immediate neighbors sit in row 3 facing north and eight persons sit in row 2 such
d. S sits opposite to E that first three persons (from the left end of the row,
Page 49 of 1334
Get a Special Paid Bundle PDF on High Level Puzzles & Seating for All Bank Clerk/ PO Mains
If there are any suggestions/ errors in our PDFs Feel Free to contact us via this email: admin@exampundit.in
Ultra Practice Bundle PDF
Reasoning Questions for SBI Clerk/ RBI Assist. Mains
considering all are facing north) face north while rest of II. E
the five persons face south. Persons, who sit in row 1 and III. K
row 3 face the persons, who sit on row 2. The distance IV. M
between each of the adjacent persons in all the rows is a. I, II and IV
same. The persons at the rightmost seat of row 2 is b. Both III and IV
opposite to the rightmost seat of row 3(considering all are c. Both I and III
facing the north direction). And, the leftmost seat of row d. Only III
2 is opposite to leftmost seat of row 1(considering all are e. Both II and IV
facing the north direction). 18) How many persons sit between B and the person,
Note: If X faces Y, then they must face towards each who sits immediate right of J?
other. a. One
C faces the person, who sits 3rd to the right of F.J sits 2nd b. Three
to the left of F and adjacent to D. D faces the person, who c. Four
sits 2nd to the right of P. Neither C nor D sits at any of the d. Two
extreme ends. P faces the person, who sits 2nd to the right e. None of these
of E. Only one person sits between E and A, who faces I.
O sits immediate right of N, who doesn’t face P.L and G 19) Who among the following pairs of persons doesn’t
sits adjacent to each other. H faces B, who doesn’t sits 2nd form a group?
to the right of M. L doesn’t sit immediate right of F. M a. H, C
doesn’t sit in row 3.K doesn’t face J. b. E, G
16) Who among the following faces the person, who sits c. P, O
immediate left of N? d. L, N
a. E e. B, J
b. M 20) Who sits opposite to H?
c. F a. A
d. D b. E
e. None of these c. B
17) Who among the following doesn’t sit in row 2? d. L
I. J e. None of these
Page 50 of 1334
Get a Special Paid Bundle PDF on High Level Puzzles & Seating for All Bank Clerk/ PO Mains
If there are any suggestions/ errors in our PDFs Feel Free to contact us via this email: admin@exampundit.in
Ultra Practice Bundle PDF
Reasoning Questions for SBI Clerk/ RBI Assist. Mains
Directions (21-25): Study the following information d. Y
carefully and answer the questions given below: e. None of these
Twenty persons A, B, C, D, E, F, G, H, I, P, Q, R, S, T, U, 22) Who among the following sits second to the left of V?
V, W, X, Y and Z are sitting in three horizontally parallel a. S
rows such that there are five persons sitting in row 1, ten b. T
persons sitting in row 2 and five persons sitting in row 3. c. C
Persons in row 1 are facing south and persons in row 3 are d. X
facing north. Five consecutive persons from the left face e. Cannot be determined
south while other five face north in row 2. All the persons 23) How many persons sit to the left of T?
in row 1 are in north-east of all the persons sitting in row a. Three
3. Row 1 is to the north of Row 2, which is to the north of b. Four
Row 3. Distance between the adjacent persons is same. c. Five
The five persons in Row 1 are sitting opposite to the five d. Two
persons in Row 2 and the other five persons in Row 2 are e. None of these
sitting opposite to the five persons in Row 3. 24) Find the odd one out.
A sits third to the left of X, who faces Q. Q sits at one of a. P
the extreme ends. Two persons sit between Q and I. B sits b. C
opposite to I and B sits adjacent to C. One person sits c. U
between X and Y such that Y does not sit adjacent to A. d. E
W sits adjacent to Z, who sits opposite to U, who is not e. R
adjacent to X. F faces north and sits exactly between G 25) Who sits third to right of G?
and E. T faces E and sits adjacent to S. R sits to the a. Z
immediate right of H. Neither V nor D sits at any end. V b. None
is an immediate neighbor of X. Z sits at the extreme left c. W
end of Row-3. d. F
21) Who among the following sits opposite to H? e. E
a. B Directions 26-30: Study the following information
b. P carefully and answer the questions given below:
c. F
Page 51 of 1334
Get a Special Paid Bundle PDF on High Level Puzzles & Seating for All Bank Clerk/ PO Mains
If there are any suggestions/ errors in our PDFs Feel Free to contact us via this email: admin@exampundit.in
Ultra Practice Bundle PDF
Reasoning Questions for SBI Clerk/ RBI Assist. Mains
There are 3 horizontally parallel rows, row 1, row 2 and b. 1
row 3. Row 1 is to the north of row 2 and row 2 is to the c. 4
north of row 3. There are five persons sitting in each row d. 2
and L is one of the persons among them. In row 1, first e. None of these
three persons sitting from left to right are facing north and 28) How many persons are sitting between Q and D?
rest are facing south. In row 3, first three persons sitting a. 2
from left to right are facing north and rest are facing south. b. 3
In row 2, first two persons sitting from left to right are c. 1
facing north and rest are facing south. Last two persons d. 4
from left end of row 1 face the first two persons from the e. None of these
left end of row 2. Last three persons from left end of row 29) Who is sitting opposite to B?
2 face first three persons from left end of row 3. a. D
F, who is sitting 2nd to the right of Q, is sitting opposite b. N
to A. Three persons are sitting between A and M. F is c. F
facing south. G is sitting 2nd to the right of C and both of d. Q
them are facing the north. E is sitting in the left of G. One e. None of these
person is sitting between E and N. B is sitting to the 30) Who is sitting in the middle of row 3?
immediate left of D, who is sitting opposite to O, who is a. D
not sitting adjacent to A. S and P are sitting opposite to b. G
each other. Neither S nor P is sitting in row 1. R is not c. R
sitting adjacent to M. d. Either (a) or (b)
26) Find the odd one out? e. None of these
a. C Directions-31-35: Study the following information
b. O carefully and answer the questions given below:
c. N There are three horizontally parallel rows row 1, row 2 and
d. F row 3. Row 1 is north of row 2, which is north of row 3.
e. Q Five persons sit on row 1 facing south, five persons sit in
27) How many persons are sitting in the left of S? row 3 facing north and ten persons sit in row 2 such that
a. 3 first five persons (from the left of the row) face north
Page 52 of 1334
Get a Special Paid Bundle PDF on High Level Puzzles & Seating for All Bank Clerk/ PO Mains
If there are any suggestions/ errors in our PDFs Feel Free to contact us via this email: admin@exampundit.in
Ultra Practice Bundle PDF
Reasoning Questions for SBI Clerk/ RBI Assist. Mains
while rest of the five persons face south. Persons, who sit 33) How many persons sit between M and the person
in row 1 and row 3 face the persons, who sit on row 2. The who faces D who sits immediate right of J?
distance between each of the adjacent persons in the row a. One
is same. No seat is vacant. Q, D, I and L are some of the b. Two
persons sitting on the seats. c. Three
H faces the person, who sits 3rd to the right of M. Two d. Four
persons sit between O and the person who faces U. O e. None of these
doesn’t sit at any extreme ends. Person, who sits to the 34) Who among the following persons sit between G
immediate right of O, faces J. Person, who faces G, sits to and the person, who faces U?
the immediate left of T. W sits to the immediate left of E. I. K
U sits to the immediate left of F, who faces K. One person II. E
sits between M and K. Neither E nor U sit at any extreme III. O
ends. J faces the person, who sits 3rd to the right of G. One a. Both II and III
person sits between E and V, who faces south. R sits to b. Only II
the immediate right of P. Neither D nor I face north. c. Both I and II
Neither N nor L sits at any extreme ends. One person sits d. Only III
between N and S. e. Only I
31) How many persons sit between N and K? 35) Who sits third to the right of N?
a. Four a. M
b. Two b. V
c. Three c. K
d. Five d. E
e. None of these e. L
32) Who sits 3rd to the left of P? Directions-36-40: Study the following information
a. J carefully and answer the questions given below:
b. Q There are three horizontally parallel rows among row 1,
c. L row 2 and row 3. Row 1 is north of row 2, which is north
d. U of row 3. Four persons sit in row 1 facing south, four
e. None of these persons sit in row 3 facing north and eight persons sit in
Page 53 of 1334
Get a Special Paid Bundle PDF on High Level Puzzles & Seating for All Bank Clerk/ PO Mains
If there are any suggestions/ errors in our PDFs Feel Free to contact us via this email: admin@exampundit.in
Ultra Practice Bundle PDF
Reasoning Questions for SBI Clerk/ RBI Assist. Mains
row 2 such that first four persons (from the left end of the e. Both I and III
row) face north while rest of the persons face south. 38) Find the odd among the following pairs of persons?
Persons, who sit in row 1 and row 3, face the persons, who a. U, K
sit in row 2. The distance between each of the adjacent b. R, Q
persons in all the rows is same. c. N, Y
Note: If A faces B, then they must face towards each other. d. T, V
Y doesn’t face R. S sits 2nd to the right of Z, who faces e. W, L
X. K sits 3rd to the right of Z. One person sits between M 39) Who and Who sit to the immediate right of each
and V, who faces K. Q sits adjacent to M. X sits at one of other?
the extreme ends. T and U sit 2nd to the right of each a. Z, T
other. L sits to the immediate right of P, who sits in row b. S, K
2. R faces south but neither faces P or L. O sits 2nd to the c. Z, U
right of N. Y and W also sit in the rows. Y doesn’t sit in d. Q, M
row 1. U is not facing M. e. None of these
36) How many persons sit between S and the person, 40) Who sits 2nd to the right of the person, who faces
who faces N? Y?
a. Four a. T
b. Three b. S
c. Two c. Z
d. One d. P
e. None of these e. None of these
37) Who among the following doesn’t sit in row 2? Directions 41-45: Study the following information
I. S carefully and answer the questions that follow:
II. Q Fifteen persons from A to O are sitting in three rows
III. Y namely Row-1, Row-2 and Row-3 facing north. Five
a. Both I and II persons are sitting in each row and seats of all rows are
b. Both II and III such that each seat is exactly behind or in front of another
c. Only II seat of neighboring row. Row-1 is to the north of Row-2
d. Only III and Row-2 is to the north of Row-3.
Page 54 of 1334
Get a Special Paid Bundle PDF on High Level Puzzles & Seating for All Bank Clerk/ PO Mains
If there are any suggestions/ errors in our PDFs Feel Free to contact us via this email: admin@exampundit.in
Ultra Practice Bundle PDF
Reasoning Questions for SBI Clerk/ RBI Assist. Mains
It is given that Engineers are sitting in row-1, Doctors are 43) How many persons are sitting between N and B?
sitting in row-2 and Teachers are sitting in row-3. a. Two
Two persons are sitting between A and F but neither of b. One
them is a doctor. N and A are sitting in the same row but c. Four
N is sitting at one of the ends and J is sitting exactly behind d. Three
or in front of him. E is a teacher and two persons are sitting e. Cannot be determined
between him and L but neither E nor L is sitting exactly 44) Who is sitting second to the left of D?
behind J. The number of persons sitting between N and O a. J
is the same as the number of persons sitting between E and b. I
H. I is sitting exactly behind O. K is neither an engineer c. M
nor sitting in the same row as I but he is sitting at one of d. G
the ends. G is sitting exactly behind A but not exactly in e. None of the above
front of L. C is sitting immediate to the right of H. D is not 45) Select the odd one out.
an engineer. M is sitting in the same row as G is sitting but a. F-I-K
they are not a neighbor of each other. Neither D nor M is b. D-O-C
sitting in front of H. c. K-N-D
41) Which doctor is sitting at one of the ends of the d. B-O-F
row? e. A-H-J
a. E Directions 46-50: Study the following information
b. F carefully and answer the questions that follow:
c. D Fifteen persons from A to O are sitting in three rows
d. K namely Row-1, Row-2 and Row-3 facing the stage which
e. J is located in the north direction. Five persons are sitting in
42) Select the pair of teachers among the following? each row and seats of all rows are such that each seat is
a. C-L-D exactly behind or in front of another seat of neighboring
b. I-E-K row. The price of a ticket for Row-1 is Rs. 1500 and that
c. K-C-H of the row-2 and row-3 are Rs. 1300 and Rs. 700
d. E-M-J respectively. The price for each row decreases as one
e. None of the above move away from the stage.
Page 55 of 1334
Get a Special Paid Bundle PDF on High Level Puzzles & Seating for All Bank Clerk/ PO Mains
If there are any suggestions/ errors in our PDFs Feel Free to contact us via this email: admin@exampundit.in
Ultra Practice Bundle PDF
Reasoning Questions for SBI Clerk/ RBI Assist. Mains
D is sitting third to the right of B and they are sitting in c. L-D
row-2. The total price of tickets of H and M is Rs. 2000 d. C-A
but H is not sitting in the same row as B. H is sitting e. M-I
second to the right of G but neither of them is sitting at 48) What is the total price of tickets of E and I?
ends. The total price of tickets of I and J is Rs. 400 more a. 2800
than the total price of tickets of M and D. Also both I and b. 2200
J are sitting next to each other. J is sitting third to the left c. 1400
of O. K is sitting second to the right of L. The ticket price d. 3000
of K is not less than Rs. 1000 and L is not sitting in the e. 2000
same row as O. Total ticket price of C and E is Rs. 2200. 49) How many persons are sitting between K and L?
Also, the ticket price of C is more than that of K and he is a. Three
sitting to the immediate left of A. E is sitting second to the b. Two
right of N but he is neither sitting exactly behind D nor c. No one
sitting next to G. d. One
46) Who is sitting third to the right of G? e. Can't be determined
a. L 50) Who among the following is not sitting at one of the
b. F ends of the rows?
c. E a. K
d. K b. L
e. O c. O
47) Select the odd one out. d. I
a. J-O e. F
b. H-F

Page 56 of 1334
Get a Special Paid Bundle PDF on High Level Puzzles & Seating for All Bank Clerk/ PO Mains
If there are any suggestions/ errors in our PDFs Feel Free to contact us via this email: admin@exampundit.in
Ultra Practice Bundle PDF
Reasoning Questions for SBI Clerk/ RBI Assist. Mains
Seating Parallel Row 2 Variable – Answer and Explanation
1. Answer-D
2. Answer-D
3. Answer-A
4. Answer-B
5. Answer-D
Solution 1-5
1. S sits third to the U, who sits opposite to N.
2. W sits in the middle of the row and immediate left of V
but neither sit at any end. 7. O sits opposite to R and Y.

3. N sits opposite to U and M. 8. Neither Y nor R are adjacent to M.

4. K sits second to the left of M and K sits in the 3rd row. 9. X sits to the immediate left of Z but not adjacent to S.

5. One person sits between V and I, who does not sit at 10. P and T are adjacent to each other.

any end. 11. P is not opposite to L. L is in 2nd row.

6. S does not sit at any end. W and K are not opposite to 12. P is not adjacent to U.

each other. So, case 1 is rejected.

So, we have two cases, So, we have,

Case 1:

Case 2:
6. Answer-B
7. Answer-E
8. Answer-B
Page 57 of 1334
Get a Special Paid Bundle PDF on High Level Puzzles & Seating for All Bank Clerk/ PO Mains
If there are any suggestions/ errors in our PDFs Feel Free to contact us via this email: admin@exampundit.in
Ultra Practice Bundle PDF
Reasoning Questions for SBI Clerk/ RBI Assist. Mains
9. Answer-A 8. One person sits between O and I.
10. Answer-E 9. J faces south and does not sit adjacent to T or Z.
Solution 6-10 So, case 2 is rejected.
1. P sits third to the right of X.
2. S and M sit to the immediate right of each other.
3. P faces K, who sits to the immediate left of S.
4. N faces W, who sits to the immediate right of V, who
does not sit at any end.
5. L faces Q, who sits second to the right of R.
6. Z sits to the immediate left of T, who does not sit
adjacent to V.
So, we have,
11. Answer-A
Case 1:
12. Answer-D
13. Answer-E
14. Answer-E
15. Answer-C
Solution 11-15
1. Y sits third to the left of T. Only two persons sitting
between K and T, so it is clear from here that Y, T and K
Case 2:
are sitting in Row 2.
So, there are two possibilities either T faces north or south
in row 2.
2. R sits third from an extreme end and faces north. Three
persons sit between Q and N, who faces R, the position of
R is fixed from here in both the cases.
Case I:

7. Z and I do not sit at extreme end.


Page 58 of 1334
Get a Special Paid Bundle PDF on High Level Puzzles & Seating for All Bank Clerk/ PO Mains
If there are any suggestions/ errors in our PDFs Feel Free to contact us via this email: admin@exampundit.in
Ultra Practice Bundle PDF
Reasoning Questions for SBI Clerk/ RBI Assist. Mains
16. Answer-C
17. Answer-B
18. Answer-C
19. Answer-D
20. Answer-C
Solution16-20
1. C faces the person, who sits 3rd to the right of F.
2. J sits 2nd to the left of F and adjacent to D.
Case II:
3. Neither C nor D sits at any of the extreme ends, so
Case I:

3. Z sits third to the right of V, who faces south, so case II


is invalid because H, is second to the right of the one who
faces Z. So, the position V is fixed in Row 2.
4. Neither G nor H sits at the extreme end of the row also
Case II:
one of them faces towards the south, so position of H is
fixed which means G faces the south and sitting in Row 1
because S faces the one who is on the immediate left of G.
5. E sits second to the left of J so, the only places where
both of them can be fixed is in Row 1.
The final arrangement is as follows:

Case III:

Page 59 of 1334
Get a Special Paid Bundle PDF on High Level Puzzles & Seating for All Bank Clerk/ PO Mains
If there are any suggestions/ errors in our PDFs Feel Free to contact us via this email: admin@exampundit.in
Ultra Practice Bundle PDF
Reasoning Questions for SBI Clerk/ RBI Assist. Mains

4. D faces the person, who sits 2nd to the right of P, this is


Solution (21-25)
not possible in case I, so case I is rejected.
1. Z sits at the extreme left end of Row-3.
5. P faces the person, who sits 2nd to the right of E, this is
2. W sits adjacent to Z, who sits opposite to U, who is not
not possible in case II, so case III is rejected.
adjacent to X.
Case II:

3. A sits third to the left of X, who faces Q.


4. Q sits at one of the extreme ends.
6. Only one person sits between E and A, who faces I. 5. Two persons sit between Q and I.
7. O sits immediate right of N, who doesn’t face P.
8. L and G sits adjacent to each other, so L sits immediate
right of E.
9. L doesn’t sit immediate right of F.
10. H faces B, who doesn’t sits 2nd to the right of M.
6. B sits opposite to I and B sits adjacent to C.
11. M doesn’t sit in row 3, so M sits in row 1.
7. One person sits between X and Y such that Y does not
12. K doesn’t face J, so K faces F.
sit adjacent to A.
The final seating arrangement is given below:
8. F faces north and sits exactly between G and E.
9. T faces E and sits adjacent to S.
Page 60 of 1334
Get a Special Paid Bundle PDF on High Level Puzzles & Seating for All Bank Clerk/ PO Mains
If there are any suggestions/ errors in our PDFs Feel Free to contact us via this email: admin@exampundit.in
Ultra Practice Bundle PDF
Reasoning Questions for SBI Clerk/ RBI Assist. Mains
4. G is sitting 2nd to the right of C and both of them are
facing the north.
5. E is sitting in the left of G.
6. One person is sitting between E and N.
Case 1: When Q is sitting 2nd from the left end in row 1.
10. R sits to the immediate right of H.
B is sitting to the immediate left of D, who is sitting
11. Neither V nor D sits at any end.
opposite to O, who is not sitting adjacent to A, so this case
12. V is an immediate neighbor of X.
is not possible.

21. Answer-D
22. Answer-B
23. Answer-C 7. S and P are sitting opposite to each other.

24. Answer-E Case 2: When Q is sitting at the left end of row 2. Neither

25. Answer-B S nor P is sitting in row 1, so this case is not possible.

26. Answer-A
27. Answer-B
28. Answer-A
29. Answer-B
30. Answer-C
Solution 26-30
1. F, who is sitting 2nd to the right of Q, is sitting opposite
8. R is not sitting adjacent to M.
to A.
Case 3: When Q is sitting at the right end of row 2.
2. No one is sitting in the left of A.
The final arrangement is as follows:
3. Three persons are sitting between A and M and F is
facing south.

Page 61 of 1334
Get a Special Paid Bundle PDF on High Level Puzzles & Seating for All Bank Clerk/ PO Mains
If there are any suggestions/ errors in our PDFs Feel Free to contact us via this email: admin@exampundit.in
Ultra Practice Bundle PDF
Reasoning Questions for SBI Clerk/ RBI Assist. Mains

Case III:
31. Answer-A
32. Answer-B
33. Answer-A
34. Answer-D
35. Answer-A
Solution 31-35 (Directly solved minimum 6 cases are
there, find attachment) 5. Two persons sit between O and the person who faces
1. H faces the person, who sits 3rd to the right of M. U.
2. U sits to the immediate left of F, who faces K. 6. O doesn’t sit at any extreme ends.
3. One person sits between M and K. 7. Person, who sits to the immediate right of O, faces J,
4. U doesn’t sit at any extreme ends, so this is not possible in case III, so case III is rejected.
Case I: Case I:

Case II: Case II:

Page 62 of 1334
Get a Special Paid Bundle PDF on High Level Puzzles & Seating for All Bank Clerk/ PO Mains
If there are any suggestions/ errors in our PDFs Feel Free to contact us via this email: admin@exampundit.in
Ultra Practice Bundle PDF
Reasoning Questions for SBI Clerk/ RBI Assist. Mains
37. Answer-B
38. Answer-D
39. Answer-A
40. Answer-C
Solution 36-40
1. S sits 2nd to the right of Z, who faces X.
8. J faces the person, who sits 3rd to the right of G. 2. X sits at one of the extreme ends.
9. Person, who faces G, sits to the immediate left of T. 3. K sits 3rd to the right of Z, so
10. W sits to the immediate left of E, who doesn’t sit at Case I:
any extreme ends.
11. One person sits between E and V, who faces south,
this contradicts case II, so case II is rejected.

Case II:

12. R sits to the immediate right of P.


13. Neither D nor I faces north.
14. Neither N nor L sits at any extreme ends.
15. One person sits between N and S.
The final seating arrangement is given below:

Case III:

36. Answer-C
Page 63 of 1334
Get a Special Paid Bundle PDF on High Level Puzzles & Seating for All Bank Clerk/ PO Mains
If there are any suggestions/ errors in our PDFs Feel Free to contact us via this email: admin@exampundit.in
Ultra Practice Bundle PDF
Reasoning Questions for SBI Clerk/ RBI Assist. Mains
Case IV:

Case IV:

8. L sits immediate right of P, who sit in row 2.


9. R faces south but neither face P or L.
10. O sits 2nd to the right of N
11. Y doesn’t sit in row 1, this is not possible in case II, so
case II is rejected.
12. Y doesn’t face R. U is not facing M.
4. One person sits between M and V, who faces K. The final seating arrangement is given below:
5. Q sits adjacent to M.
6. T and U sit 2nd to the right of each other, this is not
possible in case I and case IV, so case I and case IV are
rejected.
7. U faces south.
Case II:

41. Answer- E
42. Answer- C
43. Answer- A
44. Answer- C
45. Answer- D

Page 64 of 1334
Get a Special Paid Bundle PDF on High Level Puzzles & Seating for All Bank Clerk/ PO Mains
If there are any suggestions/ errors in our PDFs Feel Free to contact us via this email: admin@exampundit.in
Ultra Practice Bundle PDF
Reasoning Questions for SBI Clerk/ RBI Assist. Mains
Solution 41-45 Engineers Row-1
1. Two persons are sitting between A and F but neither of
Doctors Row-2 J
them is a doctor.
2. N and A are sitting in the same row but N is sitting at Teachers Row-3 A F N F
one of the ends and J is sitting exactly behind or in front
of him.
3. E is a teacher and two persons are sitting between him
Profession Row Case 1.1: Case1.2
and L but neither E nor L is sitting exactly behind J.
Engineers Row-1 N A F N In
F cases 3.1,
A 3.2, 4.1, 4.2 E and L cannot be placed. So,
these cases get eliminated.
Doctors Row-2 J J
Case 1.1.1:
Teachers Row-3 Profession Row North(↑)

Engineers Row- N A F
Profession Row Case 2.1 Case-2.2 1

Engineers Row-1 A F N F Doctors A Row-


N J
2
Doctors Row-2 J J
Teachers Row- E L
Teachers Row-3
3

Case 3.1:
Case 1.1.2:
Profession Row Case-3.1 Case- 3.2
Profession Row North(↑)

Engineers Row-1 N F
Engineers Row- N A F

Doctors Row-2 J J 1

Teachers Row-3 N A F N FDoctors A Row- J


2

Profession Row Case-4.1 Case-4.2

Page 65 of 1334
Get a Special Paid Bundle PDF on High Level Puzzles & Seating for All Bank Clerk/ PO Mains
If there are any suggestions/ errors in our PDFs Feel Free to contact us via this email: admin@exampundit.in
Ultra Practice Bundle PDF
Reasoning Questions for SBI Clerk/ RBI Assist. Mains
Teachers Row- L E Engineers Row- A F N
3 1

Doctors Row- J
Case 1.2.1: 2
Profession Row North(↑)
Teachers Row- E L
Engineers Row- N F A 3
1

Doctors Row- J Case 2.1.2:


2 Profession Row North(↑)

Teachers Row- E L Engineers Row- A F N


3 1

Doctors Row- J
Case 1.2.2: 2
Profession Row North(↑)
Teachers Row- L E
Engineers Row- N F A 3
1

Doctors Row- J Case 2.2.1:


2 Profession Row North(↑)

Teachers Row- L E Engineers Row- F A N


3 1

Doctors Row- J
Case 2.1.1: 2
Profession Row North(↑)

Page 66 of 1334
Get a Special Paid Bundle PDF on High Level Puzzles & Seating for All Bank Clerk/ PO Mains
If there are any suggestions/ errors in our PDFs Feel Free to contact us via this email: admin@exampundit.in
Ultra Practice Bundle PDF
Reasoning Questions for SBI Clerk/ RBI Assist. Mains
Teachers Row- E L Doctors Row- J I
3 2

Teachers Row- E H L
Case 2.2.2: 3
Profession Row North(↑)
Case 1.1.2:
Engineers Row- F A N Profession Row North(↑)
1
Engineers Row- N A O F
Doctors Row- J 1
2
Doctors Row- J I
Teachers Row- L E 2
3
Teachers Row- L H E
3
4. The number of persons sitting between N and O is the
same as the number of persons sitting between E and H.
Case 1.2.1:
5. I is sitting exactly behind O.
Profession Row North(↑)
6. There can be either 1 or 2 persons between N and O.
There can be either 0 or 1 person between E and H. Engineers Row- N F O A

As a number of person between N & O and E & H has to 1

be same. So, we can say that there is only one person


Doctors Row- J I
between them.
2
Case 1.1.1:
Profession Row North(↑) Teachers Row- E H L
3
Engineers Row- N A O F
1 Case 1.2.2:
Profession Row North(↑)

Page 67 of 1334
Get a Special Paid Bundle PDF on High Level Puzzles & Seating for All Bank Clerk/ PO Mains
If there are any suggestions/ errors in our PDFs Feel Free to contact us via this email: admin@exampundit.in
Ultra Practice Bundle PDF
Reasoning Questions for SBI Clerk/ RBI Assist. Mains
Engineers Row- N F O A Teachers Row- L H E
1 3

Doctors Row- J I Case 2.2.1:


2 Profession Row North(↑)

Teachers Row- L H E Engineers Row- F O A N


3 1

Doctors Row- I J
Case 2.1.1: 2
Profession Row North(↑)
Teachers Row- E H L
Engineers Row- A O F N 3
1

Doctors Row- I J Case 2.2.2:


2 Profession Row North(↑)

Teachers Row- E H L Engineers Row- F O A N


3 1

Doctors Row- I J
Case 2.1.2: 2
Profession Row North(↑)
Teachers Row- L H E
Engineers Row- A O F N 3
1

Doctors Row- I J 7. G is sitting exactly behind A but not exactly in front of


2 L.
In cases 1.1.2, 1.2.1, 2.1.2, 2.2.1 G is exactly in front of L.
So, these cases get eliminated.

Page 68 of 1334
Get a Special Paid Bundle PDF on High Level Puzzles & Seating for All Bank Clerk/ PO Mains
If there are any suggestions/ errors in our PDFs Feel Free to contact us via this email: admin@exampundit.in
Ultra Practice Bundle PDF
Reasoning Questions for SBI Clerk/ RBI Assist. Mains
Case 1.1.1: Doctors Row- G I J
Profession Row North(↑) 2

Engineers Row- N A O F Teachers Row- E H L


1 3

Doctors Row- J G I
2 Case 2.2.2:
Profession Row North(↑)
Teachers Row- E H L
3 Engineers Row- F O A N
1

Case 1.2.2: Doctors Row- I G J


Profession Row North(↑) 2

Engineers Row- N F O A Teachers Row- L H E


1 3

Doctors Row- J I G
2 8. C is sitting immediate to the right of H.
In Cases 1.1.1 and 2.1.1 there is no place for C on the
Teachers Row- L H E
immediate right of H. so, these cases get eliminated.
3
Case 1.2.2:
Profession Row North(↑)
Case 2.1.1:
Engineers Row- N F O A
Profession Row North(↑)
1
Engineers Row- A O F N
Doctors Row- J I G
1
2

Page 69 of 1334
Get a Special Paid Bundle PDF on High Level Puzzles & Seating for All Bank Clerk/ PO Mains
If there are any suggestions/ errors in our PDFs Feel Free to contact us via this email: admin@exampundit.in
Ultra Practice Bundle PDF
Reasoning Questions for SBI Clerk/ RBI Assist. Mains
Teachers Row- L H C E Profession Row North(↑)
3
Engineers Row- F O A N
1
Case 2.2.2:
Doctors Row- I G J
Profession Row North(↑)
2
Engineers Row- F O A N
Teachers Row- L H C E K
1
3
Doctors Row- I G J
2
10. D is not an engineer.
Teachers Row- L H C E 11. M is sitting in the same row as G is sitting but they are
3 not a neighbour of each other.
12. Neither D nor M is sitting in front of H.
In case 2.2.2, D and M cannot be placed. So, case 2.2.2
9. K is neither an engineer nor sitting in the same row as I
gets eliminated.
but he is sitting at one of the ends.
So, B is an engineer.
Case 1.2.2:
Case 1.2.2:
Profession Row North(↑)
Profession Row North(↑)
Engineers Row- N F O A
Engineers Row- N F O B A
1
1
Doctors Row- J I G
Doctors Row- J M I D G
2
2
Teachers Row- K L H C E
Teachers Row- K L H C E
3
3

Case 2.2.2:
46. Answer-C
Page 70 of 1334
Get a Special Paid Bundle PDF on High Level Puzzles & Seating for All Bank Clerk/ PO Mains
If there are any suggestions/ errors in our PDFs Feel Free to contact us via this email: admin@exampundit.in
Ultra Practice Bundle PDF
Reasoning Questions for SBI Clerk/ RBI Assist. Mains
47. Answer-E 3. H is sitting second to the right of G but neither of them
48. Answer-B is sitting at ends.
49. Answer-D Case 1:
50. Answer-B Price of Row Stage
Solution 46-50 ticket
1. D is sitting third to the right of B and they are sitting in
1500 Row-1
row-2.
Case 1: 1300 Row-2 B D
Price of Row Stage
700 Row-3 G H
ticket

1500 Row-1
Case 2:
1300 Row-2 B D Price of Row Stage
ticket
700 Row-3
1500 Row-1

Case 2: 1300 Row-2 B D


Price of Row Stage
700 Row-3 G H
ticket

1500 Row-1
4. The total price of tickets of I and J is rs. 400 more than
1300 Row-2 B D the total price of tickets of M and D. M and D is sitting in
the second row. So, the total price of the ticket of M and
700 Row-3
D is Rs. 2600. So, the total price of the ticket of I and J is
Rs.3000. i.e I and j belong to the first row.
2. The total price of tickets of H and M and is Rs. 2000 5. J is sitting third to the left of O.
but H is not sitting in the same row as B.i.e H is sitting in Case 1.1:
row 3 and M is sitting in row 2.

Page 71 of 1334
Get a Special Paid Bundle PDF on High Level Puzzles & Seating for All Bank Clerk/ PO Mains
If there are any suggestions/ errors in our PDFs Feel Free to contact us via this email: admin@exampundit.in
Ultra Practice Bundle PDF
Reasoning Questions for SBI Clerk/ RBI Assist. Mains
Price of Row Stage Price of Row Stage
ticket ticket

1500 Row-1 J O 1500 Row-1 J O

1300 Row-2 B D 1300 Row-2 B D

700 Row-3 G H 700 Row-3 G H

Case 1.2: 6. K is sitting second to the right of L. The ticket price of


Price of Row Stage K is not less than Rs. 1000 and L is not sitting in the same
ticket row as O. So, K and L are sitting in the second row.
M is sitting in the second row.
1500 Row-1 J O
Case 1.1:
1300 Row-2 B D Price of Row Stage
ticket
700 Row-3 G H
1500 Row-1 J O

Case 2.1: 1300 Row-2 B M L D K


Price of Row Stage
700 Row-3 G H
ticket

1500 Row-1 J O
Case 1.2:
1300 Row-2 B D Price of Row Stage
ticket
700 Row-3 G H
1500 Row-1 J O

Case 2.2: 1300 Row-2 B M L D K

Page 72 of 1334
Get a Special Paid Bundle PDF on High Level Puzzles & Seating for All Bank Clerk/ PO Mains
If there are any suggestions/ errors in our PDFs Feel Free to contact us via this email: admin@exampundit.in
Ultra Practice Bundle PDF
Reasoning Questions for SBI Clerk/ RBI Assist. Mains
700 Row-3 G H 1500 Row-1 I J C A O

1300 Row-2 B M L D K
Case 2.1:
700 Row-3 G H
Price of Row Stage
ticket
Case 1.2:
1500 Row-1 J O
Price of Row Stage
1300 Row-2 L B K M D ticket

700 Row-3 G H 1500 Row-1 J C A O I

1300 Row-2 B M L D K
Case 2.2:
700 Row-3 G H
Price of Row Stage
ticket
Case 2.1:
1500 Row-1 J O
Price of Row Stage
1300 Row-2 L B K M D ticket

700 Row-3 G H 1500 Row-1 I J C A O

1300 Row-2 L B K M D
7. Total ticket price of C and E is Rs. 2200. Also,the ticket
700 Row-3 G H
price of C is more than that of K and he is sitting to the
immediate left of A.i.e C and A are sitting in the first row.
Both I and J are sitting next to each other. Case 2.2:
Case 1.1: Price of Row Stage
Price of Row Stage ticket
ticket
1500 Row-1 J C A O I

Page 73 of 1334
Get a Special Paid Bundle PDF on High Level Puzzles & Seating for All Bank Clerk/ PO Mains
If there are any suggestions/ errors in our PDFs Feel Free to contact us via this email: admin@exampundit.in
Ultra Practice Bundle PDF
Reasoning Questions for SBI Clerk/ RBI Assist. Mains
1300 Row-2 L B K M D In case 2.1, E is either sitting behind D or next to G. So,
Case 2.1 gets eliminated.
700 Row-3 G H
Case 1.1:
Price of Row Stage
In case 1.2 and 2.2, I is not next to J. So, both the cases ticket
eliminated.
1500 Row-1 I J C A O
8. E is sitting second to the right of N but he is neither
sitting exactly behind D nor sitting next to G. 1300 Row-2 B M L D K
So, F is sitting in the third row.
700 Row-3 F G N H E

Download Seating Arrangement Practice Questions PDF


Get More Reasoning Practice Questions PDF

Seating Square 2 Variable


Directions (1-5) : Answer the questions based on the A, B, C, D, E, F, G and H are sitting around square table
information given below: in such a way that four of them sit at four corners of the
table while four of them sit in the middle of each of the

Page 74 of 1334
Get a Special Paid Bundle PDF on High Level Puzzles & Seating for All Bank Clerk/ PO Mains
If there are any suggestions/ errors in our PDFs Feel Free to contact us via this email: admin@exampundit.in
Ultra Practice Bundle PDF
Reasoning Questions for SBI Clerk/ RBI Assist. Mains
the four sides. The one who sit at the four corners face the (a) second to right
centre while those sit in the middle of the sides face (b) immediate left
outside. Each one of them likes different sweets i.e. Jalebi, (c) third to right
Barfi, Ghevar, Peda, Rasmalai, Gulabjamun, Rabri and (d) second to left
Rasgulla but not necessarily in the same order. (e) immediate right
The one who likes rasgulla does not faces inside. C sits 5. Who likes Gulabjamun?
third to the left of the one who likes rasgulla. The one who (a) A
likes rasmalai not an immediate neighbor of the one who (b) G
likes rasgulla. D sits on the immediate left of the one who (c) D
likes gulabjamun. G does not like gulabjamun. E likes (d) F
rabri. E is not an immediate neighbor of A. Only two (e) H
person sit between C and H. The one who likes jalebi sits 6. Who is third to the left of the one who likes Peda?
on the immediate right of H. The one who likes rasmalai (a) H
sits second to the right of G. G is an immediate neighbor (b) B
of neither H nor C. G does not sit third to the right of C. C (c) F
and F do not sit diagonally opposite. Only one person sit (d) D
between A and one who likes rasmalai. The one who likes (e) E
barfi is an immediate neighbor of E. The one who likes 7. B likes which of the following sweet??
peda is an immediate neighbor of F. The one who likes (a) Rabri
ghevar is not an immediate neighbor of H. (b) Barfi
3. What is the position of H with respect to the one (c) Jalebi
who likes Rasmalai? (d) Ghevar
(a) third to right (e) None of these
(b) second to left Directions (6-10) : Answer the questions based on the
(c) third to left information given below:
(d) Immediate left There are eight friends J, K, L, M, N, O, P and Q are
(e) Cannot be determined belongs to different countries i.e. USA, UK, Japan, Korea,
4. What is the position of one who likes Rabri with India, Nepal, Russia and France but not necessarily in the
respect to one who likes Barfi? same order. All of them are sitting around a square table
Page 75 of 1334
Get a Special Paid Bundle PDF on High Level Puzzles & Seating for All Bank Clerk/ PO Mains
If there are any suggestions/ errors in our PDFs Feel Free to contact us via this email: admin@exampundit.in
Ultra Practice Bundle PDF
Reasoning Questions for SBI Clerk/ RBI Assist. Mains
facing the centre and two on each side of the table but not (a) J
necessarily in the same order. (b) Q
L sits third to the right of K, who belongs to Nepal. P sits (c) L
second to the left of O, who is not an immediate neighbor (d) N
of K or L. The One who belongs to Korea is an immediate (e) K
neighbor of O. Three persons sit between K and the one 11. Who is sitting third to the left of the one who
who belongs to USA. K and L and also their immediate belongs from India?
neighbors do not belongs to Russia. Only one person sits (a) M
between the one who belongs Russia and M. Those who (b) Q
belongs to UK and France are immediate neighbors. L (c) K
neither belongs to UK nor France. Only one person sits (d) N
between J and the one who belongs to India. J doesnot (e) J
belongs to Russia or france. N does not belongs to Russia. 12. K sits opposite to the one who belongs to which
M and K do not sits one the same side of the table. country?
8. What is the position of Q with respect to J? (a) India
(a) third to right (b) Korea
(b) second to left (c) Russia
(c) third to left (d) Nepal
(d) fourth to right (e) UK
(e) Cannot be determined Directions (11-15): Answer the questions based on the
9. What is the position of O with respect to one who information given below:
belongs to Korea? There are eight friends A, B, C, D, E, F, G and H are sitting
(a) second to right around a square table and facing outside the centre in such
(b) immediate left away that two sits on each side of the table but not
(c) third to right necessarily in the same order. Among eight A, B, C and D
(d) second to left are doctors and E, F, G and H are nurses. Each of them
(e) immediate right belongs to different cities i.e. Mathura, Ranchi, Unnao,
10. Who sits opposite to the one who belongs to Hardoi, Jodhpur, Pune, Surat and Prayag but not
USA?
Page 76 of 1334
Get a Special Paid Bundle PDF on High Level Puzzles & Seating for All Bank Clerk/ PO Mains
If there are any suggestions/ errors in our PDFs Feel Free to contact us via this email: admin@exampundit.in
Ultra Practice Bundle PDF
Reasoning Questions for SBI Clerk/ RBI Assist. Mains
necessarily in the same order. There are combinations of (e) None of these
a doctor and nurse on each side of the table. 14. Who is sitting second to the right of the one who
The person who belongs to jodhpur sits immediate right belongs from Ranchi?
of C. D belong to Hardoi and sits opposite to the person (a) F
who belongs to Pune. G does not belongs to Ranchi. A sits (b) H
third to the right of E, who is not an immediate neighbor (c) B
of D. The person who belongs to Prayag sits opposite to (d) D
H. B sits second to the left of H who belongs to Mathura. (e) None of these
G sits third to the left of F, who belongs to Surat. 15. How many persons sits between A and C, when
11. Who sits to the immediate left of the one who counted in ant Clock wise direction?
belongs from Pune? (a) One
(a) H (b) Four
(b) B (c)Three
(c) F (d) Six
(d) G (e) None of these
(e) None of these Directions (16-20) : Answer the questions based on the
12. Who sits second to the left of the person who information given below:
belongs from Jodhpur? There are eight persons namely A, B, C, D, E, F, G and H
(a) A who all are sitting around a square table facing towards
(b) G the center. Four of them are sitting at the corners and rest
(c) C in the middle of the sides. They all like different news
(d) F channels i.e. Aaj tak, ABP news, India news, News18,
(e) None of these DD news, IndiaTv, NDTV and News24 but not
13. D is sitting third to the left of the one who belongs necessarily in the same order.
to? G does not like News24 and India news. C, who likes ABP
(a) Jodhpur news sits 2nd to left of the one who likes Aaj tak who sits
(b) Surat at middle side of the table. Three people sit between the
(c) Pune one who likes Aaj tak and D, who likes DD news. B does
(d) Prayag not like India news. H sits immediate left of D. E, sits
Page 77 of 1334
Get a Special Paid Bundle PDF on High Level Puzzles & Seating for All Bank Clerk/ PO Mains
If there are any suggestions/ errors in our PDFs Feel Free to contact us via this email: admin@exampundit.in
Ultra Practice Bundle PDF
Reasoning Questions for SBI Clerk/ RBI Assist. Mains
diagonally opposite to G. The one who likes India news (c) H
sits diagonally opposite to the one who likes News18. H (d) E
does not like India news. The one who likes NDTV sits (e) B
immediate left to the one who likes ABP news. A does not 30. Who sits opposite to the one who likes India Tv
sit next to C and he does not like Aaj tak. The one who news channel?
likes IndiaTv sits 2nd to the left of D. (a) F
26. What is the position of G with respect to E? (b) C
(a) third to right (c) E
(b) second to left (d) B
(c) third to left (e) G
(d) fourth to right Directions (21-25) : Answer the questions based on the
(e) Cannot be determined information given below:
27. What is the position of H with respect to one who M, N, O, P, Q, R, S and T are eight politicians from two
likes News24? parties BJP and AAP. They are sitting around a square
(a) second to right table and two sits on each side of the table but not
(b) immediate left necessarily in the same order. Four politician are from
(c) third to right BJP, who are facing towards the centre and four
(d) second to left politicians are from AAP, who are facing away from the
(e) immediate right centre.
28. Who likes NDTV news channel? Only one person of BJP has both the neighbors from AAP
(a) A while none of the persons from AAP has both the
(b) D neighbors from BJP. Q and T are facing to each other.
(c) G Both the immediate neighbors of M are from same party
(d) B as M. O and R belongs to AAP and only Q is sitting
(e) E between them. S is facing away from the centre and is
29. Who sits second to the right of one who likes Aaj sitting third to the left of T. The person immediate left of
Tak? N is from same party as N.
(a) A 21. Who is to the immediate left of M?
(b) F (a) M
Page 78 of 1334
Get a Special Paid Bundle PDF on High Level Puzzles & Seating for All Bank Clerk/ PO Mains
If there are any suggestions/ errors in our PDFs Feel Free to contact us via this email: admin@exampundit.in
Ultra Practice Bundle PDF
Reasoning Questions for SBI Clerk/ RBI Assist. Mains
(b) P (e) None of these
(c) O Directions (26-30) : Answer the questions based on the
(d) J information given below:
(e) N There are eight persons P, Q, R, S, T, U, V and W are
22. Who sits second to the left of Q? sitting around a square table in such a way that all are
(a) T facing towards the centre but not necessarily in the same
(b) P order. Four of them are sitting at the corners and rest in
(c) J the middle of the sides. All of them like different colours
(d) N i.e. Blue, Red, Black, Green, Yellow, Orange, White and
(e) None of these Pink.
23. Who is sitting third to the left of one who sits The person who likes Blue is sitting fifth to the right of Q.
second to the right of P? T is sitting third to the left of S who is also sitting third to
(a) S the left of the person who likes Blue. The person who likes
(b) L White is neither an immediate neighbor of S nor of T. R
(c) P is sitting fourth to the left of V who does not like Blue.
(d) M Only one person is sitting between U and W. The person
(e) None of these who likes Orange is sitting fifth to the left of W. The
24. Who is sitting fourth to the right of M? person who likes Green is sitting sixth to the right of the
(a) T person who likes Orange. The person who likes Red is an
(b) O immediate neighbor of the person who likes Pink. S does
(c) Q not like Pink. W does not like Yellow. The one who likes
(d) P blue is sitting at middle side of the table.
(e) None of these 26. What is the position of T with respect to the
25. Who sits opposite to the person whose immediate person who likes orange colour?
neighbors are from AAP party? (a) third to right
(a) P (b) second to left
(b) L (c) third to left
(c) T (d) fourth to right
(d) S (e) Cannot be determined
Page 79 of 1334
Get a Special Paid Bundle PDF on High Level Puzzles & Seating for All Bank Clerk/ PO Mains
If there are any suggestions/ errors in our PDFs Feel Free to contact us via this email: admin@exampundit.in
Ultra Practice Bundle PDF
Reasoning Questions for SBI Clerk/ RBI Assist. Mains
27.What is the position of S with respect to the Directions (31-35) : Answer the questions based on the
person who likes white colour? information given below:
(a) second to right There are eight friends A, B, C, D, E, F, G and H sitting
(b) third to right around a square table facing away from the centre but not
(c) immediate left necessarily in the same order. Four of them are sitting at
(d) second to left the corners and rest in the middle of the sides. Each one
(e) immediate right of them likes different movies i.e. Tanhaji, War, Baaghi,
28. Who sits third to the right of one who likes Green Malang, Chapaak, Panga, Thappad and Gully boy but not
colour? necessarily in the same order.
(a) S A likes tanhaji and is not an immediate neighbor of the
(b) P one who likes panga. The one who likes the malang sits
(c) W on the immediate left of B, who likes war. C likes panga
(d) Q and sits third to the left of E. The one who likes gullyboy
(e) R and the one who likes tanhaji are immediate neighbors of
29.Who sits immediate right of the person who likes each other. The one who likes chapaak and the one who
yellow colour? likes malang are immediate neighbors of each other but
(a) Who likes orange colour. both of them are neither an immediate neighbors of E nor
(b) Who likes green colour. C. Neither E nor D likes gullyboy. Only F sits between the
(c) Who likes blue colour. one who likes malang and the one who likes baghi. G sits
(d) Who likes pink colour. third to the left of the person who likes gullyboy.
(e) Who likes red colour. 31. Who sits to the immediate right of one who likes
30.How many person sits between R and V from Baghi movie?
right of V? (a) F
(a) Six (b) A
(b) Four (c) E
(c) None (d) B
(d) Two (e) H
(e) Three
32. Who among the following likes War movie?
Page 80 of 1334
Get a Special Paid Bundle PDF on High Level Puzzles & Seating for All Bank Clerk/ PO Mains
If there are any suggestions/ errors in our PDFs Feel Free to contact us via this email: admin@exampundit.in
Ultra Practice Bundle PDF
Reasoning Questions for SBI Clerk/ RBI Assist. Mains
(a) D facing the centre and other four are facing away from the
(b) G centre and two sits on each side of the table but not
(c) H necessarily in the same order. Each of them likes different
(d) F subjects i.e. Physics, Chemistry, English, Quant,
(e) B Reasoning, GA, Computer and Economics but not
33. Who sits third to the left of the person who likes necessarily in the same order.
Panga movie? E faces towards the centre and likes Physics. Both the
(a) A immediate neighbors of E faces away from the centre and
(b) D likes English and Quant. D faces away from the centre.
(c) G Both the immediate neighbors of D do not faces away
(d) E from the centre. E sits third to the right of F, who likes
(e) F Reasoning. C sits third to the left of F. The one who likes
34. Which movie does E like? English sits immediate right of C. The one who likes
(a) Tanhaji Chemistry is not the immediate neighbor of F and faces
(b) Gullyboy away from the centre. A sits second to the left of C and
(c) Thappad does not like Computer and Economics. The one who
(d) Baghi likes Computer sits between F and H. B faces away from
(e) Chapaak the centre and does not like Chemistry. F faces away from
35. How many persons sit between those who likes War the centre. A and F sits on the same side of the table.
and Baghi from left of War? 36. Who sits to the immediate left of one who likes
(a) One Quant?
(b) Four (a) G
(c) Six (b) A
(d) Two (c) D
(e) Three (d) E
Directions (36-40) : Answer the questions based on the (e) B
information given below: 37. Four of the following five are alike in a certain way
There are eight exampundit’s faculty A, B, C, D, E, F, G based on the given arrangement and hence form a
and H sitting around a square table and Four of them are
Page 81 of 1334
Get a Special Paid Bundle PDF on High Level Puzzles & Seating for All Bank Clerk/ PO Mains
If there are any suggestions/ errors in our PDFs Feel Free to contact us via this email: admin@exampundit.in
Ultra Practice Bundle PDF
Reasoning Questions for SBI Clerk/ RBI Assist. Mains
group. Which is the one that doesn’t belong to that There are eight persons P, Q, R, S, T, U, V and W are
group? sitting around a square table four of them are facing the
(a) A centre and other four are facing away from the centre and
(b) G two sits on each side but not necessarily in the same order.
(c) B They are working in different companies i.e. HCL, Wipro,
(d) E TATA, Reliance, AIRTEL, Jio, Idea and Bsnl but not
(e) C necessarily in the same order.
38. Who sits second to the right of A ? T sits to immediate left of S and faces outside the centre.
(a) Who likes English S works in TATA and faces to the centre.The person who
(b) Who likes Economics works in Relaince and AIRTEL are facing same
(c) Who likes Computer direction.T is sitting between the person who works in
(d) Who likes Reasoning TATA and the one who works in Bsnl. S and T sit on the
(e) Who likes Physics same side of the table. The person who works in Idea is
39. Who is second to the left of one who likes Physics? facing outside the centre and an immediate neighbor of
(a) G who works in Bsnl. T does not works in Idea. W is an
(b) B immediate neighbor of the person who works in AIRTEL
(c) H and Idea. U faces the centre and sits third to the right of
(d) C R.R works in Wipro. V sits third to the right of Q, who
(e) A works in HCL. P works in in AIRTEL.
40. How many persons sit between C and one who likes 41. Who is sitting third to the right of R?
Chemistry when counted from left of anyone? (a) W
(a) One (b) S
(b) Four (c) P
(c) Six (d) U
(d) Two (e) None of these
(e) None 42. What is the position of W with the person who
Directions (41-45) : Answer the questions based on the works in Idea?
information given below: (a) Immediate right
(b) Immediate left
Page 82 of 1334
Get a Special Paid Bundle PDF on High Level Puzzles & Seating for All Bank Clerk/ PO Mains
If there are any suggestions/ errors in our PDFs Feel Free to contact us via this email: admin@exampundit.in
Ultra Practice Bundle PDF
Reasoning Questions for SBI Clerk/ RBI Assist. Mains
(c) Third to the right There are eight sports personality i.e. A, B, C, D, E, F, G
(d) Third to the left and H representing their games i.e. Cricket, Football,
(e) None of these Chess, Tennis, Carrom, Boxing, Handball and Racing are
43. What is T’s position with respect to one who works sitting around a square table facing outside the centre.
in BSNL? Four of them are sitting at the corners and rest in the
a) Third to the right middle of the sides but not necessarily in the same order.
b) Second to the right C sits at corner. Three people sit between H and who
c) Immediate right represents Handball. Only one person sits between A and
d) Second to the left who represents Boxing. A does not represents Chess and
e) Third to the left Cricket. C sits third to the right of H. H represents Racing.
44. Who is sitting third to the right of the person who E is not an immediate neighbor of either C or H. The one
works in Wipro? who represents Football is an immediate neighbor of E. B
(a) U is not representing either Chess or Football. F sits third to
(b) P the left of E. C and H and also their immediate neighbors
(c) R do not represents Chess. Only one person sits between G
(d) W and the person who represents Chess. The person who
(e) T represents Cricket and Tennis are immediate neighbors of
45. Which one of the following statements is true each other. C represents neither Carrom nor Tennis.
according to the above mentioned arrangement? 46. Who is to the immediate left of the person who
a) P is to the immediate left of one who works in represents Handball?
TATA. (a) B
b) W sits second to the right of one who works in (b) C
HCL. (c) D
c) V sits second to right of one who works in Airtel. (d) G
d) T sits fourth to the right of one who works in Jio. (e) None of these
e) None of these 47. Who represents Tennis game?
Directions (46-50) : Answer the questions based on the (a) C
information given below: (b) E
(c) G
Page 83 of 1334
Get a Special Paid Bundle PDF on High Level Puzzles & Seating for All Bank Clerk/ PO Mains
If there are any suggestions/ errors in our PDFs Feel Free to contact us via this email: admin@exampundit.in
Ultra Practice Bundle PDF
Reasoning Questions for SBI Clerk/ RBI Assist. Mains
(d) A (c) G and H
(e) None of these (d) B and A
48. Who is sitting third to the left of B? (e) None of these
(a) A 50. Who is sitting fourth to the right of one who
(b) G represents represent
(c) D Carrom?
(d) F (a) E
(e) None of these (b) D
49. Who is the immediate neighbors of the person who (c) F
represents Chess? (d) B
(a) B and D (e) C
(b) C and D

Seating Square 2 Variable – Answer and Explanation


SOLUTION(1-5):
Explanation in detail:
1. The one who likes rasgulla does not faces inside.
2. C sits third to the left of the one who likes rasgulla.
3. Only two person sit between C and H.
4. The one who likes jalebi sits on the immediate
right of H.

Page 84 of 1334
Get a Special Paid Bundle PDF on High Level Puzzles & Seating for All Bank Clerk/ PO Mains
If there are any suggestions/ errors in our PDFs Feel Free to contact us via this email: admin@exampundit.in
Ultra Practice Bundle PDF
Reasoning Questions for SBI Clerk/ RBI Assist. Mains
7. The one who likes rasmalai not an immediate
neighbor of the one who likes rasgulla.

5. G is an immediate neighbor of neither H nor C.


6. The one who likes rasmalai sits second to the right
of G.

Page 85 of 1334
Get a Special Paid Bundle PDF on High Level Puzzles & Seating for All Bank Clerk/ PO Mains
If there are any suggestions/ errors in our PDFs Feel Free to contact us via this email: admin@exampundit.in
Ultra Practice Bundle PDF
Reasoning Questions for SBI Clerk/ RBI Assist. Mains

But it is given that G does not sit third to right of C. So


CASE2 is cancelled out.
8. Only one person sit between A and one who likes
rasmalai.
9. D sits on the immediate left of the one who likes
gulabjamun. 1. G does not like gulabjamun.
10. E likes rabri.
11. The one who likes barfi is an immediate neighbor
of E.
12. The one who likes peda is an immediate neighbor
of F.
13. E is not an immediate neighbor of of A.
So CASE1(a1) is cancelled out

Page 86 of 1334
Get a Special Paid Bundle PDF on High Level Puzzles & Seating for All Bank Clerk/ PO Mains
If there are any suggestions/ errors in our PDFs Feel Free to contact us via this email: admin@exampundit.in
Ultra Practice Bundle PDF
Reasoning Questions for SBI Clerk/ RBI Assist. Mains

14. The one who likes ghevar is not an immediate


neighbor of H.
1. a
So CASE1(a2) is cancelled out.
2. e
15. G does not sit third to the right of C.
3. e
16. C and F do not sit diagonally opposite.
4. b
5. d
SOLUTION(6-10):
Explanation in detail:
Page 87 of 1334
Get a Special Paid Bundle PDF on High Level Puzzles & Seating for All Bank Clerk/ PO Mains
If there are any suggestions/ errors in our PDFs Feel Free to contact us via this email: admin@exampundit.in
Ultra Practice Bundle PDF
Reasoning Questions for SBI Clerk/ RBI Assist. Mains
1. L sits third to the right of K, who belongs to Nepal. 1. The One who belongs to Korea is an immediate
2. P sits second to the left of O, who is not an neighbor of O.
immediate neighbor of K or L. 2. Only one person sits between J and the one who
3. Three persons sit between K and the one who belongs to India.
belongs to USA. 3. Those who belongs to UK and France are
immediate neighbors.
4. L neither belongs to UK nor France.
5. J does not belongs to Russia or France.
6. N does not belongs to Russia.

4. K and L and also their immediate neighbors do not


belongs to Russia.
5. Only one person sits between the one who belongs
Russia and M.
6. M and K do not sits one the same side of the table. 6. d
So CASE2 is cancelled out. 7. b
Page 88 of 1334
Get a Special Paid Bundle PDF on High Level Puzzles & Seating for All Bank Clerk/ PO Mains
If there are any suggestions/ errors in our PDFs Feel Free to contact us via this email: admin@exampundit.in
Ultra Practice Bundle PDF
Reasoning Questions for SBI Clerk/ RBI Assist. Mains
8. a
9. c
10. c
SOLUTION(11-15):
Explanation in detail:
1. B sits second to the left of H who belongs to
Mathura.
2. The person who belongs to Prayag sits opposite to
H.

3. A sits third to the right of E, who is not an


immediate neighbor of D.

Page 89 of 1334
Get a Special Paid Bundle PDF on High Level Puzzles & Seating for All Bank Clerk/ PO Mains
If there are any suggestions/ errors in our PDFs Feel Free to contact us via this email: admin@exampundit.in
Ultra Practice Bundle PDF
Reasoning Questions for SBI Clerk/ RBI Assist. Mains

4. G sits third to the left of F. 5. E is not an immediate neighbor of D.

There is no such space for F and G in CASE1b and There is no space for D in CASE2a so CASE2a is

CASE2b. SO CASE1b and CASE2b is cancelled out. cancelled out.


6. The person who belongs to Jodhpur sits immediate
right of C.
7. D belong to Hardoi and sits opposite to the person
who belongs to Pune.
8. G does not belongs to Ranchi.
9. F belongs to Surat.

Page 90 of 1334
Get a Special Paid Bundle PDF on High Level Puzzles & Seating for All Bank Clerk/ PO Mains
If there are any suggestions/ errors in our PDFs Feel Free to contact us via this email: admin@exampundit.in
Ultra Practice Bundle PDF
Reasoning Questions for SBI Clerk/ RBI Assist. Mains

4. The one who likes IndiaTv sits 2nd to the left of


D.
5. The one who likes NDTV sits immediate left to the
11. c one who likes ABP news.
12. b
13. a
14. d
15. c
SOLUTION(16-20):
Explanation in detail:
1. C, who likes ABP news sits 2nd to left of the one
who likes Aaj tak who sits at middle side of the
table.
2. Three people sit between the one who likes Aaj tak 6. The one who likes India news sits diagonally
and D, who likes DD news. opposite to the one who likes News18.
3. H sits immediate left of D. 7. H does not like India news.
8. E, sits diagonally opposite to G.
9. A does not sit next to C and he does not like Aaj
tak.
10. G does not like News24 and India news.
11. B does not like India news.
Page 91 of 1334
Get a Special Paid Bundle PDF on High Level Puzzles & Seating for All Bank Clerk/ PO Mains
If there are any suggestions/ errors in our PDFs Feel Free to contact us via this email: admin@exampundit.in
Ultra Practice Bundle PDF
Reasoning Questions for SBI Clerk/ RBI Assist. Mains
2. O and R belongs to AAP and only Q is sitting
between them.

3. S is facing away from the centre and is sitting third


16. d to the left of T.
17. a
18. c
19. a
20. b
SOLUTION(21-25):
Explanation in detail:
1. Q and T are facing to each other. It means Q and
T are from BJP.
Page 92 of 1334
Get a Special Paid Bundle PDF on High Level Puzzles & Seating for All Bank Clerk/ PO Mains
If there are any suggestions/ errors in our PDFs Feel Free to contact us via this email: admin@exampundit.in
Ultra Practice Bundle PDF
Reasoning Questions for SBI Clerk/ RBI Assist. Mains

There is no such space for S so CASE1 is cancelled out.


21. b
1. Only one person of BJP has both the neighbors
22. d
from AAP while none of the persons from AAP
23. a
has both the neighbors from BJP.
24. c
2. Both the immediate neighbors of M are from same
25. c
party as M.
SOLUTION(26-30):
3. The person immediate left of N is from same party
Explanation in detail:
as N.
1. The one who likes blue is sitting at middle side of
the table.
2. The person who likes Blue is sitting fifth to the
right of Q.
3. T is sitting third to the left of S.
4. S is sitting third to the left of the person who likes
Blue.

Page 93 of 1334
Get a Special Paid Bundle PDF on High Level Puzzles & Seating for All Bank Clerk/ PO Mains
If there are any suggestions/ errors in our PDFs Feel Free to contact us via this email: admin@exampundit.in
Ultra Practice Bundle PDF
Reasoning Questions for SBI Clerk/ RBI Assist. Mains

5. The person who likes White is neither an


immediate neighbor of S nor of T.
6. R is sitting fourth to the left of V who does not like
Blue.

7. The person who likes Orange is sitting fifth to the


left of W.
8. Only one person is sitting between U and W.

There is no space for U in CASE3 so CASE3 is


cancelled out.

Page 94 of 1334
Get a Special Paid Bundle PDF on High Level Puzzles & Seating for All Bank Clerk/ PO Mains
If there are any suggestions/ errors in our PDFs Feel Free to contact us via this email: admin@exampundit.in
Ultra Practice Bundle PDF
Reasoning Questions for SBI Clerk/ RBI Assist. Mains

9. The person who likes Green is sitting sixth to the


right of the person who likes Orange.

10. The person who likes Red is an immediate


neighbor of the person who likes Pink.
11. S does not like Pink.
12. W does not like Yellow.

There is
no such space for Green in CASE2 so CASE2 is
cancelled out.

Page 95 of 1334
Get a Special Paid Bundle PDF on High Level Puzzles & Seating for All Bank Clerk/ PO Mains
If there are any suggestions/ errors in our PDFs Feel Free to contact us via this email: admin@exampundit.in
Ultra Practice Bundle PDF
Reasoning Questions for SBI Clerk/ RBI Assist. Mains
3. The one who likes chapaak and the one who likes
Malang are immediate neighbors of each other but
both of them are neither an immediate neighbors
of E nor C.

26. b
27. d
28. a
29. a 4. The one who likes gullyboy and the one who likes
30. e tanhaji are immediate neighbors of each other.
SOLUTION(31-35): 5. G sits third to the left of the person who likes
Explanation in detail: gullyboy.
1. The one who likes the Malang sits on the
immediate left of B, who likes war.
2. Only F sits between the one who likes Malang and
the one who likes baghi.

Page 96 of 1334
Get a Special Paid Bundle PDF on High Level Puzzles & Seating for All Bank Clerk/ PO Mains
If there are any suggestions/ errors in our PDFs Feel Free to contact us via this email: admin@exampundit.in
Ultra Practice Bundle PDF
Reasoning Questions for SBI Clerk/ RBI Assist. Mains

6. C likes panga and sits third to the left of E.

There is no space for E so CASE2b is cancelled out.


7. The one who likes chapaak and the one who likes
malang are neither an immediate neighbors of E
nor C.

Page 97 of 1334
Get a Special Paid Bundle PDF on High Level Puzzles & Seating for All Bank Clerk/ PO Mains
If there are any suggestions/ errors in our PDFs Feel Free to contact us via this email: admin@exampundit.in
Ultra Practice Bundle PDF
Reasoning Questions for SBI Clerk/ RBI Assist. Mains

According to question one who likes chapaak and the 31. a


one who likes malang are neither an immediate 32. e

neighbors of E nor C. So CASE1 is cancelled out. 33. e


34. c
8. Neither E nor D likes gullyboy. 35. d
9. A likes tanhaji and is not an immediate neighbor SOLUTION(36-40):
of the one who likes panga. Explanation in detail:
1. F faces away from the centre.
2. E sits third to the right of F, who likes Reasoning.
3. C sits third to the left of F.

Page 98 of 1334
Get a Special Paid Bundle PDF on High Level Puzzles & Seating for All Bank Clerk/ PO Mains
If there are any suggestions/ errors in our PDFs Feel Free to contact us via this email: admin@exampundit.in
Ultra Practice Bundle PDF
Reasoning Questions for SBI Clerk/ RBI Assist. Mains
4. E faces towards the centre and likes Physics.

A and F sits on the same side of the table. So CASE2 is


cancelled out.

5. Both the immediate neighbors of E faces away from


the centre and likes English and Quant.
6. A sits second to the left of C and does not like
Computer and Economics.
7. The one who likes Computer sits between F and H.

8. D faces away from the centre.


9. Both the immediate neighbors of D do not faces away
from the centre.
Page 99 of 1334
Get a Special Paid Bundle PDF on High Level Puzzles & Seating for All Bank Clerk/ PO Mains
If there are any suggestions/ errors in our PDFs Feel Free to contact us via this email: admin@exampundit.in
Ultra Practice Bundle PDF
Reasoning Questions for SBI Clerk/ RBI Assist. Mains
10. The one who likes English sits immediate right of C. Explanation in detail:
11. The one who likes Chemistry is not the immediate 1. T sits to immediate left of S and faces outside the
neighbor of F and faces away from the centre. centre.
12. B faces away from the centre and does not like 2. S works in TATA and faces to the centre.
Chemistry. 3. The person who works in Relaince and AIRTEL
are facing same direction.
4. T is sitting between the person who works in
TATA and the one who works in Bsnl.

36. a
37. c
38. b 5. S and T sit on the same side of the table.

39. d So CASE2 is cancelled out.

40. e
SOLUTION(41-45):
Page 100 of 1334
Get a Special Paid Bundle PDF on High Level Puzzles & Seating for All Bank Clerk/ PO Mains
If there are any suggestions/ errors in our PDFs Feel Free to contact us via this email: admin@exampundit.in
Ultra Practice Bundle PDF
Reasoning Questions for SBI Clerk/ RBI Assist. Mains

6. The person who works in Idea is facing outside the


centre and an immediate neighbor of who works in 41. d
Bsnl. 42. a
7. T does not works in Idea. 43. c
8. W is an immediate neighbor of the person who 44. a
works in AIRTEL and Idea. 45. b
9. U faces the centre and sits third to the right of R. SOLUTION(46-50):
10. R works in Wipro. Explanation in detail:
11. V sits third to the right of Q, who works in HCL. 1. C sits third to the right of H.
12. P works in in AIRTEL. 2. C sits at corner.
3. H represents Racing.
4. E is not an immediate neighbor of either C or H.
5. F sits third to the left of E.
6. Three people sit between H and who represents
Handball.

Page 101 of 1334


Get a Special Paid Bundle PDF on High Level Puzzles & Seating for All Bank Clerk/ PO Mains
If there are any suggestions/ errors in our PDFs Feel Free to contact us via this email: admin@exampundit.in
Ultra Practice Bundle PDF
Reasoning Questions for SBI Clerk/ RBI Assist. Mains
13. A does not represents Chess and Cricket.
14. B is not representing either Chess or Football.

7. C and H and also their immediate neighbors do not


represents Chess.
8. Only one person sits between G and the person
who represents Chess.
9. The one who represents Football is an immediate
neighbor of E.

46. b
47. d
10. The person who represents Cricket and Tennis are
48. a
immediate neighbors of each other.
49. a
11. C represents neither Carrom nor Tennis.
50. e
12. Only one person sits between A and who
represents Boxing.

Get More Reasoning Practice Questions PDF


Page 102 of 1334
Get a Special Paid Bundle PDF on High Level Puzzles & Seating for All Bank Clerk/ PO Mains
If there are any suggestions/ errors in our PDFs Feel Free to contact us via this email: admin@exampundit.in
Ultra Practice Bundle PDF
Reasoning Questions for SBI Clerk/ RBI Assist. Mains
Seating Rectangle 2 Variable
Directions (1-5): Answer the questions based on the (d) Immediate left
information given below: (e) Cannot be determined
P, Q, R, S, T, U, V and W are sitting around a rectangular 2. What is the position of the person who teach
table and all are facing towards the centre, not necessarily Mechanical branch with respect to one who teach
in the same order. Each of them teach different Power branch?
engineering branches i.e. Civil, Mechanical, Chemical, (a) second to right
Electrical, Optical, Power, Computer and Geological. (b) immediate left
Four of them are sitting at the corners and rest in the (c) third to right
middle of the sides. (d) second to left
The person who teach Power is sitting fifth to the left of (e) immediate right
W who sits at middle of the table. The person who teach 3. Who teach Electrical branch?
Civil is sitting fifth to the right of Q. T is sitting third to (a) V
the left of S who is also sitting third to the left of the (b) R
person who teach Civil. The person who teach Computer (c) U
is neither an immediate neighbour of S nor of T. R is (d) Q
sitting fourth to the left of V who does not teach Civil. (e) T
Only one person is sitting between U and W. The person 4. Who is third to the left of one who teach
who teach Electrical is sitting sixth to the right of the Geological branch?
person who teach Power. The person who teach (a) T
Mechanical is an immediate neighbour of the person who (b) Q
teach Geological. S does not teach Geological. W does not (c) S
teach Optical. (d) W
1. What is the position of V with respect to the (e) R
person who teach Chemical branch? 5. P teaches which of the following branch?
(a) third to right (a) Power
(b) second to left (b) Electrical
(c) third to left (c) Computer

Page 103 of 1334


Get a Special Paid Bundle PDF on High Level Puzzles & Seating for All Bank Clerk/ PO Mains
If there are any suggestions/ errors in our PDFs Feel Free to contact us via this email: admin@exampundit.in
Ultra Practice Bundle PDF
Reasoning Questions for SBI Clerk/ RBI Assist. Mains
(d) Geological (b) second to left
(e) None of these (c) third to right
Directions (6-10) : Answer the questions based on the (d) fourth to right
information given below: (e) Cannot be determined
There are eight friends A, B, C, D, E, F, G and H sitting 7. What is the position of E with respect to the one
around a rectangular table and all are facing towards the who purchase N99 mask?
centre and two sits on each side of the table but not (a) second to right
necessarily in the same order. Each one of them purchased (b) immediate left
different masks i.e. K95, K99, N95, N99, P95, P99, R95 (c) third to right
and R99 from four different pharmacy i.e. Netmeds, (d) second to left
Apollo, Medlife and MG10 but not necessarily in the same (e) immediate right
order. Not more than two friends buy from same 8. Who sits third to left of the one who purchase K95
pharmacy and no two friends who shops from same mask?
pharmacy are sitting together. (a) D
H who purchases R95 sits on the immediate right of C. (b) G
There are two friends sitting between those who are (c) B
shopping from Netmeds. B sits second to the right of D, (d) A
who purchases N99. Two friends who purchases from (e) C
Apollo sits fourth to the right of each other. F does not sit 9. Who is sitting exactly between those who shop
exactly between D and B. F faces who buys N99 mask. F from MG10 pharmacy?
purchase K95 but not from MG10. E shop from MG10. (a) F
F is an immediate neighbor of B who purchases K99. The (b) C
one who purchases P99 is an immediate neighbor of those (c) A
who purchases from MG10. The one who shop from (d) G
MG10 sits second to the right of F. D does not shop from (e) E
Apollo. G shop from Medlife and he does not purchase 10. A sits opposite to the one who purchase mask
P95. C purchases R99 from Medlife. from?
6. What is the position of C with respect to D? (a) MG10
(a) third to right (b) Apollo
Page 104 of 1334
Get a Special Paid Bundle PDF on High Level Puzzles & Seating for All Bank Clerk/ PO Mains
If there are any suggestions/ errors in our PDFs Feel Free to contact us via this email: admin@exampundit.in
Ultra Practice Bundle PDF
Reasoning Questions for SBI Clerk/ RBI Assist. Mains
(c) Medlife (e) None of these
(d) Netmeds 12. Who sits second to the left of the person who
(e) None of these belongs from SGRH?
Directions (11-15) : Answer the questions based on the (a) A
information given below: (b) G
There are eight doctors A, B, C, D, E, F, G and H sitting (c) C
around a rectangular table and all are facing towards the (d) E
centre but not necessarily in the same order. Four of them (e) None of these
are sitting at the corners and rest in the middle of the sides. 13. G is sitting third to the left of whom ?
Each one of them belongs to different hospitals in Delhi (a) MAX
i.e. Apollo, Fortis, LNJP, RML, AIIMS, SGRH, MAX (b) Apollo
and Kailash but not necessarily in the same order. (c) RML
A, who is from LNJP is sitting exactly between F and H. (d) Kailash
Neither B nor H is an immediate neighbor of E. F is from (e) None of these
RML and is sitting third to the right of the person who 14. Who is sitting second to the right of one who
from Fortis. The person from SGRH is sitting second to belongs from Fortis?
the left of the person from RML. The person from MAX (a) F
is sitting second to the left of E. The person from AIIMS (b) B
is sitting second to the right of the person from LNJP. C (c) G
is sitting third to the left of F. E is sitting third to the left (d) D
of G. The person who is from Fortis is to the immediate (e) None of these
right of E and E is not from Apollo. B is sitting fourth to 15. How many persons sits between A and C?
the right of H. H is not an immediate neighbor of G. (a) One
11. Who sits to the immediate left of one who belongs (b) Four
from LNJP? (c)Three
(a) H (d) Six
(b) F (e) None of these
(c) B Directions (16-20) : Answer the questions based on the
(d) G information given below:
Page 105 of 1334
Get a Special Paid Bundle PDF on High Level Puzzles & Seating for All Bank Clerk/ PO Mains
If there are any suggestions/ errors in our PDFs Feel Free to contact us via this email: admin@exampundit.in
Ultra Practice Bundle PDF
Reasoning Questions for SBI Clerk/ RBI Assist. Mains
There are ten persons A, B, C, D, E, P, Q, R, S and T are the table. S sits third to the left of P. E is not an immediate
sitting around a rectangular table. Four of them sitting at neighbour of S. Q sits third to the left of A.
corner, two on each longer sides and one on each smaller 16. What is the position of A with respect to Q?
side of the table but not necessarily in the same order. (a) third to right
Among them five are facing to the centre and others five (b) second to left
facing away from centre. Each of them belongs to (c) third to left
different automotive industry in India i.e. Ashok, Tata, (d) fourth to right
Suzuki, Renault, Hyundai, Ford, Force, Kia, Jeep and (e) Cannot be determined
Maruti but not necessarily in the same order. 17. What is the position of D with respect to the one
Q belongs to Ashok and sits third to the left of one who who likes Hyundai?
belongs to Jeep. E belongs to Renault and sits second to (a) second to right
the left of one who belongs to Maruti. The one who (b) immediate left
belongs to Jeep sits diagonally opposite to who belongs to (c) third to right
TATA.C sits immediate right of S, who is facing away (d) second to left
from the centre. R belongs neither to Hyundai nor Kia and (e) immediate right
facing to the centre. The one who belongs to Suzuki sits 18. Who belongs to Jeep automotive industry?
on the immediate right to C, who belongs to TATA. E sits (a) C
to the immediate left of Q. P sits third to the right of T, (b) A
who belongs to Maruti. The one who belongs to Hyundai (c) G
sits second to the left of S. A sits on the immediate right (d) B
of one who belongs to Kia. T is not an immediate (e) E
neighbour of C who facing inside the centre. R sits on the 19. Who sits second to the right of the one who belongs
immediate left of A. Both B and Q sit diagonally opposite to Suzuki automotive industry?
each other and both faces opposite direction. P sits smaller (a) F
side of the table. E and P faces same direction. The one (b) A
who belongs to Ford sits on the immediate left of who (c) H
belongs to Hyundai. C is neither immediate neighbour of (d) E
P nor D. Only two persons who sits at corner facing inside (e) B

Page 106 of 1334


Get a Special Paid Bundle PDF on High Level Puzzles & Seating for All Bank Clerk/ PO Mains
If there are any suggestions/ errors in our PDFs Feel Free to contact us via this email: admin@exampundit.in
Ultra Practice Bundle PDF
Reasoning Questions for SBI Clerk/ RBI Assist. Mains
20. Who sits opposite to the one who belongs to Kia of G. G does not like Jadeja. The one who likes Rahul sits
automotive industry? immediately right to the one who likes Dhawan. A facing
(a) F away from the centre. Immediate left of A likes Dhoni,
(b) C who faces the opposite to centre. E sits second to the left
(c) E of A.
(d) B 21. Who is to the immediate left of one who likes
(e) G Rohit?
Directions (21-25) : Answer the questions based on the (a) B
information given below: (b) F
Eight persons - A, B, C, D, E, F, G and H are sitting around (c) H
a rectangular table, but not necessarily in the same order. (d) D
Some of them are facing inside while rest of them are (e) A
facing outside and two sits on each side of the table but 22. Who sits second to the left of B?
not necessarily in the same order. Each of them likes (a) Who likes Rohit
different cricket players i.e. Kohli, Dhoni, Jadeja, Pant, (b) Who likes Pant
Rahul, Dhawan, Rohit and Sanju. (c) Who likes Rahul
A and who likes Dhoni doesnot sit on the same side of the (d) Who likes Dhoni
table. F and E are facing the same side and only two (e) None of these
persons are sitting between them. F does not sit 23. Who is sitting third to the left of the person who
immediately next to A. B sits to the immediate right of E sits second to the right of H?
and both E and B are facing same direction. A and C are (a) C
facing outward direction but not as F. D sits fourth to right (b) F
of B. The one who likes Jadeja sits between F and B. Only (c) H
two persons sit between G and H. The one who likes (d) D
Dhawan sits fourth to the left of the one who likes Jadeja. (e) None of these
D is facing same direction as F. The one who likes Sanju 24. Who is sitting fourth to the right of one who likes
sits second to the right of the one who likes Jadeja. The Dhoni?
one who likes Rohit sits fourth to the right to the one who (a) H
likes Sanju. F does not like Kohli. C sits second to the left (b) D
Page 107 of 1334
Get a Special Paid Bundle PDF on High Level Puzzles & Seating for All Bank Clerk/ PO Mains
If there are any suggestions/ errors in our PDFs Feel Free to contact us via this email: admin@exampundit.in
Ultra Practice Bundle PDF
Reasoning Questions for SBI Clerk/ RBI Assist. Mains
(c) F Samsung nor Acer laptop. N and P are not immediate
(d) C neighbors. P who has not Apple laptop sits on the
(e) None of these immediate right of Q. There is only one person sitting
25. Who sits third to the left of one who likes Sanju? between P and the person who has Samsung laptop.
(a) B 26. What is the position of Q with respect to one who
(b) H have Samsung laptop?
(c) D (a) third to right
(d) A (b) second to left
(e) None of these (c) third to left
Directions (26-30) : Answer the questions based on the (d) fourth to right
information given below: (e) Cannot be determined
There are eight persons J, K, L, M, N, O, P and Q sitting 27.What is the position of O with respect to the
around a rectangular table and all are facing away from person who Asus laptop?
the centre . Four of them are sitting at the corners and rest (a) second to right
in the middle of the sides. All friends have different laptop (b) third to right
i.e. HP, Dell, Asus, Lenovo, Apple, Samsung, Acer and (c) immediate left
Sony but not necessarily in the same order. (d) second to left
There is only one person sitting between those two (e) immediate right
persons who has Samsung and Sony laptop. The person 28. Who sits third to the left of one who have Lenovo
who has Apple laptop sits second to the right of O. Q has laptop?
not Dell or HP laptop but he is an immediate neighbor of (a) L
one who has Apple laptop. There are two persons sitting (b) P
between L and the person who has Samsung laptop. L has (c) M
Asus laptop. O sits middle side of the table. There are two (d) K
persons sits between the persons who have Acer and (e) O
Apple laptop. K and N are immediate neighbors of each 29. Who sits immediate left of one who have Dell
other. The person who has HP laptop sits on the immediate laptop?
left of the person who has Sony laptop. J sits second to the (a) Who have Sony laptop
right the person who has Acer laptop. M has neither (b) Who have Lenovo laptop
Page 108 of 1334
Get a Special Paid Bundle PDF on High Level Puzzles & Seating for All Bank Clerk/ PO Mains
If there are any suggestions/ errors in our PDFs Feel Free to contact us via this email: admin@exampundit.in
Ultra Practice Bundle PDF
Reasoning Questions for SBI Clerk/ RBI Assist. Mains
(c) Who have Accer laptop an expert of CSS. The person who sit to immediate right
(d) Who have HP laptop of E is an expert of Ruby. I is not an expert in Ruby and
(e) Who have Asus laptop JavaScript. The Perl expert is sitting opposite to the person
30. How many person sit between P and O from who is an immediate neighbour of CSS expert. The expert
right of P? of C++ is facing outside. D is not expert in Python. J sits
(a) Six at corner of the table. K sits fifth to right of E, who is an
(b) Four expert in PHP. K sits immediate right of the J, who faces
(c) None outside. J is an expert in Shell and sits opposite to D.
(d) Two 31. Who sits to the immediate right of the one who
(e) Three experts in Ruby?
Directions (31-35) : Answer the questions based on the (a) E
information given below: (b) H
There are ten programming language experts are A, B, C, (c) I
D, E, F, H, I, J & K are sitting around a rectangular table (d) C
but not necessarily in the same order. They are expert in (e) G
JavaScript, Java, Python, CSS, PHP, Ruby, C++, Perl, 32. Who among the following experts in Javascript?
Shell and VimL but not necessarily in the same order. (a) B
Some of them are facing inside while some of them are (b) D
facing outside. Four sits at corners, two sits on each of the (c) C
longer side and one on each smaller side of the table. (d) G
Immediate neighbours are not facing in the same (e) A
direction. 33. Who sits third to the left of the one who experts in
The experts of C++ is sitting opposite to B. The experts of Python ?
C++ and Perl are an immediate neighbour. A sits fourth to (a) C
the left of E. The person who is an expert of VimL sits (b) I
immediate left of A, who is an immediate neighbour of a (c) B
person who is an expert of Java. B is an expert in Java and (d) F
facing inside. There is no one sit between the experts of (e) K
VimL and JavaScript. H sits between F and I and he is not 34. D experts in which language?
Page 109 of 1334
Get a Special Paid Bundle PDF on High Level Puzzles & Seating for All Bank Clerk/ PO Mains
If there are any suggestions/ errors in our PDFs Feel Free to contact us via this email: admin@exampundit.in
Ultra Practice Bundle PDF
Reasoning Questions for SBI Clerk/ RBI Assist. Mains
(a) Ruby of A. immediate neighbors of A face opposite direction
(b) CSS with respect to A. D faces towards centre. The one who
(c) Perl likes Carrot sits second to the left of the one who likes
(d) Python Banana. E sits second to the left of B who is an immediate
(e) Java neighbour of the one who likes Red velvet who is not
35. How many persons sit between who experts in Java immediate neighbor of E.
and Shell from right of Shell? 36. Who sits to the immediate left of one who likes
(a) One Banana?
(b) Four (a) G
(c) Six (b) A
(d) Two (c) D
(e) Three (d) E
Directions (36-40) : Answer the questions based on the (e) B
information given below: 37. Four of the following five are alike in a certain way
There are eight persons A, B, C, D, E, F, G and H are based on the given arrangement and hence form a
sitting around a rectangular table. Four persons sits at group. Which is the one that doesn’t belong to that
corner and four persons sits at middle of side of the table. group?
Two of them are facing inside while Six of facing outside. (a) H
Each of them like different cake’s flavours i.e. Vanila, (b) B
Chocolate, Apple, Carrot, Banana, Strawberry, Cherry (c) D
and Red velvet but not necessarily in the same order. (d) E
B sits at one of the corners of the table. H likes Cherry and (e) C
sits third to the right of the one who sits immediate to the 38. Who sits second to the right of A ?
right of E. F likes Strawberry and sits opposite to G and (a) Who likes Cherry
faces opposite direction as the one who likes Red velvet. (b) Who likes Strawberry
H faces outside to the centre. G sits second to the right of (c) Who likes Apple
the one who likes Red velvet. C likes Vanila and is an (d) Who likes Banana
immediate neighbour of both F and A, who likes (e) Who likes Chocolate
Chocolate. The one who likes Apple sits second to the left 39. Who is second to the left of one who likes Carrot?
Page 110 of 1334
Get a Special Paid Bundle PDF on High Level Puzzles & Seating for All Bank Clerk/ PO Mains
If there are any suggestions/ errors in our PDFs Feel Free to contact us via this email: admin@exampundit.in
Ultra Practice Bundle PDF
Reasoning Questions for SBI Clerk/ RBI Assist. Mains
(a) G person who has born in 2012 cannot sit with the person
(b) B who has born in 2015. There are two persons sitting
(c) H between L and the person who has born in 2017. M has
(d) C neither born in 2017 nor 2018. N and P are not immediate
(e) A neighbors. P who has not born in 2016 sits on the
40. How many persons sit between C and one who likes immediate right of Q. There is only one person sitting
Strawberry from left of C? between those two persons who has born in 2017 and
(a) One 2019. There is only one person sitting between P and the
(b) Four person who has born in 2017.
(c) Six 41. Who is sitting third to the right of Q?
(d) Two (a) M
(e) None (b) L
Directions (41-45) : Answer the questions based on the (c) P
information given below: (d) N
There are eight persons J, K, L, M, N, O, P and Q sitting (e) None of these
around a rectangular table and all are facing away from 42. What is the position of M with one who born in
the centre. All friends have born in different years i.e. 2014?
2012, 2013, 2014, 2015, 2016, 2017, 2018 and 2019 but (a) Immediate right
not necessarily in the same order. Two persons are sitting (b) Immediate left
on each side of the table. (c) Third to the right
Q and P sits on the same side of the table. K and N are (d) Third to the left
immediate neighbors of each other. The person who has (e) None of these
born in 2012 sits on the immediate left of the person who 43. What is N’s position with respect to the person who
has born in 2019. J sits second to the right the person who born in 2018 ?
has born in 2018.There are two persons sits between the (f) Third to the right
persons who have born in 2018 and 2016. The person who (g) Second to the right
has born in 2016 sits second to the right of O. Q has not (h) Immediate right
born in 2013 or 2012 and he is also an immediate neighbor (i) Second to the left
of one who has born in 2016. L has born in 2014. The (j) Third to the left
Page 111 of 1334
Get a Special Paid Bundle PDF on High Level Puzzles & Seating for All Bank Clerk/ PO Mains
If there are any suggestions/ errors in our PDFs Feel Free to contact us via this email: admin@exampundit.in
Ultra Practice Bundle PDF
Reasoning Questions for SBI Clerk/ RBI Assist. Mains
44. Who is sitting third to the right of one who born in of N. N is not an immediate neighbor Q. J sits third to the
2015? right of who belongs CPI. Only two people sit between the
(a) N person who belongs CPI and Q. The person who belongs
(b) P BJP is an immediate neighbor of both the persons who
(c) O belongs JDU and MNS. The person who belongs MNS
(d) M sits third to the right of K. K does not belongs RJD. J
(e) L doesnot like SP. P is not an immediate neighbor of the
45. Which one of the following statements is true person who belongs BJP. O is not an immediate neighbor
according to the above mentioned arrangement? of J. The person who belongs RJD and the INC are
(f) P is to the immediate left of one who born in 2015. immediate neighbors of each other.
(g) M sits second to the right of one who born in 2019. 46. Who is to the immediate left of one who belongs
(h) N sits second to right of one who born in 2014. from MNS?
(i) Q sits fourth to the right of one who born in 2013. (a) Q
(j) None of these (b) J
Directions (46-50) : Answer the questions based on the (c) K
information given below: (d) P
There are eight persons J, K, L, M, N, O, P and Q sitting (e) None of these
around a rectangular table and all are facing away from 47. Who among the following belongs from BSP party?
the centre but not necessarily in the same order. Each one (a) L
of them belongs different political parties i.e. BJP, INC, (b) P
SP, BSP, RJD, JDU, MNS and CPI but not necessarily in (c) N
the same order. Two persons are sitting on each side of the (d) J
table. (e) None of these
The person who belongs BJP is an immediate neighbor of 48. Who is sitting third to the left of one who belongs
both the persons who belongs JDU and MNS. L sits to the from JDU?
immediate right of the person who belongs SP. Neither J (a) J
nor Q belongs RJD and INC. The one who belongs RJD (b) M
is not an immediate neighbor of the person who belongs (c) P
CPI. The person who belongs BJP sits second to the left (d) K
Page 112 of 1334
Get a Special Paid Bundle PDF on High Level Puzzles & Seating for All Bank Clerk/ PO Mains
If there are any suggestions/ errors in our PDFs Feel Free to contact us via this email: admin@exampundit.in
Ultra Practice Bundle PDF
Reasoning Questions for SBI Clerk/ RBI Assist. Mains
(e) None of these 50. Who is sitting fourth to the right of one who
49. Who is immediate neighbors of the person who belongs from INC?
belongs from CPI? (a) M
(a) K and N (b) O
(b) J and L (c) L
(c) M and Q (d) J
(d) Q and P (e) Q
(e) None of these

Seating Rectangle 2 Variable – Answer and Explanation


SOLUTION(1-5):
Explanation in detail:
1. Civil is sitting on the corner side of the table.
2. The person who teaches Civil is sitting fifth to the
right of Q.

Page 113 of 1334


Get a Special Paid Bundle PDF on High Level Puzzles & Seating for All Bank Clerk/ PO Mains
If there are any suggestions/ errors in our PDFs Feel Free to contact us via this email: admin@exampundit.in
Ultra Practice Bundle PDF
Reasoning Questions for SBI Clerk/ RBI Assist. Mains
7. Only one person is sitting between U and W.

3. T is sitting third to the left of S.


4. S is sitting third to the left of the person who teaches
Civil.

8. The person who teaches Power is sitting fifth to the


5. The person who teaches Computer is neither an left of W.
immediate neighbour of S nor of T.

6. R is sitting fourth to the left of V who does not teach


Civil.

Page 114 of 1334


Get a Special Paid Bundle PDF on High Level Puzzles & Seating for All Bank Clerk/ PO Mains
If there are any suggestions/ errors in our PDFs Feel Free to contact us via this email: admin@exampundit.in
Ultra Practice Bundle PDF
Reasoning Questions for SBI Clerk/ RBI Assist. Mains

9. The person who teaches Electrical is sitting sixth to


the right of the person who teaches Power.

1. a
2. e
3. e
4. b
5. d
SOLUTION(6-10):
Explanation in detail:
1. B sits second to the right of D, who purchases N99.
There is no such space for Electrical in CASE2 So
2. F is an immediate neighbor of B who purchases
CASE2 is cancelled out.
K99.
10. The person who teaches Mechanical is an immediate
3. F does not sit exactly between D and B.
neighbour of the person who teaches Geological.
11. S does not teach Geological.
12. W does not teach Optical.

Page 115 of 1334


Get a Special Paid Bundle PDF on High Level Puzzles & Seating for All Bank Clerk/ PO Mains
If there are any suggestions/ errors in our PDFs Feel Free to contact us via this email: admin@exampundit.in
Ultra Practice Bundle PDF
Reasoning Questions for SBI Clerk/ RBI Assist. Mains
6. The one who purchases P99 is an immediate
neighbor of those who purchases from MG10.
7. F purchase K95 but not from MG10.

8. C purchases R99 from Medlife.


9. H who purchases R95 sits on the immediate right
4. F faces who buys N99 mask. of C.

10. Two friends who purchases from Apollo sits


fourth to the right of each other.
11. D does not shop from Apollo.

So CASE2 is cancelled out.

5. The one who shop from MG10 sits second to the


right of F.

Page 116 of 1334


Get a Special Paid Bundle PDF on High Level Puzzles & Seating for All Bank Clerk/ PO Mains
If there are any suggestions/ errors in our PDFs Feel Free to contact us via this email: admin@exampundit.in
Ultra Practice Bundle PDF
Reasoning Questions for SBI Clerk/ RBI Assist. Mains
12. E shop from MG10. Explanation in detail:
13. There are two friends sitting between those who 1. E is sitting third to the left of G.
are shopping from Netmeds. 2. The person who is from Fortis is to the immediate
14. G shop from Medlife and he does not purchase right of E.
P95.

3. B is sitting fourth to the right of H.


4. H is not an immediate neighbor of G.

6. d
7. b
8. a
9. c
10. c
SOLUTION(11-15):

Page 117 of 1334


Get a Special Paid Bundle PDF on High Level Puzzles & Seating for All Bank Clerk/ PO Mains
If there are any suggestions/ errors in our PDFs Feel Free to contact us via this email: admin@exampundit.in
Ultra Practice Bundle PDF
Reasoning Questions for SBI Clerk/ RBI Assist. Mains
12. E is not from Apollo.

5. Neither B nor H is an immediate neighbor of E.


So CASE2 is cancelled out.
6. F is from RML and is sitting third to the right of
the person who from Fortis.
7. The person from SGRH is sitting second to the left
of the person from RML.

11. b
12. d
13. a
14. c
15. c
8. The person from MAX is sitting second to the left SOLUTION(16-20):
of E. Explanation in detail:
9. A, who is from LNJP is sitting exactly between F
and H. 1. P sits smaller side of the table.
10. The person from AIIMS is sitting second to the 2. S sits third to the left of P.
right of the person from LNJP.
11. C is sitting third to the left of F.

Page 118 of 1334


Get a Special Paid Bundle PDF on High Level Puzzles & Seating for All Bank Clerk/ PO Mains
If there are any suggestions/ errors in our PDFs Feel Free to contact us via this email: admin@exampundit.in
Ultra Practice Bundle PDF
Reasoning Questions for SBI Clerk/ RBI Assist. Mains

6. P sits third to the right of T, who belongs to Maruti.


7. The one who belongs to Hyundai sits second to the
left of S.
8. E belongs to Renault and sits second to the left of
one who belongs to Maruti.

3. C sits immediate right of S, who is facing facing


away from the centre.
4. C is facing inside the centre.
5. The one who belongs to Suzuki sits on the
immediate right to C, who belongs to TATA.

In CASE2 there is no such space for E. so CASE2 is


cancelled out.

Page 119 of 1334


Get a Special Paid Bundle PDF on High Level Puzzles & Seating for All Bank Clerk/ PO Mains
If there are any suggestions/ errors in our PDFs Feel Free to contact us via this email: admin@exampundit.in
Ultra Practice Bundle PDF
Reasoning Questions for SBI Clerk/ RBI Assist. Mains
15. A sits on the immediate right of one who belongs
to Kia.
16. The one who belongs to Ford sits on the immediate
left of who belongs to Hyundai.
17. R belongs neither to Hyundai nor Kia and facing
to the centre.

9. Both B and Q sit diagonally opposite each other


and both faces opposite direction.
10. Q belongs to Ashok and sits third to the left of one
who belongs to Jeep.
11. E sits to the immediate left of Q.

12. Q sits third to the left of A.


13. R sits on the immediate left of A.
14. E and P faces same direction.

16. a
17. b
18. b
19. d
20. c
SOLUTION(21-25):
Explanation in detail:
Page 120 of 1334
Get a Special Paid Bundle PDF on High Level Puzzles & Seating for All Bank Clerk/ PO Mains
If there are any suggestions/ errors in our PDFs Feel Free to contact us via this email: admin@exampundit.in
Ultra Practice Bundle PDF
Reasoning Questions for SBI Clerk/ RBI Assist. Mains
1. A facing away from the centre. 10. Only two persons sit between G and H.
2. A and who likes Dhoni doesnot sit on the same side of
the table.
3. Immediate left of A likes Dhoni, who faces the
opposite to centre.
4. E sits second to the left of A.
5. F and E are facing the same side and only two persons
are sitting between them.
6. F does not sit immediately next to A.

11. The one who likes Dhawan sits fourth to the left of the
one who likes Jadeja.
12. D is facing same direction as F.
7. B sits to the immediate right of E and both E and B are 13. A and C are facing outward direction but not as F.
facing same direction.
8. D sits fourth to right of B.

14. The one who likes Sanju sits second to the right of the
one who likes Jadeja.
If we take direction of E outward then there are no space 15. The one who likes Rohit sits fourth to the right to the
for D so we take E facing inside. one who likes Sanju.
9. The one who likes Jadeja sits between F and B.

Page 121 of 1334


Get a Special Paid Bundle PDF on High Level Puzzles & Seating for All Bank Clerk/ PO Mains
If there are any suggestions/ errors in our PDFs Feel Free to contact us via this email: admin@exampundit.in
Ultra Practice Bundle PDF
Reasoning Questions for SBI Clerk/ RBI Assist. Mains
23. a
24. c
25. c
SOLUTION(26-30):
Explanation in detail:
1. The person who has Apple laptop sits second to the
right of O.
16. F does not like Kohli. 2. O sits middle side of the table.
17. C sits second to the left of G. 3. Q is also an immediate neighbor of one who has Apple
18. G does not like Jadeja. laptop.
19. The one who likes Rahul sits immediately right to the 4. P who has not Apple laptop sits on the immediate
one who likes Dhawan. right of Q.
5. There is only one person sitting between P and the
person who has Samsung laptop.

6. There are two persons sitting between L and the


person who has Samsung laptop.
7. There are two persons sits between the persons who
have Accer and Apple laptop.
8. J sits second to the right the person who has Accer
laptop.
21. b
9. L has Asus laptop.
22. d

Page 122 of 1334


Get a Special Paid Bundle PDF on High Level Puzzles & Seating for All Bank Clerk/ PO Mains
If there are any suggestions/ errors in our PDFs Feel Free to contact us via this email: admin@exampundit.in
Ultra Practice Bundle PDF
Reasoning Questions for SBI Clerk/ RBI Assist. Mains

26. c
10. M has neither Samsung nor Accer laptop.
11. K and N are immediate neighbors of each other. 27. c
28. e
12. N and P are not immediate neighbors.
29. b
30. e
SOLUTION(31-35):
Explanation in detail:
1. K sits immediate right of the J, who faces outside.
2. J sits at corner of the table.
3. Immediate neighbours are not facing in the same
13. There is only one person sitting between those two
direction.
persons who has Samsung and Sony laptop.
4. K sits fifth to right of E, who is an expert in PHP.
14. The person who has HP laptop sits on the immediate
5. J is an expert in Shell and sits opposite to D.
left of the person who has Sony laptop.
15. Q has not Dell or HP laptop.

6. A sits fourth to the left of E.


Final arrangement:
Page 123 of 1334
Get a Special Paid Bundle PDF on High Level Puzzles & Seating for All Bank Clerk/ PO Mains
If there are any suggestions/ errors in our PDFs Feel Free to contact us via this email: admin@exampundit.in
Ultra Practice Bundle PDF
Reasoning Questions for SBI Clerk/ RBI Assist. Mains
7. The person who is an expert of VimL sits
immediate left of A, who is an immediate
neighbour of a person who is an expert of Java.
8. B is an expert in Java and facing inside.
9. There is no one sit between the experts of VimL
and JavaScript.
10. The experts of C++ is sitting opposite to B.
11. The person who sit to immediate right of E is an
15. The Perl expert is sitting opposite to the person
expert of Ruby.
who is an immediate neighbour of CSS expert.
16. The expert of C++ is facing outside.
17. D is not expert in Python.

12. H sits between F and I and he is not an expert of


CSS.
13. I is not an expert in Ruby and JavaScript.
14. The experts of C++ and Perl are an immediate
neighbour.

31. a
32. e

Page 124 of 1334


Get a Special Paid Bundle PDF on High Level Puzzles & Seating for All Bank Clerk/ PO Mains
If there are any suggestions/ errors in our PDFs Feel Free to contact us via this email: admin@exampundit.in
Ultra Practice Bundle PDF
Reasoning Questions for SBI Clerk/ RBI Assist. Mains
33. e
34. b
35. d
SOLUTION(36-40):
Explanation in detail:
1. G sits second to the right of the one who likes Red
velvet.
2. F likes Strawberry and sits opposite to G and faces
opposite direction as the one who likes Red velvet.
3. C likes Vanilla and is an immediate neighbour of
both F and A, who likes Chocolate.

4. The one who likes Apple sits second to the left of


A.
5. Immediate neighbors of A face opposite direction
with respect to A.

Page 125 of 1334


Get a Special Paid Bundle PDF on High Level Puzzles & Seating for All Bank Clerk/ PO Mains
If there are any suggestions/ errors in our PDFs Feel Free to contact us via this email: admin@exampundit.in
Ultra Practice Bundle PDF
Reasoning Questions for SBI Clerk/ RBI Assist. Mains

10. D faces towards centre.


There is no space for D So CASE2 is canceled out.
11. The one who likes Carrot sits second to the left of
the one who likes Banana.

6. E sits second to the left of B and only one of them


is an immediate neighbour of the one who likes
Red velvet.
7. B sits at one of the corners of the table.
So CASE3 and CASE4 are cancelled out.
8. H likes Cherry and sits third to the right of the one
who sits immediate to the right of E.
9. H faces outside to the centre.
CASE1:

Page 126 of 1334


Get a Special Paid Bundle PDF on High Level Puzzles & Seating for All Bank Clerk/ PO Mains
If there are any suggestions/ errors in our PDFs Feel Free to contact us via this email: admin@exampundit.in
Ultra Practice Bundle PDF
Reasoning Questions for SBI Clerk/ RBI Assist. Mains

6. There are two persons sits between the persons


36. a
who have born in 2018 and 2016.
37. c
7. J sits second to the right the person who has born
38. b
in 2018.
39. d
8. There are two persons sitting between L and the
40. e
person who has born in 2017.
SOLUTION(41-45):
Explanation in detail:
1. The person who has born in 2016 sits second to the
right of O.
2. Q is an immediate neighbor of one who has born
in 2016.
3. P who has not born in 2016 sits on the immediate
right of Q. 9. K and N are immediate neighbors of each other.
4. There is only one person sitting between P and the 10. N and P are not immediate neighbors.
person who has born in 2017. 11. L has born in 2014.
5. Q and P sits one the same side of the table.

Page 127 of 1334


Get a Special Paid Bundle PDF on High Level Puzzles & Seating for All Bank Clerk/ PO Mains
If there are any suggestions/ errors in our PDFs Feel Free to contact us via this email: admin@exampundit.in
Ultra Practice Bundle PDF
Reasoning Questions for SBI Clerk/ RBI Assist. Mains
12. There is only one person sitting between those two Explanation in detail:
persons who has born in 2017. 1. J sits third to the right of who belongs CPI.
13. Q has not born in 2013 or 2012. 2. Only two people sit between the person who belongs
14. The person who has born in 2012 cannot sit with CPI and Q.
the person who has born in 2015. 3. The person who belongs RJD and the INC are
15. The person who has born in 2012 sits on the immediate neighbors of each other.
immediate left of the person who has born in 2019. 4. Neither J nor Q belongs RJD and INC.
16. M has neither born in 2017 nor 2018. 5. The one who belongs RJD is not an immediate
neighbor of the person who belongs CPI.

41. d 6. The person who belongs BJP sits second to the left of
42. a N.
43. c 7. N is not an immediate neighbor Q.
44. a 8. The person who belongs BJP is an immediate
45. b neighbor of both the persons who belongs JDU and
SOLUTION(46-50): MNS.

Page 128 of 1334


Get a Special Paid Bundle PDF on High Level Puzzles & Seating for All Bank Clerk/ PO Mains
If there are any suggestions/ errors in our PDFs Feel Free to contact us via this email: admin@exampundit.in
Ultra Practice Bundle PDF
Reasoning Questions for SBI Clerk/ RBI Assist. Mains
There is no space for BJP in CASE2 , So CASE2 is
cancelled out.

11. L sits to the immediate right of the person who belongs


SP.
12. J doesnot like SP.
There is no such space for L so CASE1a is cancelled
out.

9. The person who belongs MNS sits third to the right


of K. 13. O is not an immediate neighbor of J.
10. K does not belongs RJD. 14. P is not an immediate neighbor of the person who
belongs BJP.

Page 129 of 1334


Get a Special Paid Bundle PDF on High Level Puzzles & Seating for All Bank Clerk/ PO Mains
If there are any suggestions/ errors in our PDFs Feel Free to contact us via this email: admin@exampundit.in
Ultra Practice Bundle PDF
Reasoning Questions for SBI Clerk/ RBI Assist. Mains

46. b
47. d
48. a
49. a
50. e

Download Seating Arrangement Practice Questions PDF


Get More Reasoning Practice Questions PDF

Seating Triangle 2 Variable


Direction (1 – 5): Study the following information company viz Nokia, Vivo, Apple, Samsung, Oppo and
carefully and answer the questions given below: Redmi but not necessary in the same order.
Six persons sit saround a triangular table such that three Arnav sits immediate right to Jayant. The one who like
of them sit at the corners and the rest sits on the middle of Nokia mobile sit at the corner. Lalit sits second to the right
the side. The one who sit at the corner face away from the of Shyam who likes Apple. Jayant faces center and two
center and the one who sit on the middle of the side persons sit between Jayant and one who likes Samsung
towards the center. They also like mobile of different mobile. The one who likes Redmi sits immediate right to
the one who likes Oppo mobile. Arnav does not like Oppo.

Page 130 of 1334


Get a Special Paid Bundle PDF on High Level Puzzles & Seating for All Bank Clerk/ PO Mains
If there are any suggestions/ errors in our PDFs Feel Free to contact us via this email: admin@exampundit.in
Ultra Practice Bundle PDF
Reasoning Questions for SBI Clerk/ RBI Assist. Mains
Prabath sits at one of the corner but does not like Samsung 5) Who likes Nokia mobile?
mobile and Redmi mobile. The one who like oppo and the a. Shyam
one who like nokia are immediate neighbours. Rahul is b. Rahul
one of the persons among them c. Prabath
1) Who seat to the first right of Arnav? d. Jayant
a. Shyam e. None of these
b. Jayant Direction (6 – 10): Study the following information
c. Prabath carefully and answer the questions given below:
d. Rahul Six person Manoj, Arnav, Praveen, Farhan, Gopal and
e. None of these Retesh seating around a triangular table after particapaed
2) Who likes Vivo Mobole? in a running competition. Three person sit at the corner
a. Lalit and rest sit on the middle of the side. All are facing
b. Rahul towards the center.They all got different rank from 1 to 6
c. Arnav but not necessary in the same order.
d. Shyam Only one person sits between Farhan and Praveen. The
e. None of these sum of the rank of Farhan and Praveen is 7 but none of
3) How many persons seat between Prabath and them got rank 1 or 2. Gopal get the last rank and sit at one
Shyam? of the corner. Ritesh got rank which is odd but not 1.
a. One Ritesh doesnot sit to the immediate right of Praveen.
b. Two Farhan sit at one of the corner and he got the rank which
c. Three is even (i.e. 2, 4 or 6). Manoj sit such that he is facing
d. Four Praveen. The one who got rank 1 does not sit adjacent to
e. None of these Gopal.
4) Rahul like mobile of which company? 6) Who has rank one of all?
a. Vivo a. Gopal
b. Apple b. Manoj
c. Oppo c. Arnav
d. Samsung d. Farhan
e. None of these e. None of these
Page 131 of 1334
Get a Special Paid Bundle PDF on High Level Puzzles & Seating for All Bank Clerk/ PO Mains
If there are any suggestions/ errors in our PDFs Feel Free to contact us via this email: admin@exampundit.in
Ultra Practice Bundle PDF
Reasoning Questions for SBI Clerk/ RBI Assist. Mains
7) Who seat second to the right of the person who got Six persons Amit, Suman, Pranav, Kundan, Rahul and
rank 5? Sumit are sitting on a triangular table facing away from
a. Arnav the center. Three person sit at the corner and rest sit on the
b. Praveen middle of the side. Each of them likes different games i.e.
c. Farhan Cricket, Boxing, Badminton, Tennis, Rugby and
d. Gopal volleyball but not necessary in the same order.
e. None of these There is one person gap between Amit and the one who
8) How many person seat between Manoj and the like Badminton. Suman is the neighbor of the one who like
person who got rank 6, when counted fron the right of Badminton which was not like by sumit. Pranav likes
Manoj? Rugby. Suman sit second to the right of the one who likes
a. One Rugby. Two persons seat between Rahul and the one who
b. Two like Volleyball. Amit like tennis but does not sit at any of
c. Three the corner. Pranav and the one who likes Boxing Seat at
d. Four the corner but none of them is neighbor of Suman who
e. None of these does not like volleyball.
9) Which of the following is true? 11) Who like Cricket?
a. Manoj seat at one of the corner. a. Rahul
b. Praveen got rank 5. b. Sumit
c. Arnav and Manoj are immediate neighbours c. Suman
d. Farhan seat at one of the corner d.Pranav
e. None of these e. None of these
10) Who is facing Ritesh? 12) Rahul like which of the following game?
a. Praveen a. Cricket
b. Farhan b. Boxing
c. Gopal c. Tennis
d. Arnav d. Volleyball
e. None of these e. None of these
Direction (11 – 15): Study the following information 13) Which of the following is true?
carefully and answer the questions given below: a. Sumit like Badminton
Page 132 of 1334
Get a Special Paid Bundle PDF on High Level Puzzles & Seating for All Bank Clerk/ PO Mains
If there are any suggestions/ errors in our PDFs Feel Free to contact us via this email: admin@exampundit.in
Ultra Practice Bundle PDF
Reasoning Questions for SBI Clerk/ RBI Assist. Mains
b. Kundan like Volleyball them belongs from Chennai. E is not the neighbor of D
c. Amit seat at one of the corner and G. D belongs from Patna and seat at one of the corner.
d. Suman and Rahul are neighbours One of the person who belongs to Kolkata seat between G
e. None of these and H. I is not the neighbor of H but he belongs from
14) Which of the following is different from the group? Kolkata. C, who Belong to Kolkata, seats at one of the
a. Suman-Tennis position which is immediate neighbor to the person who
b. Sumit-Boxing is from Chennai. G Belongs from Chennai. B is not
c. Pranav-Rugby Belongs from patna.
d. Rahul-Volleyball 16) Which of the following is true?
e. Amit-Cricket a. I is from Patna
15) Pranav likes which of the following game? b. B and H are from same city
a. Cricket c. D and E are from same city
b. Tennis d. F is from Chennai
c. Volleyball e. None of these
d. Rugby 17) Who seat third to the left of the person who is
e. None of these second to the right of E?
Direction (16 – 20): Study the following information a. F
carefully and answer the questions given below: b. A
Nine persons A, B, C, D, E, F, G, H and I are seating on a c. H
triangular table. Two persons sit on each side of the table d. B
and one person seat on each of the corner. All of them are e. None of these
facing towards the center. They are from different cities 18) Which of the following are the neighbours of C?
i.e. Chennai, Patna and Kolkata but not necessary in the a. H and A
same order. Not more than three persons are from same b. E and D
city. The person belonging from same city does not seat c. G and I
adjacent to each other. d. G and H
Three person seat between D and H. H seat second to the e. None of these
right of G. A seat at one of the corner but does not belongs 19) Which of the following seat at the corner and is
from Kolkata. E and F seat on the same side and none of from Chennai?
Page 133 of 1334
Get a Special Paid Bundle PDF on High Level Puzzles & Seating for All Bank Clerk/ PO Mains
If there are any suggestions/ errors in our PDFs Feel Free to contact us via this email: admin@exampundit.in
Ultra Practice Bundle PDF
Reasoning Questions for SBI Clerk/ RBI Assist. Mains
a. A is perfect cube. Gaurav lives on highest numbred floor but
b. D not adjacent to Aryan. Alok and the person who lives on
c. B first floor are facing each other. Sumit doesnot lives on
d. G floor number 1. The person who lives on the floor number
e. Both option a & d are correct 4 sits second to the right of Sumit. The person who lives
20) How many person seat between F and B, when on floor number 7 is sitting on the outer table but he is not
counted fron the right of F? surender. The person who lives on floor number 5 does
a. One not seat on the corner. Dipak does not live on floor number
b. Three 9. Sachin seat at the corner and lives on floor number 11.
c. Four Kundan seats in front of the person who is immediate right
d. Six of Subham. Niraj is facing Kundan and lives on floor
e. None of these number 10. The person who lives on floor number 12 is
Direction (21 – 25): Study the following information facing person who lives on floor number 3.
carefully and answer the questions given below: 21) Who seat in front of Alok?
Twelve persons are sitting in two triangular table such that a. Sumit
one table is inside the other table. Six person seat on each b. Sanjay
table in such a way that one person seat at each corner and c. Kundan
one person seat on the middle of each side. The persons d. Rohit
sittingt on the outer triangular table i.e. Sachin, Rohit, e. None of these
Sumit, Sanjay, Surender and Kundan are facing towards 22) Sanjay lives o which of the following floor?
the center and the person sitting on the inner table i.e. a. 4
Gaurav, Niraj, Subham, Dipak, Alok and Aryan facing b. 3
away from the center. Each of them lives on different floor c. 6
i.e. 1 to 12 floor of a building. d. 10
Two person sit between Sachin and Sanjay who is facing e. None of these
the Person who lives on floor number 6. Subham lives on 23) Who seat second to the left of dipak?
floor number 6. Surender seats second to the left of a. Subham
Sachin. Dipak sits second to the right of Niraj. Aryan is b. Niraj
not adjacent to Dipak but lives on the flor number which c. Kundan
Page 134 of 1334
Get a Special Paid Bundle PDF on High Level Puzzles & Seating for All Bank Clerk/ PO Mains
If there are any suggestions/ errors in our PDFs Feel Free to contact us via this email: admin@exampundit.in
Ultra Practice Bundle PDF
Reasoning Questions for SBI Clerk/ RBI Assist. Mains
d. Aryan works in IBM faces away from the center. C seats
e. None of these immediate right of the person who works in Wipro. B
24) Who lives on floor number Nine? works in Google and face towards the center. Only three
a. Dipak persons face away from the center. Three person sit
b. Sanjay between E and F. F does not work in Google. The one who
c. Gaurav likes wipro faces outside.
d. Surender 26) Who wotks in TCS?
e. None of these a. T
25) The sum of the floor number of Subham and b. E
Kundan is? c. F
a. 13 d. B
b. 16 e. None of these
c. 11 27) Which of the following is true?
d. 9 a. C seats second to the left of B
e. None of these b. D seat at one of the corner
Direction (26 – 30): Study the following information c. A and C are neighbours
carefully and answer the questions given below: d. B works in Wipro
Six person A, B, C, D, E and F sitting on a triangular e. None of these
thable in such a way that one person seat on the corner and 28) Which of the following pair is different from all
one seat on the middle of each side. Some of them are others?
facing towards the center and some of them are facing a. D - L&T
away from the center. They all work in different company b. C - HCL
i.e. HCL, Google, IBM, TCS, L&T and Wipro but nit c. B - Google
necessary in same order. d. A - Wipro
E seats at one of the corner and works in IBM. The person e. E - TCS
who Works in Wipro sits second to the right of F. C is not 29) Who seat immediate right of the person who works
the neighbor of F. The one who work in Google and TCS in TCS?
are adjacent to each other. The person who works in L&T a. B
and A sits immediate left of each other. The one who b. D
Page 135 of 1334
Get a Special Paid Bundle PDF on High Level Puzzles & Seating for All Bank Clerk/ PO Mains
If there are any suggestions/ errors in our PDFs Feel Free to contact us via this email: admin@exampundit.in
Ultra Practice Bundle PDF
Reasoning Questions for SBI Clerk/ RBI Assist. Mains
c. E colour. Rohan seats third to the right of Sunil and
d. C immediate right of Shyam.
e. None of these 31) How many person seats between Shyam and Sunil
30) E works in which of the following company? when counted from the right of Shyam?
a. Wipro a. None
b. L&T b. Three
c. HCL c. Two
d. IBM d. Four
e. None of these e. None of these
Direction (31 – 35): Study the following information 32) Which of the following person likes Blue colour?
carefully and answer the questions given below: a. Rohan
Six person Arnav, Shyam, Sunil, Rohann, Budlu and Niraj b. Sunil
are seating on a triangular table such that one person seat c. Niraj
at each corner and one person seat on the middle of each d. Shyam
side. Each person likes different colours i.e. Red, Blue, e. None of these
Green, Yellow, White and Orange but not necessary in the 33) Which of the following is true?
same order. The person who seat on the corner face away a. Niraj like Orange colour
from the center and the person who seat on the middle of b. Niraj like Green colour
the side face towards the center. c. Sunil like Orange colour
Three persons seat between the person who like Red and d. Budlu like Red colour
Sunil. Shyam does not seat immediate neighbor to either e. None of these
Arnav or Sunil. The person who like yellow and Green 34) Who seat second to the right of Shyam?
face towards the center. Rohan does not like Green. Niraj a. Sunil
likes Yellow. The person who likes Orange and Yellow b. Budlu
sits immediate right to each other. Shyam who is c. Rohan
immediate neighbor of Niraj does not like Orange. Arnav d. Niraj
face away from the center. The person who likes White is e. None of these
not the neighbor of Arnav or Budlu. Arnav like Red 35) Which of the following is false?
a. Niraj likes Yellow colour
Page 136 of 1334
Get a Special Paid Bundle PDF on High Level Puzzles & Seating for All Bank Clerk/ PO Mains
If there are any suggestions/ errors in our PDFs Feel Free to contact us via this email: admin@exampundit.in
Ultra Practice Bundle PDF
Reasoning Questions for SBI Clerk/ RBI Assist. Mains
b. Arnav seat at one of the corner 37) Which of the following seat on the same side?
c. Rohan and Budlu are immediate neighbours a. D, A and B
d. Sunil is facing away from the center b. G, B and I
e. None of these c. A, D and F
Direction (36 – 40): Study the following information d. C, E and B
carefully and answer the questions given below: 38) Which of the following is true?
Nine person namely A, B, C, D, E, F, G, H and I are sitting a. B and I are immediate neighbours
around equallateral table in such a way that three person b. D seat to the immediate right of F
sit on each side of the table, facing towards the center. c. No one seat between E and I
They all have different age as 32, 37, 46, 52, 53, 44, 54, d. C and A seat on the same line
60 and 66 but not necessary in the same order. e. None of these
Three person sits between G and C who is 44 years old. 39) How many person seats between F and B when
The person who is 54 years old sits forth to the left of I. D counted from the left of F?
and the person whose age is 54 are immediate neighbor. a. Two
Only one person sit between A and E who is one year b. None
younger than F. The person who is 46 years old seat in the c. One
middle of the line but he is neither D nor E. F seat in the d. Three
middle of the line and he is not 46 years old. G and H sit e. None of these
on the same side but none of them seat on the middle of 40) Which of the following is the age of E?
the side. C is not adjacent to D. I is oldest person of all. a. 54
The sum of the age of G and H is not more than 70 years. b. 53
G is elder than H. A is not 60 years old. I seats second to c. 44
the right of C. A is second to the right of D. d. 37
36) Who is the youngest person of the group? e. None of these
a. D Direction (41 – 45): Study the following information
b.H carefully and answer the questions given below:
c. C Six persons Akash, Aryan, Arpan, Sandeep, Sumit and
d. G Sudhir are sitting around a triangular table facing away
e. None of these from the center in such a way that one person seat at each
Page 137 of 1334
Get a Special Paid Bundle PDF on High Level Puzzles & Seating for All Bank Clerk/ PO Mains
If there are any suggestions/ errors in our PDFs Feel Free to contact us via this email: admin@exampundit.in
Ultra Practice Bundle PDF
Reasoning Questions for SBI Clerk/ RBI Assist. Mains
corner and one person seat on the middle of each side. e. None of these
They all are from different city like Bangalore, Nagpur, 43) Akash is from which of the following city?
Goa, Chennai, Manglore and Jaipur but not necessary in a. Goa
the same order. b. Chennai
Three persons seat between Arpan and the person who is c. Jaipur
from Jaipur. The person who is from Chennai seat on the d. Manglore
middle of the side but he is not neighbor to the person who e. None of these
sets second to right of the person from Jaipur. Sudhir who 44) 39) How many person seats between the person
is from Goa does not seat to the immediate left of Sandeep. who is from Nagpur and Arpan when counted from the
The person who is from Nagpur and Manglore are left of Arpan?
adjacent to each other. Sandeep is not from Jaipur. Arpan a. None
is neither from Nagpur nor from Manglore. The person b. Three
who is from Banglore sit second to the left of Sumit. The c. Four
person who is from Jaipur and nagpur seat at one of the d. Two
corner. Sandeep is second from Arpan (either from left or e. None of these
right). One person seat between the person the person who 45) Which of the following pair is true?
is from Chennai and Akash. a. Aryan-Jaipur
41) Which of the following is from Goa? b. Arpan-Banglore
a. Arpan c. Sumit-Goa
b. Sudhir d. Sandeep-Manglore
c. Ayush e. None of these
d. Sandeep Direction (45 – 50): Study the following information
e. None of these carefully and answer the questions given below:
42) Who seats third to the right of the person who is Nine person A, B, C, D, P, Q, R, S and T are sitting around
from Jaipur? a triangular table such that two person seat on each side of
a. Sudhir the table facing towards the center and one person seat on
b. Sandeep each corner of the table facing away from the center. Each
c. Ayush person gave different amount of money i.e. 32, 54, 29, 20,
d. Sumit
Page 138 of 1334
Get a Special Paid Bundle PDF on High Level Puzzles & Seating for All Bank Clerk/ PO Mains
If there are any suggestions/ errors in our PDFs Feel Free to contact us via this email: admin@exampundit.in
Ultra Practice Bundle PDF
Reasoning Questions for SBI Clerk/ RBI Assist. Mains
40, 35, 50, 36 and 42 in their pocker but not necessary in a. P
the same order. b. R
Four person seats between D and R. B is not the neighbor c. D
of D. P seat at one of the corner but he is not adjacent to d. C
R. Two person seat between the person who has 35 rupees e. None of these
and P. The person who has 42 and 29 rupees seat on the 48) What is the sum of amount that S and T have?
same side. The amount of rupees contained by R is a. 69
devisible by 2. The sum of the amount of P and C is 72 b. 86
rupees but they are not adjacent to each other. B has less c. 79
anount of mohey than S. D seats at one of the corner. R d. 58
and T are adjacent to each other and both are facing e. None of these
towards the center. B and Q who has 54 rupees sit on the 49) Which of the following is true?
same side. R is third to the left of the person who has 40 a. R has 40 rupes
rupees. The person who has 35 rupees is not adjacent to b. The difference between the amount of A and P is 4
R. S and the person who has 40 rupees are adjacent to each c. T and Q are neighbours
other. A has 36 rupees. d. C seats to the left of S
46) Which of the following have maximum amount of e. None of these
rupees in the group? 50) Who seat to the immediate left of D
a. A a. C
b. T b. P
c. Q c. R
d. S d. Q
e. None of these e. None of these
47) Which of the following has 20 rupees?

Page 139 of 1334


Get a Special Paid Bundle PDF on High Level Puzzles & Seating for All Bank Clerk/ PO Mains
If there are any suggestions/ errors in our PDFs Feel Free to contact us via this email: admin@exampundit.in
Ultra Practice Bundle PDF
Reasoning Questions for SBI Clerk/ RBI Assist. Mains

Seating Triangle 2 Variable – Answer and Explanation


1. B 5) The one who likes Redmi mobile sits immediate right
2. A to the one who likes Oppo mobile. Thus Case II is not
3. B possible
4. D 6) The one who likes Redmi sits immediate right to the
5. C one who likes Oppo mobile. Arnav does not like Oppo.
Solution
1) Jayant faces center and two persons sit between Jayant
and one who likes Samsung mobile.
2) Arnav sits immediate right to Jayant. Prabath sits at one
of the corner but does not like Samsung mobile and Redmi
mobile.
3) The one who like Nokia mobile sit at the corner.
4) Lalit sits second to the right of Shyam who likes Apple.

Page 140 of 1334


Get a Special Paid Bundle PDF on High Level Puzzles & Seating for All Bank Clerk/ PO Mains
If there are any suggestions/ errors in our PDFs Feel Free to contact us via this email: admin@exampundit.in
Ultra Practice Bundle PDF
Reasoning Questions for SBI Clerk/ RBI Assist. Mains
6. C 14. C
7. A 15. D
8. E Solution
9. D 1) Amit like tennis but does not sit at any of the corner.
10. B 2) There is one person gap between Amit and the one who
1) Farhan sit at one of the corner and he got the rank which like Badminton.
is even (i.e. 2, 4 or 6). 3) Pranav and the one who likes Boxing Seat at the corner
2) Only one person sits between Farhan and Praveen. but none of them is neighbor of Suman.
3) Manoj sit such that he is facing Praveen. 4) Suman is the neighbor of the on who like Badminton.
4) The sum of the rank of Farhan and Praveen is 7 but none 5) Pranav likes Rugby.
of them got rank 1 or 2. 6) Suman sit second to the right of the one who likes
5) Gopal get the last rank and sit at one of the corner. Rugby.
6) The one who got rank 1 does not sit adjacent to Gopal. 7) Two persons seat between Rahul and the one who like
7) Ritesh got rank which is odd but not 1. Volleyball. Thus Case-II I not possible
8) Ritesh does not sit to the right of Praveen. Thus Case- 8) Sumit does not like Badminton.
II is not possible

11. C
12. B
13. E

Page 141 of 1334


Get a Special Paid Bundle PDF on High Level Puzzles & Seating for All Bank Clerk/ PO Mains
If there are any suggestions/ errors in our PDFs Feel Free to contact us via this email: admin@exampundit.in
Ultra Practice Bundle PDF
Reasoning Questions for SBI Clerk/ RBI Assist. Mains
16. C 10) C seats at one of the position which is immediate
17. B neighbor to the person who is from Chennai.
18. D
19. E
20. A
Solution (16-20)
1) D belongs from Patna and seat at one of the corner.
2) Three person seat between D and H.
3) H seat second to the right of G.
4) G Belongs from Chennai.
5) One of the person who belongs from Kolkata seat
between G and H.
6) A seat at one of the corner but does not belongs from 21. D
Kolkata. 22. A
7) E and F seat on the same side and none of them belongs 23. B
on the same side and none of them belongs from Chennai. 24. D
8) E is not the neighbor of D and G. Thus Case-II is not 25. A
possible Solution (21-25)
1) Sachin seat at the corner and lives on floor number
11.
2) Three person sit between Sachin and Sanjay who
is facing the Person who lives on floor number 6.
3) Kundan seats in front of the person who is
immediate right of Subham.
4) Subham lives on floor number 6.
5) Surender seats second to the left of Sachin.
6) Niraj is facing Kundan and lives on floor number
9) I is not the neighbor of H but he belongs from Kolkata.
10.

Page 142 of 1334


Get a Special Paid Bundle PDF on High Level Puzzles & Seating for All Bank Clerk/ PO Mains
If there are any suggestions/ errors in our PDFs Feel Free to contact us via this email: admin@exampundit.in
Ultra Practice Bundle PDF
Reasoning Questions for SBI Clerk/ RBI Assist. Mains
7) Dipak sits second to the right of Niraj.
8) Aryan is not adjacent to Dipak and lives on the
flor number which is perfect cube.
9) Saurav lives on highest numbred floor but not
adjacent to Aryan.
10) The person who lives on floor number 12 is facing
person who lives on floor number 3. Thus Case-II
is not possible.

26. C
27. C
28. E
29. B
30. D
11) Alok and the person who lives on first floor are
Solution (26-30)
facing each other.
1) The one who works in IBM faces away from the
12) Sumit doesnot lives on floor number 1.
center.
13) The person who lives on the floor number 4 sits
2) E seats at one of the corner and works in IBM.
second to the right of Sumit.
3) Three person sit between E and F.
14) The person who lives on floor number 7 is sitting
4) The person who Works in Wipro sits second to the
on the outer table but he is not surender.
right of F.
15) The person who lives on floor number 5 does not
5) C seats immediate right of the person who works
seat on the corner.
in Wipro.
16) Dipak does not live on floor number 9.
6) C is not the neighbor of F.
7) The one who work in Google and TCS are adjacent
to each other.
8) F does not work in Google.

Page 143 of 1334


Get a Special Paid Bundle PDF on High Level Puzzles & Seating for All Bank Clerk/ PO Mains
If there are any suggestions/ errors in our PDFs Feel Free to contact us via this email: admin@exampundit.in
Ultra Practice Bundle PDF
Reasoning Questions for SBI Clerk/ RBI Assist. Mains
9) The person who works in L&T and A sits 3) Arnav like Red colour.
immediate left of each other. Thus Case-II is not 4) Shyam does not seat immediate neighbor to either
possible. Arnav or Sunil.
5) Rohan seats third to the right of Sunil.
6) The person who like yellow and Green face
towards the center.
7) Rohan does not like Green.
8) Niraj likes Yellow.
9) The person who likes Orange and Yellow sits
immediate right to each other.
10) B works in Google and face towards the center. 10) Shyam does not like Orange. Thus Case-II is not
11) Only three persons face away from the center. possible
11) The person who likes White is not the neighbor of
Arnav or Budlu.Thus Case-III is not possible.

31. B
32. A
33. C
34. E
35. C
Solution (31-35)
1) Arnav face away from the center.
2) Three persons seat between the person who like
Red and Sunil. 36. B

Page 144 of 1334


Get a Special Paid Bundle PDF on High Level Puzzles & Seating for All Bank Clerk/ PO Mains
If there are any suggestions/ errors in our PDFs Feel Free to contact us via this email: admin@exampundit.in
Ultra Practice Bundle PDF
Reasoning Questions for SBI Clerk/ RBI Assist. Mains
37. C
38. C
39. A
40. B
Solution (36-40)
1) G and H sit on the same side but none of them seat
on the middle of the side.
2) Three person sits between G and C who is 44 years 11) I is oldest person of all.
old. 12) The sum of the age of G and H is not more than 70
3) I seats second to the right of C. years.
4) The person who is 54 years old sits forth to the left 13) G is elder than H. A is not 60 years old.
of I.
5) D and the person whose age is 54 are immediate
neighbor.
6) C is not adjacent to D.
7) A is second to the right of D.
8) Only one person sit between A and E who is one
year younger than F.
9) The person who is 46 years old seat in the middle
of the line but he is neither D nor E.
10) F seat in the middle of the line and he is not 46 41. B
years old. Thus Case-II is not possible 42. A
43. D
44. E
45. B
Solution (41-45)
1) The person who is from Jaipur seat at one of the
corner.

Page 145 of 1334


Get a Special Paid Bundle PDF on High Level Puzzles & Seating for All Bank Clerk/ PO Mains
If there are any suggestions/ errors in our PDFs Feel Free to contact us via this email: admin@exampundit.in
Ultra Practice Bundle PDF
Reasoning Questions for SBI Clerk/ RBI Assist. Mains
2) Three person seat between Arpan and the person 49. B
who is from Jaipur. 50. D
3) Sandeep is second from Arpan (either from left or Solution (46-50)
right). 1) D seats at one of the corner.
4) The person who is from Chennai seat on the 2) Four person seats between D and R.
middle of the side but he is not neighbor to the 3) R and T are adjacent to each other and both are
person who sets second to right of the person from facing towards the center.
Jaipur. 4) R is third to the left of the person who has 40
5) One person seat between the person the person rupees.
who is from Chennai and Akash. 5) B and Q who has 54 rupees sit on the same side.
6) Sudhir who is from Goa does not seat to the left of 6) B is not the neighbor of D.
Sandeep. 7) P seat at one of the corner but he is not adjacent to
7) The person who is from Nagpur and Manglore are R.
adjacent to each other. 8) Two person seat between the person who has 35
8) Sandeep is not from Manglore. rupees and P.
9) Arpan is neither from Nagpur nor from Manglore. 9) The person who has 35 rupees is not adjacent to R.
10) The person who is from Banglore sit second to the 10) The person who has 42 and 29 rupees seat on the
left of Sumit. Thus Case-II is not bossible. same side.
11) The amount of rupees contained by R is divisible
by 2.
12) S and the person who has 40 rupees are adjacent to
each other.
13) A has 36 rupees.
14) The sum of the amount of P and C is 72 rupees but
they are not adjacent to each other. Thus Case-II is
not possible.
46. C 15) B has more anount of mohey than S.
47. E
48. C
Page 146 of 1334
Get a Special Paid Bundle PDF on High Level Puzzles & Seating for All Bank Clerk/ PO Mains
If there are any suggestions/ errors in our PDFs Feel Free to contact us via this email: admin@exampundit.in
Ultra Practice Bundle PDF
Reasoning Questions for SBI Clerk/ RBI Assist. Mains

Seating Linear 3 Variable


(Direction 1-5): Study the following information Jayant sits second to the left of Fatima whose birthday is
carefully and answer the given questions: on the month having 31 days but before May. Fatima sits
There are nine persons i.e. Kali, Suber, Pritam, Ankur, at the middle of the row. Riyaz who is elder than Fatima
Riyaz, Iqbal, Fatima, Tilak and Jayant are sitting in a sits second to the left of Jayant and his birthday is on
linear row. Four of them are facing south and five persons February. Kali who is the eldest person is having her
are facing in the north direction. All of them have their birthday on the month of 30 days after April and she faces
birthday on different months i.e. January, February, north. Immediate neighbours of kali are facing opposite
March, April, May, June, July, August and September and direction to each other. Three persons sit between Iqbal
also, they all are of different age i.e. 8, 12, 25, 29, 30, 32, and Kali. Iqbal sits towards the left of Fatima and his age
41, 45 and 53 but not necessarily in the same order. is a prime number. Ankur and Tilak whose birthday is on
Person who sits on the right end of the row faces north and July are immediate right and immediate left of each other
the person who sits on the left end of the row faces south. respectively. Neither Ankur nor Tilak sits at the end of the

Page 147 of 1334


Get a Special Paid Bundle PDF on High Level Puzzles & Seating for All Bank Clerk/ PO Mains
If there are any suggestions/ errors in our PDFs Feel Free to contact us via this email: admin@exampundit.in
Ultra Practice Bundle PDF
Reasoning Questions for SBI Clerk/ RBI Assist. Mains
row. Ankur has his birthday on the month having 31 days a) Iqbal is 29 years old and he faces north
but neither on January nor on August also his age is four b) Fatima faces north and is the youngest person
times the age of Fatima who is an immediate neighbour of c) The eldest person sits third from the right end
the person whose birthday is on August. Pritam sits third d) Both A and B
to the left of Ankur and he is elder than both Ankur an e) Both A and C
Tilak. Suber who doesn't face south is younger than Tilak 4) Who among the following has his birthday on
but elder than Iqbal and his birthday is neither on June nor March and what is his age?
on September. Person whose birthday is on June is an a) Tilak, 53
immediate neighbour of the persons whose birthday are b) Person who sits to the immediate right of the person
on April and August. Person whose age is 12 sits second whose age is 8years and his age is 32
to the right of the person whose age is 25. Person whose c) Person who sits between Tilak and Kali and age is 45
birthday is on March doesn't sit at the end of the row and d) Person sits fourth from the right end age is 41
he is younger than the person whose birthday is on May. e) None of These
1) Which among the following combination is correct? 5) What is the age and position of Tilak as per the given
a) Suber-April-12 arrangement?
b) Ankur-July-32 a) 41, Second to the left of Pritam
c) Pritam-May-45 b) 41, second from the right end
d) Riyaz-Feb-30 c) 41, Third to the right of Iqbal
e) Iqbal-August-41 d) 41, Immediate right of Ankur
2) Four of the five are alike in a certain way and hence e) 41, Immediate right of Kali
forms a group. Which among the following does not (Direction 6-10): Study the following information
belong to that group? carefully and answer the given questions:
a) Person whose age is 45 There are nine persons i.e. Riya, Jiya, Tia, Mia, Piya, Siya,
b) Person who sits to the immediate left of Fatima Lyra, Myra and Tara are sitting in a linear row. Four are
c) Person who sits third to the right of Suber facing south and Five persons are facing north direction.
d) Person who sits exactly between Riyaz and Fatima Each of them visits different places i.e. Pune, Kanpur,
e) Person whose age is 41 Goa, Delhi, Noida, Mumbai, Jaipur, Mohali and
3) Which among the following statements is/are not Anantpur. Each of them bought different number of items
true?
Page 148 of 1334
Get a Special Paid Bundle PDF on High Level Puzzles & Seating for All Bank Clerk/ PO Mains
If there are any suggestions/ errors in our PDFs Feel Free to contact us via this email: admin@exampundit.in
Ultra Practice Bundle PDF
Reasoning Questions for SBI Clerk/ RBI Assist. Mains
i.e. 2, 5, 9, 10, 12, 15, 18, 21 and 25 but not necessarily in b) Lyra and the person who visits Jaipur
the same order. c) Person who bought 21 items and Jiya
Tara sits at one of the ends of the row and visits Anantpur d) Both A and B
also she has bought even number of items. Mia sits third e) Both A and C
to the left of Tara and bought odd number odd items. 7) Which among the following pairs sit at the end of
Person who visits Mumbai has bought 9 items and sits the row?
second to the left of Mia. Persons sit at the end of the row a) Siya and the person who visits Anantpur
faces the same direction. Piya who faces south sits at the b) Person who bought 12 items and Tara
middle of the row and has bought least number of items. c) Person who visits Delhi and Tara
Two persons sit between Piya and the person who visits d) Both A and C
Pune who is not an immediate neighbour of Tara. Tia e) All A, B and C
neither visits Goa nor Delhi. Persons who bought 21 items 8) Four of the five are alike in a certain way and hence
and 25 items are immediate neighbours of each other. forms a group. Who among the following does not
Neither Mia nor Riya bought 21 items. Jiya sits second to belong to that group?
the left of Tia who bought prime number of items. Neither a) Person who visits Pune
Jiya nor Tia sits at the end of the row. Jiya who faces north b) Person who bought least number of items.
visits neither Pune nor Jaipur. Lyra visits Mohali but she c) Person who visits Noida
is not an immediate neighbour of both Mia and Tia. Three d) Person who sits exactly between Tia and Jiya
persons sit between the persons who bought 18 items and e) Person who bought 25 items and visits Mohali
21 items. Siya has bought less items than Mia and she 9) Which of the following statements is/are true?
visits either Goa or Delhi. Immediate neighbours of Lyra a) Riya visits Goa and bought 9 items
are facing opposite direction to that of Lyra. Person visits b) Myra and Tara are immediate neighbours
Goa sits exactly between Siya and the person who visits c) Mia doesn't visit Kanpur and bought 15 items
Jaipur who sits two places away from the person visits d) Both B and C
Noida. More than three persons sit to the left of Riya. e) None is true
6) Who among the following sits second to the right 10) Who among the following sits second to the left of
and to the left of Mia respectively? Tara?
a) Person who visits Mohali and the person who bought 9 a) Person who visits Mohali
items b) Myra
Page 149 of 1334
Get a Special Paid Bundle PDF on High Level Puzzles & Seating for All Bank Clerk/ PO Mains
If there are any suggestions/ errors in our PDFs Feel Free to contact us via this email: admin@exampundit.in
Ultra Practice Bundle PDF
Reasoning Questions for SBI Clerk/ RBI Assist. Mains
c) Lyra belongs to Japan faces north and is an immediate
d) None neighbour of the persons belong to India and UK.
e) Person who visits Kanpur Immediate neighbours of Kumud faces opposite direction
(Direction 11-15): Study the following information to that of Kumud. Person speaks Hindi sits second to the
carefully and answer the given questions: left of the person speaks Russian. Mihir neither speaks
There are nine persons i.e. Kumud, Johny, Meher, Ruhi, Chinese nor Arabic but he faces south. Person who speaks
Mihir, Sejal, Praful, Riansh and Aryan are sitting in a Arabic faces the opposite as the person who speaks
linear row. Four are facing north and Five persons are Spanish who faces opposite to that of the person who
facing south direction. Each of them belongs to different speaks Urdu.
countries i.e. India, China, France, Japan, USA, UK, 11) Which of the following combination is not correct?
Spain, Russia and Nepal. Each of them also speaks a) Ruhi-Spanish
different languages Urdu, Hindi, English, Russian, b) Aryan - France
Japanese, Spanish, French Arabic and Chinese but not c) Spain - japanese
necessarily in the same order. d) Mihir - Hindi
Kumud sits third from the right end and speaks Japanese. e) All are correct
Praful and Riansh are an immediate neighbours of 12) Who among the following sits exactly between
Kumud. Neither Praful nor Riansh belongs to Spain. Ruhi and Mihir?
Person who belongs to France speaks Russian and faces a) Person who speakes Japanese
north. Mihir sits three places away from Riansh and b) Person who is from USA
belongs to UK. Two persons sit between Praful and Sejal c) Person who speakes English
who speaks French and faces south. Ruhi who speaks d) Person who is from Nepal
Spanish sits third to the left of Aryan who doesn't sit at the e) Meher
end of the row. Meher who belongs to Japan sits exactly 13) Who sits Immediate Right of the one who speaks
between Mihir and Ruhi and they are next to one other. Japanese?
Johny ,who does not speak Chinese, belongs to either a) Praful
India or USA. Person who belongs to Russia speaks Urdu b) Ruhi
and sits to the immediate left of the person belongs to c) Aryan
Spain who faces north. Person who sits at the middle of d) Mihir
the row belongs to either France or China. Person who e) Meher
Page 150 of 1334
Get a Special Paid Bundle PDF on High Level Puzzles & Seating for All Bank Clerk/ PO Mains
If there are any suggestions/ errors in our PDFs Feel Free to contact us via this email: admin@exampundit.in
Ultra Practice Bundle PDF
Reasoning Questions for SBI Clerk/ RBI Assist. Mains
14) Who sits Immediate left of the one who speaks direction as Sahil faces. Alia sits third to the left of Mona
Urdu? who likes Almond and doesn't sit at the end of the row.
a) Meher Person who sits at the left end of the row likes Blue colour
b) Mihir and eats Vanilla ice cream. Abhay and Nitin who likes
c) Kumud Red are an immediate neighbours and both are facing
d) Aryan opposite direction to each other. Abhay neither likes Black
e) None of these nor Pink and eats Apple ice-cream. Persons like Black and
15) Who sits third to the left of the one who speaks Red colour are an immediate neighbours of each other.
Urdu? Tina likes either Orange or Grey but neither likes Apple
a) Praful nor Pista. Person who likes Pink also likes Blueberry and
b) Johny sits fourth to the left of the person who likes Red. Not
c) Sejal more than two persons facing the same direction are
d) Ruhi adjacent to each other. Abhay neither likes Green nor
e) None of these Orange. Ziva faces the same direction as the Person who
(Direction 16-20): Study the following information likes Yellow who is an immediate neighbour of the person
carefully and answer the given questions: eats Chocolate ice-cream. Person eats Vanilla ice-cream
There are nine persons i.e. Mona, Alia, Manju, Ziva, sits second to the right of the person who eats Pineapple
Nitin, Jacky, Tina, Sahil and Abhay are sitting in a linear ice-cream and faces opposite to that person who likes
row. Four are facing south and Five persons are facing Mango ice-cream who sits at the end of the row.
north direction. Each of them eats different flavour of ice- 16) Which of the following combination is correct?
cream i.e. Pista, Chocolate, Mango, Apple, Pineapple, a) Nitin-Pineapple-Black
Vanilla, Blueberry, Butterscotch and Almond. Each of b) Alia-Blue-Apple
them also likes different colours i.e. Red, Yellow, Green, c) Tina-Chocolate-Orange
Blue, Black, White, Grey, Pink and Orange but not d) Mona-Green-Almond
necessarily in the same order. e) Jacky-Yellow-Blueberry
Sahil and Ziva are an immediate neighbours of Jacky who 17) Who among the following sits to the immediate left
likes White and eats Butterscotch. Sahil sits at the middle and to the immediate right of each other?
of the row and he faces north. Two persons sit between a) Person likes Black and Sahil
Manju and Jacky. Manju likes Yellow and faces the same b) Ziva and the person likes Orange
Page 151 of 1334
Get a Special Paid Bundle PDF on High Level Puzzles & Seating for All Bank Clerk/ PO Mains
If there are any suggestions/ errors in our PDFs Feel Free to contact us via this email: admin@exampundit.in
Ultra Practice Bundle PDF
Reasoning Questions for SBI Clerk/ RBI Assist. Mains
c) Person likes Pista and Jacky Hindi, English, Maths, Physics, Biology, Chemistry,
d) Both A and B History, Geography and Computer. Each of them also got
e) Both A and C different number in that subject i.e. 64, 65, 70, 72, 78, 84,
18) Who among the follwing sits fourth to the right of 88, 90 and 96 but not necessarily in the same order.
Abhay? Not more than two persons are facing the same direction
a) Person who likes Blueberry are adjacent to each other. Julia and Kajal are sitting at the
b) Person who likes Butterscotch ends of the row and both are facing north. Angel got 84
c) Person who likes Black marks in Hindi and faces south. Mishi sits three places
d) Person who likes Pista away from Julia and likes English. Person who likes
e) Person who likes Pink Chemistry got more marks than Mishi but less than Julia.
19) Which of the following statement is/are incorrect? Two persons sit between Angel and Jaggu who got 70
a) Nitin likes Apple marks in his favourite subject. Only one person sits
b) Abhay likes Grey between Kajal and Rashid who faces opposite to that of
c) Manju likes Yellow and eats Pista Kajal and likes Chemistry. Rishi sits to the immediate
d) Both A and B right of Rashid and got 88 marks in History. Only one
e) Both A and C person sits between Angel and Mishi who got 65 marks
20) What is the position of Jacky with respect to the and faces north. Julia doesn't get the highest marks.
Person eats Mango icecream? Persons who got 84 marks and 90 marks are an immediate
a) Second to the left neighbours of each other and also both are facing the same
b) Third to the right direction. Person who sits at the middle of the row likes
c) Second to the right Biology and faces the same direction as the person who
d) Third to the left likes Computer, who got the highest marks. Person who
e) Immediate left likes Maths sits second to the right of the person who likes
(Direction 21-25): Study the following information Physics. Rishi faces south
carefully and answer the given questions: 21) Which of the following combination is not correct?
There are nine persons i.e. Mitika, Angel, Kajal, Mishi, a) Rishi-History-88
Jaggu, Julia, Rashid, Kyra and Rishi are sitting in a linear b) Kyra-Maths-90
row. Four are facing south and Five persons are facing c) Kajal-Computer-64
north direction. Each of them likes different subjects i.e. d) Jaggu-Biology-70
Page 152 of 1334
Get a Special Paid Bundle PDF on High Level Puzzles & Seating for All Bank Clerk/ PO Mains
If there are any suggestions/ errors in our PDFs Feel Free to contact us via this email: admin@exampundit.in
Ultra Practice Bundle PDF
Reasoning Questions for SBI Clerk/ RBI Assist. Mains
e) All are correct (Direction 26-30): Study the following information
22) Who among the following sits exactly between carefully and answer the given questions:
Jaggu and Kajal? There are nine persons i.e. Shivam, Agrim, Neetu, Neeta,
a) Person who scores 64 marks Niti, Azhar, Raju, Pinku and Nitesh are sitting in a linear
b) Person who likes History row. Four are facing south and five persons are facing
c) Person who scores 78 marks north direction. Each of them gets the admission in
d) Person who likes Chemistry different universities i.e. Oxford, Delhi University (DU),
e) Both B and C Cornell, Duke, Amity, MIT, MNNIT, SRM and Harvard
23) What is sum of the marks scored by Mishi and the on different days of the week but not necessarily in the
person who likes Physics? same order. Not more than two persons got the admission
a) 148 on the same day and the persons who got the admission on
b) 134 same day are not adjacent to each other.
c) 314 Azhar and Nitesh are an immediate neighbour of each
d) 341 other and facing opposite direction to each other. Either
e) 143 Azhar or Nitesh sits at one of the ends of the row. Person
24) Which of the following statement is/are not true? who got admission in MIT sits at the end of the row.
a) Angel likes Hindi and Scores 88 marks Neither Azhar nor Nitesh got admitted in MIT. Nitesh
b) Rashid likes Chemistry and score 72 marks who got admission in SRM and Neetu are getting
c) Julia likes Physics and scores 70 marks admission on the same day. Two persons sit between Niti
d) Both A and B who got admission on Monday and Nitesh who faces
e) Both B and C south. Neeta sits exactly between Neetu and Niti but not
25) Which of the following combination of subjects immediate and faces the same direction as Azhar and got
liked by Mitika and Kyra respectively? admission in Harvard on Thursday. Shivam who got
a) Maths and Geography admission on Tuesday sits third to the right of the person
b) Geography and Hindi got admitted in Harvard and both are facing the same
c) Geography and Maths direction. No one get admission on the same day on which
d) Maths and English Neeta and Shivam who get admission in Delhi University
e) None of these (DU) got. Raju who got admission on Monday faces south
and he neither admitted in Oxford nor in MIT. Pinku got
Page 153 of 1334
Get a Special Paid Bundle PDF on High Level Puzzles & Seating for All Bank Clerk/ PO Mains
If there are any suggestions/ errors in our PDFs Feel Free to contact us via this email: admin@exampundit.in
Ultra Practice Bundle PDF
Reasoning Questions for SBI Clerk/ RBI Assist. Mains
admission in Amity on Sunday and sits second to the left e) Both B and C
of Shivam. Agrim got admission either in Duke or in 29) Who among the following sits exactly between Niti
Oxford on Saturday alone and faces south and he is an and Neetu?
immediate neighbour of Shivam. Niti faces the same a) Person who got admission in Harvard.
direction as Agrim faces who sits to the left of Niti. Neetu b) Person who sits second from the left end of the row
got admission neither on Sunday nor on Friday. Niti got c) Person who got admission on Tuesday
admission neither on MNNIT nor on Oxford. Person who d) Both A and B
got admission in Oxford and MNNIT are not sit at the end e) Both A and C
of the row. Person who got admission in Duke doesn't 30) Who among the following got admission in SRM
share his admission day with anyone. and on which day?
26) Which of the following combination is not correct? a) Person sits to the right of Agrim, Monday
a) Niti-Cornell- Monday b) Person sits to the immediate left of Azhar, Wednesday
b) Agrim-Oxford-Saturday c) Person sits third from the right end, Thursday
c) Azhar-Amity-Friday d) Person sits at the right end, Friday
d) Neeta-Harvard-Thursday e) Person sits between Agrim and Azhar, Saturday
e) Neetu-MIT-Wednesday (Direction 31-35): Study the following information
27) Who among the following sits third to the right of carefully and answer the given questions:
the person who get admission in Delhi university? There are nine persons i.e. Ishu, Pulkit, Nancy, Kavi,
a) Person who got admission in SRM Saanu, Gargi, Meneka, Rohit and Bhavesh are sitting in a
b) Person who got admission in Duke linear row. Four are facing south and Five persons are
c) Person who got admission in Oxford facing north direction. All of them studies in different IIT
d) Person who got admission in MNNIT colleges i.e. IIT Delhi, IIT Bombay, IIT Ropar, IIT
e) Person who got admission in Amity Roorkee, IIT Patna, IIT Goa, IIT BHU, IIT Indore and IIT
28) Which of the following statement is/are correct? Madras and have their exams in different month i.e.
a) Pinku sits between Neeta and Shivam January, February, March, April, May, June, July,
b) Agrim got admission in Oxford university October and December but not necessarily in the same
c) Person got admission in MIT sits at the right end of the order.
row Not more than two persons facing the same direction are
d) Both A and B adjacent to each other. Saanu who studied in BHU sits at
Page 154 of 1334
Get a Special Paid Bundle PDF on High Level Puzzles & Seating for All Bank Clerk/ PO Mains
If there are any suggestions/ errors in our PDFs Feel Free to contact us via this email: admin@exampundit.in
Ultra Practice Bundle PDF
Reasoning Questions for SBI Clerk/ RBI Assist. Mains
the middle of the row and faces north. Pulkit who has his 32) Who among the following pair sits to the
exam on May sits two places away from Saanu and faces immediate right and immediate left of each other
south. Person who sits at the right end of the row studied respectively?
in Bombay and faces north. Pulkit neither studied in a) Pulkit and the person who studies in IIT Goa
Madras nor in Indore. Two persons sit between the person b) Person whose exam is on October and Ishu
who studied in Ropar and has his exam in June and Saanu c) Gargi and the person who studies in IIT Delhi
who has her exam in July. Neither Rohit who faces north d) Both A and B
nor Meneka who has her exam in the month having e) Both A and C
30days in Roorkee sits at the end of the row but both are 33) Which of the following statement is/are not true
sitting to the immediate right of each other. Meneka is not regarding Meneka?
an immediate neighbour of Saanu. Gargi who faces north a) Meneka sits third from the end of the row.
sits to the immediate right of Pulkit. Nancy is an b) Nancy and Rohit are neighbours of Meneka
immediate neighbour of both Meneka and Ishu who c) Meneka does not study in IIT Roorkee.
studied in Bombay and has her exam in February. Kavi d) Meneka has her exam in April.
who faces opposite direction to that of Ishu is studied e) All are correct
either in Delhi or in Indore and has his exam in March. 34) Who among the following sits second to the right
Bhavesh doesn't sit at the end of the row and has studied of Bhavesh?
in Goa also he has his exam after the exam of Nancy a) Kavi
whose exam is held just after the exam of Saanu. Person b) Gargi
studied in Indore is not an immediate neighbour of the c) Rohit
persons who studied in Ropar or BHU. d) Saanu
31) Number of persons sits to the left of Pulkit is equal e) Pulkit
to the number of person sits to the right of___. 35) Which of the following combination is not correct?
a) Person who studies in IIT Roorkee a) Saanu-IIT BHU-July
b) Person who is a neighbour of Rohit b) Gargi-IIT Ropar-June
c) Person whose exam is on April c) Meneka-IIT Roorkee-April
d) Both A and B d) Ishu-IIT Bombay-January
e) All A, B and C e) Kavi-IIT Delhi-March

Page 155 of 1334


Get a Special Paid Bundle PDF on High Level Puzzles & Seating for All Bank Clerk/ PO Mains
If there are any suggestions/ errors in our PDFs Feel Free to contact us via this email: admin@exampundit.in
Ultra Practice Bundle PDF
Reasoning Questions for SBI Clerk/ RBI Assist. Mains
(Direction 36-40): Study the following information north is not an immediate neighbour of both Tarun and
carefully and answer the given questions: Tanuj. Person who has presentation on 19 sits third to the
There are nine persons i.e. Tarun, Tanuj, Chinky, Justin, right of the person who has presentation on Bitcoin who
Zubin, Priya, Dipti, Faiza and Rahul are sitting in a linear sits second to the left of the person who has presentation
row. Four are facing north and Five persons are facing on Pollution. Person who has presentation on a square of
south direction. All of them prepare presentation in a number sits second to the right of the person who has
different topics i.e. 5G vs 4G, Nanotech, Pollution, Radar, presentation on cube of a number. Justin is not an
Biochips, Bitcoin, Sales, E-cars and E-marketing and have immediate neighbour of Faiza but faces same direction as
their presentation in different dates i.e. 5, 8, 12, 16, 19, 24, Faiza and Dipti faces.
25, 28 and 31 but not necessarily in the same order. 36) Who among the following sits at any end of the
Faiza who faces south has her presentation on 5. Less than row?
four person sits to the left of Faiza. Chinky who sits at the a) Person has presentation on Bitcoin
left end of the row has her presentation on Radar and faces b) Person has presentation on Biochip
opposite direction to that of Justin. Dipti who has c) Person has presentation on Nanotech
presentation on 5G vs 4G sits second to the left of Tarun d) Both A and B
who has his presentation on 12. Priya who sits second to e) Both B and C
the right of Chinky has presentation on either Nanotech or 37) Which of the following combination is correct?
Pollution. Zubin who has presentation on Sales on 28 is a) Dipti-E-cars-28
an immediate neighbour of Priya but not an immediate b) Chinky-Pollution-12
neighbour of Chinky. Justin who sits second to the right c) Priya-Nanotech-8
of Tarun who has presentation on E-marketing and faces d) Justin-Bitcoin-31
north. Justin has his presentation on prime numbered date e) Zubin-Sales-24
but not on 19. Person who has presentation on Bitcoin and 38) Which of the following pair facing the same
the person who has presentation on Biochips are an direction are adjacent to each other?
immediate neighbour of each other. Neither Dipti nor a) Person who has presentation on 5G vs 4G and Faiza
Priya has presentation on Bitcoin. Person who has b) Rahul and Priya
presentation on E-cars on 16 sits second to the left of the c) Tarun and the person whose presentation is on 24
person who has presentation on 28. Tanuj has presentation d) Both A and B
neither on 16 nor on 8 and faces south. Rahul who faces e) Both A and C
Page 156 of 1334
Get a Special Paid Bundle PDF on High Level Puzzles & Seating for All Bank Clerk/ PO Mains
If there are any suggestions/ errors in our PDFs Feel Free to contact us via this email: admin@exampundit.in
Ultra Practice Bundle PDF
Reasoning Questions for SBI Clerk/ RBI Assist. Mains
39) Four of the five are alike in a certain way and hence direction to that of Surbhi. Richa sits three places away
forms a group. Who among the following does not from Surbhi and likes Mohit and uses BSNL. Only one
belong to that group? person sits between Richa and Sweety who faces north
a) Person whose presentation on Radar and likes either Alka or Shreya. Pari uses neither Airtel
b) Tarun nor Jio. Nimmi who likes Arjit and uses neither Jio nor
c) Person whose presentation on 28 Airtel sits at the left end of the row and faces the same
d) Person whose presentation on Biochip direction as Sweety faces. Preeti sits second to the right of
e) Rahul Nimmi and uses Vodafone and Both are facing opposite
40) Number of persons sits to the left of Tanuj is equal direction to each other. Neither Mahima nor Sweety uses
to the number of persons sits to the right of___. Uninor. Neither Astha nor Pari likes Shreya. Person who
a) Person whose presentation on E-cars likes Sunidhi is not an immediate neighbour of the persons
b) None who uses Vodafone and Sity. Prama sits exactly between
c) Priya the person who uses Vodafone and the person who likes
d) Person whose presentation is on 8 Neha who doesn't sit at the end of the row. Preeti does not
e) None of these like Neha. Person who uses Uninor sits third to the left of
(Direction 41-45): Study the following information the person who likes Mahadevan. Prama neither likes
carefully and answer the given questions: Mahadevan nor Alka and faces north which is opposite to
There are nine persons i.e. Prama, Richa, Preeti, Mahima, that of Richa. Pari who faces south likes Alka is an
Nimmi, Pari, Sweety, Astha and Surbhi are sitting in a immediate neighbour of the person who likes Armaan.
linear row. Four are facing north and Five persons are Astha likes Hathway and sits second to the left of the
facing south direction. All of them uses different network person who uses Aircel and both are facing opposite
i.e. Airtel, Jio, Aircel, Uninor, Hathway, Idea, Sity, direction to each other. Person who uses Airtel and the
Vodafone and BSNL and each person likes different person who uses Idea are an immediate neighbour of each
singers i.e. Mohit, Arjit, Alka, Saanu, Sunidhi, other and facing the same direction.
Mahadevan, Shreya, Armaan and Neha but not necessarily 41) Who among the following sits exactly between
in the same order. Preeti and the person who uses Sity?
Surbhi who faces south sits third from the right end of the a) Person who sits to the immediate left of Astha
row and uses Sity network. Mahima who likes Armaan is b) Person who likes Sunidhi
an immediate neighbour of Surbhi and faces opposite c) Person who faces south
Page 157 of 1334
Get a Special Paid Bundle PDF on High Level Puzzles & Seating for All Bank Clerk/ PO Mains
If there are any suggestions/ errors in our PDFs Feel Free to contact us via this email: admin@exampundit.in
Ultra Practice Bundle PDF
Reasoning Questions for SBI Clerk/ RBI Assist. Mains
d) Person who uses Hathway (Direction 46-50): Study the following information
e) Person who uses Jio but does not like Sunidhi carefully and answer the given questions:
42) Which of the following statement is/are not true? There are nine persons i.e. Dheeraj, Nimo, Jackal, Aashu,
a) Astha does not uses Hathway Vinay, Abhi, Tapan, Pranay and Varun are sitting in a
b) Pari does not like Alka linear row. Five are facing north and Four persons are
c) Surbhi likes Neha facing south direction. All of them uses laptop of different
d) Both A and B brands i.e. Dell, Asus. Acer, Apple, Lenevo, MSI, HP,
e) All are not true Microsoft and Razer and also each person likes different
43) Who among the following sits third to the right of chocolates i.e. Dairy Milk, 5-Star, Perk, Munch, Snickers,
the person who uses BSNL? Fuse, Bournville, Milky Bar and Kitkat but not necessarily
a) Person who uses Idea in the same order. Not more than two persons of same
b) Sweety direction are sitting together.
c) Person who likes Shreya Vinay sits third from the left end and Abhi sits third from
d) Person who does not like Arjit the right end of the row and both faces opposite direction
e) Both A and D to each other. Either Vinay or Abhi likes Dairy Milk.
44) Which of the following pair facing the same Jackal who uses Microsoft sits second to the right of Abhi
direction are adjacent to each other? and faces north. Dheeraj who sits three places away from
a) Prama and Mahima Vinay likes Snickers and faces the same direction as
b) Richa and Pari Jackal faces. Neither Vinay nor Varun uses Asus. Tapan
c) Sweety and Nimmi who is an immediate neighbour of Vinay likes Munch and
d) Astha and Preeti uses Lenevo. Nimo sits second to the left of Tapan and
e) All are correct uses Dell. Nimo faces opposite direction to that of Tapan.
45) Who among the following likes Alka and which Person who uses MSI and the person who uses Apple are
network does he use? immediate neighbours of each other. Number of persons
a) Mahima, Uninor sits to the left of Pranay is equal to the number of persons
b) Person who sits to the immediate left of Mahima, Aircel sits to the right of Nimo who likes Perk. Pranay uses
c) Person who sits at left end of the row, Uninor neither Asus nor HP. Person who likes Bournville sits
d) Nimmi, Idea exactly between the person who likes Fuse and the person
e) Person who sits at the right end of the row, Uninor who likes Dairy Milk. Varun who faces south uses Acer
Page 158 of 1334
Get a Special Paid Bundle PDF on High Level Puzzles & Seating for All Bank Clerk/ PO Mains
If there are any suggestions/ errors in our PDFs Feel Free to contact us via this email: admin@exampundit.in
Ultra Practice Bundle PDF
Reasoning Questions for SBI Clerk/ RBI Assist. Mains
and likes either Kitkat or Bournville but doesn't sit at the d) Person who likes Bournville
end of the row. Neither Pranay nor Jackal likes Bournville. e) Person who uses Dell
Person who likes Milky Bar sits to the right of the person 48) Which of the following combination is not correct?
who uses Razer who is not Pranay and both faces opposite a) Aashu-5-star-Razer
direction to each other. Aashu doesn't faces south and b) Dheeraj-Fuse-HP
likes 5-star. Person who likes Fuse is an immediate c) Varun-Bournville-Asus
neighbour of the persons who uses HP and the person who d) Both B and C
uses Apple brand. e) Both A and B
46) Who among the following sits second to the right 49) Who among the following sits exactly between
of the person who uses HP? Tapan and Pranay?
a) Person who likes Fuse a) Varun
b) Person who uses Apple brand b) Dheeraj
c) Person who likes Kitkat c) Abhi
d) Person who likes Milky Bar d) Vinay
e) Both B and D e) Aashu
47) Four of the five are alike in a certain way and hence 50) Which of the following statement is/are not true?
forms a group. Who among the following does not a) Person who uses Microsoft sits at the end of the row
belong to that group? b) Person who uses Acer sits to the left of Dheeraj
a) Person who uses Asus c) Person who likes Snickers does not uses HP
b) Person who likes Snickers d) Both A and B
c) Person who uses Apple e) All are true

Seating Linear 3 Variable – Answer and Explanation


Answers (1-5): Solutions (1-5):
1) C STEP I: Person who sits on the right end of the row faces
2) D north and the person who sits on the left end of the row
3) E faces south.
4) B As per these statements, the arrangement will look like
5) A this:

Page 159 of 1334


Get a Special Paid Bundle PDF on High Level Puzzles & Seating for All Bank Clerk/ PO Mains
If there are any suggestions/ errors in our PDFs Feel Free to contact us via this email: admin@exampundit.in
Ultra Practice Bundle PDF
Reasoning Questions for SBI Clerk/ RBI Assist. Mains
STEP IV: Iqbal sits towards the left of Fatima and his age
is a prime number.
As per this statement, CASE I and CASE II will get split
STEP II: Jayant sits second to the left of Fatima whose into two more cases and the arrangement will look like
birthday is on the month having 31 days but before May. this:
Fatima sits at the middle of the row. CASE I:
As per this arrangement, there will be two cases getting
framed and the arrangement will look like this:
CASE I:

CASE I (A):

CASE II:

CASE II:

STEP III: Riyaz who is elder than Fatima sits second to


the left of Jayant and his birthday is on February.
As per this statement, the arrangement will look like this:
CASE II (A):
CASE I:

STEP V: Immediate neighbours of Kali are facing


CASE II:
opposite direction to each other. Three persons sit between
Iqbal and Kali.
As per these statements, the arrangement will look like
this:
Page 160 of 1334
Get a Special Paid Bundle PDF on High Level Puzzles & Seating for All Bank Clerk/ PO Mains
If there are any suggestions/ errors in our PDFs Feel Free to contact us via this email: admin@exampundit.in
Ultra Practice Bundle PDF
Reasoning Questions for SBI Clerk/ RBI Assist. Mains
CASE I:

CASE II:
CASE I (A):

CASE II: CASE II (A):

CASE II (A): STEP VII: Ankur and Tilak whose birthday is on July are
immediate right and immediate left of each other
respectively. Neither Ankur nor Tilak sits at the end of the
row.
As per these statements, the arrangement will look like
STEP VI: Kali who is the eldest person is having her
this:
birthday on the month of 30 days after April and she faces
CASE I:
north.
As per this statement, the arrangement will look like this:
CASE I:

CASE I (A):

CASE I (A):

Page 161 of 1334


Get a Special Paid Bundle PDF on High Level Puzzles & Seating for All Bank Clerk/ PO Mains
If there are any suggestions/ errors in our PDFs Feel Free to contact us via this email: admin@exampundit.in
Ultra Practice Bundle PDF
Reasoning Questions for SBI Clerk/ RBI Assist. Mains

CASE II:
CASE II:

CASE II (A):
CASE II (A):

STEP VIII: Ankur has his birthday on the month having


31 days but neither on January nor on August also his age STEP IX: Pritam sits third to the left of Ankur and he is
elder than both Ankur and Tilak.
is four times the age of Fatima who is an immediate
neighbour of the person whose birthday is on August. As per this statement, CASE I, CASE II and CASE II
(A) will get eliminated and we will continue with CASE
As per this statement, the arrangement will look like this:
CASE I: I (A) and the arrangement will look like this:
CASE I:

CASE I (A):
CASE I (A):

Page 162 of 1334


Get a Special Paid Bundle PDF on High Level Puzzles & Seating for All Bank Clerk/ PO Mains
If there are any suggestions/ errors in our PDFs Feel Free to contact us via this email: admin@exampundit.in
Ultra Practice Bundle PDF
Reasoning Questions for SBI Clerk/ RBI Assist. Mains

CASE II: STEP XII: Person whose birthday is on March doesn't sit
at the end of the row and he is younger than the person
whose birthday is on May. Person whose age is 12 sits
second to the right of the person whose age is 25.
As per this statement, the final arrangement will look like
CASE II (A): this:
CASE I (A):

STEP X: Suber who doesn't face south is younger than


Answers (6-10):
Tilak but elder than Iqbal and his birthday is neither on
6) A
June nor on September.
7) D
As per this statement, the arrangement will look like this:
8) C
CASE I (A):
9) E
10) B
Solutions (6-10):
STEP I: Tara sits at one of the ends of the row and visits
Anantpur also she has bought even number of items.
STEP XI: Person whose birthday is on June is an
As per this statement, there will be two cases getting
immediate neighbour of the persons whose birthday are
framed and the arrangement will look like this:
on April and August.
CASE I:

As per this statement, the arrangement will look like this:


CASE I (A):
Page 163 of 1334
Get a Special Paid Bundle PDF on High Level Puzzles & Seating for All Bank Clerk/ PO Mains
If there are any suggestions/ errors in our PDFs Feel Free to contact us via this email: admin@exampundit.in
Ultra Practice Bundle PDF
Reasoning Questions for SBI Clerk/ RBI Assist. Mains
CASE I (A):

CASE II:
CASE II:

STEP II: Mia sits third to the left of Tara and bought odd
number odd items. CASE II (A):

As per this statement, the arrangement will look like this:


CASE I:

STEP IV: Persons sit at the end of the row faces the same
direction. Piya who faces south sits at the middle of the
CASE II: row and has bought least number of items.
As per these statements, the arrangement will look like
this:
CASE I:

STEP III: Person who visits Mumbai has bought 9 items


and sits second to the left of Mia.
As per this statement, CASE I and CASE II will get split
into two more cases and the arrangement will look like CASE I (A):
this:
CASE I:

CASE II:

Page 164 of 1334


Get a Special Paid Bundle PDF on High Level Puzzles & Seating for All Bank Clerk/ PO Mains
If there are any suggestions/ errors in our PDFs Feel Free to contact us via this email: admin@exampundit.in
Ultra Practice Bundle PDF
Reasoning Questions for SBI Clerk/ RBI Assist. Mains
STEP VII: Tia neither visits Goa nor Delhi. Persons who
bought 21 items and 25 items are immediate neighbours
of each other. Neither Mia nor Riya bought 21 items.
As per this statement, the arrangement will look like this:
CASE II (A):
CASE I:

STEP V: Two persons sit between Piya and the person


who visits Pune who is not an immediate neighbour of CASE I (A):

Tara.
As per this statement, the arrangement will look like this:
CASE I:

CASE II:

CASE I (A):

CASE II (A):

CASE II:

STEP VIII: Jiya sits second to the left of Tia who bought
prime number of items. Neither Jiya nor Tia sits at the end
of the row. Jiya who faces north visits neither Pune nor
CASE II (A):
Jaipur.
As per this statement, the arrangement will look like this:
CASE I:

Page 165 of 1334


Get a Special Paid Bundle PDF on High Level Puzzles & Seating for All Bank Clerk/ PO Mains
If there are any suggestions/ errors in our PDFs Feel Free to contact us via this email: admin@exampundit.in
Ultra Practice Bundle PDF
Reasoning Questions for SBI Clerk/ RBI Assist. Mains

CASE I (A): CASE II:

CASE II (A):
CASE II:

CASE II (A): STEP X: Three persons sit between the persons who
bought 18 items and 21 items.
As per this statement, the arrangement will look like this:
CASE I (A):

STEP IX: Lyra visits Mohali but she is not an immediate


neighbour of both Mia and Tia.
As per this statement, CASE I and CASE II (A) will get
eliminated and we will continue with CASE I (A) and CASE II:

CASE II and the arrangement will look like this:


CASE I:

STEP XI: Siya has bought less items than Mia and she
visits either Goa or Delhi. Immediate neighbours of Lyra
are facing opposite direction to that of Lyra. Four are
CASE I (A):
facing south and five persons are facing north direction.

Page 166 of 1334


Get a Special Paid Bundle PDF on High Level Puzzles & Seating for All Bank Clerk/ PO Mains
If there are any suggestions/ errors in our PDFs Feel Free to contact us via this email: admin@exampundit.in
Ultra Practice Bundle PDF
Reasoning Questions for SBI Clerk/ RBI Assist. Mains
As per these statements, CASE I (A) will get eliminated Solutions (11-15):
and we will continue with CASE II and the arrangement STEP I: Kumud sits third from the right end and speaks
will look like this: Japanese.
CASE I (A): As per this statement, the arrangement will look like this:

STEP II: Praful and Riansh are an immediate neighbours


CASE II:
of Kumud.
As per this statement, the arrangement will look like this:

STEP XII: Person visits Goa sits exactly between Siya


and the person who visits Jaipur who sits two places away
STEP III: Mihir sits three places away from Riansh and
from the person visits Noida. More than three persons sit
he belongs to UK.
to the left of Riya.
As per this statement, there will be two cases getting
As per these statements, the final arrangement will look
framed and the arrangement will look like this:
like this:
CASE I:
CASE II:

Answers (11-15): CASE I (A):


11) D
12) E
13) A
14) C
CASE II:
15) B

Page 167 of 1334


Get a Special Paid Bundle PDF on High Level Puzzles & Seating for All Bank Clerk/ PO Mains
If there are any suggestions/ errors in our PDFs Feel Free to contact us via this email: admin@exampundit.in
Ultra Practice Bundle PDF
Reasoning Questions for SBI Clerk/ RBI Assist. Mains
STEP V: Ruhi who speaks Spanish sits third to the left of
Aryan who doesn't sit at the end of the row. Meher who
belongs to Japan sits exactly between Mihir and Ruhi.
As per these statements, CASE I (A) and CASE II will
STEP IV: Two persons sit between Praful and Sejal who
get eliminated and we will continue with CASE I and
speaks French and faces south.
CASE II (A) and the arrangement will look like this:
As per this statement, CASE II will get spilt into one more
CASE I:
case and the arrangement will look like this:
CASE I:

CASE I (A):

CASE I (A):

CASE II:

CASE II:

CASE II (A):

CASE II (A):

STEP VI: Johny belongs to either India or USA.


As per this statement, the arrangement will look like this:
CASE I:

Page 168 of 1334


Get a Special Paid Bundle PDF on High Level Puzzles & Seating for All Bank Clerk/ PO Mains
If there are any suggestions/ errors in our PDFs Feel Free to contact us via this email: admin@exampundit.in
Ultra Practice Bundle PDF
Reasoning Questions for SBI Clerk/ RBI Assist. Mains
As per these statements, CASE II (A) will get eliminated
and we will continue with CASE I and the arrangement
will look like this:
CASE I:
CASE II (A):

CASE II (A):
STEP VII: Neither Praful nor Riansh belongs to Spain.
Person who belongs to France speaks Russian and faces
north. Person who belongs to Russia speaks Urdu and sits
to the immediate left of the person belongs to Spain who
faces north. STEP IX: Immediate neighbours of Kumud faces

As per these statements, the arrangement will look like opposite direction to that of Kumud. Person speaks Hindi

this: sits second to the left of the person speaks Russian.

CASE I: As per these statements, the arrangement will look like


this:
CASE I:

CASE II (A):

STEP X: Mihir neither speaks Chinese nor Arabic and he


faces south. Person who speaks Arabic faces the same as

STEP VIII: Person who sits at the middle of the row the person who speaks Spanish who faces opposite to that

belongs to either France or China. Person who belongs to of the person who speaks Urdu.

Japan faces north and is an immediate neighbour of the As per these statements, the final arrangement will look

persons belong to India and UK. like this:


CASE I:

Page 169 of 1334


Get a Special Paid Bundle PDF on High Level Puzzles & Seating for All Bank Clerk/ PO Mains
If there are any suggestions/ errors in our PDFs Feel Free to contact us via this email: admin@exampundit.in
Ultra Practice Bundle PDF
Reasoning Questions for SBI Clerk/ RBI Assist. Mains
As per these statements, CASE I and CASE II will further
get spit into one more case and the arrangement will look
like this:
CASE I:
Answers (16-20):
16) C
17) A
18) B
19) E
CASE I (A):
20) D
Solutions (16-20):
STEP I: Sahil and Ziva are an immediate neighbours of
Jacky who likes White and eats Butterscotch. Sahil sits at
the middle of the row and he faces north.
As per these statements, there will be two cases getting CASE II:

framed and the arrangement will look like this:


CASE I:

CASE II (A):

CASE II:

STEP III: Alia sits third to the left of Mona who likes
Almond and doesn't sit at the end of the row. Person who
STEP II: Two persons sit between Manju and Jacky. sits at the left end of the row likes Blue colour and eats
Manju likes Yellow and faces the same direction as Sahil Vanilla ice cream.
faces.
Page 170 of 1334
Get a Special Paid Bundle PDF on High Level Puzzles & Seating for All Bank Clerk/ PO Mains
If there are any suggestions/ errors in our PDFs Feel Free to contact us via this email: admin@exampundit.in
Ultra Practice Bundle PDF
Reasoning Questions for SBI Clerk/ RBI Assist. Mains
As per this statement, CASE II will get eliminated and we As per these statements, CASE II (A) will get eliminated
will continue with CASE I, CASE I (A) and CASE II (A) and we will continue with CASE I and CASE I (A) and
and the arrangement will look like this: the arrangement will look like this:
CASE I: CASE I:

CASE I (A): CASE I (A):

CASE II: CASE II (A):

CASE II (A): STEP V: Tina likes either Orange or Grey but neither
likes Apple nor Pasta. Person who likes Pink also likes
Blueberry and sits fourth to the left of the person who likes
Red.
As per these statements, CASE I (A) will get eliminated
STEP IV: Abhay and Nitin who likes Red are an and we will continue with CASE I and the arrangement
immediate neighbours and both are facing opposite will look like this:
direction to each other. Abhay neither likes Black nor Pink CASE I:
and eats Apple ice-cream. Persons like Black and Red
colour are an immediate neighbours of each other.

Page 171 of 1334


Get a Special Paid Bundle PDF on High Level Puzzles & Seating for All Bank Clerk/ PO Mains
If there are any suggestions/ errors in our PDFs Feel Free to contact us via this email: admin@exampundit.in
Ultra Practice Bundle PDF
Reasoning Questions for SBI Clerk/ RBI Assist. Mains
CASE I:

CASE I (A):
Answers (21-25):
21) C
22) D
23) E
24) B
STEP VI: Not more than two persons facing the same 25) C
direction are adjacent to each other. Abhay neither likes Solutions (21-25):
Green nor Orange. Ziva faces the same direction as the STEP I: Not more than two persons are facing the same
person who likes Yellow who is an immediate neighbour direction are adjacent to each other. Julia and Kajal are
of the person eats Chocolate ice-cream. sitting at one of the ends of the row and both are facing
As per these statements, the arrangement will look like north.
this: As per these statements, there will be two possible cases
CASE I: and the arrangement will look like this:
CASE I:

CASE II:
STEP VII: Person eats Vanilla ice-cream sits second to
the right of the person who eats Pineapple ice-cream and
faces opposite to that person who likes Mango ice-cream
who sits at the end of the row. STEP II: Mishi sits three places away from Julia and she
As per this statement, the final arrangement will look like likes English.
this: As per this statement, the arrangement will look like this:

Page 172 of 1334


Get a Special Paid Bundle PDF on High Level Puzzles & Seating for All Bank Clerk/ PO Mains
If there are any suggestions/ errors in our PDFs Feel Free to contact us via this email: admin@exampundit.in
Ultra Practice Bundle PDF
Reasoning Questions for SBI Clerk/ RBI Assist. Mains
CASE I:

STEP V: Only one person sits between Angel and Mishi


CASE II:
who got 65 marks and faces north.
As per this statement, CASE II will get split into one more
case and the arrangement will look like this:
CASE I:
STEP III: Only one person sits between Kajal and Rashid
who faces opposite to that of Kajal and likes Chemistry.
As per this statement, the arrangement will look like this:
CASE I:
CASE II:

CASE II:

CASE II (A):

STEP IV: Rishi sits to the immediate right of Rashid and


got 88 marks in History.
STEP VI: Two persons sit between Angel and Jaggu who
As per this statement, the arrangement will look like this:
got 70 marks in his favourite subject. Angel got 84 marks
CASE I:
in Hindi and faces south.
As per these statements, the arrangement will look like
this:
CASE I:

CASE II:

Page 173 of 1334


Get a Special Paid Bundle PDF on High Level Puzzles & Seating for All Bank Clerk/ PO Mains
If there are any suggestions/ errors in our PDFs Feel Free to contact us via this email: admin@exampundit.in
Ultra Practice Bundle PDF
Reasoning Questions for SBI Clerk/ RBI Assist. Mains
CASE II (A):

CASE II:
STEP VIII: Person who sits at the middle of the row likes
Biology and faces the same direction as the person who
likes Computer, who got highest marks.
As per this statement, the arrangement will look like this:
CASE II (A):
CASE I:

STEP VII: Persons who got 84 marks and 90 marks are


CASE II:
an immediate neighbours of each other and also both are
facing the same direction. Person who likes Chemistry got
more marks than Mishi but less than Julia. Julia doesn't
get the highest marks.
As per these statements, the arrangement will look like CASE II (A):
this:
CASE I:

STEP IX: Person who likes Maths sits to the right of the
person who likes Physics.
As per this statement, CASE II and CASE II (A) will get
CASE II:
eliminated and we will continue with CASE I and the
final arrangement will look like this:
CASE I:

Page 174 of 1334


Get a Special Paid Bundle PDF on High Level Puzzles & Seating for All Bank Clerk/ PO Mains
If there are any suggestions/ errors in our PDFs Feel Free to contact us via this email: admin@exampundit.in
Ultra Practice Bundle PDF
Reasoning Questions for SBI Clerk/ RBI Assist. Mains

CASE II:
CASE II:

CASE II (A):
CASE II (A):

STEP II: Person who got admission in MIT sits at the end
of the row. Neither Azhar nor Nitesh got admitted in MIT.
Answers (26-30):
As per these statements, the arrangement will look like
26) C this:
27) B
CASE I:
28) B
29) A
30) B
Solutions (26-30):
CASE I (A):
STEP I: Azhar and Nitesh are an immediate neighbour of
each other and facing opposite direction to each other.
Either Azhar or Nitesh sits at one of the ends of the row.
As per these statements, there will be four cases getting
CASE II:
framed and the arrangement will look like this:
CASE I:

CASE II (A):

CASE I (A):
Page 175 of 1334
Get a Special Paid Bundle PDF on High Level Puzzles & Seating for All Bank Clerk/ PO Mains
If there are any suggestions/ errors in our PDFs Feel Free to contact us via this email: admin@exampundit.in
Ultra Practice Bundle PDF
Reasoning Questions for SBI Clerk/ RBI Assist. Mains
STEP IV: Neeta sits exactly between Neetu and Niti but
not immediate and faces the same direction as Azhar and
got admission in Harvard on Thursday.
STEP III: Nitesh who got admission in SRM and Neetu As per this statement, the arrangement will look like this:
are getting admission on the same day. Two persons sit CASE I (A):
between Niti who got admission on Monday and Nitesh
who faces south.
As per these statements, CASE I and CASE II (A) will
get eliminated and then we will continue with CASE I (A)
CASE II:
and CASE II and the arrangement will look like this:
CASE I:

STEP V: Shivam who got admission on Tuesday sits third


CASE I (A): to the right of the person got admitted in Harvard and both
are facing the same direction.
As per this statement, the arrangement will look like this:
CASE I (A):

CASE II:

CASE II:

CASE II (A):

Page 176 of 1334


Get a Special Paid Bundle PDF on High Level Puzzles & Seating for All Bank Clerk/ PO Mains
If there are any suggestions/ errors in our PDFs Feel Free to contact us via this email: admin@exampundit.in
Ultra Practice Bundle PDF
Reasoning Questions for SBI Clerk/ RBI Assist. Mains
STEP VI: No one get admission on the same day on As per this statement, the arrangement will look like this:
which Neeta and Shivam who get admission in Delhi CASE I (A):
University (DU) got.
As per this statement, the arrangement will look like this:
CASE I (A):

CASE II:

CASE II:

STEP IX: Agrim got admission either in Duke or in


Oxford on Saturday alone and faces south and he is an
immediate neighbour of Shivam. Niti faces the same
STEP VII: Pinku got admission in Amity on Sunday and
direction as Agrim faces who sits to the left of Niti. Raju
sits second to the left of Shivam.
who got admission on Monday faces south and he neither
As per this statement, the arrangement will look like this:
admitted in Oxford nor in MIT.
CASE I (A):
As per this statement, CASE II will get eliminated and we
will continue with CASE I (A) and the arrangement will
look like this:
CASE I (A):
CASE II:

CASE II:
STEP VIII: Niti got admission neither on MNNIT nor on
Oxford. Person who got admission in Oxford and MNNIT
are not sit at the end of the row.

Page 177 of 1334


Get a Special Paid Bundle PDF on High Level Puzzles & Seating for All Bank Clerk/ PO Mains
If there are any suggestions/ errors in our PDFs Feel Free to contact us via this email: admin@exampundit.in
Ultra Practice Bundle PDF
Reasoning Questions for SBI Clerk/ RBI Assist. Mains
Azhar (North) Duke Friday

Answers (31-35):
31) E
32) A
STEP X: Neetu got admission neither on Sunday nor on
33) C
Friday. Person who got admission in Duke doesn't share
34) B
his admission day with anyone.
35) D
As per this statement, the final arrangement will look like
Solutions (31-35):
this:
STEP I: Not more than two persons facing the same
CASE I (A):
direction are adjacent to each other. Saanu who studied in
BHU sits at the middle of the row and faces north.
As per this statement, the arrangement will look like this:

Name of Person College Name Day of Admission

Neetu (North) MIT Wednesday


STEP II: Pulkit who has his exam on May sits two places
Raju (South) MNIT Monday away from Saanu and faces south.
As per this statement, there will be two possible cases and
Neeta (North) Harvard Thursday
the arrangement will look like this:
Pinku (North) Amity Sunday CASE I:

Niti (South) Cornell Monday

Shivam (North) Delhi University Tuesday


(DU)
CASE II:
Agrim (South) Oxford Saturday

Nitesh (South) SRM Wednesday

Page 178 of 1334


Get a Special Paid Bundle PDF on High Level Puzzles & Seating for All Bank Clerk/ PO Mains
If there are any suggestions/ errors in our PDFs Feel Free to contact us via this email: admin@exampundit.in
Ultra Practice Bundle PDF
Reasoning Questions for SBI Clerk/ RBI Assist. Mains
STEP III: Person who sits at the right end of the row
studied in Bombay and faces north. Pulkit neither studied
in Madras nor in Indore.
As per these statements, the arrangement will look like
CASE II:
this:
CASE I:

CASE II (A):

CASE II:

STEP V: Neither Rohit who faces north nor Meneka who


has her exam in the month having 30 days in Roorkee sits
STEP IV: Two persons sit between the person who at the end of the row but both are sitting to the immediate
studied in Ropar and has his exam in June and Saanu who right of each other. Meneka is not an immediate neighbour
has her exam in July. of Saanu.
As per this statement, CASE I and CASE II will further
As per these statements, the arrangement will look like
get spilt into one more case each and the arrangement will this:
look like this: CASE I:
CASE I:

CASE I (A):

CASE I (A):

CASE II:
Page 179 of 1334
Get a Special Paid Bundle PDF on High Level Puzzles & Seating for All Bank Clerk/ PO Mains
If there are any suggestions/ errors in our PDFs Feel Free to contact us via this email: admin@exampundit.in
Ultra Practice Bundle PDF
Reasoning Questions for SBI Clerk/ RBI Assist. Mains
CASE II (A):

CASE II (A):
STEP VII: Bhavesh doesn't sit at the end of the row and
has studied in Goa also he has his exam after the exam of
Nancy whose exam is held just after the exam of Saanu.
As per this statement, CASE I (A) will get eliminated and
STEP VI: Gargi who faces north sits to the immediate
we will continue with CASE I and the arrangement will
right of Pulkit. Nancy is an immediate neighbour of both
look like this:
Meneka and Ishu who studied in Bombay and has her
CASE I:
exam in February.
As per these statements, CASE II and CASE II (A) will
get eliminated and we will continue with CASE I and
CASE I (A) and the arrangement will look like this:
CASE I: CASE I (A):

CASE I (A): STEP VIII: Kavi who faces opposite direction to that of
Ishu is studied either in Delhi or in Indore and has his
exam in March. Person studied in Indore is not an
immediate neighbour of the persons who studied in Ropar
or BHU.
CASE II: As per these statements, the final arrangement will look
like this:

Page 180 of 1334


Get a Special Paid Bundle PDF on High Level Puzzles & Seating for All Bank Clerk/ PO Mains
If there are any suggestions/ errors in our PDFs Feel Free to contact us via this email: admin@exampundit.in
Ultra Practice Bundle PDF
Reasoning Questions for SBI Clerk/ RBI Assist. Mains

STEP II: Chinky who sits at the left end of the row has
Answers (36-40):
her presentation on Radar and faces opposite direction to
36) B
that of Justin.
37) C
As per this statement, the arrangement will look like this:
38) A
CASE I:
39) D
40) B
Solutions (36-40):
STEP I: Faiza who faces south has her presentation on 5.
Less than four person sits to the left of Faiza. CASE II:
As per these statements, there will be four possible cases
and the arrangement will look like this:
CASE I:

CASE III:

CASE II:

CASE IV:

CASE III:

STEP III: Priya who sits second to the right of Chinky


has presentation on either Nanotech or Pollution.
CASE IV: As per this statement, the arrangement will look like this:
CASE I:

Page 181 of 1334


Get a Special Paid Bundle PDF on High Level Puzzles & Seating for All Bank Clerk/ PO Mains
If there are any suggestions/ errors in our PDFs Feel Free to contact us via this email: admin@exampundit.in
Ultra Practice Bundle PDF
Reasoning Questions for SBI Clerk/ RBI Assist. Mains
CASE III:

CASE II:

CASE IV:

CASE III:

STEP V: Dipti who has presentation on 5G vs 4G sits


second to the left of Tarun who has his presentation on 12.
Justin who sits second to the right of Tarun who has
CASE IV: presentation on E-marketing and faces north.
As per these statement, CASE II and CASE IV will get
eliminated and we will continue with CASE I and CASE
III and the arrangement will look like this:
STEP IV: Zubin who has presentation on Sales on 28 is CASE I:
an immediate neighbour of Priya but not an immediate
neighbour of Chinky.
As per this statement, the arrangement will look like this:
CASE I:
CASE II:

CASE II:
CASE III:

Page 182 of 1334


Get a Special Paid Bundle PDF on High Level Puzzles & Seating for All Bank Clerk/ PO Mains
If there are any suggestions/ errors in our PDFs Feel Free to contact us via this email: admin@exampundit.in
Ultra Practice Bundle PDF
Reasoning Questions for SBI Clerk/ RBI Assist. Mains

CASE IV: CASE III:

STEP VI: Justin has his presentation on prime numbered


STEP VIII: Person who has presentation on E-cars on 16
date but not on 19.
sits second to the left of the person who has presentation
As per this statement, the arrangement will look like this:
on 28. Tanuj has presentation neither on 16 nor on 8 and
CASE I:
faces south.
As per this statement, the arrangement will look like this:
CASE I:

CASE III:

CASE III:

STEP VII: Person who has presentation on Bitcoin and


the person who has presentation on Biochips are an
immediate neighbour of each other. Neither Dipti nor
STEP IX: Rahul who faces north is not an immediate
Priya has presentation on Bitcoin.
neighbour of both Tarun and Tanuj. Person who has
As per these statements, the arrangement will look like
presentation on 19 sits third to the right of the person who
this:
has presentation on Bitcoin who sits second to the left of
CASE I:
the person who has presentation on Pollution.

Page 183 of 1334


Get a Special Paid Bundle PDF on High Level Puzzles & Seating for All Bank Clerk/ PO Mains
If there are any suggestions/ errors in our PDFs Feel Free to contact us via this email: admin@exampundit.in
Ultra Practice Bundle PDF
Reasoning Questions for SBI Clerk/ RBI Assist. Mains
As per these statements, CASE III will get eliminated and 44) C
we will continue with CASE I and the arrangement will 45) E
look like this: Solutions (41-45):
CASE I: STEP I: Surbhi who faces south sits third from the right
end of the row and uses Sity network. Mahima who likes
Armaan is an immediate neighbour of Surbhi and faces
opposite direction to that of Surbhi.
As per these statements, there will be two possible cases
CASE III:
and the arrangement will look like this:
CASE I:

STEP X: Person who has presentation on a square of a


number sits second to the right of the person who has CASE II:
presentation on cube of a number. Justin is not an
immediate neighbour of Faiza but faces same direction as
Faiza and Dipti faces.
As per this statement, the final arrangement will look like
this: STEP II: Richa sits three places away from Surbhi and
CASE I: likes Mohit and uses BSNL Only one person sits between
Richa and Sweety who faces north and likes either Alka
or Shreya.
As per these statements, the arrangement will look like
this:
Answers (41-45): CASE I:
41) B
42) D
43) A

Page 184 of 1334


Get a Special Paid Bundle PDF on High Level Puzzles & Seating for All Bank Clerk/ PO Mains
If there are any suggestions/ errors in our PDFs Feel Free to contact us via this email: admin@exampundit.in
Ultra Practice Bundle PDF
Reasoning Questions for SBI Clerk/ RBI Assist. Mains
nor Alka and faces north which is opposite to that of
Richa.
As per these statements, the arrangement will look like
this:
CASE II: CASE I:

STEP III: Nimmi who likes Arjit and uses neither Jio nor CASE II:
Airtel sits at the left end of the row and faces the same
direction as Sweety faces. Preeti sits second to the right of
Nimmi and uses Vodafone and both are facing opposite
direction to each other.
As per these statements, the arrangement will look like STEP V: Pari who faces south likes Alka is an immediate

this: neighbour of the person who likes Armaan. Astha likes

CASE I: Hathway and sits second to the left of the person who uses
Aircel and both are facing opposite direction to each other.
Pari uses neither Airtel nor Jio.
As per these statements, CASE II will get eliminated and
we will continue with CASE I and the arrangement will
CASE II: look like this:
CASE I:

STEP IV: Prama sits exactly between the person who


uses Vodafone and the person who likes Neha who doesn't CASE II:

sit at the end of the row. Prama neither likes Mahadevan

Page 185 of 1334


Get a Special Paid Bundle PDF on High Level Puzzles & Seating for All Bank Clerk/ PO Mains
If there are any suggestions/ errors in our PDFs Feel Free to contact us via this email: admin@exampundit.in
Ultra Practice Bundle PDF
Reasoning Questions for SBI Clerk/ RBI Assist. Mains
48) D
49) B
50) C
Solutions (41-50):
STEP VI: Neither Mahima nor Sweety uses Uninor.
STEP I: Vinay sits third from the left end and Abhi sits
Neither Astha nor Pari likes Shreya. Person who likes
third from the right end of the row and both faces opposite
Sunidhi is not an immediate neighbour of the persons who
direction to each other.
uses Vodafone and Sity. Preeti does not like Neha. Person
As per these statements, there will be two possible cases
who uses Uninor sits third to the left of the person who
and the arrangement will look like this:
likes Mahadevan.
CASE I:
As per these statements, the arrangement will look like
this:
CASE I:

CASE II:

STEP VII: Person who uses Airtel and the person who
uses Idea are an immediate neighbour of each other and
STEP II: Jackal who uses Microsoft sits second to the
facing the same direction.
right of Abhi and faces north. Dheeraj who sits three
As per this statement, the final arrangement will look like
places away from Vijay likes Snickers and faces the same
this:
direction as Jackal faces.
CASE I:
As per these statements, the arrangement will look like
this:
CASE I:

Answers (46-50):
46) E
47) B
Page 186 of 1334
Get a Special Paid Bundle PDF on High Level Puzzles & Seating for All Bank Clerk/ PO Mains
If there are any suggestions/ errors in our PDFs Feel Free to contact us via this email: admin@exampundit.in
Ultra Practice Bundle PDF
Reasoning Questions for SBI Clerk/ RBI Assist. Mains
CASE II: CASE I:

STEP III: Neither Vinay nor Varun uses Asus. Tapan


CASE II:
who is an immediate neighbour of Vijay likes Munch and
uses Lenevo. Nimo sits second to the left of Tapan and
uses Dell. Nimo faces opposite direction to that of Tapan.
As per these statements, CASE II will further get spilt into
one more case and the arrangement will look like this: CASE II (A):
CASE I:

STEP V: Either Vinay or Abhi likes Dairy Milk. Person


who likes Bournville sits exactly between the person who
CASE II:
likes Fuse and the person who likes Dairy Milk.
As per these statements, CASE I will further get spilt into
one more case and the arrangement will look like this:
CASE I:

CASE II (A):

CASE I (A):
STEP IV: Number of persons sits to the left of Pranay is
equal to the number of persons sits to the right of Nimo
who likes Perk.
As per this statement, the arrangement will look like this:

Page 187 of 1334


Get a Special Paid Bundle PDF on High Level Puzzles & Seating for All Bank Clerk/ PO Mains
If there are any suggestions/ errors in our PDFs Feel Free to contact us via this email: admin@exampundit.in
Ultra Practice Bundle PDF
Reasoning Questions for SBI Clerk/ RBI Assist. Mains
CASE II:

CASE II (A):
CASE II (A):

STEP VII: Person who likes Fuse is an immediate


STEP VI: Varun who faces south uses Acer and likes neighbour of the persons who uses HP and the person who
either Kitkat or Bournville but doesn't sit at the end of the uses Apple brand.
row. Neither Pranay nor Jackal likes Bournville. As per this statement, CASE II (A) will get eliminated and
As per these statements, CASE I and CASE I (A) will we will continue with CASE II and the arrangement will
get eliminated and we will continue with CASE II and look like this:
CASE II (A) and the arrangement will look like this: CASE II:
CASE I:

CASE II (A):

CASE I (A):

STEP VIII: Person who uses MSI and the person who
uses Apple are immediate neighbours of each other.
CASE II: Pranay uses neither Asus nor HP.

Page 188 of 1334


Get a Special Paid Bundle PDF on High Level Puzzles & Seating for All Bank Clerk/ PO Mains
If there are any suggestions/ errors in our PDFs Feel Free to contact us via this email: admin@exampundit.in
Ultra Practice Bundle PDF
Reasoning Questions for SBI Clerk/ RBI Assist. Mains
As per these statements, the arrangement will look like faces opposite direction to each other. Aashu doesn't faces
this: south and likes 5-star.
CASE II: As per these statements, the final arrangement will look
like this:

STEP IX: Person who likes Milky Bar sits to the right of
the person who uses Razer who is not Pranay and both

Download Seating Arrangement Practice Questions PDF


Get More Reasoning Practice Questions PDF

Seating Circle 3 Variable


Directions (1-5): Answer the questions based on the but not necessarily in the same order. Not more than two
information given below: friends buy from same malls and no two friends who
There are eight friends M, N, O, P, Q, R, S and T sitting shops from same mall are sitting together.
around a circular table and all are facing towards the R purchase shoes but not from GIP. Q shop from GIP.
centre. They all purchased different items i.e. shoes, belt, There are two friends sitting between those who are
wallet, jeans, trousers, shirts, cap and tie from four shopping from Vishalmart. N sits second to the right of P,
different malls i.e. vishalmart, indiamart, citylife and GIP who purchases jeans. Two friends who purchases from

Page 189 of 1334


Get a Special Paid Bundle PDF on High Level Puzzles & Seating for All Bank Clerk/ PO Mains
If there are any suggestions/ errors in our PDFs Feel Free to contact us via this email: admin@exampundit.in
Ultra Practice Bundle PDF
Reasoning Questions for SBI Clerk/ RBI Assist. Mains
Indiamart sits opposite to each other. R is an immediate (a) T
neighbor of N who purchases belt. The one who purchases (b) M
shirts is an immediate neighbor of those who purchases (c) N
from GIP. O purchases tie from Citylife. The one who (d) Q
shop from GIP sits second to the right of R. P does not (e) R
shop from Indiamart. T who purchases cap sits on the 8. R buys from which shopping mall?
immediate right of O. S shop from Citylife and he does (a) Indiamart
not purchase trouser. (b) GIP
4. What is the position of N with respect to the one (c) Citylife
who buys shoes? (d) Vishalmart
(a) Third to right (e) None of these
(b) Second to left Directions (6-10): Answer the questions based on the
(c) Third to left information given below:
(d) Immediate left There are eight friends J, K, L, M, N, O, P and Q are sitting
(e) Cannot be determined around a circular tableand all friends are facing to the
5. What is the position of S with respect to Q? centre, but not necessarily in the same order. They are
(a) Second to right working in different Companies i.e.Google, Walmart,
(b) Immediate left Samsung, L&T, Amazon, Reliance, Dell and Intel but not
(c) Third to right necessarily in the same order. Each person has salary
(d) Second to left account in different banks i.e. PNB, SBI, UBI, Axis, Hdfc,
(e) Immediate right BOM, OBC, and BOB.
6. Who buys cap from GIP? There is three person sit between K and the one who is
(a) S working in Reliance. Lhas saving account in Hdfc. There
(b) T is three person sit between OandN, whose salary account
(c) M in PNB. Only one person sit between K and J. N is
(d) N working in Google. The one who has account in BOM is
(e) R working in Intel.There are only two persons sit between
7. Who is third to the left of the one who buys Jand P. There is only one person sit between J and Q. O
wallet? sits at a gap of two seats to K. The one who has account
Page 190 of 1334
Get a Special Paid Bundle PDF on High Level Puzzles & Seating for All Bank Clerk/ PO Mains
If there are any suggestions/ errors in our PDFs Feel Free to contact us via this email: admin@exampundit.in
Ultra Practice Bundle PDF
Reasoning Questions for SBI Clerk/ RBI Assist. Mains
in BOB sits second to the right of K. M is not an (d) L
immediate neighbour of the person who is third to the left (e) Cannot be determined
of J. 12. In certain way O is related to K, Q is related to
The one whose account in BOM sits second to the right of M and in same way P is related to ______ of
L. The one who is working in Dell sits immediate left of thefollowing?
J. There are two persons sit between whose account in (a) L
Axis and M. The one who is working in L&T sits opposite (b)M
to N and has account in SBI. The one whose account in (c)J
OBC sits opposite to one whose account in UBI. The one (d) Q
who is working in Dell has not account in OBC. The one (e) O
who is working in Samsung sits immediate right to the one 13. Q sits opposite to who works in which of the
who is working in Walmart. Q is sitting next to P. following companies?
9. What is the position of K with respect to O? (a) Google
(a) Third to right (b)Samsung
(b) Second to left (c) Intel
(c) Fourth to right (d)Walmart
(d) Third to left (e) Reliance
(e) Cannot be determined Directions (11-15): Answer the questions based on the
10. What is the position of M with respect to who the information given below:
one works in Amazon? There are eight friends A, B, C, D, E, F, G and H sitting
(a) Second to right around a circular table and Four of them are facing the
(b) Immediate left centre and other four are facing away from the centre. All
(c) Third to right of them attend a meeting on different dates from 1st to 8th
(d) Second to left of a month. Each of them use different vehicles to attend
(e) Immediate right the meeting i.e. Train, Car, Truck, Auto, Cycle, Boat,
11. Who sits opposite to whose account in UBI? Scooter and Motorcyclebut not necessarily in the same
(a) P order.
(b) Q E who will attend on 3rd ,is sitting third to the right of C.B
(c) N uses Car and sitting adjacent to F who will attend on 7th.
Page 191 of 1334
Get a Special Paid Bundle PDF on High Level Puzzles & Seating for All Bank Clerk/ PO Mains
If there are any suggestions/ errors in our PDFs Feel Free to contact us via this email: admin@exampundit.in
Ultra Practice Bundle PDF
Reasoning Questions for SBI Clerk/ RBI Assist. Mains
Both B and F are facing towards the centre. H faces the (d) Scooter
same direction as C and he is not the neighbour of G.C is (e) None of these
not will attend on 1st and he uses Truck. G will attend on 14. Who is sitting second to the right of the one who
4th.The one who will attend on 1st uses Auto. The one who attend meeting on 4th?
will attend on 6th and uses Cycle is sitting to the immediate (a) G
left of the person who uses Boat. D who is facing towards (b) F
the centre is sitting second to the left of G. C is sitting (c) E
adjacent to both G and D and F is sitting fourth to the right (d) H
of C.G is facing outwards and uses Train.The one who (e) None of these
will attend on 2nd is sitting second to the right of D who 15. How many persons sit between the C and the one
will attend on 8th and D uses Scooter. who used Motorcycle, when counted from right of C?
11. Who sits to the immediate left of the one who used (a) One
Motorcycle? (b) Four
(a) E (c)Three
(b) B (d) Six
(c) D (e) None of these
(d) H Directions (16-20): Answer the questions based on the
(e) None of these information given below: (1 statement added)
12. Who sits second to the left of the one who attend There are eight friends A, B, C, D, E, F, G and H sitting
the meeting on 6th? around a circular table and four of them are facing the
(a) D centre and other four are facing away from the centre.
(b) H Each of them has given different subjects test i.e.
(c) E Reasoning, politics, Economics, Quant, GA,
(d) F English,Geography and History and all of them obtained
(e) None of these different marks i.e. 35, 36, 38, 40, 42, 45, 48 and 49 but
13. D is sitting third to the left of the one who used? not necessarily in the same order.
(a) Motor Cycle The adjacent members do not face the same direction. The
(b) Truck person who obtained 40 marks gives Reasoning test and is
(c) Car sitting third to the right of C. The person who gives
Page 192 of 1334
Get a Special Paid Bundle PDF on High Level Puzzles & Seating for All Bank Clerk/ PO Mains
If there are any suggestions/ errors in our PDFs Feel Free to contact us via this email: admin@exampundit.in
Ultra Practice Bundle PDF
Reasoning Questions for SBI Clerk/ RBI Assist. Mains
Politics test obtained 42 marks and sits opposite A, who 23. In certain way F is related to C, A is related to D
obtained 40 marks. B and C are the neighbours of H. B and in same way E is related to ______ of
obtained 35 marks and is facing towards the center. The thefollowing??
persons who gives Quant and GA are sitting opposite to (a) A
each other. D who obtained 36 marks gives Economics (b) G
test and he is an immediate neighbour of A. F who gives (c) H
GA test does not sit adjacent to both B and E and he does (d) B
not obtained highest mark. The person who gives Quant (e) E
test obtained 45 marks.The person who obtained 38 marks 24. Who sits second to the right of the one who
is sitting opposite to the person who gives English test. E obtained 36 marks?
who obtained marks only more than two friend is sitting (a) F
second to the left of B and gives History test. The one who (b) A
has given English test obtained more marks than the one, (c) H
who has given GA test. (d) E
21. What is the position of G with respect to the one, (e) B
who obtained40 marks? 25. Who sits opposite to the one who given Economics
(a) Second to left test?
(b) Third to left (a) F
(c) Third to right (b) C
(d) Fourth to right (c) B
(e) Cannot be determined (d) A
22. What is the position of H with respect to the one (e) G
who given Geography test? Directions (21-25): Answer the questions based on the
(a) Second to right information given below:
(b) Immediate left There are six friends A, B, C, D, E and F are sitting around
(c) Third to right a circular table and all are facing towards the centre. They
(d) Second to left have purchased different brand’s sanitizer i.e. Himalya,
(e) Immediate right Lifebuoy, Jaiwik, Ozone, Mediker and Dabur. Each

Page 193 of 1334


Get a Special Paid Bundle PDF on High Level Puzzles & Seating for All Bank Clerk/ PO Mains
If there are any suggestions/ errors in our PDFs Feel Free to contact us via this email: admin@exampundit.in
Ultra Practice Bundle PDF
Reasoning Questions for SBI Clerk/ RBI Assist. Mains
sanitizer are of different price (Prices are in integer value) (c) C
but not necessarily in the same order. (d) F
B purchased sanitizer at the second highest price but did (e) None of these
not purchased Ozone, B sits second to the left of the 23. Who is sitting third to the left of the one who
person who purchased at Rs 74. D sits opposite to who purchased Himalya?
purchased ozone. D does not sit immediate to B. E did not (a) A
purchased at lowest price. C sits immediate left of the (b) F
person who purchased Dabur and C purchased at Rs 70. (c) B
The person who purchased Lifebuoy sits second to the left (d) D
of who purchased at Rs 72. The person who purchased (e) None of these
Himalya did not purchased at highest price. One of the 24. Which among the following pair doesn’t belong to
friends purchased sanitizer at that price which is prime the group??
number. Lowest price of sanitizer among six is Rs 64. C (a) CA
is second to the right of the person who purchased (b)FB
Mediker whose price is Rs 69 which is third lowest price (c) ED
among six sanitizer. A sits immediate left to the person (d) EB
who sits opposite to who purchased Dabur, who does not (e) None of these
next to F. 25. Who sits opposite to the one who purchased
21. Who is to the immediate left of the one who Lifebuoy?
purchased Jaiwik? (a) B
(a) B (b) E
(b) C (c)F
(c) D (d) D
(d) F (e) None of these
(e) E Directions (26-30): Answer the questions based on the
22. Who sits second to the left of the one who information given below: (editions done)
purchased at Rs 67? There are eight friends A, B, C, D, E, F, G and H sitting
(a) B around a circular table and Four of them are facing the
(b) A centre and other four are facing away from the centre.
Page 194 of 1334
Get a Special Paid Bundle PDF on High Level Puzzles & Seating for All Bank Clerk/ PO Mains
If there are any suggestions/ errors in our PDFs Feel Free to contact us via this email: admin@exampundit.in
Ultra Practice Bundle PDF
Reasoning Questions for SBI Clerk/ RBI Assist. Mains
They works at different places i.e. Noida, Saket, (c) Third to left
Gurugram, Mayur vihar, Ashok nagar, Laxmi nagar, GTB (d) Fourth to right
nagar and Rajiv chowk. They works in eight different (e) Cannot be determined
banks i.e. SBI, BOB, BOI, RBI, OBC, UBI, CBI and 27.What is the position of C with respect to the one
BOM but not necessarily in the same order. who from Saket?
G faces the centre and sits third to the right of D, who (a) Second to right
works in BOM. E work at Gurugram and sits opposite to (b) Third to right
the person who works at Laxmi nagar. A, who works in (c) Immediate left
BOB but not at Rajiv chowk and sits third to the right of (d) Second to left
C, who works at Noida. The person who works at Ashok (e) Immediate right
nagar and Mayur vihar are facing the same direction. The 28. Who sits third to the right of the one, who is from
one who work in BOB is an immediate neighbor of who GTB nagar?
works in SBI and BOI. F is sitting adjacent to the person (a) E
who works at Gurugram and Rajiv chowk. The person (b) G
who works in SBI does not faces who works in UBI. B (c) B
works at Ashok nagar and D works at Saket. B is not an (d) H
immediate neighbor of who works in SBI. The person who (e) C
works at GTB nagar is facing outside and immediate 29.Who sits immediate right of the one who works in
neighbor of who works at Rajiv chowk. The person who BOI?
works in BOI faces the person who works in RBI and both (a) Who works in SBI
faces inside. H is immediate neighbor of the person who (b) Who works in BOB
works at Ashok nagar and GTB nagar. F who does not (c) Who works in ICICI
works at GTB nagar, but sits to the immediate left of E. (d) Who works in UBI
The person who works in RBI sits immediate left of who (e) Who works in RBI
works in OBC. 30. How many person sit between B and E from right
26. What is the position of D with respect to the one of E?
who works in SBI? (a) Six
(a) Third to right (b) Four
(b) Second to left (c) None
Page 195 of 1334
Get a Special Paid Bundle PDF on High Level Puzzles & Seating for All Bank Clerk/ PO Mains
If there are any suggestions/ errors in our PDFs Feel Free to contact us via this email: admin@exampundit.in
Ultra Practice Bundle PDF
Reasoning Questions for SBI Clerk/ RBI Assist. Mains
(d) Two (d) T
(e) Three (e) P
Directions (31-35): Answer the questions based on the 32. Who among the following belongs from Noida?
information given below: (a) S
There are seven persons P, Q, R, S, T, U and V are sitting (b) Q
around a circular table and all are facing towards the (c) R
centre. They likes different computer languages Python, (d) T
Java, C++, Ruby, PHP, Perl and TCL. They belongs (e) P
different cities Noida, Bhopal, Gurugram, Ranchi, Indore, 33. Who sits third to the left of the one who likes
New delhi and Patna but not necessarily in the same order. Python ?
S sits immediate left of the person who likes PHP. The (a) R
person from Indore, does not like Python. T is an (b) U
immediate neighbor of the student who likes TCL. The (c) T
person from Noida likes C++. Q is fourth to the left of the (d) S
person who likes Java. T belongs from New delhi. U likes (e) Q
TCL. The person who likes PHP is third to the left of the 34. Which language likes V?
person who likes TCL. P does not likes Java. V is not an (a) Ruby
immediate neighbor of either Bhopal or Patna. The person (b) PHP
who likes Ruby is second to the right of P, who is an (c) Perl
immediate right of U. The person who likes Java is an (d) Python
immediate neighbor of the person who belongs to Patna. (e) Java
The person who likes Perlis an immediate neighbor of P. 35. How many persons sit between the one who likes
V is third to the right of the person who likes C++. The Java and Perl from left of the one, who likes Perl?
person from Ranchi sits second to the right of Q. (a) One
31. Who sits to the immediate right of the one who likes (b) Four
ruby? (c) Six
(a) V (d) Two
(b) Q (e) Three
(c) R
Page 196 of 1334
Get a Special Paid Bundle PDF on High Level Puzzles & Seating for All Bank Clerk/ PO Mains
If there are any suggestions/ errors in our PDFs Feel Free to contact us via this email: admin@exampundit.in
Ultra Practice Bundle PDF
Reasoning Questions for SBI Clerk/ RBI Assist. Mains
Directions (36-40): Answer the questions based on the group. Which is the one that doesn’t belong to that
information given below: group?
There are eight persons P, Q, R, S, T, U, V and W are (a) VQ
sitting around a circular table facing outside the centre. (b) TR
They belong to different countries i.e. USA, UK, France, (c) WR
Italy, India, Nepal, Japan and Russia but not necessarily (d) PU
in the same order. They also like three different sports (e) SW
Cricket, Football and Tennis. At least two persons like one 38. Who sits second to the right of R ?
sports. (a) Who belongs to Italy
Q does not sits opposite to P. P is from India and likes (b) Who belongs to France
Football and sits on the immediate right of T. U is from (c) Who belongs to UK
Italy and likes Tennis. W likes the same sports as V. V is (d) Who belongs to USA
the third to the left of U. Q sits opposite the person who is (e) Who belongs to Japan
from Russia. R sits exactly opposite the person who 39. Who is second to the left of the one who belongs
belongs to USA. The persons who belongs to Nepal and from Russia?
France likes football. V is from Russia. Q is from France. (a) Q
The persons who like Football are not immediate (b) U
neighbors of each other. The one who is from Japan does (c) W
not likes football or cricket. W is not from Nepal. Only S (d) T
and T Likes cricket and S sits third to the right of T. (e) S
36. Who sits to the immediate left of the one who 40. How many persons sit between U and who belongs
belongs to Italy? from France, when counted from right of U?
(a) R (a) One
(b) W (b) Four
(c) T (c) Six
(d) S (d) Two
(e) U (e) None
37. Four of the following five are alike in a certain way Directions (41-45): Answer the questions based on the
based on the given arrangement and hence form a information given below:
Page 197 of 1334
Get a Special Paid Bundle PDF on High Level Puzzles & Seating for All Bank Clerk/ PO Mains
If there are any suggestions/ errors in our PDFs Feel Free to contact us via this email: admin@exampundit.in
Ultra Practice Bundle PDF
Reasoning Questions for SBI Clerk/ RBI Assist. Mains
There are six persons A, B, C, D, E and F are sitting in a 42. What is the position of C with respect to the one,
circular table and all of them facing to the centre. Each who stay on 1st floor?
one of them belongs to different cities i.e. Vellore, Ranchi, (a) Immediate right
Indore, Kanpur, Agra and Nashik. Each of them also stay (b) Immediate left
in a hotel on different floor from one to six, Bottommost (c) Third to the right
floor no is One and topmost floor is six but not necessarily (d) Third to the left
in the same order. (e) None of these
F stay on first floor and he is sitting to the immediate left 43. What is E’s position with respect to the one who
of the person who stay on third floor. The person who stay belongs to Ranchi?
on sixth floor is sitting diagonally opposite to who belongs a) Third to the right
to Nashik. The one who belongs to Indore is sitting to the b) Second to the right
immediate left of the person who stay on fourth floor. The c) Immediate right
person who stay on first floor is sitting to the immediate d) Second to the left
right to one who belongs to Kanpur. D stays on fifth floor e) Third to the left
and he belongs to Indore. D is sitting diagonally opposite 44. Who is sitting third to the right of the one who stay
to the one who stay on second floor. A does not stay on on 5th floor?
third floor.The one who belongs to Vellore does not stay (a) A
on neither on fourth nor sixth floor. The one who belongs (b) F
to Ranchi does not stay on sixth floor. B does not belongs (c) B
to Kanpur and he does not stay on third floor. Only one (d) C
person is sitting between E and F, and that person stay on (e) E
second floor. 45. Which among the following statement is definitely
41. Who is sitting third to the right of the one who true?
belongs to Agra? a) A is to the immediate left of the one who stay on 3rd
(a) D floor.
(b) A b) B sits second to the right of the one who stay on 3rd
(c) F floor.
(d) C c) D sits second to right of the one who stay on 3rd
(e) None of these floor.
Page 198 of 1334
Get a Special Paid Bundle PDF on High Level Puzzles & Seating for All Bank Clerk/ PO Mains
If there are any suggestions/ errors in our PDFs Feel Free to contact us via this email: admin@exampundit.in
Ultra Practice Bundle PDF
Reasoning Questions for SBI Clerk/ RBI Assist. Mains
d) C sits fourth to the right of the one who stay on 3rd (b) B
floor. (c) D
e) None of these (d) A
(e) None of these
Directions (46-50): Answer the questions based on the 47. Who among the followings have Samsung mobile
information given below: phone?
There are six persons A, B, C, D, E and F are sitting (a) C
around a circular table facing the center and have different (b) E
professions i.e. Doctor, Teacher, Engineer, Architecture, (c) D
Singer, and Lawyer but not necessarily in the same order. (d) F
They have also different brand mobile phonesi.e. Oppo, (e) None of these
Nokia, Samsung, Sony, HTC and Redmi but not 48. Who is sitting third to the left of the one who is
necessarily in the same order. Engineer?
B who is not an immediate neighbour of D and sits second (a) C
to the left of the person who has Redmi mobile phone. The (b) B
one who isArchitecture sits opposite to D. The one who is (c) D
Doctor has not Redmi. C has Sony sits second to the right (d) F
of the person who is Singer. C sits to the immediate left of (e) None of these
the person who is Lawyer. The one who isTeacher sits 49. Who are immediate neighbors of the one who has
second to the left of the person who has HTC. The one Oppo mobile phone?
who is teacher sits immediate right to the one who is (a) C and F
Singer. F has not Nokia mobile.A sits to the immediate (b) C and D
left of the person who sits opposite to the person who (c) B and D
isLawyer. The person who isLawyer is not an immediate (d) B and A
neighbour of F. The one who is Teacher has Oppo. B has (e) None of these
HTC and he is not an Architecture. 50. Four of the following five are alike in a certain way
46. Who is to the immediate left of the one who has and hence form a group. Which of the following does
Nokia mobile phone? not belong to the group?
(a) C (a) DE
Page 199 of 1334
Get a Special Paid Bundle PDF on High Level Puzzles & Seating for All Bank Clerk/ PO Mains
If there are any suggestions/ errors in our PDFs Feel Free to contact us via this email: admin@exampundit.in
Ultra Practice Bundle PDF
Reasoning Questions for SBI Clerk/ RBI Assist. Mains
(b) CA (d) FB
(c) ED (e) FA

Seating Circle 3 Variable - Answers and Explantion


SOLUTION (1-5): 3. The one who purchases shirts is an immediate
Explanation in detail: neighbor of those who purchases from GIP.
1. N sits second to the right of P, who purchases 4. The one who shop from GIP sits second to the
jeans. right of R.
2. R is an immediate neighbor of N who purchases 5. R purchase shoes but not from GIP.
belt.

Page 200 of 1334


Get a Special Paid Bundle PDF on High Level Puzzles & Seating for All Bank Clerk/ PO Mains
If there are any suggestions/ errors in our PDFs Feel Free to contact us via this email: admin@exampundit.in
Ultra Practice Bundle PDF
Reasoning Questions for SBI Clerk/ RBI Assist. Mains
6. Two friends who purchases from Indiamart sits 10. S shop from Citylife and he does not purchase
opposite to each other. trouser.
7. O purchases tie from Citylife. 11. Q shop from GIP.
8. P does not shop from Indiamart. 12. There are two friends sitting between those who
9. T who purchases cap sits on the immediate right of are shopping from Vishalmart.
O. 13. O purchases tie from Citylife.
14. T who purchases cap sits on the immediate right of
O.

There is no such space for O in CASE1 so CASE1 is 1. d

cancelled out 2. a
3. b
4. b
Page 201 of 1334
Get a Special Paid Bundle PDF on High Level Puzzles & Seating for All Bank Clerk/ PO Mains
If there are any suggestions/ errors in our PDFs Feel Free to contact us via this email: admin@exampundit.in
Ultra Practice Bundle PDF
Reasoning Questions for SBI Clerk/ RBI Assist. Mains
5. D 6. O sits at a gap of two seats to K.
SOLUTION (6-10):
Explanation in detail:
1. There are only two persons sit between J and P.
2. There is only one person sit between J and Q.
3. Q is sitting next to P.

7. The one who has account in BOB sits second to


the right of K.
8. There is three person sit between K and the one
who is working in Reliance.
4. The one who is working in Dell sits immediate left
of J.
5. Only one person sit between K and J.

Page 202 of 1334


Get a Special Paid Bundle PDF on High Level Puzzles & Seating for All Bank Clerk/ PO Mains
If there are any suggestions/ errors in our PDFs Feel Free to contact us via this email: admin@exampundit.in
Ultra Practice Bundle PDF
Reasoning Questions for SBI Clerk/ RBI Assist. Mains

12. L has salary account in Hdfc.


So CASE2 is cancelled out.
9. There is three person sit between O and N, whose
13. The one who has account in BOM is working in
salary account in PNB.
Intel.
10. N is working in Google.
14. The one whose account in BOM sits second to the
11. M is not an immediate neighbour of the person
right of L.
who is third to the left of J.

Page 203 of 1334


Get a Special Paid Bundle PDF on High Level Puzzles & Seating for All Bank Clerk/ PO Mains
If there are any suggestions/ errors in our PDFs Feel Free to contact us via this email: admin@exampundit.in
Ultra Practice Bundle PDF
Reasoning Questions for SBI Clerk/ RBI Assist. Mains

15. There are two persons sit between whose account 6.d
in Axis and M. 7.b
16. The one who is working in L&T sits opposite to N 8.a
and has account in SBI. 9.c
17. The one whose account in OBC sits opposite to 10.c
one whose account in UBI. SOLUTION (11-15):
18. The one who is working in Dell has not account in Explanation in detail:
OBC. 1. D who is facing towards the centre is sitting
19. The one who is working in Samsung sits second to the left of G.
immediate right to the one who is working in 2. C is sitting adjacent to both G and D and F is sitting
Walmart. fourth to the right of C.
3. G is facing outwards and uses Train.

Page 204 of 1334


Get a Special Paid Bundle PDF on High Level Puzzles & Seating for All Bank Clerk/ PO Mains
If there are any suggestions/ errors in our PDFs Feel Free to contact us via this email: admin@exampundit.in
Ultra Practice Bundle PDF
Reasoning Questions for SBI Clerk/ RBI Assist. Mains
4. The one who will attend on 2nd is sitting second to 10. G will attend on 4th.
the right of D who will attend on 8th and D uses
Scooter.
5. E who will attend on 3rd is sitting third to the right
of C.

11. The one who will attend on 1st uses Auto.


12. The one who will attend on 6th and uses Cycle is
sitting to the immediate left of the person who uses
Boat.
6. B uses Car and sitting adjacent to F who will
attend on 7th.
7. Both B and F are facing towards the centre.

8. H faces the same direction as C and he is not the


neighbour of G.
9. C is not will attend on 1st and he uses Truck.

Page 205 of 1334


Get a Special Paid Bundle PDF on High Level Puzzles & Seating for All Bank Clerk/ PO Mains
If there are any suggestions/ errors in our PDFs Feel Free to contact us via this email: admin@exampundit.in
Ultra Practice Bundle PDF
Reasoning Questions for SBI Clerk/ RBI Assist. Mains
12.d
13.a
14.c
15.c
SOLUTION (16-20):
Explanation in detail:
1. The adjacent members do not face the same
direction.
2. The person who obtained 40 marks gives Reasoning
test and is sitting third to the right of C.
11.b

3. The person who gives Politics test obtained 42


marks and sits opposite A, who obtained 40 marks.
4. B and C are the neighbours of H.

Page 206 of 1334


Get a Special Paid Bundle PDF on High Level Puzzles & Seating for All Bank Clerk/ PO Mains
If there are any suggestions/ errors in our PDFs Feel Free to contact us via this email: admin@exampundit.in
Ultra Practice Bundle PDF
Reasoning Questions for SBI Clerk/ RBI Assist. Mains
5. B obtained 35 marks and is facing towards the
center.
B is facing to centre so CASE2 is cancelled out.

8. The persons who gives Quant and GA are sitting


opposite to each other.
9. D who obtained 36 marks gives Economics test and
he is an immediate neighbour of A.
10. F who gives GA test does not sit adjacent to both B
and E and he does not obtained highest mark.
11. The person who gives Quant test obtained 45 marks.
12. . The one who has given English test obtained more
marks than the one, who has given GA test.

6. The person who obtained 38 marks is sitting


opposite to the person who gives English test.
7. E who obtained marks only more than two friend is
sitting second to the left of B and gives History test.
There is no such space for E in CASE1b so CASE1b is
cancelled out.

Page 207 of 1334


Get a Special Paid Bundle PDF on High Level Puzzles & Seating for All Bank Clerk/ PO Mains
If there are any suggestions/ errors in our PDFs Feel Free to contact us via this email: admin@exampundit.in
Ultra Practice Bundle PDF
Reasoning Questions for SBI Clerk/ RBI Assist. Mains
C

Mediker

Rs 70 Rs 69

3. Lowest price of sanitizer among six is Rs 64.


4. A sits immediate left to the person who sits
opposite to who purchased Dabur, who does not
next to F.
16.a
17. e
18. b
19. d
20. c
SOLUTION (21-25):
Explanation in detail:
1. C is second to the right of the person who
purchased Mediker whose price is Rs 69 which is
third lowest price among six sanitizer.
2. C sits immediate left of the person who purchased
Dabur and C purchased at Rs 70.

Mediker

Page 208 of 1334


Get a Special Paid Bundle PDF on High Level Puzzles & Seating for All Bank Clerk/ PO Mains
If there are any suggestions/ errors in our PDFs Feel Free to contact us via this email: admin@exampundit.in
Ultra Practice Bundle PDF
Reasoning Questions for SBI Clerk/ RBI Assist. Mains
Rs 70 Rs 69 Rs 64 Mediker

5. One of the friends purchased sanitizer at that price Rs 74 Rs 72 Rs 70 Rs 69 Rs 67 Rs 64


which is prime number.
There is no such space for D and Ozone in CASE2 so
6. B purchased sanitizer at the second highest price
CASE2 is cancelled out.
but did not purchased Ozone.
10. The person who purchased Lifebuoy sits second to
7. B sits second to the left of the person who
the left of who purchased at Rs 72.
purchased at Rs 74.
8. D sits opposite to who purchased Ozone.
9. D does not sit immediate to B.

A B C F

Mediker

Rs 74 Rs 72 Rs 70 Rs 69 Rs 67 Rs 64

11. E did not purchased at lowest price.


12. The person who purchased Himalya did not
purchased at highest price.

B C

Page 209 of 1334


Get a Special Paid Bundle PDF on High Level Puzzles & Seating for All Bank Clerk/ PO Mains
If there are any suggestions/ errors in our PDFs Feel Free to contact us via this email: admin@exampundit.in
Ultra Practice Bundle PDF
Reasoning Questions for SBI Clerk/ RBI Assist. Mains
Explanation in detail:
1. E work at Gurugram and its opposite to the person
who works at Laxmi nagar.
2. F who doesnot works at GTB nagar, but sits to the
immediate left of E.
3. F is sitting adjacent to the person who works at
Gurugram and Rajiv chowk.
4. The person who works at GTB nagar is facing outside
and immediate neighbor of who works at Rajiv chowk.
A B C F E D

Jaiwik Dabur Himalya Mediker Ozone Lifebuoy

Rs 74 Rs 72 Rs 70 Rs 69 Rs 67 Rs 64

21. e
22. c
23. a
24. d
25. b
5. H is immediate neighbor of the person who works at
SOLUTION (26-30):
Ashok nagar and GTB nagar.
Page 210 of 1334
Get a Special Paid Bundle PDF on High Level Puzzles & Seating for All Bank Clerk/ PO Mains
If there are any suggestions/ errors in our PDFs Feel Free to contact us via this email: admin@exampundit.in
Ultra Practice Bundle PDF
Reasoning Questions for SBI Clerk/ RBI Assist. Mains
6. B works at Ashok nagar. 12. The person who works in BOI faces the person who
7. A, who works in BOB but not at Rajiv chowk and sits works in RBI and both faces inside.
third to the right of C, who works at Noida
8. D works at Saket.
9. G faces the centre and sits third to the right of D, who
works in BOM.

The person who works in RBI faces outside so


CASE2 is cancelled out.
13. The person who works in RBI sits immediate left of
who works in OBC.
14. The person who works in SBI does not faces who
10. The person who work in BOB is an immediate works in UBI.
neighbor of who works in SBI and BOI. 15. The person who works at Ashok nagar and Mayur
11. B is not an immediate neighbor of who works in SBI. vihar are facing the same direction.

Page 211 of 1334


Get a Special Paid Bundle PDF on High Level Puzzles & Seating for All Bank Clerk/ PO Mains
If there are any suggestions/ errors in our PDFs Feel Free to contact us via this email: admin@exampundit.in
Ultra Practice Bundle PDF
Reasoning Questions for SBI Clerk/ RBI Assist. Mains
5. P is an immediate right of U.
6. S sits immediate left of the person who likes PHP.

7.

26. a
27. c
28. e
P does not sits on the immediate right to U, so
29. b
Case2 is cancelled out.
30. d
8. The person who likes Ruby is second to the right
SOLUTION (31-35):
of P.
Explanation in detail:
9. The person who likes Perl is an immediate
1. T is an immediate neighbor of the student who
neighbor of P.
likes TCL.
10. Q is fourth to the left of the person who likes Java.
2. T belongs from New delhi.
11. P does not likes Java.
3. U likes TCL.
4. The person who likes PHP is third to the left of the
person who likes TCL.
Page 212 of 1334
Get a Special Paid Bundle PDF on High Level Puzzles & Seating for All Bank Clerk/ PO Mains
If there are any suggestions/ errors in our PDFs Feel Free to contact us via this email: admin@exampundit.in
Ultra Practice Bundle PDF
Reasoning Questions for SBI Clerk/ RBI Assist. Mains

31. a
12. V is third to the right of the person who likes C++. 32. e
13. The person who likes Java is an immediate 33. e
neighbor of the person who belongs to Patna. 34. b
14. The person from Noida likes C++. 35. d
15. V is not an immediate neighbor of either Bhopal SOLUTION(36-40):
or Patna. Explanation in detail:
16. The person from Ranchi sits second to the right of 1. Q does not sits opposite to P.
Q. 2. P is from India and likes Football and sits on the
17. The person from Indore, does not like Python. immediate right of T.
3. Q sits opposite the person who is from Russia.
4. V is from Russia.
5. Q is from France.
6. Only S and T Likes cricket and S sits third to the
right of T.

Page 213 of 1334


Get a Special Paid Bundle PDF on High Level Puzzles & Seating for All Bank Clerk/ PO Mains
If there are any suggestions/ errors in our PDFs Feel Free to contact us via this email: admin@exampundit.in
Ultra Practice Bundle PDF
Reasoning Questions for SBI Clerk/ RBI Assist. Mains
14. W is not from Nepal.

7. V is the third to the left of U.


8. U is from Italy and likes Tennis.
9. W likes the same sports as V.
10. The persons who belongs to Nepal and France
likes football.
11. The persons who like Football are not immediate
neighbors of each other.

36. a
37. c
38. b
39. d
40. e
SOLUTION(41-45):
12. R sits exactly opposite the person who belongs to
Explanation in detail:
USA.
1. D is sitting diagonally opposite to the one who stay
13. The one who is from Japan does not likes football
on second floor.
or cricket.
Page 214 of 1334
Get a Special Paid Bundle PDF on High Level Puzzles & Seating for All Bank Clerk/ PO Mains
If there are any suggestions/ errors in our PDFs Feel Free to contact us via this email: admin@exampundit.in
Ultra Practice Bundle PDF
Reasoning Questions for SBI Clerk/ RBI Assist. Mains
2. Only one person is sitting between E and F, and 4. D stays on fifth floor and he belongs to Indore.
that person stay on second floor.

3. The one who belongs to Indore is sitting to the


5. F stay on first floor and he is sitting to the
immediate left of the person who stay on fourth
immediate left of the person who stay on third
floor.
floor.

Page 215 of 1334


Get a Special Paid Bundle PDF on High Level Puzzles & Seating for All Bank Clerk/ PO Mains
If there are any suggestions/ errors in our PDFs Feel Free to contact us via this email: admin@exampundit.in
Ultra Practice Bundle PDF
Reasoning Questions for SBI Clerk/ RBI Assist. Mains
8. The one who belongs to Vellore does not stay on
neither on fourth nor sixth floor.
9. The one who belongs to Ranchi does not stay on
sixth floor.

10. B does not belongs to Kanpur and he does not stay


on third floor.
11. A does not stay on third floor.

There is no such space for Third floor in CASE1 so


CASE1 is cancelled out.
6. The person who stay on first floor is sitting to the
immediate right to one who belongs to Kanpur.
7. The person who stay on sixth floor is sitting
diagonally opposite to who belongs to Nashik.

Page 216 of 1334


Get a Special Paid Bundle PDF on High Level Puzzles & Seating for All Bank Clerk/ PO Mains
If there are any suggestions/ errors in our PDFs Feel Free to contact us via this email: admin@exampundit.in
Ultra Practice Bundle PDF
Reasoning Questions for SBI Clerk/ RBI Assist. Mains

41.d
42.a 3. C has Sony sits second to the right of the person
43.c who is Singer.
44.a 4. C sits to the immediate left of the person is
45.b Lawyer.
SOLUTION(46-50):
Explanation in detail:
1. A sits to the immediate left of the person who sits
opposite to the person who is Lawyer.
2. The person who is Lawyer is not an immediate
neighbour of F.

Page 217 of 1334


Get a Special Paid Bundle PDF on High Level Puzzles & Seating for All Bank Clerk/ PO Mains
If there are any suggestions/ errors in our PDFs Feel Free to contact us via this email: admin@exampundit.in
Ultra Practice Bundle PDF
Reasoning Questions for SBI Clerk/ RBI Assist. Mains
5. The one who is Teacher sits immediate right to the
one who is Singer.
6. The one who is Teacher has Oppo.

There is no space for D in CASE2 so CASE2 is


cancelled out.
8. B has HTC and he is not an Architecture.
9. B who is not an immediate neighbour of D and sits
second to the left of the person who has Redmi
mobile phone.
10. The one who is Teacher sits second to the left of
the person who has HTC.

7. The one who is Architecture sits opposite to D.

11. The one who is Doctor has not Redmi.


12. F has not Nokia mobile.

Page 218 of 1334


Get a Special Paid Bundle PDF on High Level Puzzles & Seating for All Bank Clerk/ PO Mains
If there are any suggestions/ errors in our PDFs Feel Free to contact us via this email: admin@exampundit.in
Ultra Practice Bundle PDF
Reasoning Questions for SBI Clerk/ RBI Assist. Mains

46.b
47.d
48.a
49.a
50.e

Download Seating Arrangement Practice Questions PDF


Get More Reasoning Practice Questions PDF
Seating Square 3 Variable
Directions (1-5): Study the following information and Q sits at the corner and likes Pear fruit. Only two persons
answer the questions below: sit between Q and the one who was born in an odd
Eight persons P, Q, R, S, T, U, V and W are sitting around numbered date which is greater than 14th. V neither likes
a rectangular table in such a way that four of them sit at Apple nor Guava fruit. S sits third to the right of V and
four corners, who are facing towards center, and rest four was born on 12. S is not an immediate neighbor of Q. The
who sits at the middle of the side, are facing outside. They one who was born on 13th likes Mango fruit and faces
like different Fruits – Apple, Banana, Pear, Grape, Guava, center. Only two persons sit between U who likes Banana
Kiwi, Mango and Cherry. All of them were born on fruit and T who faces inside the center. The one who likes
different date starting from 11 to 18 of the same month of Guava fruit was born on 15th. Q was born on the date
same year. which is an odd number. R faces inside the center. U is not
an immediate neighbor of S. P sits second to the left of W

Page 219 of 1334


Get a Special Paid Bundle PDF on High Level Puzzles & Seating for All Bank Clerk/ PO Mains
If there are any suggestions/ errors in our PDFs Feel Free to contact us via this email: admin@exampundit.in
Ultra Practice Bundle PDF
Reasoning Questions for SBI Clerk/ RBI Assist. Mains
who likes Kiwi fruit. The one who likes Grape fruit sits (a) No one sits between P and W because W and P are
second to the right of the one who likes Apple fruit. The immediate neighbors of each other
youngest person likes Cherry fruit and P is not the oldest (b) S and T
person. S faces inside the center. Neither R nor P likes (c) P and S
Apple fruit. T is 2 days older than U. The one who born (d) The person whose born date is 12.
on 11 and the one who born on 17 are not immediate (e) U, S and T
neighbor. 5. If Q is made to face the opposite direction, who
1. What is position of T with respect to R? would sit to his third right?
(a) Third to the right (a) R
(b) R and T sit diagonally opposite to each other (b) Q
(c) Second to the right (c) W
(d) Second to the left (d) T
(e) Fifth to the right (e) P
2. Who sits exactly between V and U? Directions (6-10): Study the following information and
(a) Q Answer the questions below:
(b) R Eight persons A, B, C, D, E, F, G and H are sitting around
(c) P a rectangular table in such a way that four of them sit at
(d) W four corners are facing towards center, and rest four who
(e) S sits at the middle of the side, are facing outside. They all
3. Which of the following pairs represents the persons are different ages- 25, 18, 32, 7, 36, 40, 21 and 15. Each
seated in the middle of the sides who face Opposite of them likes different Color – Grey, White, Green, Blue,
to each other? Black, Yellow, Orange and Purple.
(a) S, Q Age of D is multiple of 5. C likes Orange color and sits at
(b) T, R the corner. Only two persons sit between C and the one
(c) U, P who is 21years old. The one who likes White color sits
(d) R, V third to the right of the one who is 7years old. The one
(e) T, Q who is 25years old likes Purple color and sits at the corner.
4. Who amongst the following sit between W and P G sits second to the left of the one who is 21 years old. D
when counted in clockwise direction from P? sits second to the left of the one who likes White color. E
Page 220 of 1334
Get a Special Paid Bundle PDF on High Level Puzzles & Seating for All Bank Clerk/ PO Mains
If there are any suggestions/ errors in our PDFs Feel Free to contact us via this email: admin@exampundit.in
Ultra Practice Bundle PDF
Reasoning Questions for SBI Clerk/ RBI Assist. Mains
is the oldest person of the group. B likes Grey color and 9. What is the position of H with respect to the one
sits third to the right of D. The one who likes blue color who likes Orange color?
sits opposite to G. The one who likes Purple color sits (a) Third to the right
second to the left of the one who likes Yellow color. F sits (b) Second to the left
second to the right of H. The one who likes Black color (c) Either (a) or (e)
sits second to the right of the one who is 32 years old. C (d) Immediate left
is older than B who is not the third oldest person of the (e) Fifth to the left
group. The one who is 7 years old does not sit at the 10. Four of the following five are alike in a certain way
corner. and hence form a group. Which of the following
6. The age of the person who is 2nd to the right of D does not belong to the group?
is? (a) B
(a) 40years (b) D
(b) 32years (c) G
(c) 25years (d) H
(d) 18year (e) F
(e) None of these Directions (11-15): Study the following information
7. Who among the following person sit opposite to the and Answer the questions below:
person of is immediate left of F? Eight friends S, T, U, V, W, X, Y and Z are sitting around
(a) The one whose age is 32years. a square table in such a way that four of them sit at four
(b) The one whose age is 18years. corners while four sit in the middle of each of the four
(c) The one whose age is 15years. sides, but not necessarily in the same order. Each one of
(d) The one whose age is 40years. them like different type of sports viz. Hockey, Cricket,
(e) None of these Basket Ball, Foot Ball, Boxing, Wrestling, Badminton and
8. Which among the following combination is true? Tennis. And each one of them have different weight viz.
(a) B –White 61kg , 59kg, 46kg, 71kg, 73kg, 66kg, 56kg, and 81kg. The
(b) G – Yellow one who sits in the middle of the sides face the center
(c) A- Blue while those who sits at the four corners face outside (i.e
(d) C - Grey opposite to the center).
(e) None is true
Page 221 of 1334
Get a Special Paid Bundle PDF on High Level Puzzles & Seating for All Bank Clerk/ PO Mains
If there are any suggestions/ errors in our PDFs Feel Free to contact us via this email: admin@exampundit.in
Ultra Practice Bundle PDF
Reasoning Questions for SBI Clerk/ RBI Assist. Mains
Y sits third to the right of W. The one who faces the centre (b) Boxing
likes Cricket. W sits on one of the corners of the table. T (c) Can’t be determined
does not like Basketball. The one who likes Badminton is (d) None of these
one of the immediate neighbors of X. Total weight of (e) Foot Ball
immediate neighbor of T is 112kg. The one who likes 13. Who among the following sits exactly between U
Cricket sits immediate left of the person one who likes and the Basketball Player?
Badminton. The person having 73kg weight sits between (a) The person who plays Tennis
U and the person who like Badminton. Only one person (b) The person who plays Wrestling
sits between V and Y. T is one of the immediate neighbors (c) The person who plays Badminton
of V. The one who faces the outside of the centre likes (d) Football player
Boxing. The one who likes Hockey faces the Y. The (e) None of these
persons who like Wrestling and Football sit next to each 14. How many persons sit between the one whose
other. W does not like Boxing. U sits second to the right weight is 56kg and the one whose weight is 59kg?
of T. Total weight of U and W is 152kg. Only three (a) None
persons sit between T and S and difference of their weight (b) One
is 2kg and S is heavier than T. Z sits to the immediate right (c) Two
of S. The persons who like Basketball and Boxing sit on (d) Three
the corners and opposite to each other. Immediate (e) Can’t be determined
neighbor of W’s total weight is 122kg. The person likes 15. Which of the following pairs are the immediate
wrestling sit immediate right of the person who likes neighbors of the Hockey player?
Tennis. W is heaviest person among all friends. (a) S, W
11. Who among the following is a Cricket player? (b) X, S
(a) X (c) U, T
(b) U (d) V, W
(c) T (e) W, U
(d) Can’t be determined Directions (16-20): Study the following information
(e) None of these carefully and answer the below questions.
12. V is related to which of the following games? Eight person namely – A, B, C, D, E, F, G and H are sitting
(a) Badminton in a rectangular table in such a way that one person is
Page 222 of 1334
Get a Special Paid Bundle PDF on High Level Puzzles & Seating for All Bank Clerk/ PO Mains
If there are any suggestions/ errors in our PDFs Feel Free to contact us via this email: admin@exampundit.in
Ultra Practice Bundle PDF
Reasoning Questions for SBI Clerk/ RBI Assist. Mains
sitting on each corner and one person on each edge. Person the one who likes Shakib Al Hasan doesn’t sit together.
on edge are facing center while person at each corner sits The person who likes Crys Gale neither sits adjacent to G
facing outward. Each person likes different cricketers viz. nor with the one who from Bangladesh but sits immediate
Shakib Al Hasan, Joe Root, Sourav ganguly, Kumar left of the one who likes Kene Williamson. The person
Shangakara, Crys Gale, Ricky Ponting, Kene Williamson who likes Kumar Shangakara faces E. Only two people sit
and AB de villiers. Each person also likes different Teams between C and the one who is from South Africa.
viz. India, Australia, England, South Africa, New 16. Who sits to the second to left of one who likes
Zealand, West Indices, Bangladesh and Srilanka. All the Sourav?
information are not necessary in same order. (a) The one who likes Sachin.
The person who likes Ricky Ponting sits third to right of (b) The one who likes Australia.
the one who is from Bangladesh Team. The person who (c) One who likes Joe Root.
likes Kene williamson sits third to right of the one who is (d) The one who likes Shakib.
from South Africa. D, who doesn’t sit adjacent to the (e) The one who likes AB de Villiers.
person one who likes Kumar Shangakara and sits adjacent 17. What is the position of one who likes Kumar
to the one who is from Australia. The one who is from Shangakara with respect to one who likes Kene
Bangladesh sits immediate left of the person one who likes Williamson?
Joe Root but doesn’t sit at corner of the table. C is from (a) Second to left
New Zealand, sits second to the left of the person who (b) Third to right
likes Shakib Al Hasan. G likes Ricky Ponting, sits second (c) Can’t be determined
to the right of the one who likes West Indices. E and the (d) Immediate left
person who is from England are immediate neighbours. (e) Third to Left
Only two persons sit between the person who is from West 18. Which of the following combination is not true?
Indices and the one who likes Sourav Ganguly. H, who (a) F– WI – Crus Gale
neither likes Joe Root nor from New Zealand team but sits (b) D – India – Kene
adjacent to the one who likes Sourav Ganguly. B and the (c) C – NZ – Kumar
person from New Zealand are immediate neighbours. (d) E – SA – Ricky Ponting
Only two persons sit between H and A, who is from (e) All the above given combination are true.
Srilanka team but not sit adjacent to the person who
neither likes Joe Root nor likes Shakib Al Hasan. F and
Page 223 of 1334
Get a Special Paid Bundle PDF on High Level Puzzles & Seating for All Bank Clerk/ PO Mains
If there are any suggestions/ errors in our PDFs Feel Free to contact us via this email: admin@exampundit.in
Ultra Practice Bundle PDF
Reasoning Questions for SBI Clerk/ RBI Assist. Mains
19. How many person sits between one who likes AUS The one who live in Delhi sits immediate right of the one
and one who likes Kene Williamson when counted who works in Oracle. The one who works in Wipro sits
left of one who likes AUS? 2nd to the left of N. C sits 2nd left of K who works in Tech
(a) Three Mahindra. L sits opposite of the one who works in Oracle.
(b) Four The one who works in IBM sits immediate left of the one
(c) One who works in Oracle. There are two members sit between
(d) None the one who live in Delhi and Raipur. T sits 3rd right of the
(e) Two one who sits immediate left of L. The one who lives in
20. Which of the following statement is not true? Indore sits immediate right of the one who lives in
(a) One who likes Kene sits second to right of one Chandigarh. The one who works in Cognizant sits 2nd to
who likes Srilanka. the right of the one who works in Microsoft. There are two
(b) The one who likes Ricky Ponting sits immediate members sit between L and K. P does not sit opposite side
right of one who likes Srilanka. of C. K does not lives in Chennai. O sits 3rd to the right of
(c) B sits second to left of F. the one who works in Google who sits opposite side of the
(d) One who likes India sits second to right of one who one who works in Microsoft. The one who lives in Jaipur
likes Shakib. sits opposite side of the one who works in Cognizant. The
(e) All the above given information are true. one who works in Patni Computers sits just opposite of
Directions (21-25): Study the following information that member who work sits in Wipro. P sits just near to L.
carefully and answer the below questions G does not opposite side of the one who works in Google.
Eight members of a society i.e. C, G, N, P, K, T, O and L P does not live in Raipur but sits opposite of the one who
are sitting around a square table; no one sits at corner only lives in Mumbai. O lives in Delhi and L doesn’t live in
two persons sit each of the side of square table. They are Chandigarh.
working in different companies i.e. Wipro, Tech 21. What is the position of O with respect to G?
Mahindra, Patni Computer, IBM, Google, Cognizant, (a) Third to the right
Microsoft and Oracle but not in same order. They all are (b) Second to the right
facing towards the centre and lives in different City i.e. (c) Third to the left
Chandigarh, Mumbai, Delhi, Chennai, Hyderabad, (d) Second to the left
Indore, Raipur and Jaipur but not in same order. (e) Fourth to the right

Page 224 of 1334


Get a Special Paid Bundle PDF on High Level Puzzles & Seating for All Bank Clerk/ PO Mains
If there are any suggestions/ errors in our PDFs Feel Free to contact us via this email: admin@exampundit.in
Ultra Practice Bundle PDF
Reasoning Questions for SBI Clerk/ RBI Assist. Mains
22. Who sits third to the right of the one who work in (e) P
Tech Mahindra? Directions (26-30): Study the following information
(a) G carefully and answer the below questions
(b) C Eight persons i.e. K, H, C, X, O, W, F and Z are sitting
(c) T around a square table. After completion of their
(d) P graduation, they are going to open different shops in
(e) L different months of same year. These shops are Bakery,
23. How many person sit between the person who work Garment, Electrical, Stationary, Medical, Dairy, Gift and
in IBM and the person who is from Jaipur when Liquor but not in same order. Now different opening
counter to the right of IBM? months are March, June, August, December, January,
(a) None April, February and November but not in same order. Four
(b) One persons are seated at the corner of this table and faces
(c) Two inside and Rest four persons are seated at middle of each
(d) Three side of this table and faces outside.
(e) Four O is not going to open shop in June. The one who is going
24. Which of the following statement is true? to open Liquor shop does not sit near to W. The one who
(a) The person who is from Mumbai is 2nd to the right sits at one of the corner of this table is going to open shop
of the one who is from Hyderabad. in August and sits 3rd left of the one who is going to open
(b) The person who work in Google sits 3rd to the left Medical shop. There are two persons sit between C and
of the person who is from Delhi. the one who is going open shop in August. K sits 2nd right
(c) One person between K and T of C who is going to open shop in April. The one who is
(d) G and P faces each other going to open shop in March sits 2nd left of the person
(e) None of these who is going to open shop in that month which has less
25. Which of the following person sits 3rd to the right days compare to all other months. C sits opposite to the
of the person who faces the person from Chennai? one who is going to open shop in February. X who is going
(a) C to open shop in December sits immediate right of C. The
(b) T one who is going open Electrical shop sits opposite to the
(c) O one who is going open Stationary shop. There are two
(d) N Person sit between X and W who is going to open shop in
Page 225 of 1334
Get a Special Paid Bundle PDF on High Level Puzzles & Seating for All Bank Clerk/ PO Mains
If there are any suggestions/ errors in our PDFs Feel Free to contact us via this email: admin@exampundit.in
Ultra Practice Bundle PDF
Reasoning Questions for SBI Clerk/ RBI Assist. Mains
January. Z is going to open Liquor shop but sits diagonally 29. Who sits immediate right of the one who want to
opposite to the one who is going open dairy shop. The one open Dairy Shop?
who is going to open shop in November sits 2nd to the right (a) Who want to open Medical Shop
of F. The one who is going to open Stationary shop sits (b) Who want to open Electrical Shop
near to X. The one who is going to open shop in August (c) Who want to open Gift Shop
does not want to open Gift and Bakery shop. The one who (d) Who want to open Stationary Shop
is going to open Bakery shop sits just near to F. (e) Who want to open Liquor Shop
26. What is the position of X with respect to the one 30. How many person sits between the one who open
who want to open Electrical shop? shop on August and April?
(a) Third to right (a) Six
(b) Second to left (b) Five
(c) Third to left (c) None
(d) Fourth to right (d) Two
(e) Fourth to left (e) Three
27. What is the position of Z with respect to the one Directions (31-35): Study the following information
who open his shop on March? carefully and answer the below questions
(a) Second to right E, F, G, H, I, J, K and L are 8 members of a family each
(b) Third to right having different age. They all have different hobbies (or
(c) Immediate left they like different activity) viz Cricket, Chess, Hockey,
(d) Second to left Tennis, TT, Football, Basketball and Volleyball. They are
(e) Immediate right sitting around a rectangular table facing the centre and
28. Who sits third to the right of the one, who Open his they all are seated with equal distance, but not necessarily
shop on December? in the same order. Each side of the table two person sits
(a) C together, no person sits on the corner of the table. Ages
(b) X are an integer value. There are four couples in the family.
(c) W Every Wife is at least two year younger than her husband.
(d) O Three generations are there in a family.
(e) K K does not sit opposite to L. The hobby of G’s son-in-law
is not Football and Chess. H, who is 34 years old, is son-
Page 226 of 1334
Get a Special Paid Bundle PDF on High Level Puzzles & Seating for All Bank Clerk/ PO Mains
If there are any suggestions/ errors in our PDFs Feel Free to contact us via this email: admin@exampundit.in
Ultra Practice Bundle PDF
Reasoning Questions for SBI Clerk/ RBI Assist. Mains
in-law of I and seated second to the right of K, who is the (a) Cricket, 32
wife of E. The hobby of H’s brother-in-law is Basketball. (b) Football, 30
The person whose hobby is Tennis sits second to the right (c) Basketball, 36
of the person, who is 80 years old. G is the father of F who (d) TT, 80
is not the youngest member in the family. The hobby of (e) None of these
L’s grandfather is not Volleyball. No three females are 33. Which of the following is true regarding I?
sitting together. The daughter-in-law of E is 56 years old. (a) I is immediate neighbor of K and H.
L is 6 years elder than his wife. The person who is 36 years (b) I have two son
old is seated second to the left of J’s father-in-law. The (c) Age of I is 58 years
age of E is 2.5 times the age of F. Only one couple can sit (d) Hobby of I is Chess
together. The hobby of the person whose age is 78 years (e) None of these
is neither TT nor Volleyball. No one is less than 22 years 34. H is related to Cricket in the same way as F is
old. Each parent is at least 20 years elder than his/ her related to Football based on the given
child. K is the grandmother of L. The daughter-in-law of arrangement, to whom amongst the following is J
I is not near to her. The person whose hobby Cricket, is related to the following same pattern?
immediate neighbour of the persons, whose hobby is (a) TT
Chess and the one, who is husband of J. The Person whose (b) Chess
hobby is Football sits opposite to the person, who is (c) Basketball
immediate right of F’s husband. E is seated between L’s (d) Hockey
grandmother and wife of L. (e) Volleyball
31. Who amongst the following faces the person whose 35. Who among the following is Grand Son in law of
Hobby is Hockey? E?
(a) The person whose hobby is Cricket (a) The one whose age is 30years
(b) The person whose age is 58years (b) The one whose age is 36years
(c) The person whose hobby is Basketball (c) The one whose age is 32years
(d) The person whose age is 36years (d) The one whose age is 36years
(e) None of these (e) The one whose age is 34years
32. The Hobby and Age of the person who is brother Directions (36-40): Study the following information
of F is? carefully and answer the below questions
Page 227 of 1334
Get a Special Paid Bundle PDF on High Level Puzzles & Seating for All Bank Clerk/ PO Mains
If there are any suggestions/ errors in our PDFs Feel Free to contact us via this email: admin@exampundit.in
Ultra Practice Bundle PDF
Reasoning Questions for SBI Clerk/ RBI Assist. Mains
A, B, C, D, E, F, G and H are eight boys sitting around a (a) VS
Rectangular table. Two of them are facing outside while (b) RA
rest is facing towards centre. Each side of the table two (c) BJ
person sits together, no person sits on the corner of the (d) CB
table. All of them like books of different writers, viz. CB, (e) IG
RA, VS, PB, AP, BJ, EN and IG but not necessarily in the 37. Four of the following five are alike in a certain way
same order. Each of them is carrying a bucket with a based on the given arrangement and hence form a
capacity of 3, 5 or 8 liters. Total volume of all the buckets group. Which is the one that doesn’t belong to that
of these eight friends is 40 liters. group?
D and C are sitting third and second to the left of A (a) H
respectively who faces outside. The difference between (b) A
the volumes of buckets of B and the one who likes VS is (c) C
the same as the number of persons sitting between them. (d) D
D and G are neighbors of B, who likes AP. The one who (e) E
likes PB sits on the immediate right of B. G is sitting third 38. The total sum of the person who like book writers
to the right of A and likes CB. The sum of total capacities EN, PB, BJ and RA have how many ltrs?
of the buckets of the immediate neighbors of A is greater (a) 22
than 6 but less than 10. H sits third to the right of E and (b) 18
likes EN. The capacity of the bucket of the person who (c) 20
likes IG is not the highest but greater than that of his (d) 21
immediate right neighbor. A and H are not the neighbor of (e) 19
that person who likes BJ. F does not like VS and PB. Both 39. Who is second to the left of G?
the immediate neighbors of E have buckets of 3 liters (a) D
Capacity. The person who likes IG is immediate neighbor (b) C
of the one who sits sitting opposite E. The one who likes (c) B
AP has a bucket of 5 liters. B and A are not facing each (d) E
other. The person who likes RA does not faces AP (e) F
36. Who sits to the immediate left of the one who like 40. How many persons sit between D and the person
Book writer AP? who likes book writer RA?
Page 228 of 1334
Get a Special Paid Bundle PDF on High Level Puzzles & Seating for All Bank Clerk/ PO Mains
If there are any suggestions/ errors in our PDFs Feel Free to contact us via this email: admin@exampundit.in
Ultra Practice Bundle PDF
Reasoning Questions for SBI Clerk/ RBI Assist. Mains
(a) One Financial Advisor. The Financial Advisor sits third to the
(b) Four right of Q. Q is not the Vice – President. U is not an
(c) Three immediate neighbor of P. V is not an immediate neighbor
(d) Two of the Manager. Only two people sit between the
(e) None Managing Director and W. The Vice – President and the
Directions (41-45): Study the following information Company Secretary are immediate neighbors of each
carefully to answer the given questions. other. Financial Advisor faces the person who likes
Eight Friends P, Q, R, S, T, U, V and W are sitting around Finding Nemo.
a Rectangular table facing the centre. Four of them sit at 41. Who amongst the following sits second to the left of
corners of the table. Each one of them holds a different T?
post viz. Manager, Company Secretary, Chairman, (a) V
President, Vice – President, Group Leader, Financial (b) The Manager
Advisor and Managing Director but not necessarily in the (c) P
same order. Each person likes different movies namely viz (d) The Financial Advisor
– Spider Man, Titanic, Batman, Super Man, Finding (e) Q
Nemo, Jurrasic Park, Avengers and Avatar. 42. Four of the following five are alike in a certain way
President faces the person who likes Spider-Man. Q likes based on the given arrangement and thus form a
Titanic. P sits third to the right of Managing Director. P is group. Which is the one that does not belong to that
not the Chairman. The person who likes Avengers sits group?
immediately right to the person who likes Avatar and (a) T – Chairman
faces Jurrasic Park. T is not an immediate neighbor of (b) V – President
W. R sits on the immediate right of the Chairman. Neither (c) S – Manager
P nor W is a Vice – President or a Company Secretary. (d) P – Financial Advisor
Vice –President is not an immediate neighbor of the (e) Q – Managing Director
Managing Director. The person who likes Superman sits 43. Who among the following is the Vice – President of
exactly in the middle between the persons who like the company?
Batman and Avatar. The Manager sits second to the left of (a) S
T and does not sit at corner of the table. The Manager is (b) R
an immediate neighbor of the both Group Leader and the (c) Q
Page 229 of 1334
Get a Special Paid Bundle PDF on High Level Puzzles & Seating for All Bank Clerk/ PO Mains
If there are any suggestions/ errors in our PDFs Feel Free to contact us via this email: admin@exampundit.in
Ultra Practice Bundle PDF
Reasoning Questions for SBI Clerk/ RBI Assist. Mains
(d) P not necessarily in the same order. These students are from
(e) V different Collages, viz IIT delhi, IIT Mumbai, IIT Indore,
44. Which of the following is true with respect to the IIT Bhilai, IIT Kharagpur, IIT Chennai, IIT Guwahati and
given seating arrangement? IIT Goa but not necessarily in the same order. No two
(a) T sits to the immediate left of the Managing students of the same subject group are sitting adjacent to
Director each other except those who belong to English group. The
(b) The Group Leader of the company is an immediate Students from reasoning group are sitting opposite to each
neighbor of the Vice – President other. V is neither in IIT Indore nor in IIT kharagpur and
(c) V sits second to right of S does not sit at any corners. The student who is from IIT
(d) The Group Leader and the Company Secretary are Guwahati is sitting on the immediate right of the student
immediate neighbors of each other who is from IIT Indore. U, who is in the IIT Bhilai, is in
(e) The Group Leader sits second to the left of the General Knowledge group and she is sitting on the
person who likes Super Man immediate right of Y, who is in group English. Y is neither
45. Who among the following likes Titanic? from IIT Kharagpur nor IIT Indore. T who is from IIT
(a) Chairman Delhi is neither in reasoning group nor in the subject
(b) Manager group of General Knowledge and Math’s. T is sitting
(c) Vice – President opposite to W. Only IIT Chennai Student S is sitting
(d) Company Secretary between the participant W, who is from Guwahati and IIT
(e) Financial Advisor Goa participant. Both the students of subject group
Directions (46-50): Study the following information Mathematics are sitting Immediate Right of the students
carefully to answer the given questions of subject group Reasoning. X is not the neighbor of V.
Eight students S, T, U, V, W, X, Y and Z participated in a The students of Mathematics group do not sit at middle of
Collage Quiz competition. They were sitting around a the side.
Square table in such a way that one Student is sitting on 46. Who is to the immediate left of the one who is from
each corner and one student on each edge. The students IIT Guwahati College?
who sit at corners faces outside and rest faces inside All (a) S
eight students participated in different subjects quiz (b) V
General Knowledge, Mathematics, English, and (c) Z
Reasoning, at least two students in each subject group, but (d) T
Page 230 of 1334
Get a Special Paid Bundle PDF on High Level Puzzles & Seating for All Bank Clerk/ PO Mains
If there are any suggestions/ errors in our PDFs Feel Free to contact us via this email: admin@exampundit.in
Ultra Practice Bundle PDF
Reasoning Questions for SBI Clerk/ RBI Assist. Mains
(e) None of these 49. Which of the following subject groups sits at
47. Who among the followings is from IIT Delhi and corners only?
IIT Goa respectively? (a) Reasoning
(a) V, U (b) Mathematics
(b) T, Z (c) English
(c) T, Y (d) GK
(d) T, V (e) None of these
(e) None of these 50. Four of the following five are alike in a certain way
48. Who is sitting third to the left of the one who is and hence form a group. Which of the following
from IIT Chennai and belongs to which subject does not belong to the group?
group? (a) U
(a) U, GK (b) T
(b) W, Reasoning (c) V
(c) V, Reasoning (d) W
(d) Z, GK (e) Y
(e) None of these

Page 231 of 1334


Get a Special Paid Bundle PDF on High Level Puzzles & Seating for All Bank Clerk/ PO Mains
If there are any suggestions/ errors in our PDFs Feel Free to contact us via this email: admin@exampundit.in
Ultra Practice Bundle PDF
Reasoning Questions for SBI Clerk/ RBI Assist. Mains
Seating Square 3 Variable – Answer and Explanation
SOLUTION (1-5): CASE 2
1. S sits third to the right of V and was born on 12.
2. S faces inside the center.(S sits at corner)
3. U is not an immediate neighbor of S.
4. Only two persons sit between U who likes Banana
fruit and T who face inside the center.

8. P sits second to the left of W who likes Kiwi fruit


9. Q was born on the date which is an odd number.
10. The one who likes Guava fruit was born on 15th.
11. The one who was born on 13th like Mango fruit and
faces center
5. Q sits at the corner and likes Pear fruit. Here date for Q = 11, 13, 15, 17
6. R faces inside the center.(R sits at corner) 13, 15 not possible for Q because Q like Pear fruit and
7. S is not an immediate neighbour of Q 13 and 15 like different fruits other than Q.
Here 2 cases for Q So Q = 17/ 11
CASE 1
CASE 1

Page 232 of 1334


Get a Special Paid Bundle PDF on High Level Puzzles & Seating for All Bank Clerk/ PO Mains
If there are any suggestions/ errors in our PDFs Feel Free to contact us via this email: admin@exampundit.in
Ultra Practice Bundle PDF
Reasoning Questions for SBI Clerk/ RBI Assist. Mains
CASE 2

16. The one who likes Guava fruit was born on 15th
12. V neither likes Apple nor Guava fruit. 17. The youngest person likes Cherry fruit. (youngest
13. The one who was born on 13th likes Mango fruit and means born on 18)
faces center. 18. V does not like Guava fruit.
14. The one who likes Grape fruit sits second to the right CASE 1
of the one who likes Apple fruit.
15. Neither R nor P likes Apple fruit.
CASE 1(In this case we can place Apple at T also but
after that we cannot place 13 and mango. So, we have
left with only option by placing Apple as S.)

CASE 2

CASE 2

19. T is 2 days older than U.


Page 233 of 1334
Get a Special Paid Bundle PDF on High Level Puzzles & Seating for All Bank Clerk/ PO Mains
If there are any suggestions/ errors in our PDFs Feel Free to contact us via this email: admin@exampundit.in
Ultra Practice Bundle PDF
Reasoning Questions for SBI Clerk/ RBI Assist. Mains
20. The one who born on 11 and the one who born on 17 1. b
are not immediate neighbor. 2. a
21. Only two persons sit between Q and the one who was 3. c
born in an odd numbered date which is greater than 4. d
14th. 5. e
From here it is clear that T = 14 and U = 16 SOLUTION (6-10):
And case 1 was also eliminated because 11 and 17 are 1. C likes Orange colour and sits at the corner.
immediate neighbor. 2. Only two persons sit between C and the one who is
Case 2 21years old
3. G sits second to the left of the one who is 21 years old
4. The one who likes blue colour sits opposite to G.
Two case for C and 21
CASE 1

Final Arrangement

CASE 2

5. The one who is 7 years old does not sit at the corner.

Page 234 of 1334


Get a Special Paid Bundle PDF on High Level Puzzles & Seating for All Bank Clerk/ PO Mains
If there are any suggestions/ errors in our PDFs Feel Free to contact us via this email: admin@exampundit.in
Ultra Practice Bundle PDF
Reasoning Questions for SBI Clerk/ RBI Assist. Mains
6. The one who likes White color sits third to the right of 11. F sits second to the right of H
the one who is 7years old. Here CASE 2 was eliminated because purple must in
7. D sits second to the left of the one who likes White corner which is not possible in case 2.
color. CASE 1
8. B likes Grey colour and sits third to the right of D.
We can place 7 to immediate left of C in both the cases
because 8th condition not follow.
CASE 1

12. The one who likes Black colour sits second to the right
of the one who is 32 years old.
13. Age of D is multiple of 5. (D’s age is 15)
14. E is the oldest person of the group.(E’s age id 40)
CASE 1
CASE 2

15. C is older than B. (C= 36years, B = 18years)

9. The one who is 25years old likes Purple colour and Final Arrangement

sits at the corner


10. The one who likes Purple colour sits second to the left
of the one who likes Yellow colour.

Page 235 of 1334


Get a Special Paid Bundle PDF on High Level Puzzles & Seating for All Bank Clerk/ PO Mains
If there are any suggestions/ errors in our PDFs Feel Free to contact us via this email: admin@exampundit.in
Ultra Practice Bundle PDF
Reasoning Questions for SBI Clerk/ RBI Assist. Mains
7. T is one of the immediate neighbors of V.
8. U sits second to the right of T.
9. Total weight of U and W is 152kg. ( W = 81 kg)
Here two cases for V
CASE 1

6.a
7.d
8.e
9.c
10.b
CASE 2
SOLUTION (11-15):
1. Y sits third to the right of W
2. W sits on one of the corners of the table.
3. The one who likes Hockey faces the Y
4. W is heaviest person among all friends. (W = 81kg)
5. Immediate neighbor of W’s total weight is 122kg.
So W’s immediate neighbor’s weight is
10. Only three persons sit between T and S and difference
56 + 66 = 122 kg of their weight is 2kg and S is heavier than T.
11. Z sits to the immediate right of S.
From here case 2 eliminated because T sits opposite to S
which is not possible in Case 2.
CASE 1

6. Only one person sits between V and Y.


Page 236 of 1334
Get a Special Paid Bundle PDF on High Level Puzzles & Seating for All Bank Clerk/ PO Mains
If there are any suggestions/ errors in our PDFs Feel Free to contact us via this email: admin@exampundit.in
Ultra Practice Bundle PDF
Reasoning Questions for SBI Clerk/ RBI Assist. Mains
18. The one who faces the outside of the centre likes
Boxing.
19. The persons who like Basketball and Boxing sit on the
corners and opposite to each other.
20. W does not like Boxing.
CASE 1

12. The one who likes Badminton is one of the immediate


neighbors of X.
13. The one who likes Cricket sits immediate left of the
person one who likes Badminton.
14. The person having 73kg weight sits between U and the
person who like Badminton.
21. The person likes wrestling sit immediate right of the
15. The one who faces the centre likes Cricket.
person who likes Tennis
22. The persons who like Wrestling and Football sit next
CASE 1
to each other
CASE 1

16. Total weight of immediate neighbor of T is 112kg.


(V = 66 and Y = 46)

Final Arrangement

17. T does not like Basketball.

Page 237 of 1334


Get a Special Paid Bundle PDF on High Level Puzzles & Seating for All Bank Clerk/ PO Mains
If there are any suggestions/ errors in our PDFs Feel Free to contact us via this email: admin@exampundit.in
Ultra Practice Bundle PDF
Reasoning Questions for SBI Clerk/ RBI Assist. Mains
4. Only two persons sit between the person who is from
West Indices and the one who likes Sourav Ganguly.
5. H, who neither likes Joe Root nor from New Zealand
team but sits adjacent to the one who likes Sourav
Ganguly.
Here two case for Sourav Ganguly
CASE 1

11. a
12. e
13. c
14. e
15. a
SOLUTION (16-20):
1. The one who is from Bangladesh sits immediate left CASE 2
of the person one who likes Joe Root but doesn’t sit at
corner of the table.
2. The person who likes Ricky Ponting sits third to right
of the one who is from Bangladesh Team.
3. G likes Ricky Ponting, sits second to the right of the
one who likes West Indices.

6. Only two persons sit between H and A, who is from


Srilanka team but not sit adjacent to the person who
either likes Joe Root or likes Shakib Al Hasan.
CASE 1

Page 238 of 1334


Get a Special Paid Bundle PDF on High Level Puzzles & Seating for All Bank Clerk/ PO Mains
If there are any suggestions/ errors in our PDFs Feel Free to contact us via this email: admin@exampundit.in
Ultra Practice Bundle PDF
Reasoning Questions for SBI Clerk/ RBI Assist. Mains

12. The One who likes Kumar Shangakara facing E


CASE 2 13. E and the person who is from England are immediate
neighbors.
CASE 2

7. C is from New Zealand, sits second to the left of the


person who likes Shakib Al Hasan.
8. Only two people sit between C and the one who is
from South Africa.
14. D, who doesn’t sit adjacent to the person one who likes
9. The person who likes Kene williamson sits third to
Kumar Shangakara and sits adjacent to the one who is
right of the one who is from South Africa.
from Australia.
10. The person who likes Crys Gale neither sits adjacent
15. B and the person from New Zealand are immediate
to G nor with the one who from Bangladesh but sits
neighbours.
immediate left of the one who likes Kene Williamson.
16. F and the one who likes Shakib Al Hasan doesn’t sit
11. Only two persons sit between H and A, who is from
together.
Srilanka team but not sit adjacent to the person who
Final Arrangement
neither likes Joe Root nor likes Shakib Al Hasan.
Form here Case 1 eliminated because we cannot satisfy
above 7, 8 and 9 points.
CASE 2
Page 239 of 1334
Get a Special Paid Bundle PDF on High Level Puzzles & Seating for All Bank Clerk/ PO Mains
If there are any suggestions/ errors in our PDFs Feel Free to contact us via this email: admin@exampundit.in
Ultra Practice Bundle PDF
Reasoning Questions for SBI Clerk/ RBI Assist. Mains
CASE 2

16. d
6. T sits 3rd right of the one who sits immediate left of L.
17. c
7. There are two members sit between L and K
18. a
8. C sits 2nd left of K who works in Tech Mahindra.
19. e
From here case 1 eliminated because we cannot place K
20. b
as work in Tech Mahindra.
SOLUTION (21-25):
CASE 2
1. L sits opposite of the one who works in Oracle.
2. The one who live in Delhi sits immediate right of the
one who works in Oracle.
3. The one who works in IBM sits immediate left of the
one who works in Oracle.
4. O lives in Delhi.
5. O sits 3rd to the right of the one who works in Google
who sits opposite side of the one who works in 9. P sits just near to L.
Microsoft. 10. P does not live in Raipur but sits opposite of the one
CASE 1 who lives in Mumbai.
11. The one who works in Cognizant sits 2nd to the right
to the one who works in Microsoft.
CASE 2

Page 240 of 1334


Get a Special Paid Bundle PDF on High Level Puzzles & Seating for All Bank Clerk/ PO Mains
If there are any suggestions/ errors in our PDFs Feel Free to contact us via this email: admin@exampundit.in
Ultra Practice Bundle PDF
Reasoning Questions for SBI Clerk/ RBI Assist. Mains

S
12. The one who works in Wipro sits 2nd to the left of N
13. The one who lives in Jaipur sits opposite side of the Final Arrangement

one who works in Cognizant.


14. The one who works in Patni Computers sits just
opposite of that member who work sits in Wipro
15. P does not live in Raipur
16. There are two members sit between the one who live
in Delhi and Raipur
CASE 2
21.c
22.e
23.a
24.b
25.d
SOLUTION (26-30):
1. The one who sits at one of the corner of this table is
going to open shop in August and sits 3rd left of the
17. The one who lives in Indore sits immediate right of
one who is going to open Medical shop.
the one who lives in Chandigarh.
2. There are two persons sit between C and the one who
18. K does not lives in Chennai
is going open shop in August.
CASE 2
3. K sits 2nd right of C who is going to open shop in April

Page 241 of 1334


Get a Special Paid Bundle PDF on High Level Puzzles & Seating for All Bank Clerk/ PO Mains
If there are any suggestions/ errors in our PDFs Feel Free to contact us via this email: admin@exampundit.in
Ultra Practice Bundle PDF
Reasoning Questions for SBI Clerk/ RBI Assist. Mains
4. C sits opposite to the one who is going to open shop in
February.
Two cases for C
CASE 1

CASE 2

CASE 2

9. The one who is going to open shop in November sits 2nd


to the right of F.
5. X who is going to open shop in December sits 10. The one who is going to open shop in March sits 2nd
immediate right of C. left of the person who is going to open shop in that month
6. The one who is going open Electrical shop sits which has less days compare to all other months.
opposite to the one who is going open Stationary shop. 11.The one who is going to open Bakery shop sits just near
7. The one who is going to open Stationary shop sits near to F.
to X. 12. June month must be placed at remaining seat at corner
8. There are two Person sit between X and W who is of the table. So, O must also open the shop in month Feb.
going to open shop in January. CASE 1 eliminated because we cannot place F and
CASE 1 November
CASE 2

Page 242 of 1334


Get a Special Paid Bundle PDF on High Level Puzzles & Seating for All Bank Clerk/ PO Mains
If there are any suggestions/ errors in our PDFs Feel Free to contact us via this email: admin@exampundit.in
Ultra Practice Bundle PDF
Reasoning Questions for SBI Clerk/ RBI Assist. Mains
Final Arrangement

s
13. Z is going to open Liquor shop but sits diagonally
26.a
opposite to the one who is going open dairy shop.
27.e
14. The one who is going to open Liquor shop does not sit
28.c
near to W.
29.b
CASE 2
30.d
SOLUTION (31-35):
Blood Relation
1. K is the wife of E.
2. The daughter-in-law of E

15.The one who is going to open shop in August does not


want to open Gift shop.
CASE 2

Figure 1
3. K is the grandmother of L.
4. L is 6 years elder than his wife.
From here it is clear that L is male

Page 243 of 1334


Get a Special Paid Bundle PDF on High Level Puzzles & Seating for All Bank Clerk/ PO Mains
If there are any suggestions/ errors in our PDFs Feel Free to contact us via this email: admin@exampundit.in
Ultra Practice Bundle PDF
Reasoning Questions for SBI Clerk/ RBI Assist. Mains
11. H, who is 34 years old and seated second to the right
of K.
12. E is seated between L’s grandmother (K) and wife of
L (J).
13. The Person whose hobby is Football sits opposite to
the person, who is immediate right of F’s husband.
Figure 2
(F’s Husbad is H)
5. The daughter-in-law of I.
Two Cases for K
6. H, who is 34 years old, is son-in-law of I
CASE 1
7. The hobby of H’s brother-in-law is Basketball.
From above two statements it is clear that I have two child.

Figure 3
CASE 2
From Figure 1, 2 and 3 it is clear that I is in 2nd Generation
8. G’s son-in-law, G is the father of F(means G and I are
Husband wife)
9. J’s father-in-law, husband of J(Means J is female)
10. F’s husband (Means F is female )
Final Blood Relation is

14. Each Wife is at least two year younger than her


husband
15. The person whose hobby is Tennis sits second to the
right of the person, who is 80 years old

Page 244 of 1334


Get a Special Paid Bundle PDF on High Level Puzzles & Seating for All Bank Clerk/ PO Mains
If there are any suggestions/ errors in our PDFs Feel Free to contact us via this email: admin@exampundit.in
Ultra Practice Bundle PDF
Reasoning Questions for SBI Clerk/ RBI Assist. Mains
From here it is clear that E Eldest so His age is 80years
and K is 78 years because difference between Husband
and wife is at least 2 years.
CASE 1

CASE 2(Only one couple can sit together so K and E sits


together only)

CASE 2

19. The age of E is 2.5 times the age of F.


From here we can find the age of F as E’s Age is 80

16. The hobby of H’s brother-in-law is Basketball which


is L
17. The person whose hobby Cricket, is immediate
neighbor of the persons, whose hobby is Chess and the
one, who is husband of J.(Husband of J is L)
18. K does not sit opposite to L
CASE 1(Only one couple can sit together so K and E sits
together only)

From above Blood relation diagram it is clear that age


L or J is 36
Page 245 of 1334
Get a Special Paid Bundle PDF on High Level Puzzles & Seating for All Bank Clerk/ PO Mains
If there are any suggestions/ errors in our PDFs Feel Free to contact us via this email: admin@exampundit.in
Ultra Practice Bundle PDF
Reasoning Questions for SBI Clerk/ RBI Assist. Mains
20. The person who is 36 years old is seated second to the
left of J’s father-in-law who is G.
From here Case 2 was eliminated because we cannot
place G and 36.
CASE 1

23. The hobby of the person whose age is 78 years is


neither TT nor Volleyball.(K’s hobby is Hockey)
24. The hobby of L’s grandfather is not Volleyball.
Final Arrangement

21. L is 6 years elder than his wife. (L’s wife is J so L’s


age is 36 so J’s age is 30)
Age of G is 58years because difference between husband
and wife is atleast 2 year and each parent is at least 20
years elder than his/ her child.

31.b
32.c
33.a
34.d
35.e
22. The daughter-in-law of I is not near to her
SOLUTION (36-40):
Only One Couple Sits together so I is not immediate
1. D and C are sitting third and second to the left of A
neighbor to G because K and E sits together
respectively who faces outside.
2. D and G are neighbours of B, who likes AP.

Page 246 of 1334


Get a Special Paid Bundle PDF on High Level Puzzles & Seating for All Bank Clerk/ PO Mains
If there are any suggestions/ errors in our PDFs Feel Free to contact us via this email: admin@exampundit.in
Ultra Practice Bundle PDF
Reasoning Questions for SBI Clerk/ RBI Assist. Mains
3. The one who likes PB sits on the immediate right of CASE 1
B.
4. G is sitting third to the right of A and likes CB.
5. The one who likes AP has a bucket of 5 ltrs
From here we can find that there are 2 cases for A
CASE 1

CASE 2

CASE 2

8. The person who likes IG is immediate neighbour of


the one who sits sitting opposite E.
9. Both the immediate neighbours of E have buckets of
3 ltr Capacity
10. The sum of total capacities of the buckets of the
immediate neighbours of A is greater than 6 but less
6. Two of them are facing outside while rest is facing
than 10.
towards centre.
Here possible sum of immediate neighbor of A is 7, 8 and
7. H is third to the right of E and likes EN.
9 but we cannot make sum 7 and 9 from 3, 5 and 8 so sum
From 6th point A and B faces outside in both cases and all
is 8 only by adding 3 and 5.
other person faces inside.
Page 247 of 1334
Get a Special Paid Bundle PDF on High Level Puzzles & Seating for All Bank Clerk/ PO Mains
If there are any suggestions/ errors in our PDFs Feel Free to contact us via this email: admin@exampundit.in
Ultra Practice Bundle PDF
Reasoning Questions for SBI Clerk/ RBI Assist. Mains
CASE 1 CASE 2

CASE 2

13. A and H are not the neighbor of that person who likes
BJ.
14. The capacity of the bucket of the person who likes IG
is not the highest but greater than that of his immediate
right neighbour.
CASE 1

11. The difference between the volumes of buckets of B


and the one who likes VS is the same as the number of
persons sitting between them.
12. F does not like VS and PB.
CASE 1

CASE 2

Page 248 of 1334


Get a Special Paid Bundle PDF on High Level Puzzles & Seating for All Bank Clerk/ PO Mains
If there are any suggestions/ errors in our PDFs Feel Free to contact us via this email: admin@exampundit.in
Ultra Practice Bundle PDF
Reasoning Questions for SBI Clerk/ RBI Assist. Mains
2. Only two people sit between the Managing Director
and W.
Here two cases for Managing Director.
CASE 1

15. The peeson who like RA does not faces AP.


From here CASE 1 eliminated because RA faces AP.
16. Total volume of all the buckets of these eight friends
is 40 ltr. (total 40 – all bucket’s volume = 8 ltrs for E.) CASE 2
So Final Arrangement

3. The Vice – President and the Company Secretary are


immediate neighbors of each other.
36.d
4. Neither P nor W is a Vice – President or a Company
37.b
Secretary.
38.e
5. Vice –President is not an immediate neighbor of the
39.a
Managing Director.
40.c
6. Here two more Cases for Vice President and Company
SOLUTION (41-45):
Secretary.
1. P sits third to the right of Managing Director.
CASE 1a

Page 249 of 1334


Get a Special Paid Bundle PDF on High Level Puzzles & Seating for All Bank Clerk/ PO Mains
If there are any suggestions/ errors in our PDFs Feel Free to contact us via this email: admin@exampundit.in
Ultra Practice Bundle PDF
Reasoning Questions for SBI Clerk/ RBI Assist. Mains
7. The Manager sits second to the left of T and does not
sit at corner of the table.
8. T is not an immediate neighbor of W.
9. The Manager is an immediate neighbor of the both
Group Leader and the Financial Advisor.
CASE 1a eliminated because 8th point not satisfying.
CASE 2a eliminated because we cannot place T.
CASE 1b

CASE 2a

CASE 2b

10. R sits on the immediate right of the Chairman.


11. P is not the Chairman
12. The Financial Advisor sits third to the right of Q.

Page 250 of 1334


Get a Special Paid Bundle PDF on High Level Puzzles & Seating for All Bank Clerk/ PO Mains
If there are any suggestions/ errors in our PDFs Feel Free to contact us via this email: admin@exampundit.in
Ultra Practice Bundle PDF
Reasoning Questions for SBI Clerk/ RBI Assist. Mains
13. Q is not the Vice – President 20. The person who likes Superman sits exactly in the
From here CASE2b eliminated because we cannot place middle between the persons who like Batman and
R and Chairman. Avatar.

Final Arrangement
14. President faces the person who likes Spider-Man.
15. Q likes Titanic.
16. Financial Advisor faces the person who likes Finding
Nemo.
17. U is not an immediate neighbor of P.
18. V is not an immediate neighbor of the Manager.

41.b
42.a
43.e
44.a
45.d
19. The person who likes Avengers sits immediately right SOLUTION (46-50):
to the person who likes Avatar and faces Jurasic Park.
Page 251 of 1334
Get a Special Paid Bundle PDF on High Level Puzzles & Seating for All Bank Clerk/ PO Mains
If there are any suggestions/ errors in our PDFs Feel Free to contact us via this email: admin@exampundit.in
Ultra Practice Bundle PDF
Reasoning Questions for SBI Clerk/ RBI Assist. Mains
1. The student who is from IIT Guwahati is sitting on the 5. No two students of the same subject group are sitting
immediate right of the student who is from IIT Indore. adjacent to each other except those who belong to
2. Only IIT Chennai Student S is sitting between the English group.(From here it is clear English group
participant W, who is from Guwahati and IIT Goa sitting next to each other)
participant 6. U, who is in the IIT Bhilai, is in General Knowledge
3. T is sitting opposite to W. group and she is sitting on the immediate right of Y,
4. T who is from IIT Delhi is neither in reasoning group who is in group English.
nor in the subject group of General Knowledge and 7. Y is neither from IIT Kharagpur nor IIT Indore.
Math’s.
Here two cases for Indore and Guwahati

8. V is neither in IIT Indore nor in IIT kharagpur and


does not sit at any corners.

Page 252 of 1334


Get a Special Paid Bundle PDF on High Level Puzzles & Seating for All Bank Clerk/ PO Mains
If there are any suggestions/ errors in our PDFs Feel Free to contact us via this email: admin@exampundit.in
Ultra Practice Bundle PDF
Reasoning Questions for SBI Clerk/ RBI Assist. Mains
9. X is not the neighbour of V.
From here Case 1 eliminates because Place for V is at
corners

46. a
10. Both the students of subject group Mathematics are 47. d
sitting Immediate Right of the students of subject 48. a
group Reasoning. 49. b
11. The students of Mathematics group do not sit at 50. e
middle of the side.

Final Arrangement

Download Seating Arrangement Practice Questions PDF


Get More Reasoning Practice Questions PDF
Page 253 of 1334
Get a Special Paid Bundle PDF on High Level Puzzles & Seating for All Bank Clerk/ PO Mains
If there are any suggestions/ errors in our PDFs Feel Free to contact us via this email: admin@exampundit.in
Ultra Practice Bundle PDF
Reasoning Questions for SBI Clerk/ RBI Assist. Mains
Seating Rectangle / Square New Pattern
Directions: Answer the given questions based on the red color and both sit on the same side. V has 2 more balls
information given below. than T, who does not face G. The one, who has 4 balls,
Sixteen persons are sitting around the two square tables. faces B, who likes Pink. S is not adjacent to P, who is not
One square table is inscribed in another. Two persons sit to the immediate right of T. G does not like Grey or Blue
on each side of the table. The persons on the inner table color.
have different number of balls while the persons on outer 1) Who among the following likes black color?
table like different colours. P, Q, R, S, T, U, V and W sit a. F
on one table while A, B, C, D, E, F, G and H sit on another b. H
table. Each of the persons on both tables is facing the c. G
closest person opposite to them on different tables such d. C
that the persons on inner table are facing outwards while e. None of these
persons on outer table are facing inwards. Persons on 2) How many persons sit between P and V when
inner table have different number of balls from 1 to 8. counted from the left of P?
Also, persons on outer table like different colours among, a. Three
red, black, pink, yellow, white, grey, blue and green. b. Two
Note: If A is facing B then both are sitting opposite to each c. Four
other on the different tables but on the nearest seat. d. Five
p sits opposite to one, who sits to the immediate left of R, e. None of these
who has 3 balls. Both P and R are sitting on the opposite 3) Who among the following faces S?
sides of the same table. The persons facing R is three a. The one, who likes green color
places away from D. W has twice number of balls of S, b. B
who has 3 balls more than Q. Q faces E, who likes yellow c. The one, who likes blue color
color. U sits to the immediate right of V, who has 7 balls, d. A
on the same side. A, who likes white color, faces U, who e. None of these
has 6 balls. E does not sit adjacent to A or C and Q is not 4) Who among the following sits second to the right of
adjacent to R. C, who likes green color, sits second to the T?
right of A. G sits to the immediate right of F, who likes a. W

Page 254 of 1334


Get a Special Paid Bundle PDF on High Level Puzzles & Seating for All Bank Clerk/ PO Mains
If there are any suggestions/ errors in our PDFs Feel Free to contact us via this email: admin@exampundit.in
Ultra Practice Bundle PDF
Reasoning Questions for SBI Clerk/ RBI Assist. Mains
b. The one, who has 3 balls Immediate neighbors of D are facing the opposite
c. The one, who has 7 balls direction of D. Person sitting opposite to A is not facing
d. The one, who has 4 balls away from the centre. C is the only brother of A. A is
e. None of these married.
5) Who likes grey color? B 3@ A# G%
a. H 6) How many persons are sitting between B and C,
b. D when counted from the left of B?
c. G a. 1
d. F b. 3
e. Cannot be determined c. 4
Directions: Answer the questions based on the d. 5
information given below. e. None of these
Eight members, A, B, C, D, E, F, G and H of a family are 7) Find the odd one out.
sitting at the corners and on the middle of the sides of a a. EA
square. The number of persons facing the centre is less b. DF
than those who are facing away from the centre. There is c. GE
no single parent in the family. d. BG
A 3@ means person sitting 3rd to the right of A is the e. CH
father of A. 8) What is the position of C’s father with respect to B’s
A # means A is sitting opposite to his/her mother. grandmother?
A% means Person sitting opposite to A is not the male a. 3rd to the right
member. b. 4th to the left
Two persons are sitting between F and C, who is not c. 2nd to the right
sitting at the corners and both of them are facing in the d. 3rd to the left
opposite directions from each other. E is sitting 2nd to the e. None of these
right of C. D is sitting 2nd to the right of E, who is married 9) Who is sitting opposite to the father in law of the
to F. One person is sitting between H and G, who is sitting only brother in law of B?
adjacent to B, who is sitting opposite to his only sister. H a. A
is not sitting adjacent to D. A is not sitting adjacent to B. b. B
Page 255 of 1334
Get a Special Paid Bundle PDF on High Level Puzzles & Seating for All Bank Clerk/ PO Mains
If there are any suggestions/ errors in our PDFs Feel Free to contact us via this email: admin@exampundit.in
Ultra Practice Bundle PDF
Reasoning Questions for SBI Clerk/ RBI Assist. Mains
c. G direction. Immediate neighbors of H are facing the
d. H opposite direction of H. B is sitting 2nd to the right of H.
e. F Distance between the persons sitting on the side opposite
10) How is E related to the person sitting opposite to to the side on which G is sitting is increasing
D? consecutively from 6. Both the immediate neighbors of E
a. Daughter are sitting at the corners and E is sitting exactly in the
b. Niece middle of one of the sides. L is sitting adjacent to B and
c. Daughter in law the distance between them is 9m. A is sitting opposite to
d. Sister D. F is sitting 2nd to the right of K. F and I are sitting on
e. None of these the same side. D is sitting adjacent to I. Distance between
Directions: Read the following information and D and F is 20m. Distance between A and G is equal to the
answer the questions given below. distance between F and I. Immediate neighbors of A are
A certain number of persons are sitting around a square facing the opposite direction of A. G is sitting 2nd to the
table. Four persons are sitting at the corners and others are right of E.
sitting at the sides of the table. Number of the persons 11) What is the position of F with respect to H?
facing the centre is equal to the number of the persons a. 3rd to the left
facing away from the centre. Three persons are sitting on b. 4th to the right
at least one of the sides. The area of the square is 900m2. c. 2nd to the left
Only one person sitting at the corners is facing away from d. 5th to the right
the centre. e. None of these
Distance between I and one of the corners of the side on 12) How many persons are sitting on the side of the
which I is sitting is 10m and there is no one sitting between table on which L is sitting excluding L?
them. Distance between K and G is 2m more than the a. 2
distance between I and F. K is the immediate neighbor of b. 1
G but both of them doesn’t sit at the corner. One of the c. 3
immediate neighbors of K is sitting at the corner. Both G d. Either (a) or (b)
and K are facing the direction as E. Two persons are sitting e. None of these
on the side, which is opposite to E. H is sitting 2nd to the 13) How many persons are sitting on the side of the
left of D and both of them are facing the opposite table on which H is sitting?
Page 256 of 1334
Get a Special Paid Bundle PDF on High Level Puzzles & Seating for All Bank Clerk/ PO Mains
If there are any suggestions/ errors in our PDFs Feel Free to contact us via this email: admin@exampundit.in
Ultra Practice Bundle PDF
Reasoning Questions for SBI Clerk/ RBI Assist. Mains
a. 1 A and F sit together facing in the south direction. F wears
b. 2 red shirt and A does not wear orange coloured shirt. G
c. 3 wears black and sits next to the person wearing yellow
d. Either (a) or (b) coloured shirt. H wears white shirt and is sitting between
e. None of these the persons wearing blue and orange coloured shirts. E sits
14) How many persons are sitting on the sides of the opposite to A and the person next to E wears green
table? coloured shirt.
a. 11 16) If person wearing pink shirt sits to the right of the
b. 12 person wearing orange shirt then who sits to the
c. 14 immediate left of the person wearing black shirt?
d. 9 a. F
e. 8 b. B
15) What is the total distance between A and L to the c. H
right of A? d. E
a. 41 e. None of these
b. 42 17) Who sits to the immediate left of the one who wears
c. 34 green shirt if A sits to the immwdiate right of F?
d. 39 a. A
e. 37 b. B
Directions 16-20: Answer the questions based on the c. C
information given below: d. D
Eight persons A, B, C and D (Boys) and E, F, G and H e. E
(girls) go out for a restaurant and sits around a square 18) Which direction is G facing if he is seated to the
table.Two people sit on each side of the table such that one immediate left of B?
is a boy and the other one is a girl. Also each person wears a. North
a shirt of different colour viz. red, blue, black, white, b. East
yellow, orange, green and pink. c. West
B and C sit next to each other. D sits opposite to B, d. South
between the persons with gender opposite to that of him. e. Cannot be determined
Page 257 of 1334
Get a Special Paid Bundle PDF on High Level Puzzles & Seating for All Bank Clerk/ PO Mains
If there are any suggestions/ errors in our PDFs Feel Free to contact us via this email: admin@exampundit.in
Ultra Practice Bundle PDF
Reasoning Questions for SBI Clerk/ RBI Assist. Mains
19) How many people sit between F and the one who is 2. Q is sitting third to the right of the person who is a fan
wearing blue shirt on the same side? of Thor.
a. Zero 3. The sum of distances between Q’s neighbours is 91
b. One units.
c. Two 4. The distance between V and the person who is a fan of
d. Three Superman is 64 units and there is no one between them.
e. Four Also, V is not a fan of Captain America.
20) Which coloured shirt is the person wearing who is 5. The persons who are fans of Aquaman and Hulk are
sitting diagonally opposite to D? sitting opposite to each other.
a. Red 6. W is a fan of Ironman but he is not sitting next to the
b. Black person who is a fan of Thor or Superman.
c. Yellow 7. The distance between the persons who are fans of
d. Orange Superman and Batman is 27 units which are 11 units more
e. None than the distance between the persons who are fans of
Directions: Study the following information carefully Batman and Aquaman. There is no one between superman
and answer the questions that follow: and batman as well as batman and Aquaman.
Eight people P, Q, R, S, T, U, V and W are sitting in a 8. The distance between P and the person who is a fan of
rectangular manner and facing the centre and 4 sits at the Captain America is minimum among them.
corner of the table while 4 sits at the middle of the table. 9. Three persons are sitting between S and R but S is not
Each of them is fans of different superheroes among a fan of Hulk.
Aquaman, Batman, Captain America, Hulk, Iron man, 10. The distances of R with his neighbours are 27 units
Superman, Thor and Wonder Woman. The distance and 16 units and U is not a fan of Batman.
between them is either single or two digits perfect square 21) Who among the following is the fan of Wonder
or perfect Cube. Also, all the distances are greater than 3 Woman?
units. a. R
Further, it is given that b. V
1. The distance between the persons who are fans of Thor c. W
and Aquaman is both a perfect square and a perfect Cube. d. P
e. Can't be determined
Page 258 of 1334
Get a Special Paid Bundle PDF on High Level Puzzles & Seating for All Bank Clerk/ PO Mains
If there are any suggestions/ errors in our PDFs Feel Free to contact us via this email: admin@exampundit.in
Ultra Practice Bundle PDF
Reasoning Questions for SBI Clerk/ RBI Assist. Mains
22) What is the minimum distance between the persons Directions 26-30: Study the following information
who are fans of Superman and Aquaman? carefully to answer the given questions:
a. 43 Eight persons are sitting at the corners and on the middle
b. 27 of the sides of a rectangluar arrangement. They are facing
c. 49 different directions – 5 facing away from the center and 3
d. 130 facing towards the center. They belong to eight different
e. None of the above states - Haryana, Gujarat, Punjab, UP, MP, HP, Kerala and
23) Who is sitting second to the left of the person who Delhi. They play eight different sports- Basketball,
is a fan of Thor? Football, Golf, Hockey, Badminton, Cricket, Tennis and
a. Q Volleyball. They have visited eight different rivers - Nile,
b. W Amazon, Thames, Yangtze, Danube, Ganga, Yamuna and
c. T Brahmaputra.
d. S E is sitting opposite to F but not facing each other.B is
e. R sitting opposite to H and not facing each other. F doesn’t
24) If U and R exchange their positions with Q and T play Tennis, Badminton and Cricket. The person sitting
respectively then who will be sitting second to the right adjacent to G is from UP and plays Volleyball. C sits
of R? second to adjacent of F in clockwise direction. The person
a. T sitting adjacent to H plays Tennis. G sits opposite to C and
b. Q are facing each other.The Basketball player sits between
c. R C and E. H is sitting between C and F. The person sitting
d. W between C and F is from Kerala and plays Hockey. B and
e. None of the above D are not from UP or HP. The persons sitting adjacent to
25) How many persons are sitting between P and V F faces same direction as B. H sits opposite to cricket
when counted from the left side of V? player. The tennis player is from MP. The Delhi player
a. Two sits between persons from Punjab and UP. The Basketball
b. One player sits adjacent to Football player who is from
c. Five Gujarat. The tennis player has visited River Thames. The
d. Three person from Punjab has visited River Yangtze and person
e. Four from HP has visited River Amazon. The hockey player
Page 259 of 1334
Get a Special Paid Bundle PDF on High Level Puzzles & Seating for All Bank Clerk/ PO Mains
If there are any suggestions/ errors in our PDFs Feel Free to contact us via this email: admin@exampundit.in
Ultra Practice Bundle PDF
Reasoning Questions for SBI Clerk/ RBI Assist. Mains
has visited Yamuna. E and D has not visited Nile, Yamuna a. A
or Brahmaputra. B sits between persons who have visited b. C
Nile and Ganga. D faces inside. Only one person sits in c. G
the corner facing inside. d. E
26) Who has visited River Ganga? e. D
a. F Directions 31-35: Study the following information
b. A carefully to answer the given questions:
c. Football player Eight friends A, B, C, D, E, F, G and H are sitting around
d. Person from Punjab a rectangular table, but not necessarily in the same order.
e. G All friends are facing the centre and four of them sits at
27) Who sits between B and A? the corner while another four sits middle of the table. Each
a. E person has different salaries i.e.15000, 19000, 16000,
b. Person who has visited River Brahmaputra 32000, 12000, 70000, 40000, and 60000 and are of
c. Person from UP different ages 20, 26, 56, 40, 34, 54, 43 and 30 years but
d. F not necessarily in the same order.
e. Person who has visited River Nile There is an angle of 135° between A and G.B’s salary is
28) Which of the following pair is incorrect? double of H’s but less than D’s salary. There is an angle
a. F – Tennis of 90° between A and H. F sits at 135° clockwise direction
b. H – Yamuna to B. The average of B and D’s age is equal to the age of
c. A – UP G. Salary of the one, who sits 2nd to the right of B is
d. C – Thames divisible by 4000. D is not an immediate neighbor of the
e. D – Basketball person who is third to the left of A. G’s salary is thousand
29) Which of the following statement is correct? rupees less than 1/3rd of A’s salary. There is an angle of
a. E is a football player from Punjab 180° between B and the one whose age is 54 years. C’s
b. F is from HP and has visited River Amazon. salary is third highest salary among all the salaries. There
c. C is a Tennis player from UP. is an angle of 180° between F and E, whose salary is
d. B has visited River Brahmaputra. divisible by 3000 but not 12000. B sits at 90° clockwise
e. A is a Volleyball player from Haryana. direction of A. E is the youngest in the group. Only two
30) Who is a Badminton player? pair is sitting diagonally opposite to each other has age
Page 260 of 1334
Get a Special Paid Bundle PDF on High Level Puzzles & Seating for All Bank Clerk/ PO Mains
If there are any suggestions/ errors in our PDFs Feel Free to contact us via this email: admin@exampundit.in
Ultra Practice Bundle PDF
Reasoning Questions for SBI Clerk/ RBI Assist. Mains
difference of 6 years. The salary of C is 1000 multiple of b. 20,000
its age. The one who have the highest salary is third c. 30.000
youngest in the group. d. 28,000
31) Who is sitting immediately left of G? e. 15,000
a. C Directions 36-40: Answer the questions based on the
b. D information given below:
c. A There are eight members P, Q, R, S, T, U, V and W in a
d. W family which consists of three generations and two
e. B married couples on each in first and second generation.
32) Whose salary is least among the eight persons? Each of these members sits around a square shaped table
a. B such that four sit at the corners and four along the middle
b. G of the edges facing towards the centre. Each of them has
c. E different hobbies among Writing, Reading, Gardening,
d. H Dancing, Swimming, Surfing, Mountaineering and
e. F Playing.
33) Which is the correct combination? The son-in-law of the person, who likes writing, sits to the
a. D – 70000 – 30 immediate left of Q and along one of the edges. Q is
b. A – 15000 – 20 married but doesn’t have any daughter. Mother of the
c. E – 32000 – 56 person who likes Gardening, sits 3rd to the right of the one
d. H – 12000 – 26 who likes writing. Father of the person who likes
e. None of these Gardening, sits 2nd to the right of V, who is a female. V
34) What is B’s age? sits at one of the corners of the table. S is the only brother
a. 56 years of V and sits to the immediate left of V. One person sits
b. 43 years between S and W. P sits to the immediate left of the
c. 34 years person, who likes Reading who has only two child. U sits
d. 54 years opposite to P. W is one of the two sons of the person, who
e. 26 years likes Reading. T likes Swimming. Person who likes
35) What is the difference in salary of B and A? Surfing sits to the immediate right of T. Two persons sit
a. 10,000 between the persons who like Surfing and
Page 261 of 1334
Get a Special Paid Bundle PDF on High Level Puzzles & Seating for All Bank Clerk/ PO Mains
If there are any suggestions/ errors in our PDFs Feel Free to contact us via this email: admin@exampundit.in
Ultra Practice Bundle PDF
Reasoning Questions for SBI Clerk/ RBI Assist. Mains
Mountaineering. Neither S nor V likes Dancing. If S is e. None of these.
present in the generation of the one who likes writing, then 40) The person who sits opposite to T likes ____.
there is only one person in 3rd generation and two males a. Playing
in 2nd generation. P is father of guardning b. Surfing
36) How is U related to the person who likes c. Mountaineering
Mountaineering? d. Gardening
a. Son e. None of these
b. Brother Directions 41-45: Study the following information
c. Father carefully to answer the given questions:
d. Nephew Eight friends - A, B, C, D, E, F, G, and H - are sitting
e. None of these around a rectangular table, but not necessarily in the same
37) What is the position of S with respect to the person order, four of them are facing inside and the other four are
who likes Dancing? facing outside and four of them are sitting at the corner
a. Immediate right and the other four are sitting at the middle. All eight
b. 2nd to the left friends work in different companies – Google, Walmart,
c. 3rd to the right Intel, Amazon, L&T, Dell, Reliance, and Samsung but not
d. 2nd to the right necessarily in the same order. They live in the same
e. None of these. building on different floors from 1st to 8th floor but not
38) Who sits opposite to the person who likes playing? necessarily in the same order.
a. Q The adjacent members do not face the same direction. The
b. T person who lives on the 4th floor works at L&T and is
c. R sitting third to the right of C. B lives on the 1st floor and
d. Can’t be determined. is facing towards the center. The person who is working
e. None of these. at Reliance lives on the 5th floor and sits opposite A, who
39) How is the person who likes Surfing related to P? lives immediately below him. B and C are the neighbours
a. Son of H in the square table. The person who lives on the 3rd
b. Grand son floor is sitting opposite to the person who works in Intel.
c. Nephew E who lives two floors above B is sitting second to the left
d. Wife of B and works at Google. The persons who are working
Page 262 of 1334
Get a Special Paid Bundle PDF on High Level Puzzles & Seating for All Bank Clerk/ PO Mains
If there are any suggestions/ errors in our PDFs Feel Free to contact us via this email: admin@exampundit.in
Ultra Practice Bundle PDF
Reasoning Questions for SBI Clerk/ RBI Assist. Mains
at Amazon and Dell are sitting opposite to each other. D c. F
who lives two floors below A works at Samsung and is a d. G
neighbour of A. F who works at Dell does not sit between e. C
B and E and he does not live on the top floor. The person
working at Amazon lives on the 6th floor. A sits at the 45) Which of the following sits second to the left of the
corner of the table person who lives on the 2nd floor?
41) Which of the combination is correct? a. The person who works at Google.
a. A - 7th floor - L&T b. The person who lives on the 6th floor.
b. G - 6th floor - Samsung c. The person who works at Intel.
c. F - 5th floor - Dell d. The person who lives on the 8th floor.
d. D - 2nd floor - Samsung e. Both c and d
e. None of these Directions 46-50: Study the following information
42) Four of the following five are alike. Which of the carefully to answer the given questions:
one does not follow the group? Eight students - A, B, C, D, E, F, G and H are sitting
a. B around a rectangular table, but not necessarily in the same
b. F order, four of them are facing inside and the other four are
c. A facing outside. They got different ranks in the IIT JEE
d. E exam from 1st to 8th and got the admission in different
e. H IIT’s viz. Kharagpur, Patna, Delhi, Kanpur, Mumbai,
43) What is the position of the person living on the 8th Madras, Roorkee and Mandi but not necessarily in the
floor with respect to the person working at Amazon? same order. Four of them are sitting in the corner and the
a. 3rd to the left other four are sitting in the middle of the table.
b. 2nd to the right D who is facing towards the centre is sitting second to the
c. Opposite left of G. The one who got 1st rank got admitted to IIT
d. Immediate left Kanpur. E who got 3rd rank is sitting third to the right of
e. 2nd to the left C. B got admitted to IIT Delhi and sitting adjacent to F
44) Which of the following friend works at Walmart? who got 7th rank. Both B and F are facing the same
a. D direction towards the centre. G is facing outwards and got
b. B admitted to IIT Mumbai. The one who got 2nd rank is
Page 263 of 1334
Get a Special Paid Bundle PDF on High Level Puzzles & Seating for All Bank Clerk/ PO Mains
If there are any suggestions/ errors in our PDFs Feel Free to contact us via this email: admin@exampundit.in
Ultra Practice Bundle PDF
Reasoning Questions for SBI Clerk/ RBI Assist. Mains
sitting second to the right of D who got 8th rank and D got d. The person who got 5th rank.
admitted to IIT Patna. C is sitting between G and D and F e. H
is sitting fourth to the right of C who faces outside. H faces 48) Which of the following combinations is correct?
the same direction as C and he is not the neighbour of G. a. B - 2nd - Patna
The one who got 6th rank and got admitted to IIT b. G - 3rd - Mumbai
Kharagpur is sitting to the immediate left of the person c. A - 6th - Mandi
who got admitted to IIT Madras. C is not the topper but he d. C - 5th - Roorkee
got admitted to IIT Roorkee. G got the 4th rank. B sits at e. None of these
the corner of the table 49) Four of the following are alike in some manner.
46) Who among the following students got admitted to Which one does not belong to the group?
IIT Mandi? a. C
a. A b. D
b. B c. E
c. F d. G
d. G e. H
e. H 50) What is the rank of H?
47) Who among the following students sits to the a. 6th
immediate left of A? b. 5th
a. The person who got admitted to IIT Madras. c. 1st
b. B d. 8th
c. G e. None of these

Seating Rectangle / Square New Pattern - Answers and Explanations


1. c 1. P sits opposite to one, who sits immediate left of R, who
2. b has 3 balls.
3. b Also, both sit on same table.
4. d 2. The persons facing R is three places away from D.
5. e 3. W has twice number of balls than that of S, who has 3
Solutions: 1-5 balls than Q.

Page 264 of 1334


Get a Special Paid Bundle PDF on High Level Puzzles & Seating for All Bank Clerk/ PO Mains
If there are any suggestions/ errors in our PDFs Feel Free to contact us via this email: admin@exampundit.in
Ultra Practice Bundle PDF
Reasoning Questions for SBI Clerk/ RBI Assist. Mains
So, Q, S and W have 1, 4 and 8 balls respectively.
4. Q faces E, who likes yellow color.
5. U sits immediate right of V, who has 7 balls, on the
same side.
6. A, who likes white color, faces U, who has 6 balls. C,
who likes green color, sits second to the right of A.
7. G sits to the immediate right of F, who likes red color
and both sit on the same side.
So, we have four cases,

8. E does not sit adjacent to A or C and Q is not adjacent


to R.
So, case 2(a) is rejected.
9. V has 2 more balls than T, who does not face G.
So, case 1(a) and 2(b) are rejected.
10. The one, who has 4 balls, faces B, who likes Pink.
11. S is not adjacent to P, who is not immediate right of T.
12. G does not like Grey or Blue color.
Page 265 of 1334
Get a Special Paid Bundle PDF on High Level Puzzles & Seating for All Bank Clerk/ PO Mains
If there are any suggestions/ errors in our PDFs Feel Free to contact us via this email: admin@exampundit.in
Ultra Practice Bundle PDF
Reasoning Questions for SBI Clerk/ RBI Assist. Mains
So, we have, Case 1: When F is facing away from the centre and C is
sitting 3rd to the right of F. A is not sitting adjacent to B,
so this case is not possible.
When G is sitting adjacent to D, then person sitting
opposite to A is facing away from the centre, so this case
is not possible.

Answers: 6-10
6. D
7. C
8. A
9. E 6. Immediate neighbors of D are facing the opposite
10. B direction of D.
Solutions 6-10 Case 1(a): When F is facing away from the centre and C
1. Two persons are sitting between F and C, who is not is sitting 3rd to the left of F. Person sitting opposite of A is
sitting at the corners and both of them are facing the not facing away from the centre, so this case is not
opposite directions. possible.
2. E is sitting 2nd to the right of C.
3. D is sitting 2nd to the right of E.
4. One person is sitting between H and G, who is sitting
adjacent to B. who is sitting opposite to his only sister.
5. H is not sitting adjacent to D.

Page 266 of 1334


Get a Special Paid Bundle PDF on High Level Puzzles & Seating for All Bank Clerk/ PO Mains
If there are any suggestions/ errors in our PDFs Feel Free to contact us via this email: admin@exampundit.in
Ultra Practice Bundle PDF
Reasoning Questions for SBI Clerk/ RBI Assist. Mains
Case 2(a): When F is facing the centre and C is sitting
3rd to the left of F.
The final arrangement is as follows:

7. B is sitting opposite to his only sister, so E must be the


sister of B.
8. E is married to F.
9. C is the only brother of A.
B 3@ - Person sitting 3rd to the right of B is the father of
B.
The final family tree diagram is as follows:
A# - Person sitting opposite to A is the mother of A, i.e.
G.
G% - Person sitting opposite to G is not a male member.
Case 2: when F is facing the centre and C is not sitting 3rd
to the right of F, so A nor F can be the father of B, so this
case is not possible.

Answers:
11. b
12. a
13. c
14. e
15. d
Solutions: 11-15

Page 267 of 1334


Get a Special Paid Bundle PDF on High Level Puzzles & Seating for All Bank Clerk/ PO Mains
If there are any suggestions/ errors in our PDFs Feel Free to contact us via this email: admin@exampundit.in
Ultra Practice Bundle PDF
Reasoning Questions for SBI Clerk/ RBI Assist. Mains
1. Both the immediate neighbors of E are sitting at the
corners and E is sitting exactly in the middle of one of the
sides.
2. Two persons are sitting on the side, which is opposite
to E.
3. G is sitting 2nd to the right of E.

4. K is the immediate neighbor of G but doesn’t sit at the


corner. One of the immediate neighbors of K is sitting at
the corner. Both G and K are facing the direction as E.
Case 2: When E is facing away from the centre.
5. F is sitting 2nd to the right of K. F and I is sitting on the
The final arrangement is as follows:
same side. D is sitting adjacent to I. Distance between D
and F is 20m. Distance between I and one of the corner of
the side on which I is sitting is 10m. Distance between K
and G is 2 more than the distance between I and F. H is
sitting 2nd to the left of D and both of them are facing the
opposite direction.
6. Immediate neighbors of H are facing the opposite
direction of H. B is sitting 2nd to the right of H. Distance
between the persons sitting on the side opposite to the side
on which G is sitting is increasing consecutively. L is
sitting adjacent to B and the distance between them is 9m.
Answers:
A is sitting opposite to D.
16) B
7. Distance between A and G is equal to the distance
17) E
between F and I.
18) B
Case 1: When E is facing the centre. Immediate neighbors
19) A
of A are facing the opposite direction of A, so this case is
20) B
not possible.
Solutions 16-20

Page 268 of 1334


Get a Special Paid Bundle PDF on High Level Puzzles & Seating for All Bank Clerk/ PO Mains
If there are any suggestions/ errors in our PDFs Feel Free to contact us via this email: admin@exampundit.in
Ultra Practice Bundle PDF
Reasoning Questions for SBI Clerk/ RBI Assist. Mains
1) A and F sit together facing in the south direction.
2) F wears red shirt and A does not wear orange coloured
shirt.

6)H wears white shirt and is sitting between the persons


wearing blue and orange coloured shirts.
(Therefore, C will be wearing Green shirt according to
statement 3)
3) E sits opposite to A and the person next to E wears
green coloured shirt.

7) G wears black and sits next to the person wearing


yellow coloured shirt.
Now only colour left for E is Pink.
4) D sits opposite to B, between the persons with gender
opposite to that of him.
(D is a male therefore, he will be seated between 2
females)
5) B and C sit next to each other.
(Now as C is a male, the only place left for C is the edge
where E is seated)
Answers:
Page 269 of 1334
Get a Special Paid Bundle PDF on High Level Puzzles & Seating for All Bank Clerk/ PO Mains
If there are any suggestions/ errors in our PDFs Feel Free to contact us via this email: admin@exampundit.in
Ultra Practice Bundle PDF
Reasoning Questions for SBI Clerk/ RBI Assist. Mains
21. b 7. The distance between V and the person who is a fan of
22. a Superman is 64 units. Also, V is not a fan of Captain
23. b America.
24. a
25. d
Solutions 21-25
1. The distance between the persons who are fans of Thor
and Aquaman is both a perfect square and a perfect Cube.
2. Q is sitting third to the right of the person who is a fan 8. Three persons are sitting between S and R but S is not

of Thor. a fan of Hulk.


9. The distances of R with his neighbours are 27 units and
16 units and U is not a fan of Batman.

3. The persons who are fans of Aquaman and Hulk are


sitting opposite to each other.
10. The distance between P and the person who is a fan of
4. The distance between the persons who are fans of
Captain America is 16 units .
Superman and Batman is 27 units which are 11 units more
than the distance between the persons who are fans of
Batman and Aquaman.

5. W is a fan of Ironman but he is not sitting next to the


person who is a fan of Thor or Superman.
6. The sum of distances of Q with his neighbours is 91 Answers:
units. 26. c
Page 270 of 1334
Get a Special Paid Bundle PDF on High Level Puzzles & Seating for All Bank Clerk/ PO Mains
If there are any suggestions/ errors in our PDFs Feel Free to contact us via this email: admin@exampundit.in
Ultra Practice Bundle PDF
Reasoning Questions for SBI Clerk/ RBI Assist. Mains
27. e 8. The person sitting adjacent to H plays Tennis.
28. a 9. F doesn’t play Tennis, Badminton or Cricket.
29. b So, C is a tennis player.
30. c 10. The Basketball player sits between C and E.
Solutions 26-30
1. E is sitting opposite to F but not facing each other.
2. C sits second to adjacent of F in clockwise direction.
3. G sits opposite to C and are facing each other.
4. H is sitting between C and F.

11. The persons sitting adjacent to F faces same direction


as A.
As 3 people are facing towards the center of the circle.
5. B is sitting opposite to H and not facing each other. 12. The person sitting between C and F is from Kerala and
6. The person sitting adjacent to G is from UP and plays plays Hockey.
Volleyball. 13. The Basketball player sits adjacent to Football player
7. B and D are not from UPor HP. from Gujarat.
So, person sitting between F and G is from UP and plays So, E is a football player from Gujarat.
Volleyball. As D is not from UP so D sits between C and
E and A sits between G and F.

Page 271 of 1334


Get a Special Paid Bundle PDF on High Level Puzzles & Seating for All Bank Clerk/ PO Mains
If there are any suggestions/ errors in our PDFs Feel Free to contact us via this email: admin@exampundit.in
Ultra Practice Bundle PDF
Reasoning Questions for SBI Clerk/ RBI Assist. Mains

14. H sits opposite to cricket player.


As only games left are Golf and Badminton and F is not a
Badminton player so F is a golf player and G is a
Badminton player.
15. The Delhi player sits between persons from Punjab
and UP.
So, G is from Delhi and B is from Punjab.
16. The tennis player is from MP. 20. B sits between persons who have visited Nile and
As only two countries are left: HP and Haryana. As, D is Ganga.
not from HP so D is from Haryana and F is from HP. So, E has visited Ganga, G has visited Nile and D has
visited Danube.
21. The hockey player has visited Yamuna.
So, H has visited Yamuna and A has visited Brahmaputra.

17. The person from Punjab has visited River Yangtze and
person from HP has visited River Amazon.
18. E and D has not visited Nile, Yamuna or Brahmaputra. Answers:
19. The tennis player has visited River Thames. 31. a
So, C has visited River Thames. And D and E has visited 32. e
either Danube or Ganga. 33. a
34. a
35. d

Page 272 of 1334


Get a Special Paid Bundle PDF on High Level Puzzles & Seating for All Bank Clerk/ PO Mains
If there are any suggestions/ errors in our PDFs Feel Free to contact us via this email: admin@exampundit.in
Ultra Practice Bundle PDF
Reasoning Questions for SBI Clerk/ RBI Assist. Mains
Solutions 31-35 9) G’s salary is thousand rupees less than 1/3rd of A’s
1) F sits at 135˚ clockwise direction to B. salary i.e. 60000/3 = 20000, 20000 – 1000 = 19000.
2) Salary of the one, who sits 2nd to right of Bis divisible 10) There is an angle of 180˚ between B and the one whose
by 4000. Here, the probabilities are (16000, 32000, 12000, age is 54 years.
40000, 60000) 11) D is not an immediate neighbour of the person who is
3) There is an angle of 180˚ between F and E. third to the left of A.
4) B sits at 90˚ clockwise direction of A i.e. A is 90˚ anti- Therefore, only place left for D is between A and H and
clockwise direction of B. for C is between G and F.
5) E is the youngest in the group. Therefore, E is of 20 12) C’s salary is third highest salary among all the salaries
years. i.e. 40000.

6) There is an angle of 90˚ between A and H.


7) There is an angle of 135° between A and G.
8) Only two pair is sitting diagnolly opposite to each other 13) E’s salary is divisible by 3000 but not 12000.
has age difference of 6 years. 14) B’s salary is double of H’s but less than D’s salary.
(Such two age pairs are 20, 26 and 30, 44) (32,000 is double of 16000 but less than 70000,
furthermore 12000 which is left is F’s salary).
15) The salary of C is 1000 multiple of its age. (Therefore,
age of C is 40 years, and using point 8) the age difference
of 6 to 34 is 38, therefore age of A is 34 years)

Page 273 of 1334


Get a Special Paid Bundle PDF on High Level Puzzles & Seating for All Bank Clerk/ PO Mains
If there are any suggestions/ errors in our PDFs Feel Free to contact us via this email: admin@exampundit.in
Ultra Practice Bundle PDF
Reasoning Questions for SBI Clerk/ RBI Assist. Mains
There is one couple is in 1st generation and one couple is
in 2nd generation. The son in law of the person, who likes
writing, sits to the immediate left of Q and along one of
the edges. Q is married but doesn’t have any daughter. Q
is in 2nd generation. S is the only brother of V, who is a
female. S has to be in 1st or 2nd generation. If S is present
in the generation of the one who likes writing, then there
is only one person in 3rd generation and two males in
2nd generation. If S is in 1st generation then W cannot be
16) The one who have the highest salary is third youngest in 2nd or 3rd generation so S is in 2nd generation.
in the group. (Therefore, D is of 30 years) As the son in law of the person, who likes writing, sits to
17) The average of B and D’s age is equal to the age of G. the immediate left of Q and along one of the edges.
((56 + 30) /2 = 43 years) Q is married but doesn’t have any daughter, so Q must be
the daughter of the person, who likes writing. Let the son
in law of the person, who likes writing be denoted as 1, so
Seating Arrangement:

Answers:
36. b Family tree:
37. c
38. d
39. b
40. c
Solutions 36-40
Page 274 of 1334
Get a Special Paid Bundle PDF on High Level Puzzles & Seating for All Bank Clerk/ PO Mains
If there are any suggestions/ errors in our PDFs Feel Free to contact us via this email: admin@exampundit.in
Ultra Practice Bundle PDF
Reasoning Questions for SBI Clerk/ RBI Assist. Mains
Mother of the person, who likes Gardening, sits 3rd to the S is the only brother of V and sits immediate left of V.
right of the one who likes Writing. One person sits between S and W.
Father of the person, who likes Gardening, sits 2nd to the W is one of the two sons of the person who likes Reading,
right of V, who is a female. so either Q or her husband likes Reading.
Case I: The person, who likes Gardening, is in the Seating Arrangement:
1st generation.
Q is the mother of the person, who likes Gardening. V
likes Writing.

Family Tree:

Case II: The person, who likes Gardening, is in the


2nd generation.
Let the mother of the person, who likes Gardening be
denoted as 2.

P sits immediate left of the person who likes Reading.


U sits opposite to P, so P likes Writing and the husband of
Q likes Reading.
T likes Swimming, so T is the mother of Q, V and S. Also,
R is the husband of Q and likes Reading.
Seating Arrangement:
V sits at one of the corners of the table, so case I is
rejected.

Page 275 of 1334


Get a Special Paid Bundle PDF on High Level Puzzles & Seating for All Bank Clerk/ PO Mains
If there are any suggestions/ errors in our PDFs Feel Free to contact us via this email: admin@exampundit.in
Ultra Practice Bundle PDF
Reasoning Questions for SBI Clerk/ RBI Assist. Mains

Family Tree:
Answers:
41. d
42. c
43. a
44. b
45. e
Solutions 41-45
Person who likes Surfing sits immediate right of T, so U 1) B lives on 1st floor and is facing towards the center.
likes Surfing. 2) The adjacent members do not face the same direction.
Two persons sit between the persons who like Surfing and 3) E who lives two floors above B is sitting second to the
Mountaineering, so W likes Mountaineering. left of B and works at Google.
Neither S nor V likes Dancing, so Q likes Dancing and
thus, either S or V likes Gardening or Playing.
The final seating arrangement and family tree is given
below:

Page 276 of 1334


Get a Special Paid Bundle PDF on High Level Puzzles & Seating for All Bank Clerk/ PO Mains
If there are any suggestions/ errors in our PDFs Feel Free to contact us via this email: admin@exampundit.in
Ultra Practice Bundle PDF
Reasoning Questions for SBI Clerk/ RBI Assist. Mains
Floors Friends Companies

5 Floors Friends Companies

4 8

3 E Google 7

2 6

1 B 5 H Reliance

4) B and C are the neighbors of H. 4 A L&T

(This implies that H will be at immediately right of B and


3 E Google
C at the immediate left of H)
5) The person who lives on the 4th floor works at L&T and 2

is sitting third to the right of C.


1 B
th
6) The person who is working at Reliance lives on 5 floor
and sits opposite to A, who lives immediately below him. 7) The person who lives in 3rd floor is sitting opposite to

(It implies A works at L&T and lives on 4th floor and H the person working in Intel.

who works in reliance sits opposite to A and lives on (That implies, C works at Intel).

5th floor). 8) D who lives two floors below A works at Samsung and
is a neighbour of A.
(This implies D is sitting to the immediate right of A and
works at Samsung and lives on 2nd floor)
9) The persons who are working at Amazon and Dell are
sitting opposite to each other.

Page 277 of 1334


Get a Special Paid Bundle PDF on High Level Puzzles & Seating for All Bank Clerk/ PO Mains
If there are any suggestions/ errors in our PDFs Feel Free to contact us via this email: admin@exampundit.in
Ultra Practice Bundle PDF
Reasoning Questions for SBI Clerk/ RBI Assist. Mains
(This implies that B works at Walmart) top floor, F must be living on 7th floor and remaining C
lives on 8th floor.)

Floors Friends Companies

6
Floors Friends Companies

5 H Reliance
8 C Intel

4 A L&T
7 F Dell

3 E Google
6 G Amazon

2 D Samsung
5 H Reliance

1 B Walmart
4 A L&T

10) F who works at Dell does not sit between B and E and
3 E Google
he does not live at top floor.
11) The person working at Amazon lives in 6th floor. 2 D Samsung

(This implies that F works at Dell and is sitting between 1 B Walmart


C and D and G is sitting between B and E. Hence, G works
at Amazon and lives on 6th floor. Since F does not live on Answers:
46. c
47. a
Page 278 of 1334
Get a Special Paid Bundle PDF on High Level Puzzles & Seating for All Bank Clerk/ PO Mains
If there are any suggestions/ errors in our PDFs Feel Free to contact us via this email: admin@exampundit.in
Ultra Practice Bundle PDF
Reasoning Questions for SBI Clerk/ RBI Assist. Mains
48. d 6) B got admitted to IIT Delhi and sitting adjacent to F
49. b who got 7th rank.
50. c 7) Both B and F are facing the same direction i.e. towards
Solutions 46-50 the centre.
1) D who is facing towards the centre is sitting second to
the left of G.
2) G is facing outwards and got admitted to IIT Mumbai.
3) The one who got 2nd rank is sitting second to the right
of D who got 8th rank and D got admitted to IIT Patna.

8) H faces the same direction as C and he is not the


neighbour of G.
9) The one who got 6th rank and got admitted to IIT
4) C is sitting between G and D and F is sitting fourth to
Kharagpur is sitting to the immediate left of the person
the right of C.
who got admitted to IIT Madras.
5) E who got 3rd rank is sitting third to the right of C.

10) The one who got 1st rank got admitted to IIT Kanpur.

Page 279 of 1334


Get a Special Paid Bundle PDF on High Level Puzzles & Seating for All Bank Clerk/ PO Mains
If there are any suggestions/ errors in our PDFs Feel Free to contact us via this email: admin@exampundit.in
Ultra Practice Bundle PDF
Reasoning Questions for SBI Clerk/ RBI Assist. Mains
11) C is not the topper and he got admitted to IIT Roorkee.
12) G got the 4th rank.

Download Seating Arrangement Practice Questions PDF


Get More Reasoning Practice Questions PDF
Puzzle Box
Directions(1-5): Answer the questions based on the bottommost box contains 88 bottles. Box S is just above
information given below. box W, which contains 50 bottles. Box S is not adjacent
Eight boxes P, Q, R, S, T, U, V and W are kept one above to box P. Box U does not contain 10 or 81 bottles.
another such that bottommost box is numbered as 1 while 1) Which of the following box contains maximum
the topmost box is numbered as 8. Each of them contains number of bottles?
different number of bottles among 10, 25, 28, 36, 47, 50, a. Box T
81 and 88. b. Box S
Box P is three boxes above the box, which contains 28 c. Box U
bottles. Box Q is two boxes above the box P. There is one d. Box R
box between the box containing 28 bottles and box V. The e. None of these
average of number of bottles in box R and box Q is equal 2) How many boxes are above the box T?
to number of bottles of box P. The box containing 47 a. Three
bottles is just above the one containing 25 bottles. Box R b. Four
is an odd number box. There is one box between the box c. Five
containing 28 bottles and box containing 10 bottles. The d. One
Page 280 of 1334
Get a Special Paid Bundle PDF on High Level Puzzles & Seating for All Bank Clerk/ PO Mains
If there are any suggestions/ errors in our PDFs Feel Free to contact us via this email: admin@exampundit.in
Ultra Practice Bundle PDF
Reasoning Questions for SBI Clerk/ RBI Assist. Mains
e. None of these boxes between the boxes, which contain Kiwi and
3) Which box is just above box R? Strawberries, is equal to the number of boxes between L
a. Box W and H. H, which is placed 2nd to the below of D, is placed
b. Box U above L. Box, which is placed just above H contains
c. Box P Mango. The number of boxes between the boxes, which
d. Box Q contain Mango and Apple, is a prime number. Number of
e. None of these boxes between H and M is 4. Orange is placed 2nd to the
4) How many bottles does box V contain? above of Mango. Number of boxes between the boxes,
a. 10 which contain Orange and Kiwi is equal to the number of
b. 36 boxes below K. Number of boxes above D is equal to the
c. 47 number of boxes below the box contains strawberry. R is
d. 25 placed above D. Box M is placed at the bottom. Box which
e. None of these contains Mango is placed above the box which contains
5) How many boxes are between box T and box W? Apple.
a. Three 6) How many boxes are placed below the box which
b. One contains apple?
c. Two a. 2
d. Four b. 1
e. Five c. 3
Direction(6-10): Read the following information d. Cannot be determined
carefully and answers the question given below. e. None of these
Nine boxes are stacked one above another, such that the 7) Find the odd one out.
bottommost box is 1 and the box above it is 2 and so on. a. R
Some of them contain different fruits and only some of the b. K
boxes are named as alphabetical letters. c. D
Two boxes are placed between M and L, which contains d. M
Kiwi. K is placed immediately above of L. One box is e. H
placed between K and the box, which contains Apple. 8. How many boxes are placed below the box, which
Strawberries are placed just below Apple. Number of contains Mango?
Page 281 of 1334
Get a Special Paid Bundle PDF on High Level Puzzles & Seating for All Bank Clerk/ PO Mains
If there are any suggestions/ errors in our PDFs Feel Free to contact us via this email: admin@exampundit.in
Ultra Practice Bundle PDF
Reasoning Questions for SBI Clerk/ RBI Assist. Mains
a. 7 of the boxes below G is vacant. The topmost box contains
b. 6 Trousers. E is box 7. Box, which contains shirts, is three
c. 4 boxes above G. Box, which contains shoes, are two boxes
d. 8 above F, which contains T-shirts. B is immediately above
e. None of these the box, which contains shoes. H is immediately above C.
9. Which of the following box contains Strawberries? 11) Which among the following statement(s) is/are
a. M true?
b. K a. There are three boxes above the box, which contains
c. R shoes.
d. D b. H is two boxes above the box, which contains caps.
e. None of these c. None is true.
10. Four of the five are alike in a certain way and hence d. Box 1 contains Watches.
form a group. Find the one which doesn’t belong to e. There are two boxes between E and C.
that group. 12) Which among the following two boxes are vacant?
a. R a. D, B
b. M b. E, D
c. Apple c. G, E
d. K d. E, B
e. H e. None of these.
Direction (11-15): Read the following information 13) How many boxes are above the box which contains
carefully and answer the question given below. Caps?
There are eight boxes A, B, C, D, E, F, G and H kept one a. Five
above the other like a stack. Six of these boxes contain b. Four
different items Caps, Shirts, Trousers, T-shirts, Watches c. Six
and Shoes while others are vacant. The bottommost box is d. Three
numbered 1 and the topmost box is numbered 8. e. None of these.
There are at least two boxes between the vacant boxes. 14) Which among the following boxes are between the
There are at least two boxes below G. There are at least vacant boxes?
two boxes above A, which doesn’t contain Watches. None I. C
Page 282 of 1334
Get a Special Paid Bundle PDF on High Level Puzzles & Seating for All Bank Clerk/ PO Mains
If there are any suggestions/ errors in our PDFs Feel Free to contact us via this email: admin@exampundit.in
Ultra Practice Bundle PDF
Reasoning Questions for SBI Clerk/ RBI Assist. Mains
II. H between box E and box F. Box G is kept immediately
III. D below Glass Type box. Paper Type box is below box D
IV. E and H. Metal Type box is placed immediate below
a. Both I and III Wooden Type box but not placed at the bottom most
b. Both II and IV position. Aluminium Type box is above Fibre Type box
c. Both III and IV but below Cotton Type box. Only two boxes are kept
d. Both I and II between box J and Glass Type box. One of the boxes is I
e. None of these. 16) Which of the following box is of Fibre Type?
15) Which of the following box contains shoes? a. Box D
a. G b. Box B
b. B c. Box E
c. E d. Box C
d. H e. None of these
e. None of these 17) Which of the following box is placed at the top?
Directions: (16-20) Study the below details and answer a. Box E
the following questions. b. Box D
There are 10 boxes placed in a stack, not necessarily in the c. Box F
same order. They are of different Types of Material viz. d. Box J
Wooden, Metal, Plastic, Glass, Rubber, Cotton, Steel, e. None of these
Aluminium, Paper and Fibre 18) How many boxes are between the box which is kept
More than four boxes are below box G. Only two boxes immediately below A and immediately above I?
are kept between box A and box C. H is not a Glass Type a. One
box. Steel Type box is kept immediately below C. Box A b. Four
is a Plastic Type and is kept immediate above or below c. Three
Glass Type box. There is only one box kept between Steel d. Two
Type box and Paper Type box. Only three boxes are kept e. None of these
between Rubber Type and Paper Type box. Only two 19) Box H is of which Types box?
boxes are kept between box B and Rubber Type box. Box a. Aluminium
E is kept immediately below box B. Only one box kept b. Rubber
Page 283 of 1334
Get a Special Paid Bundle PDF on High Level Puzzles & Seating for All Bank Clerk/ PO Mains
If there are any suggestions/ errors in our PDFs Feel Free to contact us via this email: admin@exampundit.in
Ultra Practice Bundle PDF
Reasoning Questions for SBI Clerk/ RBI Assist. Mains
c. Cotton contain Jersey nor Ring. Box U is lighter than the box
d. Metal which contains Ring. The box which contains Toy which
e. None of these is just heavier than box S which is heavier than box U. U
20) How many boxes are kept below Aluminium Type neither contains neither Headphones nor Bottles. Three
box? boxes are placed between the box which contain Toy and
a. None the box which contain Laptop. Third heaviest box is
b. One placed immediately below the box which contains Toy.
c. Two Only one box is placed between heaviest and second
d. Three heaviest box. The box contain Headphones is lighter than
e. None of these the box which contain Bottles. Box P does not contain
Bottles and headphones.
Direction (21-25) Study the below details and answer 21) How many boxes are heavier than the box which
the following questions. contains Ring?
Eight boxes are placed one above another. Each of them a. One
contains different items viz. Jersey, Phones, Bottles, b. Two
Diarys, Ring, Headphones, Laptop, and Toy. But not c. More than Three
necessarily in the same order. d. No one
Only two boxes are placed between box P and U. The e. Three
lightest box is placed at the top. Only two boxes are lighter 22) Which of the following box is placed immediately
than box D. Not more than two boxes are placed between below the box which contains Laptop?
the lightest box and box P. Box U is not the lightest box. a. Q
Box R contains Phones. The heaviest box contains Diary. b. S
Box Q is heavier than box D but lighter than box T. The c. T
box which contains Phone is heavier than the box which d. R
contains Jersey. Box S is placed below box Q. Two boxes e. None of these
are placed between box A and the box which contain 23) How many boxes are placed between box A and S?
Jersey. The box which contains Jersey is placed a. One
immediately above the box which contains Diary. The box b. Two
which contain Jersey is heavier than box D. Box Q neither c. More than Three
Page 284 of 1334
Get a Special Paid Bundle PDF on High Level Puzzles & Seating for All Bank Clerk/ PO Mains
If there are any suggestions/ errors in our PDFs Feel Free to contact us via this email: admin@exampundit.in
Ultra Practice Bundle PDF
Reasoning Questions for SBI Clerk/ RBI Assist. Mains
d. No one contains Strawberry. Two boxes are placed between Box
e. Three H and the box which contain Carrot. Only one box is
24) Box D contains which of the following item? between Box B and the box which contains Sugarcane.
a. Laptop Box B does not contain Strawberry. Two boxes are there
b. Diary between A and the box which contains Banana. Neither
c. Bottles Box F nor Box G contains Banana. Box A is not the
d. Jersey lowermost box. Box A is not placed just below Box E.
e. Ring Box C is placed just above the box which contains Apple.
25) Which of the following box contains Laptop? Neither Box F nor Box E contains Coconut. Two boxes
a. D are between C and the box which contains Coconut. Box
b. S D does not contain Sugarcane. Box F is placed below Box
c. T E.
d. U 26) Which of the following box contains Lemon?
e. R a. G
Directions (26-30): Refer to the data below and answer b. C
the questions that follow. c. E
There are eight boxes named A, B, C, D, E, F, G and H. d. B
They are placed one above the other. Each box contains e. None of these
different Fruits i.e., Mango, Banana, Carrot, Coconut, 27) If Box C is related to Banana, Box E is related to
Lemon, Apple, Sugarcane and Strawberry but not Strawberry, in the same way Box A is related to which
necessarily in the same order. of the following Fruit?
Only one box is between Box E and the box which a. Carrot
contains Mango. Three boxes are there between Box E b. Lemon
and Box F. There are two boxes between box G and the c. Sugarcane
box which contains Strawberry. Box G is placed above the d. Coconut
box which contains Strawberry. Box G is placed above e. None of these
Box E. Neither Box F nor Box E is placed at the topmost
or lowermost position. The number of boxes above H is 28) How many Boxes are there between Box G and the
the same as the number of boxes below the box which box which contains Sugarcane?
Page 285 of 1334
Get a Special Paid Bundle PDF on High Level Puzzles & Seating for All Bank Clerk/ PO Mains
If there are any suggestions/ errors in our PDFs Feel Free to contact us via this email: admin@exampundit.in
Ultra Practice Bundle PDF
Reasoning Questions for SBI Clerk/ RBI Assist. Mains
a. None books. The number of Biology books is 22 less than the
b. One number of books in box C. The box containing Polity
c. Two books has 23 books in total. Box F has 18 more books than
d. Three box D and 8 more books than box B. The box that contains
e. None of these Hindi books has the highest number of books and the box
29) Which of the following box contains Coconut? with English books has the second highest number of
a. E books. Box E has 7 more books than box G but 22 more
b. F books than box B. Mathematics books are odd in number.
c. H Box B contains either Chemistry or Polity books and the
d. C number of books is 8/11 of the number of books in box G.
e. None of these 31) Which subject has the least number of books?
30) Which of the following box contains lemon? a. Mathematics
a. B b. Polity
b. F c. Accounting
c. H d. Biology
d. C e. Chemistry
e. None of these345 32) Box F contains books of which subject?
Directions 31-35: Answer the questions based on the a. Hindi
information given below. b. English
Seven boxes A, B, C, D, E, F and G are kept in the c. Biology
warehouse of a printing press. The boxes contain books of d. Mathematics
different subjects which are English, Mathematics, e. Chemistry
Biology, Hindi, Polity, Chemistry and Accounting but not 33) How many books are there in box G?
necessarily in the same order. Each box contains books of a. 20
only one subject. As the thickness of the books differs b. 50
from one subject to the other, the number of books in each c. 40
box is different. d. 48
Box C does not contain books of either Polity or e. None of the above
Mathematics. Box E contains either English or Biology 34) Which box contains Hindi books?
Page 286 of 1334
Get a Special Paid Bundle PDF on High Level Puzzles & Seating for All Bank Clerk/ PO Mains
If there are any suggestions/ errors in our PDFs Feel Free to contact us via this email: admin@exampundit.in
Ultra Practice Bundle PDF
Reasoning Questions for SBI Clerk/ RBI Assist. Mains
a. Box G Cucumber is 42 kg and is placed just below the box that
b. Box C contains Black Olive. Box that contains Potato is placed
c. Box B above the Tomato box. Kawal has Capsicum. Kunal’s box
d. Box F is 5 kg heavier than Naresh, who has Potato. Vinay’s box
e. Can’t be determined contains Mushroom and the box is numbered 5 and the
35) How many books of Chemistry are there? weight of the box is multiple of 13. The one whose box
a. 20 contains Capsicum is 22 kg and is immediately above the
b. 50 box which contains Black Olive. Manoj has Onion.
c. 40 36) Which of the following Person’s box contains
d. 48 Potato?
e. None of the above a. Manoj
36-40) Directions: Answer the questions based on the b. Vishal
information given below. c. Kunal
There are eight people Naresh, Vishal, Kawal, Manoj, d. Naresh
Kunal, Chandan, Vinay and Vikash. All of them having e. Kawal
one box numbered 1 to numbered 8.The lowermost box is 37) Whose box among the following is the heaviest in
numbered 1, just above them is numbered 2 and so on. The the group?
weight of each box is 22 kg, 24 kg, 26 kg, 27 kg, 33 kg, a. Manoj
38 kg, 42 kg and 45 kg, but not necessary in same order. b. The one which contains Black Olive
Each box contains different fruits - Tomato, Onion, c. The one which contains Garlic
Potato, Capsicum, Mushroom, Cucumber, Black Olive d. Naresh
and Garlic but not necessary in same order. e. Kawal
Vikash’s box contains Garlic and his box is heavier than 38) Whose box among the following is on the topmost?
Kunal’s box, which contains Black Olive. Only one box is a. Vishal
there between Potato and the Tomato box. Only one box b. Naresh
is between the one which contains Black Olive and which c. The one whose box contains Tomato
contains Garlic, both of them is numbered odd. Vishal d. The one whose box contains Cucumber
whose box weighs 24 kg, contains Tomato and the box is e. Both (1) and (3)
numbered even but greater than 4. The box that contains 39) Which of the following is true regarding Kunal?
Page 287 of 1334
Get a Special Paid Bundle PDF on High Level Puzzles & Seating for All Bank Clerk/ PO Mains
If there are any suggestions/ errors in our PDFs Feel Free to contact us via this email: admin@exampundit.in
Ultra Practice Bundle PDF
Reasoning Questions for SBI Clerk/ RBI Assist. Mains
a. He likes Garlic the Box two places below the Box of the player who plays
b. His box is 33kg Minecraft. B plays Real Racing. The Box of Real Racing
c. His box is immediate above the Garlic box. and PUBG players are adjacent. C’s Box is between the
d. Only one box is between his box and the box which Box of D and I. D does not play PUBG or Minecraft or
contains Mushroom. Modern Combat.
e. None is true 41) What is the position of Box of Minecraft player
40) What is the weight of the box which contains with respect to the H’s Box.
Cucumber? a. Two places above
a. 24 kg b. Three places below
b. 27 kg c. Four places below
c. 33 kg d. Immediately above
d. 38 kg e. None of these
e. 42 kg 42) Which of the following is not true?
Direction (41-45) Directions: Answer the questions a. The Box of FIFA player is at the bottom.
based on the information given below: b. D plays Call of Duty.
There are nine players A, B, C, D, E, F, G, H and I in a C. There is no Box between the Box of Modern Combat
school. They play different games: PUBG, FIFA, Call of and Asphalt players.
Duty, Modern Combat, Asphalt, Teen Patti, Minecraft, d. I’s Box is just above the Box of Modern Combat player.
Pokemon and Real Racing but not necessarily in same e. All are true
order. There are 9 Boxes one above another that belongs 43) How many Box are there between Box of F the one
to these players. who plays Call of Duty?
There are five Boxes between the Box of the players who a. 1
play PUBG and Modern Combat and any of these two b. 2
Boxes is neither at the top nor at the bottom. A plays c. 3
Asphalt and only 2 Boxes are there above his Box. The d. 4
Box of the person who plays Minecraft is just above the e. None
Box of E. D’s Box is exactly in between the Box of 44) Who plays PUBG among the players?
Pokemon player and Teen Patti player. H plays Pokemon a. D
and his Box is just below A’s Box. G, the FIFA player has b. E
Page 288 of 1334
Get a Special Paid Bundle PDF on High Level Puzzles & Seating for All Bank Clerk/ PO Mains
If there are any suggestions/ errors in our PDFs Feel Free to contact us via this email: admin@exampundit.in
Ultra Practice Bundle PDF
Reasoning Questions for SBI Clerk/ RBI Assist. Mains
c. C does not contain Guava. Box G is between box A and box
d. F F. Box B placed above Box E and Box A.
e. Can’t be determined. 46) B contains which of the following fruit?
45) Find the odd one? a. Banana
a. B – F b. Strawberry
b. H – D c. Kiwi
c. C – I d. Litchi
d. A – B e. None of the above
e. E – G 47) Which of the following combinations is true?
Directions 46-50: Study the below details and answer a. 2nd Box – Kiwi – C
the following questions. b. 4th Box – Mango – F
Eight boxes A, B, C, D, E, F, G and H are placed on a c. 3rd Box – Grape – H
shelf. Each of them contains different fruits viz. Mango, d. None is true.
Banana, Guava, Grape, Kiwi, Litchi, Strawberry and e. 6th Box – Raspberry – A
Raspberry, but not necessarily in the same order. The 48) Who among the following placed on the topmost
Lowermost Box is numbered 1 and the topmost Box is stack?
numbered 8. a. D
There is only one Box between the one who contains b. F
Guava and the one who contains Raspberry. There are two c. H
Boxes between the one who contains Banana and Box C. d. A
Box F number is even and contains Strawberry. Neither e. None of the above.
Box A nor Box B contains Banana or Kiwi. The box 49) How many Boxes are there between F and C?
which contains Grapes placed on bottom. There are three a. None
Boxes between Box B and Box E. Box F placed above b. One
Box C, who does not placed on bottom but contains c. Two
Mango. Only one Box placed between the one who d. Three
contains Kiwi and the one who contains Banana. The box e. None of these
which contains Guava placed on the topmost box. Box H 50) Who among the following placed 3rd from bottom?
a. C
Page 289 of 1334
Get a Special Paid Bundle PDF on High Level Puzzles & Seating for All Bank Clerk/ PO Mains
If there are any suggestions/ errors in our PDFs Feel Free to contact us via this email: admin@exampundit.in
Ultra Practice Bundle PDF
Reasoning Questions for SBI Clerk/ RBI Assist. Mains
b. D d. None.
c. The one who contains Kiwi. e. The one who contains Mango.

Puzzle Box – Answer and Explanation


Solution (1-5) 1 88
1. Answer- C
Case 2:
2. Answer- B
Box number Box Number of bottles
3. Answer- D
4. Answer- A 8
5. Answer- B
7 Q
1. Box P is three boxes above the box, which contains 28
bottles. 6

2. Box Q is two boxes above the box P.


5 P
3. There is one box between the box containing 28 bottles
and box V. So, we have three cases, 4 V

Case 1:
3
Box number Box Number of bottles
2 28
8 Q
1 88
7
Case 3:
6 P
Box number Box Number of bottles

5 V
8 Q

4
7
3 28
6 P

2
5

Page 290 of 1334


Get a Special Paid Bundle PDF on High Level Puzzles & Seating for All Bank Clerk/ PO Mains
If there are any suggestions/ errors in our PDFs Feel Free to contact us via this email: admin@exampundit.in
Ultra Practice Bundle PDF
Reasoning Questions for SBI Clerk/ RBI Assist. Mains
4 2

3 28 1 88

2 Case 3:
Box number Box Number of bottles
1 V 88
8 Q 47

4. The average of number of bottles in box R and box Q is 7 R 25


equal to number of bottles of box P.
6 P 36
So, Q and R must have 25 and 47 bottles but not
necessarily in same order. 5
Hence, Box P contains 36 bottles.
4
5. The box containing 47 bottles is just above the one
containing 25 bottles. 3 28

6. Box R is an odd number box.


2
So, case 2 is rejected.
Case 1: 1 V 88

Box number Box Number of bottles

8 Q 47 7. There is one box between the box containing 28 bottles


and box containing 10 bottles.
7 R 25
8. The bottommost box contains 88 bottles.
6 P 36 9. Box S is just above box W, which contains 50 bottles.
10. Box S is not adjacent to box P.
5 V
Case 1
4 Box number Box Number of bottles

3 28 8 Q 47

Page 291 of 1334


Get a Special Paid Bundle PDF on High Level Puzzles & Seating for All Bank Clerk/ PO Mains
If there are any suggestions/ errors in our PDFs Feel Free to contact us via this email: admin@exampundit.in
Ultra Practice Bundle PDF
Reasoning Questions for SBI Clerk/ RBI Assist. Mains
7 R 25 Box number Box Number of bottles

6 P 36 8 Q 47

5 V 10 7 R 25

4 6 P 36

3 S 28 5 V 10

2 W 50 4 T 81

1 88 3 S 28

Case 3 2 W 50
Box number Box Number of bottles
1 U 88
8 Q 47
Solution 6-10:
7 R 25 6. Answer- A
7. Answer- A
6 P 36
8. Answer-B
5 10 9. Answer- E
10. Answer- E
4 81
1. Two boxes are placed between M and L, which contains
3 S 28 Kiwi.
2. K is placed immediately above of L.
2 W 50
3. One box is placed between K and the box, which
1 V 88 contains Apple.
4. Strawberries are placed just below Apple.
11. Box U does not contain 10 or 81 bottles.
5. Box M is placed at the bottom.
So, case 3 is rejected.
Case-1
The final arrangement is shown below,

Page 292 of 1334


Get a Special Paid Bundle PDF on High Level Puzzles & Seating for All Bank Clerk/ PO Mains
If there are any suggestions/ errors in our PDFs Feel Free to contact us via this email: admin@exampundit.in
Ultra Practice Bundle PDF
Reasoning Questions for SBI Clerk/ RBI Assist. Mains
Positions Boxes 1 M

9
6. Number of boxes between the boxes, which contain
8
Kiwi and Strawberries, is equal to the number of boxes
7 between L and H.
7. H, which is placed 2nd to the below of D, is placed
6
above L.
5 K 8. Box, which is placed just above H contains Mango.
4 L(Kiwi) 9. Box which contains Mango is placed above the box
which contains Apple.
3 (Apple)
Hence, Case-2 is eliminated
2 (Strawberries)
Positions Boxes
1 M
9
Case-2
8 D
Positions Boxes
7 Mango
9
6 H
8
5 K
7 (Apple)
4 L(Kiwi)
6 (Strawberries)
3 (Apple)
5 K
2 (Strawberries)
4 L(Kiwi)
1 M
3

2 10. Orange is placed 2nd to the above of Mango.

Page 293 of 1334


Get a Special Paid Bundle PDF on High Level Puzzles & Seating for All Bank Clerk/ PO Mains
If there are any suggestions/ errors in our PDFs Feel Free to contact us via this email: admin@exampundit.in
Ultra Practice Bundle PDF
Reasoning Questions for SBI Clerk/ RBI Assist. Mains
11. Number of boxes between the boxes, which contain 3. Box, which contains shirts, is three boxes above G.
Orange and Kiwi is equal to the number of boxes below K 4. There are at least two boxes below G, so
12. Number of boxes above D is equal to the number of Case I Case II
boxes below Strawberries.
8 (Trousers) (Trousers)
13. R is placed above D.
The final arrangement is as follows: 7 E (Shirts) E

Positions Boxes 6 (Shirts)

9 R (Orange)
5

8 D
4 G
7 (Mango)
3 G
6 H
2
5 K
1
4 L(Kiwi)

3 (Apple)
5. None of the boxes below G is vacant.
2 (Strawberries) 6. There are at least two boxes between the vacant boxes,
this is not possible in case I, so case I is rejected.
1 M
Case II(a) Case II(b)

Solution 11-15 8 (Trousers) (Trousers)

11. Answer- C
7 E(vacant) E(vacant)
12. Answer- D
13. Answer- C 6 (Shirts) (Shirts)

14. Answer- D
5
15. Answer- A
1. The topmost box contains Trousers. 4 (vacant)

2. E is box 7.
Page 294 of 1334
Get a Special Paid Bundle PDF on High Level Puzzles & Seating for All Bank Clerk/ PO Mains
If there are any suggestions/ errors in our PDFs Feel Free to contact us via this email: admin@exampundit.in
Ultra Practice Bundle PDF
Reasoning Questions for SBI Clerk/ RBI Assist. Mains
3 G(vacant) G 17. Answer- D
18. Answer- B
2
19. Answer- D
1 20. Answer- C
1. More than four boxes are below box G.
7. Box, which contains shoes, are two boxes above F,
2. Box G is kept immediately below Glass Type box.
which contains T-shirts.
3. Only two boxes are kept between box J and Glass Type
8. B is immediately above the box, which contains shoes.
box.
9. H is immediately above C, this is not possible in case
4. Box A is a Plastic Type and is kept immediate above or
II(a), so case II(a) is rejected.
below Glass Type box.
10. There are at least two boxes above A, which doesn’t
5. Only two boxes are kept between box A and box C.
contain Watches, so A is box 2 and thus, D is box 8, so C
6. Steel Type box is kept immediately below C.
contains watches and A contains caps.
From these statements, we get four possibilities:
The final arrangement is as follows:
Case-1 Case-2
8 D(Trousers)
J
7 E(vacant)

6 H (Shirts)
A Plastic A Plastic
5 C (Watches)
Glass Glass
4 B (vacant)
G G
3 G (Shoes)
C C
2 A (Caps)
J Steel Steel
1 F (T-shirts)

Solution 16-20
16. Answer- E

Page 295 of 1334


Get a Special Paid Bundle PDF on High Level Puzzles & Seating for All Bank Clerk/ PO Mains
If there are any suggestions/ errors in our PDFs Feel Free to contact us via this email: admin@exampundit.in
Ultra Practice Bundle PDF
Reasoning Questions for SBI Clerk/ RBI Assist. Mains
12. Only one box kept between box E and box F, from this
case 1 gets eliminated.
Case-2
7. There is only one box kept between Steel Type box and
Paper Type box. J
8. Only three boxes are kept between Rubber Type and
H
Paper Type box.
9. Only two boxes are kept between box B and Rubber A Plastic

Type box.
D Glass
10. Box E is kept immediately below box B.
Case-1 Case-2 G Rubber

J C

F Steel

A Plastic A Plastic B

Glass Glass E Paper

G Rubber G Rubber

C C
13. Metal Type box is placed immediate below Wooden
J Steel Steel
Type box but not placed at the bottom most position.
B B 14. Aluminium Type box is above Fibre Type box but
below Cotton Type box.
E Paper E Paper
Box Types

J Wooden

H Metal
11. Paper Type box is below box D and H. H is not a Glass
Type box.
Page 296 of 1334
Get a Special Paid Bundle PDF on High Level Puzzles & Seating for All Bank Clerk/ PO Mains
If there are any suggestions/ errors in our PDFs Feel Free to contact us via this email: admin@exampundit.in
Ultra Practice Bundle PDF
Reasoning Questions for SBI Clerk/ RBI Assist. Mains
A Plastic Case-1 Case-2 Case-3

D Glass Lightest Lightest Lightest

G Rubber P

C Cotton P

F Steel P

B Aluminium U

E Paper U

I Fibre U

Solution: 21-25
21. Answer- B
6. Box R contains Phones. The heaviest box contains
22. Answer- D
Diary.
23. Answer- B
7. Box Q is heavier than box D but lighter than box T.
24. Answer- C
8. The box which contains Phone is heavier than the box
25. Answer- D
which contains Jersey.
1. Only two boxes are placed between box P and U.
9. Box S is placed below box Q.
2. The lightest box is placed at the top.
So, from the given conditions it is clear that box A is the
3. Only two boxes are lighter than box D.
lightest box.
4. Not more than two boxes are placed between the
10. Two boxes are placed between box A and the box
lightest box and box P.
which contain Jersey.
5. Box U is not the lightest box.
11. The box which contains Jersey is placed immediately
>>>>>D>>
above the box which contains Diary.
(Diary)> > > > > D > A

Page 297 of 1334


Get a Special Paid Bundle PDF on High Level Puzzles & Seating for All Bank Clerk/ PO Mains
If there are any suggestions/ errors in our PDFs Feel Free to contact us via this email: admin@exampundit.in
Ultra Practice Bundle PDF
Reasoning Questions for SBI Clerk/ RBI Assist. Mains
Case-1 Case-2 Case-3 Toy

A A A P Toy

P Jersey P Jersey

P Diary Diary

Jersey Jersey P Jersey U Laptop

U Diary Diary Diary U Laptop

U
18. Third heaviest box is placed immediately below the
box which contains Toy.
So, P is third heaviest box.
12. The box which contain Jersey is heavier than box D. 19. The box which contain Toy which is just heavier than
13. Box Q neither contain Jersey nor Ring. box S which is heavier than box U.
14. Box U is lighter than the box which contains Ring. 20. Only one box is placed between heaviest and second
So, from this case-1 gets eliminated. heaviest box.
15. The box which contains Toy which is just heavier than So, from this case-3 gets eliminated.
box S which is heavier than box U. 21. The box contain Headphones is lighter than the box
16. U neither contains Headphones nor Bottles. which contain Bottles. Box P does not contain Bottles.
So, U contains Laptop. T>R>P>Q>S>D>U>A
17. Three boxes are placed between the box which contain Case-2 Items
Toy and the box which contain Laptop.
A Headphones
(Diary)> > > > > D > A
Case-2 Case-3 Q Toy

A A P Ring

Page 298 of 1334


Get a Special Paid Bundle PDF on High Level Puzzles & Seating for All Bank Clerk/ PO Mains
If there are any suggestions/ errors in our PDFs Feel Free to contact us via this email: admin@exampundit.in
Ultra Practice Bundle PDF
Reasoning Questions for SBI Clerk/ RBI Assist. Mains
S Jersey 5

T Diary 4

U Laptop 3 F Coconut

R Phone 2 F Coconut

D Bottles 1

Solution 26-30 5) There are two boxes between box G and the box which
26. Answer- D contains Strawberry.
27. Answer- B 6) Box G is placed above the box which contains
28. Answer- D Strawberry.
29. Answer- C 7) Box G is placed above Box E.
30. Answer- A 8) Neither Box F nor Box G contains Banana.
1) Three boxes are there between Box E and Box F.
2) Neither Box F nor Box E is placed at the topmost or
lowermost position.
3) Neither Box F nor Box E contains Coconut.
4) Box F is placed below Box E.
Two cases arise:
No. Case 1 Case 2

Box Fruit Box Fruit

7 E Coconut

6 E Coconut

Page 299 of 1334


Get a Special Paid Bundle PDF on High Level Puzzles & Seating for All Bank Clerk/ PO Mains
If there are any suggestions/ errors in our PDFs Feel Free to contact us via this email: admin@exampundit.in
Ultra Practice Bundle PDF
Reasoning Questions for SBI Clerk/ RBI Assist. Mains
No Case 1 Case 2 (I) Case 2 (II) Bo Fruit Bo Fruit Bo Fruit
. x x x
Bo Fruit Bo Fruit Bo Fruit
x x x 8 G Mango G Banana Mango

8 G Banana G Banana 7 G Banana

7 E Coconut G Banana 6 E Coconut E Coconut E Coconut

6 E Coconut E Coconut 5 Strawberr Strawberr


y y
5 Strawberr Strawberr
y y 4 Mango Strawberr
y
4 Strawberr
y 3

3 F Coconut/ 2 F Coconut/ F Coconut/ F Coconut/


Banana Banana Banana Banana

2 F Coconut/ F Coconut/ 1
Banana Banana

1 10) The number of boxes above H is the same as the


number of boxes below the box which contains
Strawberry.
9) Only one box is between Box E and the box which
No Case 2 (I)a Case 2 (I)b Case 2 (II)
contains Mango.
.
Thus, case 1 gets eliminated. Bo Fruit Bo Fruit Bo Fruit
Two subcases of case 2 (I): x x x
No Case 2 (I)a Case 2 (I)b Case 2 (II)
8 G Mango G Banana Mango
.
7 G Banana

Page 300 of 1334


Get a Special Paid Bundle PDF on High Level Puzzles & Seating for All Bank Clerk/ PO Mains
If there are any suggestions/ errors in our PDFs Feel Free to contact us via this email: admin@exampundit.in
Ultra Practice Bundle PDF
Reasoning Questions for SBI Clerk/ RBI Assist. Mains
6 E Coconut E Coconut E Coconut 6 E Apple E Cocon E Cocon E Cocon
ut ut ut
5 Strawberr Strawberr H
y y 5 Straw H Cocon H H
berry ut
4 H H Mango Strawberr
y 4 H Cocon Straw C Straw C Straw
ut berry berry berry
3
3 Apple Apple
2 F Coconut/ F Coconut/ F Coconut/
Banana Banana Banana 2 F Cocon F Cocon F Cocon J Cocon
ut/ ut/ ut/ ut/
1
Banan Banan Banan Banan
a a a a

11) Box C is placed just above the box which contains


1 Cocon
Apple.
ut
12) Two boxes are between C and the box which contains
Coconut.
Thus, case 2 (I) - b gets eliminated. 13) Two boxes are placed between Box H and the box
Three subcases of Case – 2 (II): which contain Carrot.
N Case 2 (I)a Case 2 (II)a Case 2 (II)b Case 2 (III)c 14) Two boxes are there between A and the box which
o. contains Banana.
B Fruit B Fruit B Fruit B Fruit
15) Box A is not the lowermost box.
o o o o
16) Box A is not placed just below Box E.
x x x x
Thus, case 2 (I) - a gets eliminated.
8 G Mang C Mang Mang Mang N Case 2 (II)a(i) Case 2 (II)a(ii) Case 2 (II)b(i)
o o o o o.
Box Fruit Box Fruit Box Fruit
7 C G Apple G Cocon G Banan es es es
ut a

Page 301 of 1334


Get a Special Paid Bundle PDF on High Level Puzzles & Seating for All Bank Clerk/ PO Mains
If there are any suggestions/ errors in our PDFs Feel Free to contact us via this email: admin@exampundit.in
Ultra Practice Bundle PDF
Reasoning Questions for SBI Clerk/ RBI Assist. Mains
8 C Mango C Mango A Mango 4 C Strawbe C Strawbe C Strawbe
rry rry rry
7 G Apple G Apple G Coconut
3 A Apple Apple A Apple
6 E Coconut E Banana E Coconut
2 F Carrot F Coconut F Coconut
5 H Coconut H Coconut H Banana
/ Banana / Banana
4 A Strawbe Strawbe C Strawbe
1 Coconut Coconut
rry rry rry

3 A Apple
17) Only one box is between Box B and the box which
2 F Carrot F Carrot F Carrot contains Sugarcane.
18) Box B does not contain Strawberry.
1 Banana
19) Box D does not contain Sugarcane.
Thus, all the cases get eliminated except case – 2 (II)
– a(ii).
Thus, the final table is as follows:
Boxes Fruits

C Mango
N Case 2 (II)b(ii) Case 2 (II)c(i) Case 2 (II)c(ii)
o. G Apple
Box Fruit Box Fruit Box Fruit
es es es E Banana

8 Mango A Mango Mango H Coconut

7 G Coconut G Banana G Banana D Strawberry

6 E Banana E Coconut E Banana A Sugarcane

5 H H Banana H F Carrot

Page 302 of 1334


Get a Special Paid Bundle PDF on High Level Puzzles & Seating for All Bank Clerk/ PO Mains
If there are any suggestions/ errors in our PDFs Feel Free to contact us via this email: admin@exampundit.in
Ultra Practice Bundle PDF
Reasoning Questions for SBI Clerk/ RBI Assist. Mains
B Lemon 3) Box E has 7 more books than box G but 22 more books
than box B.
(Number of books in box E = a + 7 = 8a/11 + 22, implies
a = 55 and accordingly box E has 62 books. It further
Solution 31-35 implies box B has 40 books.)
31. Answer- B (Box F has 8 more books than books B, implies box F has
32. Answer- C 48 books and box D has 18 less books than box F implies
33. Answer- E box D has 30 books.)
34. Answer- B Box Name Subject Number of Books
35. Answer- C
Box A
1) Box B contains either Chemistry or Polity books and
the number of books is 8/11 of the number of books in box Box B Chemistry 40
G.
Box C
Box Name Subject Number of Books
Box D 30
Box A
Box E 62
Box B Chemistry (8G/11)
Box F 48
Box C
Box G 55
Box D

Box E
4) The box containing Polity books has 23 books in total.
Box F 5) Box C does not contain books of either Polity or
Mathematics.
Box G G
6) Mathematics books are odd in number.
Box Name Subject Number of Books
2) Box F has 18 more books than box D but 8 more books
Box A Polity 23
than box B.

Page 303 of 1334


Get a Special Paid Bundle PDF on High Level Puzzles & Seating for All Bank Clerk/ PO Mains
If there are any suggestions/ errors in our PDFs Feel Free to contact us via this email: admin@exampundit.in
Ultra Practice Bundle PDF
Reasoning Questions for SBI Clerk/ RBI Assist. Mains
Box B Chemistry 40 9) The number of Biology books is 22 less than the
number of books in box C.
Box C
Hence, Box D cannot contain Biology books as this box
Box D 30 has only 30 books.
Box Name Subject Number of Books
Box E 62
Box A Polity 23
Box F 48
Box B Chemistry 40
Box G Mathematics 55
Box C Hindi 70

7) Box E contains either English or Biology books. Box D Accounting 30


8) The box that contains Hindi books has the highest
Box E English 62
number of books and the box containing English books
has the second highest number of books. Box F Biology 48

Box G Mathematics 55
Box Name Subject Number of Books

Box A Polity 23

Box B Chemistry 40
Solution 36-40
Box C Hindi >62 36. Answer- D
37. Answer- C
Box D 30
38. Answer- B
Box E English 62 39. Answer- D
40. Answer- E
Box F 48
1) Vinay’s box contains Mushroom and the box is
Box G Mathematics 55 numbered 5 and the weight of the box is multiple of 13.
(So Vinay’s box is of 26 kg)

Page 304 of 1334


Get a Special Paid Bundle PDF on High Level Puzzles & Seating for All Bank Clerk/ PO Mains
If there are any suggestions/ errors in our PDFs Feel Free to contact us via this email: admin@exampundit.in
Ultra Practice Bundle PDF
Reasoning Questions for SBI Clerk/ RBI Assist. Mains
2) The one whose box contains Capsicum is 22 kg and is 5 Vinay 26 Mushroom
immediately above the box which contains Black Olive.
4 22 Capsicum
3) Only one box is between the one which contains Black
Olive and which contains Garlic, both of them is 3 Black Olive
numbered odd. (This contains two possible cases)
2
Case 1(a):
Box Person Weight Fruit 1 Garlic

8
4) The box that contains Cucumber is 42 kg and is placed
7
just below the box that contains Black Olive.
6 (This cannot be possible with case 1(a) so this is invalid)
5) Vishal whose box weighs 24 kg, contains Tomato and
5 Vinay 26 Mushroom
the box is numbered even but greater than 4. (Two case
4 possible either 6 or 8)
6) Vikash’s box contains Garlic and his box is heavier
3 Garlic
than Kunal’s box, which contains Black Olive.
2 22 Capsicum Case 1(b):
Box Person Weight Fruit
1 Black Olive
8

Case 1(b): 7
Box Person Weight Fruit
6 Vishal 24 Tomato
8
5 Vinay 26 Mushroom
7
4 22 Capsicum
6
3 Kunal Black Olive

Page 305 of 1334


Get a Special Paid Bundle PDF on High Level Puzzles & Seating for All Bank Clerk/ PO Mains
If there are any suggestions/ errors in our PDFs Feel Free to contact us via this email: admin@exampundit.in
Ultra Practice Bundle PDF
Reasoning Questions for SBI Clerk/ RBI Assist. Mains
2 42 Cucumber Box Person Weight Fruit

1 Vikash Garlic 8 Naresh 33 Potato

7
Case 1(c):
6 Vishal 24 Tomato
Box Person Weight Fruit
5 Vinay 26 Mushroom
8 Vishal 24 Tomato
4 Kawal 22 Capsicum
7
3 Kunal 38 Black Olive
6
2 42 Cucumber
5 Vinay 26 Mushroom
1 Vikash Garlic
4 22 Capsicum

3 Kunal Black Olive


11) Manoj has Onion.
2 42 Cucumber (Since, Vikash’s box contains Garlic and his box is
heavier than Kunal’s box, so, Vikash must be 45 and
1 Vikash Garlic
Manoj must be 27)
The final arrangement is as follows
7) Only one box is there between Potato and the Tomato
one. Box Person Weight Fruit
8) Box that contains Potato is placed above the Tomato
8 Naresh 33 Potato
box. (This will eliminate case 1(c))
9) Kawal has Capsicum. 7 Manoj 27 Onion
10) Kunal’s box is 5 kg heavier than Naresh, who has
6 Vishal 24 Tomato
Potato. (Only one combination have difference of 5 kg. 33
– 38) 5 Vinay 26 Mushroom

Case 1(b):

Page 306 of 1334


Get a Special Paid Bundle PDF on High Level Puzzles & Seating for All Bank Clerk/ PO Mains
If there are any suggestions/ errors in our PDFs Feel Free to contact us via this email: admin@exampundit.in
Ultra Practice Bundle PDF
Reasoning Questions for SBI Clerk/ RBI Assist. Mains
4 Kawal 22 Capsicum H Pokemon

3 Kunal 38 Black Olive

2 Chandan 42 Cucumber

1 Vikash 45 Garlic

Modern
Solution 41-45 Combat
41. Answer- B
42. Answer- C
43. Answer- B
44. Answer- D Case – 2
45. Answer- D Person (Box Sport

1) There are five Boxes between the Box of the players position)

who play PUBG and Modern Combat and any of these


two Boxes is neither at the top nor at the bottom.
2) A plays Asphalt and only 2 Boxes are there above his Modern

Box. Combat

3) H plays Pokemon and his Box is just below A’s Box.


A Asphalt
Hence, there are two possible cases (as shown below):
Case – 1 H Pokemon

Person (Box Sport


position)

PUBG
PUBG
A Asphalt

Page 307 of 1334


Get a Special Paid Bundle PDF on High Level Puzzles & Seating for All Bank Clerk/ PO Mains
If there are any suggestions/ errors in our PDFs Feel Free to contact us via this email: admin@exampundit.in
Ultra Practice Bundle PDF
Reasoning Questions for SBI Clerk/ RBI Assist. Mains

4) D’s Box is exactly in between the Box of Pokemon


Modern
player and Teen Patti player.
Combat
5) D does not play PUBG or Minecraft or Modern
Combat. A Asphalt

Case – 1
H Pokemon
Person (Box Sport
position) D

Teen Patti

PUBG

A Asphalt PUBG

H Pokemon

D
6) G, the FIFA player has the Box two places below the
Teen Patti
Box of the player who plays Minecraft.
7) The Box of the person who plays Minecraft is just
above the Box of E.
Modern
Case – 1
Combat
Person (Box Sport
position)

Case – 2
PUBG
Person (Box Sport
position) A Asphalt

Page 308 of 1334


Get a Special Paid Bundle PDF on High Level Puzzles & Seating for All Bank Clerk/ PO Mains
If there are any suggestions/ errors in our PDFs Feel Free to contact us via this email: admin@exampundit.in
Ultra Practice Bundle PDF
Reasoning Questions for SBI Clerk/ RBI Assist. Mains
H Pokemon
8) B plays Real Racing.
D
Here, only possible position of B’s Box is at the top.
Teen Patti 9) The Box of Real Racing and PUBG players are
adjacent.
Minecraft
Hence, 2nd case can be ignored.
E Modern 10) C’s Box is between the Box of D and I.
Combat Case – 1
Person (Box Sport
G FIFA
position)

B Real Racing
Case – 2
Person (Box Sport F PUBG
position)
A Asphalt

H Pokemon
Modern
D Call of Duty
Combat

C Teen Patti
A Asphalt

I Minecraft
H Pokemon

E Modern
D
Combat
Teen Patti
G FIFA
Minecraft

E PUBG Solution 46-50


46. Answer- D
G FIFA
47. Answer- E
Page 309 of 1334
Get a Special Paid Bundle PDF on High Level Puzzles & Seating for All Bank Clerk/ PO Mains
If there are any suggestions/ errors in our PDFs Feel Free to contact us via this email: admin@exampundit.in
Ultra Practice Bundle PDF
Reasoning Questions for SBI Clerk/ RBI Assist. Mains
48. Answer- A Case I:
49. Answer- B Box Number Fruits Boxes
50. Answer- C
8 Guava
1. The box which contains Guava placed on the topmost
2. The box which contains Grape placed on bottom 7
3. There is only one Box between the one who contains
6 Raspberry
Guava and the one who contains Raspberry
Box Number Fruits Boxes 5 Banana

8 Guava 4 Strawberry F

7 3

6 Raspberry 2 Mango C

5 1 Grape

4
Case II:
3
Box Number Fruits Boxes
2
8 Guava
1 Grape
7 Banana

6 Raspberry
4. Box F number is even and contains Strawberry
.Here, two cases will be formed because Box F can be 5

placed on either 2nd Box or 4th Box.


4 Mango C
5. There are two Boxes between the one who contains
Banana and Box C. 3

6. Box F placed above Box C, who does not placed on


2 Strawberry F
bottom but contains Mango.

Page 310 of 1334


Get a Special Paid Bundle PDF on High Level Puzzles & Seating for All Bank Clerk/ PO Mains
If there are any suggestions/ errors in our PDFs Feel Free to contact us via this email: admin@exampundit.in
Ultra Practice Bundle PDF
Reasoning Questions for SBI Clerk/ RBI Assist. Mains
1 Grape 4 Strawberry F

3
Thus, Case II gets eliminated as it opposes statement 6.
2 Mango C
7. Box G is between box A and box F.
Case I (a): 1 Grape

Box Number Fruits Boxes

8 Guava 8. Box B placed above Box E and Box A.


Thus case I (b) gets eliminated.
7
9. There are three Boxes between Box B and Box E.
6 Raspberry A 10. Neither Box A nor Box B contains Banana or Kiwi.
Thus, the one who contains Kiwi is E.
5 Banana G
11. Box H does not contain Guava.
4 Strawberry F Case I (a):
Box Number Fruits Boxes
3
8 Guava D
2 Mango C
7 Litchi B
1 Grape
6 Raspberry A

Case I (b): 5 Banana G


Box Number Fruits Boxes
4 Strawberry F
8 Guava A
3 Kiwi E
7
2 Mango C
6 Raspberry G
1 Grape H
5 Banana

Page 311 of 1334


Get a Special Paid Bundle PDF on High Level Puzzles & Seating for All Bank Clerk/ PO Mains
If there are any suggestions/ errors in our PDFs Feel Free to contact us via this email: admin@exampundit.in
Ultra Practice Bundle PDF
SBI Clerk/ RBI Assist. Mains – Reasoning
Puzzle Floor
Directions (1-5): Study the following information a) 165 Kg
carefully and answer the below questions. b) 160 Kg
There are eight persons viz. A, B, C, D, P, Q, R, and S c) 175 Kg
lives in eight floored building numbered 1 to 8 in such a d) 145 Kg
way that the lowermost floor is marked as 1, the floor e) None of these
above it is marked as 2 and so on and the top floor is 2. Who lives immediately above the floor of S?
marked as 8. Each of them has different weight viz. 30, a) The one whose weight is 65 kg.
40, 45, 65, 95, 80, 85 and 90 kg but not necessarily in the b) The one whose weight is 40 kg.
same order. Only one person lives on each floor. c) P
The person whose weight is 65kg lives on an odd d) The One who lives fourth from the top floor
numbered floor at a gap of two floors from D, who doesn’t e) Q
live on the top most floor. S lives on the adjacent floor of 3. How many persons live between A and theone whose
the one whose weight is 85kg. Twice the weight of the weight is 40 kg?
person who lives on the floor number 4 is equal to thrice a) Two
the weight of the person who lives on the floor number 2. b) Three
Q lives at a gap of one floor from the one whose weight is c) One
85 Kg. Number of floors above C is same as the number d) Four
of floors below the one whose weight is 40kg. The weight e) None
of A is twice the weight of S. A, who neither lives on the 4. Which of the following statement is not true?
lowermost floor nor adjacent to the one whose weight is a) The one whose weight is 45 kg lives immediately
65kg, lives at a gap of three floors from S, who neither below B.
lives on the top nor his weight is 65kg. P, whose weight is b) P lives at a gap of one floor from S.
30kg lives just above the floor of R. Weight of Q is more c) D lives at a gap of three floors from the one whose
than that of the one who lives at a gap of one floor above weight is 30kg.
C. d) The One whose weight is 85 kg lives immediately
1. What is the sum of the weight of the one who lives below Q.
on the floor numbered 7 and the one who lives on the e) All the given statements are true.
floor numbered 3?
Page 312 of 1334
Subscribe The Xpress Video Course & Mock Test Package for Bank & Insurance Exams
If there are any suggestions/ errors in our PDFs Feel Free to contact us via this email: admin@exampundit.in
Ultra Practice Bundle PDF
SBI Clerk/ RBI Assist. Mains – Reasoning
5. Which of the following combination of name, floor one of the adjacent floors of the player who likes to play
and weight is true? cricket in Pune. The number of players live between P and
a) A – 90kg – 7 T is same as the number of players live between S and R.
b) 80kg – 2 – R P and V lives on the adjacent floors. V does not live on
c) S – 45kg – 5 the topmost floor. The number of players live above W is
d) 85 kg – Q – 6 same as the number of players live below the player who
e) D – 40kg – 6 likes to play cricket in Kolkata. W lives on an even
Directions (6-10) Read the given information carefully numbered floor. Q who likes to play cricket in Chennai
and answer the questions given below: lives on one of the floors above the one who likes to play
Nine cricket players viz. P, Q, R, S, T, U, V, W and N are cricket in Kolkata. U and N lives on the adjacent floors.
living ona nine storey building. The lowermost floor is The player who likes to play cricket in Trivandrum lives
numbered as one and the above floor is numbered as two on an odd numbered floor.
and so on. All the players got selected for some franchise 6.Who among the following lives on the sixth floor of
to play in IPL-2020. Each of them likes different city to the building?
play cricket such as Mumbai, Pune, Chennai, Kolkata, a) R
Bangalore, Trivandrum, Delhi, Bhubaneswar and b) S
Ahmedabad but not necessarily in the same order.Only c) P
one person stays on each floor. d) Q
P lives on an even numbered floor above the floor number e) None of these
three. The person who likes to play cricket in 7.Who among the following likes to play in
Bhubaneswar lives immediately above S who likes to play Trivandrum?
cricket in Bangalore. Two players live between P and T. a) Either U or N
S lives on the lowermost floor. There is a gap of three b) N
floors between the one who likes to play cricket in Delhi c) P
and the one who likes to play cricket in Bhubaneswar. d) V
Neither U nor N likes to play cricket in Delhi. There is no e) Either a) or d)
gap between the players who likes to play cricket in 8.Who among the following lives on the top floor of the
Mumbai and the one who likes to play cricket in Pune. The building?
player who likes to play cricket in Bhubaneswar lives on a) Q

Page 313 of 1334


Subscribe The Xpress Video Course & Mock Test Package for Bank & Insurance Exams
If there are any suggestions/ errors in our PDFs Feel Free to contact us via this email: admin@exampundit.in
Ultra Practice Bundle PDF
SBI Clerk/ RBI Assist. Mains – Reasoning
b) N age group persons. All the above information is not
c) V necessarily in the same order.
d) R Note: Nonagenarian means, the person who is between
e) None of these 91-99 years old.
9.P likes to play from which of the following city? The number of five people are living in an odd numbered
a) Trivandrum floor. The 31-39 age group people are not living in the
b) Delhi lowermost floor. The total number of people lives in odd
c) Bangalore numbered floors is same as the total number of people
d) Ahmedabad lives in even numbered floors. Six people are living in
e) None of these more than one floor between the 41-49 age group
10.How many person lives below the floor who likes to people and the 21-29 age group people. The
play for Kolkata? nonagenarian people are living exactly between the 41-49
a) Four age group people and the floor which has only 5 people.
b) Three There is no floor gap between the nonagenarian people
c) Two and 41-49 age group people. Not less than 7 number of
d) Seven people are living on the floor of 91-99 ages group people.
e) None of these The numbers of six people are living in either the
Direction (11-15): Study the following information lowermost floor or the topmost floor. The floor number
carefully and answer the below questions: and the number of people on the floor are not the same.
In an orphanage, there are eight floors and some number The number of lowest age people are more than the
of people is living on each floor and no two floors have number of highest age people. The 81-89 age group
the same number of persons. The lowermost floor is people are not living in an even numbered floor but the
numbered as one and the floor above is numbered as two number of people is an even number. The sum of the
and so on till the topmost floor is numbered as eight. In number of 61-69 age group people and the number of 71-
each floor, a minimum of two people to a maximum of 79 age group people is same as the total number of people
nine people are living. The number of people are in living on the third floor. The 61-69 age group people are
different age groups viz., 21-29, 31-39, 41-49, 51-59, 61- lives not on the adjacent floor of either the 41-49 age
69, 71-79, 81-89 and 91-99. No two floors have the same group peopleor the 81-89 age group people.

Page 314 of 1334


Subscribe The Xpress Video Course & Mock Test Package for Bank & Insurance Exams
If there are any suggestions/ errors in our PDFs Feel Free to contact us via this email: admin@exampundit.in
Ultra Practice Bundle PDF
SBI Clerk/ RBI Assist. Mains – Reasoning
11.The Nonagenarian people are living in which of the 15.Which of the following age group people are living
following numbered floor? on the 7th floor?
a) 2 a) 71-79
b) 3 b) 91-99
c) 4 c) 61-69
d) 5 d) 81-89
e) 8 e) 21-29
12.How many people are living on the floor numbered Directions (16-20):Study the following information
7 and 5? carefully and answer the questions given below:
a) 10 Eight persons T, M, D, X, K, R, G and Z lives on eight
b) 11 different floors. The lowermost floor is numbered 1, the
c) 12 floor above is numbered 2, and so on till the Top most
d) 13 floor is numbered 8. Each one of them works in different
e) 09 banks BOI, SBI, RBI, LVB, KVB, Axis bank, ICICI and
13.How many floors between the floor which has 6 SIB. All the above information is not necessarily in the
numbers of people and the 91-99 age group people? same order. Only one person lives on each floor.
a) None Three persons live between G and the one who works in
b) One RBI. More than two persons live between M and the one
c) Two who works in BOI. K lives below the one who works in
d) Three RBI. More than three persons live between R and the one
e) More than three who works in ICICI. The one who works in ICICI lives
14.If the number of people are arranged in descending above R.G does not live on the topmost floor. Z neither
order from top to bottom, how many number of people works in ICICI nor works in SIB. Only one person lives
are in the age group of 31-39? between X and the one who works in SIB. Number of
a) 6 persons live above X is one less than the number of
b) 5 persons below T. Five persons live between Z and the one
c) 7 who works in LVB. Number of persons live between X
d) 2 and the one who works in RBI is one less than Number of
e) 4 persons live between D and the one who works in SBI. K

Page 315 of 1334


Subscribe The Xpress Video Course & Mock Test Package for Bank & Insurance Exams
If there are any suggestions/ errors in our PDFs Feel Free to contact us via this email: admin@exampundit.in
Ultra Practice Bundle PDF
SBI Clerk/ RBI Assist. Mains – Reasoning
does not work in BOI. The one who works in Axis bank a) G
lives above the one who works in KVB. The one who b) X
works in RBI lives below G. There are three persons live c) T
between K and the one who works in KVB. d) M
16.How many persons live between the person who e) None of these
lives immediately above X and the one who works in 20.What is the sum of the floor numbers of the person
SBI? who lives immediately above D and the person who
a) Four lives immediately below Z?
b) Six a) 12
c) Five b) 10
d) Three c) 13
e) None of these d) 8
17.If Z is related to SBI, X is related to RBI, in the same e) None of these
way which of the following is related to SIB? Directions (21-25): Study the following information
a) R carefully and answer the questions given below:
b) G Eight persons G, H, Q, R, T, L, F and M lives on eight
c) K different floors. The lowermost floor is numbered 1, the
d) D floor above is numbered 2, and so on till the topmost floor
e) None of these is numbered 8. Each one of them are in different age 20,
18.Which of the following combination is not true? 23, 24, 45, 31, 42, 36 and 17. Person those who lives on
a) More than three persons live between G and the an even numbered floor are in an odd numbered age.
one who works in LVB Person those who lives on an odd numbered floor are in
b) T lives below the one who works in Axis bank an even numbered age. All the above information is not
c) X and the one who works in BOI lives on even necessarily in the same order.
numbered floor Two persons live between F and the one whose age is the
d) The one who works in SIB lives immediately multiple of 7. Sum of Q’s and L’s age is one less than the
above the one who works in KVB age of R. Q is younger than L. H is not the third eldest. H
e) None is true lives on an odd numbered floor.Only one person lives
19.Which of the following person works in Axis bank? between H and R. Q lives below G. More than two persons

Page 316 of 1334


Subscribe The Xpress Video Course & Mock Test Package for Bank & Insurance Exams
If there are any suggestions/ errors in our PDFs Feel Free to contact us via this email: admin@exampundit.in
Ultra Practice Bundle PDF
SBI Clerk/ RBI Assist. Mains – Reasoning
live between M and the one whose age is 31. The one who 24.Four of the following five are like in a certain way
is the eldest lives above the one who is the second based on the above arrangement. Find which one does
youngest. The one whose age is 20 does not live not belongs to the group?
immediately above or below G. The one whose age is 31 a) M
does not live below the one whose age is 23.F does not b) G
live on the top most floor.T’s age is two more than twice c) F
the age of Q.More than three persons live between H and d) L
T.the sum of the ages of G and the one who lives in second e) Q
floor is more than 70. 25.Which of the following combination is true?
21.How many persons live between M and the one who a) The one who is youngest lives below M
is the youngest? b) Sum of L and H’s age is 6 years more than the age
a) One of G
b) Two c) R lives below Q
c) Three d) F lives immediately above T
d) Four e) None is true
e) None of these Directions (26-30): Study the following information
22.If G is related to 20, L is related to 17, in the same carefully and answer the below questions.
way how R is related to? Eight persons namely –P, Q, R, S, T, U, V and W lives in
a) 20 eight floored building marked 1 to 8 in such a way that
b) 23 lowermost floor is marked as 1, floor above it is marked
c) 17 as 2 and so on till the top most floor is marked as 8. Each
d) 24 person was born on different date viz. 3, 6, 7, 12, 15, 16,
e) None of these 24 and 28. Each person likes to visit different hill station
23.How many persons live above H? viz. Manali, Tawang, Shillong, Leh, Shimla, Kullu,
a) No one Nainital and Ladakh. All the given information is not
b) One necessary in the same order.
c) Two One who was born on 24th lives at a gap of two from the
d) Three one who likes to visit Shimla. R neither lives adjacent to
e) More than three V nor adjacent to W but lives adjacent to the one who was

Page 317 of 1334


Subscribe The Xpress Video Course & Mock Test Package for Bank & Insurance Exams
If there are any suggestions/ errors in our PDFs Feel Free to contact us via this email: admin@exampundit.in
Ultra Practice Bundle PDF
SBI Clerk/ RBI Assist. Mains – Reasoning
born on 16th. TheOne who likes to visit Ladakh lives in an d) Q
even numbered floor at a gap of two floors from the one e) Can’t be determined
who was born 7th. At least four person lives between the 27.Which of the following statement is true?
one who likes to visit Shillong and U, who was not born a) One who was born on 15th lives at a gap of two
on 12th. V doesn’t live on the lowermost floor but lives floors from S.
just below the one who was born on 3rd. Only three person b) One who visits Shimla lives just above U.
lives between V and the one who likes to visit Manali. The c) One who visits Kullu lives at a gap of two floors
one who was born on 12th is not lives adjacent to S,but from the one who was born on 28th.
lives just above the one who likes Manali. S, who was not d) T lives just below one who visits Tawang.
born on 16th .The one who was born on 3rd lives on an e) All the given statements are not true.
even numbered floor but doesn’t live on the top most 28.How many person lives between T and the one who
floor. Only one person lives between the one who was likes to visit Leh?
born on 12th and W, who likes to visit Leh. One who was a) 4
born on 28th lives at a gap of one floor with the one who b) 1
likes to visit Kullu. One who visits Shimla lives just above c) 3
the one who was born on 15th. One who was born on d) 2
7th likes to visit Tawang, who doesn’t live adjacent to the e) None
one who likes Leh. P neither lives adjacent to R nor 29.Which of the following combination is true?
adjacent to W but lives at a gap of one floor with the one a) U – 15th – 7th floor – Shimla
who was born on 6th. Only two person lives between T and b) Q – 3rd – 4 th floor – Ladakh
S, who doesn’t live in the adjacent floor of the one who c) W – 6 th floor – 24th – Leh
visits Nainital. T lives on one of the floors below Q, who d) P – 12th – 6 th floor – Manali
neither live on the top most floor nor likes to visit e) All the above combination are not true.
Kullu. As the one who was born on 12th doesn’t lives 30.On which of the following date V was born?
adjacent to the one who was born on 16th. a) 16th
26.Who was born on 6th? b) 7th
a) W c) 6th
b) R d) 28th
c) V e) Can’t be determined.

Page 318 of 1334


Subscribe The Xpress Video Course & Mock Test Package for Bank & Insurance Exams
If there are any suggestions/ errors in our PDFs Feel Free to contact us via this email: admin@exampundit.in
Ultra Practice Bundle PDF
SBI Clerk/ RBI Assist. Mains – Reasoning
Directions (31-35): Study the following information 31. Which of the following person works in the
carefully and answer the questions given below: company L?
Eight persons T, K, Y, M, R, V, Q, and J live on the eight a) V
different floors. The Lowermost floor is numbered 1, the b) J
floor above is numbered 2, and so on till The Topmost c) T
floor is numbered 8. Each one of them works in the d) Cannot be determined
different company B, S, L, W, G, P, D, and X, but not e) None of these
necessarily in the same order. Only one person lives on 32. How many persons live between R and the one who
each floor. works in thecompany S?
V lives on an odd numbered floor but not on the seventh a) One
floor. Only one person lives between T and the one who b) Two
works in company P. V does not work in the company P. c) Three
T does not work in the company D. T neither lives on the d) Four
fourth floor nor on the sixth floor. Number of persons live e) None of these
above Y is two less than the number of persons live below 33. If T is related to S, Q is related to P, in the same
R. M lives immediately above the one who works in way R is related to which of the following?
company G. Number of persons live above M is two more a) P
than the number of persons live below J. J does not work b) G
in the company D. Three persons live between V and the c) D
one who works in the company D. J does not live on the d) W
lowermost floor. Q works in the company W. Only one e) None of these
person lives between K and the one who works in the 34.Which of the following combination is true?
company X. Y lives somewhere above R. Neither M nor a) More than two persons live between K and the one
J works in the company X. More than two persons live who works in the company W
between the one who works in the company L and the one b) R lives immediately below the one who works in
who works in the company B. T lives above the one who P
works in the company P. V neither works in the company c) Four persons live between Y and the one who
L nor in the company B. The one who lives in the works in the company L
lowermost floor does not work in the company P.

Page 319 of 1334


Subscribe The Xpress Video Course & Mock Test Package for Bank & Insurance Exams
If there are any suggestions/ errors in our PDFs Feel Free to contact us via this email: admin@exampundit.in
Ultra Practice Bundle PDF
SBI Clerk/ RBI Assist. Mains – Reasoning
d) The one who works in the company B lives above C's district code is an even number which is less than E.D's
V district code is 89. D lives four floors below G. A lives on
e) None is true an even numbered floor below the sixth floor. A's district
35.Four of the following five are like in a certain way code is forty-four greater than B. B lives somewhere
based on the above arrangement. Find which one does above F. E lives immediately below C.
not belongs to the group? 36.Whose district code is 45?
a) R a) The one who lives in first floor
b) Y b) The one who lives in third floor
c) The one works in the company D c) The one who lives in fourth floor
d) V d) The one who lives in fifth floor
e) The one who works in the company W e) The one who lives in topmost floor
37.Who lives on the second floor?
Directions (36-40): Study the following information a) E
carefully and answer the below questions. b) D
Eight friends A, B, C, D, E, F, G, and H are living in eight c) B
different floors of a building. Each of them belongs to d) H
different district code viz., 58, 90, 60, 23, 67, 45, 89 and e) None of these
15. All the above information is not necessarily in the 38. What is the total of E's and D's district code?
same order. The lower most floor of the building is a) 189
numbered as one, the floor above is numbered as two and b) 180
so on till the topmost floor numbered as 8.Only one person c) 38
lives on each floor.
d) 60
The one who belongs to the district code 45 and the one
e) 179
whose district code is 15 are immediate neighbours. A
39.Whose district code is 58?
lives immediately above F. F lives in an odd numbered
a) E
floor. G lives two floors above F. F's district code is three
b) C
times to that of H's district code. G's district code is thirty-
five greater than B's district code. C lives on the top most c) G
floor. The difference of C's and E's district code is thirty. d) A
e) None of these
Page 320 of 1334
Subscribe The Xpress Video Course & Mock Test Package for Bank & Insurance Exams
If there are any suggestions/ errors in our PDFs Feel Free to contact us via this email: admin@exampundit.in
Ultra Practice Bundle PDF
SBI Clerk/ RBI Assist. Mains – Reasoning
40.A belongs to which of the following district code? M. R does not use Nokia. K neither uses Samsung nor uses
a) 67 HTC.

b) 60 41.P uses which of the following mobile phones?

c) 23 a) Oppo
d) 89 b) Nokia
e) None of these c) Samsung
Directions (41-45): Study the following information d) HTC
carefully and answer the question below: e) None of these
Eight persons S, T, P, K, D, M, N and R lives on eight 42.How many persons live between N and the one who
different floors. The lowermost floor is numbered 1, the uses Samsung?
floor above is numbered 2, and so on till the Top most a) One
floor is numbered 8. Each one of them uses different b) Two
mobile phones Sony, LG, HTC, Samsung, Nokia, Vivo, c) Three
Motto and Oppo. All the above information is not
d) Four
necessarily in the same order. Only one people lives on
e) None of these
each floor.
43.How many persons live above M?
P lives on a prime numbered floor but not on the fifth
a) Six
floor. As many persons live above S is same as below the
b) Four
one who uses Sony. Two persons live between N and the
one who uses Motto. K lives immediately below the one c) Seven
who uses Motto. More than three persons live between K d) Five
and the one who uses Oppo. N lives some where above K. e) None of these
P does not use Motto. Only one person lives between D 44.Which of the following combination is true?
and the one who uses Oppo. Three persons live between P a) The one who uses HTC lives below T
and the one who uses Nokia. The one who uses Oppo does b) More than three persons live between R and the
not live immediately below P. S does not use Motto. The one who uses Sony
one who uses Vivo lives immediately below S. More than c) D lives on an even numbered floor and lives below
two persons live between T and the one who uses R
Samsung. The one who uses LG lives immediately below

Page 321 of 1334


Subscribe The Xpress Video Course & Mock Test Package for Bank & Insurance Exams
If there are any suggestions/ errors in our PDFs Feel Free to contact us via this email: admin@exampundit.in
Ultra Practice Bundle PDF
SBI Clerk/ RBI Assist. Mains – Reasoning
d) Only one person lives between T and the one who Xiaomi and ASUS are arranged one below another in the
uses LG same order in the same type of room respectively. B stays

e) None is true below D. Xiaomi, Samsung and Vivo as in the same order

45.If S is related to Motto, R is related to Sony, in the stays in a single floor below the vacant floor. A stays on

same way N is related to which of the following? the right of G and likes Lenovo. D and B does not like

a) Vivo Samsung and Vivo and does not stay on the ground floor
and both lives in same type of room. Only two people live
b) LG
on the topmost floor. H does not stay on the extreme right.
c) HTC
The one who likes Oppo stays to the immediate right room
d) Motto
of D. C, who likes Realme stays below the one who likes
e) None of these
Vivoand both lives in same type of room. J likes ASUS.
Directions (46-48): Study the following information
46.Which brand does ‘H’ like?
carefully and answer the question given below
a) Oppo
A, B, C, D, E, F, G, H, I and J not in the same order live
b) Samsung
on different floors of a building. They like different
c) Apple
Mobiles such as Samsung, Xiaomi, Vivo, Oppo, One Plus,
Realme, Apple, Nokia, ASUS and Lenovo. The lower d) One Plus
most floor is ground floor ,the floor immediate above e) ASUS
ground floor is numbered 1, the floor immediate above 1st 47.Who stays to the extreme left of E?

floor is numbered 2 and so on till the top most floor is a) J


numbered 4. b) H
Each floor has a minimum of two persons and a maximum c) The one who likes Xiaomi.
of three persons. One of the odd numbered floors is totally d) The room is vacant
vacant. Each floor has three rooms and are positioned as e) None of the above
extreme left, middle and extreme right. Except vacant 48.Who stays in the middle room of the top floor?
rooms, each room is occupied by only one person.
a) E
H does not like Apple. I and J does not like Vivo. G, who
b) A
likes One Plus, stays at the smallest odd numbered floor.
c) C
The extreme left room of the floor in which G lives is
d) F
vacant. F stays on the topmost floor. The Mobiles Nokia,
Page 322 of 1334
Subscribe The Xpress Video Course & Mock Test Package for Bank & Insurance Exams
If there are any suggestions/ errors in our PDFs Feel Free to contact us via this email: admin@exampundit.in
Ultra Practice Bundle PDF
SBI Clerk/ RBI Assist. Mains – Reasoning
e) I numbered floor. There is only one floor between the floors
on which F and G stay. E stays immediately above the
Directions (49-50): Study the following information floor on which I stay. G does not stay on the ground floor.
carefully and answer the below questions. B stays immediately above the floor on which H stay.
Nine persons A, B, C, D, E, F, G, H and I stay on a nine There are three floors between the floors on which C and
floors building, but not necessarily in the same order. Only F stay. The one who owns the silver Titan watch stays
one person stays on each floor. All of them have either immediately above G. E stays on the fourth floor. The one
Fastrack or Titan watch of different colours i.e. yellow, who owns the grey Fastrack watch stays on the third floor.
golden, red, orange, purple, black, white, silver and grey, Yellow colour watch is Fastrack.
but not necessarily in the same order. Only five of them 49.What is the colour of the Titan/ Fastrack watch of
have Titan watches. The ground floor is numbered 1, the the person who stays on the 6th floor?
floor above is numbered 2 and so on till the topmost floor a) Red
is numbered 9. b) White
There are two floors between the floors on which the c) Orange
persons having a black Fastrack and silver Titan watches. d) Silver
C owns a red Fastrack watch. F does not own a grey
e) Yellow
Fastrack watch. A stays on an even numbered floor below
50.Who stays on the 7th floor?
the even numbered floor on which H stays. The one who
a) The one who has white watch
owns a purple Titan watch stays on the fourth floor. D
b) B
stays on the second floor and owns the orange Titan
c) G
watch. The one who owns the golden Titan watch stays on
the topmost floor. A does not own a white Titan watch. F d) Both (A) and (C)
owns a black Fastrack watch and stays on an odd- e) The one who has silver watch

Page 323 of 1334


Subscribe The Xpress Video Course & Mock Test Package for Bank & Insurance Exams
If there are any suggestions/ errors in our PDFs Feel Free to contact us via this email: admin@exampundit.in
Ultra Practice Bundle PDF
SBI Clerk/ RBI Assist. Mains – Reasoning
Puzzle Floor – Answer and Explanation
Direction: (1-5)
• The person whose weight is 65kg lives on any odd
numbered floor at a gap of two floors from D, that means
D lives on even numbered floor and D doesn’t live on top
floor, that means we have four possibilities, in case (1)the
one whose weight is 65kg lives on ground floor, in case
(2) the one whoseweight is 65kg lives on floor numbered
3 and in case (3) theone whose weight is 65kg lives on
floor numbered 5and in case (4) the one whose weightis
1. Answer: b 65kg lives on floor numbered 7.
2. Answer: d • P, whose weight is 30kg lives just above floor of
3. Answer: c R, that means case (3) is not valid as in case (3) weight of
4. Answer: d D is 30 kg which is not correct and in case (1) ,case (2)and
5. Answer: e case (4) R lives on ground floor.
Explanation: • Based on above given information we get:
We have:
• Twice the weight of the person who lives on floor
number 4 is equal to thrice the weight of the person who
lives on floor number 2.
Let the weight of theperson wholives on floor number 4 is
X.
Let the weight of theperson wholives on floor number 2 is Again, we have:
Y. • A, who neither lives on the lowermost floor noran
Thus, 2X = 3Y immediate neighbour of theone whose weight is 65kg,
X/Y = 3:2 lives at a gap of three floor from S, who neither lives on
That means, one who lives on floor numbered 2 is 30 kg the top nor his weight is 65kg. by this condition, case 4
and one who lives on floor numbered 4 is 45kg. got eliminated. Now case 1 has one more possibility.
thatmeansin case 1, S lives on floor numbered 7 in case
Page 324 of 1334
Subscribe The Xpress Video Course & Mock Test Package for Bank & Insurance Exams
If there are any suggestions/ errors in our PDFs Feel Free to contact us via this email: admin@exampundit.in
Ultra Practice Bundle PDF
SBI Clerk/ RBI Assist. Mains – Reasoning
1a, S lives on floor numbered 3 and in case 2 , S lives on • Weight of A is twice the weight of S, that means
floor numbered 4. case (1b) is not valid as weight of A doesn’t follow given
• S lives on the adjacent floor of the one whose condition and in case (2) weight of A must be 90kg.
weight is 85kg and the weight of D is not 85kg, now case1 • Weight of Q is more than that of the one who lives
have one more possibility that means, in case 1, the one at a gap of one floor above C, which means weight of Q
whose weight is 85 kg lives on top floor in case (1b) A must be 95kg.
lives on the floor numbered 3 and the one whose weight Based on above given information, we get final
is 85 kg lives immediatelyabove S, in case (2) A lives on arrangement as follows:
the top floor and the one whose weight is 85kglives on
floor numbered 5 .case 1a is not valid.
• Q lives at a gap of one floor from the one whose
weight is 85 Kg, which meansnow case 2 have one more
possibility, in case (1) Q lives immediate below S, in case
(1b) Q lives on the top floor ,in case 2 ,Q lives on the floor
numbered 7 and in case (2a) , Q lives on floor numbered
In case 1 ,case2a are not valid because, Number of floors
3.
above C is same as the number of floors below the one
Based on above given information we get 4 possibilities:
whose weight is 40 kg,and case 1b Is not valid
becauseWeight of A is twice the weight of S.
So the final solution is case 2.

Again, we have:
• Number of floors above C is same as the number
of floors below the one whose weight is 40 kg, which
means case (1),case 2aare not valid, in case (1b) C lives
on floor numbered 6 and in case (2) C lives on floor
numbered 3.
Directions (6-10):

Page 325 of 1334


Subscribe The Xpress Video Course & Mock Test Package for Bank & Insurance Exams
If there are any suggestions/ errors in our PDFs Feel Free to contact us via this email: admin@exampundit.in
Ultra Practice Bundle PDF
SBI Clerk/ RBI Assist. Mains – Reasoning

The number of players live above W is same as the


number of players live below the player who likes to play

6. Answer: c cricket in Kolkata. W lives on an even numbered floor. Q

7. Answer: e who likes to play cricket in Chennai lives on one of the

8. Answer: a floors above the one who likes to play cricket in

9. Answer: b Kolkata. By this condition , case 1 is eliminated.

10. Answer: d
Explanation:
S lives on the lowermost floor. The player who likes to
play cricket in Bhubaneswar lives immediately above S
who likes to play cricket in Bangalore. There is a gap of
three floors between the one who likes to play cricket in Player U and Player N live on the immediate floors.
Delhi and the one who likes to play cricket in Neither U nor N likes to play cricket in Delhi. So, case 2
Bhubaneswar. The player who likes to play cricket in will be eliminated.
Bhubaneswar lives on one of the adjacent floor of the
player who likes to play cricket in Pune. There is no gap
between the players who likes to play cricket in Mumbai
and the one who likes to play cricket in Pune. Player P
lives on an even numbered floor above the floor number
three. Two players live between P and T. The number of
players live between P and T is same as the number of
playerslive between S and R.Based on the above The player who likes to play cricket in Trivandrum lives
conditions, we get 3 possibilities. on an odd numbered floor.
Thus we get the final Table as follows,

Page 326 of 1334


Subscribe The Xpress Video Course & Mock Test Package for Bank & Insurance Exams
If there are any suggestions/ errors in our PDFs Feel Free to contact us via this email: admin@exampundit.in
Ultra Practice Bundle PDF
SBI Clerk/ RBI Assist. Mains – Reasoning
8 8 6

7 7

6 6

5 5

4 4

3 3
Directions(11-15):
2 2

1 6 1

In each floor, a minimum of two people to a maximum of


nine people are living
In an orphanage, there areeight floors and some number
of people is living on each floorand no two floors have the
same number of persons.
The total number of odd floor people is same as the total
11.Answer: c
number of even floor people.
12. Answer: b
The floor number and the number of people in the floor
13. Answer: c
are not the same.
14. Answer: e
The numbers of 5 people are living in an odd numbered
15. Answer: d
floor.
Explanation:
Case-1 Case-1-A
The numbers of six people are living in either the
lowermost floor or the topmost floor. Flo Peop Age Flo Peop Age
Case-1 Case-2 or le gro or le gro
up up
Floor People Age Floor People Age
group group 8 8

Page 327 of 1334


Subscribe The Xpress Video Course & Mock Test Package for Bank & Insurance Exams
If there are any suggestions/ errors in our PDFs Feel Free to contact us via this email: admin@exampundit.in
Ultra Practice Bundle PDF
SBI Clerk/ RBI Assist. Mains – Reasoning
7 7 5 The nonagenarian people are living exactly between the
41-49 age group people and the floor which has only 5
6 6
people. There is no floor gap between the nonagenarian
5 5 5 5 people and 41-49 age group people.
Case-1 Case-1-A Case-2-A (I)
4 4
Flo Peop Age Flo Peop Age Flo Peop Age
3 5 3
or le gro or le gro or le gro
2 2 up up up

1 6 1 6 8 8 8 6

Case-2-A Case-2-B 7 7 5 7

Case-2 6 6 91- 6
99
Flo Peop Age Flo Peop Age Flo Peop Age
or le gro or le gro or le gro 5 41- 5 41- 5
up up up 49 49

8 6 8 6 8 6 4 91- 4 4
99
7 7 7 5
3 5 3 3 5
6 6 6
2 2 2 91-
5 5 5 5 5 5
99
4 4 4
1 6 1 6 1 41-
3 3 5 3 49

2 2 2
Case-2 Case-2-A Case-2-B
1 5 1 1

Page 328 of 1334


Subscribe The Xpress Video Course & Mock Test Package for Bank & Insurance Exams
If there are any suggestions/ errors in our PDFs Feel Free to contact us via this email: admin@exampundit.in
Ultra Practice Bundle PDF
SBI Clerk/ RBI Assist. Mains – Reasoning
Flo Peop Age Flo Peop Age Flo Peop Age Therefore to have 6 people, only combination we have
or le gro or le gro or le gro is 2+4=6.
up up up By using above info we get
Case-2-A(I) gets eliminated since above statements are
8 6 8 6 8 6
not possible simultaneously.
7 7 7 5 Case-1 Case-1-A Case-2-A (I)
[Eliminated]
6 6 6 91-
99 Flo Peop Age Flo Peop Age Flo Peop Age
or le gro or le gro or le gro
5 5 41- 5 41-
up up up
49 49
8 21- 8 8 6
4 4 91- 4
29
99
7 2/4 7 5 7
3 41- 3 5 3
49 6 4/2 6 91- 6
99
2 91- 2 2
99 5 41- 5 41- 5
49 49
1 5 1 1
4 91- 4 2 4
Six people are living in more than one floor between
99
the 41-49 agesgroup people and the 21-29 ages group
people. 3 5 3 4 3 5
The floor number and the number of people in the floor
2 2 21- 2 91-
are not the same.
29 99
As per above statement & above 6 cases, 2 floors must be
there between the 41-49 age group people and the 21-29 1 6 1 6 1 41-
age group people since 6 people is living either in 1st or 49

8th.

Page 329 of 1334


Subscribe The Xpress Video Course & Mock Test Package for Bank & Insurance Exams
If there are any suggestions/ errors in our PDFs Feel Free to contact us via this email: admin@exampundit.in
Ultra Practice Bundle PDF
SBI Clerk/ RBI Assist. Mains – Reasoning
Case-2 Case-2-A Case-2-B 2 3 5 (2+3=5)
2 4 6 (2+4=6)
Flo Peop Age Flo Peop Age Flo Peop Age
(not possible, since 6
or le gro or le gro or le gro
people lives in 1st or
up up up
8th)
8 6 8 6 21- 8 6 2 5 7 (2+5=7)
29 2 6 8 (2+6=8)

7 7 4/2 7 5 2 7 9 (2+7=9)
3 4 7 (3+4=7)
6 21- 6 2/4 6 91-
3 5 8 (3+5=8)
29 99
3 6 9 (3+6=9)
5 4 5 41- 5 41- 4 5 9 (4+5=9)
49 49 Now by using above table we can confirm that minimum
5 people are living in 3rd floor. This implies Case-1-A and
4 2 4 91- 4 2
Case-2-B stand eliminated.
99
Rest of the cases can be solved based on other statements.
3 41- 3 5 3 4 Case-1 Case-1-A [Eliminated]
49
Floor People Age Floor People Age
2 91- 2 2 21- group group
99 29
8 21-29 8
1 5 1 1
7 2/4 7 5
Note: Total we have 5 cases
6 4/2 6 91-99
The sum of the total number of 61-69 age group people
and the total number of 71-79 age group people are same 5 41-49 5 41-49
as the total number of people living in the third floor.
4 91-99 4 2
61-69 (or) (71- (71-79) (or) Third floor people
79) (61-69) (possibilities) 3 5 3 4

Page 330 of 1334


Subscribe The Xpress Video Course & Mock Test Package for Bank & Insurance Exams
If there are any suggestions/ errors in our PDFs Feel Free to contact us via this email: admin@exampundit.in
Ultra Practice Bundle PDF
SBI Clerk/ RBI Assist. Mains – Reasoning
2 2 21-29 Not less than 7 number of people are living in the floor of
91-99 ages group people.
1 6 1 6
The number of lowest age people are more than the
number of highest age people.
Case-2 Case-2-A Case-2-B (21-29)>(91-99). This implies that 91-99 doesn’t have 9
[Eliminated] people.
Therefore
Flo Peop Age Flo Peop Age Flo Peop Age
➔If (21-29) age group have 9 people and then (91-99) age
or le gro or le gro or le gro
group have 8 people (1st possibility)
up up up
➔If (21-29) age group have 9 people and then (91-99) age
8 6 8 6 21- 8 6 group have 7 people (2nd possibility)
29 ➔If (21-29) age group have 8 people and then (91-99) age
group have 7 people (3rd possibility)
7 7 4/2 7 5
Case-1 (3rd possibility) eliminated since the floor number
6 21- 6 2/4 6 91- and the number of people in the floor are not the same.
29 99 Case-1 (1st Case-1 (2nd Case-1 (3rd
possibility) possibility) possibility)
5 4 5 41- 5 41-
49 49 Eliminated

4 2 4 91- 4 2 Flo Peop Age Flo Peop Age Flo Peop Age
99 or le gro or le gro or le gro
up up up
3 41- 3 5 3 4
49 8 9 21- 8 9 21- 8 8 21-
29 29 29
2 91- 2 2 21-
99 29 7 2/4 7 2/4 7 2/4

1 5 1 1 6 4/2 6 4/2 6 4/2

Note: Total we have 3 cases


Page 331 of 1334
Subscribe The Xpress Video Course & Mock Test Package for Bank & Insurance Exams
If there are any suggestions/ errors in our PDFs Feel Free to contact us via this email: admin@exampundit.in
Ultra Practice Bundle PDF
SBI Clerk/ RBI Assist. Mains – Reasoning
5 41- 5 41- 5 41- 1 5 1 5 1 5
49 49 49

4 8 91- 4 7 91- 4 7 91- Case-2-A gets eliminated since 21-29 doesn’t have 6
99 99 99 people.
Case-2-A [Eliminated]
3 5 3 5 3 5

Floor People Age


2 2 2
group
1 6 1 6 1 6
8 6 21-29

7 4/2
Case-2 (1st Case-2 (2nd Case-2 (3rd
possibility) possibility) possibility) 6 2/4

Flo Peop Age Flo Peop Age Flo Peop Age 5 41-49
or le gro or le gro or le gro
4 91-99
up up up

3 5
8 6 8 6 8 6

2
7 7 7

1
6 9 21- 6 9 21- 6 8 21-
29 29 29

The total number of odd floor people is same as the total


5 4 5 4 5 4
number of even floor people.
4 2 4 2 4 2 No let check possibilities based on above stmt in each of
above 5 cases
3 41- 3 41- 3 41-
49 49 49
Case-1 (1st possibility): Sub-1➔even floors sum,
9+4+8=21 people living in 8th, 6th and 4th floor. So 1
2 8 91- 2 7 91- 2 7 91- person is required for 2nd floor which is not possible,
99 99 99 hence eliminated

Page 332 of 1334


Subscribe The Xpress Video Course & Mock Test Package for Bank & Insurance Exams
If there are any suggestions/ errors in our PDFs Feel Free to contact us via this email: admin@exampundit.in
Ultra Practice Bundle PDF
SBI Clerk/ RBI Assist. Mains – Reasoning
Case-1 (1st possibility): Sub-2➔even floors sum, Case-1 (2nd possibility): Sub-2➔even floors sum,
9+2+8=19 people living in 8th, 6th and 4th floor. So 3 9+2+7=18 people living in 8th, 6th and 4th floor. So 4
people are required for 2nd floor people are required for 2nd floor which is not possible
This implies 7 people living in 5th floor i.e. odd floors since 4 people are living in 7th hence eliminated
sum=4+7+5+6=22 people living in 1st, 3rd, 5th and 7th Case-1 (2nd possibility) Case-1 (2nd possibility)
floors.
Sub-1 [Eliminated] Sub-2 [Eliminated]
Case-1 (1st possibility) Case-1 (1st possibility)
Floor People Age Floor People Age
Sub-1 [Eliminated] Sub-2
group group
Floor People Age Floor People Age
8 9 21-29 8 9 21-29
group group
7 2 7 4
8 9 21-29 8 9 21-29
6 4 6 2
7 2 7 4
5 41-49 5 41-49
6 4 6 2
4 7 91-99 4 7 91-99
5 41-49 5 7 41-49
3 5 3 5
4 8 91-99 4 8 91-99
2 2
3 5 3 5
1 6 1 6
2 2 3
Case-2 (1st possibility) even floors sum, 6+9+2+8=25
1 6 1 6
people living in 8th, 6th, 4th floor and 2nd floor i.e. exceeded
22 people, hence eliminated
Case-1 (2nd possibility): Sub-1➔even floors sum, Case-2 (2nd possibility) even floors sum, 6+9+2+7=24
9+4+7=20 people living in 8th, 6th and 4th floor. So 2 people living in 8th, 6th, 4th floor and 2nd floor i.e. exceeded
people are required for 2nd floor which is not possible, 22 people, hence eliminated
hence eliminated

Page 333 of 1334


Subscribe The Xpress Video Course & Mock Test Package for Bank & Insurance Exams
If there are any suggestions/ errors in our PDFs Feel Free to contact us via this email: admin@exampundit.in
Ultra Practice Bundle PDF
SBI Clerk/ RBI Assist. Mains – Reasoning
Case-2 (3rd possibility) even floors sum, 6+8+2+7=23 The sum of the total number of 61-69 age group people
people living in 8th, 6th, 4th floor and 2nd floor i.e. exceeded and the total number of 71-79 age group people are same
22 people, hence eliminated as the total number of people living in the third floor.
Case-2 (1st Case-2 (2nd Case-2 (3rd 61-69 (or) (71- (71-79) (or) Third floor people
possibility) possibility) possibility) 79) (61-69) (possibilities)
2 3 5 (2+3=5)
Eliminated [Eliminated] [Eliminated]
2 4 6 (2+4=6)
Flo Peop Age Flo Peop Age Flo Peop Age (not possible, since 6
or le gro or le gro or le gro people lives in 1st or
up up up 8th)

8 6 8 6 8 6 2 5 7 (2+5=7)
2 6 8 (2+6=8)
7 7 7
2 7 9 (2+7=9)
6 9 21- 6 9 21- 6 8 21- 3 4 7 (3+4=7)
29 29 29 3 5 8 (3+5=8)
3 6 9 (3+6=9)
5 4 5 4 5 4
4 5 9 (4+5=9)
4 2 4 2 4 2 Third floor has 5 people i.e. (61-69) and (71-79) age
groups have 2 and 3 people, in any order.
3 41- 3 41- 3 41-
Given, 61-69 age group people are not in the adjacent
49 49 49
floor of the 41-49 age group people
2 8 91- 2 7 91- 2 7 91- Therefore (61-69) and (71-79) age groups have 3 and 2
99 99 99 people respectively
Case-1 (1st possibility)
1 5 1 5 1 5

Sub-2
Case-1 (1st possibility) Sub-2 is left further.
The 61-69 age group people are not in the adjacent floor Floor People Age
of either the 41-49 age group people or the 81-89 age group
group people.

Page 334 of 1334


Subscribe The Xpress Video Course & Mock Test Package for Bank & Insurance Exams
If there are any suggestions/ errors in our PDFs Feel Free to contact us via this email: admin@exampundit.in
Ultra Practice Bundle PDF
SBI Clerk/ RBI Assist. Mains – Reasoning
8 9 21-29 4 8 91-99

7 4 3 5 81-89

6 2 71-79 2 3 61-69

5 7 41-49 1 6 81-89

4 8 91-99 The 31-39 age group people are not living in the
lowermost floor.
3 5
My take: Finally 51-59 in 1st floor
2 3 61-69 Case-1 (1st possibility)

1 6 Sub-2

The 81-89 age group people are not living in an even Floor People Age
numbered floor but the number of people is an even group
number.
8 9 21-29
The 61-69 age group people are not in the adjacent floor
of either the 41-49 age group peopleor the 81-89 age 7 4 81-89
group people.
6 2 71-79
Case-1 (1st possibility)
5 7 41-49
Sub-2
4 8 91-99
Floor People Age
group 3 5 31-39

8 9 21-29 2 3 61-69

7 4 81-89 1 6 51-59

6 2 71-79 Directions (16-20):

5 7 41-49

Page 335 of 1334


Subscribe The Xpress Video Course & Mock Test Package for Bank & Insurance Exams
If there are any suggestions/ errors in our PDFs Feel Free to contact us via this email: admin@exampundit.in
Ultra Practice Bundle PDF
SBI Clerk/ RBI Assist. Mains – Reasoning

16. Answer: c
17. Answer: d
18. Answer: d
19. Answer: b
Z neither works in ICICI nor works in SIB. Only one
20. Answer: c
person lives between X and the one who works in SIB.
Explanation:
Number of persons live above X is one less than the
Three persons live between G and the one who works in
number of persons below T.by this condition case 1a , case
RBI. The one who works in RBI lives below G. G does
1b and case2 have one more possibility each and case 3a
not lives on the topmost floor. K lives below the one who
becomes invalid.
works in RBI.based on the given information, we get 3
possibilities.

Five persons lives between Z and the one who works in


Case 3(a) will be dropped because Number of persons live
LVB. More than three persons live between R and the one
above X is one less than the number of persons below T.
who works in ICICI. The one who works in ICICI lives
Number of persons live between X and the one who works
above R. Z does not work in ICICI.now case 1 has 3
in RBI is one less than Number of persons live between D
possibilities i.e 1a,1b,1c and case3 has two possibilities
and the one who works in SBI.case 1a ii),case 3b got
case3a, case 3b respectively.
eliminated by this condition.More than two persons live
between M and the one who works in BOI. K does not
Page 336 of 1334
Subscribe The Xpress Video Course & Mock Test Package for Bank & Insurance Exams
If there are any suggestions/ errors in our PDFs Feel Free to contact us via this email: admin@exampundit.in
Ultra Practice Bundle PDF
SBI Clerk/ RBI Assist. Mains – Reasoning
work in BOI. By this condition, case 1a i) and case 2b got
eliminated. The one who works in Axis bank lives above
the one who works in KVB.

21.Answer: a
22.Answer: d
23.Answer: e
There are three persons live between K and the one who 24.Answer: d
works in KVB.by this condition case 1b i),case 1b ii),case 25.Answer: c
1c got eliminated. Explanation:
Two persons live between F and the one whose age is the
multiple of 7. F does not live on the top most floor.

So final arrangement is,

Sum of Q’s and L’s age is one less than the age of R. Q is
younger than L.
Q+L=R-1
T’s age is two more than twice the age of Q.
T=2+2(Q)
Only two cases are possible
Directions (21-25):

H lives onan odd numbered floor.

Page 337 of 1334


Subscribe The Xpress Video Course & Mock Test Package for Bank & Insurance Exams
If there are any suggestions/ errors in our PDFs Feel Free to contact us via this email: admin@exampundit.in
Ultra Practice Bundle PDF
SBI Clerk/ RBI Assist. Mains – Reasoning
Age of H is an even number and H is not the third
eldest. Case 2 will be dropped because we can’t fix age
of H.
Case 1 is possible.
Age of H is 20.
Only one person lives between H and R. More than three Case 2will be dropped because Q’s age Is 17 hence he
persons live between H and T. should live on even numbered floor.
the sum of the ages of G and the one who lives in second
floor is more than 70.
By this condition, case 3a and case 3b will be eliminated.
in case 3c M’s age will be 45 to fulfill above condition.

Case 1 and 4 will be dropped because More than three


persons live between H and T. T’s age is an even number,
should live only on an odd numbered floor.
The one whose age is 20 does not live immediately above
or below G .Q lives below G and Q’s age is 17.by this
condition case 2 becomes invalid.
L’ s age is 24 , hence it should be placed in 5th floor So case 3c Is the final arrangement.
following the condition Person those who lives on an odd
numbered floor are in an even numbered age.
More than two persons live between M and the one whose
age is 31. The one who is the eldest lives above the one
who is the second youngest. The one whose age is 31 does
not live below the one whose age is 23.

Directions (26-30):

Page 338 of 1334


Subscribe The Xpress Video Course & Mock Test Package for Bank & Insurance Exams
If there are any suggestions/ errors in our PDFs Feel Free to contact us via this email: admin@exampundit.in
Ultra Practice Bundle PDF
SBI Clerk/ RBI Assist. Mains – Reasoning
• The one who was born on 12th is not lives adjacent
to S,but lives just above the one who likes Manali. S, who
was not born on 16th .
• Only one person lives between the one who was
born on 12th and W, who likes to visit Leh.
Based on above given information we have:

26. Answer: a
27. Answer: b
28. Answer: c
29. Answer: b Again, we have:
30. Answer: d • R neither lives adjacent to V nor adjacent to W but
Explanation: lives adjacent to the one who was born on 16th, as the one
We have: who was born on 12th doesn’t lives adjacent to the one
• One who was born on 3rd lives on an even who was born on 16th, that means in case (1a) R lives on
numbered floor but doesn’t live on the top most floor. the top most floor, in case (1b) R lives on the second floor
• V doesn’t live on the lowermost floor but lives just from the top, in case (2) R lives on the lowermost floor.
below the one who was born on 3rd, that means we have • TheOne who likes to visit Ladakh lives in an even
two possible place for V in case (1) V lives on the floor numbered floor at a gap of two floors from the one who
numbered 5, in case (2) V lives on the floor numbered 3. was born 7th.
• Only three person lives between V and the one • The who was born on 7th likes to visit Tawang,
who likes to visit Manali, that means in case (1) the one who doesn’t lives adjacent to the one who likes Leh, that
who visits Manali lives on the floor marked 1, in case (2) means case (1a) & case (1b) is not valid and in case (2)
the one who visits Manali lives on the floor marked 7. the one who visits Ladhak lives on floor marked 4.
Based on above given information we have:
Page 339 of 1334
Subscribe The Xpress Video Course & Mock Test Package for Bank & Insurance Exams
If there are any suggestions/ errors in our PDFs Feel Free to contact us via this email: admin@exampundit.in
Ultra Practice Bundle PDF
SBI Clerk/ RBI Assist. Mains – Reasoning
that means U lives on the floor marked 7 and one who
likes to visit Shillonglives on the floor marked 2.
• Only two persons live between T and S, who
doesn’t live in the adjacent floor of the one who visits
Nainital.
• T lives on one of the floors below Q, who neither
live on the top most floor nor likes to visit Kullu, that
Case (1a) & case (1b) is not valid as the one who was
means case (2a) is not valid and in case (2b) T lives on
born on 7th likes to visit Tawang, who doesn’t lives
floor marked 2.
adjacent to the one who likes Leh.
Based on above given information we have final
Again, we have:
arrangement as follow:
• The One who was born on 24th lives at a gap of
two from the one who likes to visit Shimla.
• The One who visits Shimla lives just above the one
who was born on 15th, that means the one who was born
on 24th lives on the floor marked 5.
• P neither lives adjacent to R nor adjacent to W but
lives at a gap of one floor with the one who was born on
6th, that means we have two possible place for P, in case
(2a) P lives on the floor marked 4, in case (2b) P lives on
the floor marked 8.
• One who was born on 28th lives at a gap of one Case (2a) is not valid as T who lives on any floor below

floor with the one who likes to visit Kullu. Q, who neither lives on top floor nor likes to visit

• At least four person lives between the one who Kullu.

likes to visit Shillong and U, who was not born on 12th,

Page 340 of 1334


Subscribe The Xpress Video Course & Mock Test Package for Bank & Insurance Exams
If there are any suggestions/ errors in our PDFs Feel Free to contact us via this email: admin@exampundit.in
Ultra Practice Bundle PDF
SBI Clerk/ RBI Assist. Mains – Reasoning
Directions (31-35):
R. Y livessomewhere above R. The one who lives in the
lowermost floor does not work in the company P.

31.Answer: d
M lives immediately above the one who works in the
32. Answer: b
company G. Number of persons live above M is two more
33.Answer: c
than number of persons live below J. J does not work in
34. Answer: d
the company D. J does not live on the lowermost floor. Q
35. Answer: a
works in the company W.Only one person lives between
Explanation:
K and the one who works in the company X.
V lives onan odd numbered floor but not on the seventh
floor. Three persons live between V and the one who
works in the company D.

Only one person lives between T and the one who works
Case 1: B-I and II will be dropped because neither M
in the company P. T lives on the floor above the one
nor J works in the company X.Case 1:B III will be
whoworks in the company P. V does not work in the
dropped becauseOnly one person lives between K and the
company P. T does not work in the company D. T neither
one who works in the company X.
lives on fourth nor lives on sixth floor. Number of persons
Case 2: B and Case 3 will be dropped because Number
live above Y is two less than number of persons live below
of persons live above M is two more than number of
persons live below J.
Page 341 of 1334
Subscribe The Xpress Video Course & Mock Test Package for Bank & Insurance Exams
If there are any suggestions/ errors in our PDFs Feel Free to contact us via this email: admin@exampundit.in
Ultra Practice Bundle PDF
SBI Clerk/ RBI Assist. Mains – Reasoning
More than two persons live between the one who works in Step 1:
the company L and the one who works in the company B. C lives in the top most floor.
V neither works in the company L nor works in the E lives immediately below C.
company B. F lives in an odd numbered floor.
G lives two floors above F. D lives four floors below G.
D's district is 89.

Case 2: A will be dropped because More than two persons


live between the one who works in the company L and the
one who works in the company B. Step 2:
Direction (36-40): B lives somewhere above F.
A livesimmediately above F.A liveson an even numbered
floor below the sixth floor.
F's district code is three times of H's district code. So H’s
district code is 15 and F’s district code is 45.
The one who belongs to the district code 45 and the one
whose code is 15 are immediate neighbours.

36. Answer: b
37. Answer: d
38. Answer: e
39. Answer: c
40. Answer: a
Explanation:

Page 342 of 1334


Subscribe The Xpress Video Course & Mock Test Package for Bank & Insurance Exams
If there are any suggestions/ errors in our PDFs Feel Free to contact us via this email: admin@exampundit.in
Ultra Practice Bundle PDF
SBI Clerk/ RBI Assist. Mains – Reasoning
Step 3: P lives ona prime numbered floor but not on the fifth floor.
A's district code is forty-four greater than B. So (67- Three persons live between P and the one who has Nokia.
23=44) B's district code is 23 and A's district code is 67.
The difference of C's and E's district code is thirty. C's
district code is an even number which isless than E.
G's district code is thirty-five greater than B's district code.

Two persons live between N and the one who uses Motto.
K lives immediately below the one who uses Motto. More
than three persons live between K and the one who uses
Oppo. N lives somewhere above K. P does not use Motto.
Only one person lives between D and the one who uses
Oppo. The one who uses Oppo does not live immediately
below P.
Directions (41-45):

As many persons live above S is sameas below the one


who uses Sony. S does not use Motto. The one who uses
Vivo lives immediately below S. More than two persons
live between T and the one who uses Samsung. R does not
use Nokia. The one who uses LG lives immediately below
41.Answer: d
M.
42.Answer: a
43.Answer: b
44.Answer: d
45.Answer: b
Explanation:

Page 343 of 1334


Subscribe The Xpress Video Course & Mock Test Package for Bank & Insurance Exams
If there are any suggestions/ errors in our PDFs Feel Free to contact us via this email: admin@exampundit.in
Ultra Practice Bundle PDF
SBI Clerk/ RBI Assist. Mains – Reasoning
1) G, who likes One Plus, stays at the smallest odd
numbered floor.
2) The extreme left room of the floor in which G lives is
vacant..
3) A stays to the right of G and likes Lenovo.
4) One of the odd numbered floors is vacant.Hence it must
be 3rd floor.
5) Xiaomi, Samsung and Vivo as in the same order stays
in a single floor below the vacant floor.
Case 2A,2B I)and 2B II),2C II) will be dropped because
More than two persons live between T and the one who
uses Samsung.
Case 3 will be dropped because the one who uses LG lives
immediately below M.
K neither uses Samsung nor uses HTC.
Hence the case 2C I) is not valid. The final
arrangementcase 1 is follows,

Directions (46-48):
46.Answer: b
47.Answer: c
48.Answer: d
Explanation:

Page 344 of 1334


Subscribe The Xpress Video Course & Mock Test Package for Bank & Insurance Exams
If there are any suggestions/ errors in our PDFs Feel Free to contact us via this email: admin@exampundit.in
Ultra Practice Bundle PDF
SBI Clerk/ RBI Assist. Mains – Reasoning

9) H does not stay on the extreme right.


10) I and J does not like Vivo.
11) C who likes Realme stays below the one who likes
Vivo and both lives in same type of room.

6) The Mobiles Nokia, Xiaomi and ASUS are arranged


one below another in the same order in the same type of 12) B stays below D and both lives in same type of room.
room respectively. (Case II will be eliminated here.) 13) The one who likes Oppo sits to the immediate right of
7) D and B does not like Samsung and Vivo and does not D.
stay on the ground floor. 14) Only two people live on the topmost floor.
8) J likes ASUS.

Page 345 of 1334


Subscribe The Xpress Video Course & Mock Test Package for Bank & Insurance Exams
If there are any suggestions/ errors in our PDFs Feel Free to contact us via this email: admin@exampundit.in
Ultra Practice Bundle PDF
SBI Clerk/ RBI Assist. Mains – Reasoning
D stays on the second floor and owns the orange Titan
watch.
The one who owns a purple Titan watch stays on the
fourth floor.
The one who owns the golden Titan watch stays on the
topmost floor

15) F stays on the topmost floor.


16) H does not like Apple.H does not stay on the extreme
right.
17) Thus E, H and I are the persons left.I and J does not
like Vivo. hence I likes Apple, H likes samsung and E
likes Vivo. Hence the final arrangement is,

F owns a black Fastrack watch and stays on an odd-


numbered floor.
There are three floors between the floors on which C and
F stay.
There is only one floor between the floors on which F and
G stay.
The one who owns the silver Titan watch stays
Directions (49-50):
immediately above G.
49.Answer: e
C owns a red Fastrack watch.
50.Answer: d
Hence, F can live only on 5th floor.
Explanation:
E stays on the fourth floor. Also, E stays immediately
above the floor on which I stay.

Page 346 of 1334


Subscribe The Xpress Video Course & Mock Test Package for Bank & Insurance Exams
If there are any suggestions/ errors in our PDFs Feel Free to contact us via this email: admin@exampundit.in
Ultra Practice Bundle PDF
SBI Clerk/ RBI Assist. Mains – Reasoning
Hence G will like White Titan Watch and A will like
Yellow Fastrack watch since Only five of them have Titan
watches
Yellow colour watch is Fasttrack.

A stays on an even numbered floor below the even


numbered floor on which H stays.
B stays immediately above the floor on which H stay.
The one who owns the grey Fastrack watch stays on the
third floor.
A does not own a white Titan watch.

Download Puzzles Practice Questions PDF


Get More Reasoning Practice Questions PDF

Page 347 of 1334


Subscribe The Xpress Video Course & Mock Test Package for Bank & Insurance Exams
If there are any suggestions/ errors in our PDFs Feel Free to contact us via this email: admin@exampundit.in
Ultra Practice Bundle PDF
SBI Clerk/ RBI Assist. Mains – Reasoning
Puzzle Floor with Flat
Direction: (1-4) Study the following information Raipur, Mumbai and Delhi. M does not belong to Kolkata.
carefully and answer the below questions: R doesn’t live in Flat A. U and Q doesn’t belong to
There are ten persons i.e. F, I, K, M, Q, R, S, T, U and X Bangalore. The one who belongs to Pune and the one who
are living in a building having five different floors. Such belongs to Cochin live in a different flat. X doesn’t belong
that ground floor is numbered 1 and the floor above 1 is to Delhi.
numbered as 2 and so on till the topmost floor is numbered 1) Which of the following personlives immediately
as 5. Each floor has two flats, flat A and flat B. Flat A is above Mumbai?
exactly to the west of flat B. Only one person lives in a)One who belongs to Bangalore
each flat. And they belong to different city i.e. Delhi, b) F
Mumbai, Chennai, Kolkata, Bangalore, Pune, Cochin, c) U
Kanpur, Raipur and Srinagar.All the above information d) Q
isnot necessarily in thesame order. e) T
U doesn’t live onthefourth floor. I who belongs to 2) Which of the following person lives on thetopmost
Cochinlivesonthesecond floor on flat B. Two floors are floor?
between I and F. Three floors are between F and Q. As a) One who belongs to Delhi
much as above floor of Q is same as the below floor of X. b) X
K who belongs to Kanpur lives onthesame floor of U. T’s c) F
floor is not an adjacent floor of K’s floor. No floor in d) One who belongs to Bangalore
between T and S who belongs to Chennai. One floor is e) All of these
between the one who belongs to Chennai and the one who 3) Four of the five among the following are similar in
belongs to Kolkata. Two floors are between the one who such a way to form a group, which one of the following
belongs to Kolkata and the one who belongs to Raipur. R doesn’t belong to the group?
lives immediately above U and both are living in same a) F – Srinagar
flat. M and the one who belongs to Bangalore lives in b) S – Kolkata
adifferent flat. One who belongs to Bangalore and the one c) X – Kanpur
who belongs to Mumbai lives in different flat and different d) Mumbai – Cochin
floor. One who belongs to Delhi and the one who belongs e) Raipur – Kolkata
to Pune are living in thesame flat. Q does not belong to 4) How many floors are there between X and U?
Page 348 of 1334
Subscribe The Xpress Video Course & Mock Test Package for Bank & Insurance Exams
If there are any suggestions/ errors in our PDFs Feel Free to contact us via this email: admin@exampundit.in
Ultra Practice Bundle PDF
SBI Clerk/ RBI Assist. Mains – Reasoning
a) 0 The one who works in PNB does not live immediately
b) 1 above or immediately below the one who works in Axis
c) 2 bank. K and the one who works in PNB does not stay on
d) 3 the same floor. A lives immediately below G and both
e) 4 lives in the same numbered flat. More than two persons
Direction (5-9): Study the following information live between D and the one who works in Union bank and
carefully and answer the questions given below. both lives in the same numbered flat. B stays on a flat
There are 14 persons-A, B, C, D, E, F, G, H, I, J, K, L, M which is left of L. D lives above the one who works in
and N are living in a seven storey buildingbut not Union bank. D does not live on thetopmost floor. The
necessarily in the same order. The lowermost floor is number of persons living below L is two less than the
numbered 1, the floor abovethe lowermost floor is number of persons are living above the one who works in
numbered 2 and so on till the topmost floor is numbered CBI and both of them lives in thesame numbered flat. The
7. There are two flats on each floor from left to right one who lives immediately above N lives in the flat, which
numbered 1 to 2 and only one person lives in each flat. is left of the one who works in SBI. L neither works in
Each one of them works in different banks SBI, HDFC, CBI nor works in Axis bank. The one who works in CBI
ICICI, PNB, Axis Bank, Union Bank, BOB, BOM, does not live on thelowermost floor. C works in HDFC. G
Canara bank, CBI, Corporation bank, IOB, Indian bank and C live on thesame floor. The number of persons living
and HSBC but not necessarily in the same order. above E is the same as the number of personslivingbelow
There are three floors between N and the one who works the one who works in ICICI. E does not work in ICICI. I
in Indian bank. K does not work in IOB. F does not stay and F does not live on the same floor. The one who works
on flat numbered 1. Only one person lives between F and in Canara bank and the one who works in BOB are lives
the one who works in Axis bank and both lives in the same on thesame floor. D does not work in Canara bank. Two
numbered flat. F does not live in an odd-numbered floor. persons live between H and J and both lives in thesame
The one who works in Indian bank and the one who works numbered flat. J does not live on thetopmost floor. Neither
in Axis bank does not live on the same floor. Person name K nor I work in Corporation bank. N lives above the one
starts with vowel does not live on flat number 2. The who works in Indian bank and both of them live in
number of persons livingabove K, who lives in flat thesame numbered flat. L and the one who works in CBI
number 1 is same as the number of persons living below lives in the same numbered flat. B works in HSBC. The
the one who works in PNB, who lives in flat number 2.

Page 349 of 1334


Subscribe The Xpress Video Course & Mock Test Package for Bank & Insurance Exams
If there are any suggestions/ errors in our PDFs Feel Free to contact us via this email: admin@exampundit.in
Ultra Practice Bundle PDF
SBI Clerk/ RBI Assist. Mains – Reasoning
one who works in SBI does not live on the even-numbered d) HDFC
floor. e) None of these
5.Who among the following person works in 9.How many persons live between A and the one who
corporation bank? works in BOM considering both of them lives in the
a) N same numbered flat?
b) E a) None
c) G b) One
d) A c) Two
e) None of these d) Three
6.Who among the following person lives on flat e) None of these
number 1 of floor number 7? Direction (10-14): Study the following information
a) K carefully and answer the questions given below.
b) H Eight persons live in a six storied building. The ground
c) The one who works in BOM floor is numbered as 1, the first floor is numbered as 2 and
d) The one who works in BOB so on and the topmost floor numbered 6. Each floor has
e) Both (a) and (c) two flats Flat X and Flat Y. Flat X is to the west of Flat Y.
7.Which of the following statements is true? Each of the flats has the same construction, so the building
a) A lives on flat number 1 of floor number 3 can also be said to be stalked in two columns. Only one
b) More than three floors are there between H and the person can live in one flat. Each person has a different
one who works in HDFC number of chocolates, with 2 being the least and 9 being
c) M and E stay on thesame floor of different flats. the highest number of chocolates.
d) I works in BOM G, W and D live in the same column. B, who has two more
e) None is true chocolates than that of G, lives in a flat just above C.
8.If B is related to Union bank, G is related to Axis Neither J nor M lives along the same column of the
bank, in the same way, E is related to which of the building as that of W. The person who has 8 chocolates
following bank? lives alone on the floor in which he resides. The total
a) SBI number of chocolates with the persons living on the
b) BOB topmost floor is 10, with Flat X having more chocolates
c) CBI than Flat Y. M and W does not live on two consecutive

Page 350 of 1334


Subscribe The Xpress Video Course & Mock Test Package for Bank & Insurance Exams
If there are any suggestions/ errors in our PDFs Feel Free to contact us via this email: admin@exampundit.in
Ultra Practice Bundle PDF
SBI Clerk/ RBI Assist. Mains – Reasoning
floors. No one lives on the floor in which D lives and D b) D lives in the floor immediately below the one
has 7 chocolates. No two flats on the consecutive floors in who has 6 chocolates may or may not both lives in
the same column are vacant. O and G live in the same thesame type flat.
column. At least two floors are there between O’s floor c) D lives in the floor immediately above the one who
and G’s floor. The number of floors between W and G is has 2 chocolates
same as the number of floors between D and W but not on d) D and C live on different floors
consecutive floors.O doesn’t live on the 4th floor. The e) All are true
number of floors between J and D is the same as the 13.How many chocolates does O have?
number of floors between M and C. If B lives below the a) 6
floor of W, then B does not live alone on the floor. B b) 4
doesn’t live in topmost floor of any flat. B and C lives in c) 8
same flat. d) Either a) or b)
10.Who among the following person has 8 chocolates? e) Either a) or c)
a) C 14.How many chocolates does C have?
b) J a) 2
c) W b) 4
d) O c) 9
e) M d) Either a) or c)
11.In which floor does B live? e) Either a) or b)
a) 1st Direction (15-19): Study the following information
b) 2nd carefully and answer the questions given below it.
c) 3rd Twelve persons namely – B, C, D, E, F, G, H, I, J, K, L
d) 4th and M live in five floored building marked 1 to 5 from
e) 5th bottom to top respectively. Each floor hasthree flats
12.Which of the following statements regarding D is marked 01, 02 and 03 from west to east respectively. Each
false? person also likes different numbers viz.- 3, 4, 6, 7, 9, 11,
a) D lives in the same type flat in which O lives 12, 13, 14, 16, 19 and 21. Three flats are vacant and none
of the adjacent flats isvacant. Flats adjacent to each other
means either east-west or north-south.

Page 351 of 1334


Subscribe The Xpress Video Course & Mock Test Package for Bank & Insurance Exams
If there are any suggestions/ errors in our PDFs Feel Free to contact us via this email: admin@exampundit.in
Ultra Practice Bundle PDF
SBI Clerk/ RBI Assist. Mains – Reasoning
H lives three floors above E and lives immediate north- 16.What is the sum of the number liked by B and D?
west of M and both H and E lives in same numbered flat a) 25
.One person lives between the flat of the one who likes 21 b) 19
and K, who lives immediate west of F either vertically c) 21
(north/south) or horizontally (east/west) i.e. K and the one, d) 12
who likes 21, live either in same flat or in same floor e) None of these
number. M, who likes a perfect square number, lives just 17.Which of the following statement is true?
above the flat of the one who likes 21, both are living in a) The one who likes 7 lives just above I.
same numbered flat. L, who neither lives adjacent flat of b) Two persons live between the one who likes 13
M nor adjacent flat of K, lives adjacent flat of J. K neither and the one who likes 9.
likes odd number nor lives adjacent flat of H. The one who c) The one who likes 19 lives immediate north-east
likes 9 lives second to the west of the one who likes 6. G, of J.
who likes 11, lives in the flat west of I. F, who likes a d) The one who likes 4 lives three flats below K.
number in a multiple of 7, lives just above the one who e) All the above statements are not true.
likes 12, both are living in same numbered flat. J lives two 18.Which of the following pair of person and their
floors away from I, who likes 16and both lives in the same number liked correctly represents on the floor marked
numbered flat. Flat just below the one who likes 11 is 2?
vacant in the same numbered flat. I lives immediate north- a) C (19) & L (21)
east of C. One person lives between the one who likes 13 b) L (19) & J (6)
and the one who likes 9 in the same numbered flat. The c) C (21), E (9) & L (19)
one who likes 14 lives immediate east of D. One person d) K (14), F (7) & J (6)
lives between the one who likes 19 and B in the same e) None of these
numbered flat. 19.What is the position of the one who likes 12 with
15.Who among the following person likes 21? respect to the one who likes 6?
a) C a) Second to the left
b) B b) Two floors below
c) L c) Immediate north-west
d) H d) Three floorsabove
e) None of these e) None of these

Page 352 of 1334


Subscribe The Xpress Video Course & Mock Test Package for Bank & Insurance Exams
If there are any suggestions/ errors in our PDFs Feel Free to contact us via this email: admin@exampundit.in
Ultra Practice Bundle PDF
SBI Clerk/ RBI Assist. Mains – Reasoning
Direction(20-24):Study the following information floor. The sum of U’s flat number and W’s flat number is
carefully and answer the below questions. not more than 4.
Ten persons live in the seven-floor building. Each floor 20.On which floor does U live?
contains three flats I, II and III from left to right end a) Fifth
respectively. Remaining flats are vacant in the b) Second
building.Lowermost floor is numbered 1, the floor above c) Sixth
1st floor is numbered 2 and so on until the topmost floor is d) Fourth
numbered 7.Only one person lives on each flat. e) None of these
Q lives in same numbered flat as C. The direction of G’s 21.How many floors are between S and W?
flat with respect to W’s flat is the opposite direction of R’s a) 1
flat with respect to U’sflat. Two floors are between R and b) 2
P and both of them lives in the same numberedflat. The c) 3
floor in which P lives is immediate above C’s floor. One d) None
floor is between U and S, but not necessarily in the same e) None of these
numbered flat. T lives immediate below S in the same 22.Which of the following floor is fully occupied?
numbered flat. C lives in an even-numbered flat. Neither a) Second
R norS lives in the same numbered flat as C. Q lives b) Third
immediately below of M’s floor but in thedifferent flat. P c) Fourth
lives in one of the even-numbered floors. All adjacent flats d) Fifth
(north, south, east and west) of U is a vacant flat. One floor e) None of these
is between Q and R but both of them not lives in the same 23.In the following who among them does not belongs
flat. The number of floors between U and M is the same to the group?
as between Q and W, but not necessarily in the same a) C
numbered flat. Two floors are between G and W, but not b) U
necessarily in the same numbered flat. Q lives below R. U c) Q
lives in even-numbered flat but neither on floor number 1 d) G
nor on floor number 7. The direction of M’s flat with e) P
respect to Q’s flat is not the same as the direction of Q’s 24.Which of the following floor is fully vacant?
flat with respect to P’s flat.R does not live on the third a) Second

Page 353 of 1334


Subscribe The Xpress Video Course & Mock Test Package for Bank & Insurance Exams
If there are any suggestions/ errors in our PDFs Feel Free to contact us via this email: admin@exampundit.in
Ultra Practice Bundle PDF
SBI Clerk/ RBI Assist. Mains – Reasoning
b) Third and G either horizontally (east/west) or vertically
c) Fifth (north/south). The one who works in IBM lives in any of
d) seventh the odd-numbered flat. The one who works in SBI neither
e) None of these lives adjacent vacant flat nor lives west to C. D and the
Direction (25-29): Study the following information one who works in DRDO lives on the adjacent floor. C
carefully and answer the below questions. doesn't live onthetopmost floor. B, who works in NAL,
Ten persons live in four floored building marked 1 to 4 lives two places away from J. The one who works in HP
from bottom to top respectively. Each floor has three flat doesn’t live on the immediate west of J.
marked 01, 02 & 03 from west to east respectively. One 25.F works in which of the following organization?
person lives in one flat only. Some flats are vacant. None a) HP
of the adjacent flats is vacant. Each person also works in b) PNB
different organization viz. NAL, SAIL, IBM, HP, BHEL, c) IG
DRDO, PNB, SBI, IG and TCS. All the information d) SBI
arenot necessarily in thesame order. e) None of these
One person lives between the one who works in HP and 26.Which of the following combination of person lives
the one who works in IG of the same flat. Flat just above on thefloor marked 2?
G is vacant in same numbered flat. J doesn’t live in even- a) H and C
numbered flat. K lives on odd-numbered floor immediate b) C, B and J
west of the one who works in BHEL. E and the one who c) H and B
works in IG live in different flats. The one who works in d) C and J
BHEL lives two floors away from J inthe same flat. The e) None of these
one who works in NAL and J lives on the same floor. One 27.Which of the following statement is not true?
person lives between E and the one who works in NAL a) G lives two floors away from the one who works
inthe same flat. C lives in any of the even floors above the in DRDO
one who works in IG inthesame flat. H lives above the one b) Two person lives between E and the one who
who works in SAIL in same flat number. Flat number of works in SBI
the one who works in HP and floor number of H is the c) The one who works in PNB lives immediate east
same. The one who works in TCS lives immediate east of of H
I. One person lives between the one who works in TCS

Page 354 of 1334


Subscribe The Xpress Video Course & Mock Test Package for Bank & Insurance Exams
If there are any suggestions/ errors in our PDFs Feel Free to contact us via this email: admin@exampundit.in
Ultra Practice Bundle PDF
SBI Clerk/ RBI Assist. Mains – Reasoning
d) No person lives between B and the one who works lives on the same flat. Only one person lives between D
in DRDO and the one who is adjacent (east or west) to R in the same
e) All the above given statements are true numbered flat. Dlives above R. G lives immediately above
28.Which among the following flat does not contains M in the same flat. There is no flat to the east of P. R and
vacant floors? D does not live on thesame flat. X and L does not live on
a) Flat 1 thesame floor.
b) Flat 2 30.How many persons live between the one who lives
c) Flat 3 adjacent to X in different flat and Min the same
d) Flat 1and Flat 3 numbered flat?
e) None of these. a) One
29.D works in which of the following organization? b) Two
a) HP c) Three
b) PNB d) Four
c) IG e) None of these
d) SBI 31.If M is related to P, D is related to K, in the same
e) None of these way, S is related to who among the followingperson?
a) X
Directions (30-32): Study the following information to b) R
answer the given questions: c) Z
Ten persons are living in a building with five floors d) L
(Numbered 1 to 5). Lowermost floor is numbered 1, the e) None of these
floor above 1st floor is numbered 2 and so on until the 32.Who among of the following person lives
topmost floor is numbered 5. There are two flats on each immediately below X?
floor. Flat A is to the west of Flat B. a) The one who livessame floor with G
Z is adjacent (east or west) to the one who lives b) The one who livessame floor with D
immediately below S and both Z and S lives in a different c) P
flat. Only two persons live above K on the same flat. Only d) No one
one person lives between K and P both lives in thesame e) R
flat. Three persons live between S and L and both of them

Page 355 of 1334


Subscribe The Xpress Video Course & Mock Test Package for Bank & Insurance Exams
If there are any suggestions/ errors in our PDFs Feel Free to contact us via this email: admin@exampundit.in
Ultra Practice Bundle PDF
SBI Clerk/ RBI Assist. Mains – Reasoning
Directions (33-36): Study the following information d) I
carefully and answer the below questions. e) None of these
Eight persons are living in six different floors of two flats. 35.Who among the following person lives second to the
The two flats namely A and B, which is east of A. above of J in thesame flat?
Bottommost floor is numbered 1 and floor above the 1st a) I
floor is numbered 2nd and so on. The topmost floor is six. b) E
Eight persons live in different flats on different floors and c) K
the remaining four flats are vacant.Only one person lives d) G
on each flat. e) None of these
J lives in an odd-numbered floor in flat A. Two floors are 36.Find the odd one out of the rest?
between J and G but both of them lives in different flats. I a) K
lives immediately below G in same numbered flat. Two b) E
floors are between I and F who only lives on the floor and c) J
both of them lives in thesame flat. F lives immediate d) F
above floor H but both lives in different flats. L lives two e) H
floors below of E. Both L and E live inadifferent flat. Two Directions (37-40): Read the following information
floors are between K and L, but not necessarily in the same carefully and answer the given questions.
flat. One of the even-numbered floor is fully vacant. K Eight persons A, B, C, D, E, F, G, and H are living on
lives above the H, but not necessarily in the same flat. three different floors of a building. The ground floor of the
33.How many floors are between E and H? building is numbered as floor 1, the floor above floor 1 is
a) 3 numbered as floor 2 and so on. There are three flats on
b) 1 each floor named flat 1, flat 2, and flat 3. The flats are in
c) 5 such a way that flat 1 is to the west of flat 2, flat 2 is to the
d) 2 west of flat 3. Flat 1 on the second floor is exactly above
e) None of these the flat 1 on the first floor and other flats are placed in the
34.Who livesonthesame floor as E? same way.Only one person lives on each flat.
a) G There is a one-floor gap between G and C and both are
b) L living in the same numbered flat. G is living in an even-
c) F numbered flat.

Page 356 of 1334


Subscribe The Xpress Video Course & Mock Test Package for Bank & Insurance Exams
If there are any suggestions/ errors in our PDFs Feel Free to contact us via this email: admin@exampundit.in
Ultra Practice Bundle PDF
SBI Clerk/ RBI Assist. Mains – Reasoning
The flat of E, who is living immediately above A in the e) The persons living on floor 1 are F, C, and H.
same flat, is to the west of the flat of B. E is not living on Directions (41-45): Study the following information
the second floor. carefully and answer the questions given below.
H is living to the east of F, but not to the east of A. Ten flats viz. M, N, O, P, Q, R, S, T, U and V are in five-
The flat of C is to the south-west to the flat of D. storey building and each floor has two blocks A and
37.On which floor, D is living? B,where block A is to the west of block B. These ten flats
a) 1 are not necessarily in the same order. The breadth of all
b) 2 the flats remains same i.e., 16m whereas the length of each
c) 3 flat changes. The total area of the floor is 736m2. The
d) Either 2 or 3 lowermost floor is numbered 1 and the floor above 1st
e) None of these floor is numbered 2 and so on and the topmost floor is
38.What is the flat and floor number respectively numbered 5. The flat to thewest in floor 1 is immediately
which is vacant? below the flat to the west in floor 2 and soon.
a) second flat on the first floor Note: Area = length * breadth
b) third flat on the second floor Flat O is in an even-numbered floor and the Flat with
c) first flat on the third floor length 28m is to the east of Flat O. There are two floors
d) second flat on the second floor between the flats P and N and P is to the north of N. Flat
e) third flat on the third floor T is two floors above the flat with 18m lengthin the same
39.Who is living immediate above H? flat. Flat P is to the east of the flat having length 21m. The
a) D flat immediately below the flat length of 25m is 10m long
b) G and is to the east of Flat M. Only the topmost floor has
c) A equal length of flats. Flat Q is to the west of Flat U. Flat S
d) E is 15m long. Flat V is not to the east of the flat with length
e) None of these 36m.
40.Which of the following statement is false? 41. What is the length of the Flat N?
a) G is living on floor 3. a) 36m
b) A is living in flat 1. b) 28m
c) D is living to the south-east of E. c) 31m
d) The persons living on flat 3 are E, G, and B. d) 10m

Page 357 of 1334


Subscribe The Xpress Video Course & Mock Test Package for Bank & Insurance Exams
If there are any suggestions/ errors in our PDFs Feel Free to contact us via this email: admin@exampundit.in
Ultra Practice Bundle PDF
SBI Clerk/ RBI Assist. Mains – Reasoning
e) 15m e) None
42. Which of the following flat has theleast area? Directions (46-50): Study the following information
a) Flat O carefully and answer the questions given below Four
b) FlatR blocks viz. A, B, C and D are in the straight line from west
c) FlatS to east and 12 persons viz. O, P, Q, R, S, T, U, V, W, X,
d) FlatT Y and Z are living on different houses of the blockbut not
e) FlatQ necessarily in the same order. Block A and block C are
43. What will be the difference in area between the facing toward north and block B and blockD are facing
flats M and V? towards thesouth and there isacertain number of floors in
a) 128m2 each block. The bottom-most floor is numbered 1 and the
b) 48 m2 floor above is 2 and so on. Each person has different pets
c) 80 m2 viz. Cat, Dog, Duck, Fish, Goat, Hamster, Hen, Horse,
d) 208 m2 Mouse, Parrot, Pigeon and Sheep, but not necessarily in
e) 176 m2 the same order. Each floor has only one house and not
44. Four of the five are alike in acertain way and hence more than one person lives in each house. No houses
forms a group. Find the one that does not belong to the remain unoccupied.
group? The person who has Horse lives in block B. X lives in an
a) FlatQ odd numbered floor on a block facing north. P lives
b) FlatP immediately below X and is to the right of S. The person
c) FlatN who has Goat lives in a block facing south andthe block
d) FlatM hasthesecond-lowest house. Q lives to the right of W, who
e) FlatT has Hen. There are no floors to the immediate right of the
45. How many meters are to be reduced in flat V to persons having Parrot and Hamster, who are the
equalise the length with the flat neighbouring it on the immediate neighbours in the same block. The person
same floor? having Duck and Mouse are facingthesame direction but
a) 3m on different blocks. U who has cat lives on the bottommost
b) 10m floor of block A. The person having Pigeon lives
c) 4m immediately above the person having Goat. O and R lives
d) 13m on the topmost floor of different blocks. Y who has Dog

Page 358 of 1334


Subscribe The Xpress Video Course & Mock Test Package for Bank & Insurance Exams
If there are any suggestions/ errors in our PDFs Feel Free to contact us via this email: admin@exampundit.in
Ultra Practice Bundle PDF
SBI Clerk/ RBI Assist. Mains – Reasoning
lives only with S. The person having Sheep lives e) Pigeon
immediately above the person having Mouse. Neither the 48. How many numbersof floors are there in block C?
person with pigeon nor the person with Goat lives with a) 2
V.Z and O doesn’t live on the same block. T, with b) 3
Hamster, lives on the topmost floor among all other c) 4
blocks.V and the person having fish live on the same d) 1
block. e) None of these
Block A and blockB have even number of floors whereas 49. Four of the five are alike in a certain way and hence
block C and blockD havethe same number of floors. The forms a group. Find the one that does not belong to the
highest numbered house block and the lowest numbered group?
house block are together. a) Z
46. Who among the following person has Fish? b) The person having Hen.
a) V c) S
b) X d) The person having Dog.
c) O e) P
d) Q 50. If X is related to the person having Sheep and W is
e) R related to the person with Dog, then who among the
47. Which of the following animal is to the immediate following is related to S?
right of the person having Goat? a) The person with Goat.
a) Duck b) P
b) Parrot c) The person with Mouse.
c) Mouse d) V
d) Fish e) U

Puzzle Floor with Flat – Answer and Explanation


Answers: Explanation:
1) B I who belongs to Cochinlivesonthesecond floor on flat B.
2) E Two floors are between I and F.
3) E
4) B
Page 359 of 1334
Subscribe The Xpress Video Course & Mock Test Package for Bank & Insurance Exams
If there are any suggestions/ errors in our PDFs Feel Free to contact us via this email: admin@exampundit.in
Ultra Practice Bundle PDF
SBI Clerk/ RBI Assist. Mains – Reasoning

Three floors are between F and Q.


As much as above floor of Q is same as the below floor of
X.

One floor is between the one who belongs to Chennai and


the one who belongs to Kolkata.
M does not belong to Kolkata.
Hence R belongs to Kolkata in all cases.
Two floors are between the one who belongs to Kolkata
and the one who belongs to Raipur.
Q does not belong to Raipur, Mumbai and Delhi.
Hence T belongs to Raipur in all cases.
M and the one who belongs to Bangalore lives in a
different flat. U and Q doesn’t belong to Bangalore.
By this condition, X belongs to Bangalore in case 1a and
case 1b, F belongs to Bangalore in case 2a, case 2b.
One who belongs to Bangalore and the one who belongs
U doesn’t live on thefourth floor. K who belongs to
to Mumbai lives in different flat and different floor.
Kanpur lives on the same floor of U.
One who belongs to Pune and the one who belongs to
T’s floor is not an adjacent floor of K’s floor.
Cochin live in adifferent flat. One who belongs to Delhi
No floor in between T and S who belongs to Chennai.
and the one who belongs to Pune are living on the same
R lives immediately above U and both are living in same
flat.By this condition, case 1b and case 2b got eliminated.
flat.
X doesn’t belong to Delhi.
R doesn’t live in Flat A.
Page 360 of 1334
Subscribe The Xpress Video Course & Mock Test Package for Bank & Insurance Exams
If there are any suggestions/ errors in our PDFs Feel Free to contact us via this email: admin@exampundit.in
Ultra Practice Bundle PDF
SBI Clerk/ RBI Assist. Mains – Reasoning
By this condition, case 2a got eliminated.
Hence case 1a is the final arrangement.

There are three floors between N and the one who works
in Indian bank. N lives above the one who works in Indian
Hence the Final Arrangement is bank and both of them live in the same numbered flat. The
one who lives immediately above N lives in the flat, which
is left of the one who works in SBI. The one who works
in SBI does not live on the even-numbered floor. This
implies the one who works in SBI lives on flat number 2
of floor numbered 7.

5. B
6. E
7. C
8. B
9. E
Explanation:
Direction (5-9):

F does not stay on flat number 1. Only one person lives


between F and the one who works in Axis bank and both

Page 361 of 1334


Subscribe The Xpress Video Course & Mock Test Package for Bank & Insurance Exams
If there are any suggestions/ errors in our PDFs Feel Free to contact us via this email: admin@exampundit.in
Ultra Practice Bundle PDF
SBI Clerk/ RBI Assist. Mains – Reasoning
lives in thesame numbered flat. F does not live in an odd- works in CBI lives in the same numbered flat. L neither
numbered floor. The one who works in Indian bank and works in CBI nor works in Axis bank. The one who works
the one who works in Axis bank does not live on thesame in CBI does not live on the lowermost floor.
floor. Person name starts with vowel does not live on flat
Floor number of F will be 2, 4 and 6. There are three number 2. A lives immediately below G. Hence person
possibilities. live on flat number 1-A, E, I, K, N, G and B
So, D must lives on flat number 2. More than two person
lives between D and the one who works in Union bank
and both lives in the same numbered flat. D lives above
the one who works in Union bank. D does not live on
thetopmost floor.
Case 1:
The number of persons living above K who lives in flat
number 1 is the same as the number of persons are living
below the one who works in PNB who lives in flat number
2. K and the one who works in PNB does not stay on the
same floor. The one who works in PNB does not live
immediately above or immediately below the one who
works in Axis bank.

C works in HDFC. G and C live on the same floor. A lives


immediately below G and both lives in the same numbered
flat.

B stays on a flat which is left of L. Hence L lives on flat


numbered 2.Bworks in HSBC. The number of persons
living below L is two less than the number of persons
livingabove the one who works in CBI. L and the one who

Page 362 of 1334


Subscribe The Xpress Video Course & Mock Test Package for Bank & Insurance Exams
If there are any suggestions/ errors in our PDFs Feel Free to contact us via this email: admin@exampundit.in
Ultra Practice Bundle PDF
SBI Clerk/ RBI Assist. Mains – Reasoning
works in Axis bank.

B stays on a flat which is left of L. Hence L lives on flat


numbered 2.Bworks in HSBC. Number of personsare
living below L is two less than the number of personsare
living above the one who works in CBI. L and the one who

Case 2: works in CBI lives in the same numbered flat. L neither

Number of persons living above K who lives in flat works in CBI nor works in Axis bank. The one who works

number 1 is same as the number of persons are living in CBI does not live on the lowermost floor.

below the one who works in PNB who lives in flat number Person name starts with vowel does not live on flat

2.K and the one who works in PNB does not stay on the number 2. A lives immediately below G. Hence person

same floor. The one who works in PNB does not live live on flat number 1-A, E, I, K, N, G and B

immediately above or immediately below the one who So, D must live on flat number 2. More than two-person
lives between D and the one who works in Union bank
and both lives in the same numbered flat. D lives above
the one who works in Union bank. D does not live on
thetopmost floor.

Page 363 of 1334


Subscribe The Xpress Video Course & Mock Test Package for Bank & Insurance Exams
If there are any suggestions/ errors in our PDFs Feel Free to contact us via this email: admin@exampundit.in
Ultra Practice Bundle PDF
SBI Clerk/ RBI Assist. Mains – Reasoning
C works in HDFC. G and C live on the same floor. A lives
immediately below G.

Number of personsare living above K who lives in flat


The number of persons living above E is the same as the number 1 is same as the number of persons are living
number of persons living below the one who works in below the one who works in PNB who lives in flat number
ICICI. E does not work in ICICI. I and F does not live on 2. K and the one who works in PNB does not stay on the
thesame floor. same floor. The one who works in PNB does not live
immediately above or immediately below the one who
works in Axis bank.

B stays on a flat which is left of L. Hence L lives on flat


numbered 2.Bworks in HSBC. The number of persons
living below L is two less than the number of persons
living above the one who works in CBI. L and the one who
Case 3:
works in CBI lives in the same numbered flat. L neither
Page 364 of 1334
Subscribe The Xpress Video Course & Mock Test Package for Bank & Insurance Exams
If there are any suggestions/ errors in our PDFs Feel Free to contact us via this email: admin@exampundit.in
Ultra Practice Bundle PDF
SBI Clerk/ RBI Assist. Mains – Reasoning
works in CBI nor works in Axis bank. The one who works
in CBI does not live on the lowermost floor.
Person name starts with vowel does not live on flat
number 2. A lives immediately below G. Hence person
live on flat number 1-A, E, I, K, N, G and B
So, D must live on flat number 2. More than two-person
lives between D and the one who works in Union bank
and both lives in the same numbered flat. D lives above
the one who works in Union bank. D does not live on
thetopmost floor.

The number of persons living above E is the same as the


number of persons living below the one who works in
ICICI. E does not work in ICICI. I and F does not live on
thesame floor. Two persons live between H and J and both
lives in thesame numbered flat. J does not live on
thetopmost floor. Neither K nor I work in Corporation
C works in HDFC. G and C live on the same floor. A lives bank.
immediately below G. K does not work in IOB. The one who works in Canara
bank and the one who works in BOB are on thesame floor.
D does not work in Canara bank.
So the final arrangement will be as follows.

Page 365 of 1334


Subscribe The Xpress Video Course & Mock Test Package for Bank & Insurance Exams
If there are any suggestions/ errors in our PDFs Feel Free to contact us via this email: admin@exampundit.in
Ultra Practice Bundle PDF
SBI Clerk/ RBI Assist. Mains – Reasoning
number of chocolates, with 2 being the least and 9 being
the highest number of chocolates.
From the above statements, the possible combinations
are (8,2), (7,3) and (6,4).
The person who has 8 chocolates lives alone on the floor
in which he resides. No one lives on the floor in which D
lives and D has 7 chocolates.
From the above statements, it is clear that the persons
who have 8 and 7 chocolates live alone. So, (6,4) is the
only possibility of being at the topmost floor.

10. B
11. D
12. C
13. D
14. D
Explanation:
G, W and D live in the same column. The number of floors
between W and G is the same as the number of floors
between D and W but not on consecutive floors. No one
lives on the floor in which D lives and D has 7 chocolates.
O and G live in the same column. At least two floors are
there between O’s floor and G’s floor.
O doesn’t live on the 4th floor.hence O lives in 6th floor.

The total number of chocolates with the persons living on


the topmost floor is 10, with Flat X having more
chocolates than Flat Y. Each person consists ofa different
Page 366 of 1334
Subscribe The Xpress Video Course & Mock Test Package for Bank & Insurance Exams
If there are any suggestions/ errors in our PDFs Feel Free to contact us via this email: admin@exampundit.in
Ultra Practice Bundle PDF
SBI Clerk/ RBI Assist. Mains – Reasoning

B, who has two more chocolates than that of G, lives in a No two flats on the consecutive floors in the same

flat just above C. column are vacant.Case I placing B on thesecond floor

B doesn’t live in topmost floor of any flat. is not applicable.

B and C lives in same flat. Case II: Placing B onthe third floor

B-G=2. The possible combinations for (B,G) are (9,7) B, who has two more chocolates than that of G, lives in a

(8,6) (7,5) (6,4) (5,3) and (4,2). The pairs in which flat just above C. The number of floors between J and D

either 7 or 6 or 4 gets cancelled because the person with is same as the number of floors between M and C. Neither

7 chocolates lives alone and G neither has 6 nor 4 J nor M lives along the same column of the building as

chocolates. B doesn’t live in topmost floor of any flat. that of W. M and W does not live on two consecutive

This implies that B don’t have 4 or 6 chocolates. floors. No two flatson the consecutive floors in the same

So the only possibility is (B,G)=(5,3). column are vacant.

Case I: Placing B on thesecond floor


B, who has two more chocolates than that of G, lives in a
flat just above C.If B lives below the floor of W, then B
does not live alone on the floor.So in the second floor,
two-person will live. No two flats on the consecutive
floors in the same column are vacant.

The position of D violates the condition that no two


flatson the consecutive floors in the same column are

Page 367 of 1334


Subscribe The Xpress Video Course & Mock Test Package for Bank & Insurance Exams
If there are any suggestions/ errors in our PDFs Feel Free to contact us via this email: admin@exampundit.in
Ultra Practice Bundle PDF
SBI Clerk/ RBI Assist. Mains – Reasoning
vacant. So, Case II placing B on the third floor is also
not applicable.
Case III: Placing B on the fourth floor
B, who has two more chocolates than that of G, lives in a
flat just above C. The number of floors between J and D
is same as the number of floors between M and C. Neither
J nor M lives along the same column of the building as
that of W. M and W does not live on two consecutive
floors. No two flats on the consecutive floors in the same We have:
• H lives three floors above E and lives immediate
column are vacant. The person who has 8 chocolates lives
alone on the floor in which he resides. north-west of M and both H and E lives in same
numbered flat.
• M, who likes a perfect square number, lives just
above the flat of the one who likes 21, both are
living in same numbered flat.
That means, in case (1) H lives in the flat marked 501, in
case (2) H lives in the flat marked 502, in case (3) H lives
in the flat marked 401, in case (4) H lives in the flat
marked 402.
These are the two possible arrangements.
• One person lives between the flat of the one who
15. A
likes 21 and K, who lives immediate west of F
16. E
either vertically (north/south) or horizontally
17. E
(east/west) i.e. K and the one, who likes 21, live
18. E
either in same flat or in same floor number.
19. D
• K neither likes odd number nor lives adjacent flat
Explanation:
of H.
That means, in case (1) K lives in the flat marked 102, in
case (2) K lives in the flat marked 301, in case (3) K lives
in the flat marked 502, in case (4) K lives in the flat
marked 201.
Page 368 of 1334
Subscribe The Xpress Video Course & Mock Test Package for Bank & Insurance Exams
If there are any suggestions/ errors in our PDFs Feel Free to contact us via this email: admin@exampundit.in
Ultra Practice Bundle PDF
SBI Clerk/ RBI Assist. Mains – Reasoning
Based onthe above given information we have: Since, M likes perfect square and only remaining perfect
square is 4, thus M likes 4.
Based onthe above given information we have:

Again, we have:
• F, who likes a number in a multiple of 7, lives just
above the one who likes 12, both are living in same
numbered flat.
That means, case (1) is not valid.
• L, who neither lives adjacent flat of M nor adjacent
flat of K, lives adjacent flat of J.
• J lives two floors away from I who likes 16and
both lives in the same numbered flat.
• I lives immediate north-east of C.
Case (1) are not valid as F, who likes a number
• G, who likes 11, lives in the flat west of I.
multiple of 7, lives just above the one who likes 12, case
• Flat just below the one who likes 11 is vacant in
(4) & case (2) are not valid as K doesn’t like an odd
the same numbered flat.
number
That means, in case (2) L lives in the flat marked 203, in
Again, we have:
case (3a) L lives in the flat marked 102, in case (3b) L
• The one who likes 14 lives immediate east of D.
lives in the flat marked 203, in case (4) L lives in the flat
• Three flats are vacant
marked 103.
• One person lives between the one who likes 19 and
• One person lives between the one who likes 13 and
B in the same numbered flat.
the one who likes 9 in the same numbered flat.
That means, in case (3b) B likes 12 and case (3a) is
• The one who likes 9 lives second to thewest of the
not valid.
one who likes 6.
Since, F likes a number multiple of 7, that means F likes
Since, K doesn’t like anodd number, thus in case (3a) &
7 and D likes 3.
case (3b) E likes 9 and H likes 13, case (2) &case (4) is
not valid.
Page 369 of 1334
Subscribe The Xpress Video Course & Mock Test Package for Bank & Insurance Exams
If there are any suggestions/ errors in our PDFs Feel Free to contact us via this email: admin@exampundit.in
Ultra Practice Bundle PDF
SBI Clerk/ RBI Assist. Mains – Reasoning
Based onthe above given information we have the final Ten persons live in the seven-floor building. Each floor
arrangement as follows: contains three flats I, II and III from left to right end.
Remaining flats are vacant in the building.Lowermost
floor is numbered 1, the floor above 1st floor is numbered
2 and so on until the topmost floor is numbered 7.Only
one person lives on each flat.
The floor in which P lives is immediate above C’s floor.
P lives in one of the even-numbered floors. C lives in
even-numbered flat.Two floors are between R and P and
both of them lives in the same numberedflat.Neither R nor
S lives in the same numbered flat as C. R does not live on
Case (3a) is not valid as one person lives between the the third floor.
one who likes 19 and B.
20. C
21. B
22. C
23. E
24. D
Explanation:

One floor is between Q and R but both of them not lives


in the same flat.Q lives in same numbered flat as C. Q lives
immediate below of M’s floor but in thedifferent flat. Q

Page 370 of 1334


Subscribe The Xpress Video Course & Mock Test Package for Bank & Insurance Exams
If there are any suggestions/ errors in our PDFs Feel Free to contact us via this email: admin@exampundit.in
Ultra Practice Bundle PDF
SBI Clerk/ RBI Assist. Mains – Reasoning
lives below R. By this condition, case 2a and case 4a got
eliminated and new cases added.

One floor is between U and S, but not necessarily in the


same numbered flat.
All adjacent flats (north, south, east and west) of U is a T lives immediate below S in the same numbered flat.
vacant flat.
U lives in even-numbered flat but neither on floor number
1 nor on floor number 7.
So, U lives in between vacant flats from 4 sides such
north, south, west and east.

Page 371 of 1334


Subscribe The Xpress Video Course & Mock Test Package for Bank & Insurance Exams
If there are any suggestions/ errors in our PDFs Feel Free to contact us via this email: admin@exampundit.in
Ultra Practice Bundle PDF
SBI Clerk/ RBI Assist. Mains – Reasoning
➔R is south-east of U, Then G is north-west of W
(opposite direction)

Case 1b and case 3b got eliminated because there are two


• The number of floors between U and M is the same floors between W and G but we cannot fix G in P’s floor
as between Q and W, but not necessarily in the since only one person lives in the floor in which P lives.
same numbered flat. The direction of M’s flat with respect to Q’s flat is not
• Two floors are between G and W, but not same as the direction of Q’s flat with respect to P’s flat.
necessarily in the same numbered flat. Case—1: M is north-east of Q and Q is north-east of P
• The direction of G’s flat with respect to W’s flat is (same direction, hence eliminated)
the opposite direction of R’s flat with respect to Case-1-a: M is north-west of Q and Q is north-east of
U’sflat. P (different direction)
Hints: Case-3: M is north-west of Q and Q is north-west of P
➔If R is south-west of U, Then G is north-east of W (same direction, hence eliminated)
(opposite direction) Case-3-a: M is north-east of Q and Q is north-west of
P (different direction)

Page 372 of 1334


Subscribe The Xpress Video Course & Mock Test Package for Bank & Insurance Exams
If there are any suggestions/ errors in our PDFs Feel Free to contact us via this email: admin@exampundit.in
Ultra Practice Bundle PDF
SBI Clerk/ RBI Assist. Mains – Reasoning
25. D
26. A
27. E
28. A
29. E
Explanation:

We have:
• K lives on an odd numbered floor immediate west
of the one who works in BHEL.
The sum of U’s flat number and W’s flat number is not
• The one who works in BHEL lives two floors
more than 4.by this condition, case 3a got eliminated.
away from J in the same flat.
• J is not lives on an even numbered flat.
That means, in case (1) K lives in the flat marked 102 and
J lives in the flat marked 303, in case (2) K lives in the flat
302 and J lives in the flat marked 103.
• The one who works in NAL and J lives on the
same floor.
• B, who works in NAL lives two places away from
J. That means, in case (1) B lives in the flat marked
301, in case (2) B lives in the flat marked 101.

Page 373 of 1334


Subscribe The Xpress Video Course & Mock Test Package for Bank & Insurance Exams
If there are any suggestions/ errors in our PDFs Feel Free to contact us via this email: admin@exampundit.in
Ultra Practice Bundle PDF
SBI Clerk/ RBI Assist. Mains – Reasoning
Based on theabove given information we have:

Again, we have:
• One person lives between E and the one who
works in NAL in the same flat.
• E and the one who works in IG live in different
flats.
• C lives in any of the even floors above the one who
• One person lives between the one who works in
works in IG in the same flat. C is not lives on
HP and the one who works in IG of the same flat.
thetopmost floor
That means, in case (1) C lives just above K, in case (2A • The one who works in HP doesn’t live on the

i), case (3A i) & case (4a i) C lives in flat marked 202. immediate west of J.

In case (2A ii), case (3A ii) and in case (4A ii) C lives in By this condition, case 1 has one more possibility. i.e case

flat marked 203. 1a in which the one who works in HP lives in flat marked
402 and case 2A ii), 3A ii) and 4A ii) got eliminated.
Since, the one who works in HP doesn’t live on the
immediate west of J, thus case (1) is not valid.

Page 374 of 1334


Subscribe The Xpress Video Course & Mock Test Package for Bank & Insurance Exams
If there are any suggestions/ errors in our PDFs Feel Free to contact us via this email: admin@exampundit.in
Ultra Practice Bundle PDF
SBI Clerk/ RBI Assist. Mains – Reasoning

Again, we have:
• The one who works in TCS lives immediate east
of I.
• One person lives between the one who works in
• Flat number of the one who works in HP and floor TCS and G either horizontally (east/west) or
number of H is the same. vertically (north/south)
• H lives above the one who works in SAIL in same • Flat just above G is vacant in same numbered flat.
flat number. That means, in case (1a) G works in BHEL and the one
That means, in case (2A i), case (3A i) & case (4A i) J who works in TCS lives in the flat marked 403, case (2A
works in SAIL and H lives in the flat marked 203 and in i), case (3A i) and case (4A i) are not valid.
case 1A, H lives in the flat marked 201 and E works in
SAIL.

Based on theabove given information we have:

Page 375 of 1334


Subscribe The Xpress Video Course & Mock Test Package for Bank & Insurance Exams
If there are any suggestions/ errors in our PDFs Feel Free to contact us via this email: admin@exampundit.in
Ultra Practice Bundle PDF
SBI Clerk/ RBI Assist. Mains – Reasoning
In Case 2A i) and in case 4A i) there is no place to fix I 30. B
hence these cases are not valid. In case 3A i) A,there is no 31. C
place to fix the person who works in TCS. In case 3A i) 32. B
B, there is no place to fix I. Hence these cases are also not Explanation:
valid. Directions (30-32):
• The one who works in SBI neither lives adjacent
vacant flat nor lives west to C.
That means, the one who works in SBI lives in the flat
marked 401.
• D and the one who works in DRDO lives on
anadjacent flat.
That means, D works in TCS.
• The one who works in IBM lives in any of the odd
numbered flat.
That means,the one who works in IBM lives in the flat There is no flat to the east of P. Only two persons live
marked 201. above K on the same flat. Only one person lives between
The only person left is F, hence F works in SBI. K and P both lives in thesame flat.
The only organisation left is PNB and hence C works in
PNB.
Based on theabove given information we have thefinal
arrangement as follows:

Three persons live between S and L and both of them lives


on the same flat. Z is adjacent (east or west) to the one
Page 376 of 1334
Subscribe The Xpress Video Course & Mock Test Package for Bank & Insurance Exams
If there are any suggestions/ errors in our PDFs Feel Free to contact us via this email: admin@exampundit.in
Ultra Practice Bundle PDF
SBI Clerk/ RBI Assist. Mains – Reasoning
who lives immediately below S and both Z and S lives in
a different flat. Only one person lives between D and the
one who is adjacent (east or west) to Rin the same
numbered flat. D lives above R. G lives immediately
above M in the same flat
R and D does not live on the same flat. X and L does not
live on the same floor.

J lives in an odd numbered floor in flat A. Two floors are


between J and G but both of them lives in different flats. I
lives immediately below G in same numbered flat.

Case 1 will be dropped because X and L does not live on


thesame floor. case 1b will be dropped because G lives
immediately above of M.
33. A
34. D
35. B
36. D
Explanation: Two floors are between I and F who only lives on the floor
and both of them lives in thesame flat. F lives immediate
above floor of H but both lives in different flats.

Page 377 of 1334


Subscribe The Xpress Video Course & Mock Test Package for Bank & Insurance Exams
If there are any suggestions/ errors in our PDFs Feel Free to contact us via this email: admin@exampundit.in
Ultra Practice Bundle PDF
SBI Clerk/ RBI Assist. Mains – Reasoning

One of the even numbered floors is fully vacant. L lives


two floors below of E. Both L and E live inadifferent flat.
Two floors are between K and L, but not necessarily in the 37. B
same flat. 38. D
K lives above the H, but not necessarily in the same flat. 39. A
40. D
Explanation:
There is a one-floor gap between G and C and both are
living in the same numbered flat. G is living in an even-
numbered flat.
The flat of C is to the south-west to the flat of D. So, Case
2 can be eliminated as there is no possibility to place
So the final arrangement is,
D. Case 1 has one more possibility.

Page 378 of 1334


Subscribe The Xpress Video Course & Mock Test Package for Bank & Insurance Exams
If there are any suggestions/ errors in our PDFs Feel Free to contact us via this email: admin@exampundit.in
Ultra Practice Bundle PDF
SBI Clerk/ RBI Assist. Mains – Reasoning
The flat of E, who is living immediately above A in the Floor
Flats
same flat, is to the west of the flat of B. E is not living on
floor 2. By this condition, case 1a got eliminated. Q U
5 23m 23m
T P
4 21m 25m
M R
3 36m 10m
O V
2 18m 28m

H is living to the east of F, but not to the east of A. So the S N

final arrangement is, 1 15m 31m


46. D
47. C
48. B
49. E
50. A
Solution:

Block A Block B Block C Block D


(Facing (Facing (Facing (Facing
Floors
North) South) North) South)
41. C T
42. B 4 Hamster
43. A X O R
44. D 3 Parrot Sheep Pigeon
45. B P S V Z
Solution: 2 Duck Horse Mouse Goat
U Y Q W
1 Cat Dog Fish Hen

Page 379 of 1334


Subscribe The Xpress Video Course & Mock Test Package for Bank & Insurance Exams
If there are any suggestions/ errors in our PDFs Feel Free to contact us via this email: admin@exampundit.in
Ultra Practice Bundle PDF
SBI Clerk/ RBI Assist. Mains – Reasoning
Puzzle Month
Directions (1-4): Study the following information 2) Who attends seminar on 21st April?
carefully and answer the questions given below it. A) J
Ten persons attends seminar in four different month viz.- B) B
January, April, May and September. Seminar in each C) F
month was scheduled on three dates viz.- 12th, 19th and D) C
21st. One person attends seminar on one day only. There E) None of these.
were holidays on two dates. There was no holiday on two 3) How many person attends seminar before C?
consecutive dates. A) Three
K attends seminar just after H’s seminar. G and C attends B) One
seminar in same month. At least two persons attend C) Two
seminar before I. K attends seminar in one of the months D) More than three
having 31days but not on prime date. Three persons attend E) None
seminar between G and H. No person attends seminar 4) Which of the following statement is not true?
between B and J. I attends seminar on 19th of the month A) One person attends seminar between B and H.
having 30 days. G doesn’t attend seminar on 19th of any B) Two persons attend seminar between C and J.
month. Three persons attend seminar between H and I. C) D attends seminar just after E and there is no holiday
There is no seminar just after G’s seminar. Number of between them.
persons attending seminar between C and J is one less than D) Three persons attend seminar between G and H.
number of persons attending seminar between E and F. E E) More than one statement are not true.
attends seminar before F and doesn’t attend seminar 0n Direction (5-9): Read the following information
21st. D and F attends seminar in the same month. B attends carefully and answer the questions given below.
seminar on any day before D. Ten persons J, K, L, M, P, Q, R, S, T and V are born
1) Who attends seminar just before B? in five different months viz., January, August, April, June
A) C and December in the same year. They all are born in two
B) K different dates viz., 7th and 18th of the given months and
C) E each one likes different fruits namely Papaya, Apple,
D) J Kiwi, Mango, Strawberry, Grapes, Banana, Guava, Pear
E) None of these and Plums but not necessary in the same order.
Page 380 of 1334
Subscribe The Xpress Video Course & Mock Test Package for Bank & Insurance Exams
If there are any suggestions/ errors in our PDFs Feel Free to contact us via this email: admin@exampundit.in
Ultra Practice Bundle PDF
SBI Clerk/ RBI Assist. Mains – Reasoning
V born in an odd numbered date in one of the months, D) Guava
which does not has 30 days. There are only two persons E) Mango
were born between V and the one who likes Grapes. The 6) Four of the following five are alike in a certain way
one who likes Kiwi born immediately after S, who does and hence form a group. Which one of the following
not born in first or last months of the year. There are three does not belong to that group?
persons were born between T and the one who likes A) The one who likes Plums
Banana, who is elder than T. S and the one who likes Kiwi B) K
are born in the same month. The one who likes Guava C) The one who likes Grapes
born in 18th of August. As many persons born before the D) L
one who likes Mango is one more than the number of E) R
persons born after T, who does not like grapes. S and V 7) How many persons are born between S and the one
do not like Mango. There are four persons were born who likes Apple?
between R and Q, who was born in April. The one who A) None
likes Strawberry does not born in same month as the one B) One
who likes Plums. The number of persons born between L C) Two
and K is same as the number of persons born between K D) Three
and P. L does not like Banana and is elder than the one E) Five
who likes Mango. P does not born in the month which has 8) Who among the following was born immediately
30 days. The one who likes Pear does not born in odd after the one who likes Grapes?
numbered date. More than three persons were born A) Q
between Q and the one who likes Pear. The one who likes B) T
Papaya does not born in January and does not born in the C) S
same month as P was born. The one who likes Strawberry D) P
was born before M and not the eldest person. L does not E) V
born immediately after V. 9) If V is related to Plums and J is related to Mango in
5) M likes which of the following fruits? a certain way. Then S is related to which of the
A) Kiwi following?
B) Grapes A) Kiwi
C) Papaya B) Papaya

Page 381 of 1334


Subscribe The Xpress Video Course & Mock Test Package for Bank & Insurance Exams
If there are any suggestions/ errors in our PDFs Feel Free to contact us via this email: admin@exampundit.in
Ultra Practice Bundle PDF
SBI Clerk/ RBI Assist. Mains – Reasoning
C) Grapes than box Z. Box X has 30kg, is bought in the month which
D) Guava has less than 30 days. There is only one box between the
E) Banana boxes which has 18kg and S, which has 32kg. The box
Directions (10-14): Study the following information having 34kg was not bought in the same month of V.
carefully and answer the questions given below. 10) Which among the following boxes are bought in the
Twelve boxes O, P, Q, R, S, T, U, V, W, X, Y, and Z are month of March?
bought in the four different months i.e. January, February, a) O, R, U
March, and April. Each month has three different dates 7, b) X, P, T
14, and 21. Each box contains different weight 12, 14, 16, c) Z, Q, W
18, 20, 22, 24, 26, 28, 30, 32, and 34 but not necessarily d) Y, S, V
in the same order. e) None of these
Box P is bought in the month which has less than 31 days. 11) Which one of the following box is bought just after
Box Z is bought just after the box which has 20kg. There box O?
are three boxes between P and V, which has 20kg. Box P a) The box which has 34kg
is bought one of the boxes before box V. Box T is bought b) S
on 21st in any one of these months. Box U is bought just c) U
after the box has 34kg and both the boxes bought in same d) The box which has 28kg
month. Box T is bought in the month of neither January e) None of these
nor March. There are two boxes between T and the box 12) What is the total weight of the boxes which are
which has 16kg. Box Z is neither 18kg nor 16kg. Box R is bought in the month of February?
bought just before the box which has 16kg. Box U is a) 68kg
bought in the month which has not less than 31 days. b) 70kg
There are three boxes between Q and Y, which has 14kg. c) 72kg
Box O has 26kg, is bought on 7th but in the month which d) 74kg
has not less than 31 days. Box Y is bought in the month e) None of these
neither April nor February. Box Q does not buy on 21st. 13) How many boxes between the box U and the box
The box which has 12kg is bought just before W and both which has 18kg?
boxes bought in same month. Box W is bought neither a) None
14th nor February. Box W is heavier than box P but lighter b) One

Page 382 of 1334


Subscribe The Xpress Video Course & Mock Test Package for Bank & Insurance Exams
If there are any suggestions/ errors in our PDFs Feel Free to contact us via this email: admin@exampundit.in
Ultra Practice Bundle PDF
SBI Clerk/ RBI Assist. Mains – Reasoning
c) Two two more than number of persons attending the seminar
d) Three after G. Two persons attend the seminar between G and
e) None of these the one who likes red. The one who likes orange attends
14) Box Q bought in which one of the following the seminar before H. The one who likes blue attends the
months? seminar after the one who likes green. C does not like
a) January Blue.
b) February 15) Which of the following statement is true?
c) March a) Two persons attend the seminar between H and the one
d) April who likes red
e) None of these b) F attends the seminar before C
Directions (15-19): Study the following information c) More than three persons attend the seminar between H
carefully and answer the questions given below: and the one who likes Black
Eight persons A, B, C, D, E, F, G and H attend the seminar d) The one who likes red attends the seminar after E
in eight different months January, March, April, June, e) None is true
May, July, September and November in the same year. 16) Who among the following likes orange?
Persons name start with consecutive letters does not attend a) H
the seminar consecutively. For example, A does not b) G
attends the seminar immediately before or after B. c) C
Three persons attend the seminar between A and the one d) A
who likes Black. D attends the seminar immediately after e) None of these
the one who likes Black. A attends the seminar before the 17) Four of the following five are alike in a certain way
one who likes Black. Two persons attend the seminar based on the above arrangement. Find which one does
between D and the one who likes yellow. F attends the not belongs to the group?
seminar immediately before the one who likes white. Two a) C-Red
persons attend the seminar between F and E. The one who b) B-Black
likes Brown attends the seminar immediately before E. E c) E-Yellow
neither attends the seminar in first nor attends the seminar d) A-Orange
in last. E does not like yellow. F attends the seminar after e) G-Blue
A. Number of persons attending the seminar before B is

Page 383 of 1334


Subscribe The Xpress Video Course & Mock Test Package for Bank & Insurance Exams
If there are any suggestions/ errors in our PDFs Feel Free to contact us via this email: admin@exampundit.in
Ultra Practice Bundle PDF
SBI Clerk/ RBI Assist. Mains – Reasoning
18) If E is related to Yellow, F is related to Red, in the before the one who likes Hockey. Only four persons were
same way B is related to which of the following? born between A and the one who likes Table Tennis. A
a) Green was born after the one who likes Table Tennis. Only three
b) Brown persons were born between C and G. The one who likes
c) Black Badminton was born exactly between H and the one who
d) Yellow likes Rugby. Only three persons were born between the
e) Orange one who likes Tennis and B.H does not like Table Tennis.
19) How many persons attend the seminar between A 20) How many persons were born between the one who
and the one who likes black? likes Cricket and E?
a) None a) One
b) One b) Two
c) Two c) No one
d) Three d) Four
e) None of these e) None of these
Direction (20-24): Study following information 21) Which of the following combinations is correct?
carefully and answer the questions given below a) B-Golf
Eight friends-A, B, C, D, E, F, G and H are born in the b) C-Hockey
eight different months, March, April, May, June, July, c) E-Table Tennis
September, October and November in the same year. Each d) B-Cricket
of them likes different games, Cricket, Football, Hockey, e) None of these
Golf, Tennis, Rugby, Table Tennis and Badminton, but 22) Four of the following five are alike in certain way
not necessarily in the same order. and thus form a group as per the given arrangement.
The one who likes Tennis was born on the month which Which of the following does not belong to that group?
has 31 days. The one who likes Golf was born on either a) Golf
March or November. C was born exactly between the one b) Cricket
who likes Table Tennis and F. Only one person was born c) Hockey
between the one who likes Cricket and the one who likes d) Rugby
Hockey. D was born on October but does not like Tennis. e) Badminton
The one who likes Cricket was born on one of the months

Page 384 of 1334


Subscribe The Xpress Video Course & Mock Test Package for Bank & Insurance Exams
If there are any suggestions/ errors in our PDFs Feel Free to contact us via this email: admin@exampundit.in
Ultra Practice Bundle PDF
SBI Clerk/ RBI Assist. Mains – Reasoning
23) F is related to C in a certain way based on the given persons have appointment between D and E. The one who
arrangement. In the same way D is related to A. To likes Cat has an appointment on before F. Only two
Which of the following person is B related to following persons have appointment between the one who likes Rat
the same pattern? and the one who likes Horse. The number of persons have
a) G appointment after A is same as the number of persons
b) D have appointment before the one who likes Dog. Only
c) H three persons have appointment between B and D. G has
d) E an appointment on immediately before the one who likes
e) None of these Elephant. The one who likes tiger has an appointment on
24) Who among the following likes Hockey? one of the months before the one who likes Lion. F has an
a) C appointment in the month which has 31 days but before E.
b) H The one who likes Elephant has an appointment on one of
c) E the months after the one who likes Horse. B does not have
d) A appointment in the month which has the least number of
e) None days.
Direction (25-29): Study following information 25) Which of the following combinations is correct?
carefully and answer the questions given below a) F-Rat
Seven friends-A, B, C, D, E, F and G have appointment b) A-Cat
on seven different months, February, March, April, May, c) C-Horse
September, October and November in the same year. Each d) D-Elephant
of them likes different animals, Cat, Rat, Tiger, Lion, e) None of these
Elephant, Dog and Horse, but not necessarily in the same 26) How many persons have appointment between C
order. and the one who likes Tiger?
Only three persons have appointment between C and the a) Three
one who likes Cat. The one who likes Rat has an b) Two
appointment on one of the months which has 31 days. c) Four
Only one person has an appointment between the one who d) One
likes Horse and the one who likes Elephant. Only two e) None of these
persons have appointment between G and B. Only two 27) Who among the following likes Dog?

Page 385 of 1334


Subscribe The Xpress Video Course & Mock Test Package for Bank & Insurance Exams
If there are any suggestions/ errors in our PDFs Feel Free to contact us via this email: admin@exampundit.in
Ultra Practice Bundle PDF
SBI Clerk/ RBI Assist. Mains – Reasoning
a) B Patna, Mumbai, Hyderabad, Shillong and Goa, but not
b) F necessarily in the same order.
c) G U was born in one of the months before S. Only one
d) A person was born between the one who belongs to
e) None of these Hyderabad and the one who belongs to Pune. Only five
28) Four of the following five are alike in certain way persons were born between the Z and X. Only one person
and thus form a group as per the given arrangement. was born between the one who belongs to Goa and S. Only
Which of the following does not belong to that group? one person was born between R and the one who belongs
a) A-F to Shillong. Only three persons were born between Y and
b) E-B the one who belongs to Bangalore. Only three persons
c) D-G were born between W and V. The one who belongs to
d) C-E Hyderabad was born immediately before the one who
e) F-B belongs to Chennai. Only two persons were born between
29) A is related to Horse in a certain way based on the T and X. Only two persons were born between the one
given arrangement. In the same way C is related to who belongs to Delhi and the one who belongs to
Dog. To Which of the following animals is F related to Shillong. S was born in the month which has 30 days but
following the same pattern? before the month of August. The one who belongs to Goa
a) Lion and the one who belongs to Hyderabad does not born in
b) Rat the month of April and March respectively. The one who
c) Elephant belongs to Shillong was born immediately after T. Z was
d) Tiger born in the month of January. The one who belongs to
e) None of these Patna was born immediately before W. Only three persons
Direction (30-34): Study following information were born between S and U. R does not belong to
carefully and answer the questions given below Mumbai. T was born on one of the months before the one
Ten persons-P, R, S, T, U, V, W, X, Y and Z are born in10 who belongs to Bangalore.
different months in the same year starting from January 30) How many persons were born between T and the
and ending on October. Each of them belongs to different one who belongs to Bangalore?
cities, viz. Chennai, Bangalore, Delhi, Kolkata, Pune, a) One
b) Four

Page 386 of 1334


Subscribe The Xpress Video Course & Mock Test Package for Bank & Insurance Exams
If there are any suggestions/ errors in our PDFs Feel Free to contact us via this email: admin@exampundit.in
Ultra Practice Bundle PDF
SBI Clerk/ RBI Assist. Mains – Reasoning
c) Five c) October
d) Three d) August
e) None of these e) None of these
31) Who among the following belongs to Hyderabad? Direction (35-39): Study following information
a) Z carefully and answer the questions given below
b)Y Eight friends-S, T, U, V, W, X, Y and Z will have to take
c) V leave in different months in the same year, March, April,
d) R May and June. Each will take the leave on two different
e) None of these dates either 18th or 21st of the given months. Each of them
32) Four of the following five are alike in certain way likes different mobile brands, viz. Nokia, Samsung,
and thus form a group as per the given arrangement. Apple, Sony, Moto, Lenovo, Vivo and Oppo, but not
Which of the following does not belongs to that group? necessarily in the same order.
a) Y V will take leave on one of the days before Z. Only three
b) X persons will take leave between the one who likes Oppo
c) P and X. Only two persons will take leave between T and
d) T the one who likes Sony. The number of persons will take
e) R leave before the one who likes Apple is same as the
33) U is related to R in a certain way based on the given number of persons will take leave after the one who likes
arrangement. In the same way V is related to Y. To Moto. S will take leave on one of the days before W. Only
Which of the following persons is W related following two persons will take leave between the one who likes
the same pattern? Sony and the one who likes Apple. Neither V nor Z will
a) S take leave on 21stof the given month. Only two persons
b) X will take leave before the one who likes Oppo. The one
c) U who likes Nokia will takes leave immediately after Z. The
d) T number of persons will take leave between V and Z is
e) None of these same as the number of persons will take leave between the
34) Which of the following is the birth month of V? one who likes Apple and the one who likes Samsung. Only
a) September two persons will take leave between the one who likes
b) May Moto and Y. W is not takes a leave on June. The number

Page 387 of 1334


Subscribe The Xpress Video Course & Mock Test Package for Bank & Insurance Exams
If there are any suggestions/ errors in our PDFs Feel Free to contact us via this email: admin@exampundit.in
Ultra Practice Bundle PDF
SBI Clerk/ RBI Assist. Mains – Reasoning
of person will take a leave after X is same as the number 39) Apple is related to Nokia in a certain way based on
of the person will take a leave before T. Only three persons the given arrangement. In the same way Oppo is
will take a leave between the one who likes Nokia and the related to Moto. Which of the following mobile brand
one who likes Vivo. is Samsung related to following the same pattern?
35) How many persons will take leave between the one a) Vivo
who likes Moto and U? b) Nokia
a) Five c) Lenovo
b) Three d) Apple
c) Two e) None of these
d) Four Direction (40-42): Study following information
e) None of these carefully and answer the questions given below.
36) Which of the following combinations is correct? Seven friends-A, B, C, D, E, F and G wrote exam on seven
a) X-Moto different months on the same year, January, February,
b) V-Oppo March, April, August, November and December. Each of
c) T-Apple them got different ranks in the exam, 1, 2, 3, 4, 5, 6 and 7
d) W-Nokia but not necessarily in the same order.
e) None of these B got second rank and wrote exam on March. E wrote
37) Which of the following statement is correct? exam immediately before F. The one who got first rank
a) V likes Oppo and will take leave on April 18. wrote exam on November. The one who got seventh rank
b) X likes Moto and will take leave on June 18. did not write exam on August or any of the months before
c) T likes Apple August. The one who got third rank wrote exam
d) U will take leave on June 21. immediately before G. F did not get fifth rank. D did not
e) None of these write exam on December. E did not write exam on any of
38) Who among the following likes Lenovo? the months after B. D did not get third rank. C did not
a) U write exam on any of the months before A. The one who
b) W got sixth rank did not write exams immediately before and
c) X after B. The one who wrote exam on August neither got
d) S fifth rank nor sixth rank.
e) None of these 40) Which of the following combinations is correct?

Page 388 of 1334


Subscribe The Xpress Video Course & Mock Test Package for Bank & Insurance Exams
If there are any suggestions/ errors in our PDFs Feel Free to contact us via this email: admin@exampundit.in
Ultra Practice Bundle PDF
SBI Clerk/ RBI Assist. Mains – Reasoning
a) A-April Volleyball and Tennis but not necessarily in the same
b) D-November order.
c) G-August More than five people attend the lecture between Puja and
d) C-December the one who likes Tennis. Waqar attends the lecture before
e) None of these the one who likes Badminton. Xing does not like
41) How many persons wrote exam between the one Badminton. Two people attend the lecture between
who got sixth rank and the one who got third rank? Qureshi and the one who likes Football. Only one person
a) One attends the lecture between Riya and the one who likes
b) Two Hockey. As many people attend the lecture after Xing,
c) Three attend the lecture before the one who likes Basketball.
d) Four Qureshi does not attend the lecture immediately before or
e) None of these after Xing. Qureshi does not like Basketball. As many
42) C is related to 5th rank in a certain way based on people attend the lecture after the one who likes
the given arrangement. In the same way D is related to Badminton attend the lecture before Waqar. Xing attends
6th rank. To Which of the following rank is G related the lecture immediately before the one who likes Tennis.
, following the same pattern? Qureshi does not like Hockey. Waqar does not like
a) 1st Basketball. Only one person attends the lecture between
b) 2nd Tina and the one who likes Swimming. Vishesh attends
c) 4th the lecture immediately before the one who likes
d) 7th Swimming. The one who likes Swimming attends the
e) None of these lecture before the one who likes Volleyball.
Direction (43-45): Study the following information 43) Suraj likes which of the following sport?
carefully and answer the questions following it. A. Football
Eight people - Puja, Qureshi, Riya, Suraj, Tina, Vishesh, B. Tennis
Waqar and Xing attend the lecture in the months January, C. Badminton
August, November, March, October, April, September D. Volleyball
and June. Each one of them likes different Sports, Hockey, E. None of these.
Football, Cricket, Swimming, Basketball, Badminton,

Page 389 of 1334


Subscribe The Xpress Video Course & Mock Test Package for Bank & Insurance Exams
If there are any suggestions/ errors in our PDFs Feel Free to contact us via this email: admin@exampundit.in
Ultra Practice Bundle PDF
SBI Clerk/ RBI Assist. Mains – Reasoning
44) Four of the following five are like in a certain way who visits Delhi attends ceremony in March. Mike and
based on the above arrangement. Find the one which Shreya is a married couple. Luv does not visit on April
does not belong to the group? Month and there is three months gap between the months
A. The one who likes Cricket in which Maya and Luv visits (Considering the months
B. Tina given in the information alone). Luv visit Chennai but
C. The one who likes Basketball neither married to Diya nor visits in any of the months
D. The one who likes Football having 31 days. Rohit, who doesn’t visits Varanasi,
E. Xing attends ceremony just after Maya. Tanya, who is neither
45) If Puja is related to Badminton, Waqar is related married to Rohit nor visits Varanasi, visits two months
to Tennis in a certain way, then in the same way before the one who visits Chennai. Maya neither visits
Vishesh is related to which of the following? Delhi nor visits in any of the month after June. Sam
A. Hockey attends ceremony just after Kavita and visits Jaipur. The
B. Swimming one who visits Lucknow attends ceremony two months
C. Football after Priya (considering the months given in the
D. Tennis information alone). One person attends ceremony
E. None of these between Shreya and the one who visits Varanasi. Priya is
Direction (46-50): Study the following information neither married to Sam nor Mike. Neither Deep nor
carefully and answer the questions given below it. Shreya attends ceremony in Delhi. Kavita neither visits
Eight married couples attend marriage ceremony in Chennai nor married to Hari. Shweta visits Patna in any of
different months viz. - January, March, April, June, July, the months before Kavita. At most two couples visits
August, September and October to different places viz.- between the one who visits Pune and Anuj, who is not
Delhi, Pune, Patna, Agra, Jaipur, Lucknow, Chennai and married to Shweta. Deep neither married to Tanya nor
Varanasi. Male persons are – Sam, Mike, Luv, Rohit, married to Shweta.
Tom, Anuj, Deep and Hari while female members are – 46) Which of the following combination is true?
Maya, Ritu, Shreya, Kavita, Shweta, Priya, Tanya and A. Anuj – Priya – April – Patna
Diya. B. Rohit – Kavita – June – Varanasi
Hari, who neither visits Jaipur nor visits Lucknow, attends C. Sam – Diya – Jaipur – July
ceremony four months after Maya. The one who visits D. Deep – Maya – Pune – April
Lucknow, visits on the month of having 30 days. The one E. Mike – Shreya – Agra – Varanasi

Page 390 of 1334


Subscribe The Xpress Video Course & Mock Test Package for Bank & Insurance Exams
If there are any suggestions/ errors in our PDFs Feel Free to contact us via this email: admin@exampundit.in
Ultra Practice Bundle PDF
SBI Clerk/ RBI Assist. Mains – Reasoning
47) Who among the following person visits Agra? 49) Who is married to Sam?
A. Tanya A. Diya
B. Mike B. Shweta
C. Anuj C. Ritu
D. Shweta D. Kavita
E. None of these E. None of these
48) Which of the following city is visited in July? 50) Kavita is married to whom among the following?
A. Jaipur A. Luv
B. Pune B. Deep
C. Chennai C. Rohit
D. Patna D. Tom
E. None of these E. None of these

Puzzle Month - Answers and Explanation


Direction (1-5): • At least two persons attend seminar before I.
That means, in case (1) I attends seminar on 19thApril,
in case (2) I attends seminar on 19th September.
• K attends seminar in one of the months having 31days
but not on prime date.
• K attends seminar just after H’s seminar.
• Three persons attend seminar between H and I.
That means, in case (1a) H attends seminar on
21st May, in case (1b) H attends seminar on
We have: 19th September, in case (2) H attends seminar on
• I attends seminar on 19th of the month having 30 days. 21st April.

Page 391 of 1334


Subscribe The Xpress Video Course & Mock Test Package for Bank & Insurance Exams
If there are any suggestions/ errors in our PDFs Feel Free to contact us via this email: admin@exampundit.in
Ultra Practice Bundle PDF
SBI Clerk/ RBI Assist. Mains – Reasoning
• Three persons attend seminar between G and H.
• There is no seminar just after G’s seminar.
• G doesn’t attends seminar on 19th.
That means, in case (2) G attends seminar on 12th January,
case (1a)& case (1b) are not valid.
Based on above given information we have:

1) Answer: A
Clearly, C attends seminar Just before B.
Hence, option A is correct choice.
Case (1a) & case (1b) are not valid as three persons
2) Answer: E
attend seminar between G and H.
Clearly, H attends seminar on 21st April.
Again, we have:
Hence, option E is correct choice.
• G and C attends seminar in same month.
3) Answer: B
• Number of persons attending seminar between C
Clearly, one person attends seminar before C.
and J is one less than number of number of persons
Hence, option B is correct choice.
attending seminar between E and F.
4) Answer: E
• E attends seminar before F and doesn’t attends
Clearly, both statement B and C are not true.
seminar 0n 21st.
Hence, option E is correct choice.
• D and F attends seminar in the same month.
Directions (5-9):
• B attends seminar on any day before D.
• No person attends seminar between B and J.
That means, B attends seminar on 12th April and F attends
seminar on 21st September.
Based on above given information we have final
arrangement as follows:

Page 392 of 1334


Subscribe The Xpress Video Course & Mock Test Package for Bank & Insurance Exams
If there are any suggestions/ errors in our PDFs Feel Free to contact us via this email: admin@exampundit.in
Ultra Practice Bundle PDF
SBI Clerk/ RBI Assist. Mains – Reasoning

There are three persons were born between T and the one
who likes Banana, who is elder than T. As many persons
born before the one who likes Mango is one more than the
number of persons born after T, who does not like grapes.
V born in odd numbered date in one of the months, which
S and V does not like Mango.
does not has 30 days. There are only two persons were
There are four persons were born between R and Q, who
born between V and the one who likes Grapes.
was born in April. So, Case-4a and Case-4b will be
dropped.

The one who likes Kiwi born immediately after S, who The number of persons born between L and K is same as

does not born in first or last months of the year. S and the the number of persons born between K and P. L does not

one who likes Kiwi are born in the same month. The one like Banana and is elder than the one who likes Mango. P

who likes Guava born in 18th of August. does not born in the month which has 30 days. L does not
born immediately after V.
So, Case-1 and case 3a will be dropped.

Page 393 of 1334


Subscribe The Xpress Video Course & Mock Test Package for Bank & Insurance Exams
If there are any suggestions/ errors in our PDFs Feel Free to contact us via this email: admin@exampundit.in
Ultra Practice Bundle PDF
SBI Clerk/ RBI Assist. Mains – Reasoning

The one who likes Pear does not born in odd numbered
date. More than three persons were born between Q and
the one who likes Pear.
So, Case-3b will be dropped.
The one who likes Strawberry does not born in same 5) Answer: B

month as the one who likes Plums. The one who likes 6) Answer: A

Papaya does not born in January and does not born in the 7) Answer: D

same month as P was born. The one who likes Strawberry 8) Answer: C

was born before M and not the eldest person. 9) Answer: B


Directions (10-14):

Explanation:
• There are three boxes between P and V, which has
20kg.
• Box P is bought one of the boxes before box V.

Page 394 of 1334


Subscribe The Xpress Video Course & Mock Test Package for Bank & Insurance Exams
If there are any suggestions/ errors in our PDFs Feel Free to contact us via this email: admin@exampundit.in
Ultra Practice Bundle PDF
SBI Clerk/ RBI Assist. Mains – Reasoning
• Box P is bought in the month which has less than 31 • Box Q does not buy on 21st.
days. • Box O has 26kg, is bought on 7th but in the month
• Box T is bought on 21st in any one of these months. which has not less than 31 days.
• Box T bought in the month of neither January nor
March. There are two boxes between T and the box
which has 16kg. Box Z is neither 18kg nor 16kg.
• Box R is bought just before the box having 16kg.
• Box Z is bought just after the box which has 20kg.

• Box U is bought just after the box having 34kg and


both the boxes bought in same month.
• Box U is bought in the month which has not less than
31 days. The box having 34kg was not bought in the
same month of V.

• In case 3b, Box O has 26kg, is bought on 7th but in


the month has not less than 31 days, and this
condition is not satisfied so eliminated
• The box which has 12kg is bought just before W and
both boxes bought in same month. Box W is bought
neither 14th nor February.
• There are three boxes between Q and Y, which has
14kg.
• Box Y is bought in the month neither April nor
February.

Page 395 of 1334


Subscribe The Xpress Video Course & Mock Test Package for Bank & Insurance Exams
If there are any suggestions/ errors in our PDFs Feel Free to contact us via this email: admin@exampundit.in
Ultra Practice Bundle PDF
SBI Clerk/ RBI Assist. Mains – Reasoning

• In case 1a, 2b, 2c Box X has 30kg, is bought in the


month has less than 30 days, this condition is not
satisfied so rejected.
• In case 3a i) and Case 3c, there is only one box
between the box which has 18kg and S, which has
• In case 3a ii), The box which has 12kg is bought just
32kg this condition not satisfied so rejected.
before W and Box W is bought neither 14th nor
• In case 1b box P should be lesser than box W. this
February, these conditions are not satisfied so rejected.
condition not satisfied so rejected. So the final
• Box X has 30kg, is bought in the month has less than
arrangement is,
30 days.
• Box Z is neither 18kg nor 16kg.
• Box W is heavier than box P but lighter than box Z.
• There is only one box between the boxes which has
18kg and S, which has 32kg.

10) Answer: D
11) Answer: A

Page 396 of 1334


Subscribe The Xpress Video Course & Mock Test Package for Bank & Insurance Exams
If there are any suggestions/ errors in our PDFs Feel Free to contact us via this email: admin@exampundit.in
Ultra Practice Bundle PDF
SBI Clerk/ RBI Assist. Mains – Reasoning
12) Answer: B first nor attends the seminar in last. E does not like yellow.
13) Answer: C F attends the seminar after A.
14) Answer: D
Directions (15-19):

Three persons attend the seminar between A and the one


who likes Black. D attends the seminar immediately after
the one who likes Black. A attends the seminar before the
one who likes Black. Two persons attend the seminar
between D and the one who likes yellow.
Number of persons attending the seminar before B is two
more than number of persons attending the seminar after
G. Two persons attend the seminar between G and the one
who likes red.

F attends the seminar immediately before the one who


likes white. Two persons attend the seminar between F
and E. The one who likes Brown attends the seminar
immediately before E. E neither attends the seminar in

Page 397 of 1334


Subscribe The Xpress Video Course & Mock Test Package for Bank & Insurance Exams
If there are any suggestions/ errors in our PDFs Feel Free to contact us via this email: admin@exampundit.in
Ultra Practice Bundle PDF
SBI Clerk/ RBI Assist. Mains – Reasoning
Case 1 will be dropped because C does not like Blue.
So the final arrangement is,

15) Answer: C)
16) Answer: C)
17) Answer: d)
18) Answer: b)
19) Answer: d)
Case 2 and Case 3b will be dropped because Number of Direction (20-24):
persons attending the seminar before B is two more than • The one who is likes Tennis was born on the month
number of persons attending the seminar after G. which has 31 days.
The one who likes orange attends the seminar • Only three persons were born between the one who
immediately before H. The one who likes blue attends the likes Tennis and B.
seminar after the one who likes green. C does not like • The one who likes Golf was born on either March or
Blue. November.
• D was born on October but does not like Tennis.
• Now we have 3 Cases.

Page 398 of 1334


Subscribe The Xpress Video Course & Mock Test Package for Bank & Insurance Exams
If there are any suggestions/ errors in our PDFs Feel Free to contact us via this email: admin@exampundit.in
Ultra Practice Bundle PDF
SBI Clerk/ RBI Assist. Mains – Reasoning
• C was born exactly between the one who likes Table
Tennis and F.
• Only four persons were born between A and the one
who likes Table Tennis .A was born after the one who
likes Table Tennis.
• Only three persons were born between C and G.
• From the above condition Case 2 and Case 3 was
dropped. 20) Answer: C
21) Answer: C
22) Answer: E
23) Answer: A
24) Answer: B
Direction (25-29):
• The one who likes Rat has an appointment on one of
the months which has 31 days.
• Only two persons have appointment between the one
who likes Rat and the one who likes Horse.
• Only one person has an appointment between the one
• Only one person was born between the one who who likes Horse and the one who likes Elephant.
likes Cricket and the one who likes Hockey. • The one who likes Elephant has an appointment on one
• The one who likes Cricket was born on one of the of the months after the one who likes Horse.
months before the one who likes Hockey. • G has an appointment on immediately before the one
• The one who likes Badminton was born exactly who likes Elephant.
between H and the one who likes Rugby. • Only two persons have appointment between G and B.
• H does not like Table Tennis. • B does not have appointment in the month which has
• So the final arrangement is. the least number of days.
• Only three persons have appointment between B and
D.
• Now we have 3 Cases.

Page 399 of 1334


Subscribe The Xpress Video Course & Mock Test Package for Bank & Insurance Exams
If there are any suggestions/ errors in our PDFs Feel Free to contact us via this email: admin@exampundit.in
Ultra Practice Bundle PDF
SBI Clerk/ RBI Assist. Mains – Reasoning

• Only two persons have appointment between D and E.


• F has an appointment in the month which has 31 days
but before E.
• The one who likes Cat has an appointment before F.
25) Answer: B
• Only three persons have appointment between C and
26) Answer: B
the one who likes Cat.
27) Answer: A
• From the above condition Case 2 was dropped.
28) Answer: E
29) Answer: A
Direction (30-34):
• Only five persons were born between the Z and X.
• Z was born in the month of January.
• S was born in the month which has month has 30 days
• The number of persons have appointment after A is
but before the month of August.
same as the number of persons have appointment
• Only one person was born between one who belongs to
before the one who likes Dog.
Goa and S.
• The one who likes tiger has an appointment on one of
• Only three persons were born between S and U.
the months before the one who likes Lion.
• U was born in one of the months before S.
• From the above condition Case 1 was dropped.
• Now we have 2 Cases.
• So the final arrangement is.

Page 400 of 1334


Subscribe The Xpress Video Course & Mock Test Package for Bank & Insurance Exams
If there are any suggestions/ errors in our PDFs Feel Free to contact us via this email: admin@exampundit.in
Ultra Practice Bundle PDF
SBI Clerk/ RBI Assist. Mains – Reasoning
• The one who belongs to Goa and the one who belongs
to Hyderabad does not born in the month of April and
March respectively.
• From the above condition Case 1 was dropped.
• The one who belongs to Shillong was born
immediately after T.
• Only two persons were born between T and X. • Only three persons were born between W and V.
• Only one person was born between R and the one who • The one who belongs to Hyderabad was born
belongs to Shillong. immediately before the one who belongs to Chennai.
• R does not belong to Mumbai.
• The one who belongs to Goa and the one who belongs
to Hyderabad does not born in the month of April and
March respectively.
• The one who belongs to Patna was born in immediately
before W.
• Only one person was born between the one who
belongs to Hyderabad and the one who belongs to
• Only two persons were born between the one who
Pune.
belongs to Delhi and the one who belongs to Shillong.
• From the above condition Case 2(a) was dropped.
• Only three persons were born between Y and the one
• So the final arrangement is.
who belongs to Bangalore.
• T was born one of the months before the one who
belongs to Bangalore.
• From the above condition Case 2 derives further 2
cases.

Page 401 of 1334


Subscribe The Xpress Video Course & Mock Test Package for Bank & Insurance Exams
If there are any suggestions/ errors in our PDFs Feel Free to contact us via this email: admin@exampundit.in
Ultra Practice Bundle PDF
SBI Clerk/ RBI Assist. Mains – Reasoning
30) Answer: a) • Only two persons will take leave between the one
31) Answer: c) who likes Moto and Y.
32) Answer: d) • Neither V nor Z will take leave on 21th of the given
33)Answer: a) month.
34) Answer: a) • V will take leave on one of the days before Z.
Directions(35-39): • The one who likes Nokia will takes leave on
• Only two persons will take leave before the one who immediately after Z.
likes Oppo. • Only three persons will take leave between the one
• Only three persons will take leave between the one who who likes Nokia and the one who likes Vivo.
likes Oppo and X. • From the above condition Case 1 derive further 2
• The number of person will take leave after X is same Cases.
as the number of the person will take leave before T.
• Only two persons will take leave between T and the one
who likes Sony.
• Only two persons will take leave between the one who
likes Sony and the one who likes Apple.
• Now we have 2 Cases.

• The number of persons will take leave between V and


Z is same as the number of persons will take leave
between the one who likes Apple and the one who likes
Samsung.
• W is not takes leave on June.
• S will take leave on one of the days before W.
• From the above condition Case 1(b) and Case 2 was
dropped.
• The number of persons will take leave before the one • So the final arrangement is..
who likes Apple is same as the number of persons will
take leave after the one who likes Moto.

Page 402 of 1334


Subscribe The Xpress Video Course & Mock Test Package for Bank & Insurance Exams
If there are any suggestions/ errors in our PDFs Feel Free to contact us via this email: admin@exampundit.in
Ultra Practice Bundle PDF
SBI Clerk/ RBI Assist. Mains – Reasoning

• The one who got third rank wrote exam immediately


35) Answer: E before G.
36) Answer: D • Now we have 2 Cases.
37) Answer: D
38) Answer: C
39) Answer: A
Direction (40-42):
• The one who got seventh rank did not write exam on
August or any of the months before August.
• The one who got first rank wrote exam on November.
• B got second rank and wrote exam on March.
• D did not write exam on December.
• E wrote exam immediately before F.
• D did not get third rank.
• E did not write exam on any of the months after B.
• C did not write exam on any of the months before A.

Page 403 of 1334


Subscribe The Xpress Video Course & Mock Test Package for Bank & Insurance Exams
If there are any suggestions/ errors in our PDFs Feel Free to contact us via this email: admin@exampundit.in
Ultra Practice Bundle PDF
SBI Clerk/ RBI Assist. Mains – Reasoning
Direction (43-45):
Explanation
i) More than five persons attend the lecture between Puja
and the one who likes Tennis.
ii) Xing attends the lecture immediately before the one
who likes Tennis.
iii) As many persons attend the lecture after Xing, attend
the lecture before the one who likes Basketball.
• F did not get fifth rank.
• The one who got sixth rank did not write exam on
immediately before and after B.
• The one who wrote exam on August neither got fifth
rank nor sixth rank.
• Case 1 was not satisfied all the conditions so it was
dropped.
• So the final arrangement is..
iv) Two persons attend the lecture between Qureshi and
the one who likes Football.
v) Qureshi does not attend the lecture immediately before
or after Xing. Qureshi does not like Basketball.
From the above statements, we have 3 cases:
Case 1:

40) Answer: d)
41) Answer: c)
42) Answer: b)

Page 404 of 1334


Subscribe The Xpress Video Course & Mock Test Package for Bank & Insurance Exams
If there are any suggestions/ errors in our PDFs Feel Free to contact us via this email: admin@exampundit.in
Ultra Practice Bundle PDF
SBI Clerk/ RBI Assist. Mains – Reasoning
vi) As many persons attend the lecture after the one who
likes Badminton attend the lecture before Waqar.
vii) Waqar attends the lecture before the one who likes
Badminton. Waqar does not like Basketball. Xing does
not likes Badminton.
viii) Only one person attends the lecture between Tina and
the one who likes Swimming.
ix) Vishesh attends the lecture immediately before the one
who likes Swimming.
x) The one who likes Swimming attends the lecture before
the one who likes Volleyball.
Case 1:

Case 3:

Case 3:

Page 405 of 1334


Subscribe The Xpress Video Course & Mock Test Package for Bank & Insurance Exams
If there are any suggestions/ errors in our PDFs Feel Free to contact us via this email: admin@exampundit.in
Ultra Practice Bundle PDF
SBI Clerk/ RBI Assist. Mains – Reasoning

xi) Only one person attends the lecture between Riya and
the one who likes Hockey.
xii) Qureshi does not like Hockey.
So Case 1 & 2 gets eliminated.
Hence the final arrangement is given below: We have:
• Luv visit Chennai but neither married to Diya nor
visits in any on the months of 31 days.
• Luv does not visit on April Month.
• Tanya, who is neither married to Rohit nor visits
Varanasi, visits two months before the one who
visits Chennai.
That means, in case (1) Luv visits Chennai in June, in case
(2) Luv visits Chennai in September.
43) Answer: B • The one who visits Delhi attends ceremony in
44) Answer: D March.
45) Answer: C Based on above given information we have:
Direction (46-50):
Explanation

Page 406 of 1334


Subscribe The Xpress Video Course & Mock Test Package for Bank & Insurance Exams
If there are any suggestions/ errors in our PDFs Feel Free to contact us via this email: admin@exampundit.in
Ultra Practice Bundle PDF
SBI Clerk/ RBI Assist. Mains – Reasoning
That means, case (2a) Priya attends ceremony in January,
in case (2b) Priya attends ceremony in March & case (1)
is invalid.
Based on above given information we have:

Again, we have:
• Rohit, who doesn’t visits Varanasi, attends
ceremony just after Maya. Case 1 is invalid because there is three months gap

• Maya neither visits Delhi nor visits in any of the between the months in which Maya and Luv visits

months after June. (considering the months given in the information alone).

• Hari, who neither visits Jaipur nor visits Lucknow, Again, we have:
• Mike and Shreya is married couple.
attends ceremony four months after Maya.
• One person attends ceremony between Shreya and
That means, in case (1) Maya attends ceremony in June
and Hari attends ceremony in October, in case (2) Maya the one who visits Varanasi.
• Neither Deep nor Shreya attends ceremony in
attends ceremony in April and Hari attends ceremony in
August. Delhi.

• There is three months gap between the months in That means, in case (2a) & case (2b (ii)) Shreya visits in

which Maya and Luv visits. (Considering the October, in case (2b (i)) Shreya visits in January.
• Shweta visits Patna in any of the months before
months given in the information alone).
• The one who visits Lucknow attends ceremony Kavita.
• Kavita neither visits Chennai nor married to Hari.
two months after Priya. (considering the months
• Sam attends ceremony just after Kavita and visits
given in the information alone) . The one who
visits Lucknow, visits on the month having 30 Jaipur.

days That means, in case (2b (ii)) Sam attends seminar in July

• Priya is neither married to Sam nor Mike. and Shweta attends seminar in January, case (2a), case (2b
(i)) are not valid.

Page 407 of 1334


Subscribe The Xpress Video Course & Mock Test Package for Bank & Insurance Exams
If there are any suggestions/ errors in our PDFs Feel Free to contact us via this email: admin@exampundit.in
Ultra Practice Bundle PDF
SBI Clerk/ RBI Assist. Mains – Reasoning
• Deep neither married to Tanya nor married to
Shweta.
That means, Deep is married to Maya. Neither Deep nor
Shreya attends ceremony in Delhi.
Based on above given information we have:

Case (2a) & case (2b (i)) is not valid as Sam attends
ceremony just after Kavita and visits Jaipur.
Again, we have: 46) Answer: D
Since, Luv is not married to Diya, thus Hari must be Clearly, combination D is correct choice.
married to Diya and Luv is married to Ritu. Hence, option D is correct choice.
• At most two couples visits between the one who 47) Answer: B
visits Pune and Anuj, who is not married to Clearly, Mike and Shreya visits Agra.
Shweta. Hence, option B is correct choice.
That means, Anuj is married to Priya and Deep visits 48) Answer: A
Pune. Clearly, Jaipur is visited in July.
Based on above given information we have final Hence, option A is correct choice.
arrangement as follows: 49) Answer: E
Clearly, Sam is married to Tanya.
Hence, option E is correct choice.
50) Answer: C
Clearly, Rohit is married to Kavita.
Hence, option C is correct choice.

Download Puzzles Practice Questions PDF


Get More Reasoning Practice Questions PDF
Page 408 of 1334
Subscribe The Xpress Video Course & Mock Test Package for Bank & Insurance Exams
If there are any suggestions/ errors in our PDFs Feel Free to contact us via this email: admin@exampundit.in
Ultra Practice Bundle PDF
SBI Clerk/ RBI Assist. Mains – Reasoning
Puzzle Month and Date
Directions (1-5): Answer the questions based on the (c) 18th June
information given below: (d) 12th June
There are eight friends A, B, C, D, E, F, G and H and they (e) None of these
have their joining dates in different banks in four months 2. Who among the following joins Icici bank?
i.e. April, May, June and July on either 12th or 18th of the (a) H
month. They join Eight different banks i.e. Hdfc,SBI, (b) A
Icici, BOB, BOM, BOI, Indian and Canara bank but not (c) E
necessarily in the same order. (d) C
G does not join BOM. Joining of A is in month having 31 (e) None of these
days but before the one who joins Indian Bank. There are 3. D joins which among the following bank?
two joining between the joining of A and C. The joining (a) SBI
of one who joins BOB is immediately after C’s joining. A (b) BOB
joins just after B. There are two joinings after the one who (c) BOI
joins Indian bank. There are two joinings between the (d) Canara
joining of H and the one who joins Canara and either one (e) None of these
of them has joins at last. B’s joining is on 18th of the month 4. How many students join banks between G and A?
having 30 days. B does not join in the same month in (a) 5
which C joins. The joining of one who joins BOI is just (b) 4
after the joining of B. H and the one who joins SBI joins (c) 6
on same date. There are three joinings between the joining (d) 3
of F and the one who joins Icici. B and C do not join Icici. (e) None of these
H does not join SBI. E joins Hdfc and his joining is before 5. Four of the following five are alike in a certain way
the one who joins Indian bank. The joining of one who and hence they form a group. Which one of the
join SBI and BOM are next to each other. G and the one following does not belong to that group?
who joins BOM joins on the same date. (a) A
1. H joins on which of the following date? (b) F
(a) 12th April (c) H
(b) 18th July (d) D
Page 409 of 1334
Subscribe The Xpress Video Course & Mock Test Package for Bank & Insurance Exams
If there are any suggestions/ errors in our PDFs Feel Free to contact us via this email: admin@exampundit.in
Ultra Practice Bundle PDF
SBI Clerk/ RBI Assist. Mains – Reasoning
(e) B (a) M
Directions (6-10): Answer the questions based on the (b) D
information given below: (c) A
There are eight friends A, B, C, D, J, K, L and M were (d) C
born on in March, April, June and July on either 10th or (e) None of these
20th of month. The ones who were born in a month having 7. Which among the following likes Orange?
30 days likes different fruits i.e. Apple, Mango, Grapes (a) D
and Banana not necessarily in the same order. The ones (b) K
who were born in a month having 31 days like different (c) L
colours i.e. Black, Brown, Orange, Pink not necessarily in (d) M
the same order. (e) A
No person was born between the ones who like Brown 8. How many persons born between A and J?
colour and the one who likes Grapes. The one who likes (a) Three
Brown colour was not born on 20th of any month. No (b) Two
person was born between D and the one who likes Banana. (c) Four
One person was born between A and the one who likes (d) One
Apple. The one who likes Mango and Apple were born (e) None
either on the same date or in the same month. One person 9. Who among the following born immediate after one
was born between B and M. A likes Black colour. K does who likes Pink colour?
not like fruits. No person was born between C and one (a) L
who likes Orange colour. M does not like fruits. D was (b) B
born in April. One person was born between D and J. J (c) A
does not like any colour. Same number of people was born (d) K
before B as after J. The number of people born between (e) None of these
the one who likes Pink colour and the one who likes 10. Four of the following five are alike in a certain way
Mango is same as the number of people born between the and hence they form a group. Which one of the
one who likes Orange colour and the one who likes following does not belong to that group?
Grapes. (a) D
6. Who among the following borns on 10th July? (b) B

Page 410 of 1334


Subscribe The Xpress Video Course & Mock Test Package for Bank & Insurance Exams
If there are any suggestions/ errors in our PDFs Feel Free to contact us via this email: admin@exampundit.in
Ultra Practice Bundle PDF
SBI Clerk/ RBI Assist. Mains – Reasoning
(c) L 11. Who among the following attend a meeting in
(d) J August and November respectively?
(e) M (a) T, W
Directions (11-15): Answer the questions based on the (b) T, V
information given below: (c) U, S
There are eight persons S, T, U, V, W, X, Y and Z attend (d) X, Y
a meeting in different months i.e. August, September, (e) V, Z
November and December on two different dates 9th or 12. How many persons attend a meeting between the
18th. They all belongs to different cities i.e. Jaipur, one who belongs to Nagaur and Bikaner?
Jodhpur, udaipur, Kota, Ajmer, Nagaur, Tonk and Bikaner (a) None
but not neccesarily in the same order. (b) One
One person attend meeting between the ones who belong (c) Three
to Nagaur and Tonk. Five persons ateend meeting between (d) More than three
T and X, who attend meeting after T. X does not attend (e) Two
meeting at last.V attend meeting before Y and both of 13. X belongs to which of the following city?
them attend meeting in the same month. S attend the (a) Nagaur
meeting in September. One person attend the meeting (b) Jodhpur
between S and who belongs to Tonk, who does not attend (c) Kota
the meeting in August. No one attend meeting before who (d) Udaipur
belongs to udaipur. The number of person attend meeting (e) Ajmer
before W is same as the number of person attend meeting 14. The one who attend meeting on 18th August belongs
after the one who belongs to Tonk. No one attend meeting to which city?
between S and the one who belongs to Jodhpur. U does (a) Tonk
not belongs to Jodhpur. U attend the meeting before Z but (b) Ajmer
not immediately before. Four persons attend meeting (c) Bikaner
between Z, who belongs to Ajmer and the one who (d) Jaipur
belongs to Kota. Z attend meeting after who belongs to (e) Udaipur
Kota. One of the person who attend meeting in November 15. As per the given arrangement Jaipur is related to
belongs to Jaipur. V and Jodhpur is related to S, following a certain

Page 411 of 1334


Subscribe The Xpress Video Course & Mock Test Package for Bank & Insurance Exams
If there are any suggestions/ errors in our PDFs Feel Free to contact us via this email: admin@exampundit.in
Ultra Practice Bundle PDF
SBI Clerk/ RBI Assist. Mains – Reasoning
pattern, which of the following is related to X following month but not in July. L plays after the one who belong to
the same pattern? RR.
(a) Bikaner 16. N belong to which of the following team?
(b) Udaipur (a) ST
(c) Ajmer (b) CSK
(d) Jodhpur (c) PS
(e) Tonk (d) RCB
Directions (16-20): Answer the questions based on the (e) None of these
information given below: 17. Who among the following player belong to IPL
There are eight friends J, K, L, M, N, O, P and Q play team?
practice match on 2nd, 4th, 9th and 11th in the month of July (a) N
and August. The person who belongs to IPL team practice (b) O
on that day which is pefectly square, the IPL team are (c) M
RCB, CSK, DD and RR. The person who belongs to BBL (d) L
team practice on that day which is prime numbers, the (e) None of these
BBL team are ST, PS, MR and BH. 18. Who among the following play on 11th August?
N do not belong to IPL team. O does not belong to ST (a) Q
team. There is no person play between one who belongs (b) L
to MR and RR team. The number of person play after the (c) P
one who belongs to RR is one less than the number of (d) J
person play before J. The one who belongs to ST play (e) None of these
immediately before K. K do not belong to MR. The 19. How many players play after O?
number of person play before K is same as number of (a) 5
person play after P. Three person play between the one (b) 4
who belongs to CSK and Q. The one who belongs to RCB (c) 6
play in the month of July. No person play between M and (d) 3
who belongs to RCB . M does not belongs to IPL team. (e) None of these
Three persons play between M and one who belongs to
MR.Q and the one who belongs to PS play in the same

Page 412 of 1334


Subscribe The Xpress Video Course & Mock Test Package for Bank & Insurance Exams
If there are any suggestions/ errors in our PDFs Feel Free to contact us via this email: admin@exampundit.in
Ultra Practice Bundle PDF
SBI Clerk/ RBI Assist. Mains – Reasoning
20. Four of the following five are alike in a certain way between U and N and none of them attend meeting in
and hence they form a group. Which one of the January and February. V did not attend meeting on an odd
following does not belong to that group? date. The one who belongs to Vellore did not attend
(a) K meeting on 12th of any month. Q does not belong to Jaipur.
(b) Q Two persons attend meeting between O and M who
(c) J belongs to Agra. O attend meeting before M and does not
(d) M belong to Indore. Three persons attend meeting between
(e) P the one who belongs to Jaipur and the one who belongs to
Directions (21-25): Answer the questions based on the Prayagraj.
information given below: 21. Who among the following attend lecturer on 15th
There are ten person M, N, O, P, Q, R, S, T, U and V go February?
to attend Mr. Sukumar sir’s lecture on 12th and 15th of (a) O
different month i.e. January, February, April, June and (b) P
July but not neccesarily in the same order. All of them (c) R
belongs to different cities i.e. Ranchi, Indore, Vellore, (d) V
Erode, Noida, Agra, Jaipur, Patna, Lucknow and (e) None of these
Prayagraj. 22. Who among the following belongs to Erode?
The one who attend meeting on 12th June belongs to Patna. (a) U
S attend meeting on 15th of a month which have less than (b) S
31 days but not in February. Three person attend meeting (c) Q
between S and T who belong to Noida. U did not attend (d) M
meeting in June. V and P attend meeting in the same (e) T
month. V belongs to Lucknow. R attend meeting 23. How many persons attend lectures between O and
immediate after S. Three persons attend meeting between U?
R and Q who neither belongs to Ranchi nor Indore. One (a) Three
of them who attend meeting in April belongs to Jaipur. T (b) Two
did not attend meeting at last. The one who belongs to (c) Four
Vellore and the one who belongs to Ranchi did not attend (d) One
meeting on the same date. Two person attend meeting (e) None

Page 413 of 1334


Subscribe The Xpress Video Course & Mock Test Package for Bank & Insurance Exams
If there are any suggestions/ errors in our PDFs Feel Free to contact us via this email: admin@exampundit.in
Ultra Practice Bundle PDF
SBI Clerk/ RBI Assist. Mains – Reasoning
24. Who among the following attend lecture who does not go to mother dairy immediate to S. More than
belongs to Prayagraj? three persons go to mother dairy between Y and V and
(a) S none of them go at first and last. More than three persons
(b) T go to mother dairy between Y and the one who purchased
(c) R 2 litre milk. V and Y do not purchased 7 litre milk. The
(d) P one who purchased 7 litre milk goes to mother dairy
(e) None of these immediate before U. R go to mother dairy immediately
25. Four of the following five are alike in a certain way after Y.
and hence they form a group. Which one of the 26. Who among the following purchased 3 litre and 7
following does not belong to that group? litre milk respectively?
(a) Agra (a) T, X
(b) Erode (b) Y, S
(c) Noida (c) V, T
(d) Prayagraj (d) X, W
(e) Vellore (e) U, V
Directions (26-30): Answer the questions based on the 27. How many people purchased milk between the U
information given below: and T?
There are eight persons R, S, T, U, V, W, X and Y go to (a) None
mother dairy to purchase milk on 4th and 7th of different (b) One
month i.e. March, April, June and August but not (c) Three
necessarily in the same order. (d) Two
The one who purchased 1 litre milk go to mother dairy (e) More than three
immediately after V. Number of persons go to mother 28. S purchased milk in which of the following month?
dairy before R are same as the number of persons go to (a) None of these
mother dairy after the one who purchased 8 litre milk. (b) August
Two person go to mother dairy between W and U and W (c) March
goes after U. T goes on even date and purchased 3 litre (d) April
milk. One person go to mother dairy between X and who (e) June
purchased 6 litre milk. The one who purchased 4 litre milk 29. How much litre milk purchased by R?

Page 414 of 1334


Subscribe The Xpress Video Course & Mock Test Package for Bank & Insurance Exams
If there are any suggestions/ errors in our PDFs Feel Free to contact us via this email: admin@exampundit.in
Ultra Practice Bundle PDF
SBI Clerk/ RBI Assist. Mains – Reasoning
(a) 6 litre belongs to KGMU. The number of doctors go before E is
(b) 4 litre same as the number of doctors go after who belongs Max
(c) 5 litre hospital. No one go between A and the one who belongs
(d) 3 litre to Fortis. C does not belong to Fortis. C goes to hospital
(e) 8 litre before H but not immediately before. Four persons go
30. As per the given arrangement W is related to S and hospital between H who belongs to RML and who belongs
V is related to U, following a certain pattern, which of to PGI. H go after the one who belongs to PGI. One doctor
the following is related to R following the same go in month of December belongs to AIIMS.
pattern? 31. C goes to which of the following hospital?
(a) X (a) Vedanta
(b) W (b) Apollo
(c) Y (c) PGI
(d) U (d) Fortis
(e) S (e) None of these
Directions (31-35): Answer the questions based on the 32. Who among the following doctors goes to hospital
information given below: on 14th September?
There are eight doctors A, B, C, D, E, F, G and H go to (a) F
hospitals on 8th or 14th of different months i.e. August, (b) E
September, November and December. They all belongs to (c) D
different hospitals AIIMS, Fortis, KGMU, PGI, RML, (d) H
Apollo, Max and Vedanta but not necessarily in the same (e) None of these
order. 33. Who among the following doctors goes to AIIMS?
A go to hospital in month of September. One doctor go to (a) G
hospital between A and who belongs to Max hospital, who (b) D
does not go in August. One doctor go between who (c) B
belongs to Max hospital and Apollo hospital. Five doctors (d) A
go between B and F, who go after B. F did not go on 14th (e) None of these
December. D go before G and both of them go hospital in 34. How many doctors goes to hospital between F and
the same month. There is no doctors go before who C?

Page 415 of 1334


Subscribe The Xpress Video Course & Mock Test Package for Bank & Insurance Exams
If there are any suggestions/ errors in our PDFs Feel Free to contact us via this email: admin@exampundit.in
Ultra Practice Bundle PDF
SBI Clerk/ RBI Assist. Mains – Reasoning
(a) 5 only four shopkeepers donate food items between M and
(b) 4 P. S and R donate food items in the same month.
(c) 6 36. Which of the following donate food items in August
(d) 3 and September respectively?
(e) None of these (a) N, S
35. Four of the following five are alike in a certain way (b) O, Q
and hence they form a group. Which one of the (c) O, M
following does not belong to that group? (d) T, P
(a) E (e) P, Q
(b) F 37. How many people donate food items between the N
(c) B and Q?
(d) H (a) None
(e) D (b) One
Directions (36-40): Answer the questions based on the (c) More than three
information given below: (d) Two
There are eight shopkeepers M, N, O, P, Q, R, S and T (e) Three
who donate food items on 18th and 24th in different months 38. Q donate food items in which of the following
i.e. August, September, October and November but not month?
necessarily in the same order. (a) September
S donate food items before the date in which R donates. T (b) October
does not donate food items in October. The number of (c) August
shopkeepers who donate food items between the O and (d) November
M is the same as the number of shopkeepers who donate (e) Cannot be determined
food items between the O and T. S and R donate food 39. On which of the following date P donate food
items in the month which has 30 days. P does not donate items?
food items in August. Q donate food items immediately (a) 18th August
after that date in which O donates. N does not donate food (b) 18th October
items just before that date in which O donates. There are (c) 18th September
(d) 18th November

Page 416 of 1334


Subscribe The Xpress Video Course & Mock Test Package for Bank & Insurance Exams
If there are any suggestions/ errors in our PDFs Feel Free to contact us via this email: admin@exampundit.in
Ultra Practice Bundle PDF
SBI Clerk/ RBI Assist. Mains – Reasoning
(e) None of these W’s age is in a multiple of three. Q and S born in same
40. As per the given arrangement T is related to O and month but not in June. There is one member born between
P is related to R, following a certain pattern, which of Q and X. There is a gap of two members born between R
the following is related to S following the same and the one whose age is 16 year. R born before the one
pattern? whose age is 16 year. O and the one whose age is 18 year
(a) M born in same month. The total age of Q and S is 20 year.
(b) T The age of Q is greater than the age of R and also greater
(c) P than the age of S. The age of W is greater than the age of
(d) R N by 24 year. The one who born in May is older than N
(e) Q but less than 30 year. There are gap of more than three
Directions (41-45): Answer the questions based on the members between U and V. O’s age is 22 years.
information given below: 41. What is the age of X?
Twelve family members M, N, O, P, Q, R, S, T, U, V, W (a) 19
and X are born on 11th and 17th of different month i.e. (b) 16
March, May, June, September, October and December. (c) 49
All of them have different ages i.e. 1, 16, 18, 19, 22, 26, (d) 53
27, 29, 49, 51, 53 and 55 years. If the month contains odd (e) None of these
number of days, then age will be in odd number and if the 42. Who among the following borns on 11th March?
month contains even number of days, then age will be in (a) N
even number. (b) V
M borns on 11th September. T born before M but not (c) T
immediately before. T’s age is 55 year and he borns on (d) M
17th of a month. There is a gap of one member between (e) None of these
T’s and V’s born. V’s age is six year less than T’s age. 43. Who among the following is 19 years old?
There is a gap of three members between P and the one (a) M
whose age is 26 years. P born before the person whose age (b) T
is 26 year. P does not born in March. There is a gap of two (c) Q
members born between W and the one whose age is 26 (d) R
year. W’s age is greater than V’s age but less than T’s age. (e) None of these

Page 417 of 1334


Subscribe The Xpress Video Course & Mock Test Package for Bank & Insurance Exams
If there are any suggestions/ errors in our PDFs Feel Free to contact us via this email: admin@exampundit.in
Ultra Practice Bundle PDF
SBI Clerk/ RBI Assist. Mains – Reasoning
44. How many persons born between who is 18 years month of July.Three persons born between D and the one
and 19 years old? who likes Orange colour. D born after the one who likes
(a) 5 Orange colour. B born immediately before the one who
(b) 4 likes Green colour. B does not born on odd number date.
(c) 6 F likes Pink colour. D and B do not likes Indigo colour.
(d) 3 The one who likes Violet born immediately after F. F
(e) None of these borns on odd date of January. E does not like Orange
45. Four of the following five are alike in a certain way colour. Only one person born between A and the one who
and hence they form a group. Which one of the likes Orange colour. E born after A. C born after H. The
following does not belong to that group? one who likes Indigo colour born on 7th of a month which
(a) R has 31 days, but not on January. Only one person born
(b) T between I and the one who likes Black colour. E does not
(c) O like Red colour. No person born between the one who
(d) M likes Black colour and the one who likes Red colour.
(e) U 46. Who among the following likes Red colour?
Directions (46-50): Answer the questions based on the (a) H
information given below: (b) D
Ten persons A, B, C, D, E, F, G, H, I and J born on 7th (c) B
and 14th of the different months i.e. January, February, (d) G
June, July and September. Each one of them likes (e) None of these
different colour i.e. Pink, Violet, Indigo, Blue, Green, 47. Which among the following colour likes by who
White, Black, Yellow, Orange and Red but not born on 7th February?
necessarily in the same order. (a) Green
C and H born on same month but not in February. Only (b) Pink
one person born between C and the one who likes White (c) Orange
colour. H does not born in February month. J does not (d) White
born immediate after A. Two persons born between the (e) Yellow
one who likes Indigo colour and E. Three persons born 48. How many persons born between who likes Yellow
between E and the one who likes Blue colour. D born in and H?

Page 418 of 1334


Subscribe The Xpress Video Course & Mock Test Package for Bank & Insurance Exams
If there are any suggestions/ errors in our PDFs Feel Free to contact us via this email: admin@exampundit.in
Ultra Practice Bundle PDF
SBI Clerk/ RBI Assist. Mains – Reasoning
(a) Three (e) None of these
(b) Two 50. Four of the following five are alike in a certain way
(c) Four and hence they form a group. Which one of the
(d) One following does not belong to that group?
(e) None (a) B
49. Who among the following born on 7th September? (b) H
(a) J (c) D
(b) A (d) E
(c) F (e) J
(d) B

Puzzle Month and Date – Answer and Explanation


SOLUTION(1-5): 4. There are two joining between the joining of A and
Explanation in detail: C.
1. B’s joining is on 18th of the month having 30 days. 5. The joining of one who joins BOB is immediately
2. The joining of one who joins BOI is just after the after C’s joining.
joining of B.
3. A joins just after B.

Page 419 of 1334


Subscribe The Xpress Video Course & Mock Test Package for Bank & Insurance Exams
If there are any suggestions/ errors in our PDFs Feel Free to contact us via this email: admin@exampundit.in
Ultra Practice Bundle PDF
SBI Clerk/ RBI Assist. Mains – Reasoning

6. There are two joinings after the one who joins


Indian bank. In CASE2 A is placed after Indian bank but according
7. Joining of A is in month having 31 days but before to question it is false statement, So CASE2 is cancelled
the one who joins Indian Bank. out.
8. There are two joinings between the joining of H
and the one who joins Canara and either one of
them has joins at last.
9. The joining of one who join SBI and BOM are
next to each other.
10. H and the one who joins SBI joins on same date.
11. H does not join SBI.

Page 420 of 1334


Subscribe The Xpress Video Course & Mock Test Package for Bank & Insurance Exams
If there are any suggestions/ errors in our PDFs Feel Free to contact us via this email: admin@exampundit.in
Ultra Practice Bundle PDF
SBI Clerk/ RBI Assist. Mains – Reasoning

There is no space for F in CASE1b so CASE1b is


cancelled out.
12. E joins Hdfc and his joining is before the one who 14. B does not join in the same month in which C
joins Indian bank. joins.
13. There are three joinings between the joining of F 15. B and C do not join Icici.
and the one who joins Icici. 16. G and the one who joins BOM joins on the same
date.
17. G does not join BOM.

Page 421 of 1334


Subscribe The Xpress Video Course & Mock Test Package for Bank & Insurance Exams
If there are any suggestions/ errors in our PDFs Feel Free to contact us via this email: admin@exampundit.in
Ultra Practice Bundle PDF
SBI Clerk/ RBI Assist. Mains – Reasoning
So J does not born in January.

1. d
1. Same number of people was born before B as after
2. a
J.
3. b
2. One person was born between B and M.
4. b
3. M does not like fruits.
5. e
4. No person was born between D and the one who
SOLUTION(6-10):
likes Banana.
Explanation in detail:
1. D was born in April.
2. One person was born between D and J.
3. J does not like any colour.
Page 422 of 1334
Subscribe The Xpress Video Course & Mock Test Package for Bank & Insurance Exams
If there are any suggestions/ errors in our PDFs Feel Free to contact us via this email: admin@exampundit.in
Ultra Practice Bundle PDF
SBI Clerk/ RBI Assist. Mains – Reasoning
6. The one who likes Brown colour was not born on
20th of any month.

5. No person was born between the ones who like


Brown colour and the one who likes Grapes.

Page 423 of 1334


Subscribe The Xpress Video Course & Mock Test Package for Bank & Insurance Exams
If there are any suggestions/ errors in our PDFs Feel Free to contact us via this email: admin@exampundit.in
Ultra Practice Bundle PDF
SBI Clerk/ RBI Assist. Mains – Reasoning

7. A likes Black colour.


8. One person was born between A and the one who
likes Apple.
9. The one who likes Mango and Apple were born
either on the same date or in the same month.
So only possibility of A in month of March.

10. No person was born between C and one who likes


Orange colour.
11. K does not like fruits.
There is only one place for Mango.

Page 424 of 1334


Subscribe The Xpress Video Course & Mock Test Package for Bank & Insurance Exams
If there are any suggestions/ errors in our PDFs Feel Free to contact us via this email: admin@exampundit.in
Ultra Practice Bundle PDF
SBI Clerk/ RBI Assist. Mains – Reasoning
one who likes Orange colour and the one who likes
Grapes.
So there is no space for Pink in CASE1 and CASE2a
to satisfy the above statement so CASE1 and CASE2a
is cancelled out.

6. d
7. b
12. The number of people born between the one who 8. a
likes Pink colour and the one who likes Mango is 9. c
same as the number of people born between the 10. e
SOLUTION (11-15):

Page 425 of 1334


Subscribe The Xpress Video Course & Mock Test Package for Bank & Insurance Exams
If there are any suggestions/ errors in our PDFs Feel Free to contact us via this email: admin@exampundit.in
Ultra Practice Bundle PDF
SBI Clerk/ RBI Assist. Mains – Reasoning
Explanation in detail: 6. No one attend meeting before who belongs to
1. Five persons ateend meeting between T and X, Udaipur.
who attend meeting after T. 7. The number of person attend meeting before W is
2. X does not ateend meeting at last. same as the number of person attend meeting after
3. S attend the meeting in September. the one who belongs to Tonk.
4. Only one person attend the meeting between S and
who belongs to Tonk, who does not attend the
meeting in August.

8. Four persons attend meeting between Z, who


belongs to Ajmer and the one who belongs to
Kota.
9. Z attend meeting after who belongs to Kota.
10. One of the person who attend meeting in
5. V attend meeting before Y and both of them attend
November belongs to Jaipur.
meeting in the same month.

Page 426 of 1334


Subscribe The Xpress Video Course & Mock Test Package for Bank & Insurance Exams
If there are any suggestions/ errors in our PDFs Feel Free to contact us via this email: admin@exampundit.in
Ultra Practice Bundle PDF
SBI Clerk/ RBI Assist. Mains – Reasoning

11. One person attend meeting between the ones who


belong to Nagaur and Tonk. 13. No one attend meeting between S and the one who
12. U attend the meeting before Z but not immediately belongs to Jodhpur.
before. 14. U does not belongs to Jodhpur.
There is no particular place for Jodhpur in CASE2 so
CASE2 is cancelled out.

Page 427 of 1334


Subscribe The Xpress Video Course & Mock Test Package for Bank & Insurance Exams
If there are any suggestions/ errors in our PDFs Feel Free to contact us via this email: admin@exampundit.in
Ultra Practice Bundle PDF
SBI Clerk/ RBI Assist. Mains – Reasoning
2. No person play between M and who belongs to
RCB .
3. M does not belongs to IPL team.
4. Three persons play between M and one who
belongs to MR.

11. b
12. d
13. a
14. c
15. c
SOLUTION(16-20):
Explanation in detail:
1. The one who belongs to RCB play in the month of
July.
Page 428 of 1334
Subscribe The Xpress Video Course & Mock Test Package for Bank & Insurance Exams
If there are any suggestions/ errors in our PDFs Feel Free to contact us via this email: admin@exampundit.in
Ultra Practice Bundle PDF
SBI Clerk/ RBI Assist. Mains – Reasoning
5. There is no person play between one who belongs 6. The number of person play after the one who
to MR and RR team. belongs to RR is one less thn the number of person
play before J.
The one who belongs to IPL team play only on 7. The one who belongs to ST play immediately
4th and 9th of the month. before K.
8. K do not belong to MR.
9. The number of person play before K is same as
number of person play after P.
The one who belongs to BBL team play only on 2nd and
11th of the month.
In CASE2 there is no place for ST team in 2nd August
bacause statement for P can be invalid.

Page 429 of 1334


Subscribe The Xpress Video Course & Mock Test Package for Bank & Insurance Exams
If there are any suggestions/ errors in our PDFs Feel Free to contact us via this email: admin@exampundit.in
Ultra Practice Bundle PDF
SBI Clerk/ RBI Assist. Mains – Reasoning

10. Three person play between the one who belongs to


CSK and Q. 12. L plays after the one who belong to RR.
11. Q and the one who belongs to PS play in the same 13. N donot belong to IPL team.
month but not in July. 14. O does not belong to ST team.
Q can’t play match in month of July So CASE1 is
cancelled out.
The one who belongs to BBL team play only on 2nd and
11th of the month.

Page 430 of 1334


Subscribe The Xpress Video Course & Mock Test Package for Bank & Insurance Exams
If there are any suggestions/ errors in our PDFs Feel Free to contact us via this email: admin@exampundit.in
Ultra Practice Bundle PDF
SBI Clerk/ RBI Assist. Mains – Reasoning

16. a
17. e
18. b
19. d
20. d
SOLUTION(21-25):
Explanation in detail:
5. R attend meeting immediate after S.
1. The one who attend meeting on 12th June belongs
6. V and P attend meeting in the same month.
to Patna.
7. Three persons attend meeting between R and Q
2. S attend meeting on 15th of a month which have
who neither belongs to Ranchi nor Indore.
less than 31 days but not in February.
8. V belongs to Lucknow.
3. Three peeerson attend meeting between S and T
9. V did not attend meeting on an odd date.
who belong to Noida.
4. T did not attend meeting at last.

Page 431 of 1334


Subscribe The Xpress Video Course & Mock Test Package for Bank & Insurance Exams
If there are any suggestions/ errors in our PDFs Feel Free to contact us via this email: admin@exampundit.in
Ultra Practice Bundle PDF
SBI Clerk/ RBI Assist. Mains – Reasoning

12. Two persons attend meeting between O and M


who belongs to Agra.
13. O attend meeting before M and does not belong to
Indore.

10. Two person attend meeting between U and N and


none of them attend meeting in January and
February.
11. U did not attend meeting in June.

Page 432 of 1334


Subscribe The Xpress Video Course & Mock Test Package for Bank & Insurance Exams
If there are any suggestions/ errors in our PDFs Feel Free to contact us via this email: admin@exampundit.in
Ultra Practice Bundle PDF
SBI Clerk/ RBI Assist. Mains – Reasoning

14. One of them who attend meeting in April belongs 16. Three persons attend meeting between the one

to Jaipur. who belongs to Jaipur and the one who belongs to

15. Q does not belong to Jaipur. Prayagraj.

In CASE2 Q can not belongs to Jaipur So CASE2 is 17. One of them who attend meeting in April belongs

cancelled out. to Jaipur.


The one who belongs to Jaipur can’t attend
meeting on 12th April because then there is no space
for the one who belongs to Ranchi.

18. The one who belongs to Vellore and the one who
belongs to Ranchi did not attend meeting on the
same date.
Page 433 of 1334
Subscribe The Xpress Video Course & Mock Test Package for Bank & Insurance Exams
If there are any suggestions/ errors in our PDFs Feel Free to contact us via this email: admin@exampundit.in
Ultra Practice Bundle PDF
SBI Clerk/ RBI Assist. Mains – Reasoning
19. The one who belongs to Vellore did not attend 25. b
meeting on 12th of any month. SOLUTION(26-30):
20. O does not belong to Indore. Explanation in detail:
21. R and Q neither belongs to Ranchi nor Indore. 1. More than three persons go to mother dairy
between Y and V and none of them go at first and
last.
2. R go to mother dairy immediately after Y.
3. The one who purchased 1 litre milk go to
mother dairy immediately after V.

4. More than three persons go to mother dairy


between Y and the one who purchased 2 litre milk.
21. e
5. Number of persons go to mother dairy before R are
22. c
same as the number of persons go to mother dairy
23. a
after the one who purchased 8 litre milk.
24. d

Page 434 of 1334


Subscribe The Xpress Video Course & Mock Test Package for Bank & Insurance Exams
If there are any suggestions/ errors in our PDFs Feel Free to contact us via this email: admin@exampundit.in
Ultra Practice Bundle PDF
SBI Clerk/ RBI Assist. Mains – Reasoning

There is no space for 7 litre so CASE2 is cancelled out.


8. T goes on even date and purchased 3 litre milk.
9. One person go to mother dairy between X and who
purchased 6 litre milk.
10. The one who purchased 4 litre milk does not go to
mother dairy immediate to S.
11. V and Y do not purchased 7 litre milk.
6. Two person go to mother dairy between W and U
and W goes after U.
7. The one who purchased 7 litre milk goes to mother
dairy immediate before U.

Page 435 of 1334


Subscribe The Xpress Video Course & Mock Test Package for Bank & Insurance Exams
If there are any suggestions/ errors in our PDFs Feel Free to contact us via this email: admin@exampundit.in
Ultra Practice Bundle PDF
SBI Clerk/ RBI Assist. Mains – Reasoning

26. a
27. c
4. D goes before G and both of them go hospital in
28. e
the same month.
29. b
5. There is no doctors go before who belongs to
30. d
KGMU.
SOLUTION(31-35):
Explanation in detail:
1. A go to hospital in month of September.
2. Five doctors go between B and F, who go after B.
3. F did not go on 14th December.

6.

Page 436 of 1334


Subscribe The Xpress Video Course & Mock Test Package for Bank & Insurance Exams
If there are any suggestions/ errors in our PDFs Feel Free to contact us via this email: admin@exampundit.in
Ultra Practice Bundle PDF
SBI Clerk/ RBI Assist. Mains – Reasoning
9. One doctor goes to hospital between A and who
belongs to Max hospital who does not go in
August.
10. One doctor goes between who belongs to Max
hospital and Apollo hospital.

7. Four persons go hospital between H who belongs


to RML and who belongs to PGI.
8. H go after the one who belongs to PGI.

11. The number of doctors go before E is same as the


number of doctors go after who belongs Max
hospital.
12. No one go between A and the one who belongs to
Fortis.
13. C does not belong to Fortis.

Page 437 of 1334


Subscribe The Xpress Video Course & Mock Test Package for Bank & Insurance Exams
If there are any suggestions/ errors in our PDFs Feel Free to contact us via this email: admin@exampundit.in
Ultra Practice Bundle PDF
SBI Clerk/ RBI Assist. Mains – Reasoning
14. C goes to hospital before H but not immediately
before.
15. One doctor go in month of December belongs to
AIIMS.

31. a
32. e
33. e
34. b
35. d
SOLUTION(36-40):
Explanation in detail:
1. S and R donate food items in the same month.
2. S donate food items before the date in which R
donates.
3. S and R donate food items in the month which has
30 days.

So in CASE1 there is no place for AIIMS so CASE1 is


cancelled out.

Page 438 of 1334


Subscribe The Xpress Video Course & Mock Test Package for Bank & Insurance Exams
If there are any suggestions/ errors in our PDFs Feel Free to contact us via this email: admin@exampundit.in
Ultra Practice Bundle PDF
SBI Clerk/ RBI Assist. Mains – Reasoning

4. There are only four shopkeepers donate food items


8. Q donate food items immediately after O donates.
between M and P.
9. N does not donate food items just before O
5. T does not donate food items in October.
donates.
6. The number of shopkeepers who donate food
There is no space for Q so CASE1 is cancelled out.
items between the O and M is the same as the
number of shopkeepers who donate food items
between the O and T.
7. P does not donate food items in August.

Page 439 of 1334


Subscribe The Xpress Video Course & Mock Test Package for Bank & Insurance Exams
If there are any suggestions/ errors in our PDFs Feel Free to contact us via this email: admin@exampundit.in
Ultra Practice Bundle PDF
SBI Clerk/ RBI Assist. Mains – Reasoning
3. T’s age is 55 year and he borns on 17th of a month.
4. There is a gap of one member between T’s and V’s
born.
5. V’s age is six year less than T’s age.
So age of V = T-6= 55-6= 49 years
V=49 years
According to Question V should be born in March
because age of V is odd.

36. a
37. c
38. b
39. d
40. e
SOLUTION(41-45):
Explanation in detail:
1. M borns on 11th September.
2. T borns before M but not immediately before.

Page 440 of 1334


Subscribe The Xpress Video Course & Mock Test Package for Bank & Insurance Exams
If there are any suggestions/ errors in our PDFs Feel Free to contact us via this email: admin@exampundit.in
Ultra Practice Bundle PDF
SBI Clerk/ RBI Assist. Mains – Reasoning
6. There are gap of three members between P and the 10. W’s age is greater than V’s age but less than T’s
one whose age is 26 years. age.
7. P borns before the person whose age is 26 year. 11. W’s age is in a multiple of three.
8. P does not born in March. 12. Q and S born in same month but not in June.
9. There are gap of two members born between W 13. There is one member born between Q and X.
and the one whose age is 26 year. W’s age must be 51 years according to question
According to Question whose age is 26 years should be 55>51>49
born in June and September. Q must be born on 11th December because if Q borns
on 17th December then there is no place for X.

Page 441 of 1334


Subscribe The Xpress Video Course & Mock Test Package for Bank & Insurance Exams
If there are any suggestions/ errors in our PDFs Feel Free to contact us via this email: admin@exampundit.in
Ultra Practice Bundle PDF
SBI Clerk/ RBI Assist. Mains – Reasoning

14. There is a gap of two members born between R


and the one whose age is 16 year.
15. R borns before the one whose age is 16 year.
16. O and the one whose age is 18 year born in same
month.
17. O’s age is 22 years.
According to Question whose age is 16/18/22/26
years should be born in June and September.

18. There are gap of more than three members


between U and V.
CASE2 is cancelled out because there are not more
than three members born between U and V.

Page 442 of 1334


Subscribe The Xpress Video Course & Mock Test Package for Bank & Insurance Exams
If there are any suggestions/ errors in our PDFs Feel Free to contact us via this email: admin@exampundit.in
Ultra Practice Bundle PDF
SBI Clerk/ RBI Assist. Mains – Reasoning

21. W is greater than the age of N by 24 year.


22. The age of the one who born in May is older than
N but less than 30 year.
W=N+24=51 so N=27 years old.

19. The total age of Q and S is 20 year.


20. The age of Q is greater than the age of R and also
greater than the age of S.
Q is older than R means Q>18 but Q+S=20 years.
Also Q>S so Q must be 19 years old.

Page 443 of 1334


Subscribe The Xpress Video Course & Mock Test Package for Bank & Insurance Exams
If there are any suggestions/ errors in our PDFs Feel Free to contact us via this email: admin@exampundit.in
Ultra Practice Bundle PDF
SBI Clerk/ RBI Assist. Mains – Reasoning

41. d
42. a
43. c
44. a
45. b
SOLUTION(46-50):
Explanation in detail:
1. The one who likes Indigo colour born on 7th of a
month which has 31 days, but not on January. 4. D born in month of July.

2. Two persons born between the one who likes 5. Three persons born between D and the one who

Indigo colour and E. likes Orange colour.

3. Three persons born between E and the one who 6. D born after the one who likes Orange colour.
7. D and B do not likes Indigo colour.
likes Blue colour.

Page 444 of 1334


Subscribe The Xpress Video Course & Mock Test Package for Bank & Insurance Exams
If there are any suggestions/ errors in our PDFs Feel Free to contact us via this email: admin@exampundit.in
Ultra Practice Bundle PDF
SBI Clerk/ RBI Assist. Mains – Reasoning

According to Question E does not like Orange colour


8. The one who likes Violet born immediately after so CASE2 is cancelled out.
F. 11. Only one person born between A and the one who
9. F borns on odd date of January. likes Orange colour.
10. E does not likes Orange colour. 12. E born after A. C born after H.
13. C and H born on same month but not in Febraury.
14. Only one person born between C and the one who
likes White colour.
15. H does not born in February month.

Page 445 of 1334


Subscribe The Xpress Video Course & Mock Test Package for Bank & Insurance Exams
If there are any suggestions/ errors in our PDFs Feel Free to contact us via this email: admin@exampundit.in
Ultra Practice Bundle PDF
SBI Clerk/ RBI Assist. Mains – Reasoning

19. Only one person born between I and the one who
likes Black colour.
20. E does not likes Red colour.
21. No person born between the one who likes Black
colour and the one who likes Red colour.
22. J does born immediate after A.

There is no space for C and H in CASE1b so CASE1b


is cancelled out.
16. B born immediately before the one who likes
Green colour.
17. B does not born on odd number date.
18. F likes Pink colour.

Page 446 of 1334


Subscribe The Xpress Video Course & Mock Test Package for Bank & Insurance Exams
If there are any suggestions/ errors in our PDFs Feel Free to contact us via this email: admin@exampundit.in
Ultra Practice Bundle PDF
SBI Clerk/ RBI Assist. Mains – Reasoning
46. b
47. d
48. a
49. a
50. e

Download Puzzles Practice Questions PDF


Get More Reasoning Practice Questions PDF
Puzzle Days
Direction (1-3): Study the following information immediately before T. Neither L nor R works in the
carefully and answer the questions given below: company Z. Only one person attends the seminar between
Seven persons K, J, L, R, T, P and M attend the seminar M and the one who works in the company W. M does not
in a week starting from Monday and ends on Sunday. All work in the company D. T and P does not work in the
of them are working in different companies namely, W, company B.
D, N, V, Z, Y and B, but not necessarily in the same order. 1. Who among the following person works in the
L attends the seminar between Tuesday and Friday. The company B?
one who works in the company Y attends the seminar a) K
immediately before L. Three persons attend the seminar b) J
between K and R. R attends the seminar immediately after c) M
the one who works in the company D. R does not attend d) T
the seminar on Saturday. The one who works in the e) None of these
company D does not attend the seminar on the first day. R 2.How many persons attend the seminar between the
attends the seminar after K. T attends the seminar one who works in the company Y and T?
immediately before the one who works in the company Z. a) None
The one who works in the company N attends the seminar b) One

Page 447 of 1334


Subscribe The Xpress Video Course & Mock Test Package for Bank & Insurance Exams
If there are any suggestions/ errors in our PDFs Feel Free to contact us via this email: admin@exampundit.in
Ultra Practice Bundle PDF
SBI Clerk/ RBI Assist. Mains – Reasoning
c) Two Banana and U, who visits New York. Only one person
d) Three visits between the one who likes Orange and one who
e) None of these likes Apple. Only two person visits before the one who
3.If J is related to the company D, L is related to the likes Mango. The one who likes Guava visits on one of the
company V, in the same way R is related to which of day before the one who likes Papaya and R doesn’t like
the following company? Apple. The one who likes orange neither visits on Sunday
a) D nor visits New Delhi. The one who likes Guava does not
b) Y visit Singapore.
c) W 4. Which of the following statement is true?
d) Z a) Q visits London just before one who likes Mango
e) None of these b) One who visits on Tuesday visits Paris
(Directions 4–8): Study the following information c) U visits New York just after one who visits Rome
carefully and answer the questions given below it d) R visits Italy and likes Orange
Seven friends namely – P, Q, R, S, T, U, and V like e) All the above given statements are false
different fruits viz. Apple, Banana, Mango, Orange, 5. Which of the following combination is true?
Guava, Grapes and Papaya. Each of them visits different a) Tuesday – New Delhi – Q – Guava
cities viz. Rome, Singapore, London, Paris, New Delhi, b) Orange – New York – Thursday – U
New York and Italy on a different day starting from c) R – Thursday – Grapes – Italy
Monday to Sunday. All the given information is not d) Saturday – Papaya – V – Rome
necessary in the same order. e) Monday – Q – Singapore – Banana
T, who neither visits London nor he likes Mango, visits on 6. Who visits on Sunday?
one of the day before Thursday. Q, who neither likes a) One who likes Apple
Grapes nor visits Singapore, visits on one of the day b) One who visits New York
before R. One who likes Grapes visits Italy on Thursday. c) One who visits just after one who likes Orange
P visits Rome on Saturday. V visits Paris on one of the d) Either (i) or (ii)
day before the one who likes Apple. Only one person e) Either (i) or (iii)
visits between the one who likes Mango and the one who 7. Who visits just after the one who likes Mango?
visits London. T visits Just after the one who likes Banana. a) R
There are three persons visit between the one who likes b) T

Page 448 of 1334


Subscribe The Xpress Video Course & Mock Test Package for Bank & Insurance Exams
If there are any suggestions/ errors in our PDFs Feel Free to contact us via this email: admin@exampundit.in
Ultra Practice Bundle PDF
SBI Clerk/ RBI Assist. Mains – Reasoning
c) P immediatelybefore the person who has bought Ford. Rolls
d) S Royce is neither Blue nor White incolour. Only three
e) Q people have bought cars between the person who has
8. How many person visits after the one who visits bought Honda and the person who has bought a Yellow
Paris? coloured car. Only three people have bought cars between
a) Two R and T. Tesla has been bought after Hyundai but neither
b) One immediately after nor on Monday. The Blue coloured car
c) Four is bought after the Black coloured carbut before the White
d) Three coloured car. Q has bought a car on one of the days before
e) None the person who has bought MG HECTOR. The number of
Directions (9-13): Refer to the data below and answer cars bought before Tesla is as same as the number of cars
the questions that follow. bought after Hyundai. Q has bought a Silver car. Only two
P, Q, R, S, T, U, V and W are eight different people who people have bought the cars before S. T has not bought a
have bought eight different cars – Honda, Mercedes, MG car on the last day. P and S have bought neither aMercedes
HECTOR, Tata, Tesla, Rolls Royce, Ford and Hyundai on nor aTata. Mercedes car is not Silverincolour. Red
eight different days starting from Monday to Monday. The colouredcar is bought immediately after Greycoloured car
cars are of eight different colours viz. Red, Blue, White, but immediately before Black coloured car.
Yellow, Black, Grey, Beige and Silver. All the 9.Which of the following car is of White colour?
information is not necessarily in the same order. a) Honda
Four people have bought the cars between Q and the b) Mercedes
person who has bought MG HECTOR. At least two c) Hyundai
people have bought cars after the person who has bought d) MG HECTOR
Ford. Grey and Blackcoloured cars have not been bought e) Ford
immediately before or after a Yellow coloured car. U has 10.Whichof the following car does P bought?
bought a car immediately after W. Hyundai is neither Red a) Honda
nor Silver incolour. Neither P nor U has bought MG b) Tata
HECTOR. Beige colouredcar is boughtthree days before c) Rolls Royce
Mercedes was bought. S has bought a car either d) Mercedes
immediately or on one of the days after V but e) MG HECTOR

Page 449 of 1334


Subscribe The Xpress Video Course & Mock Test Package for Bank & Insurance Exams
If there are any suggestions/ errors in our PDFs Feel Free to contact us via this email: admin@exampundit.in
Ultra Practice Bundle PDF
SBI Clerk/ RBI Assist. Mains – Reasoning
11.In which of the following day does W bought the the one who is an Actor. As many persons attend the
car? lecture after the one who is an Engineer is one more than
a) Tuesday as many persons attend the lecture before R. Three
b) Wednesday persons attend the lecture between T and the one who is a
c) Thursday teacher, who attends the lecture before T. Only one person
d) Friday attends the lecture between K and the one who is an
e) Saturday Engineer. As many persons attend the lecture before S as
12.Who among the following person has bought the after X. G attends the lecture before X. As many persons
Red coloured car? attend the lecture before G as after the one who is a
a) P lawyer. The one who is a doctor attends the lecture before
b) W M. X is not an auditor.
c) S 14.How many persons attend the lecture after M?
d) Q a) 1
e) U b) 2
13.What is the colour of the MG HECTOR car? c) 3
a) White d) 4
b) Black e) None of these
c) Beige 15.If S is related to Lawyer, G is related to Architect,
d) Blue then M is related to which of the following?
e) Yellow a) Engineer
Direction (14-16): Study the following information b) Doctor
carefully to answer the given questions. c) Actor
Seven persons attend the seminar in a week starting from d) Auditor
Monday and ends on Sunday. Each one of them are in e) None of these
different professionviz.Doctor, Lawyer, Engineer, 16.Which of the following statement is not true?
Auditor, Architect, Teacher and Actor, but not necessarily a) More than three persons attend the lecture before
in the same order. G
K attends the seminar on Wednesday. T is not an
Engineer. Two persons attend the lecture between X and

Page 450 of 1334


Subscribe The Xpress Video Course & Mock Test Package for Bank & Insurance Exams
If there are any suggestions/ errors in our PDFs Feel Free to contact us via this email: admin@exampundit.in
Ultra Practice Bundle PDF
SBI Clerk/ RBI Assist. Mains – Reasoning
b) As many persons attend the lecture between S and in the company B. T attends the lecture immediately
the one who is a teacher as between T and the one before the one who works in the company H. K does not
who is an Engineer work in the company V. M doesn’t attends the lecture on
c) Two persons attend the lecture between M and the Monday.
one who is a doctor 17) Who among the following persons works in the
d) Less than one person attends the lecture between company L?
G and the one who is a lawyer a) G
e) All are true b) P
Direction (17-21): Study the following information c) K
carefully and answer the questions given below. d) R
Seven persons K, M, G, P, F, R and T attend the lecture in e) None of these
a week starting from Monday to Sunday. Each one of them 18.) How many persons attend the lecture between P
works in different company viz., V, H, B, Q, L, S and Z. and the one who works in the company Z?
All the above information is not necessarily in the same a) One
order. b) Two
The one who works in the company S attends the lecture c) Three
between Wednesday and Saturday. R attends the lecture d) Four
before the one who works in the company S. The number e) None of these
of persons attend the lecture between T and the one who 19.) If F is related to H, R is related to S, in the same
works in the company Q is one more than the number of way who among the following is related to Q?
persons attend the lecture between F and the one who a) K
works in the company S. Neither M nor K works in the b) G
company Z. Three persons attend the lecture between R c) M
and the one who works in the company Q. T does not d) P
attend the lecture either on Saturday or on Monday. T does e) None of these
not work in the company Q. Two persons attend the 20) Who among the following person attends the
lecture between F and M. Three persons attend the lecture lecture immediately after the one who works in the
between K and the one who works in the company B. G company V?
attends the lecture immediately before K. R does not work a) K

Page 451 of 1334


Subscribe The Xpress Video Course & Mock Test Package for Bank & Insurance Exams
If there are any suggestions/ errors in our PDFs Feel Free to contact us via this email: admin@exampundit.in
Ultra Practice Bundle PDF
SBI Clerk/ RBI Assist. Mains – Reasoning
b) P between A and F’s seminar. Neither the person from
c) M China nor the one from Egypt takes seminar before G. C
d) G is from Canada and he takes seminar immediately before
e) None of these F, but not on Monday. B does not belong to France but
21) Four of the following five are alike in a certain way both of them takesseminar on Tuesday. Neither H nor G
and hence they form a group. Which one of the is from France. G does not take seminar before H.
following does not belong to that group? 22.G belongs to which of the following country?
a) F-Tuesday a) Greece
b) R-Thursday b) Italy
c) G-Saturday c) Brazil
d) K-Monday d) France
e) T-Wednesday e) Kuwait
Directions (22-26): Study the following information 23.Which of the following statement about F is false?
carefully and answer the questions given below. An a) As many persons take seminar before F as after H
international seminar was organized from 16th May 2020, b) F either belongs to China or Egypt
which isTuesday, to 23rd May 2020. Eight persons A, B, c) F takes seminar immediately after the one who
C, D, E, F, G and H from various countries namely, Brazil, belongs to Canada
Canada, China, Egypt, France, Greece, Italy and Kuwait d) F takes seminar on Tuesday
takes the seminar on different days starting from the first e) All are true
day, but not necessarily in the same order. Only one 24.If B is related to Kuwait and G is related to Canada,
person from one country attends the seminar on each day. then in the same way, C is related to which of the
The person from Brazil takes seminar immediately before following countries?
the person from Greece. The number of seminars taken a) Egypt
after the person from Italy is more than the number of b) Italy
seminars taken between the persons from Greece and c) France
Italy. There is a gap of at least 2 days between D’s seminar d) Greece
and the person from Brazil. D takes seminar on 20th May. e) China
A takes seminar immediately before the person from Italy 25.The person from China takes seminar on which of
but he is not from Greece. Three seminars are taken the following dates?

Page 452 of 1334


Subscribe The Xpress Video Course & Mock Test Package for Bank & Insurance Exams
If there are any suggestions/ errors in our PDFs Feel Free to contact us via this email: admin@exampundit.in
Ultra Practice Bundle PDF
SBI Clerk/ RBI Assist. Mains – Reasoning
a) 21st a) None
b) 20th b) 1
c) 23rd c) 2
d) 22nd d) 3
e) None of these e) None of these
26.Who among the following takes seminar on 18th 28.If physics is related to Tuesday, Zoology is related
May 2020? to Friday, then off day is related to which of the
a) The person from Kuwait following?
b) B a) Sunday
c) C b) Monday
d) H c) Wednesday
e) E d) Saturday
Direction(27-28): Study the following information and e) None of these
answer the given questions. [WRONG] Direction(29-33):Study the following information
Six persons take lectures on different subjects in a week given below.
starting from Monday and ends on Sunday. One day is off Seven persons attends the class in a week starting from
day. Monday and ends on Sunday and they bought different
Two lectures were given between the one who taught items T-shirt, Shoes, Top, Jeans, Bag, TV and
English and the one who taught chemistry. The one who Refrigerator, but not necessarily in the same order.
taught Zoology givesthe lecture immediately after the one Three persons attend the class between R and the one who
who taught Botany. Only one day was there between the bought Refrigerator. Only two persons attend the class
one who taught physics and the one who taught Botany. after the one who bought Jeans. R attends the class before
As many persons givelecture before the one who taught the one who bought Refrigerator. Only one person attends
chemistry as after the one who taught English. At least two the class between N and the one who bought Top. The one
lectures were given after the one who taught Zoology. One who bought T-shirt attends the class immediately before
of the person taught Mathematics. the one who bought Top. W does not boughtTop. The one
27.If English lecture was not on Tuesday, then how who bought Bag attends the class immediately before R.
many the lecture/s is/are there between the one who As many persons attend the class before W asafter N. G
taught English and the one who taught Zoology? attends the class immediately after the one who bought

Page 453 of 1334


Subscribe The Xpress Video Course & Mock Test Package for Bank & Insurance Exams
If there are any suggestions/ errors in our PDFs Feel Free to contact us via this email: admin@exampundit.in
Ultra Practice Bundle PDF
SBI Clerk/ RBI Assist. Mains – Reasoning
TV. N and G does not attend the class consecutively. K e) W
attends the class immediately after L. W does not 33. Four of the following five are like in a certain way
boughtShoes. P is one of the persons. based on the above arrangement. Find which of the
29.How many persons attend the class after the one following one does not belongs to the group?
who bought shoes? a) G-Jeans
a) No one b) R-Shoes
b) 1 c) N-Top
c) 2 d) W-T-shirt
d) 3 e) L-TV
e) None of these Direction(34-38):Study the following information
30.Whoamong the following person bought Top? carefully and answer the questions given below it.
a) L Six persons namely – G, Q, T, R, M and K attends seminar
b) R on different days of a week starting from Monday to
c) W Sunday. One person attends seminar on one day only.
d) K Each person was born in different year from 1992 to 1997.
e) None of these Each one of them works in different company. There is a
31.If P is related to Bag, R is related to Top, in the same holiday on one of the days.
way K is related to which of the followingitem? Two persons attend the seminar between G and the one
a) Refrigerator who was born in 1993. The one who works in Wipro
b) Shoes attends the seminar immediately after the one who was
c) Jeans born in 1993. More than three persons attend the seminar
d) TV between G and Q. Q does not work in Accenture. The one
e) None of these who works in HCL attends the seminar immediately
32. Who among the following person attends the class before M. Two persons attend the seminar between the
immediately after K? one who works in HCL and the one who was born in 1996.
a) G The one who works in Wipro was born in an odd
b) The one who bought Refrigerator numbered year. The one who works in CTS was born in
c) The one who bought Bag 1994. The one who works in Accenture attends the
d) P seminar before Wednesday. T attends the seminar before

Page 454 of 1334


Subscribe The Xpress Video Course & Mock Test Package for Bank & Insurance Exams
If there are any suggestions/ errors in our PDFs Feel Free to contact us via this email: admin@exampundit.in
Ultra Practice Bundle PDF
SBI Clerk/ RBI Assist. Mains – Reasoning
the one who works in CTS. G attends the seminar after Q. e) None of these
Holiday is not on Sunday. Three persons attend the 37.Four of the following five are like in a certain way
seminar between T and the one who works in Infosys. T based on the above arrangement. Find which of the
does not born in 1994. At least two persons attend the following one does not belongs to the group?
seminar after K. R does not attend the seminar on Friday. a) 1995-TCS
M is younger than T. Q was born in an even numbered b) 1996-CTS
year. The one who was born in 1997 does not attend the c) M-HCL
seminar immediately before the holiday. One of the d) R-Wipro
companies is TCS. One who attends seminar on Sunday e) T-Infosys
works in Infosys. 38.Which of the following day is holiday?
34.What is the age difference between the one who a) Monday
works in CTS and T (in years)? b) Tuesday
a) 1 c) Wednesday
b) 2 d) Thursday
c) 3 e) None of these
d) 4 Direction(39-42):Study the following information
e) None of these carefully and answer the given questions:
35.If Q is related to Tuesday, R is related to Sunday, in Seven people viz.Siraj, Asif, Junaid, Mohit, Deepak,
the same way K is related to which of the following? Roshan and Abhay, voluntarily donated some amount to a
a) Monday charity which works for the education of orphan kids.
b) Wednesday These seven people also chose to teach the kids for a few
c) Thursday hours, they taught on different days of the week from
d) Friday Monday to Sunday. The minimum amount that can be
e) None of these donated is Rs. 1000 and also no two people donated the
36.How many persons attend the seminar after K? same amount.
a) One The person who taught on Thursday donated Rs. 1800
b) Two more than Junaid. The average donation made by the
c) Three people who taught the kids on Monday and Saturday is Rs.
d) Four 2000, also the difference in their donation is Rs. 1000. The

Page 455 of 1334


Subscribe The Xpress Video Course & Mock Test Package for Bank & Insurance Exams
If there are any suggestions/ errors in our PDFs Feel Free to contact us via this email: admin@exampundit.in
Ultra Practice Bundle PDF
SBI Clerk/ RBI Assist. Mains – Reasoning
difference between the donation of Deepak and Abhay is b) Rs. 6000
Rs. 1600, where Abhay is not the highest donor. Asif c) Rs. 5400
taught three days after Roshan and his donation is four d) Rs. 4200
times as that of Roshan. Deepak taught the students three e) None of the above
days after Abhay but two days before the person who Directions (43-47): Study the following information
donated Rs. 4200. Junaid taught four days after Siraj and carefully and answer the questions.
his donation is double as that of Siraj. The donation made Eight gift boxes B1, B2, B3, B4, B5, B6, B7 and B8 are
by Deepak is double as that of Mohit. purchased from eight different E-commerce websites viz.
39.Who taught the kids a day after Mohit did? Alibaba, Jabong, Quikr, Flipkart, Amazon, Snapdeal,
a) Junaid Ebay and Myntra which are delivered to Eight different
b) Deepak places viz. Delhi, Mumbai, Hyderabad, Lucknow,
c) Asif Kanpur, Indore, Ahmedabad and Ludhiana on six
d) Siraj different days starting from Monday to Saturday. All the
e) Roshan above information is not necessarily in the same order.
40.What is the average donation of the people who Note:At least one and not more than two gift boxes are
taught the kids on Monday and Sunday? delivered on a same day. Not more than one Box is bought
a) Rs. 3000 fromthe same E-commerce website and not more than one
b) Rs. 2550 box is delivered to the same place.
c) Rs. 1850 Three boxes are delivered between B2 and B1. B5 is not
d) Rs. 2850 bought from Jabong. B3 is not delivered to Mumbai. B4
e) Rs. 2300 is delivered to Hyderabad on Tuesday. The box which is
41.Who donated the least amount? delivered to Ludhiana is bought immediately before the
a) Mohit box which is bought from Quikr. The number of boxes
b) Siraj delivered after B5 is as same as the number of boxes
c) Roshan delivered before B8. Only one box is delivered on the last
d) Asif day. B6 is not bought from Alibaba. The box which is
e) None of the above bought from Snapdeal is delivered to Lucknow. B2 is
42.What is the highest amount donated to the charity? delivered on one of the days after B3. B1 is delivered to
a) Rs. 4000 Delhi. The box which is delivered to Kanpur is delivered

Page 456 of 1334


Subscribe The Xpress Video Course & Mock Test Package for Bank & Insurance Exams
If there are any suggestions/ errors in our PDFs Feel Free to contact us via this email: admin@exampundit.in
Ultra Practice Bundle PDF
SBI Clerk/ RBI Assist. Mains – Reasoning
immediately before the day of the box which is delivered b) B5, B2, B3
to Indore. B5 is delivered immediately before or c) B1, B6, B8
immediately after the day on which B4 is delivered. B1 is d) B2, B4, B8
delivered on the first day. The boxes which are bought e) B4, B7, B8
from Flipkart and Jabong are delivered on a same day. 46.To which of the following place is box B3 delivered?
One of the boxes delivered on Wednesday is bought from a) Lucknow
Alibaba. The box which is bought from Alibaba is b) Kanpur
delivered in a day immediately after the day which is c) Mumbai
bought from Amazon but immediately before the day d) Indore
which is bought from Quikr. B3 and B6 are delivered on e) None of these
a same day. B7 is bought from Ebay. Only one box is 47.To which of the following place is Box B5 delivered?
delivered between B1 and B3. a) Lucknow
43.From which of the following websites, the gift boxes b) Kanpur
B6 and B8 were bought respectively? c) Mumbai
a) Myntra, Snapdeal d) Indore
b) Amazon, Quikr e) None of these
c) Ebay, Alibaba (Directions 48–50): Study the following information
d) Myntra, Quikr carefully and answer the questions given below it.
e) Flipkart, Alibaba Seven employees of a company namely Akhil, Apurva,
44.Which of the following statement is incorrect? Karan, Mayank, Pratik, Sahil and Saket attends seminar
a) B5 box is delivered on Thursday on seven consecutive daysstarting from Friday, but not
b) The box which is bought from Alibaba is delivered necessary in same order. Each employee attends seminar
to Ahmedabad on only one day. Pratik attends seminar on Wednesday.
c) B8 box is bought from Snapdeal Only two persons attend seminar between Mayank and
d) Both 1) and 3) Pratik. Sahil attends seminar three days before Saket.
e) B6 box is delivered on Wednesday Sahil does not attend seminar on Saturday. Saket does not
45.Which of the following boxes are delivered on attend seminar on the last day. Kanan attends seminar
Tuesday, Friday and Saturday respectively? after Akhil. Kanan does not attend seminar onthe last day.
a) B4, B6, B8 48.Whendoes Kanan attend seminar?

Page 457 of 1334


Subscribe The Xpress Video Course & Mock Test Package for Bank & Insurance Exams
If there are any suggestions/ errors in our PDFs Feel Free to contact us via this email: admin@exampundit.in
Ultra Practice Bundle PDF
SBI Clerk/ RBI Assist. Mains – Reasoning
a) Thursday c) Mayank
b) Tuesday d) Akhil
c) Saturday e) None of these
d) Friday 50.How many persons are attending seminar between
e) None of these Saket and Apurva?
49.Whoamong the following attends seminar on a) 3
Thursday? b) 1
a) Kanan c) 4
b) Apurva d) 2
e) None of these

Puzzle Days - Answer and Explanation


Direction (1-3):
1. Answer: b
2. Answer: c
3. Answer: d
Explanation:

Three persons attend the seminar between K and R. R


attends the seminar immediately after the one who works
in the company D. R does not attend the seminar on
Saturday. The one who works in the company D does not
attend the seminar on the first day. R attends the seminar
after K.
L attends the seminar between Tuesday and Friday. The
one who works in the company Y attends the seminar
immediately before L.

Page 458 of 1334


Subscribe The Xpress Video Course & Mock Test Package for Bank & Insurance Exams
If there are any suggestions/ errors in our PDFs Feel Free to contact us via this email: admin@exampundit.in
Ultra Practice Bundle PDF
SBI Clerk/ RBI Assist. Mains – Reasoning

➢ T attends the seminar immediately before the one


who works in the company Z.
➢ The one who works in the company D does not
attend the seminar on the first day. 4. Answer: e
➢ Neither L nor R works in the company Z. 5. Answer: c
➢ The one who works in the company N attends the 6. Answer: a
seminar immediately before T. 7. Answer: a
➢ Only one person attends the seminar between M 8. Answer: c
and the one who works in the company W. Explanation:
➢ M does not work in the company D. We have:
• The one who likes Grapes visits Italy on Thursday.
P visits Rome on Saturday and only two person
visits before the one who likes Mango, which
means the one who likes Mango visits on
Wednesday.
• Only one person visits between the one who likes
Mango and the one who visits London, that means
Direction(4-8):
we have two possibility for the one who visits
London, thus in case (1) the one who visits London
visits on Monday and in case (2) the one who visits
London visits on Friday.
Based on above given information we have:

Page 459 of 1334


Subscribe The Xpress Video Course & Mock Test Package for Bank & Insurance Exams
If there are any suggestions/ errors in our PDFs Feel Free to contact us via this email: admin@exampundit.in
Ultra Practice Bundle PDF
SBI Clerk/ RBI Assist. Mains – Reasoning
one who likes Orange, who neither visits on
Sunday nor visits New Delhi, and the one who
likes Apple, that means V must visit on
Wednesday and the one who likes Apple visits on
Sunday.
• The one who likes Guava visits on one of the day
before the one who likes Papaya, that means T
must like Guava.
Again, we have:
• Q, who neither likes Grapes nor visits Singapore,
• T, who neither visits London nor likes Mango,
visits on one of the day before R and R doesn’t like
visits on one of the day before Thursday and T
Apple that means R must visit on Thursday and Q
visits Just after the one who likes Banana, that
visits on Monday.
means T must visits one Tuesday.
Based on given information we have final arrangement as
• Only three person visits between the one who likes
follow:
Banana and U who visits New York, that means U
must visit on Friday, thus case (2) is not valid.
Based on above given information we have:

Directions (9-13):
Case (2) is not valid as U visits New York after three 9.Answer: b
day from one who likes Banana, thus we can’t place U 10.Answer: c
on Friday in case (2). 11.Answer: e
Again, we have: 12.Answer: c
• V visits Paris on one of the day before the one who 13.Answer: d
likes Apple and only one person visits between the Explanation:
Page 460 of 1334
Subscribe The Xpress Video Course & Mock Test Package for Bank & Insurance Exams
If there are any suggestions/ errors in our PDFs Feel Free to contact us via this email: admin@exampundit.in
Ultra Practice Bundle PDF
SBI Clerk/ RBI Assist. Mains – Reasoning
1) Only two people have bought the cars before S.
2) Four people have bought the cars between Q and the
person who has bought MG HECTOR.
3) Q has bought a car on one of the days before the person
who has bought MG HECTOR.
4) Q has bought a Silver car.
5) The number of cars bought before Tesla is as same as
the number of cars bought after Hyundai.
6) Hyundai is neither Red nor Silver in colour.
7) Tesla has been bought after Hyundai but neither
immediately nor on Monday.

8) S has bought a car either immediatelyor on one of the


days after V but immediatelybefore the person who has
bought Ford.
9) At least two people have bought cars after the person
who has bought Ford.
10) Only three people have bought cars between R and T.
11) T has not bought a car on the last day.

Page 461 of 1334


Subscribe The Xpress Video Course & Mock Test Package for Bank & Insurance Exams
If there are any suggestions/ errors in our PDFs Feel Free to contact us via this email: admin@exampundit.in
Ultra Practice Bundle PDF
SBI Clerk/ RBI Assist. Mains – Reasoning

12) U has bought a car immediately after W.


13) Neither P nor U has bought MG HECTOR.
Thus, all three cases get eliminated except case 1 (a)

Page 462 of 1334


Subscribe The Xpress Video Course & Mock Test Package for Bank & Insurance Exams
If there are any suggestions/ errors in our PDFs Feel Free to contact us via this email: admin@exampundit.in
Ultra Practice Bundle PDF
SBI Clerk/ RBI Assist. Mains – Reasoning
16) Only three people have bought cars between the
person who has bought Honda and the person who has
bought a yellow colouredcar.
17) Rolls Royce is neither Blue nor White in colour.
So, S has bought Honda and P has bought Rolls Royce.

14) P and S have bought neither Mercedes nor Tata.


15) Mercedes car is not Silver in colour.
So, R has bought Mercedes and Q has bought Tata.

18) Grey and Blackcoloured cars have not been bought


immediately before or after a Yellow coloured car.
19) The Blue colouredcar is bought after the Black
coloured carbut before the White coloured car.
20) Red colouredcar is bought immediately after
Greycoloured car but immediately before Black coloured
car.
21) Beige colouredcar is boughtthree days before
Mercedes was bought.
Final Arrangement

Page 463 of 1334


Subscribe The Xpress Video Course & Mock Test Package for Bank & Insurance Exams
If there are any suggestions/ errors in our PDFs Feel Free to contact us via this email: admin@exampundit.in
Ultra Practice Bundle PDF
SBI Clerk/ RBI Assist. Mains – Reasoning
Explanation:
->K attends the seminar on Wednesday.
->Only one person attends the lecture between K and the
one who is an Engineer.
->As many persons attend the lecture after the one who is
an Engineer is one more than as many persons attend the
lecture before R.

Direction (14-16):
14. Answer: c
->Three persons attend the lecture between T and the one
15. Answer: d
who is a teacher, who attends the lecture before T. T is not
16. Answer: d
an Engineer.
->Two persons attend the lecture between X and the one
who is an Actor.
->As many persons attend the lecture before S as after X.
G attends the lecture before X.
One case added by myself (table modified)

Case-1 [Eliminated] Case-2 [Eliminated] Case-2-A

No place S and X No place S and X

Day Persons Profession Day Persons Profession Day Persons Profession

Monday X/S Engineer Monday X/S Monday S

Page 464 of 1334


Subscribe The Xpress Video Course & Mock Test Package for Bank & Insurance Exams
If there are any suggestions/ errors in our PDFs Feel Free to contact us via this email: admin@exampundit.in
Ultra Practice Bundle PDF
SBI Clerk/ RBI Assist. Mains – Reasoning
Tuesday S Tuesday R Tuesday R Teacher

Wednesday K Teacher Wednesday K Teacher Wednesday K

Thursday S Thursday S Thursday G/ Actor

Friday S Friday S Engineer Friday G/ Engineer

Saturday R Saturday S Saturday T

Sunday T Sunday T Sunday X

->case 1 & 2 is eliminated because as many persons attend 20.Answer: D


the lecture before S as after X. 21.Answer: D
As many persons attend the lecture before G after the one
who is a lawyer.
->The one who is a doctor attends the lecture before M. X
is not an auditor.

The one who works in the company S attends the lecture


between Wednesday and Saturday. Three persons attend
the lecture between R and the one who works in the
company Q. R attends the seminar before the one who
works in the company S.

Directions (17-21):
17.Answer: C
18.Answer: E
19.Answer: B

Page 465 of 1334


Subscribe The Xpress Video Course & Mock Test Package for Bank & Insurance Exams
If there are any suggestions/ errors in our PDFs Feel Free to contact us via this email: admin@exampundit.in
Ultra Practice Bundle PDF
SBI Clerk/ RBI Assist. Mains – Reasoning

Number of persons attend the lecture between T and the Case 2 will be dropped because G attends the lecture
one who works in the company Q is one more than number immediately before K.
of persons attend the lecture between F and the one who Case 5(b) will be dropped because three persons attend the
works in the company S. T attends the lecture immediately lecture between K and the one who works in company B.
before the one who works in the company H. T does not Case 1, 3 and 4 will be dropped because R does not work
attend the lecture either on Saturday or on Monday. T does in company B.
not work in the company Q. Two persons attend the K does not work in the company V. Neither M nor K
lecture between F and M. M doesn’t attendthe lecture on works in the company Z.
Monday.

Directions (22-25):
22.Answer: b
Three persons attend the lecture between K and the one 23.Answer: d
who works in the company B. G attends the lecture 24.Answer: c
immediately before K. R does not work in the company B. 25.Answer: e
26.Answer: a

Page 466 of 1334


Subscribe The Xpress Video Course & Mock Test Package for Bank & Insurance Exams
If there are any suggestions/ errors in our PDFs Feel Free to contact us via this email: admin@exampundit.in
Ultra Practice Bundle PDF
SBI Clerk/ RBI Assist. Mains – Reasoning

Three seminars are taken between A and F’s seminar. C is


from Canada and he takes seminar immediately before F,
but not on Monday.

Explanation:
D takes seminar on 20th May. There is a gap of at least 2
days between D’s seminar and the person from Brazil. The
person from Brazil takes seminar immediately before the
person from Greece.

Since C does not take seminar on Monday, Case 1 and


Case 2a gets cancelled.
B does not belong to France but both of them takes
seminar on Tuesday. Neither H nor G is from France. G
does not take seminar before H. Neither the person from
China nor the one from Egypt takes seminar before G.

The number of seminars taken after the person from Italy


is more than the number of seminars taken between the
persons from Greece and Italy. A takes seminar
immediately before the person from Italy but he is not
from Greece.

Page 467 of 1334


Subscribe The Xpress Video Course & Mock Test Package for Bank & Insurance Exams
If there are any suggestions/ errors in our PDFs Feel Free to contact us via this email: admin@exampundit.in
Ultra Practice Bundle PDF
SBI Clerk/ RBI Assist. Mains – Reasoning
Direction (27-28):
27. Answer: a
28. Answer: d
Explanation:

Direction(29-33):
29. Answer: e
30. Answer: a
31. Answer: d
->Two lectures were given between the one who taught 32. Answer: e
English and the one who taught chemistry. 33. Answer: d
->As many persons give lecture before the one who taught Seven persons attends the class in a week starting from
chemistry as after the one who taught English. Monday and ends on Sunday and they bought different
In both case 1 and 2 off day is between Tuesday and items T-shirt, Shoes, Top, Jeans, Bag, TV and
Saturday. Refrigerator, but not necessarily in the same order.
Day Persons Items

Monday

Tuesday

Wednesday

->The one who taught Zoology gives the lecture Thursday


immediately after the one who taught Botany.
Friday
->Only one day was there between the one who taught
physics and the one who taught Botany. Saturday
-> One of the person taught Mathematics. sAt least two
Sunday
lectures were given after the one who taught Zoology.

Page 468 of 1334


Subscribe The Xpress Video Course & Mock Test Package for Bank & Insurance Exams
If there are any suggestions/ errors in our PDFs Feel Free to contact us via this email: admin@exampundit.in
Ultra Practice Bundle PDF
SBI Clerk/ RBI Assist. Mains – Reasoning
Only two persons attend the class after the one who Wednes Wednes R
bought Jeans. day day
Day Persons Items
Thursda Thursda
Monday y y

Tuesday Friday Jeans Friday Jeans

Wednesday Saturda Refriger Saturda


y ator y
Thursday
Sunday Sunday Refriger
Friday Jeans
ator

Saturday

Sunday Only one person attends the class between N and the one
who bought Top.
The one who bought T-shirt attends the class immediately
Three persons attend the class between R and the one who
before the one who bought Top.
bought Refrigerator.
Case-1 Case-2
R attends the class before the one who bought
Refrigerator. Day Perso Items Day Perso Items

The one who bought Bag attends the class immediately ns ns

before R.
Monday N/ Bag Monday
Case-1 Case-2
Tuesday R T-Shirt Tuesday N Bag
Day Perso Items Day Perso Items
ns ns Wednes Top Wednes R T-Shirt
day day
Monday Bag Monday
Thursda Thursda Top
Tuesday R Tuesday Bag
y y

Page 469 of 1334


Subscribe The Xpress Video Course & Mock Test Package for Bank & Insurance Exams
If there are any suggestions/ errors in our PDFs Feel Free to contact us via this email: admin@exampundit.in
Ultra Practice Bundle PDF
SBI Clerk/ RBI Assist. Mains – Reasoning
Friday N/ Jeans Friday Jeans Day Perso Items Day Perso Items
ns ns
Saturda Refriger Saturda
y ator y Monday N Bag Monday

Sunday Sunday Refriger Tuesday R T-Shirt Tuesday N Bag


ator
Wednes Top Wednes R T-Shirt
day day
Case-1-A Case-2-A
Thursda Thursda Top
Day Perso Items Day Perso Items y y
ns ns
Friday Jeans Friday Jeans
Monday Bag Monday
Saturda Refriger Saturda W
Tuesday R Tuesday Bag y ator y

Wednes T-Shirt Wednes R T-Shirt Sunday W Sunday Refriger


day day ator

Thursda Top Thursda Top


y y Case-1-A [Eliminated] Case-2-A

Friday Jeans Friday Jeans No place for W

Saturda N Refriger Saturda N Day Perso Items Day Perso Items


y ator y ns ns

Sunday Sunday Refriger Monday Bag Monday


ator
Tuesday R Tuesday W Bag
As many persons attend the class before W asafter N.
Wednes T-Shirt Wednes R T-Shirt
W does not boughtTop.
day day
Case-1 Case-2

Page 470 of 1334


Subscribe The Xpress Video Course & Mock Test Package for Bank & Insurance Exams
If there are any suggestions/ errors in our PDFs Feel Free to contact us via this email: admin@exampundit.in
Ultra Practice Bundle PDF
SBI Clerk/ RBI Assist. Mains – Reasoning
Thursda Top Thursda Top Sunday W Sunday G Refriger
y y ator

Friday Jeans Friday Jeans


Case-2-A [Eliminated]
Saturda N Refriger Saturda N
y ator y 2nd statement violates

Sunday Sunday Refriger Day Persons Items


ator
Monday

Tuesday W Bag
G attends the class immediately after the one who bought
TV. Wednesday R T-Shirt
N and G does not attend the class consecutively.
Thursday Top
Case-1 Case-2
Friday Jeans
Day Perso Items Day Perso Items
ns ns Saturday N TV

Monday N Bag Monday Sunday G Refrigerator

Tuesday R T-Shirt Tuesday N Bag K attends the class immediately after L.


W does not boughtShoes.
Wednes Top Wednes R T-Shirt
P is one of the persons.
day day
Case-1 [Eliminated] Case-2
Thursda TV Thursda Top
2nd statement violates
y y
Day Perso Items Day Perso Items
Friday G Jeans Friday Jeans
ns ns
Saturda Refriger Saturda W TV
Monday N Bag Monday P Shoes
y ator y

Page 471 of 1334


Subscribe The Xpress Video Course & Mock Test Package for Bank & Insurance Exams
If there are any suggestions/ errors in our PDFs Feel Free to contact us via this email: admin@exampundit.in
Ultra Practice Bundle PDF
SBI Clerk/ RBI Assist. Mains – Reasoning
Tuesday R T-Shirt Tuesday N Bag Sunday. One person attends seminar on one day only.
Each person was born in different year from 1992 to 1997.
Wednes Top Wednes R T-Shirt
Each one of them works in different company. There is a
day day
holiday on one of the days.
Thursda TV Thursda L Top Day Persons Company Year
y y
Monday
Friday G Jeans Friday K Jeans
Tuesday
Saturda Refriger Saturda W TV
Wednesday
y ator y

Thursday
Sunday W Shoes Sunday G Refriger
ator Friday

Saturday

Sunday

One who attends seminar on Sunday works in Infosys.


Three persons attend the seminar between T and the one
who works in Infosys.
Holiday is not on Sunday
Direction (34-38): Case-1 (off day is Monday or Tuesday)
34.Answer: a
Day Persons Company Year
35.Answer: d
36.Answer: c Monday
37.Answer: e
Tuesday
38.Answer: c
Explanation: Wednesday T
Six persons namely – G, Q, T, R, M and K attends seminar
on different days of a week starting from Monday to
Page 472 of 1334
Subscribe The Xpress Video Course & Mock Test Package for Bank & Insurance Exams
If there are any suggestions/ errors in our PDFs Feel Free to contact us via this email: admin@exampundit.in
Ultra Practice Bundle PDF
SBI Clerk/ RBI Assist. Mains – Reasoning
Thursday Day Pers Comp Ye Day Pers Comp Ye
ons any ar ons any ar
Friday
Monda OFF DAY Monda Q
Saturday
y y
Sunday Infosys
Tuesda Q Tuesda OFF DAY
y y

Case-2: If off day is there b/w T and Infosys


Wedne T Wedne T
Case-2 (off day b/w T and Infosys)
sday sday

Day Persons Company Year


Thursd Thursd

Monday ay ay

Tuesday T Friday Friday

Wednesday Saturd Saturd


ay ay
Thursday
Sunday G Infos Sunday G Infos
Friday
ys ys

Saturday

Sunday Infosys My take on case-2: If G on Saturday, we can’t make off


day b/w T and Infosys. Thus G must on Sunday
Case-2 (off day b/w T and Infosys)
More than three persons attend the seminar between G and
Q. Day Persons Company Year

G attends the seminar after Q.


Monday Q
Case-1 (off day is Monday or Case-1 (off day is Monday or
Tuesday) Tuesday) Tuesday T

Wednesday

Page 473 of 1334


Subscribe The Xpress Video Course & Mock Test Package for Bank & Insurance Exams
If there are any suggestions/ errors in our PDFs Feel Free to contact us via this email: admin@exampundit.in
Ultra Practice Bundle PDF
SBI Clerk/ RBI Assist. Mains – Reasoning
Thursday Saturd Saturd
ay ay
Friday
Sunda G Infosy Sunda G Infosy
Saturday
y s y s
Sunday G Infosys

Case-2 (off day b/w T and Infosys)


Q does not work in Accenture.
Day Persons Company Year
The one who works in Accenture attends the seminar
before Wednesday. Monday Q Accenture
Case-1 (off day is Monday or Case-1 (off day is Monday or
Tuesday T Accenture
Tuesday) Tuesday)
Wednesday
Eliminated 1st & 2nd Eliminated 1st & 2nd
statement not possible statement not possible Thursday

Day Pers Comp Ye Day Pers Comp Ye Friday


ons any ar ons any ar
Saturday
Monda OFF DAY Monda Q Accen
Sunday G Infosys
y y ture

Tuesda Q Accen Tuesda OFF DAY


Two persons attend the seminar between G and the one
y ture y
who was born in 1993.
Wedne T Wedne T The one who works in Wipro attends the seminar
sday sday immediately after the one who was born in 1993
Case-2 (off day b/w T and Case-2-A (off day b/w T and
Thursd Thursd
Infosys) Infosys)
ay ay

2nd statement not possible,


Friday Friday
Eliminated

Page 474 of 1334


Subscribe The Xpress Video Course & Mock Test Package for Bank & Insurance Exams
If there are any suggestions/ errors in our PDFs Feel Free to contact us via this email: admin@exampundit.in
Ultra Practice Bundle PDF
SBI Clerk/ RBI Assist. Mains – Reasoning
Day Pers Comp Ye Day Pers Comp Ye Wedn 19 Wedn 19
ons any ar ons any ar esday 93 esday 93

Mond Q Mond Q Thursd Wipro Thursd Wipro


ay ay ay ay

Tuesd T Accen Tuesda T Accen Friday OFF DAY Friday


ay ture y ture
Saturd Saturd OFF DAY
Wedn OFF DAY Wedn 19 ay ay
esday esday 93
Sunda G Infosy Sunda G Infosy
Thursd 19 Thursd OFF DAY y s y s
ay 93 ay

Friday Wipro Friday T attends the seminar before the one who works in CTS.
The one who works in CTS was born in 1994.
Saturd Saturd
T does not born in 1994
ay ay
Case-2 (off day b/w T and
Sunda G Infosy Sunda G Infosy Infosys)
y s y s
Day Pers Comp Ye
ons any ar
Case-2-B (off day b/w T and Case-2-C (off day b/w T and
Mond Q
Infosys) Infosys)
ay
Day Pers Comp Ye Day Pers Comp Ye
Tuesd T Accen
ons any ar ons any ar
ay ture
Mond Q Mond Q
Wedn OFF DAY
ay ay
esday
Tuesd T Accen Tuesda T Accen
ay ture y ture

Page 475 of 1334


Subscribe The Xpress Video Course & Mock Test Package for Bank & Insurance Exams
If there are any suggestions/ errors in our PDFs Feel Free to contact us via this email: admin@exampundit.in
Ultra Practice Bundle PDF
SBI Clerk/ RBI Assist. Mains – Reasoning
Thursd 19 Sunda G Infosy Sunda G Infosy
ay 93 y s y s

Friday Wipro
The one who works in HCL attends the seminar
Saturd CTS 19
immediately before M.
ay 94
Two persons attend the seminar between the one who
Sunda G Infosy works in HCL and the one who was born in 1996.
y s One of the companies is TCS.
Case-2 (off day b/w T and
Case-2-B (off day b/w T and Case-2-C (off day b/w T and
Infosys)
Infosys) Infosys)

Day Pers Comp Ye


Day Pers Comp Ye Day Pers Comp Ye
ons any ar
ons any ar ons any ar

Mond Q TCS
Mond Q Mond Q
ay
ay ay

Tuesd T Accen
Tuesd T Accen Tuesda T Accen
ay ture
ay ture y ture

Wedn OFF DAY


Wedn 19 Wedn 19
esday
esday 93 esday 93

Thursd HCL 19
Thursd Wipro Thursd Wipro
ay 93
ay ay

Friday M Wipro
Friday OFF DAY Friday CTS 19
94 Saturd CTS 19
ay 94
Saturd CTS 19 Saturd OFF DAY
ay 94 ay

Page 476 of 1334


Subscribe The Xpress Video Course & Mock Test Package for Bank & Insurance Exams
If there are any suggestions/ errors in our PDFs Feel Free to contact us via this email: admin@exampundit.in
Ultra Practice Bundle PDF
SBI Clerk/ RBI Assist. Mains – Reasoning
Sunda G Infosy 19 Day Persons Company Year
y s 96
Monday Q TCS
Case-2-B (off day b/w T and Case-2-C (off day b/w T and
Tuesday T Accenture
Infosys) Infosys)
Wednesday OFF DAY
Day Pers Comp Ye Day Pers Comp Ye
ons any ar ons any ar Thursday K HCL 1993

Mond Q TCS Mond Q Friday M Wipro


ay ay
Saturday R CTS 1994
Tuesd T Accen Tuesda T Accen
Sunday G Infosys 1996
ay ture y ture

Wedn HCL 19 Wedn HCL 19


Case-2-B (off day b/w T and Case-2-C (off day b/w T and
esday 93 esday 93
Infosys) Infosys)
Thursd M Wipro Thursd M Wipro
2nd statement violates,
ay ay
Eliminated
Friday OFF DAY Friday CTS 19
Day Pers Comp Ye Day Pers Comp Ye
94
ons any ar ons any ar
Saturd CTS 19 Saturd OFF DAY
Mond Q TCS Mond Q
ay 94 ay
ay ay
Sunda G Infosy 19 Sunda G Infosy 19
Tuesd T Accen Tuesda T Accen
y s 96 y s 96
ay ture y ture

Wedn K HCL 19 Wedn K HCL 19


At least two persons attend the seminar after K.
esday 93 esday 93
R does not attend the seminar on Friday.
Case-2 (off day b/w T and Infosys)

Page 477 of 1334


Subscribe The Xpress Video Course & Mock Test Package for Bank & Insurance Exams
If there are any suggestions/ errors in our PDFs Feel Free to contact us via this email: admin@exampundit.in
Ultra Practice Bundle PDF
SBI Clerk/ RBI Assist. Mains – Reasoning
Thursd M Wipro Thursd M Wipro Saturday R CTS 1994
ay ay
Sunday G Infosys 1996
Friday OFF DAY Friday R CTS 19
94
Case-2 (off day b/w T and Infosys)
Saturd R CTS 19 Saturd OFF DAY
Day Persons Company Year
ay 94 ay
Monday Q TCS 1992
Sunda G Infosy 19 Sunda G Infosy 19
y s 96 y s 96 Tuesday T Accenture 1995

Wednesday OFF DAY


Q was born in an even numbered year.
Thursday K HCL 1993
The one who works in Wipro was born in an odd
numbered year. Friday M Wipro 1997
M is younger than T.
Saturday R CTS 1994
The one who was born in 1997 does not attend the seminar
immediately before the holiday. Sunday G Infosys 1996
Case-2-B (off day b/w T and Infosys)

4th statement violates Eliminated

Day Persons Company Year

Monday Q TCS 1992

Tuesday T Accenture 1995

Wednesday K HCL 1993


Direction(39-42):
Thursday M Wipro 1997 39.Answer: a
40.Answer: d
Friday OFF DAY
41.Answer: c
42.Answer: b
Page 478 of 1334
Subscribe The Xpress Video Course & Mock Test Package for Bank & Insurance Exams
If there are any suggestions/ errors in our PDFs Feel Free to contact us via this email: admin@exampundit.in
Ultra Practice Bundle PDF
SBI Clerk/ RBI Assist. Mains – Reasoning
Explanation:
1) Deepak taught the students three days after Abhay but
two days before the person who donated Rs. 4200.
(Implies, Abhay taught the class five days before the
person who made the donation of Rs. 4200. Hence, there
are only two possibilities, i.e. either Abhay taught the
students on Monday or Tuesday and we can accordingly
map the person who donated Rs. 4200.)
3) Junaid taught four days after Siraj and his donation is
2) The average donation made by the people who taught
double as that of Siraj.
the kids on Monday and Saturday is Rs. 2000, also the
(There is only one possibility for this to happen, i.e. if
difference in their donation is Rs. 1000.
Siraj taught the kids on Wednesday and Junaid taught on
(Implies, total donation made on Monday and Saturday is
Sunday.
Rs. 4000. From statement 1, let us assume that Abhay
Also, it implies that Junaid donated Rs. 4200 to the
taught the class on Monday, implies the person who
charityNGOwhich further implies Siraj donated Rs. 2100)
donated Rs. 4200 taught the class on Saturday. But
4) Asif taught three days after Roshan and his donation is
clearly, the person taught on Saturday must have donated
four times as that of Roshan.
less than Rs. 4000 (as a total of Mon and Sat is Rs. 4000).
(There is only one possibility for this to happen, i.e.
Hence, our assumption was wrong, thus Abhay taught on
Roshan taught on Monday and Asif taught on Thursday.
Tuesday, Deepak taught on Friday and the person who
Also, as only Mohit is left to be mapped, it is clear that he
donated Rs. 4200 taught on Sunday.
taught the class on Saturday.)
Also, people who taught on Monday and Saturday donated
Rs. 1000 less than the other,
Thus, X + X – 1000 = Rs. 4000
Implies, one person donated Rs. 2500 and other donated
Rs. 1500)

Page 479 of 1334


Subscribe The Xpress Video Course & Mock Test Package for Bank & Insurance Exams
If there are any suggestions/ errors in our PDFs Feel Free to contact us via this email: admin@exampundit.in
Ultra Practice Bundle PDF
SBI Clerk/ RBI Assist. Mains – Reasoning
5) The person who taught on Thursday donated Rs. 1800
more than Junaid.
(Implies, Asif donated Rs. 6000 which further implies
Roshan donated Rs. 1500 (Acc. to statement 4).
Also, Mohit who taught on Saturday donated Rs. 2500.
(Acc. to statement 2))
6) The donation made by Deepak is double as that of
Mohit. Directions (43-47):
(Implies, Deepak donated Rs. 5000) 43. Answer: a
44. Answer: a
45. Answer: e
46. Answer: e
47. Answer: c
Explanation:

7) The difference between the donation of Deepak and


Abhay is Rs. 1600,Abhay is not the highest donor.
(If Abhay had donated Rs. 1600 more than Deepak, then
he would become the highest donor, implies Abhay
donated Rs. 3400).
1) B1 is delivered on the first day.
2) Only one box is delivered on the last day.
3) Three boxes are delivered between B2 and B1.

Page 480 of 1334


Subscribe The Xpress Video Course & Mock Test Package for Bank & Insurance Exams
If there are any suggestions/ errors in our PDFs Feel Free to contact us via this email: admin@exampundit.in
Ultra Practice Bundle PDF
SBI Clerk/ RBI Assist. Mains – Reasoning

4) B2 is delivered on one of the days after B3. Only one


box is delivered between B1 and B3.
5) B4 is delivered to Hyderabad on Tuesday.

Page 481 of 1334


Subscribe The Xpress Video Course & Mock Test Package for Bank & Insurance Exams
If there are any suggestions/ errors in our PDFs Feel Free to contact us via this email: admin@exampundit.in
Ultra Practice Bundle PDF
SBI Clerk/ RBI Assist. Mains – Reasoning

6)B3 and B6 are delivered on a same day.


7) B5 is delivered immediately before or immediately
(Case 1(a) and 2(b) will be eliminated here.)
after the day on which B4 is delivered.
(Case 1(b) will be eliminated here.)

Page 482 of 1334


Subscribe The Xpress Video Course & Mock Test Package for Bank & Insurance Exams
If there are any suggestions/ errors in our PDFs Feel Free to contact us via this email: admin@exampundit.in
Ultra Practice Bundle PDF
SBI Clerk/ RBI Assist. Mains – Reasoning
8) The number of boxes delivered after B5 is as same as
the number of boxes delivered before B8.
(Only one day left for B7 i.e. Friday.)

13) The boxes which are bought from Flipkart and Jabong
are delivered on a same day.
14) One of the boxes delivered on Wednesday is bought
9) B5 is not bought from Jabong. B3 is not delivered to from Alibaba.
Mumbai. B6 is not bought from Alibaba. Case 2(b)

12) B1 is delivered to Delhi.

Page 483 of 1334


Subscribe The Xpress Video Course & Mock Test Package for Bank & Insurance Exams
If there are any suggestions/ errors in our PDFs Feel Free to contact us via this email: admin@exampundit.in
Ultra Practice Bundle PDF
SBI Clerk/ RBI Assist. Mains – Reasoning
15) The box which is bought from Alibaba is delivered in
a day immediately after the day which is bought from
Amazon but immediately before the day which is bought
from Quikr.
16) The box which is delivered to Ludhiana is bought
immediately before the box which is bought from Quikr.
17) B7 is bought from Ebay.

18) The box which is delivered to Kanpur is delivered


immediately before the day of the box which is delivered
to Indore.
(Two cases arise here.)

19) The box which is bought from Snapdeal is delivered


to Lucknow.
(Case 2(b)ii will be eliminated here.)

Page 484 of 1334


Subscribe The Xpress Video Course & Mock Test Package for Bank & Insurance Exams
If there are any suggestions/ errors in our PDFs Feel Free to contact us via this email: admin@exampundit.in
Ultra Practice Bundle PDF
SBI Clerk/ RBI Assist. Mains – Reasoning

48.Answer: b
49.Answer: b
50.Answer: d
Explanation:
1) Pratik attends seminar on Wednesday. Only two
Final Arrangement: persons attend seminar between Mayank and
(B5 and B3 will be delivered to Mumbai and Ahmedabad Pratik.
respectively; and only one place left for Myntra i.e. 2) Sahil attends seminar three days before Saket.
Wednesday.) Sahil does not attend seminar on Saturday. Saket
does not attend seminar on the last day.

3) Kanan attends seminar after Akhil. Kanan does not


attend seminar on the last day.
(Directions 48–50):
Page 485 of 1334
Subscribe The Xpress Video Course & Mock Test Package for Bank & Insurance Exams
If there are any suggestions/ errors in our PDFs Feel Free to contact us via this email: admin@exampundit.in
Ultra Practice Bundle PDF
SBI Clerk/ RBI Assist. Mains – Reasoning

Download Puzzles Practice Questions PDF


Get More Reasoning Practice Questions PDF
Puzzle Year
Directions (1-5): Study the following information one who was born on Wednesday is a prime and was born
carefully and answer the below questions three years before Sonu. At least three persons were born
Seven persons namely – Hari, Deep, Guru, Payal, Monty, between the one who Deep and the one who was born on
Sonu and Rinku were born in different years starting from Tuesday. Age of Hari is neither a prime number nor was
1988 to 1994. Each person was also born in different days born on Tuesday. Age of Deep is not an even number. The
from Monday to Sunday. No two personswas born on one who was born on Thursday was born in any year after
either same day or same year. All the calculations are done the one who was born on Saturday.
from base year 2019. 1) Which of the combination of Name and Day is true?
Age of the one who was born on Friday is a multiple of 5. a) Rinku – Sat
Two persons were born between the one who was born on b) Payal – Wed
Monday and Hari. The one who was born on Sunday was c) Hari – Sat
born in 1990. Guru was born in 1992 but was neither born d) Sonu – Mon
on Tuesday nor Monday. Payal was neither born on e) Deep – Thu
Saturday nor born on Thursday. Guru and Monty were 2) Who was born just after the one who was born in
born in the adjacent years. The one who was born on Tuesday?
Monday was born just after Rinku’s birth year. Age of the a) Hari

Page 486 of 1334


Subscribe The Xpress Video Course & Mock Test Package for Bank & Insurance Exams
If there are any suggestions/ errors in our PDFs Feel Free to contact us via this email: admin@exampundit.in
Ultra Practice Bundle PDF
SBI Clerk/ RBI Assist. Mains – Reasoning
b) Payal 1986, 1988 and 1992. Each person works in different
c) Deep company viz.- IG, NAL, DRDO, ISRO, BHEL, TCS, SBI
d) Guru and SAIL. All the information are not necessarily in the
e) None of these same order. (Take Base Year as 2019 for age calculation).
3) Which of the following statement is not true? Difference of the ages of the one who works in SAIL and
a) The one who was born on Saturday was born just after U is a perfect square. P neither works in NAL nor works
Sonu. in IG. The one who works in BHEL is elder to U but not
b) Payal was born two years before the one who was born as much as the one who works in SAIL and the one who
on Tuesday. works in TCS. Neither V nor R works in SAIL. Number
c) Deep was born on Wednesday. of persons born before the one who works in BHEL is
d) Monty was born on Thursday. same as the number of persons born after Q. At least three
e) More than one statement is not true. persons were born between P and the one who was born
4) How many person was born after the one who was in DRDO. Two persons were born after the one who
born on Thursday? works in DRDO. Age of the one who works in SAIL is
a) Two neither odd nor is eldest among all. The one who works in
b) One IG is neither eldest nor was born after the one who works
c) More than three in ISRO. Age of the one who works in TCS is a prime
d) Three number. Q neither works in TCS nor was born in even
e) None year. The one who works in SBI is elder to S but not as
5) What is the age of Deep? much as the one who works in IG. V is younger to the one
a) 27 who works in NAL and T but not as much as W and the
b) 31 one who works in ISRO.
c) 28 6) Who is eldest among all the below persons?
d) 29 A.T
e) None of these B.R
Direction (6-10): Study the following information C.Q
carefully and answer the questions given below it. D.P
Eight persons namely – P, Q, R, S, T, U, V and W were E.None of these
born in different years viz.- 1971, 1976, 1977, 1980, 1983,

Page 487 of 1334


Subscribe The Xpress Video Course & Mock Test Package for Bank & Insurance Exams
If there are any suggestions/ errors in our PDFs Feel Free to contact us via this email: admin@exampundit.in
Ultra Practice Bundle PDF
SBI Clerk/ RBI Assist. Mains – Reasoning
7) The one who was born in 1977 works in which of the Seven persons namely –A, B, C, D, E, F and G were born
following company? in different years with the base year is taken as 2018. None
A.BHEL of them was born before 1950 and not after 2004. Each
B.TCS person likes different fruits viz. Mango, Apple, Banana,
C.SAIL Orange, Grapes, Papaya and Cherry.
D.NAL Note: If it is mentioned that a person’s age is considered
E.None of these as the last two digits of another person’s birth year, then it
8)How many persons were younger to the one who will be at any sequence. For example, if the age of ‘P’ is
works in IG? considered as last two digits of birth year of ‘Q’ which is
A.1 1975; then the age of ‘P’ might be either 75 or 57 years.
B.4 Age of C is 46 years and likes Cherry. A’s age is equal to
C.2 the last two digits of birth year of G. The difference
D.3 between the age of C and E is 12 years. B’s age is equal
E.5 to the last two digits of birth year of E. F likes Orange.
9) Which of the following order of the persons is The difference betweenthe ages of G and B is 6 years. Age
correct as per their ages(in descending order)? of A is not less than 54 years. One who was born in 1978
A.SVQPTR likes Grapes. The difference betweenthe ages of one who
B.PTQVUW likes Orange and A is 28 years. One who likes Apple was
C.SUVQPR born after the year 1972. B likes Mango. F and D were not
D.RTVQWU born on adjacent years. Difference betweenthe ages of one
E.None of these who likes Banana and one who likes Mango is 16 years.
10) Which of the following combination is not true? 11) What is the birth year of the one who likes Papaya?
A.NAL – 1971 - R A.1954
B.SAIL – T – 1977 B.1984
C.W – DRDO - 1988 C.1982
D.Q – IG – 1983 D.1958
E.S – 1992 - ISRO E.None of these
(Directions 11-15): Study the following information 12) Which of the following combination of birth year
carefully and answer the questions given below it. and fruit is true for A?

Page 488 of 1334


Subscribe The Xpress Video Course & Mock Test Package for Bank & Insurance Exams
If there are any suggestions/ errors in our PDFs Feel Free to contact us via this email: admin@exampundit.in
Ultra Practice Bundle PDF
SBI Clerk/ RBI Assist. Mains – Reasoning
A.1954 – Papaya with respect to the base year 2018. None of the person was
B.1978 – Banana born before 1952 and none of them was born after 2004.
C.1982 – Orange Not more than one person born in the same year. Each
D.1954 – Banana person likes different pet viz. Cat, Rat, Dog, Parrot, Deer,
E.1982 – Papaya Duck, Pigeon and Horse. The one who likes animal was
13) What is the age of the one who likes Orange? born in an even year and the one who likes bird was born
A.32 in an odd year. All the information is not necessary to be
B.48 in the same order.
C.36 Note: If it is mentioned that a person’s age is considered
D.46 as the last two digits of another person’s birth year, then it
E.None of these will be at any sequence. For example, if the age of P is
14) which of the following is the sum of the ages of the considered as last two digits of birth year of Q which is
one who likes orange and the one who likes apple? 1975; then the age of P might be either 57 or 75.
A. 65 F was born in 1993 and likes Pigeon. The difference
B. 78 betweenthe ages of C and G is 21 years. The difference
C. 70 between the ages of D and F is 15 years. G neither likes
D. 62 Deer nor he likes Rat. The difference betweenthe ages of
E. 90 D and A, who likes Dog, is 14 years. C’s age is equal to
15)which of the following is the correct order of the last two digits of the birth year of A. The difference
persons in descending order according to their ages? betweenthe agesof the one who likes Parrot and the one
A) A>B>D>C>F>G>E who likes Deer is 23 years. B neither likes Cat nor he was
B) A>G>B>C>D>F>E born after 1980. C likes bird but not a Parrot. H neither
C) B>D>E>C>A>G>F likes Rat nor he likes Parrot. The difference between the
D) D>E>C>A>B>F>G ages of the one who likes Cat and G is 12 years. B’s age
E) C>D>E>F>A>B>G is equal to the last two digits of the birth year of the one
(Directions 16-20): Study the following information who likes Cat. E was born in even year but neither likes
carefully and answer the questions given below it. Deer or Rat.
Eight persons A, B, C, D, E, F, G and H are born in the 16) How many people were born between the one who
same month in different years. Their ages are calculated likes Rat and G?

Page 489 of 1334


Subscribe The Xpress Video Course & Mock Test Package for Bank & Insurance Exams
If there are any suggestions/ errors in our PDFs Feel Free to contact us via this email: admin@exampundit.in
Ultra Practice Bundle PDF
SBI Clerk/ RBI Assist. Mains – Reasoning
A.1 A. F,A
B.3 B. B,F
C.2 C. C,F
D.4 D. D,E
E.None E. A,B
17) Which of the following pet is liked by the one who Directions (21-25): Study the following information
was born in 1968? carefully and answer the below questions.
A.Horse Eight persons namely – A, B, C, D, E, F, G and H are born
B.Deer in different years with the base year is taken as 2019. None
C.Cat of them are born after 2006 or before 1940. They work in
D.Rat different companies viz. IBM, Amazon, Dell, Google,
E.Can’t be determined. HCL, Zomato, HCL and Yahoo. Each person also visits
18) Which of the following year is the birth year of C? different cities viz. Lucknow, Pune, Delhi, Chennai,
A.1956 Ranchi, Shimla, Jaipur and Patna. All the information is
B.1964 not necessarily in the same order. No two person’s age is
C.1978 same.
D.1989 Note: If it is mentioned that a person’s age is considered
E.None of these as the last two digits of another person’s birth year, then it
19) four of the following five are alike in a certain way will be at any sequence. For example, if the age of X is
and forms a group. find the one who doesn’t belongs considered as last two digits of birth year of Y which is
to that group. 1984; then the age of X might be either 48 or 84.
A. D B was born in 1964 and works in HCL. C’s age is equal to
B. G the last two digits of the birth year of the one who visits
C. F Shimla and works in IBM. The one who works in Google
D. A also visits Delhi but not born after 1990. The difference
E. H betweenthe ages of the one who works in HCL and the
20) Find the pair which contains eldest and youngest one who visits Patna is 28 years. Age of the one who
persons respectively among all of them(considering works in Dell is equal to the last two digits of the birth
eight persons). year of the one who works in IBM. G neither works in HP

Page 490 of 1334


Subscribe The Xpress Video Course & Mock Test Package for Bank & Insurance Exams
If there are any suggestions/ errors in our PDFs Feel Free to contact us via this email: admin@exampundit.in
Ultra Practice Bundle PDF
SBI Clerk/ RBI Assist. Mains – Reasoning
nor visits Jaipur. D’s age is equal to the last two digits of C.E – Jaipur – 1994 – Dell
the birth year of the one who visits Patna and works in D.G – 1970 – Delhi – HCL
Yahoo. The one who visits Ranchi neither works in IBM E.All the above given statements are not true.
nor Yahoo nor was born after 1975. A neither visits Patna 23) Who among the following works in Amazon?
nor works in Google. The difference betweenthe ages of A.A
the one who visits Chennai and H is 10 years. The B.F
difference betweenthe ages of the one who works in C.D
Yahoo and the one who visits Shimla is 14 years. The D.G
difference betweenthe ages of the one who works in 24) which of the following statements is/are true
Amazon and the one who visits Ranchi is 34 years. The regarding F.
one who works in Dell neither visits Shimla nor he was A. F was born before 1990.
born before 1950. The difference betweenthe ages of the B. F is the second youngest person among them.
one who visits Pune and A is 12 years. The difference C. F is working in yahoo.
betweenthe ages of the one who works in Dell and the one D. F visits neither patna nor chennai.
who works in Google is 24 years. F was born before the E. more than one statement is true.
one who works in Dell. The difference betweenthe ages of 25)who is the eldest person among all.
the one who visits Delhi and H is 28 years. The difference A. H
between the ages of the one who visits Lucknow and the B. A
one who works in Zomato is 48 years. C. F
21) What is the birth year of the one who visits D. B
Lucknow? E. G
A.1976 Direction (26-30): Study the following information
B.1990 carefully and answer the given questions.
C.1992 There are eight persons A, B, C, D, P, Q, R and S born in
D.1994 different years viz., 1956, 1962, 1970, 1977, 1985, 1990,
E.None of these 1995 and 2001 but not necessarily in the same order. They
22) Which of the following combination is true? are working in different cities namely Bangalore,
A.C – 1994 – IBM – Chennai Chennai, Delhi, Hyderabad, Kochi, Lucknow, Mumbai
B.F – Patna – 1992 – Amazon and Pune but not necessarily in the same order.

Page 491 of 1334


Subscribe The Xpress Video Course & Mock Test Package for Bank & Insurance Exams
If there are any suggestions/ errors in our PDFs Feel Free to contact us via this email: admin@exampundit.in
Ultra Practice Bundle PDF
SBI Clerk/ RBI Assist. Mains – Reasoning
Calculations are done with respect to the year 2018 and 27) What is the total age of both A and the one who is
assuming months and date to be the same. working in Delhi?
There are only two persons born between the one who is a) 118 years
working in Bangalore and A, who does not work in Pune. b) 97 years
The age of S is three times the age difference between the c) 89 years
one who is working in Mumbai and B. C is not elder than d) 74 years
the one, who is working in Delhi. There is only one person e) 51 years
born between the one who is working in Pune and Q, 28) Four of the following are alike in a certain way and
whose age is a prime number. Not more than two and less thus form a group. Which of the following one who
than one person born between R and the one who is does not belongs to the group?
working in Lucknow. P is born in one of the odd a) B
numbered year and do not work in Bangalore and Kochi. b) Q
A’s age is a multiple of four and does not born after 1980. c) P
As many persons born between D and the one who is d) S
working in Pune is same as the persons born between Q e) D
and the one who is working in Chennai. C is five years 29) Who among the following is working in Bangalore?
younger than B. P’s age is a prime number and born a) The one whose age is 48
immediately before the one who is working in Mumbai. b) D
Sum of the ages of P and Q is equal to the perfect square c) The one who born in 1985
of a number. The one who is working in Lucknow born in d) The one who born immediately after C
an even numbered year and the one who is working in e) None of these
Delhi born in an odd numbered year. 30) What is the age of the person who is working in
26) Who among the following person(s) born between Hyderabad?
B and the one who is working in Mumbai? a) 48 years
a) C b) 33 years
b) The one who born in 2001 c) 23 years
c) The one who is working in Hyderabad d) 56 years
d) The one whose age is 48 e) None of these
e) Both a) and c)

Page 492 of 1334


Subscribe The Xpress Video Course & Mock Test Package for Bank & Insurance Exams
If there are any suggestions/ errors in our PDFs Feel Free to contact us via this email: admin@exampundit.in
Ultra Practice Bundle PDF
SBI Clerk/ RBI Assist. Mains – Reasoning
Direction (31-35): Study the following information B.10
carefully and answer the questions given below: C.12
Eight persons were born in different years 1975, 1979, D.11
1982, 1986, 1991, 1995, 1997 and 2001. Ages were E.None of these
calculated as with the based year of 2019. Each one of 33) How many persons were born between R and P?
them are indifferent weight (in kg) 21, 38, 39, 40, 42, 48, A.No one
51 and 56 but not necessarily in the same order. B.2
D was born in an even numbered year. Two persons were C.3
born between C and P, who was born after C. As many D.1
persons born before P is same as the persons born after B, E.None of these
who was born immediately before the one whose weight 34) How many persons weight is more than A?
is 38kg. P’s weight is not 38kg. D’s weight is anodd A. No one
number. R’s weight is twice of B’s weight. Three persons B. 2
were born between Q and the one who is the heaviest. C. 1
Three persons were born between D and the one whose D. 3
weight is 48kg. R was born after Q. A was born before E. 4
R. The one who is second heaviest was born immediately 35) which among the following is the sum of the
before E. Difference between A and D’s weight is at least weights of C and the one who was born in 1986?
2 kg. A. 106
31) If C is related to 1982, P is related to 1995, in the B. 91
same way, E is related to which of the following? C. 90
A.1991 D. 72
B.1986 E. 93
C.1997 Directions (36-40): Study the following information
D.2001 carefully and answer the questions given below
E.None of these Eight persons S, L, M, O, P, Q, R and N were born in eight
32) What is the age difference between A and P (in different years 1972, 1981, 1992, 1999, 1995, 1986, 1976
years)? and 1969. Each one of them likes different fruit Apple,
A.8 Avocado, Mango, Jackfruit, Banana, Orange, Kiwi and

Page 493 of 1334


Subscribe The Xpress Video Course & Mock Test Package for Bank & Insurance Exams
If there are any suggestions/ errors in our PDFs Feel Free to contact us via this email: admin@exampundit.in
Ultra Practice Bundle PDF
SBI Clerk/ RBI Assist. Mains – Reasoning
Pineapple. All the above information is not necessarily in C.The one who likes Jackfruit was born immediately
the same order. before L
Note: All these persons were born in the same month on D.More than two persons were born between M and the
same date. Ages were calculated as of 2019. one who likes Kiwi
Q was neither youngest nor eldest. Three persons were E.Q was born immediately after (considering the given
born between Q and the one who likes Jackfruit. Age years only) the one who likes Banana
difference between S and L is 3 years. S is elder than L. 38) If N is related to Pineapple, P is related to Kiwi, in
Two persons were born between N and the one who likes the same way which of the following is related to
Banana. Only one person was born between L and the one Banana?
who likes Mango. Q’s age is even number. Three persons A.L
were born between R and the one who likes Apple. N was B.M
born immediately after(considering the given years only) C.R
the one who likes Apple. P does not like Banana. As many D.Q
persons were born before the one who likes Mango is E.None of these
same as many persons were born after P. Age difference 39) Four of the following five are alike in a certain way
between O and the one who likes Kiwi is 4 years. The one and forms a group. Find the one which doesn’t belongs
who likes Pineapple was born immediately to that group.
after(considering the given years only) M. O does not like A. L
Avocado. B. N
36) How many persons were born between O and the C. P
one who likes pineapple? D. R
A.None E. M
B.1 40) which of the following is the sum of the ages of the
C.2 one who was born immediately before(considering the
D.3 given years only) N and the one who likes banana?
E.None of these A. 75
37) Which of the following is true? B. 74
A.P was born after the one who likes orange C. 78
B.Age difference between Q and P is 5 years D. 76

Page 494 of 1334


Subscribe The Xpress Video Course & Mock Test Package for Bank & Insurance Exams
If there are any suggestions/ errors in our PDFs Feel Free to contact us via this email: admin@exampundit.in
Ultra Practice Bundle PDF
SBI Clerk/ RBI Assist. Mains – Reasoning
E. 72 Green nor Grey colour and she was not born after 1970.
Direction: (41-45) Study the following information There is a pair of two persons whose birth year and age
carefully and answer the below questions: are same.
Eight persons namely- Amit, Ajay, Arpit, Akash, Aruna, 41)What is the birth year of the one who likes the
Avni, Aishwarya and Anuja were born in different years Grey colour?
with the current base year as 2020. Each persons like a) 1954
different colours viz. Yellow Orange, Red, Green, Black, b) 1984
Blue, Grey and White. c) 1982
None of them was born before 1952 and not after 2006. d) 1972
Note: If it is mentioned that a person’s age is considered e) None of these
as the last two digits of another person’s birth year, then it 42) Which of the following combination of birth year
will be at any sequence. For example, if the age of X is and colour is true for Ajay?
considered as the last two digits of birth year of Y which a) 1954- Yellow
is 1985, then the age of X might be either 85 or 58 years. b) 1972- Black
(If mentioned, then only this condition applies.) c) 1982- Blue
There is a difference of 10 years between the one who was d) 1972- Orange
born in 1978 and the one who likes White. Age of Arpit is e) 1982- Grey
48 years and likes Red colour. Amit’s age is related to the 43) Which of the following person likes Black and has
last two digits of the birth year of Aishwarya. There is a birth year as 1984?
difference of 12 years between Arpit and Aruna. Ajay’s I) The person who is 36 years old.
age is obtained from the birth year of Aruna. Avni likes II) Aishwarya
Blue colour. There is a difference of 6 years between the III) Aruna
ages of Aishwarya and Ajay. Age of Amit is not less than a) Only II
56 years. One who was born in 1978 likes Green colour. b) Only I
The difference of the ages of the one who likes Blue c) Only III
colour and Amit is 28 years. One who likes Black colour d) Both I and III
was born after 1972. Ajay likes Orange colour. Difference e) Both II and III
of the ages of the one who likes Orange colour and the one 44) What was the age of Avni 13 years ago?
who likes Yellow colour is 18 years. Anuja neither likes a) 20 years old

Page 495 of 1334


Subscribe The Xpress Video Course & Mock Test Package for Bank & Insurance Exams
If there are any suggestions/ errors in our PDFs Feel Free to contact us via this email: admin@exampundit.in
Ultra Practice Bundle PDF
SBI Clerk/ RBI Assist. Mains – Reasoning
b) 22 years old Jim’s age is an odd number but not a prime number. Jeff
c) 29 years old is 59 years old. The age of the one who likes Lamborghini
d) 15 years old Veneno Roadster is obtained by the birth year of Jeff.
e) 25 years old There is a difference of 20 years between the age of
45) Combination of which of the following pairs has Bernard and the person who likes Lamborghini Veneno
the minimum age when summed up together? Roadster and Bernard’s age is an even number. Jim
a) Amit and Ajay doesn’t like McLaren P1 LM. The age of the one who
b) Arpit and the one who likes Orange colour. likes LykanHypersport is derived by the birth year of
c) The one who was born in year 1984 and the one who Bernard. Larry likes Aston Martin Valkyrie. One of the
likes Blue colour. person was born in 1973 and that person is not Mark,
d) Aishwarya and the one who likes White colour. Larry and Jim. The age of Bill and Warren is an even
e) Akash and the one who was born in year 1954. number. The one who likes Bugatti Divo is 6 years
Direction: (46-50) Study the following information younger than Amancio. Bill doesn’t
carefully and answer the below questions likesLykanHypersport. The age of Alice is obtained by
Top 9 richest persons in the world all are born in different the birth year of Jim. The sum of the ages of the one who
years. Their ages are calculated with respect to 2020. likes Bugatti Veyron by MansoryVivere and the one who
They all are fond of costliest cars in the world and those likes LykanHypersport is equal to the Larry’s present
cars are Rolls Royce Sweptail, Mercedes Benz Maybach, age. The one who likes Bugatti Veyron by
Bugatti Divo, KoenigseggCCXR Trevita, Lamborghini MansoryVivere was born in an even numbered year. The
Veneno Roadster, McLaren P1 LM, LykanHypersport, one who likes Mercedes Benz Maybach is elder than the
Bugatti Veyron by MansoryVivere and Aston Martin one who likes Rolls Royce Sweptail and these two
Valkyrie but not necessarily in the same order. persons are not Alice and Jim. The age of Jim is derived
None of them was born before 1936. by the birth year of Warren.
Note: If it is mentioned that a person’s age is considered 46) What is the age of the one who likes Bugatti
as the last two digits of another person’s birth year, then Veyron by MansoryVivere and the one who likes
it will be at any sequence. For example, if the age of X is Rolls Royce Sweptail respectively?
considered as the last two digits of birth year of Y which a) 36,48
is 1985, then the age of X might be either 85 or 58 years. b) 48,84
(If mentioned then only this condition applies.) c) 59,16

Page 496 of 1334


Subscribe The Xpress Video Course & Mock Test Package for Bank & Insurance Exams
If there are any suggestions/ errors in our PDFs Feel Free to contact us via this email: admin@exampundit.in
Ultra Practice Bundle PDF
SBI Clerk/ RBI Assist. Mains – Reasoning
d) None of these b) Only II and III
e) 36,47 c)All I, II and III
47) Which of the following statement is/are true? d) Only I and II
a) Bill is 59 years old. e) Only I and III
b) None of the given statement is true 49) What is the sum of the ages of the eldest and the
c) The one who was born in 1972 likes youngest person?
LykanHypersport. a) 100 years
d) Jim was born in the year 2005. b) 101 years
e) Alice was born in the year 1972. c) 102 years
48) Which of the following statement is/are true? d) 103 years
I) Three persons were born between Bill and the one e) 104 years
who likes Bugatti Divo. 50) What is the difference between the ages of the one
II) The number of persons born after the one who likes who likes Aston Martin Valkyrie and the one who was
Bugatti Veyron by MansoryVivere is one more than the born in the year of 1961?
number of persons born before the one who likes a) 25 years
Mercedes Benz Maybach. b) 36 years
III) The difference between the ages of Bernard and c) 16 years
Warren is same as the difference between the ages of d) 09 years
Amancio and Jeff. e) None of these
a) Only I

Page 497 of 1334


Subscribe The Xpress Video Course & Mock Test Package for Bank & Insurance Exams
If there are any suggestions/ errors in our PDFs Feel Free to contact us via this email: admin@exampundit.in
Ultra Practice Bundle PDF
SBI Clerk/ RBI Assist. Mains – Reasoning
Puzzle Year - Answer and Explanation
Directions (1-5):

Again, we have:
• Age of the one who was born on Wednesday is
prime and was born three years before Sonu.
That means, in case (1a) & case (1b) Sonu was born in
1991, case (2a) & case (2b) are not valid.
We have: • Two persons were born between the one who
• Guru was born in 1992 but was neither born on was born on Monday and Hari.
Tuesday nor on Monday. • The one who was born on Monday was born
• The one who was born on Sunday was born in 1990. just after Rinku’s birth year.
• Guru and Monty were born in adjacent years. • Age of Hari is neither prime number nor was
That means, in case (1) Monty was born in 1993, in born on Tuesday.
case (2) Monty was born in 1991. That means, in case (1a) & case (1b) Rinku was born in
• Age of the one who was born on Friday is multiple of 1990.
5. • At least three persons were born between Deep
Since, only such birth year is 1989 and 1994, thus in case and the one who was born on Tuesday.
(1a) & case (2a) the one who was born on Friday was born • Age of Deep is not an even number.
in 1994, in case (1b) & case (2b) the one who was born on Since, neither Hari nor Guru was born on Tuesday, thus in
Friday was born in 1989. case (1a)& case (1b) Monty was born on Tuesday and
Based on above given information we have: Deep was born in 1988.
Based on above given information we have:

Page 498 of 1334


Subscribe The Xpress Video Course & Mock Test Package for Bank & Insurance Exams
If there are any suggestions/ errors in our PDFs Feel Free to contact us via this email: admin@exampundit.in
Ultra Practice Bundle PDF
SBI Clerk/ RBI Assist. Mains – Reasoning
Case (1a) is not valid as the one who was born on
Thursday was born in any year after the one who was
born on Saturday.
1) Answer: D
Clearly, only combination D is correct.
Case (2a) & case (2b) Age of the one who was born on Hence, option D is correct choice.
Wednesday is prime and was born three years before 2) Answer: A
Sonu. Explanation:
Again, we have: Clearly, Hari was born just after the one who was born on
• Payal was neither born on Saturday nor was Tuesday.
born on Thursday. Hence, option A is correct choice.
• The one who was born on Thursday was born 3) Answer: E
in any year after the one who was born on Clearly, both statement B and D are not true.
Saturday. Hence, option E is correct choice.
That means, in case (1b) Hari was born on Thursday and 4) Answer: E
case (1a) is not valid. Clearly, no person was born after the one who was born
Based on above given information we have final on Thursday.
arrangement as follows: Hence, option E is correct choice.
5) Answer: B
Clearly, age of Deep is 31 years.
Hence, option B is correct choice.
Direction (6-10):
Explanation

Page 499 of 1334


Subscribe The Xpress Video Course & Mock Test Package for Bank & Insurance Exams
If there are any suggestions/ errors in our PDFs Feel Free to contact us via this email: admin@exampundit.in
Ultra Practice Bundle PDF
SBI Clerk/ RBI Assist. Mains – Reasoning

Again, we have:
• The one who works in BHEL is elder to U but not
as much as the one who works in SAIL and the one
who works in TCS.
SAIL, TCS > BHEL > U
• Age of the one who works in TCS is prime
We have:
number.
• Two persons were born after the one who works in
That means, the one who works in TCS was born in 1976.
DRDO.
• Number of persons born before the one who works
• At least three persons were born between P and the
in BHEL is same as number of persons born after
one who was born in DRDO.
Q.
• P neither works in NAL nor works in IG.
• Q neither works in TCS nor was born in even year.
That means, in case (1) P was born in 1976, in case (2) P
That means, in case (1a) & case (2a) Q was born in 1983,
was born in 1971.
case (1b) & case (2b) are not valid.
• Difference of the ages of the one who works in
Based on above given information we have:
SAIL and U is perfect square.
• Age of the one who works in SAIL is neither odd
nor is eldest among all.
Thus, only possible such combinations are (42, 33) & (36,
27).
That means, in case (1a) & case (2a) the one who works
in SAIL was born in 1977, in case (1b) & case (2b) the Case (1b) is not valid as number of persons born before
one who works in SAIL was born in 1983. the one who works in BHEL is same as number of
Based on above given information we have: persons born after Q, case (2b) is not valid as Q doesn’t
works in TCS.
Page 500 of 1334
Subscribe The Xpress Video Course & Mock Test Package for Bank & Insurance Exams
If there are any suggestions/ errors in our PDFs Feel Free to contact us via this email: admin@exampundit.in
Ultra Practice Bundle PDF
SBI Clerk/ RBI Assist. Mains – Reasoning
Again, we have: Clearly, R is eldest among all.
• The one who works in SBI is elder to S but not as Hence, option B is correct choice.
much as the one who works in IG. 7) Answer: C
IG > SBI > S Clearly, the one who works in SAIL was born in 1977.
• The one who works in IG is neither eldest nor was Hence, option C is correct choice
born after the one who works in ISRO. 8) Answer: D
That means, the one who works in IG was born in 1983 Clearly, three persons were born after the one works in IG.
and S is youngest among all. Hence, option D is correct choice.
• Neither V nor R works in SAIL. 9) Answer: E
• V is younger to the one who works in NAL and T Clearly, none of the order is correct.
but not as much as W and the one who works in Hence, option E is correct choice.
ISRO. 10) Answer: C
NAL, T > V > W, ISRO. Clearly, only combination C is not true.
Since, P doesn’t works in NAL, thus case (2a) is not valid. Hence, option C is correct choice.
That means, in case (1a) R works in NAL and is eldest (Directions 11-15):
among all.
Based on above given information we have final
arrangements as follows:

We have:
• Age of C is 46 years and likes Cherry, that means
C was born in 1972.
Case (2a) is not valid as P doesn’t works in NAL. • The difference between the ages of C and E is 12
6) Answer: B years, that means we have two possible years for
Page 501 of 1334
Subscribe The Xpress Video Course & Mock Test Package for Bank & Insurance Exams
If there are any suggestions/ errors in our PDFs Feel Free to contact us via this email: admin@exampundit.in
Ultra Practice Bundle PDF
SBI Clerk/ RBI Assist. Mains – Reasoning
E, in case (1) E was born in 1984, in case (2) E was • A’s age is equal to the last two digits of birth year
born in 1960. of G, that means in case (1a)& case (2b) possible
• B’s age is equal to the last two digits of birth year age of A is either 64 or 46 years, as C was born in
of E, that means in case (1) age of B must be either 1972 thus A must born in 1954, in case (1b)
48 or 84 years but 84 is not valid as no person was possible age of A is either 67 or 76 years, as no
born before 1950 thus as B was born in 1970, in person was born before 1950 thus A must born in
case (2) age of B must be either 06 or 60 years but 1951, in case (2a) possible age of A is either 52 or
06 is not valid as no person was born after 2004 25 years, as A was not born after 1964 (Age of A
thus as B was born in 1958 and B likes Mango. is not less than 54 years.), that means case (2a) is
Based on above given information we have: not valid.
Based on above given information we have:

Case (2a) is not valid as A was not born after 1965.


Again, we have:
• The difference between the ages of one who likes
Orange and A is 28 years and F likes Orange, that
means in case (1a) & (2b) F must born in 1982, in
case (1b) F must born in 1979.
• One who was born in 1978 likes Grapes, that
means D was born in 1978.
Again, we have:
• As F and D was not born on adjacent years, that
• The difference betweenthe ages of G and B is 6
means case (1b) is not valid.
years, that means we have four possible birth years
• Difference between the ages of one who likes
for G, in case (1a) G was born in 1964, in case (1b)
Banana and one who likes Mango is 16 years, that
G was born in 1976, in case (2a) G was born in
1952, in case (2b) G was born in 1964.
Page 502 of 1334
Subscribe The Xpress Video Course & Mock Test Package for Bank & Insurance Exams
If there are any suggestions/ errors in our PDFs Feel Free to contact us via this email: admin@exampundit.in
Ultra Practice Bundle PDF
SBI Clerk/ RBI Assist. Mains – Reasoning
means in case (1a) A must likes Banana and B was 15) Answer: B
born in 1970, case (2b) is not valid. Clearly, the correct order of persons in descending order
• One who likes Apple was born after 1972, that according to their ages is A>G>B>C>D>F>E
means E must likes Apple. Hence , option B is correct choice.
Based on above given information we have final
arrangement as follow: (Directions 16–20):

Case (1b) is not valid as F and D was not born on


adjacent years and case (2b) is not valid as Difference
of the ages of one who likes Banana and one who likes We have:
Mango is 16 years. • F was born in 1993 and likes Pigeon.
11) Answer: E • The difference between the ages of D and F is 15
Clearly, the one who likes Papaya was born in 1964. years, that means D must born in 1978, as no
Hence, option E is correct choice. person was born after 2004.
12) Answer: D • The difference between the ages of D and A, who
Clearly, A was born in 1954 and likes Banana. likes Dog, is 14 years, that means there are two
Hence, option D is correct choice. possible ages for A, i.e in case (1) A was born in
13) Answer: C 1964 and in case (2) A was born in 1992.
Clearly, age of the one who likes Orange is 36 years. • C’s age is equal to the last two digits of the birth
Hence, option C is correct choice. year of A, that means in case (1) possible age of C
14) Answer: C is either 46 or 64 years and in case (2) possible age
Clearly, the sum of the ages of the one who likes orange of C is either 29 or 92 years. As, C likes bird but
and the one who likes apple = 36+34=70 not a Parrot, that means birth year of C must be
Hence, option C is correct choice. odd, that means case (1) is not valid and as no

Page 503 of 1334


Subscribe The Xpress Video Course & Mock Test Package for Bank & Insurance Exams
If there are any suggestions/ errors in our PDFs Feel Free to contact us via this email: admin@exampundit.in
Ultra Practice Bundle PDF
SBI Clerk/ RBI Assist. Mains – Reasoning
person was born before 1955, that means in case • E was born in even year but neither likes Deer or
(2) age of C is 29 years. Rat, , that means E must likes Cat.
• As F likes Pigeon and C doesn’t like Parrot, that • B doesn’t like Cat nor he was born after 1980
means C must likes Duck. • H neither likes Rat nor he likes Parrot
Based on above given information we have: • As, the one who likes Parrot was born in an odd
year, that means B must like Parrot.B’s age is
equal to the last two digits of the birth year of the
one who likes Cat, that means in case (2a) possible
age of B is 80 or 08 , but both of these values are
not valid because none of the person was born
before 1952 and none of them was born after
2004.In case (2b) possible age of B is 56 or 65
year, as B likes Parrot that means B must born in
Case (1) is not valid as C likes bird which is not odd year, thus B was born in 1953.
possible from last two digits of the A’s birth year. • Based on above given information we have:
Again, we have:
• The difference between the ages of C and G is 21
years.
• G neither likes Deer nor he likes Rat, as no person
was born after 2004, that means G was born in
1968.
• The difference between the ages of the one who
likes Cat and G is 12 years, that means we have
Case (2a) is not valid as B’s age is equal to the last two
two possible birth years for the one who likes Cat,
digits of the birth year of the one who likes Cat
in case (2a) one who likes Cat was born in 1956,
Again, we have:
in case (2b) one who likes Cat was born in 1980.
• The difference in the ages of the one who likes
• As, G neither likes Deer nor likes Rat, thus only
Parrot and the one who likes Deer is 23 years, that
possible pet liked by G is Horse, birth year of G is
means the one who likes Deer was born in 1976.
even.

Page 504 of 1334


Subscribe The Xpress Video Course & Mock Test Package for Bank & Insurance Exams
If there are any suggestions/ errors in our PDFs Feel Free to contact us via this email: admin@exampundit.in
Ultra Practice Bundle PDF
SBI Clerk/ RBI Assist. Mains – Reasoning
Based on above given information we have final Directions (21-25):
arrangement as follow:

We have:
• No two person’s age is same
• B was born in 1964 and works in HCL.
16) Answer: A
• The difference between the ages of the one who
Clearly, one person was born between the one who likes
works in HCL and the one who visits Patna is 28
Rat and G.
years, as none of the person was born before 1940,
Hence, option A is correct choice.
that means, the one who visits Patna was born in
17) Answer: A
1992.
Clearly, the one who was born in 1968 likes Horse.
• D’s age is equal to the last two digits of the birth
Hence, option A is correct choice.
year of the one who visits Patna and works in
18) Answer: D
Yahoo, age of D might be either 92 years or 29
Clearly, C was born in 1989.
years, but none of the person was born before
Hence, option D is correct choice.
1940, that means D must born in 1990.
19) Answer: C
• The difference between the ages of the one who
Clearly, only F was born in odd year remaining all were
works in Yahoo and the one who visits Shimla is
born in even years.
14 years, that means we have two possible birth
Hence, option C is correct choice.
years for the one who visits Shimla, in case (1) the
20) Answer: B
one who visits Shimla was born in 1976, in case
Clearly, B and F were eldest and youngest persons
(2) the one who visits Shimla was born in 2004.
respectively among all .
Hence, option B is correct choice.
Page 505 of 1334
Subscribe The Xpress Video Course & Mock Test Package for Bank & Insurance Exams
If there are any suggestions/ errors in our PDFs Feel Free to contact us via this email: admin@exampundit.in
Ultra Practice Bundle PDF
SBI Clerk/ RBI Assist. Mains – Reasoning
• C’s age is equal to the last two digits of the birth works in Google was born in 1961, in case (1b) the
year of the one who visits Shimla and works in one who works in Google was born in 1970, in
IBM, that means we have three possible birth years case (1c) the one who works in Google was born
for C, in case (1a) C was born in 1943, in case (1b) in 1943
C was born in 1952, in case (2) C was born in • Based on above given information we have:
1979.
Based on above given information we have:

Again, we have:
• The difference between the ages of the one who
visits Delhi and H is 28 years.
Again, we have:
• The difference between the ages of the one who
• Age of the one who works in Dell is equal to the
visits Chennai and H is 10 years, that means in
last two digits of the birth year of the one who
case (1b) H was born in 1942 and C visits Chennai
works in IBM.
and case (1a), case (1c) are not valid.
• The one who works in Dell neither visits Shimla
• The one who visits Ranchi neither works in IBM
nor he was born before 1950, that means in case
nor Yahoo nor was born after 1975.
(1a) the one who works in Dell was born in 1985,
• The difference in the ages of the one who works in
in case (1b) the one who works in Dell was born
Amazon and the one who visits Ranchi is 34 years,
in 1994, in case (1c) the one who works in Dell
which means H must visits Ranchi and the one
was born in 1967, in case (2) the one who works
who was born in the year 1976 works in Amazon.
in Dell was born in 1940, as The one who works
Based on above given information we have:
in Dell neither visits Shimla nor he was born
before 1950 hence case 2 is not valid.
• The one who works in Google also visits Delhi but
not born after 1990.
• The difference between the ages of the one who
works in Dell and the one who works in Google is
24 years, that means in case (1a) the one who
Page 506 of 1334
Subscribe The Xpress Video Course & Mock Test Package for Bank & Insurance Exams
If there are any suggestions/ errors in our PDFs Feel Free to contact us via this email: admin@exampundit.in
Ultra Practice Bundle PDF
SBI Clerk/ RBI Assist. Mains – Reasoning
Case (1a), case (1c) are not valid as the difference of 22) Answer: C
the ages of the one who visits Chennai and H is 10 Clearly, “E – Jaipur – 1994 – Dell” is true.
years. Hence, option C is correct choice.
Again, we have: 23) Answer: A
• A neither visits Patna nor works in Google. Clearly, A works in Amazon
• The difference in the ages of the one who visits Hence, option A is correct choice.
Pune and A is 12 years, that means B visits Pune 24) Answer: B
and A visits Shimla. Clearly ,F is the second youngest person among them.
• The difference between the ages of the one who Hence, option B is the correct choice.
visits Lucknow and the one who works in Zomato 25)Answer: A
is 48 years, which means D visits Lucknow and H Clearly, H is the eldest person among all of them
works in Zomato. Hence, option A is the correct choice.
• G neither works in HP nor visits Jaipur, which Direction (26-30):
means G was born in 1970.
• F was born before the one who works in Dell,
which means F works in HP.
Based on above given information we have final
arrangement as follow:

• There is only one person born between the one


who is working in Pune and Q, whose age is
prime number.
21) Answer: B
Clearly, the one who visits Lucknow was born in 1990.
Hence, option B is correct choice.
Page 507 of 1334
Subscribe The Xpress Video Course & Mock Test Package for Bank & Insurance Exams
If there are any suggestions/ errors in our PDFs Feel Free to contact us via this email: admin@exampundit.in
Ultra Practice Bundle PDF
SBI Clerk/ RBI Assist. Mains – Reasoning

• There are only two persons born between the


one who is working in Bangalore and A, who
does not working in Pune. • C is five years younger than B (C=B-5). So, C’s
• A’s age is multiple of four and does not born age is definitely 28 and B’s age is 33.
after 1980. • The age of S is three times the age difference
• P’s age is prime number and born immediately between the one who is working in Mumbai
before the one who is working in Mumbai. and B. (S=3(Mumbai~B)). So, age of S is
• So, Case-3 will be dropped. definitely 48.
• Case-2 will be dropped.

• Sum of the ages of P and Q is equal to perfect


square of a number.
• So, Case-4 will be dropped.

• As many persons born between D and the one


who is working in Pune is same as the persons

Page 508 of 1334


Subscribe The Xpress Video Course & Mock Test Package for Bank & Insurance Exams
If there are any suggestions/ errors in our PDFs Feel Free to contact us via this email: admin@exampundit.in
Ultra Practice Bundle PDF
SBI Clerk/ RBI Assist. Mains – Reasoning
born between Q and the one who is working in Clearly, only S was born in even numbered year.
Chennai. C is not elder than the person who is Hence, option d is the correct choice.
working in Delhi. 29. Answer: c)
• Not more than two and less than one person Clearly, The one who born in 1985 is working in
born between R and the one who is working in bangalore.
Lucknow. P is born in one of the odd numbered Hence , option c is the correct choice.
years and do not work in Bangalore and Kochi. 30. Answer: c)
• The one who is working in Lucknow born in Clearly, the age of the person who is working in
even numbered year and the one who is Hyderabad is 23 years.
working in Delhi born in odd numbered year. Hence , option c is the correct choice.
• So the final arrangement is,
Direction (31-35):
Explanation

D was born in even numbered year. Three persons were


born between D and the one whose weight is 48kg.
26. Answer: e)
Clearly both a and c statements are correct
Hence ,option e is the correct choice.
27. Answer: b)
Clearly, the total age of both A and the one who is working
in Delhi 97 years.
Hence, option b is correct choice.
28. Answer: d)
Page 509 of 1334
Subscribe The Xpress Video Course & Mock Test Package for Bank & Insurance Exams
If there are any suggestions/ errors in our PDFs Feel Free to contact us via this email: admin@exampundit.in
Ultra Practice Bundle PDF
SBI Clerk/ RBI Assist. Mains – Reasoning
Two persons were born between C and P, who was born Hence , option D is the correct choice
after C. As many persons born before P was born after B, 33. Answer: C
who was born immediately before the one whose weight Clearly, three persons were born between R and P.
is 38kg. P’s weight is not 38kg. D’s weight is in odd Hence , option c is the correct choice
number. R’s weight is twice of B’s weight. 34. Answer: B
Clearly , Two persons weight is more than A
Hence , option B is the correct choice
35. Answer: B
Clearly, the sum of the weights of C and the one who was
born in 1986 is 91 years.
Three persons were born between Q and the one who is Hence , option B is the correct choice
the heaviest. R was born after Q. A was born before R.
The one who is second heaviest was born immediately Directions (36-40):
before E. Difference between A and D’s weight is at least
2 kg.

Case 1(b) will be dropped because the one who is second


heaviest was born immediately before E.
Q’s age is even number. Q was neither youngest nor
Case 2 will be dropped because Difference between A and
eldest. Three persons were born between Q and the one
D’s weight is at least 2 kg.
who likes Jackfruit.
31. Answer: B
Clearly, E is related to 1986.
Hence , option B is the correct choice
32. Answer: D
Clearly ,the age difference between A and P (in years) is
11 years.
Page 510 of 1334
Subscribe The Xpress Video Course & Mock Test Package for Bank & Insurance Exams
If there are any suggestions/ errors in our PDFs Feel Free to contact us via this email: admin@exampundit.in
Ultra Practice Bundle PDF
SBI Clerk/ RBI Assist. Mains – Reasoning
Difference between S and L is 3 years. S is elder than L. Case 1: A will be dropped because the one who likes
Only one person was born between L and the one who Pineapple was born immediately after M.
likes Mango. As many persons were born before the one
who likes Mango is same as many persons were born after 36. Answer: (c)
P. Clearly,two persons were born between O and the one
Case 1 who likes pineapple
Hence , option C is the correct choice
37. Answer: (e)
Clearly, Q was born immediately after (considering the
given years only) the one who likes Banana
Hence , option E is the correct choice
Case 1: B will be dropped because as many persons were 38. Answer: (b)
born before the one who likes Mango is same as many Clearly, M is related to Banana.
persons were born after P. Hence , option B is the correct choice
Three persons were born between R and the one who likes 39. Answer: (e)
Apple. N was born immediately after(considering the Clearly, only M was born in odd numbered year.
given years only) the one who likes Apple. P does not like Hence , option E is the correct choice
Banana. Two persons were born between N and the one 40. Answer: (b)
who likes Banana. Age difference between O and the one Clearly, the sum of the ages of the one who was born
who likes Kiwi is 4 years. The one who likes Pineapple immediately before(considering the given years only) N
was born immediately after M(considering the given years and the one who likes bananais 74 years.
only). O does not like Avocado. Hence , option B is the correct choice

Direction(41-45)
Detailed Explanation:
Step 1: Age of Arpit is 48 years and likes Red colour.
There is a difference of 12 years between Arpit and
Aruna. Avni likes Blue colour

Page 511 of 1334


Subscribe The Xpress Video Course & Mock Test Package for Bank & Insurance Exams
If there are any suggestions/ errors in our PDFs Feel Free to contact us via this email: admin@exampundit.in
Ultra Practice Bundle PDF
SBI Clerk/ RBI Assist. Mains – Reasoning
As per the data, age of Aruna can be 60/36 years (48+/- Ajay 60 1960
12 years). Arpit 48 1972 Red
When we take the base year as 2020, Akash
Birth year of Arpit (48 years old) - 2020-48 years=1972 Aruna 60 1960
birth year Avni Blue
Two possibilities for Birth year of Aruna Aishwarya 66/54 1954/1966
i) Birth year of Aruna (60 years old) - 2020-60 years= Anuja
1960 birth year
ii) Birth year of Aruna (36 years old) – 2020-36 years= In this case, we assume age of Aruna as 60 years, then by
1984 birth year birth year of Aruna, the age of Ajay become 60 or 6 years
Name Age Birth year Colour old but no one should born before 1952 and after 2006
Amit means the youngest and eldest age are 14 and 68 years
Ajay old respectively as per base year 2020.
Arpit 48 1972 Red So the age of Ajay become 60 years in this case.
Akash By following the same concept, we obtained the age and
Aruna 60/36 1960/1984 birth year of Aishwarya from the age of Ajay as per data.
Avni Blue Age of Aishwarya–(60 years+/- 6 years= 66/54)
Aishwarya Birth year of Aishwarya – (1960+/- 6 years= 1954/1966)
Anuja Case (2)
Name Age Birth year Colour
Step 2: Ajay’s age is obtained from the birth year of Amit
Aruna. There is a difference of 6 years between the ages Ajay 48 1972
of Aishwarya and Ajay. Arpit 48 1972 Red
Here, by following the conditions, we can make two Akash
cases. Aruna 36 1984
Avni Blue
Case (1) Aishwarya 54/42 1966/1978
Name Age Birth year Colour Anuja
Amit

Page 512 of 1334


Subscribe The Xpress Video Course & Mock Test Package for Bank & Insurance Exams
If there are any suggestions/ errors in our PDFs Feel Free to contact us via this email: admin@exampundit.in
Ultra Practice Bundle PDF
SBI Clerk/ RBI Assist. Mains – Reasoning
In this case, we assume the age of Aruna as 36 years and next possibility as birth year 1966, we get age of Amit as
further following the same above concept and method. 66 years by satisfying every condition.
Step 3:Amit’s age is related to the last two digits’ of the
birth year of Aishwarya. Age of Amit is not less than 56 Case (2)
years. Name Age Birth year Colour
In this case 1, by taking year 1954 as the birth year of Amit 66 1954
Aishwarya, we obtained the age of Amit as 45/54 years . Ajay 48 1972
But Amit’s age cannot be less than 56 years so this case Arpit 48 1972 Red
is not valid. Akash
Again In this case 1, by taking year 1966 as the birth year Aruna 36 1984
of Aishwarya, we obtained the Amit’s age as 66 years . Avni Blue
Now the birth year of Amit as per base year 2020, is Aishwarya 54 1966
1954. Anuja
Case (1)
Name Age Birth year Colour Now the birth year of Amit as per base year 2020, is
Amit 66 1954 1954.
Ajay 60 1960 Step 4: Ajay likes Orange colour. One who was born in
Arpit 48 1972 Red 1978 likes Green colour. Anuja neither likes Green nor
Akash Grey colour and she was not born after 1970.
Aruna 60 1960 In both the cases, By observing the table, Akash is the
Avni Blue only person who satisfy the given condition.
Aishwarya 54 1966 Age of Akash – 2020-1978= 42 years old
Anuja Case (1)
Name Age Birth year Colour
Now consider another case 2: Amit 66 1954
By taking the birth year of Aishwarya as 1978, the age of Ajay 60 1960 Orange
Amit become 78 or 87 years but the eldest age can be 68 Arpit 48 1972 Red
years old as per the data given. Now moving towards the Akash 42 1978 Green
Aruna 60 1960

Page 513 of 1334


Subscribe The Xpress Video Course & Mock Test Package for Bank & Insurance Exams
If there are any suggestions/ errors in our PDFs Feel Free to contact us via this email: admin@exampundit.in
Ultra Practice Bundle PDF
SBI Clerk/ RBI Assist. Mains – Reasoning
Avni Blue Amit 66 1954
Aishwarya 54 1966 Ajay 60 1960 Orange
Anuja Arpit 48 1972 Red
Akash 42 1978 Green
Case (2) Aruna 60 1960
Name Age Birth year Colour Avni Blue
Amit 66 1954 Aishwarya 54 1966
Ajay 48 1972 Orange Anuja 52 1968 White
Arpit 48 1972 Red
Akash 42 1978 Green Case (2)
Aruna 36 1984 Name Age Birth year Colour
Avni Blue Amit 66 1954
Aishwarya 54 1966 Ajay 48 1972 Orange
Anuja Arpit 48 1972 Red
Akash 42 1978 Green
Step 5: There is a difference of 10 years between the one Aruna 36 1984
who was born in 1978 and the one who likes White. Avni Blue
Anuja neither likes Green nor Grey colour and she was Aishwarya 54 1966
not born after 1970. Anuja 52 1968 White
In both the cases, Difference between Akash and the
person who likes White 10 years – (1978+/-10 = Step 6: One who likes Black colour was born after 1972.
1968/1988). It means White colour must has combination So, black colour must be liked by Aruna.
with the year 1968/1988. This condition is not satisfied with case 1 hence it is
This can be placed in the table only for Anuja but Anuja invalid. in case (2), Aruna was born in 1984.
must be born before 1970 so the birth year of Anuja must Case (1)
be 1968. Name Age Birth year Colour
Age of Anuja – 2020-1968= 52 years old Amit 66 1954
Case (1) Ajay 60 1960 Orange
Name Age Birth year Colour Arpit 48 1972 Red

Page 514 of 1334


Subscribe The Xpress Video Course & Mock Test Package for Bank & Insurance Exams
If there are any suggestions/ errors in our PDFs Feel Free to contact us via this email: admin@exampundit.in
Ultra Practice Bundle PDF
SBI Clerk/ RBI Assist. Mains – Reasoning
Akash 42 1978 Green Akash 42 1978 Green
Aruna 60 1960 Black Aruna 36 1984 Black
Avni Blue Avni 38 1982 Blue
Aishwarya 54 1966 Aishwarya 54 1966
Anuja 52 1968 White Anuja 52 1968 White

Case (2) Step 8: Difference of the ages of the one who likes
Name Age Birth year Colour Orange colour and the one who likes Yellow colour is 18
Amit 66 1954 years.
Ajay 48 1972 Orange In case (2), the difference between the person who like
Arpit 48 1972 Red Orange colour and Yellow colour fits or satisfy the
Akash 42 1978 Green condition for Amit because of the age difference between
Aruna 36 1984 Black Amit and Ajay (66-48=18 years).
Avni Blue Remaining Grey colour is liked by Aishwarya.
Aishwarya 54 1966 Case (2)
Anuja 52 1968 White Name Age Birth year Colour
Step 7: The difference of the ages of the one who likes Amit 66 1954 Yellow
Blue colour and Amit is 28 years. Ajay 48 1972 Orange
In case 2, the one who likes Blue colour – (66+/- Arpit 48 1972 Red
28=94/38 years old) . As we know earlier, the maximum Akash 42 1978 Green
age can be 68 years old, so the age of Avni must be 38 Aruna 36 1984 Black
years. Avni 38 1982 Blue
Birth year of Avni (likes blue colour) – 2020-38= 1982 Aishwarya 54 1966 Grey
birth year Anuja 52 1968 White
Case (2) This is the final step.
Name Age Birth year Colour 41) Answer: E
Amit 66 1954 Clearly,the birth year of the one who likes the Grey colour
Ajay 48 1972 Orange is 1966.
Arpit 48 1972 Red None of the answer options is right.

Page 515 of 1334


Subscribe The Xpress Video Course & Mock Test Package for Bank & Insurance Exams
If there are any suggestions/ errors in our PDFs Feel Free to contact us via this email: admin@exampundit.in
Ultra Practice Bundle PDF
SBI Clerk/ RBI Assist. Mains – Reasoning
Hence , option E is the correct choice
42) Answer: D By the birth year of Jeff, we have two cases for the age
Clearly, combination of birth year and colour Ajay is of person who likes Lamborghini Veneno Roadster as
1972- Orange follows-
Hence , option D is the correct choice Case 1:-Age of the person who likes Lamborghini- 61
43) Answer: D years old, birth year is 1959
Clearly, person who likes Black and has birth year as Case 2:- Age of the person who likes Lamborghini- 16
1984 is Aruna , who is 36 years old. years old, birth year is 2004
Hence , option D is the correct choice Step 2: There is a difference of 20 years between the age
44) Answer: E of Bernard and the person who likes Lamborghini
Clearly, the age of Avni 13 years ago is 25 years. Veneno Roadster and Bernard’s age is an even number.
Hence , option E is the correct choice From the above given statements,
45) Answer: C Bernard’s age – age of the person who likes Lamborghini
Clearly,the combination in which The one who was born = 20 years
in year 1984 and the one who likes Blue colour is correct. Case 1:- If we take case 1, means the person who likes
Hence , option C is the correct choice Lamborghini is of 61 years old, then by following the
Direction (46-50)
Name Age Birth Year Car
Detailed Explanation:
Jeff 59 1961
Step 1: Jeff is 59 years old. The age of the one who likes
Bernard 36 1984
Lamborghini Veneno Roadster is obtained by the birth
Larry Aston Martin Valkyrie
year of Jeff.
above equation- we only get the Bernard’s age is in odd
In this sentence at the initial level we don’t know the
number but bernard’s age is an even number.
names of all 9 persons so we create table as per
Bernard’s age = 20 + 61 years = 81 years old (odd
statements.
number).
As per the base year 2020,
Bernard’s age = 61-20 years = 41 years old (odd number).
Birth year of Jeff – 2020-59 = 1961 year
So, case (1) is not valid.
Case 2:- In this case,
Name Age Birth Year Car
Age of the person who likes Lamborghini = 16 years old,
Jeff 59 1961
birth year is 2004
Page 516 of 1334
Subscribe The Xpress Video Course & Mock Test Package for Bank & Insurance Exams
If there are any suggestions/ errors in our PDFs Feel Free to contact us via this email: admin@exampundit.in
Ultra Practice Bundle PDF
SBI Clerk/ RBI Assist. Mains – Reasoning
By the above equation-
Bernard’s age = 20 + 16 years = 36 years old. (Even Name Age Birth Year Car
number), Bernard birth year is 1984 Jeff 59 1961
Bernard’s age = 20 - 16 years = 4 years old. (Even Bernard 36 1984
number), Bernard birth year becomes 2024 but ages are Larry Aston Martin
calculated with respect to 2020. So this possibility is not Valkyrie
valid. Jim
Name Age Birth Year Car Mark
Jeff 59 1961 Amancio
Bernard 36 1984 Alice

Step 3:- The age of the one who likes LykanHypersport Bill

is derived by the birth year of Bernard. Larry likes Aston Warren

Martin Valkyrie.
Age of the person who likes LykanHypersport – 48/84 Mark, Larry and Jim were not born in 1973 as given.
years old. If we assume, Alice was born in 1973 then the age of Jim
As per the base year 2020, become 37 or 73 years old as per condition but both are
Birth year of the person who likes LykanHypersport – prime numbers, so Alice cannot be born in 1973.
For 48 years old- (2020 – 48 = 1972 year) Age of Bill and Warren must be even numbers so their
For 84 years old- (2020 – 84 = 1936 year) age cannot be 47 years old.
Step 4:- Jim’s age is an odd number but not prime As per the above conditions, only Amancio can be born
number. One of the person was born in 1973 and that in 1973 or 47 years old.
person is not Mark, Larry and Jim. The age of Alice is Name Age Birth Year Car
obtained by the birth year of Jim. The age of Bill and Jeff 59 1961
Warren is an even number. The one who likes Bugatti Bernard 36 1984
Divo is 6 years younger than Amancio. Larry Aston Martin Valkyrie
Age of the person who was born in 1973 – (2020-1973= Jim

47 years old) Mark


Here we come to know the names of all 9 richest Amancio 47 1973

personalities given in the question. Alice

Page 517 of 1334


Subscribe The Xpress Video Course & Mock Test Package for Bank & Insurance Exams
If there are any suggestions/ errors in our PDFs Feel Free to contact us via this email: admin@exampundit.in
Ultra Practice Bundle PDF
SBI Clerk/ RBI Assist. Mains – Reasoning
Bill We knew that, the age of eldest person can be 84 years
Warren old
Age of eldest person can be- 2020-1936= 84 years old.
Step 5:- The one who likes Bugatti Divo is 6 years Case 1:- Age of the person who likes LykanHypersport
younger than Amancio. is 84 years old, then by condition,
Age of the person who likes Bugatti Divo- 47-6 = 41 Larry’s age = age of person who likes Bugatti Veyron +
years old. age of person who likes LykanHypersport
Birth year of the person who likes Bugatti Divo- 2020-
41= 1979 year.

Name Age Birth Year Car


Jeff 59 1961
Bernard 36 1984
Larry Aston Martin
Valkyrie
Jim In this case, age of Larry becomes more than 84 years

Mark Name Age Birth Year Car

Amancio 47 1973 Jeff 59 1961

Alice Bernard 36 1984

Bill Larry Aston Martin

Warren Valkyrie
Jim

Step 6:- The sum of the ages of the one who likes Bugatti Mark 41 1979 Bugatti Divo

Veyron by MansoryVivere and the one who likes Amancio 47 1973

LykanHypersport is equal to the Larry’s present age. Alice

None of them was born before 1936. Bill

As in step 3, the age of the person who likes Warren

LykanHypersport can be 48 or 84 years old. old.


So, the one who likes LykanHypersport is 84 years old is
not valid.
Page 518 of 1334
Subscribe The Xpress Video Course & Mock Test Package for Bank & Insurance Exams
If there are any suggestions/ errors in our PDFs Feel Free to contact us via this email: admin@exampundit.in
Ultra Practice Bundle PDF
SBI Clerk/ RBI Assist. Mains – Reasoning
Case 2:- Age of person who likes LykanHypersport is 48 Step 8:- Bill doesn’t likes LykanHypersport.
years old and this satisfies the other conditions as well. Earlier in step 2 & 6, we already knew-
Name Age Birth Year Car Age of the person who likes Lamborghini Veneno
Jeff 59 1961 Roadster is 16 years old.
Bernard 36 1984 Age of the person who likes LykanHypersport is 48 years
Larry Aston Martin old.
Valkyrie Alice’s age is not 16 years or 48 years because it doesn’t
Jim satisfy jim’s age condition.
Mark Age of Bill and Warren must be even numbers and we
Amancio 47 1973 have only these two even ages because the age of Jim
Alice must be in odd number.
Bill Name Age Birth Year Car
Warren Jeff 59 1961
Bernard 36 1984
Step 7:- Jim’s age is an odd number but not prime Larry Aston Martin Valkyrie
number. The age of Bill and Warren is an even number. Jim
The age of Alice is obtained by the birth year of Jim. Mark 41 1979 Bugatti Divo
As in step 5, the person who likes Bugatti Divo is of 41 Amancio 47 1973
years old and born in 1979. Alice
By observing the blank spaces in table, we come to know Bill 16 2004 Lamborghini Veneno
that Bill and Warren cannot like Bugatti Divo because Roadster
their age must be even number. Warren 48 1972 LykanHypersport
Age of Jim cannot be 41 years old because it is a prime
number and so it cannot be related to Alice as well Step 9:- Jim’s age is an odd number but not prime
because if jim born in either 1941 or 2014 then alice age number. The age of Jim is derived by the birth year of
will be 41 years but 1941 as 79 years old(odd number but Warren. The age of Alice is obtained by the birth year of
it is prime number also) & 2014 as 6 years old(even Jim.
number) so both the years are not valid for jim. Birth year of Warren is 1972 so the age of Jim becomes
So, the car, Bugatti Divo must be liked by Mark. 27 or 72 years old.

Page 519 of 1334


Subscribe The Xpress Video Course & Mock Test Package for Bank & Insurance Exams
If there are any suggestions/ errors in our PDFs Feel Free to contact us via this email: admin@exampundit.in
Ultra Practice Bundle PDF
SBI Clerk/ RBI Assist. Mains – Reasoning
Age of Jim must be odd number but not prime, then he The one who likes Mercedes Benz Maybach is elder than
must be 27 years old and born in 1993 as per base year the one who likes Rolls Royce Sweptail and these two
2020. persons are not Alice and Jim. Jim doesn’t like McLaren
Age of Alice by the birth year of Jim can be 93 or 39 P1 LM.
years old but the eldest age can be 84 years old then he Jeff (59 years old) is elder than Amancio (47 years old).
must be 39 years old. The remaining car is Koenigsegg CCXR Trevita.
Birth year of Alice- 2020-39 = 1981 year.
Name Age Birth Car Name Age Birth Car
Year Year
Jeff 59 1961 Jeff 59 1961 Mercedes Benz
Bernard 36 1984 Maybach
Larry Aston Martin Valkyrie Bernard 36 1984 Bugatti Veyron
Jim 27 1993
Larry 84 1936 Aston Martin Valkyrie
Mark 41 1979 Bugatti Divo
Amancio 47 1973 Jim 27 1993 Koenigsegg
Alice 39 1981
Bill 16 2004 Lamborghini Veneno Mark 41 1979 Bugatti Divo
Roadster
Amancio 47 1973 Rolls Royce Sweptail
Warren 48 1972 LykanHypersport

Alice 39 1981 McLaren P1 LM


Step 10:- The one who likes Bugatti Veyron by
MansoryVivere was born in an even numbered year. Bill 16 2004 Lamborghini Veneno

So Bugatti Veyron by MansoryVivere must be liked by Roadster

Bernard who was born in 1984 and 36 years old . Warren 48 1972 LykanHypersport

Larry’s age = 36 + 48 = 84 years old.


This is our final step.
(The sum of the ages of the one who likes Bugatti Veyron
46) ANSWER: E
by MansoryVivere and the one who likes
Clearly, the age of the one who likes Bugatti Veyron by
LykanHypersport is equal to the Larry’s present age.)
MansoryVivere and the one who likes Rolls Royce
Birth year of Larry – (2020-84= 1936 year)
Sweptail respectively is 36,47
Page 520 of 1334
Subscribe The Xpress Video Course & Mock Test Package for Bank & Insurance Exams
If there are any suggestions/ errors in our PDFs Feel Free to contact us via this email: admin@exampundit.in
Ultra Practice Bundle PDF
SBI Clerk/ RBI Assist. Mains – Reasoning
Hence option E is correct choice. Clearly, the sum of the ages of the eldest and the youngest
47) ANSWER: C persons is 100 years.
Clearly, The one who was born in 1972 likes Hence option A is correct choice
LykanHypersport is true among all statements. 50) ANSWER: A
Hence option C is correct choice. Clearly, the difference between the ages of the one who
48) ANSWER: E likes Aston Martin Valkyrie and the one who was born in
Clearly, statements I and III are true. the year of 1961 is 25 years.
Hence option E is correct choice Hence option A is correct choice
49) ANSWER: A

Download Puzzles Practice Questions PDF


Get More Reasoning Practice Questions PDF
Puzzle Random
Direction (1-5): Answer the questions based on the which is pink coloured box. Four boxes are between W
information given below. and O, which is green coloured box. Pink colour box is
above of Decagon shape box. Bigon shape box is three
Ten boxes A, B, C, D, M, N, O, U, V and W are placed place away from yellow coloured box, which places in the
above one and another. Each box surface is different even numbered position from the bottom. Sides of A is
colours such as Pink, Blue, White, Violet, Black, Red, less than the sides of box B, which is perfect square. Less
Green, Yellow, Cyan and Brown and different shape of than three boxes are above W, which is decagon shape.
polygons such as Monogon, Bigon, Triangle, Tetragon, Sides of O is more than sides of N and multiple of sides
Pentagon, Hexagon, Heptagon, Octagon, Nonagon and of box V, which is above of yellow coloured box. Blue
decagon. No two boxes are in the same colour and shape coloured box is below of Cyan and above of White, which
of Polygon. All the information are not necessarily in the is Hexagon. Brown coloured box places in the even
same order. numbered position from the bottom. V has 3 sides and red
One box is between B and black coloured box, which is colour
pentagon shape and below of B. Sum of the sides of the 1) Which box is a Tetragon shape?
box N and M is a perfect square. Number of boxes are
between N and M as same as number of box between O a) Blue coloured box
and N, which box is sides of Perfect cube. Two boxes are b) Box D
between O and N, which is Violet. Neither box M nor V c) Box B
is Black and Blue. Sum of the sides of Violet coloured box d) Green coloured Box
is above of Black coloured box. Number of boxes are e) None of these
above C as same as number of boxes are below of D,

Page 521 of 1334


Subscribe The Xpress Video Course & Mock Test Package for Bank & Insurance Exams
If there are any suggestions/ errors in our PDFs Feel Free to contact us via this email: admin@exampundit.in
Ultra Practice Bundle PDF
SBI Clerk/ RBI Assist. Mains – Reasoning
2) How many boxes are between Red box and white No two boxes are in the same product and quatity. All the
box? information are not necessarily in the same order.

a) Two One box is between Pen box and phone box, which has
b) Three quantity below of pen box. Number of boxes are between
c) Five Z and B as same as number of boxes are between L and
d) Seven B, which box is below of Z. Two boxes are between A
e) Four and Z, which has book and quantity is multiple of 6. A is
above of Z. Three boxes are between Note box and Laptop
3) Four of them following are in the same group, which box, which is above of Note box but below of Y. Neither
is not belongs to that group? P nor K has Laptop and Charger. Five boxes are between
a) B-4 Y and L, which box has phone and quantity is perfect
b) C-6 square. P places two box above of box C, which has Note
c) D-3 and quantity is Prime number. Sum of the quantity of A
d) M-1 and Z as same as the quantity of Pen box, which is perfect
e) O-9 square. One box is between Y and one which has Glass,
which is above of Y and L. Number of boxes are below of
4) Which box is the Red coloured box? L as same as number of boxes are above of Q, which has
Glass. K places above of R and below of J, which has
a) Tetragon Pencil. Sum of the quantity of R and Z as same as sum of
b) Bigon the quantity of Q and A, which has Soap. Sum of the
c) Heptagon quantity of Soap box and K, which has vessel as same as
d) Triangle number of perfect square. Quantity of Paste box is
e) Monogon multiple of 7. Quantity of Pencil is more than Charger.
5) Which of the following pair is in the Top and 6) Which box has Paste?
Bottom?
a) K
a) UD b) P
b) CU c) Y
c) UM d) B
d) CM e) R
e) None of these
7) What is sum of the quantity of Note box and Pencil
Direction (6-10): Answer the questions based on the box?
information given below.
a) 36
Eleven boxes A, B, C, J, K, L, P, Q, R, Y and Z are placed b) 28
above one and another. Each box has different products c) 32
such as Glass, Book, Note, Charger, Phone, Pencil, d) 20
Vessel, Laptop, Soap, pen and Paste and different quantity e) 47
of products such as 3, 5, 7, 8, 9, 11, 12, 13, 14, 25 and 36.
Page 522 of 1334
Subscribe The Xpress Video Course & Mock Test Package for Bank & Insurance Exams
If there are any suggestions/ errors in our PDFs Feel Free to contact us via this email: admin@exampundit.in
Ultra Practice Bundle PDF
SBI Clerk/ RBI Assist. Mains – Reasoning
8) How many boxes are between Paste box and Anbu and Arun, who has Audi and not live in the topmost
Charger box? floor. Akil, who likes English, lives immediately below of
one who like Botany. Number of person live above of
a) Two Anbu as same as number of person live below of Anu,
b) Five who has Benz. Aadhi, who has BMW, lives two floor
c) Four below of Arjun, who likes GK. No one lives above of one
d) Six who likes Physics and below of one who has TVS. Two
e) None person live between Anu and one who like Botany, who
9) Four of them following are in the same group, which lives in the even numbered floor but below of 8th floor.
is not belongs to that group? Ajay lives below of Abi, who has Suzuki. Four person live
between Anu and Anbu, who likes Ethics and lives above
a) L-pen Anu. Neither Aakash nor Amir likes Zoology and Tamil.
b) R-vessel Ajay lives not immediate neighbour of one who has TVS
c) Y-Glass and has Ford. One who lik Zoology lives immediately
d) B-Note above of one who likes Hindi, who lives in the odd
e) Z-Charger numbered floor. One who like Maths and one who has
Honda live immediate floors. Number of person live
10) What is the quantity of Vessel Box? between one who has Honda and Volvo as same as
a) 36 number of person live between one who has Tata and
b) 3 Volvo.
c) 11 11) Who has Jeep?
d) 7
e) 8 a) Amir
b) Arjun
Direction (11-15): Answer the questions based on the c) Akil
information given below. d) Anbu
Ten person namely Arun, Ajay, Anu, Amir, Abi, Arjun, e) Aakash
Aadhi, Akil, Aakash and Anbu are living in the ten floor 12) How many person live between one who has Volvo
building but not necessarily in the same order. Lowest and one who live Economy?
floor is floor 1 and above it is Floor 2 so on until topmost
floor is Floor 10. All of them are like different subject a) Three
such as Physics, Tamil, English, Hindi, Ethics, Maths, b) Six
Botany, Zoology, Gk, Economy and different cars such as c) Two
Audi, Ford, Honda, TVS, Suzuki, Tata, Volvoo, Benz, d) Seven
BMW and Jeep. All the information are not necessarily in e) None
the same order.
13) If Amir is related to TVS and Aadhi is related to
Number of person live between Arjun and Akil as same Audi. In the same way, who is related to Volvo?
as number of person live between Aakash and Akil, who
lives in the odd numbered floor. One person lives between a) Arjun
Page 523 of 1334
Subscribe The Xpress Video Course & Mock Test Package for Bank & Insurance Exams
If there are any suggestions/ errors in our PDFs Feel Free to contact us via this email: admin@exampundit.in
Ultra Practice Bundle PDF
SBI Clerk/ RBI Assist. Mains – Reasoning
b) Anbu top floor. Two person live between Doctor and one who
c) Abi like football, who lives immediate floor of Director. Five
d) Arun floor are between R and P, who is Manager and live in the
e) Anu odd numbered floor. Neither C nor L is police and
Lawyer. J, who is a lawyer, lives three floors below of one
14) Who likes Maths? who like badminton. Numbe of person live between C and
a) Amir one who like Volleyball is one less than number of person
b) Aakash live between one who like badminton and C, who lives in
c) Ajay the even numbered floor below 5. One who like
d) Aadhi Volleyball and badminton live not in the immediate floor.
e) None of these One who like Chess lives two floor below of one who like
Sprint. One who like Snooker lives in the even numbered
15) Which of the following pair is correct? floor but above Singer and above 6th floor.

a) Amir-Volvo 16) Who likes Squash?


b) Aakash-Economy
c) Aadhi-Tamil a) L
d) Anbu-Tata b) C
e) Both C and D c) A
d) B
Direction (16-20): Answer the questions based on the e) K
information given below.
17) How many person live between Singer and
Nine person A, B, C, J, K, L, P, Q and R are living in the Dancer?
ten floor building but not necessarily in the same order.
Lowest floor is floor 1 and above it is Floor 2 so on until a) Four
topmost floor is Floor 9. All of them are different b) Three
profession such as Doctor, Engineer, Lawyer, Dancer, c) Two
Singer, Director, Teacher, Police and different sports such d) Five
as Sprint, Chess, Badminton, Tennis, Volleyball, e) None
Football, Cricket, Squash and Snooker. All the 18) Who live in the 7th floor?
information are not necessarily in the same order.
a) Singer
Number of person live between B and L as same as b) One who like Squash
number of person live between B and Dancer. Two person c) Lawyer
live between A and K, who is an Engineer and lives below d) One who like Sprint
of Teacher but live in the even numbered floor below 5. e) Both A and D
Less than two person live abbve of P, who lives three floor
away from one who like Cricket. Police lives three floor 19) Four of them following are in the same group,
away from Q, who is a Doctor. B, who is a Teacher, lives which is not belongs to that group?
two floor above of R, who likes tennis and B is not in the
a) Volleyball
Page 524 of 1334
Subscribe The Xpress Video Course & Mock Test Package for Bank & Insurance Exams
If there are any suggestions/ errors in our PDFs Feel Free to contact us via this email: admin@exampundit.in
Ultra Practice Bundle PDF
SBI Clerk/ RBI Assist. Mains – Reasoning
b) Squash one who born in India. Sum of the age of one who born in
c) Badminton Kenya and Pakistan is multiple age of one who born in
d) Snooker USA. Sum of the age of one who born in Singapore and
e) Cricket one who born in China is one year less than age of one
who born in Malaysia. One who born in China is elder
20) Who is a Singer? than USA and Younger than one who born in Singapore.
a) C 21) Who was born in Finland?
b) R
c) A a) D
d) J b) R
e) None of these c) V
d) W
Direction (21-25): Answer the questions based on the e) Q
information given below.
22) Who was born in the year 2006?
Twelve person A, B, C, D, P, Q, R, S, U, V, W and Z were
born in the different years viz 1959, 1963, 1968, 1971, a) W
1975, 1978, 1979, 1981, 1989, 1995, 2006 and 2012. All b) V
the person ages calculate from the base year of 2020. All c) P
of them were in the different countries such as India, d) Z
Pakistan, China, USA, UK, France, Russia, Japan, Kenya, e) B
Singapore, Malaysia and Finland. No two person was
born in the same year. All the information are not 23) What is the sum of the age of one who born in
necessarily in the same order. Finland and USA?

Note: If one person was born in the year 2006 that person a) 57
age is 14 based on the base year of 2020 b) 33
c) 39
One who born in Russia is four years elder than Z, who d) 50
was born in Malaysia. Number of person were born before e) 47
R as same as number of person were born after C, who
was born in Kenya. Two person were born between D and 24) Four of them following are in the same group,
R, who is elder than D and age is 42. S, who was born in which is not belongs to that group?
Japan, is younger than U and elder than V, who is the age a) A-UK
of perfect square below 30. Sum of the age of A and W is b) R-Pakistan
one year less than age of one who born in Russia. Two c) Q-China
person were born between Z and B, who was born in UK d) R-Singer
and B, who is the age multiple of 5. Five person were born e) Z-Japan
between D and A, who was born in the leap year and born
in France. Number of person were born between B and Q 25) Which of the following pair is youngest and eldest?
as same as number of person were born between Q and
Page 525 of 1334
Subscribe The Xpress Video Course & Mock Test Package for Bank & Insurance Exams
If there are any suggestions/ errors in our PDFs Feel Free to contact us via this email: admin@exampundit.in
Ultra Practice Bundle PDF
SBI Clerk/ RBI Assist. Mains – Reasoning
a) WV 26) Who has completed M.Sc degree?
b) WU
c) UA a) M
d) UZ b) J
e) WD c) A
d) K
Direction (26-30): Answer the questions based on the e) C
information given below.
27) How many person were born between B and J?
Ten person A, B, C, J, K, L, M, N, S and T were born in
the different years viz 1962, 1969, 1972, 1981, 1984, a) Two
1990, 1993, 1998, 2004 and 2009. All the person ages b) Four
calculate from the base year of 2020. All of them have c) Three
completed different degree such as B.E, MBA, MCA, d) Five
M.Sc, B.Com, MBBS, MS, LLB, B.Ed and B.A. No two e) One
person was born in the same year. All the information are 28) What is the age of J?
not necessarily in the same order.
a) 58
Note: If one person was born in the year 2004 that person b) 36
age is 16 based on the base year of 2020 c) 11
One person was born between K and T, who is the age of d) 51
perfect square below 30 and has completed B.Com. e) 16
Difference between the year of S and L is number of 29) Which of the following pair is true?
perfect square but not more than 20. Two person were
born between N and S, who is younger than N. One who a) C-MCA
has MBA degree was born in the leap year but not younger b) A-MBBS
than C. Neither M nor J has MBA and M.Sc degree. Four c) S-LLB
person were born between C and N, who has completed d) J-B.Com
B.E and born in the leap year. Sum of the age of L and age e) B-LLB
of one who has M.S degree is one year less than age of A,
who was born immediately before N. Less than three 30) Which year M was born?
person were born after C, who is the age multiple of prime a) 2009
number which has two digits and both digits are same. b) 1981
One who has MCA degree was born after L, who has c) 2004
completed B.Ed but born before S. Number of person d) 1962
were born between one who has MBBS and MCA as same e) None of these
as number of person were born between one who has B.A
and one who has M.Sc, who was born immediatly before Direction (31-35): Answer the questions based on the
N. More than three person were born between one who information given below.
has LLB and M.

Page 526 of 1334


Subscribe The Xpress Video Course & Mock Test Package for Bank & Insurance Exams
If there are any suggestions/ errors in our PDFs Feel Free to contact us via this email: admin@exampundit.in
Ultra Practice Bundle PDF
SBI Clerk/ RBI Assist. Mains – Reasoning
Ten persons namely Ravi, Rajan, Ram, Raghu, Rekha, b) CM
Revathi, Roja, Rishi, Rooban and Raghavan are working c) PL
as employee of the university. Each of them has different d) PR
designations such as Clerk (CL), Lab-technician (TE), e) VP
Assistant professor (AP), Professor (PR), HOD (HD),
Vice principal (VP), Principal (PL), Vice-Chancellor 33) How many person work between Raghu and
(VC), Chancellor (CR), Chairman (CM). Designations are Revathi?
in the ascending order (Clerk is a lowest position of the a) Four
university while Chairman is the highest position of the b) One
university). All of them are like different subjects such as c) Two
GK, English, Maths, Physics, History, CA, Hindi, Tamil, d) Three
Polity and Botany. All the information are not necessarily e) Five
in the same order.
34) Four of them following are in the same group,
One who like Zoology is senior to Rishi and Junior to which is not belongs to that group?
Ravi. Less than two person senior to Raghavan, who likes
Tamil. Rajan, one who like Zoology is two position senior a) AP-Maths
to one who like History. Number of person rank before b) PL-Hindi
Ravi as same as number of person Rank after Rishi, who c) HD-english
likes Maths. Rajan is two position below Ravi, who is VC. d) VC-CA
Neither HD nor PL likes History and CA. Number of e) PR-History
person work before Revathi as same as number of person
work after Ram, who is Junior to one who like Zoology. 35) Which of the fllowing statement is false?
TE, who likes Botany is junior to Roja and Rekha but a) Rajan is VP
senior to Rooban. Five person work between Raghavan b) Raghu likes Botany
and one who like Maths, who is junior to Raghavan. Two c) Four person work between Ram and Revathi
person work between one who like Polity and one who d) Ravi is senior to Rajan
like History, who is senior to Polity. One who likes e) All are true
English is junior tone who like CA. Two person work
between one who like GK and Roja, who likes English. Direction (36-40): Answer the questions based on the
information given below.
31) Who is CM?
Ten person M, N, O , P, Q, R, W, X, Y and Z were born
a) Ram in the different months of the same year viz January,
b) Rekha February, April, May, July, August, September, October,
c) Raghavan November and December. All of them are like different
d) Rajan colours such as Red, White, Blue, Black, Orange, Yellow,
e) Revathi Green, Cyan, Gold and Silver. All of them were born in
32) What is the position of one who like Hindi? the different places such as Delhi, Goa, Chennai,
Bangalore, Kochin, Patna, Haryana, panjab, Kolkata and
a) VC
Page 527 of 1334
Subscribe The Xpress Video Course & Mock Test Package for Bank & Insurance Exams
If there are any suggestions/ errors in our PDFs Feel Free to contact us via this email: admin@exampundit.in
Ultra Practice Bundle PDF
SBI Clerk/ RBI Assist. Mains – Reasoning
Mumbai. No two person was born in the same month. All 38) Who was born in the month of May?
the information are not necessarily in the same order.
a) N
M was born before Z and likes Cyan but born in the month b) One who like Orange
of below 31 days. Two person were born between one c) M
who born in Mumbai and one who born in Kolkata, who d) One who like White
is elder than more than two person. Z was born after Y, e) None of these
who likes Green colour and who was born in the month of
below 30 days. One person was born between O and W, 39) Four of them following are in the same group,
who likes Yellow. O, who was born in Bangalore and P which is not belongs to that group?
were born in the month of 31 days and P was born after a) R
O. Four person were born between Y and Z, who was born b) N
in Kolkata. Number of person were born before one who c) Z
born in Mumbai as same as number of person were born d) O
after Q, who likes Black. W was exactly born between M e) P
and X, who was born in Kochin and born below W. R,
who was born in Goa, is three months younger than one 40) If X is related to Kolkata and W is related to Goa.
who born in Delhi. One person was born between one who In the same way, who is related to Delhi?
born in panjab and one who born in Haryana. One who
like Red was born after White and before Orange, who a) R
was born in the month of 30 days. One person was born b) N
between one who like Silver and N, who likes Gold and c) W
not born in Patna. Neither M nor W was born in Panjab d) M
and Patna. e) Z

36) Who was born in Patna? Direction (41-45): Answer the questions based on the
information given below.
a) N
b) M Nine person A, B, C, M, N, O , P, Q and R were born in
c) Q the different months of the same year viz January, March,
d) W April, June, July, August, September, October and
e) Y November. All of them have different exams such as
NTPC, Bank, UPSC, JEE, SSC, TNPSC, RRB, Army,
37) How many person were born between N and one Police and different marks for each exam such as 75, 100,
who like Blue? 150, 200, 250, 300, 400, 600 and 1000. No two person
was born in the same month. All the information are not
a) Two necessarily in the same order.
b) Four
c) Five Number of person have exam before one who has bank
d) Three exam as same as number of person have exam after M,
e) Six who has exam marks four times marks of TNPSC exam
and has exam after R. Neither A nor P has UPSC exam
Page 528 of 1334
Subscribe The Xpress Video Course & Mock Test Package for Bank & Insurance Exams
If there are any suggestions/ errors in our PDFs Feel Free to contact us via this email: admin@exampundit.in
Ultra Practice Bundle PDF
SBI Clerk/ RBI Assist. Mains – Reasoning
and Army exam. Sum of the exam marks of UPSC and 44) Who has exam on July?
SSC as same as marks of M, who has JEE exam. One who
has Bank exam is conduct four months before of UPSC a) Q
exam and bank exam has lowest mark. Five person have b) N
exam between R and O, who has NTPC exam and who c) C
has exam in the month of 31 days. Three person have d) A
exam between A and C, who has TNPSC exam. One e) P
person has exam between R and one who hass UPSC 45) Which of the following pair is not correct?
exam, who has exam before R. Number of person has
exam between one who has NTPC and Q as same as a) N-150
number of person have exam between Q and N, who has b) C-250
exam mark 4times of Bank exam mark. Marks ratio of c) P-400
UPSC and SSC is 3:2 repectively and N attends the exam d) O-100
in the month of 30 days. Two person have exam between e) Q-600
one who has Army exam and Police exam, who has exam
in the month of 30 days but after TNPSC exam. Marks of Direction (46-50): Answer the questions based on the
Army exam is two times the marks of the Bank exam and information given below.
is higher than Marks of the NTPC. Seven persons namely Kavi, Kalai, Karna, Kanmani,
41) Who has UPSC exam? Kumar, Kiran and Karan are attending the meeting in the
different days of the week starting from Monday to
a) C Sunday. All of them are different age persons such as 19,
b) Q 21, 24, 25, 29, 34 and 40. All of them have completed
c) N different degree such as LLB, B.A, MBBS, MBA, CA,
d) B M.S and MCA. No two person is attending the meeting in
e) A the same day. All the information are not necessarily in
the same order.
42) What is the sum of the mark of NTPC and RRB?
One person attends the meeting between one who has
a) 450 MBA and Karna, who is the age multiple of 6 and 12.
b) 225 Kumar attends the meeting two days away from one who
c) 400 has LLB degree. More than two person attend the meeting
d) 750 between Kanmani and one who has LLB degree. Neither
e) 850 Kumar nor Kanmani has degree of MBA and B.A. Two
43) How many person have exam between N and A? person attend the meeting between one who has CA
degree and Kavi, who attends the meeting immediate
a) Two before Kumar but not on Monday. Number of person
b) Three attend the meeting before one who has MBBS degree as
c) Five same as number of person attend the meeting after Kiran,
d) Six who is elder than Kumar. Karna attends the meeting
e) Seven immediately before one who has MCA degree. Karan
Page 529 of 1334
Subscribe The Xpress Video Course & Mock Test Package for Bank & Insurance Exams
If there are any suggestions/ errors in our PDFs Feel Free to contact us via this email: admin@exampundit.in
Ultra Practice Bundle PDF
SBI Clerk/ RBI Assist. Mains – Reasoning
attends the meeting before one who has B.A degree, who 48) How many person attend the meeting between one
is the age of Prime number above 15. Sum of the age of who has M.S and MCA degree?
Karan and Karna is the perfect square. Three person
attend the meeting between Kumar and Kanmani, who is a) Four
the age multiple of 8 and attends the meeting after Kumar. b) Three
Sum of the age of Kalai and Kavi as same as the age of c) Two
one who has MCA degree. d) Five
e) None
46) Who has M.S degree?
49) Four of them following are in the same group,
a) Kavi which is not belongs to that group?
b) Kanmani
c) Karna a) Tuesday-29
d) Kumar b) Friday-21
e) Karan c) Monday-24
d) Wednesday-29
47) Who is attend the meeting on Friday? e) Sunday-34

a) 29 years old person 50) What is the sum of the age of Kiran and Karna?
b) 40 years old person
c) 25 years old person a) 53
d) 34 years old person b) 58
e) None of these c) 64
d) 45
e) 49

Puzzle Random – Answer and Explanation


Answers:
1) C
2) B
3) C
4) D
5) E
Solution (1-5):
• Four boxes are between W and O, which is green
coloured box.
• Less than three boxes are above W, which is
decagon shape.

Page 530 of 1334


Subscribe The Xpress Video Course & Mock Test Package for Bank & Insurance Exams
If there are any suggestions/ errors in our PDFs Feel Free to contact us via this email: admin@exampundit.in
Ultra Practice Bundle PDF
SBI Clerk/ RBI Assist. Mains – Reasoning

Case 2 is not valid, because


• Pink colour box is above of Decagon shape box. • Number of boxes are between N and M as same as
number of box between O and N, which box
issides of Perfect cube.
➔Sides of N=perfect cube (8, 1)
Hence, 1+8=9/ 1+3=4

• Sum of the sides of the box N and M is a perfect


square.
• Two boxes are between O and N, which is Violet.

➔Sum of the sides of M+N= perfect square (4, 9, 16)

Page 531 of 1334


Subscribe The Xpress Video Course & Mock Test Package for Bank & Insurance Exams
If there are any suggestions/ errors in our PDFs Feel Free to contact us via this email: admin@exampundit.in
Ultra Practice Bundle PDF
SBI Clerk/ RBI Assist. Mains – Reasoning
So, side of N=8 and M=1

• Number of boxes are above C as same as number


of boxes are below of D, which is pink coloured
• One box is between B and black coloured box,
box.
which is pentagon shape and below of B.

• Sum of the sides of Violet coloured box is above Case 2 is not valid, because
of Black coloured box.
➔Sides of N > Sides of Black box (pentagon-5)
Page 532 of 1334
Subscribe The Xpress Video Course & Mock Test Package for Bank & Insurance Exams
If there are any suggestions/ errors in our PDFs Feel Free to contact us via this email: admin@exampundit.in
Ultra Practice Bundle PDF
SBI Clerk/ RBI Assist. Mains – Reasoning
• Bigon shape box is three place away from
yellow coloured box, which places in the even
numbered position from the bottom.
• Sides of A is less than the sides of box B, which is
perfect square.
➔Sides of A< Sides of B (4, 9)

• Neither box M nor V is Black and Blue.


• Blue coloured box is below of Cyan and above of
• Sides of O is more than sides of N and multiple of White, which is Hexagon.
sides of box V, which is above of yellow coloured • Brown coloured box places in the even numbered
box. position from the bottom.

➔Sides of O> sides of N ➔


So, O=9 (Nonagon) Cyan
➔Sides of O=Multiple of sides of V ➔3*3=9 Blue
So, V=3 White-Hexagon
Hence, remainind perfect square B=4 and A=2
Hence, Final arrangement as follows

Page 533 of 1334


Subscribe The Xpress Video Course & Mock Test Package for Bank & Insurance Exams
If there are any suggestions/ errors in our PDFs Feel Free to contact us via this email: admin@exampundit.in
Ultra Practice Bundle PDF
SBI Clerk/ RBI Assist. Mains – Reasoning

Case 2 is not valid, because


• Number of boxes are below of L as same as
number of boxes are above of Q, which has
Glass.

Answers:
6) B
7) E
8) C
9) D
10) B
Solution (6-10):
• Five boxes are between Y and L, which box has • Number of boxes are between Z and B as same as
phone and quantity is perfect square. number of boxes are between L and B, which box
• One box is between Y and one which has Glass, is below of Z.
which is above of Y and L.

➔Quantity of Box L= Perfect square (9, 25, 36)

Page 534 of 1334


Subscribe The Xpress Video Course & Mock Test Package for Bank & Insurance Exams
If there are any suggestions/ errors in our PDFs Feel Free to contact us via this email: admin@exampundit.in
Ultra Practice Bundle PDF
SBI Clerk/ RBI Assist. Mains – Reasoning

Case 2 is not valid, because


• Two boxes are between A and Z, which has
book and quantity is multiple of 6.
• A is above of Z.
• Sum of the quantity of A and Z as same as the
quantity of Pen box, which is perfect square.
➔Quantity of Box Z= multiple of 12 (12, 36)
Quantity of pen= Perfect square (9, 25, 36)
Quantity of A+Z= Pen (13 + 12 =25)
So, A=13, Z=12, Pen=25 and L=9 (Pen< Phone)

• One box is between Pen box and phone box, which


has quantity below of pen box.
➔Quantity of pen> Phone (9, 25, 36)

Page 535 of 1334


Subscribe The Xpress Video Course & Mock Test Package for Bank & Insurance Exams
If there are any suggestions/ errors in our PDFs Feel Free to contact us via this email: admin@exampundit.in
Ultra Practice Bundle PDF
SBI Clerk/ RBI Assist. Mains – Reasoning
Case 2 is not valid, because
• Neither P nor K has Laptop and Charger.
• K places above of R and below of J, which has
Pencil.

• Three boxes are between Note box and Laptop


box, which is above of Note box but below of Y.
• P places two box above of box C, which has Note
and quantity is Prime number.
• Sum of the quantity of R and Z as same as sum of
➔Quantity of C = Prime number (11, 7, 3) the quantity of Q and A, which has Soap.

➔Quantity of R+Z= Q+A


Only, 12+8=13+3
So, R=8 and Q=7

Page 536 of 1334


Subscribe The Xpress Video Course & Mock Test Package for Bank & Insurance Exams
If there are any suggestions/ errors in our PDFs Feel Free to contact us via this email: admin@exampundit.in
Ultra Practice Bundle PDF
SBI Clerk/ RBI Assist. Mains – Reasoning

• Sum of the quantity of Soap box and K, which has


vessel as same as number of perfect square. Answers:
• Quantity of Paste box is multiple of 7. 11) B
• Quantity of Pencil is more than Charger. 12) C
➔Quantity of Vessel + Soap= perfect square (16, 25, 36) 13) D
Hence, remaining 3+13=16 14) B
So, vessel=3 and Soap=13 15) E
Quantity of paste= Multiple of 7 (14) Solution (11-15):
Quantity of Charger< Pencil (So, remaining 5<36) • Two person live between Anu and one who like
Botany, who lives in the even numbered floor but
Hence, Final arrangement as follows below of 8th floor.
• Four person live between Anu and Anbu, who
likes Ethics and lives above Anu.

Page 537 of 1334


Subscribe The Xpress Video Course & Mock Test Package for Bank & Insurance Exams
If there are any suggestions/ errors in our PDFs Feel Free to contact us via this email: admin@exampundit.in
Ultra Practice Bundle PDF
SBI Clerk/ RBI Assist. Mains – Reasoning

Case 2 is not valid, because


• Number of person live above of Anbu as same
as number of person live below of Anu, who has
Benz.

• Number of person live between Arjun and Akil as


same as number of person live between Aakash
and Akil, who lives in the odd numbered floor.
• Akil, who likes English, lives immediately below
of one who like Botany.

• One person lives between Anbu and Arun, who


has Audi and not live in the topmost floor.

Case 2 is not valid, because


• Aadhi, who has BMW, lives two floor below of
Arjun, who likes GK.

Page 538 of 1334


Subscribe The Xpress Video Course & Mock Test Package for Bank & Insurance Exams
If there are any suggestions/ errors in our PDFs Feel Free to contact us via this email: admin@exampundit.in
Ultra Practice Bundle PDF
SBI Clerk/ RBI Assist. Mains – Reasoning

• No one lives above of one who likes Physics and


below of one who has TVS.

• One who like Zoology lives immediately above of


one who likes Hindi, who lives in the odd
numbered floor.

• Ajay lives below of Abi, who has Suzuki. Case 2 is not valid, because
• Ajay lives not immediate neighbour of one who
has TVS and who has Ford. • Neither Aakash nor Amir likes Zoology and
Tamil.
Page 539 of 1334
Subscribe The Xpress Video Course & Mock Test Package for Bank & Insurance Exams
If there are any suggestions/ errors in our PDFs Feel Free to contact us via this email: admin@exampundit.in
Ultra Practice Bundle PDF
SBI Clerk/ RBI Assist. Mains – Reasoning
• One who like Maths and one who has Honda
live immediate floors.

• Number of person live between one who has


Honda and Volvo as same as number of person
live between one who has Tata and Volvo.

Hence, Final arrangement as follows Answers:


16) B
17) A
18) E
19) A
20) C
Solution (16-20):
• B, who is a Teacher, lives two floor above of R,
who likes tennis and B is not in the top floor.
• Five floor are between R and P, who is Manager
and live in the odd numbered floor.

Page 540 of 1334


Subscribe The Xpress Video Course & Mock Test Package for Bank & Insurance Exams
If there are any suggestions/ errors in our PDFs Feel Free to contact us via this email: admin@exampundit.in
Ultra Practice Bundle PDF
SBI Clerk/ RBI Assist. Mains – Reasoning
• Police lives three floor away from Q, who is a
Doctor.
• Two person live between Doctor and one who like
football, who lives immediate floor of Director.

Case 2 is not valid, because


• Less than two person live abbve of P, who lives
three floor away from one who like Cricket.

• Number of person live between B and L as same


as number of person live between B and Dancer.

• Two person live between A and K, who is an


Engineer and lives below of Teacher but live in the
even numbered floor.
• Neither C nor L is police and Lawyer.
• Numbe of person live between C and one who like
Volleyball is one less than number of person live
between one who like badminton and C, who lives
in the even numbered floor.

Page 541 of 1334


Subscribe The Xpress Video Course & Mock Test Package for Bank & Insurance Exams
If there are any suggestions/ errors in our PDFs Feel Free to contact us via this email: admin@exampundit.in
Ultra Practice Bundle PDF
SBI Clerk/ RBI Assist. Mains – Reasoning

Case 2 is not valid, because • One who like Volleyball and badminton live not in
• J, who is a lawyer, lives below of one who like the immediate floor.
badminton. • One who like Chess lives two floor below of one
who like Sprint. One who like Snooker lives in the
even numbered floor but above Singer.
Hence, Final arrangement as follows

Answers:
21) B
22) C
23) D
24) D
25) B
Solution (21-25):
• Two person were born between D and R, who is
elder than D.
• Five person were born between D and A, who was
born in the leap year and born in France.

Page 542 of 1334


Subscribe The Xpress Video Course & Mock Test Package for Bank & Insurance Exams
If there are any suggestions/ errors in our PDFs Feel Free to contact us via this email: admin@exampundit.in
Ultra Practice Bundle PDF
SBI Clerk/ RBI Assist. Mains – Reasoning
➔Age of A+W=Age of one who born in Russia -1
Hence, 52+8=61-1
So, Age of A=52/8 and W=52/8

• Number of person were born before R as same as


number of person were born after C, who was born
• Two person were born between Z and B, who was
in Kenya.
born in UK and B, who is the age multiple of 5.
➔Age of B=multiple of 5 (25, 45)

Case 2 is not valid, because


• Sum of the age of A and W is one year less
than age of one who born in Russia. • One who born in Russia is four years elder than Z,
who was born in Malaysia.
Page 543 of 1334
Subscribe The Xpress Video Course & Mock Test Package for Bank & Insurance Exams
If there are any suggestions/ errors in our PDFs Feel Free to contact us via this email: admin@exampundit.in
Ultra Practice Bundle PDF
SBI Clerk/ RBI Assist. Mains – Reasoning
• S, who was born in Japan, is younger than U and
elder than V, who is the age of perfect square.

➔Age of V= perfect Square (25)

• Number of person were born between B and Q as


same as number of person were born between Q
and one who born in India.

• Sum of the age of one who born in Kenya and


Pakistan is multiple age of one who born in USA.
➔Age of one who born in Kenya+ Pakistan= Multiple of
Age of one who born in USA
Hence, 41+39=80/ 41+31=72 (because both multiple of
8)

Page 544 of 1334


Subscribe The Xpress Video Course & Mock Test Package for Bank & Insurance Exams
If there are any suggestions/ errors in our PDFs Feel Free to contact us via this email: admin@exampundit.in
Ultra Practice Bundle PDF
SBI Clerk/ RBI Assist. Mains – Reasoning

Hence, Final arrangement as follows


• Sum of the age of one who born in Singapore
and one who born in China is one year less than
age of one who born in Malaysia.
• One who born in China is elder than USA and
Younger than one who born in Singapore.
➔Age of one who born in Singapore + China= age of
one who bron in Malaysia – 1
Hence, 31+25= 57-1
So, Age of D=31, Z= 57 and V=25
Singapore
China
USA

Answers:
26) C
Page 545 of 1334
Subscribe The Xpress Video Course & Mock Test Package for Bank & Insurance Exams
If there are any suggestions/ errors in our PDFs Feel Free to contact us via this email: admin@exampundit.in
Ultra Practice Bundle PDF
SBI Clerk/ RBI Assist. Mains – Reasoning
27) B
28) C
29) C
30) D
Solution (26-30):
• Four person were born between C and N, who has
completed B.E and born in the leap year.
• Less than three person were born after C, who is
the age multiple of prime number.

• Difference between the year of S and L is number


of perfect square.
➔Difference of the age of S+L= Perfect Square (9, 36)
Hence, 39-30= 9 / 58-22= 36

• Two person were born between N and S, who is


younger than N.

Page 546 of 1334


Subscribe The Xpress Video Course & Mock Test Package for Bank & Insurance Exams
If there are any suggestions/ errors in our PDFs Feel Free to contact us via this email: admin@exampundit.in
Ultra Practice Bundle PDF
SBI Clerk/ RBI Assist. Mains – Reasoning
Case 2 is not valid, because
• Sum of the age of L and age of one who has M.S
degree is one year less than age of A, who was
born immediately before N.
➔Age of L+ one who has M.S = age of A-1
Hence, 39+11= 51-1
So, age of L=39, A=51 and M.S=11

Case 2 is not valid, because


• One who has MBA degree was born in the leap
year but not younger than C.
• Neither M nor J has MBA and M.Sc degree.

• One person was born between K and T, who is the


age of perfect square and has completed B.Com.
➔Age of T= Perfect Square (16, 36)

Page 547 of 1334


Subscribe The Xpress Video Course & Mock Test Package for Bank & Insurance Exams
If there are any suggestions/ errors in our PDFs Feel Free to contact us via this email: admin@exampundit.in
Ultra Practice Bundle PDF
SBI Clerk/ RBI Assist. Mains – Reasoning
• One who has MCA degree was born after L, who
has completed B.Ed but born after S.

• More than three person were born between one


who has LLB and M.
Hence, Final arrangement as follows
Case 2 is not valid, because
• Number of person were born between one who
has MBBS and MCA as same as number of
person were born between one who has B.A
and one who has M.Sc, who was born before N.

Answers:
Page 548 of 1334
Subscribe The Xpress Video Course & Mock Test Package for Bank & Insurance Exams
If there are any suggestions/ errors in our PDFs Feel Free to contact us via this email: admin@exampundit.in
Ultra Practice Bundle PDF
SBI Clerk/ RBI Assist. Mains – Reasoning
31) B
32) C
33) A
34) D
35) C
Solution (31-35):
• Less than two person senior to Raghavan, who
likes Tamil.
• Five person work between Raghavan and one who
like Maths, who is junior to Raghavan.

Case 2 is not valid, because


• Number of person rank before Ravi as same as
number of person Ran after Rishi, who likes
Maths.

Case 2 is not valid, because


• Rajan is two position away from Ravi, who is • Rajan, one who like Zoology is two position senior
VC. to one who like History.

Page 549 of 1334


Subscribe The Xpress Video Course & Mock Test Package for Bank & Insurance Exams
If there are any suggestions/ errors in our PDFs Feel Free to contact us via this email: admin@exampundit.in
Ultra Practice Bundle PDF
SBI Clerk/ RBI Assist. Mains – Reasoning
• Number of person work before Revathi as same as
number of person work after Ram, who is Junior
to one who like Zoology.

Case 3 is not valid, because


• One who like Zoology is senior to Rishi and
Junior to Ravi.
• Two person work between one who like Polity and
one who like History, who is senior to Polity.

Page 550 of 1334


Subscribe The Xpress Video Course & Mock Test Package for Bank & Insurance Exams
If there are any suggestions/ errors in our PDFs Feel Free to contact us via this email: admin@exampundit.in
Ultra Practice Bundle PDF
SBI Clerk/ RBI Assist. Mains – Reasoning
• One who likes English is junior tone who like CA.
• Two person work between one who like GK and
Roja, who likes English.
Hence, Final arrangement as follows

Case 2 is not valid, because


• TE, who likes Botany is junior to Roja and
Rekha but senior to Rooban.

Answers:
36) C
37) B
38) D
39) C
40) A
Solution (36-40):
• Z was born after Y, who likes Green colour and
who was born in the month of below 31 days.
• Four person were born between Y and Z, who was
born in Kolkata.

• Neither HD nor PL likes History and CA.

Page 551 of 1334


Subscribe The Xpress Video Course & Mock Test Package for Bank & Insurance Exams
If there are any suggestions/ errors in our PDFs Feel Free to contact us via this email: admin@exampundit.in
Ultra Practice Bundle PDF
SBI Clerk/ RBI Assist. Mains – Reasoning

Case 2 is not valid, because • M was born before Z and likes Cyan but born in
• Two person were born between one who born the month of below 31 days.
in Mumbai and one who born in Kolkata, who • W was exactly born between M and X, who was
is elder than more than two person. born in Kochin.

Case is not valid, because


• One person was born between O and W, who
• Number of person were born before one who
likes Yellow.
born in Mumbai as same as number of person
were born after Q, who likes Black.

Page 552 of 1334


Subscribe The Xpress Video Course & Mock Test Package for Bank & Insurance Exams
If there are any suggestions/ errors in our PDFs Feel Free to contact us via this email: admin@exampundit.in
Ultra Practice Bundle PDF
SBI Clerk/ RBI Assist. Mains – Reasoning

Case 2 is not valid, because • R, who was born in Goa, is three months younger
• O, who was born in Bangalore and P were born than one who born in Delhi.
in the month of 31 days and P was born after O.

• One who was born between one who like Silver


and N, who likes Gold and not born in Patna.

Case 2 is not valid, because


Page 553 of 1334
Subscribe The Xpress Video Course & Mock Test Package for Bank & Insurance Exams
If there are any suggestions/ errors in our PDFs Feel Free to contact us via this email: admin@exampundit.in
Ultra Practice Bundle PDF
SBI Clerk/ RBI Assist. Mains – Reasoning

• One person was born between one who born in


panjab and one who born in Haryana.
• Neither M nor W was born in Panjab and Patna.

Answers:
41) B
42) C
43) D
44) A
45) A
Solution (41-45):
• Five person have exam between R and O, who has
NTPC exam and who has exam in the month of 31
days.
• One person has exam between R and one who hass
UPSC exam, who has exam before R.

• One who like Red was born after White and before
Orange, who was born in the month of 30 days

Hence, Final arrangement as follows

Page 554 of 1334


Subscribe The Xpress Video Course & Mock Test Package for Bank & Insurance Exams
If there are any suggestions/ errors in our PDFs Feel Free to contact us via this email: admin@exampundit.in
Ultra Practice Bundle PDF
SBI Clerk/ RBI Assist. Mains – Reasoning

• One who has Bank exam is conduct three months • Three person have exam between A and C, who
before of UPSC exam and which exam has lowest has TNPSC exam.
mark.
➔Marks of Bank exam= Lowest (75)

Case 2 is not valid, because Case 2 is not valid, because


• Number of person have exam before one who • Neither A nor P has UPSC exam and Army exam.
has bank exam as same as number of person • Sum of the exam marks of UPSC and SSC as
have exam after M, who has exam marks four same as marks of M, who has JEE exam.
times marks of TNPSC exam and who has exam ➔Marks of UPSC+ SSC= JEE (1000, 400, 400)
after R. (600+400=1000/ 150=250=400/ 400+200=600)
➔Marks of exam M= 4 times mark of Tnpsc Exam
(250*4=1000/ 100*4=400/ 150*4=600)

Page 555 of 1334


Subscribe The Xpress Video Course & Mock Test Package for Bank & Insurance Exams
If there are any suggestions/ errors in our PDFs Feel Free to contact us via this email: admin@exampundit.in
Ultra Practice Bundle PDF
SBI Clerk/ RBI Assist. Mains – Reasoning

• Number of person has exam between one who has Case 2 is not valid, because
NTPC and Q as same as number of person have • Marks ratio of UPSC and SSC is 3:2 repectively
exam between Q and N, who has exam mark and N attends the exam in the month of 30 days.
multiple of Bank exam mark.
➔Marks of UPSC: SSC= 3:2
➔Marks of exam N= multiple of 75 (150, 300, 600)
Hence, 600:400
So, Marks of UPSC=600 and SSC=400
➔Based on the above JEE=1000 and TNPSC=250

• Two person have exam between one who has


Army exam and Police exam, who has exam in the
month of 30 days but after TNPSC exam.
• Marks of Army exam is two times the marks of the
Bank exam, which is higher than Marks of the
NTPC.

Hence, Final arrangements as follows

Page 556 of 1334


Subscribe The Xpress Video Course & Mock Test Package for Bank & Insurance Exams
If there are any suggestions/ errors in our PDFs Feel Free to contact us via this email: admin@exampundit.in
Ultra Practice Bundle PDF
SBI Clerk/ RBI Assist. Mains – Reasoning

Answers:
46) C
47) D
48) E
49) D
50) B Case 2, 3 and 4 are not valid, because
Solution (45-50): • More than two person attend the meeting
• Three person attend the meeting between Kumar between Kanmani and one who has LLB
and Kanmani, who is the age multiple of 8 and degree.
attends the meeting after Kumar.
• Kumar attends the meeting two days away from
one who has LLB degree.
➔Age of Kanmani= Multiple of 8 (24, 40)

Page 557 of 1334


Subscribe The Xpress Video Course & Mock Test Package for Bank & Insurance Exams
If there are any suggestions/ errors in our PDFs Feel Free to contact us via this email: admin@exampundit.in
Ultra Practice Bundle PDF
SBI Clerk/ RBI Assist. Mains – Reasoning
• One person attends the meeting between one who
has MBA and Karna, who is the age multiple of 6.
• Neither Kumar nor Kanmani has degree of MBA
and B.A.
➔Age of Karna= multiple of 6 (24)
So, Age of Kanmani= 40

Case 2 is not valid, because


• Karna attends the meeting immediately before
one who has MCA degree.

• Two person attend the meeting between one who


has CA degree and Kavi, who attends the meeting
before Kumar but not on Monday.

• Sum of the age of Kalai and Kavi as same as the


age of one who has MCA degree.

➔Age of Kalai+ Kavi= Age of one who has MCA


degree
Hence, 19+21=40
So, Age of Kalai=19/21 and Kavi= 19/21

Page 558 of 1334


Subscribe The Xpress Video Course & Mock Test Package for Bank & Insurance Exams
If there are any suggestions/ errors in our PDFs Feel Free to contact us via this email: admin@exampundit.in
Ultra Practice Bundle PDF
SBI Clerk/ RBI Assist. Mains – Reasoning
➔Age of Kavi= prime number = 19

• Number of person attend the meeting before one


• Sum of the age of Karan and Karna is the perfect
who has MBBS degree as same as number of
person attend the meeting after Kiran, who is elder square.
than Kumar. ➔Age of Karna+ Karan= perfect square
Hence, 24+25=49
➔Age of Kiran> Age of kumar So, Age of Karan=25

Hence, Final arrangement as follows

• Karan attends the meeting before one who has B.A


degree, who is the age of Prime number.
Download Puzzles Practice Questions PDF
Get More Reasoning Practice Questions PDF
Page 559 of 1334
Subscribe The Xpress Video Course & Mock Test Package for Bank & Insurance Exams
If there are any suggestions/ errors in our PDFs Feel Free to contact us via this email: admin@exampundit.in
Ultra Practice Bundle PDF
SBI Clerk/ RBI Assist. Mains – Reasoning
Puzzle Profession
Directions (1-5): Read the following information (a) Brown
carefully and answer the questions given beside. (b) Blue
Seven employees of an organization namely P to V belong (c) Orange
to different ranks/roles in the hierarchy. Their (d) Black
designations (not necessarily in same order) in descending (e) Pink
order are mentioned below: 3. If all the persons are arranged in alphabetical
Chairman > Managing Director > HOD> Senior order from top to bottom with respect to
Manager> Assistant Manager> Executive> Trainee designation order then T’s designation is
They wear shirts of different colors as per the dress code a. Managing Director
decided by organization's HR for each role. The one who b. HOD
is an Executive wears Orange shirt. R whose designation c. Senior Manager
is higher than that of S and he wears Black shirt. The one d. Assistant Manager
who wears Pink shirt is three ranks higher than the one e. Executive
who wears Orange Shirt. U is Chairman and wears Beige 4. How many people are there between the one who
Color shirt. The shirt worn by V is of Grey color. T is 2 like Beige color and the one who like Orange color?
ranks lower than U. The one having third lowest (a) 3
designation wears Brown shirt. Neither P nor Q wears (b) 4
Brown shirt. Senior Manager wears Blue shirt. V does not (c) 6
belong to the top three ranks. (d) 5
1. What is the rank of P from Top as per given (e) 2
designation order? 5. If U related to ‘Pink color’ and R related to ‘Blue
(a) 3 color’ then S related to which color?
(b) 4 (a) Black
(c) 6 (b) Grey
(d) Either (b) or (c) (c) Brown
(e) 2 (d) Orange
2. The person whose designation is HOD wears which (e) None of these
color shirt?
Page 560 of 1334
Subscribe The Xpress Video Course & Mock Test Package for Bank & Insurance Exams
If there are any suggestions/ errors in our PDFs Feel Free to contact us via this email: admin@exampundit.in
Ultra Practice Bundle PDF
SBI Clerk/ RBI Assist. Mains – Reasoning
Directions (6-10): These questions are based on the (a) R
following information. (b) W
P, Q, R, S, T, U, V and Ware eight Employee are working (c) S
in three different Universities, Disha University, Arihant (d) Q
University, G.M. University and not more than three (e) None of these
persons are working in the same University. Each of them 8. Which among the following groups of Designations
has different designations viz. Director, Principal, are from Disha University?
Professor, Associate Professor, Assistant Professor, (a) Associate Professor, Assistant Professor and
Lecturer, Lab Assistant and Librarian, not necessarily in Director
the same order. Designation order is given in descending (b) Principal, Professor-Librarian and Lab Assistant
order. S works neither at Disha University nor works in (c) Librarian, Lab-Assistant and Principal
the same university as U works at. R and U work in the (d) Professor, Director and Lecturer
same University and R designation is not 4th from bottom. (e) None of these
Q is principal works at Arihant University and T is the 9. Four of the following five are alike in a certain way
only other person who works in Arihant University. based on the given information and so form a
Neither the Professor nor the Assistant Professor works in group. Find the one which does not belong to the
GM University. P designation is highest among them but group.
does not work in G.M. University. Neither V nor W holds (a) Q - Principal
4th rank designation from Top. U is a Lecturer and V is not (b) S - Professor
least rank in designation. (c) U- Assistant Professor
6. Which among the following groups works in G.M. (d) W- Lecturer
University? (e) V-Librarian
(a) Q, R, S 10. Which among the following is True as per Given
(b) T, V, U Information?
(c) Q, S, T (a) S- G.M.University - Professor
(d) P, S, R (b) Q- ArihantUniversity –Lab Assistant
(e) V, W, S (c) T – Arihant University – Assistant Professor
7. Among the Following, Whose Designation is (d) W-Disha University -Lab Assistant
Librarian? (e) None of these

Page 561 of 1334


Subscribe The Xpress Video Course & Mock Test Package for Bank & Insurance Exams
If there are any suggestions/ errors in our PDFs Feel Free to contact us via this email: admin@exampundit.in
Ultra Practice Bundle PDF
SBI Clerk/ RBI Assist. Mains – Reasoning
Directions (11-15): These questions are based on the 11. What is the experience of the person who is HOD
following information (fully modified) of Chemical department?
Eight People A, B, C, D, E, F, G and H work in different (a) 14
Engineering department as HOD posts. Each of them had (b) 10
experience in their field of certain number of years, which (c) 8
was different for each of them. These years were among (d) 20
27, 24, 20, 16, 14, 10, 8 and 3 (27 years experienced (e) None of these
person is senior most person and 3 years experienced 12. What is the difference in the years of experience of
person is junior most person). Also it is known that the person who is HOD of Civil and Mechanical
different departments is Computer Science, Electronics, departments?
Civil, Mechanical, Electrical, I.T., Chemical, and (a) 7
Biotechnology but not in a same order. (b) 4
(c) 13
Further it is known that G who had an experience more (d) 11
than B but less than C and is HOD of Chemical (e) None of these
department. B had an experience of odd number of years. 13. E is HOD of which department?
No one is there between the experience of H and F, who (a) Computer Science
is not the HOD of Electrical department. Four persons are (b) Electronics
there between the experience of F and E. HOD of (c) Civil, Mechanical,
Electronics department is junior to the HOD of Computer (d) Electrical
department and both of their experiences are in multiple (e) Cannot be determined
of 2. H had an experience of more than 20 years but not 14. Who is HOD of Computer Science Department?
the highest. C who had 16 years of experience works as (a) B
HOD of Civil department. The person who is HOD of (b) C
Electrical department had an experience of 20 years. A (c) A
had an experience less than D but more than G and E. (d) E
HOD of Mechanical department is immediately senior to (e) None of the above
the HOD of Biotechnology department and difference 15. Which among the following pair is odd from the
between their experiences is not more than three years. rest?

Page 562 of 1334


Subscribe The Xpress Video Course & Mock Test Package for Bank & Insurance Exams
If there are any suggestions/ errors in our PDFs Feel Free to contact us via this email: admin@exampundit.in
Ultra Practice Bundle PDF
SBI Clerk/ RBI Assist. Mains – Reasoning
(a) F,27 (c) The one who like black color
(b) H,24 (d) Q
(c) G,8 (e) None of these
(d) D,20 17. Who among the following Green color?
(e) B,10 (a) R
Directions (16-20): These questions are based on the (b) P
following information. (c) L
Seven persons namely L, M, N, O, P, Q and R working as (d) Q
a Trainee, AM( Assistant Manager), MGR(Manager), SM (e) None of these
(Senior Manager), MD (Managing Director), CMD (Chief 18. Which of the following color between O and P?
Managing Director), CEO(Chief Executive Officer) not in (a) Green
the same order in a Company. Each likes a different color (b) Blue
namely Red, Green, Yellow, White, Blue, Black and (c) Red
Brown not in same order. (d) Yellow
NOTE: positions are in the Ascending order of their (e) None of these
experience. CEO is most experienced person and Trainee 19. How many personsisSenior to O?
least experienced. (a) Four
Only two persons senior to N. O does not like Yellow (b) One
color. P is senior to the Person who likes White color who (c) Three
does not the least junior. CMD is junior to the Person who (d) More than four
likes Black color. Person who likes Yellow color is senior (e) None
to Q but junior to P. R is junior to P who is junior to MD. 20. Who among the following MD of the company?
R is senior to Q who likes Blue color. M is senior to P but (a) Q
not the senior most. MD does not like Brown color. O is (b) P
junior to R who is junior to L. MD is senior to the one who (c) M
likes Green color. (d) L
16. Who among the following is CEO of the company? (e) N
(a) P Directions (21-25): Study the following information
(b) M carefully to answer the given questions:

Page 563 of 1334


Subscribe The Xpress Video Course & Mock Test Package for Bank & Insurance Exams
If there are any suggestions/ errors in our PDFs Feel Free to contact us via this email: admin@exampundit.in
Ultra Practice Bundle PDF
SBI Clerk/ RBI Assist. Mains – Reasoning
Six people from a college participated in a cultural event (c) F
held in Indore. Each of these six people had different (d) C
designations. The designation of these people in (e) None of these
decreasing order is as Chairman, Dean, HOD, Professor, 23. What is the age difference between B and E?
Asst. Professor and Lecturer. Each of them was an expert (a) 22years
in different engineering branch. No one is older than (b) 23years
65years. (c) 24years
C is junior than F, who is not Dean. The one who is Dean (d) 25years
is 5years younger than HOD. More than two persons are (e) None of these
junior than A. Age of the one who is Lecturer is a perfect 24. How many persons are junior to F?
cube of an odd number. Age of the one who is Lecture is (a) Four
not less than 25years. B is Junior than D but senior than (b) One
C. Age of the one who is HOD is just double the age of (c) Three
the one who is Lecturer. F is senior to E who is not (d) Two
Lecturer. D is not Professor. Age of the one who is (e) None of these
Professor is a perfect square of number and 13years 25. Who among the following Lecturer of the College?
younger than D. The one who is Chairman is 11years older (a) A
than the one who is HOD. D is senior to A. F’ age is an (b) C
odd number. E is 7years younger than D. B is 9years older (c) D
than C. (d) E
21. Who among the following is HOD of the College? (e) None of these
(a) F Directions (26-30): Study the following information to
(b) D answer the given questions:
(c) The one who is 32years old There are six employees of a company and all of them are
(d) A working on six different positions – AM (Assistant
(e) None of these Manager), MG (Manager), DGM (Deputy General
22. Who among the following is 65years old? Manager), GM (General Manager), CGM (Chief General
(a) E Manager), ED (Executive Director). The positions given
(b) B are in increasing order of seniority. (AM is the least junior

Page 564 of 1334


Subscribe The Xpress Video Course & Mock Test Package for Bank & Insurance Exams
If there are any suggestions/ errors in our PDFs Feel Free to contact us via this email: admin@exampundit.in
Ultra Practice Bundle PDF
SBI Clerk/ RBI Assist. Mains – Reasoning
and the ED is the most senior.). Each person likes different (a) AAJTAK
TV Channels and Weight of each person is different. (b) Sony
M is senior to P, but the Weight of P is heavier than M. R (c) Star
likes Sony Channel and junior to N. Less than three person (d) ZEE
are senior to O. Weight of O is lighter than N but heavier (e) None of these
than R. The one who likes Aajtak is senior to P but junior 28. What may be the height of R?
to the one who likes ABP channel. M does not like (a) 173 kg
AAJTAK. P is not AM in the company. Weight of M is (b) 184 kg
the lightest and he neither ED nor GM in the company. (c) 170 kg
The Weight of the second lightest person is 171kg. The (d) 185 kg
one who like Star is junior to the one who like ZEE. R is (e) 186 kg
senior to Q. Weight of the R is not odd number. Weight of 29. Who among the following is MG of the company?
the one who is CGM of the company is lighter than R. (a) The one who is just junior to Q
Weight of the one who is AM is lighter than the one who (b) The one whose weight is more than R
is GM. R is junior to the one who like IBC24. Weight of (c) The one who likes AAJTAK
the second Heaviest person is 184kg and O is not second (d) The one who likes ABP
Heaviest person. Weight of the one who likes ABP is less (e) The one who likes ZEE
than the one who is AM. The one who likes ZEE is not 30. Which of the following statement is true about O?
DGM of the company. Q is not Heaviest person but (a) O is junior to M
heavier than P. O does not like Aajtak. M neither likes (b) O likes IBC24
ZEE nor ABP. (c) Weight of O is more than Q
26. The one who is CGM of the company likes which (d) Only three persons are lighter than R
of the following TV Channels? (e) None is true
(a) ABP Directions (31-35): Study the following information
(b) IBC24 carefully and answer the Questions. (Partially
(c) ZEE modified)
(d) Star Seven persons works in Police at different Ranks. Their
(e) None of these ranks are C.P. (Commissioner of Police), D.C.P (Deputy
27. Q likes which of the following TV Channels? Commissioner of Police), A.C.P (Assistant Commissioner

Page 565 of 1334


Subscribe The Xpress Video Course & Mock Test Package for Bank & Insurance Exams
If there are any suggestions/ errors in our PDFs Feel Free to contact us via this email: admin@exampundit.in
Ultra Practice Bundle PDF
SBI Clerk/ RBI Assist. Mains – Reasoning
of Police), P.I (Police Inspector), A.P.I (Assistant Police (d) CP
Inspector), S.I. (Sub Inspector), A.S.I (Assistant Sub (e) DCP
Inspector), not necessarily in the same order. (Note: Rank 33. How many persons are their between the person
decreases on moving from left to right). Each of them has whose rank is CP and the person whose weight is
different weight in kg viz, 41, 51, 44, 35, 57, 19 and 25, 51?
but not necessarily in the same order. (a) 3
The one whose weight is 44 kg is either ACP or API. The (b) 4
rank of both J and N were lower than the person whose (c) 2
weight is 41 kg. Only three persons are there in ranks (d) 5
between the one whose weight is 44 kg and the one whose (e) 1
weight is 25 kg. Only two persons are there in ranks 34. If all the weight are arranged in increasing order
between J and the one whose weight is 35 kg who is senior from west to east then what is the weight of the
to J. N is not the lightest. At least five people junior to K. person whose rank is P.I.?
J is just senior to N. M rank is immediately after the one (a) 41
whose weight is 35 kg. P is just senior to the one who is (b) 44
the second heaviest. Only one person is there in ranks (c) 51
between K and the person whose weight is 41 kg. Only (d) 57
two persons are there in ranks between M and L. One of (e) 35
the persons is O. 35. Difference of weight of the person whose rank is
31. What is the weight of N? DCP and ASI is?
(a) 44 (a) 29
(b) 35 (b) 30
(c) 57 (c) 31
(d) 25 (d) 33
(e) 41 (e) 38
32. What is the rank of the person whose weight 44? Directions (36-40): Study the following information
(a) ACP carefully and answer the Questions (fully blunder
(b) API mistake and completely modified)
(c) SI

Page 566 of 1334


Subscribe The Xpress Video Course & Mock Test Package for Bank & Insurance Exams
If there are any suggestions/ errors in our PDFs Feel Free to contact us via this email: admin@exampundit.in
Ultra Practice Bundle PDF
SBI Clerk/ RBI Assist. Mains – Reasoning
Each people P, Q, R, S, T, U, V and W works in the same 36. The person who lives in Pune works in which
office but in a different department on the basis of department?
experience namely Research and Development (R&D), (a) HR
Public Relations (PR), Human Resources (HR), Quality (b) R&D
Management (QM), Production (PO), Accounts (ACC), (c) ACC
Marketing & Sales (M&S) and Administration (ADMIN) (d) ADMIN
in the same order. Each person has been allocated to a (e) PR
department as per decreasing order of experience with the 37. How many persons are there between the person
one in ADMIN being the least experienced while the one who works in HR department and Q?
in R&D being the most experienced. They live in different (a) 2
cities of India like Raipur, Delhi, Mumbai, Chennai, (b) 3
Hyderabad, Bhopal, Pune and Indore (not necessarily in (c) 4
same order). (d) 5
Three departments are there between the person who lives (e) 1
in Bhopal & U who is more experience than the person 38. Which of the following combinations are true for
who lives in Bhopal. T’s is more experience than the V?
person who lives in Raipur. Neither the person who lives (a) HR – Hyderabad
in Chennai nor the person who lives in Indore has most (b) M&S - Raipur
experience. W’s is more experience than S but less (c) ADMIN - Mumbai
experience than T. R & P lives in Hyderabad and Mumbai (d) PR - Chennai
respectively. No person lies between V & R. Neither U (e) ACC - Raipur
nor Q lives in Pune. Neither W nor S lives in Raipur. Three 39. How many person have more experience than that
person are there between P & the person who lives in the person who lives in Raipur?
Raipur. Q works in marketing & Sales department. Two (a) 2
person lies between U & T.. One person lies between T & (b) 3
the person who lives in Pune. V is less experience than P. (c) 4
The person who lives in Indore is more experienced than (d) 5
the person who lives in Chennai. (e) 6

Page 567 of 1334


Subscribe The Xpress Video Course & Mock Test Package for Bank & Insurance Exams
If there are any suggestions/ errors in our PDFs Feel Free to contact us via this email: admin@exampundit.in
Ultra Practice Bundle PDF
SBI Clerk/ RBI Assist. Mains – Reasoning
40. Four of the following five pairs are alike in a two people have more experience than the one who works
certain way based on their positions in the above for Vasan healthcare. R works for Narayana health and
arrangement and so form a group. Which of the has more experience than the one who works for Vasan
following does not belong to the group? healthcare. O has less experience than the one is
(a) U- Chennai specialized in Gynecology, but more experience than the
(b) R – Indore one who works for Apollo hospital. P neither has the least
(c) S - Bhopal experience nor did he specialize in Nephrology. M is not
(d) Q – Mumbai specialized in Nephrology. The one who works for
(e) T - Bhopal wockhard hospital not specialized of Gynecology.
Directions (41-45): Study the following information 41. Which of the following is Cardiology Specialist?
carefully and answer the Questions (a) L
Seven Person namely L, M, N, O, P, Q and R are works in (b) M
different specialization of different hospitals namely (c) N
Apollo Hospitals, Care Hospitals, Fortis Hospitals, Mittal (d) Q
Hospitals,Narayana Health,Vasan Healthcare,Wockhard (e) O
Hospitals but not necessarily in the same order. Each 42. Person who works for Fortis Hospital is specialized
doctor works in the different hospitals but in a different in which department?
specialization department on the basis of experience (a) Radiology
namely Accident and emergency (A&E), Physiotherapy, (b) Gynecology
Cardiology, Gynecology, Nephrology, Neurology and (c) Neurology
Radiology, in the same order. Each person has been (d) Nephrology
allocated to a department as per increasing order of (e) Cardiology
experience with the one in Accident and emergency 43. Person who works for Vasan Healthcare is
(A&E) being the least experienced while the one in experienced than how many persons?
Radiology being the most experienced. (a) 1
Only one person has less experience than Q. The one who (b) 3
has less experience than Q works for Care Hospital. Only (c) 4
one person has more experience than L. The one who is (d) 2
specialized in neurology works for Forties hospital. Only (e) 5

Page 568 of 1334


Subscribe The Xpress Video Course & Mock Test Package for Bank & Insurance Exams
If there are any suggestions/ errors in our PDFs Feel Free to contact us via this email: admin@exampundit.in
Ultra Practice Bundle PDF
SBI Clerk/ RBI Assist. Mains – Reasoning
44. Which of the following hospitals doctor is most Caption but rank above to the one who likes Border. P’s
experience? Rank is above the one who likes MangalPandey. R’s rank
(a) Fortis below P but rank above to the one who likes Border.
(b) Apollo 46. Which if the following Movie Liked by T?
(c) Mittal (a) MangalPndey
(d) Narayana (b) URI
(e) Vasan (c) Fuji
45. How many person are their between the L and the (d) Razzi
person who is specialist of Cardiology? (e) Border
(a) 2 47. Who among the following likes Border?
(b) 3 (a) P
(c) 1 (b) U
(d) 4 (c) T
(e) 5 (d) S
Directions (46-50): Study the following information (e) Cannot be Determined
carefully to answer the given questions: 48. How many persons are junior to the person who
P, Q, R, S, T and U work in Army at different levels. Their like Airlift?
ranks are General, Brigadier, Colonel, Major, Captain and (a) One
Lieutenant. (Note: Rank decreasing on moving from left (b) Two
to right). They like different movies URI, MangalPandey, (c) Three
Border, Raazi, Fauji and Airlift. (d) More than three
Only four persons are Rank lies above Q. Only Three (e) None of these
persons rank below the likes MangalPandey. U does not 49. The one who’s Rank Is Brigadier like which among
hold lowest Rank but Rank below to the one who likes the following Movie?
Airlift. The person who is General rank does not like (a) URI
Airlift. S Rank is not below the one who likes Fauji. T (b) Razzi
does not like URI. R has lower Rank than Colonal. Neither (c) Airlift
U nor P likes Raazi. More than two persons Rank’s below (d) Fauji
the one who likes Raazi. The one who likes Fauji is not (e) None of these

Page 569 of 1334


Subscribe The Xpress Video Course & Mock Test Package for Bank & Insurance Exams
If there are any suggestions/ errors in our PDFs Feel Free to contact us via this email: admin@exampundit.in
Ultra Practice Bundle PDF
SBI Clerk/ RBI Assist. Mains – Reasoning
50. Who among the following is just junior to P? (c) Q
(a) T (d) R
(b) S (e) U

Puzzle Profession- Answer and Explanation


SOLUTION(1-5): 4. The one having third lowest designation wears Brown
Chairman > Managing Director > HOD> Senior shirt.
Manager > Assistant Manager > Executive > Trainee 5. The one who wears Pink shirt is three ranks higher
Explanation in detail: than the one who wears Orange Shirt
1. The one who is an Executive wears Orange shirt. 6. Senior Manager wears Blue shirt
2. U is Chairman and wears Beige Color shirt.
3. T is 2 ranks lower than U.

7. The shirt worn by V is of Grey color.


8. V does not belong to the top three ranks.
9. R whose designation is higher than that of S and he
wears Black shirt.

Page 570 of 1334


Subscribe The Xpress Video Course & Mock Test Package for Bank & Insurance Exams
If there are any suggestions/ errors in our PDFs Feel Free to contact us via this email: admin@exampundit.in
Ultra Practice Bundle PDF
SBI Clerk/ RBI Assist. Mains – Reasoning

10. Neither P nor Q wears Brown shirt 1. d


2. e
3. d
4. b
5. b
SOLUTION(6-10):
Explanation in detail:
1. Q is Principal works at Arihant University and T is the
only other person who works in ArihantUniversity.
2. U is a Lecturer.

11. Only S is left now.


12. Position of P and Q not fixed.
Final Arrangement

Page 571 of 1334


Subscribe The Xpress Video Course & Mock Test Package for Bank & Insurance Exams
If there are any suggestions/ errors in our PDFs Feel Free to contact us via this email: admin@exampundit.in
Ultra Practice Bundle PDF
SBI Clerk/ RBI Assist. Mains – Reasoning
7. R designation is not 4th from bottom

3. P designation is highest among them but does not


work in G.M
4. S works neither at Disha University nor works in the 8. Neither V nor W holds 4th rank designation.
same university as U works at. 9. V is not least rank in designation
5. R and U work in the same University

Final arrangement:

6. Neither the Professor nor the Assistant Professor


works in GM University.
Page 572 of 1334
Subscribe The Xpress Video Course & Mock Test Package for Bank & Insurance Exams
If there are any suggestions/ errors in our PDFs Feel Free to contact us via this email: admin@exampundit.in
Ultra Practice Bundle PDF
SBI Clerk/ RBI Assist. Mains – Reasoning
16 C Civil

14

10

3. H had an experience of more than 20 years but not


the highest.
4. Further it is known that G who had an experience
more than B but less than C and is HOD of Chemical
6. e
department.
7. b
5. B had an experience of odd number of years.
8. d
Case-1
9. a
10. c Experience Person Department

SOLUTION (11-15):
27
Explanation in detail: (fully modified)
1. C who had 16 years of experience works as HOD of 24 H

Civil department.
20 Electrical
2. The person who is HOD of Electrical department had
an experience of 20 years. 16 C Civil

14 G Chemical
Experience Person Department
10
27
8
24
3 B
20 Electrical

Page 573 of 1334


Subscribe The Xpress Video Course & Mock Test Package for Bank & Insurance Exams
If there are any suggestions/ errors in our PDFs Feel Free to contact us via this email: admin@exampundit.in
Ultra Practice Bundle PDF
SBI Clerk/ RBI Assist. Mains – Reasoning
Case-2 3 B

Experience Person Department 6. No one is there between the experience of H and F,


who is not the HOD of Electrical department.
27
7. Four persons are there between the experience of F
24 H and E.
Case-1
20 Electrical
Experience Person Department
16 C Civil

27 F
14
24 H
10 G Chemical
20 Electrical
8

16 C Civil
3 B
14 G Chemical

Case-3 10 E

Experience Person Department 8

27 3 B

24 H
Case-2 (Rejected)
20 Electrical
Four persons are there between the
16 C Civil
experience of F and E.
14
Experience Person Department
10
27 F
8 G Chemical

Page 574 of 1334


Subscribe The Xpress Video Course & Mock Test Package for Bank & Insurance Exams
If there are any suggestions/ errors in our PDFs Feel Free to contact us via this email: admin@exampundit.in
Ultra Practice Bundle PDF
SBI Clerk/ RBI Assist. Mains – Reasoning
24 H Case-1 (Rejected)

20 Electrical A had an experience less than D but


more than G and E.
16 C Civil
Experience Person Department
14
27 F
10 G Chemical
24 H
8
20 D Electrical
3 B
16 C Civil

Case-3 14 G Chemical

Experience Person Department 10 E

27 F 8 A

24 H 3 B

20 Electrical
Case-3
16 C Civil
Experience Person Department
14
27 F
10 E
24 H
8 G Chemical
20 D Electrical
3 B
16 C Civil

8. A had an experience less than D but more than G 14 A

and E.

Page 575 of 1334


Subscribe The Xpress Video Course & Mock Test Package for Bank & Insurance Exams
If there are any suggestions/ errors in our PDFs Feel Free to contact us via this email: admin@exampundit.in
Ultra Practice Bundle PDF
SBI Clerk/ RBI Assist. Mains – Reasoning
10 E
Case-3
8 G Chemical
Experience Person Department
3 B
27 F Mechanical

9. HOD of Mechanical department is immediately 24 H Biotechnology


senior to the HOD of Biotechnology department
20 D Electrical
and difference between their experiences is not
more than three years. 16 C Civil
Case-3
14 A Computer

Experience Person Department


10 E Electronics
27 F Mechanical
8 G Chemical
24 H Biotechnology
3 B IT

20 D Electrical

16 C Civil Final arrangement:


Case-3
14 A
Experience Person Department
10 E
27 F Mechanical
8 G Chemical
24 H Biotechnology
3 B
20 D Electrical

10. HOD of Electronics department is junior to the 16 C Civil

HOD of Computer department and both of their


14 A Computer
experiences are in multiple of 2.
Finally B belongs to HOD of IT

Page 576 of 1334


Subscribe The Xpress Video Course & Mock Test Package for Bank & Insurance Exams
If there are any suggestions/ errors in our PDFs Feel Free to contact us via this email: admin@exampundit.in
Ultra Practice Bundle PDF
SBI Clerk/ RBI Assist. Mains – Reasoning
10 E Electronics

8 G Chemical

3 B IT

11. c
12. d
13. b
14. c
15. e
SOLUTION (16-20):
Explanation in detail:
CASE 1 a
1. Only two persons senior to N
2. CMD is junior to the Person who likes Black color.

CASE 2

3. P is senior to the Person who likes White color who


does not the least junior.
4. P who is junior to MD
For this three case are their
CASE 1
Page 577 of 1334
Subscribe The Xpress Video Course & Mock Test Package for Bank & Insurance Exams
If there are any suggestions/ errors in our PDFs Feel Free to contact us via this email: admin@exampundit.in
Ultra Practice Bundle PDF
SBI Clerk/ RBI Assist. Mains – Reasoning
CASE1a

5. Person who likes Yellow color is senior to Q but junior


to P.
9. The one who likes Yellow color is senior to Q but
6. R is junior to P who is junior to MD.
junior to P.
7. R is senior to Q who likes Blue color.
CASE1
8. M is senior to P but not the senior most
CASE 2 Eliminated
CASE 1

CASE1a

Page 578 of 1334


Subscribe The Xpress Video Course & Mock Test Package for Bank & Insurance Exams
If there are any suggestions/ errors in our PDFs Feel Free to contact us via this email: admin@exampundit.in
Ultra Practice Bundle PDF
SBI Clerk/ RBI Assist. Mains – Reasoning

Final Arrangement

10. O does not like Yellow color.


11. MD is senior to the one who likes Green color.
CASE 1a Eliminated
CASE 1
16. C
17. B
18. D
19. D
20. E

SOLUTION (21-25): (fully modified)


Explanation in detail:
1. Age of the one who is Lecture is not less than
25years.
2. Age of the one who is HOD is just double the age
of the one who is Lecturer.
3. The one who is Chairman is 11years older than the
one who is HOD

Page 579 of 1334


Subscribe The Xpress Video Course & Mock Test Package for Bank & Insurance Exams
If there are any suggestions/ errors in our PDFs Feel Free to contact us via this email: admin@exampundit.in
Ultra Practice Bundle PDF
SBI Clerk/ RBI Assist. Mains – Reasoning
4. The one who is Dean is 5years younger than HOD.
5. Age of the one who is HOD is just double the age
of the one who is Lecturer

6. D is senior to A.
7. More than two persons are junior than A.
8. B is Junior than D but senior than C.
9. C is junior than F, who is not Dean.
10. F is senior to E who is not Lecturer.
11. D is not Professor.
12. F’ age is an odd number.
Brief hints:
D>A
D>B>C (D is not professor)
F>C (F is not Dean)
F>E (E is not lecturer)

13. Age of the one who is Professor is a perfect square


of number and 13years younger than D.
Note: Case-1 & Case-2 gets rejected since D’s age
is 65 years & professor age is 13 years younger

Page 580 of 1334


Subscribe The Xpress Video Course & Mock Test Package for Bank & Insurance Exams
If there are any suggestions/ errors in our PDFs Feel Free to contact us via this email: admin@exampundit.in
Ultra Practice Bundle PDF
SBI Clerk/ RBI Assist. Mains – Reasoning
than D i.e. 65-13=52 years, but it is not a perfect 25. b
square SOLUTION (26-30): (some modified case-2-A added)
Explanation in detail:
1. Weight of M is the lightest.
2. Weight of P is heavier than M.
3. Weight of O is lighter than N but heavier than R.
4. The Weight of the second lightest person is 171cm.
5. Weight of the R is not odd number.
6. Weight of the second Heaviest person is 184cm and O
is not second Heaviest person.
7. Q is not Heaviest person but heavier than P.
14. E is 7years younger than D.
From above information we arrange all the person in
15. B is 9 years older than C.
Decreasing order of their weight.
Note: E’s age is 42 (49-7=42) and B’s age is 36
Final Arrangement

8. Less than three person are senior to O.


9. The one who likes Aajtak is senior to P but junior to
the one who likes ABP channel. M does not like
Aajtak.
10. Weight of the one who is CGM of the company is
lighter than R.
11. M is senior to P.
CASE 1

21. d
22. c
23. E
24. e
Page 581 of 1334
Subscribe The Xpress Video Course & Mock Test Package for Bank & Insurance Exams
If there are any suggestions/ errors in our PDFs Feel Free to contact us via this email: admin@exampundit.in
Ultra Practice Bundle PDF
SBI Clerk/ RBI Assist. Mains – Reasoning

CASE 2

16. The one who likes Aajtak is senior to P but junior to


the one who likes ABP channel.
17. M does not like Aajtak. O does not like Aajtak
18. Weight of the one who likes ABP is less than the one
who is AM
19. R likes Sony Channel
20. M neither likes ZEE nor ABP.

12. R likes Sony Channel and junior to N.


13. R is senior to Q.
14. Weight of the one who is AM is lighter than the one
who is GM.
15. P is not AM in the company.

CASE 1 is Eliminated because O is GM and then


AM must be lighter than GM i.e. either R or P, but
P is not AM and R is senior to Q.
CASE 2

Page 582 of 1334


Subscribe The Xpress Video Course & Mock Test Package for Bank & Insurance Exams
If there are any suggestions/ errors in our PDFs Feel Free to contact us via this email: admin@exampundit.in
Ultra Practice Bundle PDF
SBI Clerk/ RBI Assist. Mains – Reasoning

26. b
27. c
21. The one who likes ZEE is not DGM of the company.
28. a
22. The one who like star is junior to the one who like
29. e
ZEE.
30. E
23. M neither likes ZEE nor ABP
SOLUTION (31-35): (modified)
Case-2-A gets rejected since P must like ZEE, but the one
Explanation in detail:
who like ZEE is not the DGM.
1. The one whose weight is 44 kg is either ACP or API.
CASE 2
2. Only three persons are there in ranks between the one
whose weight is 44 kg and the one whose weight is 25
kg
CASE 1

Final arrangement:

Page 583 of 1334


Subscribe The Xpress Video Course & Mock Test Package for Bank & Insurance Exams
If there are any suggestions/ errors in our PDFs Feel Free to contact us via this email: admin@exampundit.in
Ultra Practice Bundle PDF
SBI Clerk/ RBI Assist. Mains – Reasoning

CASE 2

CASE 2

CASE 2a

3. At least five people junior to K.


4. Only one person is there in ranks between K and the
person whose weight is 41 kg.
CASE 1

Page 584 of 1334


Subscribe The Xpress Video Course & Mock Test Package for Bank & Insurance Exams
If there are any suggestions/ errors in our PDFs Feel Free to contact us via this email: admin@exampundit.in
Ultra Practice Bundle PDF
SBI Clerk/ RBI Assist. Mains – Reasoning
5. The rank of both J and N were lower than the person
whose weight is 41 kg.
6. Only two persons are there in ranks between J and the
one whose weight is 35 kg who is senior to J.
7. M rank is immediately after the one whose weight is
35 kg.
8. Only two persons are there in ranks between M and L.

From here case 1 and case2a eliminated (8th


statement not satisfied)
CASE1 Final Arrangement
11. One of the persons is O.

9. P is just senior to the one who is the second heaviest.


10. N is not the lightest
CASE 1
31. c
32. b
33. b
34. a
35. e
SOLUTION (36-40): (One case added)
Explanation in detail:
Page 585 of 1334
Subscribe The Xpress Video Course & Mock Test Package for Bank & Insurance Exams
If there are any suggestions/ errors in our PDFs Feel Free to contact us via this email: admin@exampundit.in
Ultra Practice Bundle PDF
SBI Clerk/ RBI Assist. Mains – Reasoning
1. Q works in marketing & Sales department.
2. Three department are their between the person who
lives in Bhopal & U who is more experience than the
person who lives in Bhopal.
3. Two person lies between U & T
4. One person lies between T & the person who lives in
Pune
5. W’s is more experience than S but less experience than
T.
CASE 1

CASE 3

CASE 2

Page 586 of 1334


Subscribe The Xpress Video Course & Mock Test Package for Bank & Insurance Exams
If there are any suggestions/ errors in our PDFs Feel Free to contact us via this email: admin@exampundit.in
Ultra Practice Bundle PDF
SBI Clerk/ RBI Assist. Mains – Reasoning

11. Neither U nor Q lives in Pune.


12. Neither the person who lives in Chennai nor the person
who lives in Indore has most experience
13. W’s is more experience than S but less experience than
T.
6. R & P lives in Hyderabad and Mumbai respectively. 14. The person who lives in Indore is more
7. No person lies between V & R. experienced than the person who lives in Chennai
8. T’s is more experience than the person who lives in (newly added)
Raipur CASE 3
9. Three person are their between P & the person who
lives in Raipur
10. V is less experience than P.

From here CASE 1, Case-2 and CASE 4


Eliminated (simultaneously all the above
statements not possible)

CASE 3

Page 587 of 1334


Subscribe The Xpress Video Course & Mock Test Package for Bank & Insurance Exams
If there are any suggestions/ errors in our PDFs Feel Free to contact us via this email: admin@exampundit.in
Ultra Practice Bundle PDF
SBI Clerk/ RBI Assist. Mains – Reasoning
Final arrangement:

6. R works for Narayana health and has more experience


than the one who works for Vasan healthcare.
36. a 7. O has less experience than the one is specialized in
37. b Gynecology, but more experience than the one who
38. e works for Apollo hospital.
39. d
40. c
SOLUTION(41-45):
Explanation in detail:
1. Only one person has less experience than Q.
2. The one who has less experience than Q works for
Care Hospital.
3. Only one person has more experience than L.
4. The one who is specialized in neurology works for
Forties hospital.
5. Only two people have more experience than the one
who works for Vasan healthcare.

Page 588 of 1334


Subscribe The Xpress Video Course & Mock Test Package for Bank & Insurance Exams
If there are any suggestions/ errors in our PDFs Feel Free to contact us via this email: admin@exampundit.in
Ultra Practice Bundle PDF
SBI Clerk/ RBI Assist. Mains – Reasoning
8. P neither has the least experience nor did he specialize 42. c
in Nephrology. 43. c
9. M is not specialized in Nephrology. 44. d
10. The one who works for wockhard hospital not 45. a
specialized of Gynecology. SOLUTION(46-50): (fully mistake now modified
completely)
Explanation in detail:
1. Only four persons are Rank lies above Q.
2. Only Three persons rank below the likes Mangal
Pandey

Final Arrangement

3. P’s Rank is above the one who likes Mangal


Pandey.
4. R has lower Rank than Colonal.
5. R’s rank below P but rank above to the one who
likes Border.

41. e

Page 589 of 1334


Subscribe The Xpress Video Course & Mock Test Package for Bank & Insurance Exams
If there are any suggestions/ errors in our PDFs Feel Free to contact us via this email: admin@exampundit.in
Ultra Practice Bundle PDF
SBI Clerk/ RBI Assist. Mains – Reasoning

6. More than two persons Rank’s below the one who


likes Raazi.
8. The person who is General rank does not like
7. Neither U nor P likes Raazi.
Airlift.
9. U does not hold lowest Rank but Rank below to
the one who likes Airlift.
10. Neither U nor P likes Raazi.

Page 590 of 1334


Subscribe The Xpress Video Course & Mock Test Package for Bank & Insurance Exams
If there are any suggestions/ errors in our PDFs Feel Free to contact us via this email: admin@exampundit.in
Ultra Practice Bundle PDF
SBI Clerk/ RBI Assist. Mains – Reasoning

11. S Rank is not below the one who likes Fauji.


12. The one who likes Fauji is not Caption but rank 46. E
above to the one who likes Border. 47. C
13. T does not like URI. 48. d
49. c
Final Arrangement 50. e

Download Puzzles Practice Questions PDF


Get More Reasoning Practice Questions PDF
Puzzle Timing Based
Directions (1-5): Study the following information and TruJet Airlines is the earliest to depart. Its departure time
Answer the questions below: is not after 8 a.m. and its travel time is two hours less than
Seven Airlines – Indigo, Air India, Tru Jet, Go Air, Spice the travel time of Vistara Airlines. Since Indigo Airlines
Jet, Vistara and Air Asia depart from Mumbai Airport on has to cover the least distance, it leaves last, but before 8
Monday. Each flight departure at different times 5 AM, 8 p.m. and still completes its journey at 10p.m. Go Air
AM, 9 AM, 11 AM, 2 PM, 4 PM and 7 PM not necessarily Airline leaves Mumbai Airport at Monday 11 a.m. and
in the same order. takes 19 hours to reach its destination. There is a four hour
gap between the departure times of Spice Jet, leaving at 9

Page 591 of 1334


Subscribe The Xpress Video Course & Mock Test Package for Bank & Insurance Exams
If there are any suggestions/ errors in our PDFs Feel Free to contact us via this email: admin@exampundit.in
Ultra Practice Bundle PDF
SBI Clerk/ RBI Assist. Mains – Reasoning
a.m. and Tru Jet Airline. Spice Jet Airline takes one hour (e) Indigo Airline
more than the travel time of Tru Jet Airline to reach its 4. What is the difference between the departure
destination. Indigo Airline leaves five hours after Air timing of Go air and Indigo Airlines?
India Airlines departs but takes only three hours to reach (a) 2 hours
its destination. Vistara Airlines and Air Asia Airlines take (b) 4 hours
9 hours and 7 hours to reach their respective destinations. (c) 6 hours
Vistara Airlines leaves at 4 p.m. and there is gap of 8 hours (d) 8 hours
between the departure times of Vistara Airlines and Air (e) 10 hours
Asia Airlines. One flight reaches its destination at 7 p.m. 5. Which of the following statement is true as per
1. Which of the following flights reach the destination departure time and Arrival time at Destination in
on Tuesday? the given information?
(a) Indigo airline (a) Spice Jet Airlines - 9:00 AM - 4:00 PM
(b) Air India Air line (b) Go Air Airlines - 11:00 AM - 6:00 AM
(c) Both Go Air and Vistara Airline (c) Tru Jet Airlines - 4:00 AM - 1:00 PM
(d) Air Asia Airline (d) VistaraAirlines- 5:00 PM - 2:00 AM
(e) Tru Jet Airline (e) Air Asia Airlines- 8:00 AM - 12:00 noon
2. How many flights departure between Air India and Directions (6-10): Study the following information and
Vistara Airline? Answer the questions below:
(a) None P, Q, R, S, T, U and V are seven friends who study in
(b) One Exam Pundit at three different cities namely Chennai,
(c) Two Hyderabad and Visakhapatnam such that not less than two
(d) Three friends study in the same city. Each friend also has a
(e) Four different favorite’s subject viz. English, Reasoning,
3. Which of the following flight departure last on Polity, Maths, General Awareness, Geography and
Monday? Science but not necessarily in the same order. Each of the
(a) Spice Jet Airline students attends the classes of his favorite’s subjects at
(b) Air India Airline different time.
(c) Go Air Airline Q attends his classes at 11 AM. U attends his classes at 8
(d) Tru jet Airline AM. P likes Maths and studies in the Chennai with only

Page 592 of 1334


Subscribe The Xpress Video Course & Mock Test Package for Bank & Insurance Exams
If there are any suggestions/ errors in our PDFs Feel Free to contact us via this email: admin@exampundit.in
Ultra Practice Bundle PDF
SBI Clerk/ RBI Assist. Mains – Reasoning
one other friend who likes General Awareness. V studies (e) None of these
with two other friends. There is only one hour gap 8. Who amongst the following studies in the Chennai
between the time of class of the persons study at City?
Hyderabad and the one whose favorite’s subject is (a) Q
General Awareness.. S neither studies in Chennai nor (b) V
Visakhapatnam but studies with only one person and does (c) U
not like Geography. The one whose favorite’s subject is (d) S
Science takes his class at 9 AM. V attends his class at 9 (e) Both P and R
‘O clock. The persons whose favorite’s subjects are 9. Which of the following combinations is definitely
Science and Polity do not have their classes at the same correct?
time. Both the friends who study with V like Reasoning (a) V and Reasoning
and English. The one who likes Polity neither study in (b) Q and English
Chennai nor Hyderabad. T does not like General (c) P and G.A.
Awareness subject. R does not like English, Reasoning or (d) T and Reasoning
Geography. P and the one whose favorite subject is (e) S and Science
Geography have their class at 1 PM. R attends his class 10. Which of the following subjects does U like?
just after S. T studies with only one friend. (a) Either Maths or Reasoning
6. Which combination represents T’s favorite subject (b) Either Reasoning or English
and the city in which he studies? (c) Either Reasoning or Science
(a) Mathsand Chennai (d) Either Reasoning or G.A
(b) Science and Hyderabad (e) Either Civics or Geography
(c) Polity and Visakhapatnam Directions (11-15): Study the following information
(d) Geography and Hyderabad and Answer the questions below:
(e) Reasoning and Hyderabad Six lecturers – B, A, S, K, R and M will visit a college
7. Q attends his class at? from 12th November to 18th November to give lectures
(a) 11 AM but not necessarily as the same order. There is a Sunday
(b) 8 AM in between and no visit is scheduled on Sunday. Each of
(c) 10 AM the lectures has different time duration: 30 mins. 45 mins,
(d) 9 AM

Page 593 of 1334


Subscribe The Xpress Video Course & Mock Test Package for Bank & Insurance Exams
If there are any suggestions/ errors in our PDFs Feel Free to contact us via this email: admin@exampundit.in
Ultra Practice Bundle PDF
SBI Clerk/ RBI Assist. Mains – Reasoning
60 mins, 75 mins, 100 mins and 120 mins. Again not (d) 60
necessary as the same order. (e) 45
18th November is not Sunday and lecture of 30 mins is 14. Which of the following Combination is true?
scheduled on that day. B's lecture is for less than 60 mins (a) S -45min
and is scheduled immediately before R's lecture. There are (b) R -120min
two lectures Scheduled between M's lecture which is for (c) K - 30min
120 mins and S's lecture which is for 60 mins. R's lecture (d) A - 30 min
is before Sunday and there are two days between Sunday (e) B- 60min
and B's lecture. K's lecture which is for 75 mins is not 15. How many lectures are scheduled after the one who
scheduled as 12th November. The lecture scheduled on give lectures on Thursday?
Saturdays is of 120 mins. (a) 1
11. On which date no lecturers Schedule? (b) 2
(a) 12 (c) 3
(b) 13 (d) 4
(c) 14 (e) 5
(d) 15 Directions (16-20): Study the following information
(e) 16 carefully and answer the given questions below.
12. How many lectures are scheduled between Seven films A, B, C, D, E, F and G are screened in
Thursday and Monday? different time slots/Sessions. The timing can be of 40
(a) 3 minutes, 55 minutes or one hour ten minutes. There is no
(b) 2 gap between two sessions; it means if session 1 ended at
(c) 4 12.10pm then session 2 starts at 12.10pm and so on. Each
(d) 1 film is screened at different theatres I, J, K, L, M, N and
(e) None O on the same building but not necessarily in the same
13. On Friday how many minutes lecture are order.
scheduled? In theatre N, the film was screened at 1:40PM. E and the
(a) 100 film which was screened in theatre N had consecutive
(b) 120 slots. The last session ended at 5:15PM. C and the film
(c) 75 which was screened at theatre L had sessions of

Page 594 of 1334


Subscribe The Xpress Video Course & Mock Test Package for Bank & Insurance Exams
If there are any suggestions/ errors in our PDFs Feel Free to contact us via this email: admin@exampundit.in
Ultra Practice Bundle PDF
SBI Clerk/ RBI Assist. Mains – Reasoning
55minutes. The film which was screened in theatre M took (e) All are correct
its session immediately after a session of 40minutes. The 19. Four of the five among the following are similar in
film F was screened at theatre O. There was one film such a way to form a group, which one of the
screened between F and G. F is screened before G. The following doesn’t belong to the group?
film screened in theatre K was started at 12.05PM. The (a) B – N
film B was screened immediately after the film C. The (b) E- O
film screened in Theatre I had its session immediately (c) C – N
after D’s session. Number of sessions after D’s session (d) A- L
was twice as that of the number of sessions after E’s (e) F- J
session. E was screened in Theatre J. A was screened at 20. Time Duration of how many theatres are 55mins?
3.40pm. (a) 1
16. Which of the following films was screened at (b) 2
theatre L? (c) 3
(a) B (d) 4
(b) E (e) 5
(c) F Directions (21-25): Study the following information
(d) A carefully and answer the given questions below:
(e) G (One statement modified)
17. Film E was screened between _____? There are five Boxers F, G, H, I and J who are fighting
(a) 1.00pm – 2.20pm with five Opponent viz. V, W, X, Y and Z but not
(b) 4.20pm – 5.15pm necessarily in the same order. Each fight is of one hour
(c) 3.40pm-4.20pm duration. No break after each fight and all the fight are in
(d) 2.20pm-3.00pm continuous manner. All five fights are taking place at
(e) 3.00pm-3.30pm different cities of India viz. Delhi, Mumbai, Chennai,
18. Which of the following is correct? Hyderabad and Ahmedabad. No Female boxer fights with
(a) Three films are screened between D and G a male boxer.
(b) Film G screened at theatre M Among Boxer only three are female are- The one who
(c) Total time duration of film A is 40minutes fights with W, The one whose fight is at 8 PM and the one
(d) Flim B screened immediately after film E whose fight is in Hyderabad. The female whose fight is at

Page 595 of 1334


Subscribe The Xpress Video Course & Mock Test Package for Bank & Insurance Exams
If there are any suggestions/ errors in our PDFs Feel Free to contact us via this email: admin@exampundit.in
Ultra Practice Bundle PDF
SBI Clerk/ RBI Assist. Mains – Reasoning
10 PM fights in Chennai. One of fights is at 6PM, but not (b) Chennai
takes place in Mumbai. X’s fight is neither in Mumbai nor (c) Delhi
in Hyderabad. Y does not fight in Mumbai. The Boxer (d) Mumbai
who has a fight with Y is a female but she is not I. J fights (e) Ahmadabad
at 10 PM. F and H is not females. F’s fight is at 9 PM and 25. How many fights is/are there after fight between F
fights neither with X nor with the one whose fight is in and Z?
Mumbai. G fights at 7 PM. V’s fight is in Delhi and he (a) 2
does not fight with F. (b) 1
21. What is the Gender of X and X is opponent of (c) 3
which boxer? (d) 4
(a) Male, H (e) 5
(b) Male, F Directions (26-30): Study the following information
(c) Female, J carefully and answer the given questions below:
(d) Female , I There are Seven Lecturers – P, Q, R, S, T, U and V taught
(e) None of these seven subjects, viz., Reasoning, History, Geography,
22. At what time fight between G and Y started? Polity, Science, General Awareness and Hindi on one day
(a) 7pm in a week starting from Monday and ending on Sunday (of
(b) 8pm the same week). There will be separate timings for each
(c) 9pm lecture.
(d) 10pm Note: Total hours taken by all the lecturers is 18 hours.
(e) None of these The minimum and maximum timing of any lecture will be
23. The Boxer who Fight just after H is? one hour and five hour respectively. There are two pairs
(a) J of timings that can be followed by four lecturers.
(b) I Polity is taught on Thursday. General Awareness is
(c) F neither taught on Tuesday nor on Saturday. The
(d) None of these Geography professor gave lecture immediately after the
(e) G lecturer P. Q is not a Polity Professor. Reasoning is taught
24. The fight between I and W is on which place? for one hour. The Professor Q gave his lecture on one of
(a) Hyderabad the days before Friday. Neither the professor T nor the

Page 596 of 1334


Subscribe The Xpress Video Course & Mock Test Package for Bank & Insurance Exams
If there are any suggestions/ errors in our PDFs Feel Free to contact us via this email: admin@exampundit.in
Ultra Practice Bundle PDF
SBI Clerk/ RBI Assist. Mains – Reasoning
professor U gave his lecture on Sunday. Professor U gave (e) General Awareness – Sunday
his lecture immediately after T. Lecturer P spent more 28. If all the persons are made to arrange in
time than Lecturer R. Time taken by lecturer R is the sum alphabetical order from Monday to Sunday,
of time taken by the lecturers Q and U. Subjects positions of how many persons will remain
Reasoning & History are taught for same duration. The unchanged?
lecturer who took maximum time is immediately preceded (a) Four
by the person who took less than one hour of maximum (b) Three
time. The difference between the subjects taught on Friday (c) None
and Sunday is equaled to the time taken by the lecturer P. (d) Two
Professor who gave Reasoning lecture immediately (e) One
preceded and followed by R and V respectively. Professor 29. Professor S gave lecture for how many hours?
who gave lecture on Sunday spent less than three hours. (a) One hour
Only one lecture is held between Polity and Geography. (b) Three hours
History is taught after two days of reasoning lecture. hindi (c) Four hours
is neither taught on Monday nor Sunday. Geography is not (d) Two hours
taught on the immediate next day on which History is (e) None of the given options is true
taught. Science is taught on Monday. 30. Who among the following gave lecture immediately
26. Which of the following Subject is taught by P ? after U?
(a) Science (a) T
(b) Geography (b) Q
(c) Reasoning (c) S
(d) Hindi (d) V
(e) Polity (e) R
27. Which of the following combinations is True with Directions (31-35): Study the following information
respect to the given arrangement? carefully and answer the given questions below:
(a) History – Wednesday Seven Exams are to be scheduled by a College on different
(b) Science – Friday timings on different days. The Subjects are History,
(c) Polity – Wednesday Maths, English, Hindi, Economics, Science and Social
(d) History – Friday Science were to be schedule on 2 PM, 12 PM, 5 PM, 7

Page 597 of 1334


Subscribe The Xpress Video Course & Mock Test Package for Bank & Insurance Exams
If there are any suggestions/ errors in our PDFs Feel Free to contact us via this email: admin@exampundit.in
Ultra Practice Bundle PDF
SBI Clerk/ RBI Assist. Mains – Reasoning
PM, 10 PM, 6 PM and 4 PM not necessarily in the same (e) None of these
order and the exam started in a week from Monday and 33. In which of the following time Science was
ending on Sunday. scheduled?
Economics was scheduled on 7PM but after Thursday. (a) 4 PM
More than two exams were scheduled between Economics (b) 2 PM
and Maths which is not scheduled at 5PM. Number of (c) 5 PM
Exam between English and Science was same as between (d) 6 PM
Science and Social Science. Social Science was schedules (e) 12 PM
after science. Science was neither scheduled on 10PM nor 34. If 5 PM is related to 6 PM and English is related to
12PM. The exam scheduled on Monday was scheduled on Hindi in a certain way, then to which of the
5PM. Only two exams were scheduled between Social following would 2 PM be related to, following the
Science and the exam scheduled on 2PM. English exam same pattern?
was not scheduled in 5PM. Exam in 12 PM was scheduled (a) 10 PM
immediately before the exam which scheduled on 10PM. (b) 4 PM
Hindi was not scheduled on Monday. Exam scheduled on (c) 12 PM
4 PM was not scheduled after exam on 2PM. (d) 6 PM
31. How many Days Gap between History and Social (e) 7 PM
Science exams? 35. Which of the following exam was held on Tuesday?
(a) Four (a) Economics
(b) One (b) History
(c) Two (c) Social Science
(d) Three (d) Science
(e) Five (e) English
32. Which exam was scheduled just next day after Directions (36-40): Study the following information
Hindi exam? carefully and answer the given questions below (one
(a) Economics statement added)
(b) History Ten students namely viz P, Q, R, S, T, U, V, W X and Y
(c) Social Science of ten different colleges but not necessarily in the same
(d) Science order have seminar on five different days starting from

Page 598 of 1334


Subscribe The Xpress Video Course & Mock Test Package for Bank & Insurance Exams
If there are any suggestions/ errors in our PDFs Feel Free to contact us via this email: admin@exampundit.in
Ultra Practice Bundle PDF
SBI Clerk/ RBI Assist. Mains – Reasoning
Monday to Friday of the same week. Each student have (c) Thursday, 1pm
seminar at two different time slots, i.e 10.00 AM and 1 (d) Monday, 10am
P.M. Each student likes different Fruits namely viz – (e) Tuesday, 10am
Apple, Banana, Orange, Grape, Pineapple, Cranberry, 37. In which of the following Day R Gives his seminar?
Strawberry, Kiwi, Peach and Watermelon. (a) Monday
Persons who like Kiwi, Apple, Grape, Cranberry and (b) Tuesday
Peach have seminar at 10:00 A.M. Persons who like (c) Wednesday
Strawberry, Banana, Orange, Pineapple and Watermelon (d) Friday
have seminar at 1:00 P.M. W has a seminar on Tuesday. (e) Thursday
The number of people who have seminar between V and 38. How many person gives seminar between W and
S is same as the number of people who have seminar P?
between R and Y. Persons who like Kiwi and Pineapple (a) Four
have seminar on Monday and the persons who like Peach (b) Three
and Watermelon have Seminar on Friday. Q has a seminar (c) Five
immediately before W. W does not have seminar on any (d) Two
of the days before V. Y does not has seminar at 1 P.M. S 39. Which of the following Person and Days
has a seminar immediately after the day of one who has combinations is true with respect to the given
seminar on Monday. U does not have seminar at 1 P.M. S arrangement?
does not has seminar on any one of the days after T. Only (a) P – Wednesday
three people have seminar between V and T. Persons who (b) V – Friday
like Grape and Orange went to seminar neither on (c) Q – Wednesday
Tuesday nor Thursday.Only two people have seminar (d) S – Friday
between U and X. Persons who like Apple and Banana (e) T – Wednesday
does not have seminar on Thursday. U doesn’t like 40. Which of the following Student Give seminar just
Cranberry. after X.
36. In which of the following day and time P gives (a) P
Seminar? (b) Y
(a) Friday, 10am (c) R
(b) Thursday, 10am (d) S

Page 599 of 1334


Subscribe The Xpress Video Course & Mock Test Package for Bank & Insurance Exams
If there are any suggestions/ errors in our PDFs Feel Free to contact us via this email: admin@exampundit.in
Ultra Practice Bundle PDF
SBI Clerk/ RBI Assist. Mains – Reasoning
(e) T 42. How many grooming session are there between J
Directions (41-45): Study the following information and O?
carefully and answer the given questions below (a) 3
Eight persons I, J, K, L, M, N, O and P visited a Saloon (b) 5
for grooming in eight consecutive slots. The slots were of (c) 4
30min, 45min or 1hour. There is no gap between two (d) 2
Consecutive Slots. It means if session 1 ended at 10:00am (e) None
then session 2 starts at 10.00am. The slot started at 43. How many grooming session is/are of 30min?
11:00am. (a) 2
Number of persons who went for the session before K was (b) 1
thrice as that of the number of person who went after the (c) 3
session that ended at 4:15pm. K did not go in the 1st (d) 4
session. There were 3 sessions between P’s session and (e) 5
M’s session. P had his session before M. K’s session 44. At what time last grooming session ends?
started at 1:15pm. Number of sessions before J’s session (a) 4:30pm
was equal to the number of sessions after L’s session. J (b) 5:15pm
had his session before L. O and L had equal slot duration. (c) 5:00pm
N and I did not have the equal duration slots. O’s slot (d) 4:45pm
started at 3:00pm or 3:30pm. K and I had consecutive (e) None of these
sessions. M had a session of 30minutes. There were two 45. Which of the following persons grooming
sessions between M’s and N’s session. P’s session ended immediately before and after M?
at 12:15pm. The first Session was of 45minutes. (a) J, N
41. At what time I’s session was started? (b) N, K
(a) 12:15pm (c) J, P
(b) 1:15pm (d) I, O
(c) 4:15pm (e) K, I
(d) 3:30pm Directions (46-50): Study the following information
(e) 2:15pm carefully and answer the given questions below

Page 600 of 1334


Subscribe The Xpress Video Course & Mock Test Package for Bank & Insurance Exams
If there are any suggestions/ errors in our PDFs Feel Free to contact us via this email: admin@exampundit.in
Ultra Practice Bundle PDF
SBI Clerk/ RBI Assist. Mains – Reasoning
In a sports meet, different teams – Arsenal, Chelsea, 46. At what time and day Team Porto play game?
Liverpool, Manchester United, AC Milan,Inter Milan, La (a) 4 PM, Monday
Liga, Real Madrid,Barcelona,Porto, Kerala Blasters and (b) 1 PM, Monday
Flamingo play different games – Athletics, Baseball, (c) 1 PM, Sunday
Boxing, Cricket, Cycling, Diving, Golf, Handball, (d) 1 PM, Wednesday
Hockey, Judo, Rowing and Shooting in different time (e) 1 PM, Friday
slots – 9:00 am, 1:00 pm and 4:00 pm on different days of 47. Which of the following game play at 9 am Sunday?
the week – Monday, Wednesday, Friday and Sunday. (a) Golf
Diving is played before Shooting. Team La Liga plays (b) Cycling
Golf. Rowing is played on the same day on which (c) Diving
Handball is organized. Athletics is played by neither (d) Baseball
Liverpool nor Porto team. Team Real Madrid plays (e) Judo
Boxing at 1:00pm. Only 2 teams play between Manchester 48. How many games played between team Real Madrid
United and Kerala Blaster teams. There are 3 teamsplay and Flamingo?
between Flamingo and Manchester United team. Team (a) 4
Flamingo does not play Hockey. Baseball is played by AC (b) 2
Milan team. Hockey is played in the last slot of last day of (c) 1
the week. The number of games played between Judo and (d) 6
Hand Ball is equivalent to the number of games played (e) 5
between Baseball and Cycling. Both Judo and Hand ball 49. Which of the following team play his game on Friday
are played before Cricket. Athletics is played immediately at 9 am?
before Rowing. Team Arsenal plays Judo. Team (a) AC Milan
Barcelona plays Cricket on Wednesday in the third slot. (b) La Liga
The number of teams playing before the Handball is (c) Chelsea
equivalent to the number of teams playing after Team (d) Manchester United
Flamingo. Cycling is played by team Chelsea immediately (e) Liverpool
before Hockey. The number of games played before 50. Which of the following Time and Game combinations
Cricket is equivalent to the number of games played after is true with respect to the given arrangement?
Baseball.Team Liverpool does not play Hand ball. (a) 4 PM - Baseball

Page 601 of 1334


Subscribe The Xpress Video Course & Mock Test Package for Bank & Insurance Exams
If there are any suggestions/ errors in our PDFs Feel Free to contact us via this email: admin@exampundit.in
Ultra Practice Bundle PDF
SBI Clerk/ RBI Assist. Mains – Reasoning
(b) 4 PM - Cycling (d) 1 PM - Judo
(c) 1 PM – Hand ball (e) 1 PM - Shooting

Puzzle Timing Based – Answer and Explanation


Solution (1 - 5):
1. TruJet Airlines is the earliest to depart & its departure
time is not after 8 a.m. (here time is given of departure
earliest means 5am)
2. Indigo Airlines has to cover the least distance, it leaves
last, but before 8 p.m. and still completes its journey
at 10p.m.(indgo leaves last means its departure time is
7pm and travel time is 3 hour) 5. Indigo Airline leaves five hours after Air India
Airlines departs.
6. Vistara Airlines and Air Asia Airlines take 9 hours and
7 hours to reach their respective destinations.
7. Vistara Airlines leaves at 4 p.m. and there is gap of 8
hours between the departure times of Vistara Airlines
and Air Asia Airlines.

3. Go Air Airline leaves Mumbai Airport at Monday


11a.m. and takes 19 hours to reach its destination.(Go
Air reaches his destination on 6am Tuesday )
4. There is a four hour gap between the departure times
of Spice Jet, leaving at 9 a.m. and Tru Jet Airline.

8. Tru Jet travel time is two hours less than the travel
time of Vistara Airlines.
9. Spice Jet Airline takes one hour more than the travel
time of Tru Jet Airline to reach its destination

Page 602 of 1334


Subscribe The Xpress Video Course & Mock Test Package for Bank & Insurance Exams
If there are any suggestions/ errors in our PDFs Feel Free to contact us via this email: admin@exampundit.in
Ultra Practice Bundle PDF
SBI Clerk/ RBI Assist. Mains – Reasoning

5. V studies with two other friends.


6. T studies with only one friend.
7. T does not like General Awareness subject.
10. One flight reaches its destination at 7 p.m. (Air india
8. P likes Maths and studies in the Chennai with only one
reaches at 7pm )
other friend who likes General Awareness
Final Arrangement
9. S studies neither in Chennai nor Visakhapatnam but
studies with only one person and does not like
Geography.
10. V studies with two other friends.(Means V is from
Vishakhapatnam)
11. V attends his class at 9 ‘O clock
12. R does not like English, Reasoning or Geography.
From 8th and 9th point it is clear that number of Person
1.c from Chennai and Hyderabad has only 2 persons and T
2.a studies with only one friend so he is from Hyderabad.
3.e
4.d
5.b
Solution (6 - 10):
1. Q attends his classes at 11 AM.
2. U attends his classes at 8 AM.
3. P likes Maths and studies in the Chennai
4. S neither studies in Chennai nor Visakhapatnam.
Page 603 of 1334
Subscribe The Xpress Video Course & Mock Test Package for Bank & Insurance Exams
If there are any suggestions/ errors in our PDFs Feel Free to contact us via this email: admin@exampundit.in
Ultra Practice Bundle PDF
SBI Clerk/ RBI Assist. Mains – Reasoning
13. Both the friends who study with V like Reasoning
and English( Means They both belongs to
Visakhapatnam which is Q and U)
Hints explanation: R doesn’t like English, & Reasoning.
This implies R is not from Vishakhapatnam. T studies
with only one friend. This implies T is not from
Vishakhapatnam
Thus Q and U are from Vishakhapatnam (they like
Reasoning and English in any order) 20. R attends his class just after S.

14. The one who likes Polity neither study in Chennai nor Final Arrangement
Hyderabad.

6.d
7.a
15. P likes Maths and studies in the Chennai with only one
8.e
other friend who likes General Awareness.(R like
9.e
G.A. and he study at Chennai because minimum two
10.b
friends study in any of the city)
Solution (11 - 15):
16. P and the one whose favorite subject is Geography
1. 18th November is not Sunday and lecture of 30 mins is
have their class at 1 PM
scheduled on that day.
17. S does not like Geography.
2. B's lecture is for less than 60 mins and is scheduled
18. The one whose favorite’s subject is Science takes his
immediately before R's lecture.( B = 30mins or 45
class at 9 AM
mins )
19. The persons whose favorite’s subjects are Science and
3. There are two days between Sunday and B's lecture.
Polity do not have their classes at the same time.
Page 604 of 1334
Subscribe The Xpress Video Course & Mock Test Package for Bank & Insurance Exams
If there are any suggestions/ errors in our PDFs Feel Free to contact us via this email: admin@exampundit.in
Ultra Practice Bundle PDF
SBI Clerk/ RBI Assist. Mains – Reasoning
4. R's lecture is before Sunday 5. There are two lectures Scheduled between M's lecture
which is for 120 mins and S's lecture which is for 60
mins
6. The lecture scheduled on Saturdays is of 120 mins.
.
CASE 3 eliminated because 5 point is not possible

7. K's lecture which is for 75 mins is not scheduled as


12th November.
From here CASE 1 eliminated because we cannot
place K at 12.

Page 605 of 1334


Subscribe The Xpress Video Course & Mock Test Package for Bank & Insurance Exams
If there are any suggestions/ errors in our PDFs Feel Free to contact us via this email: admin@exampundit.in
Ultra Practice Bundle PDF
SBI Clerk/ RBI Assist. Mains – Reasoning
3. E was screened in Theatre J.
4. In theatre N, the film was screened at 1:40PM.
5. E and the film which was screened in theatre N had
consecutive slots.

Now only A and 100min is there


Final Arrangement

11.e
12.b
13.a
14.d
15.d
Solution (16- 20):
In this type of question we fix the all slot/ session from 1
to 7 \
1. The film screened in Theatre I had its session
immediately after D’s session.
2. Number of sessions after D’s session was twice as that
of the number of sessions after E’s session.
Page 606 of 1334
Subscribe The Xpress Video Course & Mock Test Package for Bank & Insurance Exams
If there are any suggestions/ errors in our PDFs Feel Free to contact us via this email: admin@exampundit.in
Ultra Practice Bundle PDF
SBI Clerk/ RBI Assist. Mains – Reasoning

6. The film F was screened at theatre O.


7. There was one film screened between F and G.
8. F is screened before G.
9. The film B was screened immediately after the film C.
10. C had sessions of 55minutes.

13. The film screened in theatre K was started at


12.05PM.(in theatre K film D was screened)

11. The last session ended at 5:15PM


14. L had sessions of 55minutes.
12. A was screened at 3:40pm.
15. The film which was screened in theatre M took its
session immediately after a session of 40minutes.
Page 607 of 1334
Subscribe The Xpress Video Course & Mock Test Package for Bank & Insurance Exams
If there are any suggestions/ errors in our PDFs Feel Free to contact us via this email: admin@exampundit.in
Ultra Practice Bundle PDF
SBI Clerk/ RBI Assist. Mains – Reasoning
Here for Theatre M only 1 possibility is there i.e. Film A 18.c
and also we find the duration of theatre M. 19.a (Film and Theatre combination)
20.b
Solution (21- 25):

1. J fights at 10 PM.
2. F’s fight is at 9 PM
From here we can find the time slot of theatre O.
3. G fights at 7 PM
4. F and H is not females

5. The one whose fight is at 8 PM is female. so I is female


6. One of fights is at 6PM, but not takes place in
Mumbai. (From here it is clear that one of the fight is

Final Arrangement at 6pm so H’s fight is at 6pm)


7. The female whose fight is at 10 PM fights in Chennai.

8. F’s fight is at 9 PM and fights neither with X nor with


16.e the one whose fight is in Mumbai.
17.d 9. V’s fight is in Delhi and he does not fight with F.
Page 608 of 1334
Subscribe The Xpress Video Course & Mock Test Package for Bank & Insurance Exams
If there are any suggestions/ errors in our PDFs Feel Free to contact us via this email: admin@exampundit.in
Ultra Practice Bundle PDF
SBI Clerk/ RBI Assist. Mains – Reasoning
10. The Opponent whose fight is in Mumbai does not fight
at 6 PM(Two case for Mumbai either G or I)
From Above points it is clear that F’s Fight is at
Ahmadabad. Because Hyderabad is not possible F is male.

15. F not Fight with X.(F is male so he cannot fight with


W also so he F fight with Z )
16. X’s fight is neither in Mumbai nor in Hyderabad.
17. The one who fights with W is female.
In case 2 we can fixed Y so one more case is there

11. The one whose fight is in Hyderabad is female.


12. Y does not fight in Mumbai.
13. The Boxer who has a fight with Y is a female but she
is not I.
14. V’s fight is in Delhi.

Here CASE 1 and CASE 2b is eliminated because X is not


from Hyderabad.
Final Arrangement
Page 609 of 1334
Subscribe The Xpress Video Course & Mock Test Package for Bank & Insurance Exams
If there are any suggestions/ errors in our PDFs Feel Free to contact us via this email: admin@exampundit.in
Ultra Practice Bundle PDF
SBI Clerk/ RBI Assist. Mains – Reasoning

21.c
22.a
23.e
5. History is taught after two days of reasoning lecture.
24.d
6. Geography is not taught on the immediate next day on
25.b
which History is taught.
Solution (26- 30): 7. Reasoning is taught for one hour.
1. Polity is taught on Thursday
2. Only one lecture is held between Polity and
Geography.
3. The Geography professor gave lecture immediately
after the lecturer P
4. Science is taught on Monday.
Here two case for Geography.

8. Professor who gave Reasoning lecture immediately


preceded and followed by R and V respectively.

Page 610 of 1334


Subscribe The Xpress Video Course & Mock Test Package for Bank & Insurance Exams
If there are any suggestions/ errors in our PDFs Feel Free to contact us via this email: admin@exampundit.in
Ultra Practice Bundle PDF
SBI Clerk/ RBI Assist. Mains – Reasoning
9. Q is not a Polity Professor. 18. The lecturer who took maximum time is immediately
10. The Professor Q gave his lecture on one of the days preceded by the person who took less than one hour of
before Friday maximum time.(maximum is 5 and one less then
11. Subjects Reasoning & History are taught for same maximum is 4)
duration 19. The difference between the subjects taught on Friday
and Sunday is equaled to the time taken by the lecturer
P.

12. Either the professor T or the professor U not gave his


lecture on Sunday.
13. Professor U gave his lecture immediately after T
14. Hindi is neither taught on Monday nor Sunday.
20. Total hours taken by all the lecturers is 18 hours.
15. General Awareness is neither taught on Tuesday nor
21. Professor who gave lecture on Sunday spent less than
on Saturday.
three hours
16. Time taken by lecturer R is the sum of time taken by
Here total hour is 18 so we have to place 2hour at Sunday.
the lecturers Q and U. (Q =1hr, U= 1hr then R = 2hr)
Final Arrangement

17. Lecturer P spent more time than Lecturer R.


26.a
Page 611 of 1334
Subscribe The Xpress Video Course & Mock Test Package for Bank & Insurance Exams
If there are any suggestions/ errors in our PDFs Feel Free to contact us via this email: admin@exampundit.in
Ultra Practice Bundle PDF
SBI Clerk/ RBI Assist. Mains – Reasoning
27.e (Subject and Day’s)
28.b
29.d
30.c
Solution (31- 35):
1. The exam scheduled on Monday was scheduled on
5PM
2. Economics was scheduled on 7PM but after Thursday.
3. More than two exams were scheduled between
Economics and Maths which is not scheduled at 5PM.

4. Only two exams were scheduled between Social


Science and the exam scheduled on 2PM.
5. Number of Exam between English and Science was
same as between Science and Social Science.
6. Social Science was schedules after science.
7. English exam was not scheduled in 5PM.
8. Exam scheduled on 4 PM was not scheduled after
exam on 2PM.

Page 612 of 1334


Subscribe The Xpress Video Course & Mock Test Package for Bank & Insurance Exams
If there are any suggestions/ errors in our PDFs Feel Free to contact us via this email: admin@exampundit.in
Ultra Practice Bundle PDF
SBI Clerk/ RBI Assist. Mains – Reasoning

9. Hindi was not scheduled on Monday.


10. Exam in 12 PM was scheduled immediately before the
exam which scheduled on 10AM.
11. Science was neither scheduled on 10AM nor 12PM.

Page 613 of 1334


Subscribe The Xpress Video Course & Mock Test Package for Bank & Insurance Exams
If there are any suggestions/ errors in our PDFs Feel Free to contact us via this email: admin@exampundit.in
Ultra Practice Bundle PDF
SBI Clerk/ RBI Assist. Mains – Reasoning

31.a
32.c
33.d
34.a
35.e
Solution (36- 40):
1. W has a seminar on Tuesday.
2. Q has a seminar immediately before W.
3. W does not have seminar on any of the days before V.

Final Arrangement

Page 614 of 1334


Subscribe The Xpress Video Course & Mock Test Package for Bank & Insurance Exams
If there are any suggestions/ errors in our PDFs Feel Free to contact us via this email: admin@exampundit.in
Ultra Practice Bundle PDF
SBI Clerk/ RBI Assist. Mains – Reasoning

4. Only three people have seminar between V and T.


5. S does not have seminar on any one of the days after
T.
6. S has a seminar immediately after the day of one who
has seminar on Monday.

7. Y does not have seminar at 1 P.M.

Page 615 of 1334


Subscribe The Xpress Video Course & Mock Test Package for Bank & Insurance Exams
If there are any suggestions/ errors in our PDFs Feel Free to contact us via this email: admin@exampundit.in
Ultra Practice Bundle PDF
SBI Clerk/ RBI Assist. Mains – Reasoning
8. The one who has seminar at 10.00 A.M. Immediately 12. Persons who like Kiwi and Pineapple have seminar
before X (not given in summary of the puzzle, actually on Monday and the persons who like Peach and
no use of this statement) Watermelon have Seminar on Friday.
9. U does not have seminar at 1 P.M. 13. Persons who like Grape and Orange went to seminar
10. Only two people have seminar between U and X. neither on Tuesday nor Thursday
11. The number of people who have seminar between V 14. U doesn’t like Cranberry
and S is same as the number of people who have In the instruction part it is given which fruit is
seminar between R and Y. 10am and 1pm

15. Persons who like Apple and Banana does not have
seminar on Thursday
Final Arrangement

Page 616 of 1334


Subscribe The Xpress Video Course & Mock Test Package for Bank & Insurance Exams
If there are any suggestions/ errors in our PDFs Feel Free to contact us via this email: admin@exampundit.in
Ultra Practice Bundle PDF
SBI Clerk/ RBI Assist. Mains – Reasoning
5. M had a session of 30minutes.
6. There were two sessions between M’s and N’s session.
7. Number of sessions before J’s session was equal to the
number of sessions after L’s session. J had his session
before L.

36.c
37.d
38.a
39.e
40.b
Solution (41- 45):
1. The slot started at 11:00am.
2. The first Session was of 45minutes.
3. P’s session ended at 12:15pm.
4. There were 3 sessions between P’s session and M’s
session. P had his session before M 8. K’s session started at 1:15pm.
9. Number of persons who went for the session before K
was thrice as that of the number of person who went
after the session that ended at 4:15pm.
10. K did not go in the 1st session.

Page 617 of 1334


Subscribe The Xpress Video Course & Mock Test Package for Bank & Insurance Exams
If there are any suggestions/ errors in our PDFs Feel Free to contact us via this email: admin@exampundit.in
Ultra Practice Bundle PDF
SBI Clerk/ RBI Assist. Mains – Reasoning

Final Arrangement

41.e
42.b
43.a
44.c

11. K and I had consecutive sessions. 45.d

12. O’s slot started at 3:00pm or 3:30pm. Solution (46- 50):


13. P had his session before M. 1. Team Barcelona plays Cricket on Wednesday in the
14. O and L had equal slot duration. third slot.
2. Hockey is played in the last slot of last day of the
week
3. The number of games played before Cricket is
equivalent to the number of games played after
Baseball.
4. Baseball is played by AC Milan team.

15. N and I did not have the equal duration slots.

Page 618 of 1334


Subscribe The Xpress Video Course & Mock Test Package for Bank & Insurance Exams
If there are any suggestions/ errors in our PDFs Feel Free to contact us via this email: admin@exampundit.in
Ultra Practice Bundle PDF
SBI Clerk/ RBI Assist. Mains – Reasoning

5. Cycling is played by team Chelsea immediately


before Hockey.
6. Rowing is played on the same day on which
Handball is organized.
10. The number of games played between Judo and Hand
7. Athletics is played immediately before Rowing
Ball is equivalent to the number of games played
8. Team Flamingo does not play Hockey.
between Base ball and Cycling.
9. The number of teams playing before the Handball is
11. Both Judo and Hand ball are played before Cricket.
equivalent to the number of teams playing after Team
12. Team Arsenal plays Judo.
Flamingo.

Page 619 of 1334


Subscribe The Xpress Video Course & Mock Test Package for Bank & Insurance Exams
If there are any suggestions/ errors in our PDFs Feel Free to contact us via this email: admin@exampundit.in
Ultra Practice Bundle PDF
SBI Clerk/ RBI Assist. Mains – Reasoning
19. Team Liverpool does not play Hand ball.

13. Team Real Madrid plays Boxing at 1:00pm.


14. Only 2 teams play between Manchester United and Final Arrangement
Kerala Blaster teams.
15. There are 3 teams between play between Flamingo
and Manchester United team.
16. Team La Liga plays Golf.

46.d
47.a
48.e
49.a
17. Diving is played before Shooting. 50.c
18. Athletics is played by neither Liverpool nor Porto
team.

Download Puzzles Practice Questions PDF


Page 620 of 1334
Subscribe The Xpress Video Course & Mock Test Package for Bank & Insurance Exams
If there are any suggestions/ errors in our PDFs Feel Free to contact us via this email: admin@exampundit.in
Ultra Practice Bundle PDF
SBI Clerk/ RBI Assist. Mains – Reasoning
Puzzle Scheduling
Direction (1-5): Study the given information carefully a. Same as the number of persons who join after Q.
to answer the following questions: b. Same as the number of persons joining between U and
Eight persons, P, Q, R, S, T, U, V and W joined a company T.
on 19th of different months, January, March, April, June, c. Same as the number of persons joining after W.
July, September, October and December of different d. Cannot be determined
years, 1989, 1992, 1997, 1999, 2007, 2010, 2014 and e. None of these
2016. Difference in the years is considered as per the 3) Who among the following joins immediately after
joining year, not considering the month of joining. W?
U, who joins in April, joins just before the person, who a. P
joins in December. R joins just before W. Two persons b. Q
join the company between T and the person, who joins in c. V
October. Both P and W don’t join in October; exactly one d. S
person doesn’t join between Q and R. The difference in e. T
the years of joining of P and the person, who joins in 4) Four of the following forms a group. Which among
March, is 7 years. Person, who joins in June, joins the the following does not belongs to that group?
company just before the person, who joins in March. The a. P
one joins in January joins before the one joins in July. Two b. Q
persons join the company between V and the person, who c. V
joins in June. T joins in September but not in an odd d. S
numbered year. e. T
1) How many persons join after the person, who joins 5) S joins in which of the month?
in December? a. March
a. 4 b. October
b. 2 c. July
c. 6 d. December
d. 3 e. None of these
e. Cannot be determined Directions (6-10): Study the given information
2) How many persons join before S? carefully to answer the following questions:
Page 621 of 1334
Subscribe The Xpress Video Course & Mock Test Package for Bank & Insurance Exams
If there are any suggestions/ errors in our PDFs Feel Free to contact us via this email: admin@exampundit.in
Ultra Practice Bundle PDF
SBI Clerk/ RBI Assist. Mains – Reasoning
Sanjana planned to study for one subject per day during 7. What is the average number of hours spent by
one week of ‘Preparation Leaves’ starting from Monday. Sanjana on difficult subjects?
She decided one among them as the rest day. She planned a. 4
to prepare for six subjects namely, History, Math, Science, b. 7
Economics, Civics and English. She also planned the c. 6
number of hours to be spent on each subject. According to d. 10
Sanjana, Science and English are difficult subjects e. 14
whereas Math and Civics are easy subjects. 8. Which of the following statement is false as per the
Sanjana cannot study two hard subjects on consecutive given information?
days. She planned to spend only two hours on one of the a. Sanjana planned to spend ten hours on Science
easy subject’s preparation. She decided to start with preparation.
Science preparation two days after civics preparation. She b. Sanjana planned to spend five hours on Civics
planned to study for ten hours on an immediately one day preparation.
before the rest day. She decided to spend five hours on c. Sanjana planned to spend six hours on Economics
Civics preparation on the day, which is four days before preparation.
the History’s preparation day. She planned to take rest on d. Sanjana planned to spend two hours on Math
an immediately following day after preparing for two preparation.
difficult subjects. She also decided to spend four hours on e. All of the above.
Science preparation and end it before Thursday. She 9) Which subjects did Sanjana study on Friday and
decided to spend six hours on Economics but not on Tuesday respectively?
Thursday. a. None, English
6. Which among the following day is the rest day as per b. Math, Economics
Sanjana’s planning? c. English, None
a. Tuesday d. History, Economics
b. Monday e. Math, English
c. Thursday 10) Which subject did Sanjana study on Wednesday?
d. Sunday a. Civics
e. None of these b. Economics
c. Math

Page 622 of 1334


Subscribe The Xpress Video Course & Mock Test Package for Bank & Insurance Exams
If there are any suggestions/ errors in our PDFs Feel Free to contact us via this email: admin@exampundit.in
Ultra Practice Bundle PDF
SBI Clerk/ RBI Assist. Mains – Reasoning
d. History a. Wednesday
e. Science b. Monday
11-15) Study the given information carefully to answer c. Tuesday
the following questions: d. Thursday
There are seven different Coffee shops – A, B, C, D, E, F e. Friday
and G sold coffee one on each day starting from Monday 13) Which of the following combinations of “number
to Sunday (of the same week) not necessarily in same of Coffee sold - shop” is true for Monday?
order. The number of coffee cups sold by the seven coffee a. 81 – D
shops in seven different days of a same week is 100, 77, b. 81 – E
68, 81, 118, 53 and 106 (not necessarily in the same c. 77 – D
order). d. 100 – A
D sold 77 cups of coffee. 53 cups of coffee is been sold by e. 106 – B
B. 81 coffee cups were sold on Wednesday and 3 days 14) Which of the following number of coffee is sold by
after Wednesday B sold the coffee. Shop which sells 68 shop F?
cups of coffee does not sell it either on Monday or a. 81
Thursday. On Thursday a shop sold 118 coffee cups and b. 118
on the day after Thursday F sold 106 coffee cups. G sold c. 106
81 cups of coffee while the shop which sold 100 cups of d. 77
coffee does not sell it either on Monday or Sunday. C does e. 68
not sell coffee either on Saturday or Sunday. C sell coffee 15) Which of the following combination is not true?
after E. a. Monday-77-D
11) The least number of coffee were sold on? b. Tuesday-100-E
a. Tuesday c. Wednesday- 81-G
b. Friday d. Sunday- 100-F
c. Sunday e. Friday-106-F
d. Saturday 16-20)Study the given information carefully to answer
e. None of the given options is true the following questions1:
12) Two shops sold coffee in difference of 12, on Eight persons A, B, C, D, E, F, G and H have different
successive days. Less coffee is sold on which day? exams on different dates (of the same month and year)

Page 623 of 1334


Subscribe The Xpress Video Course & Mock Test Package for Bank & Insurance Exams
If there are any suggestions/ errors in our PDFs Feel Free to contact us via this email: admin@exampundit.in
Ultra Practice Bundle PDF
SBI Clerk/ RBI Assist. Mains – Reasoning
among 2nd, 4th, 5th, 7th, 9th, 10th, 12th and 13th. Each e. Only III
of these persons belongs to different places. They also 17) Person from where has exam on 2nd?
have different exams a. Agra
G has exam one day before the person, who is from b. Can’t be determined
Indore. There is a gap of four days between the exams of c. Gwalior
G and the person, who has UPSC exam. A has exam two d. Bhopal
days before the person, who has UPSC exam. Only one e. None of these
person has exam after the person, who has PO exam. F, 18) Who among the following has UPSC exam?
who is from Bhopal, and A have exams one after the other. a. Person from Mumbai.
F doesn’t have exam on 7th. Three persons have exam b. Person from Patna.
between F and D. E has exam immediately after the one, c. Person from Gwalior.
who has SSC exam. D is not from Indore. E doesn’t have d. Can’t be determined
PO exam. C has exam after B but not on the last day. B e. None of these.
has clerk exam. Person from Patna has exam five days 19) Who has exam five days before the person from
after B. Neither B nor G is from Lucknow. Person, who Pune?
has CHSL exam, has exam immediately after the person a. A
from Lucknow. C doesn’t have Insurance exam. Person, b. E
from Mumbai, has exam three days before the person, who c. C
has Insurance exam. Persons from Agra and Gwalior have d. F
exams before 10th. One of the persons is from Pune. e. None of these
16) Who among the following has exam after the 20) Who among the following has SSC exam?
person from Pune? a. B
I. H b. C
II. D c. A
III. C d. D
a. Only II e. None of these.
b. Both I and II 21-25) Read the following information carefully to
c. Both II and III answer the question that follows:
d. Only I

Page 624 of 1334


Subscribe The Xpress Video Course & Mock Test Package for Bank & Insurance Exams
If there are any suggestions/ errors in our PDFs Feel Free to contact us via this email: admin@exampundit.in
Ultra Practice Bundle PDF
SBI Clerk/ RBI Assist. Mains – Reasoning
Nine candidates have exams on different dates (5th, 18th 23) Who among the following has exam of the subject
and 23th) on 3 different months (April, May and June) of in which we study about the gravity?
the same year. They have exams of different subject viz. I. F
(Physics, Chemistry, Biology, Mathematics, History, II. D
Hindi, English, Economics and Polity III. J
Three candidates have exams between H and F who does a. Either I or II
not have exam of Biology. G has exam of Hindi but after b. Only II
B and before A. J has exam of Physics but it is scheduled c. Only III
immediately before F. D’s exam which is not of Biology d. Only I
is immediately after E’s exam but they have exam in e. None of these
different months. C has exam of Polity but immediately 24) Which among the following group has a exam of
after H and neither of them has exam on 5th. B and E does Language?
not give the exam of biology. H gives exam of the subject a. B, G
in which the topics are financial. The exam of chemistry b. F, G
is scheduled immediately after the exam of Mathematics. c. G, H
F does not give the exam of English but gives the exam d. G, D
after the exam of English. e. G, C
21) B has exam on which date and of which subject? 25) Who has exam of History?
a. 18th May , Hindi a. None of these
b. 18th April , English b. G
c. 5th May, History c. F
d. 5th April, English d. B
e. None of these e. A
22)G has exam on which date? 26-30) Study the given information carefully to answer
a. 18th May the following questions:
b. 5th May 8 different organizations - A, F, H, K, N, P, S and V have
c. 18th April scheduled their seminars in eight different months -
d. 23th May January, February, March, April, May, June, July and
e. None of these August, not necessarily in the same order.

Page 625 of 1334


Subscribe The Xpress Video Course & Mock Test Package for Bank & Insurance Exams
If there are any suggestions/ errors in our PDFs Feel Free to contact us via this email: admin@exampundit.in
Ultra Practice Bundle PDF
SBI Clerk/ RBI Assist. Mains – Reasoning
Only two dates have been fixed for seminar for each c. Four
organization i.e. 25th and 31th. Seminar of N will be held d. Five
on third month after the seminar of S but their dates are e. Six
different. Seminar of S will be held after February. 29) Four of the following five options are alike in a
Seminar of A will be held on the second month after the certain way, hence they form a group, find out the odd
P’s seminar but immediately after the seminar of H. P’s pair?
seminar date is different from the seminar dates of A and a. A and F
H both. Seminar of K will be held before the seminar of F b. H and N
and V both. Seminar of F will neither be held in June nor c. N and V
in August. Date of seminars of N and V is same but of V d. K and V
and P are different. Seminar of F and the organization e. P and S
whose seminar is in June, both will be on the same date. 30) Which of the following combination is not correct?
26) Seminar of K will be held in which month? a. A - March - 25
a. January b. F - July - 30
b. February c. H - February - 25
c. March d. N - August - 25
d. April e. S - May – 31
e. May 31-35) Study the given information carefully to answer
27) Which organization seminar will be held the following questions:
immediate after the S’s seminar? Eight candidates A, B, C, D, E, F, G and H has NTPC
a. A exams on different days either on 7th or 20th of four
b. F months among August, September, October and
c. N November in 2020. Each of them belongs to different
d. P places. One of the persons is from Agra.
e. V Three persons have exam before C’s exam. G is from
28) How many organizations have their seminars on Jabalpur and his exam is immediately after F’s exam but
25th? not in the same month. Two persons have exam between
a. Two E’s and H’s exam. Person from Kanpur has exam
b. Three immediately after E’s exam but not in September. D is

Page 626 of 1334


Subscribe The Xpress Video Course & Mock Test Package for Bank & Insurance Exams
If there are any suggestions/ errors in our PDFs Feel Free to contact us via this email: admin@exampundit.in
Ultra Practice Bundle PDF
SBI Clerk/ RBI Assist. Mains – Reasoning
from Jhansi and has exam immediately after B’s exam but a. Person from Agra
not on 20th August. Person from Bhopal has exam b. Person from Indore
immediately after A’s exam. B is not from Bhopal. Person c. F
from Lucknow has exam immediately after the one from d. None of these
Indore. Person from Gwalior doesn’t have exam at last. e. H
31) How many persons have exam between A and the 35) From which does B belongs to?
person from Agra? a. Agra
a. Four b. Kanpur
b. One c. Indore
c. Three d. Jhansi
d. Five e. None of these
e. None of these 36-40) Study the given information carefully to answer
32) Person from Gwalior has exam on which date? the following questions:
a. 7th September There are 12 members (A, B, C, D, E, F, G, H, I, J, K and
b. 7th August L) each has decided to play a different game in their
c. 20th August college for the tournament that is going to be held in a
d. 7th October week from Monday to Saturday. 2 games will be played
e. None of these in a day such that 1 in the morning and 1 in the evening.
33) Who among the following has exam after the E likes to play carom which is going to be played on
person from Lucknow? Monday evening. H will play kabaddi on Thursday
I. Person from Jhansi morning. F plays handball which is the last game of the
II. Person from Agra. tournament. C likes to play badminton which is scheduled
III. Person from Gwalior. on Tuesday evening. B plays football which is exactly
a. Both I and III between C and F. Tennis tournament is immediately
b. Only II before kabaddi. G will play between E and C. Cycling
c. Only I tournament will be held exactly between the tournaments
d. Both I and II in which B and F will participate. The first tournament is
e. Both II and III Chess. K loves swimming and I loves volleyball. K will
34) Who has exam immediately after G’s exam? play before I. J will play before I and there will be only

Page 627 of 1334


Subscribe The Xpress Video Course & Mock Test Package for Bank & Insurance Exams
If there are any suggestions/ errors in our PDFs Feel Free to contact us via this email: admin@exampundit.in
Ultra Practice Bundle PDF
SBI Clerk/ RBI Assist. Mains – Reasoning
one game between them. L will play in the morning. One 40) Cycling tournament will be held on which of the
person plays Rugby. following day?
36) Which sport will be played between Swimming and a. Tuesday
Kabaddi? b. Saturday
a. Handball c. Friday
b. Badminton d. Wednesday
c. Carrom e. None of these
d. Cycling 41-45) Study the below details and answer the
e. Tennis following questions.
37) G played on which of the following day? 12 persons (M, N, O, P, Q, R, S, T, U, V, W and X) are
a. Monday told to prepare a seminar on chapters from 1 to 12 such
b. Tuesday that no one prepare for the same chapter. The seminar
c. Wednesday takes place such that each day two persons are allowed,
d. Thursday one at 8 AM and the other at 2 PM from Monday to
e. Friday Saturday.
38) How many persons played between G and J? X is the 2nd person to take seminar and he chose chapter
a. 4 1. P prepares the last chapter for seminar and has the
b. 7 seminar at 8 AM on Wednesday. R does the seminar on
c. 6 Thursday at 2 PM on chapter 2. V, Q, and S has seminars
d. 5 immediately one after the other and they prepare for the
e. None of these consecutive chapters. R has the seminar between the
39) If Rugby will be played before D, then Who will people who prepared chapter 9 and 10 for seminar. The
play tennis? person who prepared chapter 8 has the seminar
a. K immediately after the person who has seminar on chapter
b. D 3. N has the seminar on Thursday but did not prepare
c. A chapter 10. W chose chapter 4 and has seminar on
d. B Wednesday. U has the seminar between X and O.V does
e. Cannot be determined not prepare for chapter 10.
41) Who has the seminar immediately after R?

Page 628 of 1334


Subscribe The Xpress Video Course & Mock Test Package for Bank & Insurance Exams
If there are any suggestions/ errors in our PDFs Feel Free to contact us via this email: admin@exampundit.in
Ultra Practice Bundle PDF
SBI Clerk/ RBI Assist. Mains – Reasoning
a. M d. 9
b. X e. 11
c. T 46-50) Study the below details and answer the
d. V following questions.
e. Cannot be determined Eight kids, A, B, C, D, E, F, G, and H were born on
42) How many persons had seminar between V and O? different dates among 3rd, 5th, 17th, 18th, 20th, 22nd,
a. 4 23rd and 27th of different months in year 2019. No kid
b. 5 was born in January, February, June and August. Only the
c. 6 months are considered not the dates while counting the
d. 7 number of months between the birth of two kids.
e. Cannot be determined A was born in a month which consists of 30 days but not
43) Who among the following had the seminar on in November. Two kids were born between A and the one,
Saturday? who was born on 22nd. G was born in October. B was
a. S born on 18th and in the month which is three months
b. V before the month in which G was born. G was born after
c. Q the one, who was born on 22nd. D was born immediately
d. P after B. The kid, who was born on 5th, was born in the
e. Both 1 and 3 month, which is at most five months before the month in
44) Which of the following chapter did V prepare? which G was born. Neither D nor A was born on 5th. F
a. 9 was born on 3rd but before 5th. E was born before
b. 6 November. The kid, who was born on 20th, was born
c. 7 immediately before C. The kid, who was born on 27th,
d. 5 was born after H.
e. None of these 46) Who was born immediately after G?
45) If M does not choose chapter 10 then T takes a. C
seminar on which of the following Chapter? b. H
a. 8 c. D
b. 10 d. E
c. 12 e. None of these

Page 629 of 1334


Subscribe The Xpress Video Course & Mock Test Package for Bank & Insurance Exams
If there are any suggestions/ errors in our PDFs Feel Free to contact us via this email: admin@exampundit.in
Ultra Practice Bundle PDF
SBI Clerk/ RBI Assist. Mains – Reasoning
47) Who was born on 17th of a month? e. Both II and III
a. C 49) G was born on which date?
b. H a. 17th
c. D b. 18th
d. E c. 20th
e. A d. 22nd
48) Who among the following was/were born after H? e. Cannot be determined
I. C 50) How many kids were born between H and the one,
II. B who was born on 3rd of a month?
III. D a. Four
a. Both I and II b. Five
b. Only II c. One
c. Only III d. Three
d. Only I e. None of these

Puzzle Scheduling - Answer with Explanation


Solution 1-5
1. The difference in the years of joining of P and the person, who joins in March, is 7 years.
2. Person, who joins in June, joins the company just before the person, who joins in March.
Case-1 Case-2 Case-3 Case-4

Year Person Month Person Month Person Month Person Month

1989 June

1992 P March

1997 June

Page 630 of 1334


Subscribe The Xpress Video Course & Mock Test Package for Bank & Insurance Exams
If there are any suggestions/ errors in our PDFs Feel Free to contact us via this email: admin@exampundit.in
Ultra Practice Bundle PDF
SBI Clerk/ RBI Assist. Mains – Reasoning
1999 June March P

2007 P March

2010 June

2014 March P

2016

3. Two persons join the company between V and the person, who joins in June. Hence, Case-4 is eliminated
4. T joins in September but not in an odd numbered year.
Case-1 Case-1 (a)

Year Person Month Year Person Month

1989 1989

1992 T September 1992

1997 V 1997 V

1999 1999

2007 P 2007 P

2010 June 2010 June

2014 March 2014 March

2016 2016 T September

Case-2 Case-2 (a) Case-2 (b)

Year Person Month Person Month Person Month

Page 631 of 1334


Subscribe The Xpress Video Course & Mock Test Package for Bank & Insurance Exams
If there are any suggestions/ errors in our PDFs Feel Free to contact us via this email: admin@exampundit.in
Ultra Practice Bundle PDF
SBI Clerk/ RBI Assist. Mains – Reasoning
1989 V V V

1992 T Septemb
er

1997

1999 June June June

2007 March March March

2010 T September

2014 P P P

2016 T September

Case-3 Case-3 (a)

Year Person Month Person Month

1989

1992 P P

1997 June June

1999 March March

2007

2010 V V

2014 T September

2016 T September

Page 632 of 1334


Subscribe The Xpress Video Course & Mock Test Package for Bank & Insurance Exams
If there are any suggestions/ errors in our PDFs Feel Free to contact us via this email: admin@exampundit.in
Ultra Practice Bundle PDF
SBI Clerk/ RBI Assist. Mains – Reasoning
5. Two persons join the company between T and the person, who joins in October.
6. Both P and W don’t join in October.
Hence, Case-1, Case -1(a), Case-2, Case-2(b), Case-3 is eliminated
Case-2 (a) Case-3 (a)

Year Person Month Person Month

1989 V

1992 P

1997 October June

1999 June March

2007 March October

2010 T September V

2014 P

2016 T September

7. R joins just before W. Exactly one person doesn’t join between Q and R.
8. U, who joins in April, joins just before the person, who joins in December
Hence, Case-2(a) is eliminated.
9. The one joins in January joins before the one joins in July.
Year Person Month

1989 U April

1992 P December

1997 R June

1999 W March

Page 633 of 1334


Subscribe The Xpress Video Course & Mock Test Package for Bank & Insurance Exams
If there are any suggestions/ errors in our PDFs Feel Free to contact us via this email: admin@exampundit.in
Ultra Practice Bundle PDF
SBI Clerk/ RBI Assist. Mains – Reasoning
2007 S October

2010 V January

2014 Q July

2016 T September

1. C
2. C
3. D
4. D
5. B
Solution 6-10 Tuesday
1) She decided to spend five hours on Civics preparation
Wednesday Science 4
on the day, which is four days before the History’s
preparation day. Thursday
So, there are possibilities of three days for Civics
Friday History
preparation i.e. Monday, Tuesday and Wednesday.
2) She decided to spend four hours on Science preparation Saturday

and end it before Thursday.


Sunday
So, there are possibilities of three days for Science
preparation i.e. Monday, Tuesday and Wednesday. 4) She cannot study two hard subjects on consecutive
3) She also planned to start with Science preparation two days.
days after civics preparation. (i.e. English is the difficult subject and can be studied on
(This condition satisfies only if preparation days of Civics either Saturday or Sunday.)
and Science are Monday and Wednesday respectively.) 5) She planned to take rest on an immediately following
Day Subject No of day after preparing for two difficult subjects.
hours (That implies, she prepares for an English subject on
Saturday and takes rest on Sunday.)
Monday Civics 5
6) She planned to study for ten hours on an immediate one
day before the rest day.
Page 634 of 1334
Subscribe The Xpress Video Course & Mock Test Package for Bank & Insurance Exams
If there are any suggestions/ errors in our PDFs Feel Free to contact us via this email: admin@exampundit.in
Ultra Practice Bundle PDF
SBI Clerk/ RBI Assist. Mains – Reasoning
Day Subject No of Saturday English 10
hours
Sunday Rest day
Monday Civics 5
6. D
Tuesday 7. B
8. A
Wednesday Science 4
9. D
Thursday 10. E
Solution 11-15
Friday History
1) On Thursday a shop sold 118 coffee cups and on the
Saturday English 10 day after Thursday F sold 106 coffee cups.
Day Coffee cups Shop
Sunday Rest day
sold
7) She planned to spend six hours on Economics but not
Monday
on Thursday.
(That implies Economics will be studied on Tuesday.) Tuesday
8) She planned to spend only two hours on one of the easy
Wednesday
subject’s preparation.
(Given that, Math and Civics are easy subjects.) Thursday 118
Day Subject No of
Friday 106 F
hours
Saturday
Monday Civics 5
Sunday
Tuesday Economics 6
2) 53 cups of coffee is been sold by B.
Wednesday Science 4
3) 81 coffee cups were sold on Wednesday and 3 days
Thursday Math 2 after Wednesday B sold the coffee.
Day Coffee cups Shop
Friday History
sold

Page 635 of 1334


Subscribe The Xpress Video Course & Mock Test Package for Bank & Insurance Exams
If there are any suggestions/ errors in our PDFs Feel Free to contact us via this email: admin@exampundit.in
Ultra Practice Bundle PDF
SBI Clerk/ RBI Assist. Mains – Reasoning
Monday Monday 77 D

Tuesday Tuesday 100

Wednesday 81 Wednesday 81 G

Thursday 118 Thursday 118

Friday 106 F Friday 106 F

Saturday 53 B Saturday 53 B

Sunday Sunday 77 D

4) D sold 77 cups of coffee. 6) Shop which sells 68 cups of coffee does not sell it either
Day Coffee cups Shop on Monday or Thursday.
sold Day Coffee cups Shop
sold
Monday 77 D
Monday 77 D
Tuesday 77 D
Tuesday 100
Wednesday 81
Wednesday 81 G
Thursday 118
Thursday 118
Friday 106 F
Friday 106 F
Saturday 53 B
Saturday 53 B
Sunday 77 D
Sunday 68
5) G sold 81 cups of coffee while the shop which sold 100
cups of coffee does not sell it either on Monday or Sunday. 7) C does not sell coffee either on Saturday or Sunday.
Day Coffee cups Shop 8) C sell coffee after E.
sold

Page 636 of 1334


Subscribe The Xpress Video Course & Mock Test Package for Bank & Insurance Exams
If there are any suggestions/ errors in our PDFs Feel Free to contact us via this email: admin@exampundit.in
Ultra Practice Bundle PDF
SBI Clerk/ RBI Assist. Mains – Reasoning
Day Coffee cups Shop 11. D
sold 12. E
13. C
Monday 77 D
14. C
Tuesday 100 E 15. D
Solutions 16-20
Wednesday 81 G
1. Only one person has exam after the person, who has PO
Thursday 118 C exam,
2. G has exam one day before the person from Indore.
Friday 106 F
3. There is a gap of four days between the exams of G and
Saturday 53 B the person, who has UPSC exam.
4. A has exam two days before the person, who has UPSC
Sunday 68
exam, so, G has exam either on 4th or 9th or 12th.
Since the left shop is A, thus final arrange would be as Case I:
following.
2nd
Day Coffee cups Shop
sold 4th G

5th Indore
Monday 77 D
7th A
Tuesday 100 E
9th UPSC
Wednesday 81 G
10th
Thursday 118 C
12th PO
Friday 106 F
13th
Saturday 53 B Case II:

Sunday 68 A 2nd A

4th UPSC

Page 637 of 1334


Subscribe The Xpress Video Course & Mock Test Package for Bank & Insurance Exams
If there are any suggestions/ errors in our PDFs Feel Free to contact us via this email: admin@exampundit.in
Ultra Practice Bundle PDF
SBI Clerk/ RBI Assist. Mains – Reasoning
5th 7th A

7th 9th F UPSC Bhopal

9th G 10th

10th Indore 12th PO

12th PO 13th

13th Case II:

Case III: 2nd A

2nd 4th F UPSC Bhopal

4th 5th

5th A 7th

7th UPSC 9th G

9th 10th D Indore

10th 12th PO

12th G PO 13th

13th Indore Case III:

5. F, who is from Bhopal and A has exams one after the 2nd

other. 4th F Bhopal


6. F doesn’t have exam on 7 .
th
5th A
7. Three persons have exam between F and D.
Case I: 7th UPSC

2nd D 9th

4th G 10th D

5th Indore 12th G PO

Page 638 of 1334


Subscribe The Xpress Video Course & Mock Test Package for Bank & Insurance Exams
If there are any suggestions/ errors in our PDFs Feel Free to contact us via this email: admin@exampundit.in
Ultra Practice Bundle PDF
SBI Clerk/ RBI Assist. Mains – Reasoning
13th Indore 12th G PO

8. D is not from Indore, so case II is rejected. 13th H Indore


9. E has exam immediately after the one, who has SSC
13. Person from Patna has exam five days after B, this is
exam.
not possible in case I, so case I is rejected.
10. E doesn’t have PO exam.
14. Person, who has CHSL exam, has exam immediately
11. C has exam after B but not on the last day, so H has
after the person from Lucknow.
exam on 13th.
15. Neither B nor G is from Lucknow.
12. B has clerk exam.
2nd B Clerk
Case I:
4th F Bhopal
2nd D

4th G SSC 5th A SSC

5th E Indore 7th E UPSC Patna

7th A 9th C Lucknow

9th F UPSC Bhopal 10th D CHSL

10th B Clerk 12th G PO

12th C PO 13th H Indore

16. C doesn’t have insurance exam.


13th H
17. Person, from Mumbai, has exam three days before the
Case III:
person, who has Insurance exam.
nd
2 B Clerk 18. Person from Agra and Gwalior has exams before 10th,

4th F Bhopal so person from Pune has exam on 12th.


The final table is given below:
5th A SSC
Date Person Exam Place
7th E UPSC
2nd B Clerk Agra/Gwalior
th
9 C
4th F Railways/BPSC Bhopal
th
10 D

Page 639 of 1334


Subscribe The Xpress Video Course & Mock Test Package for Bank & Insurance Exams
If there are any suggestions/ errors in our PDFs Feel Free to contact us via this email: admin@exampundit.in
Ultra Practice Bundle PDF
SBI Clerk/ RBI Assist. Mains – Reasoning
5th A SSC Gwalior/Agra 13th H Insurance Indore

7th E UPSC Patna 16. D


17. B
9th C BPSC/Railways Lucknow
18. B
th
10 D CHSL Mumbai 19. B

12th G PO Pune 20. C

Solution 21-25
1. C has exam of Polity but immediately after H and neither of them has exam on 5th
2. Three candidates have exams between H and F who does not have exam of Biology
3. J has exam of Physics but it is scheduled immediately before F.
Case I Case II Case III

Months 5th 18th 23th 5th 18th 23th 5th 18th 23th

April H C (Polity) J(Physics)

May J (Physics) F (Biology) H C(Polity) F

(Biology)

June J (Physics) F (Biology) H C(Polity)

4. D’s exam which is not of Biology is immediately after E’s exam but they have exam in different months. This is not
possible in case I, so case I is rejected.
Case II Case III

Months 5th 18th 23th 5th 18th 23th

April E J(Physics)

Page 640 of 1334


Subscribe The Xpress Video Course & Mock Test Package for Bank & Insurance Exams
If there are any suggestions/ errors in our PDFs Feel Free to contact us via this email: admin@exampundit.in
Ultra Practice Bundle PDF
SBI Clerk/ RBI Assist. Mains – Reasoning
May D H C(Polity) F E

(Biology) (Biology)

June J F D H C(Polity)
(Physics)
(Biology) (Biology)

5. G has exam of Hindi but after B and before A.


6. B and A together don’t have exam on 5th.
This is not possible in case II, so case II is rejected.

Case III

Months 5th 18th 23th

G
April B (Hindi) J(Physics)

F
May (Biology) A E

D
June (Biology) H C(Polity)

7. B and E does not give the exam of biology


8. H gives exam of the subject in which the topics are financial
So, H gives the exam of Economics
Now, only A is left so A gives the exam of Biology

Months 5th 18th 23th

April B G (Hindi) J(Physics)

May F A (Biology) E

Page 641 of 1334


Subscribe The Xpress Video Course & Mock Test Package for Bank & Insurance Exams
If there are any suggestions/ errors in our PDFs Feel Free to contact us via this email: admin@exampundit.in
Ultra Practice Bundle PDF
SBI Clerk/ RBI Assist. Mains – Reasoning
H
June D (Economics) C(Polity)

9. The exam of chemistry is scheduled immediately after the exam of Mathematics


10. F does not give the exam of English but gives the exam after the exam of English.
Hence, B gives the exam of English and Now only F is left so F gives the exam of History
The final schedule will be

Months 5th 18th 23th

April B (English) G (Hindi) J(Physics)

E
May F (History) A (Biology) (Mathematics)

D H
June (Chemistry) (Economics) C(Polity)

21. D
22. C
23. C
24. A
25. C
Solutions 26-30 H
1) Seminar of N will be held on third month after the
K
seminar of S but their dates are different.
2) Seminar of S will be held after February. N June 25
Case 1
P
Organization Month Date
S March 31
A
V
F
Case 2

Page 642 of 1334


Subscribe The Xpress Video Course & Mock Test Package for Bank & Insurance Exams
If there are any suggestions/ errors in our PDFs Feel Free to contact us via this email: admin@exampundit.in
Ultra Practice Bundle PDF
SBI Clerk/ RBI Assist. Mains – Reasoning
Organization Month Date 3) Seminar of A will be held in the third month of the year
after the P’s seminar but immediately after the seminar of
A
H.
F P→H→A
This condition doesn’t satisfy in case 1 so this case gets
H
eliminated.
K 4) P’s seminar date is different from the seminar dates of
A and H both.
N July 31
In case 3, we have two sub-cases as
P January, February, March → This satisfies condition 4).
February, March, April → This doesn’t satisfy condition
S April 25
4) as if February = 25th, then March and April =
V 31st which is not possible as April has only 30 days.
Case 2
Organization Month Date
Case 3
Organization Month Date A March 25

A F

F H February 25

H K

K N July 31

N August 31/25 P January 31

P S April 25

S May 25/31 V

V Case 3

Page 643 of 1334


Subscribe The Xpress Video Course & Mock Test Package for Bank & Insurance Exams
If there are any suggestions/ errors in our PDFs Feel Free to contact us via this email: admin@exampundit.in
Ultra Practice Bundle PDF
SBI Clerk/ RBI Assist. Mains – Reasoning
Organization Month Date K April 25

A March 25 N August 25

F P January 31

H February 25 S May 31

K V June 25

N August 31/25
8) Seminar of F and of the organization whose seminar is
P January 31
in June, both will be on the same date.
S May 25/31 Organization Month Date

V A March 25

5) Seminar of K will be held before the seminar of F and F July 25


V both.
H February 25
6) Seminar of F will neither be held in June nor in August.
This condition doesn’t satisfy in case 2 so this case gets K April 25
eliminated.
N August 25
7) Date of seminars of N and V is same but of V and P are
different. P January 31
Since, June has 30 days, so N and V seminar dates will be
S May 31
25th.
Case 3 V June 25

Organization Month Date


26. D
A March 25 27. E
28. E
F July
29. E
H February 25 30. B
Solution 31-35
Page 644 of 1334
Subscribe The Xpress Video Course & Mock Test Package for Bank & Insurance Exams
If there are any suggestions/ errors in our PDFs Feel Free to contact us via this email: admin@exampundit.in
Ultra Practice Bundle PDF
SBI Clerk/ RBI Assist. Mains – Reasoning
1. Three persons have exam before C’s exam. October E (Kanpur)
2. G is from Jabalpur and his exam is immediately after
F’s exam but not in the same month, so G has exam on November H

either 7th September or 7th November: Case II(a):


Case I: 7th 20th
7th 20th
August H
August F
September C
September G(Jabalpur) C
October E F(Kanpur)
October
November G(Jabalpur)
November Case II(b):
Case II: 7th 20th
7th 20th
August
August
September C
September C
October E F(Kanpur)
October F
November G(Jabalpur) H
November G(Jabalpur) 5. D from Jhansi has exam immediately after B’s exam
3. Two persons have exam between E’s and H’s exam. but not on 20th August, this is not possible in case II(a), so
4. Person from Kanpur has exam immediately after E’s case II(a) is rejected. And thus, A’ exam is on 7th August.
exam but not in September. 6. Person from Bhopal has exam immediately after A’s
Case I: exam, so either F or B is from Bhopal.

7th 20th Case I:

7th 20th
August F
August A F(Bhopal)
September G(Jabalpur) C

Page 645 of 1334


Subscribe The Xpress Video Course & Mock Test Package for Bank & Insurance Exams
If there are any suggestions/ errors in our PDFs Feel Free to contact us via this email: admin@exampundit.in
Ultra Practice Bundle PDF
SBI Clerk/ RBI Assist. Mains – Reasoning
September G(Jabalpur) C 33. D
34. B
October E B(Kanpur) 35. B

November D(Jhansi) H Solutions 36-40


1) E likes to play carom which is going to be played on
Case II(b):
Monday evening.
th th
7 20
2) H will play kabaddi on Thursday morning.

August A B(Bhopal) 3) F plays handball which is the last game of the


tournament.
September D(Jhansi) C
4) C likes to play badminton which is scheduled on
October E F(Kanpur) Tuesday evening.
Days Person Game
November G(Jabalpur) H
Monday Morning
7. B is not from Bhopal, so case II(b) is rejected.
8. Person from Lucknow has exam immediately after the Evening E Carrom
one from Indore, so E is from Lucknow and C is from
Tuesday Morning
Indore.
9. Person from Gwalior doesn’t have exam at last, so A is Evening C Badminton
from Gwalior and H is from Agra.
Wednesday Morning
The final schedule table is given below:

7th 20th Evening

August A(Gwalior) F(Bhopal) Thursday Morning H Kabaddi

September G(Jabalpur) C(Indore) Evening

October E(Lucknow) B(Kanpur) Friday Morning

November D(Jhansi) H(Agra) Evening

31. E Saturday Morning


32. B

Page 646 of 1334


Subscribe The Xpress Video Course & Mock Test Package for Bank & Insurance Exams
If there are any suggestions/ errors in our PDFs Feel Free to contact us via this email: admin@exampundit.in
Ultra Practice Bundle PDF
SBI Clerk/ RBI Assist. Mains – Reasoning
Evening F Handball Monday Morning Chess

5) Tennis tournament is immediately before kabaddi. Evening E Carrom


6) G will play between E and C.
Tuesday Morning G
Days Person Game
Evening C Badminton
Monday Morning
Wednesday Morning K Swimming
Evening E Carrom
Evening Tennis
Tuesday Morning G
Thursday Morning H Kabaddi
Evening C Badminton
Evening B Football
Wednesday Morning
Friday Morning
Evening Tennis
Evening Cycling
Thursday Morning H Kabaddi
Saturday Morning
Evening
Evening F Handball
Friday Morning
10) K loves swimming and I loves volleyball. K will play
Evening
before I.
Saturday Morning 11) J will play before I and there will be only one game
between them.
Evening F Handball
12) L will play in the morning.
7) B plays football which is exactly between C and F. Days Person Game
8) Cycling tournament will be held exactly between the
Monday Morning L Chess
tournaments in which B and F will participate.
9) The first tournament is Chess. Evening E Carrom
Days Person Game
Tuesday Morning G

Page 647 of 1334


Subscribe The Xpress Video Course & Mock Test Package for Bank & Insurance Exams
If there are any suggestions/ errors in our PDFs Feel Free to contact us via this email: admin@exampundit.in
Ultra Practice Bundle PDF
SBI Clerk/ RBI Assist. Mains – Reasoning
Evening C Badminton Tuesday 8 am

Wednesday Morning K Swimming 2 pm

Evening A/D Tennis Wednesday 8 am P 12

Thursday Morning H Kabaddi 2 pm

Evening B Football Thursday 8 am

Friday Morning J 2 pm R 2

Evening A/D Cycling Friday 8 am

Saturday Morning I Volleyball 2 pm

Evening F Handball Saturday 8 am

36. E 2 pm
37. B
4) N has the seminar on Thursday but did not prepare
38. D
chapter 10.
39. E
5) W chose chapter 4 and has seminar on Wednesday.
40. C
Days Time Person Chapter
Solution 41-45
1) X is the 2ndperson to take seminar and he chose chapter Monday 8 am
1.
2 pm X 1
2) P prepares the last chapter for seminar and has the
seminar at 8 AM on Wednesday. Tuesday 8 am

3) R does the seminar on Thursday at 2 PM on chapter 2.


2 pm
Days Time Person Chapter
Wednesday 8 am P 12
Monday 8 am
2 pm W 4
2 pm X 1

Page 648 of 1334


Subscribe The Xpress Video Course & Mock Test Package for Bank & Insurance Exams
If there are any suggestions/ errors in our PDFs Feel Free to contact us via this email: admin@exampundit.in
Ultra Practice Bundle PDF
SBI Clerk/ RBI Assist. Mains – Reasoning
Thursday 8 am N 10 2 pm Q

2 pm R 2 Saturday 8 am S

Friday 8 am 2 pm

2 pm Case 2:
Days Time Person Chapter
Saturday 8 am
Monday 8 am
2 pm
2 pm X 1
6) U has the seminar exactly between X and O.
7) V, Q, and S has seminars immediately one after the Tuesday 8 am U
other in the same order and they prepare for the
2 pm O
consecutive chapters.
Case 1: Wednesday 8 am P 12
Days Time Person Chapter
2 pm W 4
Monday 8 am
Thursday 8 am N 10
2 pm X 1
2 pm R 2
Tuesday 8 am U
Friday 8 am
2 pm O
2 pm V
Wednesday 8 am P 12
Saturday 8 am Q
2 pm W 4
2 pm S
Thursday 8 am N 10
7) R has the seminar between the people who prepared
2 pm R 2 chapter 9 and 10 for seminar.
8) V does not prepare for chapter 10.
Friday 8 am V
Therefore, case 1 gets eliminated.

Page 649 of 1334


Subscribe The Xpress Video Course & Mock Test Package for Bank & Insurance Exams
If there are any suggestions/ errors in our PDFs Feel Free to contact us via this email: admin@exampundit.in
Ultra Practice Bundle PDF
SBI Clerk/ RBI Assist. Mains – Reasoning
9) The person who prepared chapter 8 has the seminar 2 pm X 1
immediately after the person who has seminar on chapter
Tuesday 8 am U 3
3.
Case 2: 2 pm O 8
Days Time Person Chapter
Wednesday 8 am P 12
Monday 8 am
2 pm W 4
2 pm X 1
Thursday 8 am N 9
Tuesday 8 am U 3
2 pm R 2
2 pm O 8
Friday 8 am M/T 10
Wednesday 8 am P 12
2 pm V 5
2 pm W 4
Saturday 8 am Q 6
Thursday 8 am N 9
2 pm S 7
2 pm R 2
41. E
Friday 8 am 10 42. B
43. E
2 pm V 5
44. D
Saturday 8 am Q 6 45. B
Solution 46-50
2 pm S 7
1. A was born in a month which consists of 30 days not in
Now the only chapter left is 11 and the people are M and November, so A was born either in April or in September.
T. 2. Two kids were born between A and the one, who was
Days Time Person Chapter born on 22nd.
3. G was born in a month, which consists of 31 days.
Monday 8 am M/T 11
4. G was born after the one, who was born on 22nd.

Page 650 of 1334


Subscribe The Xpress Video Course & Mock Test Package for Bank & Insurance Exams
If there are any suggestions/ errors in our PDFs Feel Free to contact us via this email: admin@exampundit.in
Ultra Practice Bundle PDF
SBI Clerk/ RBI Assist. Mains – Reasoning
5. B was born on 18th and in the month which is three July B 18th
months before the month in which G was born, so
Sept 22nd
Case I:

March Oct G

April A Nov

May Dec

Case II: May 5th


March
July B 18th
April 22nd
Sept D 22nd
May
Oct G
July B 18th
Nov
Sept A
Dec
Oct G 8. F was born on 3rd but before 5th, so F was born in March.

Nov 9. E was born before November, E was born in May.

March F 3rd
Dec

6. D was born immediately after B, this is not possible in April A

case II, so case II is rejected. May E 5th


7. The kid, who was born on 5th, was born in the month,
July B 18th
which is at most five months before the month in which G
was born. Sept D 22nd
Case I: Oct G
March
Nov
April A
Dec

Page 651 of 1334


Subscribe The Xpress Video Course & Mock Test Package for Bank & Insurance Exams
If there are any suggestions/ errors in our PDFs Feel Free to contact us via this email: admin@exampundit.in
Ultra Practice Bundle PDF
SBI Clerk/ RBI Assist. Mains – Reasoning
10. The kid, who was born on 20th, was born immediately Sept D 22nd
before C.
th
Oct G 23rd/17th
11. The kid, who was born on 27 , was born after H, so C
was born on 27th December and H was born on Nov H 20th
20th November. Dec C 27th
The final schedule table is given below:
46. B
rd
March F 3 47. E
April A 17th/23rd 48. D
49. E
May E 5th
50. B
th
July B 18

Download Puzzles Practice Questions PDF


Get More Reasoning Practice Questions PDF
Puzzle Age
Directions (1 - 5): Answer the questions based on the years elder to O. Not more than two persons were born
information given below: between S and R. The one born in Raipur was born after
Eight persons L, M, N, O, P, Q, R and S were born in the 1983 but not on even year. N was born in Indore at a gap
years 1963, 1967, 1976, 1979, 1983, 1987, 1990, and of 4 persons from Q. The one born in Noida was born just
1997 not necessarily in the same order. They were born in before M but not eldest. Nobody was born between the
the places Indore, Raipur, Hydrabad, Pune, Kanpur, ones born in Vizag and Raipur. The person born in
Manipur, Vizag and Noida not necessarily in the same Manipur was aged 36 years old. O was not born in Raipur
order. They were born on the same date i.e. on 17th and L was not born in Kanpur. The person born in
October. Make the age calculations as on 17th October Hyderabad was elder to the one born in Kanpur.
2019. 1. Who among the following person is from Pune?
S was 4 years elder to the one born in Pune. The one born (a) P
in Pune was born at a gap of 4 persons from P. Number (b) Q
of persons elder to the one born in Kanpur was one more (c) L
than that of number of persons younger to R. R was 8 (d) R
Page 652 of 1334
Subscribe The Xpress Video Course & Mock Test Package for Bank & Insurance Exams
If there are any suggestions/ errors in our PDFs Feel Free to contact us via this email: admin@exampundit.in
Ultra Practice Bundle PDF
SBI Clerk/ RBI Assist. Mains – Reasoning
(e) O Directions (6 - 10): Answer the questions based on the
2. Which of the following statements is true? information given below:
(a) M age is 40year Ten persons M, N, O, P, Q, R, S, T, U and V joined their
(b) N age is 52year company in the months of May, August, September,
(c) O age is 29year October and November on either 21st or 23rd of these
(d) Q age is 24 year months. No two persons joined on the same day. They
(e) L age is 52 year joined in the same year. Each person was aged differently.
3. Four of the following are alike in a certain way and The persons whose age was odd valued joined the office
thus form a group. Which of the following does not in the months with odd number of days and the persons
belong to the group? whose age was even valued joined the office in the months
(a) N - Noida with even number of days.
(b) M - Kanpur The person who joined the Company on 21st May was 17
(c) P - Raipur years old. Number of persons who joined the office after
(d) S - Pune M was thrice as the number of persons who joined before
(e) O - Noida the one aged 21. There were 3 persons who joined the
4. If ‘Hydrabad’ is related to ‘43’ and ‘Indore’ is office between the one who was aged 21 and V. V and O
related to ‘36’ in a certain way. Then who among joined the company in the same month. S did not join in
the following will be related to ‘Manipur’? November. P joined immediately before S. S was aged
(a) 24 twice as that of Q who joined the office just after T. U
(b) 29 joined the office just after R. R joined the company on a
(c) 32 23rd. There were 4 persons who joined the company
(d) 36 between T and the one who was aged 35 years old. The
(e) None of them one who was 35 years old joined the company before T.
5. Who among the following is eldest? M and T were born in consecutive years and T's age was
(a) S a perfect square. P was 5 years elder to R whose age was
(b) M a perfect square greater than 14. The person who was 33
(c) O years old and P joined company in consecutive months,
(d) N not necessarily on consecutive turns. The number of
(e) None of them persons, who joined before U, was equal to the number of

Page 653 of 1334


Subscribe The Xpress Video Course & Mock Test Package for Bank & Insurance Exams
If there are any suggestions/ errors in our PDFs Feel Free to contact us via this email: admin@exampundit.in
Ultra Practice Bundle PDF
SBI Clerk/ RBI Assist. Mains – Reasoning
persons who joined after the one who was aged 29 years. (d) September 21
Q was elder to U and they were born in consecutive years. (e) None of these
The eldest person was 44 years old. 10. How many persons between U and the one who
6. On which of the month and date M joined the joined company in November 23?
company? (a) Two
(a) May -21 (b) One
(b) August -21 (c) Five
(c) August – 23 (d) Three
(d) October -21 (e) Six
(e) October -23 Direction (11 -15): Study the given information
7. Which of the following is the age of S? carefully and answer the following questions.
(a) 21 Eight persons A, B, C, D, E, F, G and H are born in a
(b) 35 different month i.e. January, February, March, April,
(c) 44 May, June, July, August of different years. Then ages are
(d) 22 calculated with respect to 2020. Not consider month for
(e) None of these age calculation. No two person’s age is same.
8. What is the age difference between the one who Note: If it is mentioned that a person’s age is considered
joined company in May 21 and the one who joined as the last two digits of another person’s birth year, then it
the company in November 21? will be at any sequence. For example, if the age of X is
(a) 19 considered as last two digit of birth year of Y which is
(b) 24 1934. Then the age of X might be either 34 or 43. None of
(c) 26 the persons were born before 1970 and after 2010.
(d) 17 A was born on1988. A’s birth month is neither August nor
(e) 36 he born in first half of the year. H’s age is a square number
9. P has joined office on which of the following date? greater that A’s age and both are born on consecutive
(a) November 21 months. The difference between the age of A and E is 6
(b) May 21 years. C’s age is equal to the last two digit or birth year of
(c) September 23 E and his birth month is not April. F's age is a cube
number between E’s age and C’s age. D’s age is half of

Page 654 of 1334


Subscribe The Xpress Video Course & Mock Test Package for Bank & Insurance Exams
If there are any suggestions/ errors in our PDFs Feel Free to contact us via this email: admin@exampundit.in
Ultra Practice Bundle PDF
SBI Clerk/ RBI Assist. Mains – Reasoning
the age of G’s age whose age is last two digit of H’s birth (a) 55 years
year. The Average of ages of A and D is two less than that (b) 52 years
of the age of B who was born just next month on which D (c) 62 years
born. Person’s whose birth year is odd on the month, (d) 49 years
which has even number of days .The Person who is (e) None of these
youngest neither born in 30days of month nor born in 15. Which of the following statement is true?
31days Month. (a) F is younger than B
11. What is the age of H? (b) E is three year elder than G
(a) 49 years (c) None of the options are true
(b) 26 years (d) D is not the youngest person
(c) 49 years (e) Number of person born after B is not same as
(d) 36 Yeas before A.
(e) 48years Directions (16 - 20): Answer the questions based on the
12. Which of the following is the birth month - year of information given below:
C? There are eight persons E, F, G, H, U, V, W and X are
(a) January – 1988 born in different years viz., 2001, 1997, 1992, 1987, 1979,
(b) April - 1993 1972, 1964, and 1956 but not necessary in the same order.
(c) Feb – 1996 They are working for different company namely TCS,
(d) June – 1971 Infosys, Cognizant, Wipro, HCL, Jindal, Idea, and Airtel
(e) August – 1984 but not necessary in the same order. Age Calculations are
13. If I was born between A and F then which of the done with respect to the year 2020 and assuming months
following can be the birth year of I? and date to be same.
(a) 1987 U was born in one of the odd numbered year and does not
(b) 1985 working in TCS. There are only two persons born between
(c) 1982 the one who works for TCS and E, who does not, works
(d) 1993 for Airtel. The age of X is four times the age difference
(e) 1991 between the one who works for ldea and F. Age of the
14. What well be the sum of ages of Second youngest person who works for HCL is perfect square. Sum of the
and Third eldest person in the group? age of U and V is equal to perfect cube of a number. G is

Page 655 of 1334


Subscribe The Xpress Video Course & Mock Test Package for Bank & Insurance Exams
If there are any suggestions/ errors in our PDFs Feel Free to contact us via this email: admin@exampundit.in
Ultra Practice Bundle PDF
SBI Clerk/ RBI Assist. Mains – Reasoning
not elder than the person one who works for cognizant. (b) 22
There is only one person born between the one who works (c) 23
for Airtel and V, whose age is prime number. As many (d) 24
persons born between H and the one who works for Airtel (e) 20
is same as the persons born between V and the one who 19. What is the age of the person who works for jindal?
works for Infosys. Not more than three and less than two (a) 33
person born between W and the one who works for Jindal. (b) 48
E's age is multiples of three and does not born after 1982. (c) 41
F is five year elder then G. U’s age is Prime number and (d) 28
born immediately before the one who works for Idea. The (e) 23
person who works for Cognizant was born after the person 20. The person whose age is third highest works for
who works for Jindal. which company?
16. What is the age of the person who works for (a) TCS
Cognizant? (b) Idea
(a) 33years (c) Wipro
(b) 48 years (d) Infosys
(c) 28years (e) Jindal
(d) 17 years Directions (21 -25): Answer the questions based on the
(e) None of these information given below:
17. How many persons between the person whose birth Eight persons namely P, Q, R, S, T, U, V and W lives in
year is 1964 and the person who works for Wipro eight different floored building. Floors of building are
Company? marked 1 to 8 in such a way that floors are marked from
(a) 1 bottom to top. Each person was born in different year viz.
(b) 2 1985, 1993, 1978, 1995, 1997, 1994, 1998 and 1989. All
(c) 3 the information is not necessary in same order. Age of the
(d) 4 person is calculated from current year 2020.
(e) 5 Q lives on any floor below the one who was born in
18. The difference between age of G and E is? 1993. Number of person below the U is one third the
(a) 21 number of person below the one whose age is 25 years.

Page 656 of 1334


Subscribe The Xpress Video Course & Mock Test Package for Bank & Insurance Exams
If there are any suggestions/ errors in our PDFs Feel Free to contact us via this email: admin@exampundit.in
Ultra Practice Bundle PDF
SBI Clerk/ RBI Assist. Mains – Reasoning
At least one person lives between Q and the one whose 23. W born in which of the following year?
age is 27 years. Only two person lives between the one (a) 1997
whose age is 25 years and Q, whose age is not 27 years. (b) 1995
Sum of ages of the one who lives on floor 2 and Q is 53 (c) 1989
years. Two person lives between T and the one who was (d) 1998
born in 1993. Average sum of ages of the one who lives (e) 1993
on lowermost floor and V is one more than the age of 24. What is the age difference between the person
S. T, whose age is not odd number, lives just above the immediately above W and the person live on top
one whose age is multiple of five. W does not live even floor?
numbered floor, was neither born in 1985 nor is his age (a) Less than 10
is an even number. Age of S is neither odd number nor (b) Greater than 20
he lives below P. Number of person below T is two (c) Between 16 and 25
more than number of person above P. Age of Q is (d) Between 14 and 18
greater than V. More than two floor between R and S. (e) None of these
The one whose age is 25 years do not live in odd 25. Four of the following are alike in a certain way and
numbered floor. thus form a group. Which of the following does not
21. What is the difference of ages of R and Q? belong to the group?
(a) 9 (a) Q
(b) 8 (b) T
(c) 7 (c) P
(d) 6 (d) W
(e) Cannot be determined (e) V
22. Who lives in fourth floor and the age of that person Directions (26 - 30): Answer the questions based on the
is? information given below:
(a) V, 31 Eight Person B, F, H, J, U, W, Y and Z are born in
(b) E, 22 different years -1978, 1981, 1983, 1985, 1989, 1994, 1997
(c) R, 25 and 1999. Each person’s age are calculated with respect to
(d) P, 25 the current year (2020), same month and same date. Each
(e) None of these

Page 657 of 1334


Subscribe The Xpress Video Course & Mock Test Package for Bank & Insurance Exams
If there are any suggestions/ errors in our PDFs Feel Free to contact us via this email: admin@exampundit.in
Ultra Practice Bundle PDF
SBI Clerk/ RBI Assist. Mains – Reasoning
one of them has different number of chocolate from 1 to 8 (a) 1
but not in the same order. (b) 2
B and Y’s age difference and their difference of chocolate (c) 3
are same, in which their difference of ages will be the (d) 4
number of chocolate for J. Z is 4 years older than Y. Sum (e) 5
of number of chocolate of H and Z's is 8. U was born 29. Four of the following five are alike in a certain way
before 1997. Number of chocolate of U is below 3. and hence they form a group. Which one of the
Number of chocolate of J is above 4. Both H and Z's does following does not belong to that group?
not have even number of chocolate. Number of chocolate (a) J
of Y is three times of number of chocolate of Z. Three (b) Z
persons were born between H and W. W was born after H. (c) W
less than three persons were born between J and Y. More (d) F
than three persons were born between F and U. F was born (e) B
on odd number year. Sum of number of chocolate of F and 30. What is the sum of ages of the person who have 3
U is 6. and 4 chocolates respectively?
26. Who among following have 6 chocolate? (a) 50
(a) U (b) 49
(b) W (c) 52
(c) H (d) 56
(d) F (e) 48
(e) B Direction (31 - 35): Study the following information
27. What is the age of the person having 8 chocolate? carefully and answer the questions given below:
(a) 23 yr Eight people – A, B, C, D, E, F, G and H are born in
(b) 21yr different years from 1981 to 1988. Each person age are
(c) 26 yr calculated with respect to the current year (2020), same
(d) 35 yr month and same date. Each one of them likes different
(e) 39 yr brand of ice cream viz. Amul, Kwality Walls, Baskin
28. How many person are their between the one whose Robbins, Vadilal, Havmor, Dinshaw's, Mother Dairy,
birth year is 1981 and the one who has 4 chocolate? Naturals Ice Cream but not necessarily in the same order.

Page 658 of 1334


Subscribe The Xpress Video Course & Mock Test Package for Bank & Insurance Exams
If there are any suggestions/ errors in our PDFs Feel Free to contact us via this email: admin@exampundit.in
Ultra Practice Bundle PDF
SBI Clerk/ RBI Assist. Mains – Reasoning
Person name starts with consecutive alphabet does not 32. How many persons were born between H and the
born in consecutive years. one who likes Havmor?
A was born after 1982 but before 1986. Three persons (a) None
were born between A and the one who likes Amul Ice (b) One
cream. More than four persons were born between the one (c) Two
who likes Amul Ice cream and the one who likes Baskin (d) Three
Robbins. The one who likes Baskin Robbins is neither (e) None of these
born in 1986 nor in 1988. Two persons were born between 33. Which of the following person likes Amul Ice
B and the one who likes Mother Dairy. B was born after Cream?
the one who likes Mother Dairy. B neither born after 1986 (a) H
nor like Amul. The one who likes Kwality Walls was born (b) B
immediately after C. D likes Vadilal and is not the third (c) C
eldest. More than three persons were born between the one (d) A
who likes Vadilal and the one who likes Dinshaw's. The (e) None of these
one who like Dinshaw’s born after the one who like 34. If A is related Baskin Robbins, C is related to
Vadilal. E was born immediately before the one who likes Naturals Ice Cream, in the same way E is related to
Dinshaw's. E neither likes Naturals Ice Cream nor likes which of the following?
Kwality Walls. F neither likes Kwality Walls nor likes (a) Mother Dairy
Baskin Robbins. Neither B nor F likes Naturals Ice Cream. (b) Havmor
H does not like Naturals Ice Cream and Dinshaw's. (c) Dinshaw's
Neither G nor A likes Havmor. G does not like Baskin (d) Amul
Robbins. G is not the eldest. Neither F nor H likes Amul. (e) Naturals Ice Cream
D does not born immediately before G. 35. Which of the following person was born
31. G likes which of the following Ice Cream? immediately after C?
(a) Baskin Robbins (a) F
(b) Havmor (b) H
(c) Kwality Walls (c) A
(d) Amul (d) D
(e) Naturals Ice Cream (e) G

Page 659 of 1334


Subscribe The Xpress Video Course & Mock Test Package for Bank & Insurance Exams
If there are any suggestions/ errors in our PDFs Feel Free to contact us via this email: admin@exampundit.in
Ultra Practice Bundle PDF
SBI Clerk/ RBI Assist. Mains – Reasoning
Directions (36-40): Study the following information room. W is the eldest male member of the family and lives
carefully and answer the given questions: in E2. The persons on second generation are from same
There are nine members of a family E, F, G, H, I, U, V, W parent. U is cousin of F. X is the only daughter of W. X
and X which has 3 generations. Each were born in has only one daughter and no son whose age is either a
different years viz. 1965, 1951, 1971, 2000, 1973, 1996, cube or square of a number. V is the nephew of I, who has
1998, 1992 and 1994 but not necessarily in same order. only one daughter. G’s age is a prime number and doesn’t
Their ages were calculated with respect to 2021. They all live in F2. The person whose age is prime number is from
stayed in a multi-storey building in three different floors same parents. The person who is third eldest has not more
containing two rooms in each floor. Not more than two than one child.
and less than one stays in the same room; one room is left 36. How many males are there in the family?
vacant. The simple outline of the building is as shown in (a) One
the fig. below. (b) Two
(c) Three
(d) More than four
(e) Cannot be determined
37. In which room and with whom does I lives?
(a) E2 – V
(b) F2 – F
The difference between the sum of the ages of people
(c) G3 – F
staying in F2 and F3 is 4 years. G stays with his paternal
(d) E3 – W
uncle who has only one son. The person, who is half the
(e) E2 – V
age of H stays alone in G2 and is a female. Only F, who
38. How is U related to W and in which room does U
stays with his father, is the second eldest of the family. G
lives?
and his elder sister whose age is 29years, lives in same
(a) Mother – E3
floor, but not in same room. The people who born in leap
(b) Daughter – G2
year were not siblings. The minimum age difference
(c) Grandson – E2
between each generation is 12 years. There is only one
(d) Sister – G3
floor between the floor in which the room is vacant and
(e) Granddaughter – G2
the floor in which a person stays alone. The Grandfather
39. What is the age difference between G and H?
and the one whose age is 21 were staying in the same
Page 660 of 1334
Subscribe The Xpress Video Course & Mock Test Package for Bank & Insurance Exams
If there are any suggestions/ errors in our PDFs Feel Free to contact us via this email: admin@exampundit.in
Ultra Practice Bundle PDF
SBI Clerk/ RBI Assist. Mains – Reasoning
(a) 21 years Citizen. Aadish does not born in odd numbered year. More
(b) 27 years than two persons born between Aanshi and one who wears
(c) 28 years Diesel. As many persons born between Aabhas and the
(d) 31 years one who wears Gucci is same as the persons born after the
(e) 35 years one who wears Rotary. There are two persons born
40. Which of the following statement is true? between Aadish and the one who wears Rolex. Age of the
(a) X is the Paternal Aunt of V one who wears Citizen is not a prime number but born in
(b) U is the only Granddaughter of W one of the months before the one who wears Rotary. There
(c) F is 4 years elder to V are only two persons born between the one who wears
(d) Both F and X stays together in the building Citizen and the one wears Boss. Only one person born
(e) Age difference between G and H is 23 years between Aadarsh and the one who wears Rolex, who does
Direction (41-45): Read the following information not born in the year 1964. Aadesh and the one who wears
carefully and answer the questions given below. Diesel are born in different month. Aabhas was born
Eight persons Aabhas, Aadarsh, Aadesh, Aadish, immediately after the one who wears Aansh. Aanshi does
Aaghosh, Aansh, Aapeksha and Aanshi are born in four not like Gucci. Aansh does not born immediately before
different years viz., 1964, 1977, 1982 and 1991 and also or after the one who wear Rotary.
in two different months viz., May and November but not 41. Who among the following is the Youngest?
necessary in the same order. All they are wearing eight (a) The one who wears Armani
different watches namely Casio, Boss, Rotary, Citizen, (b) Aanshi
Diesel, Gucci, Rolex and Armani. No two persons born in (c) The one who born immediately after Aansh
same month of a year. Age of the persons is calculated as (d) Aabhas
on December 2020. (e) Both (a) and (b)
Each year two persons were born in two different months. 42. The one who wears Casio born in which month and
Months are not considered for age calculation. The person year?
who wears Casio watch born immediately before Aansh. (a) May, 1977
Aansh was born in the month which has 31 days. The sum (b) November, 1982
of the ages of Aadarsh and the one who wears Boss is (c) May, 1982
equal to perfect square of a number. There are only three (d) November, 1964
persons born between Aagosh and the one who wears (e) None of these

Page 661 of 1334


Subscribe The Xpress Video Course & Mock Test Package for Bank & Insurance Exams
If there are any suggestions/ errors in our PDFs Feel Free to contact us via this email: admin@exampundit.in
Ultra Practice Bundle PDF
SBI Clerk/ RBI Assist. Mains – Reasoning
43. Four of the following alike in a certain way and day of 2020 as their date of births. (Note: consider only
thus form a group. Which of the following one that numerical value not its units).
does not belong to the group? S’s age is same as that of Q’s height. Sum of U and S’s
(a) Aadish height is equal to 77. P is three years elder than R. T born
(b) Aansh in a year which is not less than 366 days and its resultant
(c) Aanshi of sum of digits is divisible by 5. V is younger than T. Q’s
(d) Aadesh age is square of a number which is not more than S’s
(e) Aapeksha height. The difference between birth year of R and X is
44. If ‘Monika’ is born between ‘Aadrash’ and 17. Height of W is six times the difference of ages of U
‘Aadesh’, then what can be the age of ‘Monika’? and Q. P’s height is divisible by both 5 and 9. V was born
(a) 33 years in 1969. Square of the difference between Q’s age and T’s
(b) 56 years height is 289. Difference of twice the height of V and one
(c) 29 years fourth of U’s age is 73. U born after X whose age
(d) 38 years difference is 120 months. X is shorter than R both of them
(e) None of these has odd number heights. P’s age is as same as that of S’s
45. How many person born between the person who height. U’s height is prime number which is below 41 cm.
born in November 1964 and Aapeksha? S is 4 years elder than X. T is not taller than X. Age of W
(a) One is average height of Q and S. Average of R and X’s height
(b) Two is 52. Difference between the height of both R and X is
(c) Three not more than 2cm.
(d) Four 46. U’s height and X’s age is?
(e) More than four (a) 37 and 44
Direction (46-50): Study the following information (b) 37 and 54
carefully and answer the given questions. (c) 55 and 37
Nine persons P, Q, R, S, T, U, V, W and X are born in (d) 37 and 53
same months of different year. All are have different Ages (e) None of these
and Height in cm. Their heights and ages are between 34 47. What is the sum of Q’s height and S’s age?
- 62. Their ages are considered as on the same month and (a) 105
(b) 115

Page 662 of 1334


Subscribe The Xpress Video Course & Mock Test Package for Bank & Insurance Exams
If there are any suggestions/ errors in our PDFs Feel Free to contact us via this email: admin@exampundit.in
Ultra Practice Bundle PDF
SBI Clerk/ RBI Assist. Mains – Reasoning
(c) 116 (a) 6
(d) 106 (b) 5
(e) None of these (c) 8
48. T is how many years younger than S? (d) 9
(a) 7 (e) None of these
(b) 6 50. What is W’s age?
(c) 5 (a) 52
(d) 4 (b) 57
(e) None of these (c) 49
49. If X’s height is five cm more than T’s height, what (d) 36
is difference of height between X and S’s height? (e) None of these

Page 663 of 1334


Subscribe The Xpress Video Course & Mock Test Package for Bank & Insurance Exams
If there are any suggestions/ errors in our PDFs Feel Free to contact us via this email: admin@exampundit.in
Ultra Practice Bundle PDF
SBI Clerk/ RBI Assist. Mains – Reasoning
Puzzle Age – Answer and Explanation
SOLUTION (1-5):
Explanation in detail:
13. S was 4 years elder to the one born in Pune.
14. The one born in Pune was born at a gap of 4 persons
from P.
15. The person born in Manipur was aged 36 years old.
CASE1:

CASE2:

CASE2:
18. Number of persons elder to the one born in Kanpur
was one more than that of number of persons younger
to R.
From this statement CASE 2 was wrong. We can
go with CASE 1
CASE1:

16. R was 8 years elder to O.


17. Not more than two persons were born between S and
R
CASE1:

Page 664 of 1334


Subscribe The Xpress Video Course & Mock Test Package for Bank & Insurance Exams
If there are any suggestions/ errors in our PDFs Feel Free to contact us via this email: admin@exampundit.in
Ultra Practice Bundle PDF
SBI Clerk/ RBI Assist. Mains – Reasoning
7. The one born in Raipur was born after 1983 but not on
even year.
8. Nobody was born between the ones born in Vizag and
Raipur
CASE1:

1. c
2. e
3. e
4. b
5. a
9. The one born in Noida was born just before M but not SOLUTION (6-10):
eldest Explanation in detail:
10. N was born in Indore at a gap of 4 persons from Q. 10. The person who joined the Company on 21st May was
CASE1: 17 years old.
11. Number of persons who joined the office after M was
thrice as the number of persons who joined before the
one aged 21.
12. There were 3 persons who joined the office between
the one who was aged 21 and V.
13. V and O joined the company in the same month.
CASE 1

11. O was not born in Raipur and L was not born in


Kanpur
12. The person born in Hyderabad was elder to the one
born in Kanpur.
So final arrangement

Page 665 of 1334


Subscribe The Xpress Video Course & Mock Test Package for Bank & Insurance Exams
If there are any suggestions/ errors in our PDFs Feel Free to contact us via this email: admin@exampundit.in
Ultra Practice Bundle PDF
SBI Clerk/ RBI Assist. Mains – Reasoning
15. S was aged twice as that of Q who joined the office
just after T
From these statements CASE 3 eliminated.
CASE 1

CASE 2

CASE 2

CASE 3

7. U joined the office just after R.


8. R joined the company on a 23rd. P joined immediately
before S (simulataneoulsy not possible so this CASE
1 is eleminated.
CASE 2
14. There were 4 persons who joined the company
between T and the one who was aged 35 years old

Page 666 of 1334


Subscribe The Xpress Video Course & Mock Test Package for Bank & Insurance Exams
If there are any suggestions/ errors in our PDFs Feel Free to contact us via this email: admin@exampundit.in
Ultra Practice Bundle PDF
SBI Clerk/ RBI Assist. Mains – Reasoning
15. Q was elder to U and they were born in consecutive
years.
16. The eldest person was 44 years old.
17. S was aged twice as that of Q
CASE 2

9. P was 5 years elder to R whose age was a perfect


square greater than 14
10. P joined immediately before S.
11. The person who was 33 years old and P joined
company in consecutive months, not necessarily on
consecutive turns.
So final arrangement
12. M and T were born in consecutive years and T's age
was a perfect square.
13. P joined immediately before S.
CASE 2

6. c
7. c
8. a

14. The number of persons, who joined before U, was 9. d

equal to the number of persons who joined after the 10. e

one who was aged 29 years. SOLUTION (11-15):


Explanation in detail:
Page 667 of 1334
Subscribe The Xpress Video Course & Mock Test Package for Bank & Insurance Exams
If there are any suggestions/ errors in our PDFs Feel Free to contact us via this email: admin@exampundit.in
Ultra Practice Bundle PDF
SBI Clerk/ RBI Assist. Mains – Reasoning
Ages was calculated with respect to 2020 94, and 49. As mention in the instruction. C’s age
1. A was born on1988 hence age is 32. A’s birth month is is 28 and 49.
neither August nor he born in first half of the year. CASE 1
2. The difference between the age of A and E is 6 years.
3. A’s birth month is neither August nor he born in first
half of the year. It is clear that A birth month is July
CASE1:

CASE 2

CASE2:

5. F's age is a cube number between E’s age and C’s age.
Age of F is 27.
From here CASE1 is eliminated.
CASE 2

4. C’s age is equal to the last two digit or birth year


of E and his birth month is not April. C’s is 82, 28,

Page 668 of 1334


Subscribe The Xpress Video Course & Mock Test Package for Bank & Insurance Exams
If there are any suggestions/ errors in our PDFs Feel Free to contact us via this email: admin@exampundit.in
Ultra Practice Bundle PDF
SBI Clerk/ RBI Assist. Mains – Reasoning

6. H’s age is a square number greater that A’s age and


both are born on consecutive months.
7. Person’s whose birth year is odd on the month, which 9. The Average of ages of A and D is two less than that

has even number of days. of the age of B who was born just next month on which

CASE 2 D born.
CASE 2

8. D’s age is half of the age of G’s age whose age is


last two digit of H’s birth year.
10. The Person who is youngest Neither born in 30days of
CASE 2
month nor born in 31days Month
11. C’s birth month is not April
12. Person’s whose birth year is odd on the month, which
has even number of days.

Page 669 of 1334


Subscribe The Xpress Video Course & Mock Test Package for Bank & Insurance Exams
If there are any suggestions/ errors in our PDFs Feel Free to contact us via this email: admin@exampundit.in
Ultra Practice Bundle PDF
SBI Clerk/ RBI Assist. Mains – Reasoning
Note: In the year 2020 February month has 29 days. So
persons born in April and June are odd numbered years.
CASE 2

11. d
12. d
13. e
13. B who was born just next month on which D born. 14. c
14. For E and G month is not given. So 15. A
CASE2 SOLUTION (16-20):
Explanation in detail:
12. U born in one of the odd numbered year
13. Sum of the age of U and V is equal to perfect cube of
a number
14. U’s age is Prime number and born immediately before
the one who works for Idea.
CASE1:

Final Arrangement

Page 670 of 1334


Subscribe The Xpress Video Course & Mock Test Package for Bank & Insurance Exams
If there are any suggestions/ errors in our PDFs Feel Free to contact us via this email: admin@exampundit.in
Ultra Practice Bundle PDF
SBI Clerk/ RBI Assist. Mains – Reasoning

CASE2: CASE 2b

15. There is only one person born between the one who 17. There are only two persons born between the one who
works for Airtel and V, whose age is prime number. works for TCS and E, who does not, works for Airtel.
16. E's age is multiples of three and does not born after From here CASE 2b also eliminated
1982. 18. F is five year elder then G.
CASE1 is eliminated and CASE 2 has 2 19. The age of X is four times the age difference between
possibilities. the one who works for ldea and F

CASE 2a CASE 2a

Page 671 of 1334


Subscribe The Xpress Video Course & Mock Test Package for Bank & Insurance Exams
If there are any suggestions/ errors in our PDFs Feel Free to contact us via this email: admin@exampundit.in
Ultra Practice Bundle PDF
SBI Clerk/ RBI Assist. Mains – Reasoning
So Final Arrangement

20. Age of the person who works for HCL is perfect


square.
21. G is not elder than the person one who works for 16. a

cognizant 17. d

22. Age of the person who works for HCL is perfect 18. e

square. 19. c

23. As many persons born between H and the one who 20. d

works for Airtel is same as the persons born between SOLUTION (21-25): (fully modified)

V and the one who works for Infosys. First Calculate age as shown in figure

24. Not more than three and less than two person born
between W Only one person sits between Z’s mother
and W.
CASE 2a

Explanation in detail:
14. Number of person below the U is one third the
number of person below the one whose age is 25
years.
Page 672 of 1334
Subscribe The Xpress Video Course & Mock Test Package for Bank & Insurance Exams
If there are any suggestions/ errors in our PDFs Feel Free to contact us via this email: admin@exampundit.in
Ultra Practice Bundle PDF
SBI Clerk/ RBI Assist. Mains – Reasoning
15. The one whose age is 25 years do not live in odd Case-1
numbered floor. Floor Person Age year
Case-1 8
Floor Person Age year 7
8 6
7 5
6 4 25 1995
5 3
4 25 1995 2 U 22 1998
3 1 Q 31 1989
2 U
1 Case-2 (rejected)
16. Only two person lives between the one whose age At least one person lives between Q
is 25 years and Q, whose age is not 27 years. and the one whose age is 27 years.
17. At least one person lives between Q and the one Floor Person Age year
whose age is 27 years. 8 27 1993
18. Sum of ages of the one who lives on floor 2 and Q 7 Q 27
is 53 years. 6
19. Q lives on any floor below the one who was born 5
in 1993. 4 25 1995
20. Age of Q is greater than V 3
Brief hints: 2 U
2 possibilities to make 53 (31, 22) & (27, 26) 1
➔In case-1 (27, 26) combination is not possible for Q and 21. Two person lives between T and the one who was
2nd floor born in 1993.
Note: If Q’s age is 22 and then V’s age must be less than 22. T, whose age is not odd number, lives just above
22 years which is not possible as per age calculation table. the one whose age is multiple of five.
Hence Q’s age must be 31 years and U’s age must be 22 23. At least one person lives between Q and the one
years. whose age is 27 years.

Page 673 of 1334


Subscribe The Xpress Video Course & Mock Test Package for Bank & Insurance Exams
If there are any suggestions/ errors in our PDFs Feel Free to contact us via this email: admin@exampundit.in
Ultra Practice Bundle PDF
SBI Clerk/ RBI Assist. Mains – Reasoning
24. Number of person below T is two more than 5 T
number of person above P. 4 25 1995
Case-1 3
Floor Person Age year 2 U 22 1998
8 1 Q 31 1989
7 25. Age of S is neither odd number nor he lives below
6 T P.
5 P 26. More than two floor between R and S.
4 25 1995 Case-1
3 27 1993 Floor Person Age year
2 U 22 1998 8
1 Q 31 1989 7 S
6 T
Case-3 5 P
Floor Person Age year 4 25 1995
8 T 3 R 27 1993
7 2 U 22 1998
6 1 Q 31 1989
5 27 1993
4 25 1995 Case-1-A
3 P Floor Person Age year
2 U 22 1998 8 S
1 Q 31 1989 7
6 T
Case-4 5 P
Floor Person Age year 4 R/ 25 1995
8 27 1993 3 R/ 27 1993
7 2 U 22 1998
6 P 1 Q 31 1989

Page 674 of 1334


Subscribe The Xpress Video Course & Mock Test Package for Bank & Insurance Exams
If there are any suggestions/ errors in our PDFs Feel Free to contact us via this email: admin@exampundit.in
Ultra Practice Bundle PDF
SBI Clerk/ RBI Assist. Mains – Reasoning
Floor Person Age year
Case-3 (Rejected) 8
More than two floor between R and 7 S
S (not possible) 6 T
Floor Person Age year 5 P
8 T 4 25 1995
7 3 R 27 1993
6 2 U 22 1998
5 27 1993 1 Q 31 1989
4 25 1995
3 P Case-1-A
2 U 22 1998 Floor Person Age year
1 Q 31 1989 8 S
7 W/
Case-4 6 T
Floor Person Age year 5 P
8 S 27 1993 4 R/ 25 1995
7 S 3 R/W 27 1993
6 P 2 U 22 1998
5 T 1 Q 31 1989
4 25 1995
3 R Case-4 (Rejected)
2 U 22 1998 W does not live in even numbered floor
1 Q 31 1989 Floor Person Age year
8 27 1993
27. W does not live even numbered floor, was neither 7 S
born in 1985 nor is his age is an even number. 6 P
Case-1 (Rejected) 5 T
W does not live in even numbered floor 4 25 1995

Page 675 of 1334


Subscribe The Xpress Video Course & Mock Test Package for Bank & Insurance Exams
If there are any suggestions/ errors in our PDFs Feel Free to contact us via this email: admin@exampundit.in
Ultra Practice Bundle PDF
SBI Clerk/ RBI Assist. Mains – Reasoning
3 R 5 P 35 1985
2 U 22 1998 4 R 25 1995
1 Q 31 1989 3 W 27 1993
28. Average sum of ages of the one who lives on 2 U 22 1998
lowermost floor and V is one more than the age of 1 Q 31 1989
S. 21. d
29. Age of Q is greater than V. 22. c
Note: S’s age is even number. T’s age is even number. T 23. e
lives immediately above the one, whose age is multiple of 24. A
5. 25. b
Even numbered ages are 42, 26 and 22. SOLUTION (26-30):
As we know U’s age is 22. Therefore S and T ages are 42 Explanation in detail:
and 26, but in any order. 24. Z is 4 years older than Y.
Q’s age is 31 (lowermost floor) CASE 1
To find V’s age:
Given, (Q+V)/2=1+S
If S age is 26, Q+V=54 and then V’s age is 23 (54-31=23)
If S age is 42, Q+V=86 and then V’s age is 55 (86-31=55,
not possible as age 55 is not given)
Thus S’s age is 26, T’s age is 42 and V’s age is 23.
Also V must live on 7th floor. This implies W lives on 3rd
floor and R lives on 4th floor.
Finally, P’s age is 35 years.
So Final Arrangement is
CASE 2
Case-1-A
Floor Person Age year
8 S 26 1994
7 V 23 1997
6 T 42 1978

Page 676 of 1334


Subscribe The Xpress Video Course & Mock Test Package for Bank & Insurance Exams
If there are any suggestions/ errors in our PDFs Feel Free to contact us via this email: admin@exampundit.in
Ultra Practice Bundle PDF
SBI Clerk/ RBI Assist. Mains – Reasoning

25. Sum of number of chocolate of H and Z's is 8


26. H and Z's does not have even number of chocolate. 28. B and Y’s age difference and their difference of

27. Number of chocolate of Y is three times of number of chocolate are same, in which their difference of ages

chocolate of Z. will be the number of chocolate for J.

H + Z = 8 => 7 + 1= 8 and number of chocolate Y is 3 B –Y = 5(chocolate and age ) => 8 – 3 = 5

CASE 1 29. Number of chocolate of J is above 4. Number of


chocolate of J is 5
CASE 2 eliminated
CASE1

CASE 2

30. Number of chocolate of U is below 3

Page 677 of 1334


Subscribe The Xpress Video Course & Mock Test Package for Bank & Insurance Exams
If there are any suggestions/ errors in our PDFs Feel Free to contact us via this email: admin@exampundit.in
Ultra Practice Bundle PDF
SBI Clerk/ RBI Assist. Mains – Reasoning
31. Three persons were born between H and W. W was
born after H.
32. Sum of number of chocolate of F and U is 6.
F + U = 6 => 4 + 2 = 6 and W’s chocolate is 6
CASE 1

26. b
27. c
28. e
29. e
30. c
33. Less than three persons were born between J and Y.
SOLUTION (31-35):
34. More than three persons were born between F and U.
Explanation in detail:
35. F was born on odd number year.
12. A was born after 1982 but before 1986.
CASE1
13. Three persons were born between A and the one who
likes Amul Ice cream.
14. More than four persons were born between the one
who likes Amul Ice cream and the one who likes
Baskin Robbins.
15. The one who likes Baskin Robbins is neither born in
1986 nor in 1988.
CASE 1

So final answer is
Page 678 of 1334
Subscribe The Xpress Video Course & Mock Test Package for Bank & Insurance Exams
If there are any suggestions/ errors in our PDFs Feel Free to contact us via this email: admin@exampundit.in
Ultra Practice Bundle PDF
SBI Clerk/ RBI Assist. Mains – Reasoning

CASE2 CASE 2a

16. Two persons were born between B and the one who
CASE2b
likes Mother Dairy.
17. B was born after the one who likes Mother Dairy.
18. B neither born after 1986 nor like Amul.
CASE 1

Page 679 of 1334


Subscribe The Xpress Video Course & Mock Test Package for Bank & Insurance Exams
If there are any suggestions/ errors in our PDFs Feel Free to contact us via this email: admin@exampundit.in
Ultra Practice Bundle PDF
SBI Clerk/ RBI Assist. Mains – Reasoning

19. D likes Vadilal and is not the third eldest. CASE 2a


20. More than three persons were born between the one
who likes Vadilal and the one who likes Dinshaw's.
21. The one who like Dinshaw’s born after the one who
like Vadilal.
CASE 1

22. E was born immediately before the one who likes


Dinshaw's
23. The one who likes Kwality Walls was born
immediately after C.
CASE 2a Eliminated

CASE 2

Page 680 of 1334


Subscribe The Xpress Video Course & Mock Test Package for Bank & Insurance Exams
If there are any suggestions/ errors in our PDFs Feel Free to contact us via this email: admin@exampundit.in
Ultra Practice Bundle PDF
SBI Clerk/ RBI Assist. Mains – Reasoning

CASE1

CASE2

CASE2

From here CASE2 eliminated. By “Neither F nor H likes


Amul” This statements

24. F neither likes Kwality Walls nor likes Baskin 29. Neither B nor F likes Naturals Ice Cream

Robbins. 30. Neither G nor A likes Havmor.

25. Neither B nor F likes Naturals Ice Cream Final arrangement


26. G does not like Baskin Robbins.
27. G is not the eldest.
28. Neither F nor H likes Amul.
CASE1

Page 681 of 1334


Subscribe The Xpress Video Course & Mock Test Package for Bank & Insurance Exams
If there are any suggestions/ errors in our PDFs Feel Free to contact us via this email: admin@exampundit.in
Ultra Practice Bundle PDF
SBI Clerk/ RBI Assist. Mains – Reasoning

16. W is grandfather
17. X is the only daughter of W.
18. X has only one daughter and no son whose age is
either a cube or square of a number.
31. c
32. c
33. e
34. a
35. e
SOLUTION (36-40): (fully modified)
From the given statements first we find the ages and leap
year as per year 2021.
Figure-1
19. The minimum age difference between each generation
is 12 years.

Figure-2
Explanation in detail:
20. G stays with his paternal uncle who has only one son.
15. W is the eldest male member of the family and lives
21. G and his elder sister whose age is 29years, lives in
in E2. It means W is grandfather
same floor, but in same room.

Page 682 of 1334


Subscribe The Xpress Video Course & Mock Test Package for Bank & Insurance Exams
If there are any suggestions/ errors in our PDFs Feel Free to contact us via this email: admin@exampundit.in
Ultra Practice Bundle PDF
SBI Clerk/ RBI Assist. Mains – Reasoning
22. G’s age is a prime number and doesn’t live in F2.

Figure 3
23. U is cousin of F.
Figure-5
24. V is the nephew of I, who has only one daughter.
28. We know that F’s father’s (male) age is 56 (figure-4)
25. Only F stays with his father, who is the second eldest
and H’s age is 50 (figure-5). Therefore by combining
of the family.
figure-1 & figure-4 with figure -5 we can get that W’s
only daughter is X, whose age is 48.
29. Now X and H are from 2nd generation. Therefore I
must be from 2nd generation (from figure-4). This
implies that I must be the father of F.

Figure-4
26. The person, who is half the age of H stays alone in G2
and is a female.
27. There is only one floor between the floor in which the
room is vacant and the floor in which a person stays
alone.
30. Now we can get clear that E, F, U, V, and G are from
3rd generation.
31. From figure-3 we can confirm that there are two males
in second generation. I’s brother must be H. Thus we
get
32. The persons on second generation are from same
parent. From here it clear that H and I are son of W.

Page 683 of 1334


Subscribe The Xpress Video Course & Mock Test Package for Bank & Insurance Exams
If there are any suggestions/ errors in our PDFs Feel Free to contact us via this email: admin@exampundit.in
Ultra Practice Bundle PDF
SBI Clerk/ RBI Assist. Mains – Reasoning

From figure-3 we can confirm that G must be the son


of I (If G is son of H, G can’t have elder sister) 37. The Grandfather and the one whose age is 21 were
That is G’s paternal uncle is H, whose only son is V. staying in the same room.
E is only daughter of I and U is the daughter of X 38. G and his elder sister whose age is 29years, lives in
33. The person who is third eldest has not more than one same floor, but not in same room.
child 39. G’s age is a prime number and doesn’t live in F2

40. G stays with his paternal uncle


34. The people who born in leap year were not siblings 41. Only F, who stays with his father
35. The person whose age is prime number is from same
parents.
36. U is cousin of F
From here it is clear that F and G is Son of I.
Final blood relation

Now only X is left


42. The difference between the sum of the ages of people
staying in F2 and F3 is 4 years
Page 684 of 1334
Subscribe The Xpress Video Course & Mock Test Package for Bank & Insurance Exams
If there are any suggestions/ errors in our PDFs Feel Free to contact us via this email: admin@exampundit.in
Ultra Practice Bundle PDF
SBI Clerk/ RBI Assist. Mains – Reasoning
Final Arrangement CASE 2

36. d
37. c
38. e
39. b CASE 3
40. a
SOLUTION(41-45):
Explanation in detail:
11. Aadish does not born in odd numbered year.
12. There are two persons born between Aadish and the
one who wears Rolex.
13. Only one person born between Aadarsh and the one
who wears Rolex, who does not born in the year 1964.
14. The sum of the ages of Aadarsh and the one who wears
CASE 4
Boss is equal to perfect square of a number.
CASE 1

Page 685 of 1334


Subscribe The Xpress Video Course & Mock Test Package for Bank & Insurance Exams
If there are any suggestions/ errors in our PDFs Feel Free to contact us via this email: admin@exampundit.in
Ultra Practice Bundle PDF
SBI Clerk/ RBI Assist. Mains – Reasoning
15. The sum of the ages of Aadarsh and the one who wears 19. The person who wears Casio watch born immediately
Boss is equal to perfect square of a number before Aansh
16. Age of the one who wears Citizen is not a prime 20. Age of the one who wears Citizen is not a prime
number but born in one of the months before the one number but born in one of the months before the one
who wears Rotary who wears Rotary
17. There are only two persons born between the one who From this CASE 4 is eliminated now we go with
wears Citizen and the one wears Boss. CASE 1.
18. There are only three persons born between Aagosh CASE 1
and the one who wears Citizen
Case 3 and case 2 eliminated
CASE 1

21. As many persons born between Aabhas and the one


who wears Gucci is same as the persons born after the
one who wears Rotary
CASE 4
22. Aadesh and the one who wears Diesel are born in
different month.
23. Aabhas was born immediately after the one who wears
Aansh.
CASE 1

Page 686 of 1334


Subscribe The Xpress Video Course & Mock Test Package for Bank & Insurance Exams
If there are any suggestions/ errors in our PDFs Feel Free to contact us via this email: admin@exampundit.in
Ultra Practice Bundle PDF
SBI Clerk/ RBI Assist. Mains – Reasoning
41. e
42. d
43. d
44. a
45. b
SOLUTION(46-50):
Explanation in detail:
Their heights and ages are between 34-62. So birth year

24. More than two persons born between Aanshi and are between 1958 – 1988.

one who wears Diesel. 1. Sum of U and S’s height is equal to 77.

25. Aanshi does not like Gucci. 2. V was born in 1969.


3. P’s age is as same as that of S’s height.
4. U’s height is prime number which is below 41 cm.

Final arrangement

5. P is three years elder than R.


6. Q’s age is square of a number which is not more than
S’s height.
7. The difference between birth year of R and X is 17.
8. U born after X whose age difference is 120 months.

Page 687 of 1334


Subscribe The Xpress Video Course & Mock Test Package for Bank & Insurance Exams
If there are any suggestions/ errors in our PDFs Feel Free to contact us via this email: admin@exampundit.in
Ultra Practice Bundle PDF
SBI Clerk/ RBI Assist. Mains – Reasoning
9. X is shorter than R both of them has odd number
heights.
10. Average of R and X’s height is 52

15. S’s age is same as that of Q’s height.


16. V is younger than T.
17. Age of W is average height of Q and S
11. S is 4 years elder than X.
18. Square of the difference between Q’s age and T’s
12. Height of W is six times the difference of ages of U
height is 289.
and Q.
13. P’s height is divisible by both 5 and 9.
14. T born in a year which is not less than 366 days and
its resultant of sum of digits is divisible by 5. Hence
only two possibilities is there 1960 and 1964. Sum is
divisible by 5 hence 1964 is T’s birth year.

Page 688 of 1334


Subscribe The Xpress Video Course & Mock Test Package for Bank & Insurance Exams
If there are any suggestions/ errors in our PDFs Feel Free to contact us via this email: admin@exampundit.in
Ultra Practice Bundle PDF
SBI Clerk/ RBI Assist. Mains – Reasoning
19. Difference of twice the height of V and one fourth of
U’s age is 73. .

46. b
47. C
48. E
Final arrangement 49. A
50. C

Download Puzzles Practice Questions PDF


Get More Reasoning Practice Questions PDF
Seating with Ages
Directions (1-5): Read the following information direction as the person who is fifty years old. The age of
carefully and answer the questions given below. immediate neighbors of L are 18 and 21 years old. V and
Eight persons V, H, B, G, L, J, T and N are sitting around J are fifty and fifty five years old respectively. L sits to the
a circular table but not necessarily in the same order. Four immediate left of T. The person who is twenty six years
of them are facing inside and the other four are facing old is facing outward and is an immediate neighbor of the
outside. All eight persons have different ages. person who is twenty one years old. N is an immediate
G sits third to the right of H, who is one year older than neighbor of the persons who are fifty and twenty six years
the person who sits to the immediate left of L. The person old. B faces the centre and sits third to the right of J. T is
who is sitting second to the left of G is facing the same eighteen years old and faces the person who is thirty years

Page 689 of 1334


Subscribe The Xpress Video Course & Mock Test Package for Bank & Insurance Exams
If there are any suggestions/ errors in our PDFs Feel Free to contact us via this email: admin@exampundit.in
Ultra Practice Bundle PDF
SBI Clerk/ RBI Assist. Mains – Reasoning
old. Age of L is equal to the number obtained by reversing 5. Who among the following sits third to the left of J?
the digits of H's age. V does not sit second to the left of G. (a) N
1. Who among the following is 26 years old? (b) H
(a) E (c) B
(b) J (d) L
(c) H (e) T
(d) G Directions (6-10): Solve the questions based on the
(e) B following given information carefully.
2. What is the age of H? There are eight people A, B, C, D, E, F, G and H sitting
(a) 22 years around a circular table for the lunch. Some of them are
(b) 19 years facing towards the center of the table while some are
(c) 24 years facing opposite to the center of the table. They all are of
(d) 25 years different age viz. 11, 13, 15, 17, 18, 19, 27 and 31 year but
(e) 26 years not necessarily in the same order. A is sitting third to the
3. Who among the following sits second to the left of right of C. A's age is 31 years old. Only one person is
B? sitting between A and B. D is sitting fourth to the right of
(a) H B. The one whose age is 15 years old sits second to the
(b) T right of D. Three persons sit between the one whose age
(c) J is 15 years and the one whose age is 31 years . The one
(d) G whose age is 18 years sits second to the right of one
(e) N whose age is 31 years. D is not 18 years old. The one who
4. Four of the following five are alike in a certain way is 17 years old sits third to the right of the one who is 18
and hence they form a group. Which one of the years old. C's age is 17 years. There are two persons sit
following does not belong to that group? between D and E. Three persons sit between the one
(a) B whose age is 13 years and the one whose age is 19 years.
(b) H E's age is neither 13 nor 19 years old. E faces the center of
(c) V the table. There is only one person sits between the one
(d) T whose age is 19 years and the one whose age is 27 years.
(e) N The one who is 15 years old sits third to the right of the

Page 690 of 1334


Subscribe The Xpress Video Course & Mock Test Package for Bank & Insurance Exams
If there are any suggestions/ errors in our PDFs Feel Free to contact us via this email: admin@exampundit.in
Ultra Practice Bundle PDF
SBI Clerk/ RBI Assist. Mains – Reasoning
one whose age is 27 years. The one who is 11 years old (c) 3
sits third to the right of the one whose age is 19 years. G (d) 2
sits fourth to the right of the one whose age is 13 years. F (e) 1
is not an immediate neighbor of G. The one whose age is 10. What is the total sum of age of the immediate
13 years sits third to the left of the one whose age is 15 neighbors of G?
years and both faces the same direction. (a) 32
6. What is the age of E? (b) 30
b. 15 years old (c) 29
c. 11 years old (d) 25
d. 27 years old (e) 33
e. 19 years old Directions (11-15): Solve the questions based on the
f. 18 years old following given information carefully.
7. Who sits third to the right of the one whose age is Ten persons are sitting in two parallel rows containing five
11 years? seats in each row in such a way that there is an equal
(a) C distance between adjacent persons. In row 1 – E, F, G, H
(b) F and I – are seated and all of them are facing in north
(c) E direction but not necessarily in the same order and in row
(d) G 2 – U, V, W, X and Y – are seated and all of them are
(e) H facing in south direction but not necessarily in the same
8. Which of the following are immediate neighbors of order. Every person in row1 faces exactly another person
A? in row2. The ages of the persons who are sitting in row1
(a) B, E are multiple of 4. The ages of the persons who are sitting
(b) The one whose age is 27 and 13 years. in row 2 are multiple of 3. No two persons are of the same
(c) H, E age.
(d)The one whose age is 27 and 11 years. The person who faces F sits third to the right of the one
(e) G, B whose age is 36 years. The difference between the ages of
9. How many people are facing towards the centre? E and W is 17 years. Only one person sits between G and
(a) 5 H. Neither G nor H faces the person whose age is 36 years.
(b) 4 Either U or X is of 36 years old. Four persons sit to the left

Page 691 of 1334


Subscribe The Xpress Video Course & Mock Test Package for Bank & Insurance Exams
If there are any suggestions/ errors in our PDFs Feel Free to contact us via this email: admin@exampundit.in
Ultra Practice Bundle PDF
SBI Clerk/ RBI Assist. Mains – Reasoning
of the person, who is 2 years older than F and faces either 14. Who sits opposite to U?
G or H. Only two persons sit between W and X. F is four (a) E
years older than U. Only one person is sitting between U (b) G
and Y who sits at the extreme end. E sits to the left of I. (c) H
The person who sits third to the left of I is of 52 years old. (d) I
The persons sitting to the left of F are younger than W. G (e) F
is six years older than Y. The age of W is between the age 15. Four of the following five are alike in a certain way
of Y and G. The sum of the ages of V and I is equal to the and hence form a group. Find out the one which
sum of the ages of H and Y. Sum of the ages of H and Y does not belong to that group.
is 110 years. The age of I is a multiple of 11. (a) U, 60 years old
11. What is the sum of the ages of G and I (in years)? (b) G, 48 years old
(a) 110 (c) V, 40 years old
(b) 100 (d) X, 36 years old
(c) 120 (e) F, 57 years old
(d) 90 Directions (16-20): Solve the questions based on the
(e) 104 following given information carefully
12. Which of the following statement is true? Six people G, H, I, J, K and L of six different countries V,
(a) F is older than V. W, X, Y, Z and M are sitting around a circular table facing
(b) E sits to the left of F. the centre. They all have different ages such as viz. 41, 44,
(c) X faces I. 48, 50, 52 and 54 (in years) but not necessarily in the same
(d) U and X are adjacent to each other. order. I sits second to the right of the person who is from
(e) None of these country Y. The one whose age is 54 years is not I. The one
13. Who sits to the immediate right of the person whose age is 48 years, sits second to the left of the person
whose age is 60 years? who is from country M who is not G and I. H sits opposite
(a) E to the person who is from country Z whose age is 44years.
(b) H There is only person between the person who is from
(c) G country M and Z but he is not from country V. Person
(d) I from country Z sits second to the right of the person from
(e) None of these country M. The person whose age is 50years sits second

Page 692 of 1334


Subscribe The Xpress Video Course & Mock Test Package for Bank & Insurance Exams
If there are any suggestions/ errors in our PDFs Feel Free to contact us via this email: admin@exampundit.in
Ultra Practice Bundle PDF
SBI Clerk/ RBI Assist. Mains – Reasoning
to the right of G. The person whose age is 52years does (b) J
not sit near to the one whose age is 44years. L sits to the (c) G
immediate left of the person who is from country X but (d) H
not near to G. The person from the country V doesn’t sit (e) K
near to the person from the country W but to the 20. Who is sitting opposite to K?
immediate right of J. (a) Person from country X
16. Who is sitting opposite to the person belongs to (b) The one whose age is 41years
country W? (c) G
(a) I (d) L
(b) J (e) -H
(c) H Directions (21-25): Solve the questions based on the
(d) Person from country V following given information carefully
(e) L Eight persons P, Q, R, S, T, U, V and W are sitting around
17. What is the position of the person whose age is 41 a rectangular table in such a way that four of them sit at
years with respect to the one whose age is 48 years? four corners, who are facing towards the center, and rest
(a) Second to the right four who sits at the middle of the side, are facing outside.
(b) Immediate right They like different fruits – Banana, Grapes, Guava, Apple,
(c) Immediate left Blueberry, Coconut, Litchi and Mango. All of them have
(d) Opposite different ages starting from 21 to 28 (in years).
(e) Second to the left Q sits at the corner and likes Guava fruit. Only two
18. Which of the following statement is true? persons sit between Q and the one whose age is greater
(a) L is the person from country M than 24years but odd number. V neither likes Grapes nor
(b) I is second to the right of H but not near to M Apple. S sits third to the right of V. S’s age is 22 years old.
(c) H’s age is 52years. S is not an immediate neighbor of Q. The one whose age
(d) Person from country X is opposite to the person is 23 years, likes Litchi fruit and faces centre but does not
from country Y sit to the immediate left of U. Only two persons sit
(e) I’s age is 50years. between the U, who likes Banana and T who faces inside
19. Who among the following is 41 years old? the center. The person who likes Apple fruit is 25 years
(a) I old. Q’s age is an odd number. R faces towards the center.

Page 693 of 1334


Subscribe The Xpress Video Course & Mock Test Package for Bank & Insurance Exams
If there are any suggestions/ errors in our PDFs Feel Free to contact us via this email: admin@exampundit.in
Ultra Practice Bundle PDF
SBI Clerk/ RBI Assist. Mains – Reasoning
U is not an immediate neighbor of S. P sits second to the (a) W
left of W who likes Coconut fruit. The one who likes (b) V
Blueberry fruit sits second to the right of the one who likes (c) U
Grapes fruit. The youngest person likes Mango fruit and (d) R
W is not the oldest person. S faces inside the center. (e) P
Neither R nor P likes Grapes fruit. T is 2 years older than 25. Who among the following sits second to left of the
U. one whose age is 21 years?
21. Who among the following is 24 years old? (a) 22 years old
(a) V (b) 24 years old
(b) S (c) 26 years old
(c) P (d) 21 years old
(d) R (e) 27 years old
(e) W Directions (26-30): Solve the questions based on the
22. What is the age of P? following given information carefully
(a) 25 years old Nine people Q, G, L, M, J, R, K, N and I are sitting in a
(b) 22 years old row. Some of them are facing in North while some are
(c) 27 years old facing in South direction. Persons face in opposite
(d) 26 years old direction means (If one faces south, the other faces north
(e) 28 years old and vice versa). Their age lies in between 35-65 years old.
23. Who among the following sits second to the left of None of them has same age.
S? R sits in the middle of the row, and one person sits
(a) W between N and R. Q sits second to the left of N, and faces
(b) Q in opposite direction with respect to both R and N. G and
(c) T L are immediate neighbors of each other and one of them
(d) R sits at any extreme end of row and both face same
(e) U direction with respect to Q. Number of persons sitting
24. Four of the following five are alike in a certain between L and R is same as in between I and R. One
way and hence they form a group. Which one person sits between I and the one whose age is a perfect
of the following does not belong to that group? square of a number which is more than 6. M and J are

Page 694 of 1334


Subscribe The Xpress Video Course & Mock Test Package for Bank & Insurance Exams
If there are any suggestions/ errors in our PDFs Feel Free to contact us via this email: admin@exampundit.in
Ultra Practice Bundle PDF
SBI Clerk/ RBI Assist. Mains – Reasoning
immediate neighbors of each other and difference between (a) None
both of their ages is 1 and the age of one of them is a (b) Two
perfect square of a number which is more than 5. Number (c) Three
of persons sitting between the one whose age is 37 years (d) Four
and one who is sitting to the immediate right of R, is one (e) One
more than number of persons sitting between G and M. 29. If L is related to J and N is related to Q in a certain
The age of immediate neighbors of R is a perfect square way, then following same pattern K is related to
number. M, who is 48 years old, sits third to the left of K. whom?
The person who is 55 years old sits sixth to right of L. Q’s (a) J
age is 10 years less than his neighbor and I faces in (b) I
opposite direction with respect to both M and N. R is 3 (c) G
years older than J. Difference between the ages of N and (d) Q
L is 5 years and L is older than N, who is 2 years younger (e) None of these
than I. Not more than 3 persons are facing in south 30. Which group represents the group of people facing
direction and G is the youngest among all nine members. in south direction?
26. How many persons are sitting to the right of M? (a) K, I, Q
(a) Four (b) G, L, M
(b) Three (c) M, R, N
(c) Two (d) J, K, Q
(d) One (e) None of these
(e) None of these Directions (31-35): Solve the questions based on the
27. Which group represents the people whose age is a following given information carefully
perfect square number? Seven persons Ramesh, Ravi, Rakesh, Roshan, Rahul,
(a) I, Q Ratan and Rishab are sitting in the straight line. Some of
(b) K, N them are facing towards the south while some are facing
(c) L, J towards the north. Each of them has different ages (in
(d) J, G years) viz., 72, 74, 75, 78, 80, 83 and 85 but not
(e) J, K necessarily in the same order.
28. How many persons are sitting between I and M?

Page 695 of 1334


Subscribe The Xpress Video Course & Mock Test Package for Bank & Insurance Exams
If there are any suggestions/ errors in our PDFs Feel Free to contact us via this email: admin@exampundit.in
Ultra Practice Bundle PDF
SBI Clerk/ RBI Assist. Mains – Reasoning
Ratan is facing south and his age is the multiple of 5. (c) One
Ramesh is facing in north and sits second to the right of (d) More than three
both Roshan and Rakesh. Rahul is facing south and he is (e) None of these
youngest. Difference between the age of Ramesh and 33. What is the difference between the age of the
Rishab is 4 and also both are immediate neighbors of each person who sits in the middle of the line and the one
other. Difference between the ages of Ravi and Rakesh is who sits immediate right of Ravi?
10. The person who is oldest among all, doesn’t sit at an (a) Three
extreme end of the line. Ravi is an immediate neighbor of (b) Two
Roshan but not an immediate neighbor of Ramesh. Rahul (c) Four
and Rakesh don’t sit at an extreme end of the line. The (d) Five
immediate neighbors of Roshan are facing in opposite (e) None of these
directions. The second eldest and youngest person is next 34. What is the total age of Rakesh, Roshan and the
to each other. The difference between the ages of Rakesh person who sits extreme left end of the row?
and the person who sits immediate right of Rakesh is at (a) Above 240
least 10.The sum of the ages of the person who sits at right (b) Above 190
end of the row and his neighbor is at least 160. Not more (c) Below 200
than two consecutive persons are facing the same (d) Below 240
direction. (e) None of these
31. Who among the following is second eldest among 35. What is the position of Roshan with respect to the
all? one whose age is 74 years?
(a) Ravi (a) Third to the right
(b) Rahul (b) Third to the left
(c) Roshan (c) Immediate right
(d) Ramesh (d) Immediate left
(e) None of these (e) Second to the right
32. How many persons are between Rishab and the one
who sits immediate right of Roshan?
(a) Two Directions (36-40): Solve the questions based on the
(b) Three following given information carefully

Page 696 of 1334


Subscribe The Xpress Video Course & Mock Test Package for Bank & Insurance Exams
If there are any suggestions/ errors in our PDFs Feel Free to contact us via this email: admin@exampundit.in
Ultra Practice Bundle PDF
SBI Clerk/ RBI Assist. Mains – Reasoning
Eight persons- H, N, S, I, D, A, R, and E are sitting around 37. What is the difference between the age of I and one
a circular table but not necessarily in the same order. Four who was born on 2000?
of them are facing inside and others are facing outside the (a) Four
circle. All the eight persons have different ages. They are (b) Five
born in the same month and same date of different years. (c) Eight
Their ages are calculated as base year 2020. (d) Seven
S’s age is Forty-nine years old. H is one year older than (e) None of these
the person who sits to the immediate left of E. N faces 38. What is the position of R with respect to the person
center of the table and sits third to the right of A. The whose age is 49?
person who sits second to the left of I is facing the same (a) Immediate right
direction as the person who was born in 1971. Immediate (b) Immediate Left
neighbors of E are seventeen and twenty years old. E is (c) Second to the left
not the youngest person. D is seventeen years old and (d) Second to the right
faces the person who was born in 1990. A was born in (e) None of these
1966. E sits to the immediate left of D. The person who is 39. Which of the following combinations are correct?
twenty-five years old is facing outwards and is an (a) I-1995
immediate neighbor of the person who is twenty years old. (b) R-25
R is an immediate neighbor of the person who is Forty- (c) S-1969
nine years old and the person who was born in 1995. Age (d) N-1990
of E is equal to the number obtained by reversing the digits (e) None of these
of H’s age. S does not sit second to the left of I. 40. Who is the third youngest person of the group?
36. How many persons are sit between R and the one (a) H
who is the oldest person in the group? (b) N
(a) Three (c) D
(b) Four (d) I
(c) Two (e) None of these
(d) One Directions (41-45): Solve the questions based on the
(e) None of these following given information carefully

Page 697 of 1334


Subscribe The Xpress Video Course & Mock Test Package for Bank & Insurance Exams
If there are any suggestions/ errors in our PDFs Feel Free to contact us via this email: admin@exampundit.in
Ultra Practice Bundle PDF
SBI Clerk/ RBI Assist. Mains – Reasoning
Twelve persons are sitting in a two parallel row, Row 1 42. How many people younger than P?
and Row 2. G, H, I, L, N and O are sitting in row 2 and (a) One
facing South direction. P, Q, R, U, V and W are sitting in (b) Two
row 1 and facing North direction. They have different ages (c) Three
viz, 65, 64, 63, 50, 49, 39, 36, 30, 28, 25, 20 and 15. The (d) More than three
person whose age is an even number are not sit adjacent (e) Y is the youngest person
to each other and the person whose age is an odd number 43. Who sits second to the left of the one who sits
are not sit adjacent to each other. opposite to U?
G is elder than U. V is 63 years old and sits third from the (a) The one who is the eldest person
extreme end of the row. L’s age is twice that of Q and sits (b) The one who is 49 years old
opposite to V. There are two person sit in between L and (c) The one who is youngest person
H, who is the youngest person among the group. Q sits (d) None of those given as option
opposite to the one whose age is 64 years. I’s age is (e) D
multiple of 5 and N sits to the immediate left of I. N is not 44. Four of the five among the following are similar in
64 years old. P is 20 years younger than L and sits opposite such a way to form a group, which one of the
to I. The age of G and U is a square number below 50. W following doesn’t belong to the group?
sits somewhere to the left of Q. G sits second to the left of (a) 15
the one who is opposite to W. The number of persons sits (b) 28
to the right of O is one less than that of the number of (c) 39
persons sits left of the one whose age is 36 years. The one (d) 50
whose age is 65 years sits to the immediate right of the (e) 20
one whose age is 20 years old. Q is not the eldest person. 45. What is the sum of ages of W, O and R?
R is elder than U. Q is younger than W. (a) 121
41. What will be the age of R? (b) 131
(a) 25 (c) 148
(b) 49 (d) 139
(c) 36 (e) None of those given as options
(d) 39 Directions (46-50): Solve the questions based on the
(e) None of those given as options following given information carefully

Page 698 of 1334


Subscribe The Xpress Video Course & Mock Test Package for Bank & Insurance Exams
If there are any suggestions/ errors in our PDFs Feel Free to contact us via this email: admin@exampundit.in
Ultra Practice Bundle PDF
SBI Clerk/ RBI Assist. Mains – Reasoning
Fourteen people are sitting in two parallel rows of eight K is older than M but younger than R, who is younger than
seats in each row and one seat is vacant in each row. L, E and is an immediate neighbor of H but not F. The one
M, N, O, P, Q and R are sitting in row 1 facing South E, who likes Jupiter faces the one who likes Activa. The
F, G, H, I, J and K are sitting in row 2 facing north. Each person who likes the Splender and Jawa are adjacent to
likes different two wheelers i.e Splendor, Royal Enfield, each other. Vacant seat in row 1 is not an immediate
Yamaha, Activa, Pulsar, KTM, Apache, Honda Shine, neighbor of O. K neither likes Platina nor Activa. F sits at
Suzuki, Platina, Jawa, Jupiter, Access-125 and Dio but not one of the extreme ends of the row 2. G does not like
necessarily in the same order. Each person has different Splendor and Jawa. F is not an immediate neighbor of J.
age – 2, 3, 4, 6, 7, 8, 9, 10, 11, 12, 13, 14, 15 and 16 but Vacant seat of row 1 does not face H who does not sit at
not necessarily in the same order. any of the extreme ends of the row. The person who likes
The difference between the age of M and N is 3. H sits Splendor is 3 years old. The age of P is the half of the age
fourth to the right of G and likes Activa. Only three people of I. The age of L, G and H is the square of the age of O,
sit between F and the vacant seat. Q is 10 years old and is P and L respectively. Neither O nor H is 4 years old.
an immediate neighbor of O and M. F does not like Pulsar 46. Who among the following is 2 years old?
or Platina. P is not an immediate neighbor of N. M likes (a) L
Access 125. The persons who sit at the extreme end of the (b) M
line have ages in consecutive order. Neither F nor I is 8 (c) N
years old. Both the immediate neighbors of vacant seat in (d) O
both rows have age in odd number. The one who likes (e) P
Yamaha faces the one who likes Honda Shine. R likes 47. Which of the following combination is correct?
Suzuki and sits third from the left end. The one who likes (a) Splendor – 3
Yamaha sits opposite to the one who sits fourth to the right (b) Access125 – 16
of the person who sits opposite to H. N is not an immediate (c) Activa – 7
neighbor of O. I, who likes neither Pulsar nor Platina nor (d) Pulsar – 6
KTM and does not face R. Neither H nor G sits at the (e) None of these
extreme ends of the row. O faces J. J sits third to the left 48. How many persons sit between O and N?
of E and is 14 years old. Vacant seats are not opposite to (a) None
each other. Three seats are there between N and M who (b) One
sits fourth right of the one who likes Apache. J likes Dio. (c) Two

Page 699 of 1334


Subscribe The Xpress Video Course & Mock Test Package for Bank & Insurance Exams
If there are any suggestions/ errors in our PDFs Feel Free to contact us via this email: admin@exampundit.in
Ultra Practice Bundle PDF
SBI Clerk/ RBI Assist. Mains – Reasoning
(d) Three 50. Four of the following are alike in a certain way and
(e) Four hence form a group. Which of the following does
49. Who among the following is 15 years old? not belong to that group?
(a) J (a) P
(b) E (b) N
(c) R (c) G
(d) M (d) I
(e) N (e) F

Seating with Ages – Answer and Explanation


SOLUTION (1-5):
Explanation in detail:
1. H, who is one year older than the person who sits to
the immediate left of L
2. Immediate neighbours of L are 18 and 21 years old.
3. Age of L is equal to the number obtained by reversing
the digits of H's age
From here we can find the possible age of L and H 6. Immediate neighbors of L are 18 and 21 years old.
7. The person who is twenty six years old is facing
outwards and is an immediate neighbor of the person
who is twenty one year old.(L’s age is 26 not possible
as shown in figure 1)
Figure 1
4. T is Eighteen years old and faces the person who is
thirty years old. (from here it is clear that T faces
inside then only T faces 30 year old)
5. L sits to the immediate left of T.

Page 700 of 1334


Subscribe The Xpress Video Course & Mock Test Package for Bank & Insurance Exams
If there are any suggestions/ errors in our PDFs Feel Free to contact us via this email: admin@exampundit.in
Ultra Practice Bundle PDF
SBI Clerk/ RBI Assist. Mains – Reasoning
8. N is an immediate neighbor of the persons who are
fifty and twenty six years old.
9. V is fifty years old.
.

14. As per instruction four of them faces inside and four


faces outside, then L and V face outside the circle and
satisfy the conditons.
15. Age of L is equal to the number obtained by reversing
the digits of H's age (from figure 1 we cannot take L’s
age is 22 years because then the age of H and L
become same which is not possible so we take age of
10. J is fifty five years old.
L is 91 years old and H’s age is 19 years)
11. B faces the centre and sits third to the right of J.
Final Arrangement

12. G sits third to the right of H.


13. The person who is sitting second to the left of G is
1.d
facing the same direction as the person who is fifty
2.b
years old. So, H must faces inside.
3.e
4.c
5.a
SOLUTION (6-10):

Page 701 of 1334


Subscribe The Xpress Video Course & Mock Test Package for Bank & Insurance Exams
If there are any suggestions/ errors in our PDFs Feel Free to contact us via this email: admin@exampundit.in
Ultra Practice Bundle PDF
SBI Clerk/ RBI Assist. Mains – Reasoning
Explanation in detail: 8. The age of D is not the 18 year.
1. A is sitting third to the right of C. 9. There is one person sitting between A and B.
2. A's age is 31 years. 10. D is sitting fourth to the right of B.
3. C's age is 17 years. CASE 1
From here, we have two cases (C faces inside or outside)
CASE 1

CASE 2

CASE 2

4. Three persons sit between the one whose age is 15 11. There are three persons sit between the one whose age

years and the one whose age is 31 years. is 13 years and the one whose age is 19 years.

5. The one whose age is 17 years sits third to the right of 12. The one whose age is 11 years sits third to the right of

the one whose age is 18 years. the one whose age is 19 years.

6. The one whose age is 18 years sits second to right of 13. There is only one person sits between the one whose

one whose age is 31 years. age is 19 years and the one whose age is 27 years.

7. The one whose age is 15 years sits second to the right 14. G sits fourth to the right of the one whose age is 13

of D. years.

Page 702 of 1334


Subscribe The Xpress Video Course & Mock Test Package for Bank & Insurance Exams
If there are any suggestions/ errors in our PDFs Feel Free to contact us via this email: admin@exampundit.in
Ultra Practice Bundle PDF
SBI Clerk/ RBI Assist. Mains – Reasoning
15. The one whose age is 15 years sits third to the right of CASE 1
the one whose age is 27 years.
CASE 1

Final Arrangement

CASE 2

6.c
16. The one whose age is 13 years sits third to left of one
7.d
whose age is 15 years and both faces the same
8.b
direction.
9.a
17. F is not immediate neighbour of G.
10.e
18. There is two persons sits between D and E.
SOLUTION (11-15):
19. E's age is neither 13 nor 19 years.
Explanation in detail:
20. E faces towards the Center of table.
1. The person who faces F sits third to the right of the
From here CASE 2 eliminated (because we can not place
one whose age is 36 years.
E as E faces inside)
2. Only one person sits between G and H.
Page 703 of 1334
Subscribe The Xpress Video Course & Mock Test Package for Bank & Insurance Exams
If there are any suggestions/ errors in our PDFs Feel Free to contact us via this email: admin@exampundit.in
Ultra Practice Bundle PDF
SBI Clerk/ RBI Assist. Mains – Reasoning
3. Neither G nor H faces the person whose age is 36
years.
4. E sits to the left of I.
Here, we have two Cases for the person who is 36
years old and F.
CASE 1 7. F is four years older than U.(F = U +4=> 52 - 4 = 48 )
F = 52 years old, U =48 years old
8. Either U or X is of 36 years old.(X= 36)
9. Only two persons sit between W and X.
10. Only one person is sitting between U and Y
CASE 1
CASE 2

11. G is six years older than Y. (G= Y+6=>54 +6 =>60)


5. Four persons sit to the left of the person who is 2 years
12. The age of W is between the ages of Y and G.
older than F and faces either G or H.
6. The person who sits third to the left of I is of 52 years
old.
From here CASE 2 eliminated
(By condition 6, age of F is 52 years and the person The persons sitting to the left of F are younger than W.

sits at the extreme right end of row 2 becomes 54 years ( It means H sits to the immediate left of F)

old and faces either G or H. Four person sit to the left CASE 1

of the person whose age is 54 years, faces G or H,


which is not possible in CASE 2 ).
CASE 1

Page 704 of 1334


Subscribe The Xpress Video Course & Mock Test Package for Bank & Insurance Exams
If there are any suggestions/ errors in our PDFs Feel Free to contact us via this email: admin@exampundit.in
Ultra Practice Bundle PDF
SBI Clerk/ RBI Assist. Mains – Reasoning

13. The difference between the ages of E and W is 17


years.

Age of E cannot be 74 years old because it is not a Final Arrangement

multiple of 4.
14. Sum of the ages of H and Y is 110 years.

11.e
12.c
13.a
14.b
15. The sum of the ages of V and I is equal to the sum of
15.d
the ages of H and Y. (V +I = 110)
SOLUTION (16-20):
16. The age of I is a multiple of 11.(here only possiblw
Explanation in detail:
value for I is 44 because it is aslo divisible by 4)
Person G, H, I, J, K and L
Countries V, W, X, Y, Z and M
1. The one whose age is 48 years is seated second to the
left of the person who is from country M.
2. Person from country Z sits second to the right of the
person from country M.
3. H is seated opposite to the person who is from country
Z whose age is 44years

Page 705 of 1334


Subscribe The Xpress Video Course & Mock Test Package for Bank & Insurance Exams
If there are any suggestions/ errors in our PDFs Feel Free to contact us via this email: admin@exampundit.in
Ultra Practice Bundle PDF
SBI Clerk/ RBI Assist. Mains – Reasoning
6. V sits to the immediate right of J.
7. There is only person between the person who is from
country M and Z but he is not from country V.
Here 1 more cases for CASE 1 and CASE 2
CASE 1a

4. I is seated second to the right of the person who is from


country Y.
5. G and I are not from country M.
There are two cases for Y and I
CASE 1

CASE1b

CASE 2 CASE 2a

Page 706 of 1334


Subscribe The Xpress Video Course & Mock Test Package for Bank & Insurance Exams
If there are any suggestions/ errors in our PDFs Feel Free to contact us via this email: admin@exampundit.in
Ultra Practice Bundle PDF
SBI Clerk/ RBI Assist. Mains – Reasoning
CASE 2b
11. The person whose age is 52 years is not seated near to
the one whose age is 44 years.
12. The one whose age is 54 years is not I.

CASE 1b

8. The person whose age is 50 years is seated second to


the right of G.
9. L sits to the immediate left of the person who is from
country X but not near to G.
10. The person from the country V is not seated near to
the person from the country W
Final Arrangement
From here CASE 1a eliminated (because W is always
nearer to V which is not possible)
CASE 2a also eliminated (because we can not place X
and L)
CASE 2b also eliminated (because we can not place X
and L also G is not placed)
CASE 1b

16.d
17.d
18.c
19.a
20.a
SOLUTION (21-25):

Page 707 of 1334


Subscribe The Xpress Video Course & Mock Test Package for Bank & Insurance Exams
If there are any suggestions/ errors in our PDFs Feel Free to contact us via this email: admin@exampundit.in
Ultra Practice Bundle PDF
SBI Clerk/ RBI Assist. Mains – Reasoning
Explanation in detail:
1. S sits third to the right of V.
2. S’s age is 22 years old.
3. S faces inside the center.
4. U is not an immediate neighbour of S
5. Only two persons sit between U who likes Banana and
T who faces inside the center.
From here it is clear that U sit at the middle of the side CASE 2a

6. Q sits at the corner and likes Guava Fruit.


7. Only two persons sit between Q and the one whose age
CASE 2b
is odd number greater than 24 years.(25/27)
8. The person who likes Apple fruit was 25 years old.
9. V neither likes Grapes nor Apple.
10. R faces inside the center
11. Q’s age is odd number.(Q = 21, 23, 25, 27)
From here we can find Q’s age i.e 27,
21 notpossible → Mango
23 notpossible→ Litchi 12. P sits second to the left of W who likes Coconut.
25 notpossible →Apple From here CASE 2a eliminated (we can not place W)
Now three cases for Q CASE 1
CASE 1

Page 708 of 1334


Subscribe The Xpress Video Course & Mock Test Package for Bank & Insurance Exams
If there are any suggestions/ errors in our PDFs Feel Free to contact us via this email: admin@exampundit.in
Ultra Practice Bundle PDF
SBI Clerk/ RBI Assist. Mains – Reasoning

CASE 2b

CASE 2b

13. The one whose age is 23 years likes Litchi fruit and
faces centre and not immediate left of U.
14. Neither R nor P likes Grapes fruit
15. The one who like Blackberry fruit sits second to the
right of the one who likes Grapes fruits.
16. The youngest person likes Mango fruit and W is not From here CASE 2b eliminated as 23 years person does

the oldest person. not sit to the left of U

CASE 1 17. T is 2 years older than U.


18. W is not the oldest person.(remaining ages 24, 26)
So from CASE 1
T=U+2
28 = 26 + 2
26 = 24 + 2 (Not possible because here W age is 28 which
is not possible as per given statement so W is 24)

Page 709 of 1334


Subscribe The Xpress Video Course & Mock Test Package for Bank & Insurance Exams
If there are any suggestions/ errors in our PDFs Feel Free to contact us via this email: admin@exampundit.in
Ultra Practice Bundle PDF
SBI Clerk/ RBI Assist. Mains – Reasoning

CASE 2

3. G and L both are immediate neighbours of each other


Final Arrangement and one of them sits at any extreme end of row and
both face same direction with respect to Q.
4. Number of persons sitting between L and R & I and R
is same.(So G sits at the extreme end)
5. The person who is 55 years old sits sixth to right of L
6. I face opposite direction with respect to both M and N
CASE 1

21. e
22. a
CASE 2
23. b
24. d
25. c
SOLUTION (26-30):
7. Not more than 3 persons are facing south direction
Explanation in detail:
Here CASE 2 eliminated.
1. R sits in the middle of the row, and one person sits
8. M and J are immediate neighbours of each other.
between N and R.
9. M, who is 48 years old sits third to left of K.
2. Q sits second to left of N, and faces opposite direction
CASE 1
with respect to both R and N.
There are two cases for N here
CASE 1

Page 710 of 1334


Subscribe The Xpress Video Course & Mock Test Package for Bank & Insurance Exams
If there are any suggestions/ errors in our PDFs Feel Free to contact us via this email: admin@exampundit.in
Ultra Practice Bundle PDF
SBI Clerk/ RBI Assist. Mains – Reasoning
10. difference between M and J’s ages is one year and the
age of one of them is a perfect square of a number
which is more than 5. (Age of J = 48 +1 = 49 years old
which perfect square of 7.)
11. R’s age is 3 years more than J Final Arrangement

12. The age of both of the neighbours of R is a perfect


square number
13. One person sits between I and the one whose age is a
perfect square of a number which is more than 6.
26.c
14. (By following the above conditions, age of K become
27.e
64 which is perfect square of 8.)
28.d
CASE 1
29.b
30.c
SOLUTION (31-35):
Explanation in detail:
15. Difference between the ages of N and L is 5 years and
1. Ramesh is facing north and sits second to the right of
L is older than N, who is 2 years younger than I.
both Roshan and Rakesh
Age of N = 55 – 2 = 53 years old.
2. Ravi is an immediate neighbour of Roshan but not an
Age of L = 53 + 5 = 58 years old
immediate neighbour of Ramesh.
16. Number of persons sitting between the one whose age
3. Rahul and Rakesh doesn’t sit at an extreme end of the
is 37 and one who is sitting immediate right of R is
line.
one more than number of persons sitting between G
There are two Cases for Roshan and Ramesh
and M.
CASE 1
17. Q’s age is 10 years less than his neighbour.(Q’s age is
45years)
18. G is youngest among all nine members.
Youngest age mention in the question is 37 years old, CASE 2
so the age of G becomes 37 years old.
CASE 1

Page 711 of 1334


Subscribe The Xpress Video Course & Mock Test Package for Bank & Insurance Exams
If there are any suggestions/ errors in our PDFs Feel Free to contact us via this email: admin@exampundit.in
Ultra Practice Bundle PDF
SBI Clerk/ RBI Assist. Mains – Reasoning
4. Ratan is facing south and his age is multiple of 5. 8. Not more than two consecutive persons are facing the
5. Difference between the ages of Ravi and Rakesh is 10. same direction
6. The person who is oldest doesn’t sit at an extreme end 9. The immediate neighbours of Roshan are facing
of the line. opposite directions.
(As per above condition, only three ages i.e. 75,80 and From here we can find direction of all the person here
85 are multiples of 5 but 75 and 85 is the only pair
which has a difference of 10 that is useful for the ages
of Ravi and Rakesh. Hence, the age of Ratan becomes
80 years old.) 10. Difference between the age of Ramesh and Rishab is
Ratan sits at an extreme end of the line because Rahul 4
does not sit at an end and Rishab is an immediate
neighbor of Ramesh, so we have only one place
remaining for Ratan.
11. The second eldest and youngest person are next to
each other.(means 83 and 72 immediate neighbor)
12. The difference between the ages of Rakesh and the
person who sits immediate right of Rakesh is at least
CASE 1 10 (From here clear that Ramesh age is 78 and Rishab
age is 74)
13. Rahul is facing in south and he is youngest among all.

CASE 2

Final Arrangement

7. The sum of the ages of the person who sits at right end
of the row and his neighbour is at least 160
From here CASE 2 eliminated because in right end 75 is
31.c
there and we can not make sum 160 using 75
32.c

Page 712 of 1334


Subscribe The Xpress Video Course & Mock Test Package for Bank & Insurance Exams
If there are any suggestions/ errors in our PDFs Feel Free to contact us via this email: admin@exampundit.in
Ultra Practice Bundle PDF
SBI Clerk/ RBI Assist. Mains – Reasoning
33.d
34.a
35.b
SOLUTION (36-40):
Explanation in detail:
1. Immediate neighbours of E are seventeen and twenty
years old.
2. E is not the youngest person. 5. E sits to the immediate left of D

3. H is one year older than the person who sits to the 6. D is seventeen years old and faces the person who was

immediate left of E. born in 1990.(2020-1990 = 30)

4. Age of E is equal to the number obtained by reversing 7. The person who is twenty-five years old is facing

the digits of H’s age. outwards and is an immediate neighbor of the person

From here we can find H’s age. who is twenty years old.

The person who is immediate left E is either 17 or 20. From here we can find the direction of D which is

H’s age is 18 or 21 years and it is given that E’s age is towards the center because D faces 30 years old

reverse of H’s age so E’s age is either 12 or 81. E is not person.

youngest so 12 is not possible Hence E’s age is 81 and Case 2 was eliminated because D’s face is outside to

H’s is 18. make sit E left of D.

There are two Cases for E-


CASE 1

8. R is an immediate neighbor of the persons who is


forty-nine years old and the person who was born in
CASE 2
1995 (25 years)

Page 713 of 1334


Subscribe The Xpress Video Course & Mock Test Package for Bank & Insurance Exams
If there are any suggestions/ errors in our PDFs Feel Free to contact us via this email: admin@exampundit.in
Ultra Practice Bundle PDF
SBI Clerk/ RBI Assist. Mains – Reasoning
9. S’s age is Forty-nine years old.

15. In given instructions, 4 persons face inside and 4


10. N faces center of the table and sits third to the right of persons face outside the circle.
A Final Arrangement
11. A was born in 1966 (2020-1966 = 54 years old)

12. We have found earlier that H’s age is 18 so remaining


place allot to I and his age becomes 25 years.
13. S does not sit second to the left of I
14. The person who sits second to the left of I is facing the
same direction as the person who was born in
1971(49years). By this, S faces outside the circle.

Page 714 of 1334


Subscribe The Xpress Video Course & Mock Test Package for Bank & Insurance Exams
If there are any suggestions/ errors in our PDFs Feel Free to contact us via this email: admin@exampundit.in
Ultra Practice Bundle PDF
SBI Clerk/ RBI Assist. Mains – Reasoning

CASE 2
36.c
37.b
38.b
39.a
40.b
SOLUTION (41-45):
Explanation in detail:
1. V is 63 years old and sits third from the extreme end 4. N sits to the immediate left of I.
of the row. 5. P is 20 years younger than L and sits opposite to I.(P
2. There are two persons sit between L and H who is the = L – 20➔ 50- 20 = 30) P =30 years old.
youngest person among the group.(H = 15 years old) 6. G sits second to the left of the one who is opposite to
3. L’s age is twice that of Q and sits opposite to V. W.
(Only the pair 25 and 50 is there which satisfies the Form here one more case of CASE 2
condition of twice the another number. It means if age
of Q is 25 years old then only by the given set of ages, CASE 1 (I and N are not possible between L and H
age of L becomes 50 years old.) because we can not place G)

There are two cases for V here


CASE 1

Page 715 of 1334


Subscribe The Xpress Video Course & Mock Test Package for Bank & Insurance Exams
If there are any suggestions/ errors in our PDFs Feel Free to contact us via this email: admin@exampundit.in
Ultra Practice Bundle PDF
SBI Clerk/ RBI Assist. Mains – Reasoning
L’s age is twice that of Q.(L = 50, Q = 25)
9. Q sits opposite to the one whose age is 64 years.
10. N is not 64 years old.
CASE 1

CASE 2a

CASE 2a

CASE 2b

CASE 2b

7. G is elder than U
8. The age of G and U is a square number below 50.
Here for G and U two possibilities i.e 49 and 36.
G is elder than U so G = 49 years old, U =36 yaers old.

Page 716 of 1334


Subscribe The Xpress Video Course & Mock Test Package for Bank & Insurance Exams
If there are any suggestions/ errors in our PDFs Feel Free to contact us via this email: admin@exampundit.in
Ultra Practice Bundle PDF
SBI Clerk/ RBI Assist. Mains – Reasoning
11. The number of persons sits to the right of O is one less 14. R is elder than U.
than that of the number of persons sits left of the one 15. Q is younger than W.
whose age is 36 years. CASE 2b
12. The one whose age is 65 years sits to the immediate
right of the one whose age is 20 years old.
13. I’s age is a multiple of 5. (Age of I is 65 years old.)
From here CASE 2a eliminated because it does not
satisfy the 11th condition.
CASE 1
Final Arrangement

41. d
CASE 2b 42.d
43.a
44.d
45.b
SOLUTION (46-50):
Explanation in detail:
1. R likes Suzuki and sits third from the left end.
2. H sits fourth to the right of G and likes Activa.

From here Case 1 is eliminated because it is given that: 3. Neither H nor G sits at the extreme ends of the row.

The persons whose age are in even numbers are not sit 4. The one who likes Yamaha sits opposite to the one

adjacent to each other and the person whose age is the odd who sits fourth right of the person who sits opposite to

numbers are not sit adjacent to each other. H.


Page 717 of 1334
Subscribe The Xpress Video Course & Mock Test Package for Bank & Insurance Exams
If there are any suggestions/ errors in our PDFs Feel Free to contact us via this email: admin@exampundit.in
Ultra Practice Bundle PDF
SBI Clerk/ RBI Assist. Mains – Reasoning
5. The one who likes Yamaha faces the one who likes 16. K and H are immediate neighbor.
Honda Shine.
6. The one who likes Jupitar faces the one who likes
Activa
From here it is clear that only one place for G and H.

17. Three seats are there between N and M who sits fourth
right of the one who likes Apache.
18. Vacant seat present in row 1 is not an immediate
7. J sits third to the left of E and 14 years old neighbor of person O.
8. O faces J 19. Vacant seats are not opposite to each other.
9. Q is 10 years old and is an immediate neighbor of O 20. P is not an immediate neighbor of N.
and M. 21. N is not an immediate neighbor of O.
10. M likes Access 125.
11. J likes Dio

22. The age of L, G and H is the square of the age of O, P


and L respectively.
12. F sits at one of the extreme ends of the row 2. 23. Neither O nor H is 4 years old.
13. F is not an immediate neighbor of J. (From here it is 24. The person who likes Splendor is 3 years old.
clear that F sits at right end) 25. The age of P is the half the age of I.
14. Only three people sit between F and the vacant seat. 26. The person who likes the Splendor and Jawa are
15. I, who likes neither Pulsar nor Platina nor KTM and adjacent to each other.
do not, face R.

Page 718 of 1334


Subscribe The Xpress Video Course & Mock Test Package for Bank & Insurance Exams
If there are any suggestions/ errors in our PDFs Feel Free to contact us via this email: admin@exampundit.in
Ultra Practice Bundle PDF
SBI Clerk/ RBI Assist. Mains – Reasoning

Here 4, 9, 16 is square of 2, 3, 4 So we can take


O = 2, P = 3
We cannot take any other value for O because square of
O is L and Square of L is H
31. The difference between the age of M and N is 3.
We get

32. Both the neighbors of vacant seat in both rows have


P =1/2 I ➔ I = 6 and P = 3 age in odd number.(means E, R and M have odd
number age)
33. K is older than M but younger than R who is younger
than E.
E>R>K>M

27. K neither likes Platina nor Activa.


28. F does not like Pulsar or Platina. Means F like KTM
29. The persons who sits at the extreme end of the line
have ages in consecutive order
30. I neither likes Pulsar nor Platina nor KTM. Means I
like Royal Enfield. Final Arrangement

Page 719 of 1334


Subscribe The Xpress Video Course & Mock Test Package for Bank & Insurance Exams
If there are any suggestions/ errors in our PDFs Feel Free to contact us via this email: admin@exampundit.in
Ultra Practice Bundle PDF
SBI Clerk/ RBI Assist. Mains – Reasoning
46.d
47.a
48.e
49.b
50.c

Seating Linear with Blood Relation


Directions (1-5): Answer the questions based on the in-law. Z is married to U. Y is the son-in-law of Z. Y’s
information given below: (4 statements added) only brother-in-law is married to T. T’s sister-in-law’s
A family of eight members - S, T, U, V, W, X, Y and Z father-in-law is X. V’s father is not Y.
are seated in a straight line with an equal distance between 1. What is the position of V with respect to her
them, but not necessarily in the same order. Some of them grandmother?
are facing north and some are facing south. This family is (a) Third to right
having three generations. The immediate neighbors of X (b) Second to left
face same directions. Both W and T face a direction (c) Third to right
opposite to that of V. W is the only daughter of U. Z is (d) Four to the left
female member. U’s granddaughter does not sit at an (e) Cannot be determined
extreme end of the line. S’s mother sits on the immediate 2. What is the position of S with respect to his
left of her son. U’s wife is not an immediate neighbor of sister?
W. The immediate neighbors of W’s brother face opposite (a) Fifth to right
directions. The persons sitting at the extreme ends face (b) Immediate left
opposite directions. Z’s daughter-in-law sits second to the (c) Third to right
left of W’s father-in-law. X faces north. X is not an (d) Second to left
immediate neighbor of V’s aunt. U’s daughter is an (e) Immediate right
immediate neighbor of one who is sitting at an extreme 3. How is V related to W?
end of the line. Only three people sit between Y and his (a) sister
wife. T’s daughter sits second to the right of S’s brother- (b) Niece

Page 720 of 1334


Subscribe The Xpress Video Course & Mock Test Package for Bank & Insurance Exams
If there are any suggestions/ errors in our PDFs Feel Free to contact us via this email: admin@exampundit.in
Ultra Practice Bundle PDF
SBI Clerk/ RBI Assist. Mains – Reasoning
(c) Daughter extreme end of the line. Daughter of A is sitting 2nd to the
(d) Sister-in-law right of the mother of A. I is sitting facing south direction
(e) Cannot be determined which is opposite of what A’s daughter is facing. The
4. Who is the husband of T? immediate neighbors of A’s daughter are facing same
(a) brother of Y direction as I. From nine members, one is son of A. There
(b) Father of V is one person sitting between A and G and they both are
(c) Grandfather of X facing opposite directions. H faces north and only two
(d) Son of W persons are sitting to the right of H, who is A’s sister. D is
(e) Father of Z sitting between H and B.G faces north. H and C are not
5. How is Z related to V? neighbors.
(a) sister 6. What is the position of D with respect to F?
(b) Aunt (a) Third to right
(c) Mother (b) Second to left
(d) Mother-in-law (c) Third to right
(e) Grandmother (d) Fourth to right
Directions (6-10): Answer the questions based on the (e) Cannot be determined
information given below: (4 statements added) 7. What is the position of C with respect to her
There are nine members in a family – A, B, C, D, E, F, G, brother?
H, I. They all are sitting in a straight line from left to right. (a) Second to right
All of them have a relationship with A. Some of them are (b) Immediate right
facing north and others are facing south. The following (c) Third to right
information is known about them: (d) Second to left
E who is the father of A is sitting to the immediate of G’s (e) Immediate left
wife. A’s wife is sitting immediately left of B who is 8. How is I related to A?
sitting at 5th position from left end of row (when seen in (a) sister-in-law
north direction). C is sitting fourth to the right of A’s (b) Niece
father. A is sitting 3rd to the right of D, who faces south. (c) Daughter
Mother and brother of A are facing the same direction and (d) Sister
both of them are sitting together with one of them at an (e) Cannot be determined

Page 721 of 1334


Subscribe The Xpress Video Course & Mock Test Package for Bank & Insurance Exams
If there are any suggestions/ errors in our PDFs Feel Free to contact us via this email: admin@exampundit.in
Ultra Practice Bundle PDF
SBI Clerk/ RBI Assist. Mains – Reasoning
9. Who is one of the brothers of H? (a) M
(a) Brother of E (b) P
(b) Father of G (c) K
(c) Father of B (d) J
(d) Son of I (e) None of these
(e) Cannot be determined 12. Who sits second to the left of grandson of L?
10. How is F related to A? (a) O
(a) sister (b) L
(b) Aunt (c) Q
(c) Mother (d) M
(d) Mother-in-law (e) None of these
(e) Cannot be determined 13. Who is sitting third to the left of N?
Directions (11-15): Answer the questions based on the (a) J
information given below: (2 statements added) (b) Q
There are eight members J, K, L, M, N, O, P and Q. Each (c) L
of them is related to J in some way or the other and all are (d) P
sitting in a straight line facing north, but necessarily in the (e) None of these
same order. 14. Who is sitting second to the right of daughter of
N is sitting second to right of J’s son. Two people are M?
sitting between N and K’s brother. Two people are sitting (a) O
between J’s son and J’s father. J’s brother is sitting second (b) J
to left of J. Two people are sitting between M and J’s (c) P
sister. J is not an immediate neighbor of N. Q’s father is (d) L
sitting to the immediate right of J’s daughter. J’s wife is (e) None of these
immediate neighbor of Q. P is younger than L.O is an 15. What is relation between K with respect to O?
immediate neighbor of J’s father. O is sitting at one of (a) Son
extreme ends. M and J’s brother are sitting together. O is (b) Uncle
married to L. (c) Daughter
11. Who is to the immediate left of daughter of L? (d) Father

Page 722 of 1334


Subscribe The Xpress Video Course & Mock Test Package for Bank & Insurance Exams
If there are any suggestions/ errors in our PDFs Feel Free to contact us via this email: admin@exampundit.in
Ultra Practice Bundle PDF
SBI Clerk/ RBI Assist. Mains – Reasoning
(e) None of these (d) Fourth to right
Directions (16-20): Answer the questions based on the (e) Cannot be determined
information given below: (some edited & 3 statements 27. What is the position of A with respect to father of
added) E?
Eight people of a family A, B, C, D, E, F, G and H are (a) Second to right
sitting in a row some facing north direction and some (b) Immediate left
facing south direction but not necessarily in the same (c) Third to right
order. (d) Second to left
H is a grandmother of G and C is a son in law of B who (e) Immediate right
has two daughters. There is only one married couple in a 28. How is E related to A?
family of which the wife is sitting third to the left of her (a) sister
husband. E’s maternal grandfather is sitting second from (b) Niece
one of the ends. E, who is a female and E’s maternal (c) Daughter
grandfather have at least one child, but both of them does (d) Sister-in-law
not have spouse. A has only one brother, who is a (e) Cannot be determined
grandfather of F. C is not an immediate neighbor of G. 29. Who is the husband of G?
Both the immediate neighbors of E face south. Immediate (a) A
neighbors of G face opposite direction. A and D face the (b) F
same direction as E. D is sister in law of C. Only three (c) D
people sit between A and G. E sits exactly between A and (d) C
G. Both the sister in laws are sitting at the ends. H sits (e) Cannot be determined
third to the right of E. B is an immediate neighbor of H 30. How is G related to F?
and faces south. A sits at one of the extreme ends of the (a) sister
line. C sits second to the right of F. Youngest person is a (b) Aunt
male. (c) Grandmother
26. What is the position of H with respect to E? (d) Mother-in-law
(a) Third to right (e) Cannot be determined
(b) Second to left Directions (21-25): Answer the questions based on the
(c) Third to left information given below: ((1 statement added)

Page 723 of 1334


Subscribe The Xpress Video Course & Mock Test Package for Bank & Insurance Exams
If there are any suggestions/ errors in our PDFs Feel Free to contact us via this email: admin@exampundit.in
Ultra Practice Bundle PDF
SBI Clerk/ RBI Assist. Mains – Reasoning
Five person of different age of two generations live in a 23. Who is sitting third to the left of the one, whose is
family. They have different designations i.e. CEO, COO, 29 years?
GM, AM, BM and they are sitting in a line facing north (a) A
direction in the above order of designations from left to (b) D
right. (c) E
Two persons sit between the persons, whose ages are 40 (d) B
and 44 years. Difference of the age of E and D is 4 years (e) None of these
less than the age of the person who is CEO. D is unmarried 24. Who is sitting second to the right of daughter in
and is sister in law of C. A is the father in law of the person law of E?
who is CEO. Husband of E is COO, and he does not have (a) C
even numbered age. Age of the person who sits immediate (b) D
right of the person whose age is 40 is 71 years. Average (c) A
age of A, E and D is 55 years. There are as many persons (d) B
sitting to the left of A as to the right of the only son of E. (e) None of these
Person who is BM is the sibling of B and both sit next to 25. What is relation of E with respect to the one whose
each other. CEO is younger than AM. age is 71 years?
21. Who is to the immediate left of the one who is (a) Son
COO? (b) Wife
(a) D (c) Daughter
(b) E (d) Father
(c) A (e) None of these
(d) B Directions (26-30): Answer the questions based on the
(e) C information given below:
22. Who sits second to the left of son of A? There are nine members A, B, C, D, E, F, G, H and I in a
(a) C family of four generation having three married couples.
(b) E They all sit in a row facing north direction but not
(c) A necessarily in the same order. E is only married child of
(d) D A. E sits second to the right of C. A’s brother in law sit at
(e) None of these the extreme right end. G has two sons. G is an immediate

Page 724 of 1334


Subscribe The Xpress Video Course & Mock Test Package for Bank & Insurance Exams
If there are any suggestions/ errors in our PDFs Feel Free to contact us via this email: admin@exampundit.in
Ultra Practice Bundle PDF
SBI Clerk/ RBI Assist. Mains – Reasoning
neighbor of B. C and H both are siblings. Only one person (b) E
sit between F and E’s child who is an immediate neighbor (c) F
of H. H and I is unmarried. G is mother in law of A. Sister (d) C
in law of D sits fourth from the extreme end. E’s father (e) B
sits to the immediate right of I’s aunt. Daughter in law of 30. How is G related to A?
A sits second to the left of G who is married to F. B is (a) sister
daughter of A. (b) Daughter-in-law
26. What is the position of B with respect to D? (c) Grandmother
(a) Third to left (d) Mother-in-law
(b) Second to left (e) Cannot be determined
(c) Third to right Directions (31-35): Answer the questions based on the
(d) Fourth to right information given below:
(e) Cannot be determined Eight persons of a family J, K, L, M, N, O, P and Q sit in
27. What is the position of F with respect to father a linear row and all face in north direction but not
of I? necessarily in the same order. There are three married
(a) Second to right couples in the family. P is grandchild of N and sits to the
(b) Third to right immediate right of O’s Mother. J has two children. O is
(c) Immediate left not sister in law of L. M is younger than K who is spouse
(d) Second to left of J. J is not a female member of the family. J and O’s
(e) Immediate right mother in law does not sit next to each other. K is not
28. How is I related to C? mother in law of O. Two person sit between K and P’s
(a) sister father who sits at one of the extreme ends. O is mother of
(b) Niece P and sits third to the left of her own sister. O’s mother in
(c) Daughter law is the only neighbor of M’s father. L is aunt of P. Q is
(d) Sister-in-law grandfather of P but not father of O. K sits exactly in
(e) Cannot be determined between O’s father and L.
29. Who is the husband of one who sits immediate 31. Who sits to the immediate left of husband of K?
left of A? (a) M
(a) G (b) Q

Page 725 of 1334


Subscribe The Xpress Video Course & Mock Test Package for Bank & Insurance Exams
If there are any suggestions/ errors in our PDFs Feel Free to contact us via this email: admin@exampundit.in
Ultra Practice Bundle PDF
SBI Clerk/ RBI Assist. Mains – Reasoning
(c) O Directions (36-40): Answer the questions based on the
(d) N information given below: (fully edited & 3 statements
(e) L added)
32. Who among the following son of Q? Eight members A, B, C, D, E, F, G and H are sitting in a
(a) N row facing north direction but not necessarily in the same
(b) J order. They buy different items Mug, Bag, Bottle, Desk,
(c) O Clock, Brush, Pen and cup but not necessarily in the same
(d) L order.
(e) M There is only one person sit between C and A, who is
33. Who sits third to the left of N? granddaughter of C. E’s wife sit on the odd place value
(a) M from right to left but not extreme right end. A sits one of
(b) J the three seats from right end. G is mother of B, who buys
(c) Q Mug and B sits exactly between E’s wife and C. Three
(d) L people sit between E’s wife and C, who buy Bottle. The
(e) K number of person sits to the right of C is same as left to
34. Who sits immediate left to daughter in law of N? the E, who buys clock. D is father of A. The one who buy
(a) M cup sits immediate right to the one who buy desk. E’s wife
(b) J does not buy cup. D sits immediate left to the one who
(c) K buys brush. B is brother of H, who married C. F, who sits
(d) Q immediate right to her father. G’s daughter buys a bag. B
(e) P is unmarried. G has only two children. F and the one, who
35. How many persons sit between daughter in law of buys Pen, are not neighbors. The one, who buys pen and
Q and spouse of N? the one, who buy desk are couples.
(a) One 36. Who sits to the immediate left of one who buys
(b) Four clock?
(c) Six (a) G
(d) Two (b) D
(e) Three (c) B
(d) H

Page 726 of 1334


Subscribe The Xpress Video Course & Mock Test Package for Bank & Insurance Exams
If there are any suggestions/ errors in our PDFs Feel Free to contact us via this email: admin@exampundit.in
Ultra Practice Bundle PDF
SBI Clerk/ RBI Assist. Mains – Reasoning
(e) F There are seven members i.e. A, B, C, D, E, F and G in a
37. How many persons sit between son in law of G and family and only three male members. All are sitting in a
wife of E? row some are facing north and some facing south
(a) One direction. Each one of them likes different subjects i.e.
(b) Six maths, reasoning, English, history, geography, politics
(c) Three and economics but not necessarily in same order.
(d) Two C sits at extreme end. F who is sister of B sits second to
(e) Four the right of A’s sister. E’s mother sits second to the left of
38. Who is daughter of C? F. D is son in law of C and father of E. E does not sit left
(a) B of F. The one who likes English sits second to the right of
(b) F the one who sits second to the right of the one who likes
(c) H politics. E is grandson of A. C likes maths. A, who likes
(d) A reasoning, is father of B. The one who likes economics
(e) G sits second to the right of the one who likes history.
39. Who is second to the right of son of G? Mother of B sits second to the left of her. D is not an
(a) A immediate neighbor of C and faces the same direction as
(b) F F faces. Not more than three persons face south direction.
(c) D A, who faces opposite direction of C, sits to the right of E.
(d) C F is not married to D. C and E face opposite direction with
(e) E respect to each other.
40. How many persons sit between son and husband of 41. Who is sitting third to the right of C?
G? (a) C
(a) One (b) A
(b) Four (c) H
(c) Six (d) D
(d) Two (e) None of these
(e) None 42. What is the position of A with respect to wife of A?
Directions (41-45): Answer the questions based on the (a) Immediate right
information given below: (b) Immediate left

Page 727 of 1334


Subscribe The Xpress Video Course & Mock Test Package for Bank & Insurance Exams
If there are any suggestions/ errors in our PDFs Feel Free to contact us via this email: admin@exampundit.in
Ultra Practice Bundle PDF
SBI Clerk/ RBI Assist. Mains – Reasoning
(c) Third to the right There are six members in a family of two generations U,
(d) Third to the left V, W, X, Y and Z. all of them are sitting in a row facing
(e) None of these south direction but not necessarily in the same order.
43. What is B’s position with respect to one who likes Y is sitting to the immediate right of X. Z is sitting at one
English? of the extreme ends. V’s father in law is sitting to the
a. Third to the right immediate right of V. X is a male. Two persons sit
b. Second to the right between both the daughters of U. Z is not daughter of U.
c. Fourth to the right V’s sister in law sits only with V. X’s sister is sitting at
d. Second to the left the extreme right end. U is mother of only two daughters
e. Third to the left and is sitting second to the right of her husband. V is
44. Who is sitting third to the right of one who likes married male.
maths? 46. Who is to the immediate left of Z?
(a) A (a) Y
(b) G (b) U
(c) F (c) X
(d) D (d) W
(e) None of these (e) None of these
45. Which one of the following statements is true 47. Who is son-in-law of X?
according to the above mentioned arrangement? (a) U
a. G is to the immediate left of wife of A. (b) Z
b. D sits immediate right to one of the daughters (c) W
of C. (d) V
c. F sits second to right of her niece. (e) None of these
d. E sits fourth to the right of A. 48. Who is sitting third to the left of wife of V?
e. None of these (a) W
Directions (46-50): Answer the questions based on the (b) X
information given below: (c) Z
(d) Y
(e) None of these

Page 728 of 1334


Subscribe The Xpress Video Course & Mock Test Package for Bank & Insurance Exams
If there are any suggestions/ errors in our PDFs Feel Free to contact us via this email: admin@exampundit.in
Ultra Practice Bundle PDF
SBI Clerk/ RBI Assist. Mains – Reasoning
49. Who is daughter of U? 50. Who is sitting fourth to the right of brother in law
(a) Y and W of W?
(b) X and Z (a) U
(c) Y and V (b) Y
(d) V and Z (c) X
(e) None of these (d) V
(e) Z

Seating Linear with Blood Relation – Answer and Explanation


SOLUTION (1-5):
Explanation in detail: (Completely modified)
1. W is the only daughter of U.
2. U’s daughter is an immediate neighbor of one who
is sitting at an extreme end of the line.

6. Y’s only brother-in-law is married to T.


7. T’s sister-in-law’s father-in-law is X

8. V’s father is not Y.


9. This family is having three generations.

3. Z is married to U. 10. Only three people sit between Y and his wife.
4. Y is the son-in-law of Z. CASE1:
5. Z is female member

Page 729 of 1334


Subscribe The Xpress Video Course & Mock Test Package for Bank & Insurance Exams
If there are any suggestions/ errors in our PDFs Feel Free to contact us via this email: admin@exampundit.in
Ultra Practice Bundle PDF
SBI Clerk/ RBI Assist. Mains – Reasoning
18. The immediate neighbors of W’s brother face
opposite directions.
19. X is not an immediate neighbor of V’s aunt.
20. U’s wife is not an immediate neighbor of W.

CASE2:

11. T’s daughter sits second to the right of S’s brother-


in-law.
12. U’s granddaughter does not sit at an extreme end
of the line.

So CASE 2 is cancelled out (Z’s daughter-in-law sits


second to the left of W’s father-in-law, not satisfied)
21. Both W and T face a direction opposite to that of
V.

13. The persons sitting at the extreme ends face


opposite directions.
Blood relation:
14. Z’s daughter-in-law sits second to the left of W’s
father-in-law.
15. X faces north.
16. The immediate neighbors of X face same
directions.
17. S’s mother sits on the immediate left of her son.
Page 730 of 1334
Subscribe The Xpress Video Course & Mock Test Package for Bank & Insurance Exams
If there are any suggestions/ errors in our PDFs Feel Free to contact us via this email: admin@exampundit.in
Ultra Practice Bundle PDF
SBI Clerk/ RBI Assist. Mains – Reasoning

4. D is sitting between H and B.


Final arrangement: 5. A is sitting 3rd to the right of D, who faces south
6. There is one person sitting between A and G and
they both are facing opposite directions.
7. G faces north.
1.d
2.a
3.b
4.b
8. C is sitting fourth to the right of A’s father.
5.e
9. Mother and brother of A are facing the same
SOLUTION (6-10):
direction and both of them are sitting together with
Explanation in detail: (completely modified)
one of them at an extreme end of the line.
1. A’s wife is sitting immediately left of B who is
10. H and C are not neighbors.
sitting at fifth position from left end of row (when
11. E who is the father of A is sitting to the immediate
seen in north direction)
of G’s wife
2. H faces north and only two persons are sitting to
the right of H, who is A’s sister.
3. E who is the father of A is sitting to the immediate
of G’s wife

Daughter of A is sitting 2nd to the right of the


mother of A.

Page 731 of 1334


Subscribe The Xpress Video Course & Mock Test Package for Bank & Insurance Exams
If there are any suggestions/ errors in our PDFs Feel Free to contact us via this email: admin@exampundit.in
Ultra Practice Bundle PDF
SBI Clerk/ RBI Assist. Mains – Reasoning
12. I is sitting facing south direction which is opposite 7.b
of what A’s daughter is facing. 8.a
13. The immediate neighbors of A’s daughter are 9.c
facing same direction as I. 10.c
14. From nine members, one is son of A. SOLUTION (11-15):
15. A’s wife is sitting immediately left of B who is Explanation in detail:
sitting at fifth position from left end of row (when 1. O is sitting at one of extreme ends.
seen in north direction) 2. O is an immediate neighbour of J’s father.
3. Two people are sitting between J’s son and J’s
father.
4. N is sitting second to right of J’s son.

CASE1:

CASE2:

Blood relation:

5. Two people are sitting between N and K’s brother.


6. M and J’s brother are sitting together.
So final arrangement
7. J’s brother is sitting second to left of J.
CASE1:

6.d
Page 732 of 1334
Subscribe The Xpress Video Course & Mock Test Package for Bank & Insurance Exams
If there are any suggestions/ errors in our PDFs Feel Free to contact us via this email: admin@exampundit.in
Ultra Practice Bundle PDF
SBI Clerk/ RBI Assist. Mains – Reasoning

CASE2: Blood Relation:

8. Two people are sitting between M and J’s sister.


So CASE1 is cancelled out
CASE1: Final arrangement:

11.b
12.d
CASE2: 13.a
14.a
15.c
SOLUTION (16-20):
Explanation in detail:
1. A sits at one of the extreme ends of the line.
9. J is not an immediate neighbour of N.
2. Only three people sit between A and G.
10. Q’s father is sitting to the immediate right of J’s
3. E sits exactly between A and G.
daughter.
4. H is a grandmother of G and C is a son in law of B
11. J’s wife is immediate neighbour of Q.
who has two daughters.
12. P is younger than L.
13. O is married to L

Page 733 of 1334


Subscribe The Xpress Video Course & Mock Test Package for Bank & Insurance Exams
If there are any suggestions/ errors in our PDFs Feel Free to contact us via this email: admin@exampundit.in
Ultra Practice Bundle PDF
SBI Clerk/ RBI Assist. Mains – Reasoning
11. Immediate neighbors of G faces opposite
direction.
12. A and D face the same direction as E.
13. There is only one married couple in a family of
which the wife is sitting third to the left of her
husband.
Final arrangement:

Blood relation:
14. D is sister in law of C.
5. H sits third to the right of E. 15. E’s maternal grandfather is sitting second from
6. B is an immediate neighbor of H and faces south one of the ends.
7. C sits second to the right of F. 16. E, who is a female and E’s maternal grandfather
8. C is not an immediate neighbor of G have at least one child, but both of them does not
9. Both the immediate neighbors of E faces south. have spouse.
10. Both the sister in laws are sitting at the ends. 17. A has only one brother, who is a grandfather of F.
So CASE2 is cancelled out (C is not an immediate 18. Youngest person is a male.
neighbor of G) Note: Direction of G not known

16.a

Page 734 of 1334


Subscribe The Xpress Video Course & Mock Test Package for Bank & Insurance Exams
If there are any suggestions/ errors in our PDFs Feel Free to contact us via this email: admin@exampundit.in
Ultra Practice Bundle PDF
SBI Clerk/ RBI Assist. Mains – Reasoning
17. e
18. b
19. d
20. c
SOLUTION (21-25):
Explanation in detail: 8. Difference of the age of E and D is 4 years less
1. CEO, COO, GM, AM, BM and they are sitting in than the age of the person who is CEO.
a line facing north direction in the above order of 9. D is unmarried and is sister in law of C.
designations from left to right. 10. Average age of A, E and D is 55 years.
2. There are as many persons sitting to the left of A
as to the right of the only son of E.
3. Two persons sit between the persons, whose ages
are 40 and 44 years.
4. CEO is younger than AM.
5. Person who is BM is the sibling of B and both sit
next to each other.
6. A is the father in law of the person who is CEO.
Final arrangement:
Husband of E is COO, and he does not have even
numbered age.
7. Age of the person who sits immediate right of the
person whose age is 40 is 71 years.

Blood relation:

21. e
22. c
Page 735 of 1334
Subscribe The Xpress Video Course & Mock Test Package for Bank & Insurance Exams
If there are any suggestions/ errors in our PDFs Feel Free to contact us via this email: admin@exampundit.in
Ultra Practice Bundle PDF
SBI Clerk/ RBI Assist. Mains – Reasoning
23. a
24. e
25. b
SOLUTION (26-30):
Explanation in detail:
1. Daughter in law of A sits second to the left of G
who is married to F.
2. B is daughter of A.
3. E is only married child of A.
4. G has two sons.

Final arrangement:

5. Sister in law of D sits fourth from the extreme end.


6. E’s father sits to the immediate right of I’s aunt. Blood relation:
7. C and H both are siblings. H and I is unmarried.
8. G is mother in law of A.

26. c
27. C
28. e
9. E sits second to the right of C. A’s brother in law
29. b
sit at the extreme right end.
30. d
10. Only one person sit between F and E’s child who
SOLUTION (31-35):
is an immediate neighbor of H.
Explanation in detail:
11. G is an immediate neighbor of B.
Page 736 of 1334
Subscribe The Xpress Video Course & Mock Test Package for Bank & Insurance Exams
If there are any suggestions/ errors in our PDFs Feel Free to contact us via this email: admin@exampundit.in
Ultra Practice Bundle PDF
SBI Clerk/ RBI Assist. Mains – Reasoning
1. O is mother of P and sits third to the left of her own 13. Two person sit between K and M who sits at one
sister. of the extreme ends.
2. O’s mother in law is the only neighbor of M’s 14. P sits to the immediate right of K.
father. 15. K sits exactly in between J and L.
3. L is aunt of P. O is not sister in law of L. 16. O sits third to the left of L.
4. Q is grandfather of P but not father of O.
CASE1:

CASE2:
5. K sits exactly in between O’s father and L.
6. M is younger than K who is spouse of J.
7. J is not a female member of the family.
8. J and O’s mother in law does not sit next to each
other. 17. N is the only neighbor of Q. (diagram modified)

9. K is not mother in law of O. CASE1:

10. P is grandchild of N and sits to the immediate right


of O’s Mother.
11. J has two children.
12. Two people sit between K and P’s father who sits
at one of the extreme ends.
CASE2:

18. J and N do not sit next to each other.


So CASE 2 is cancelled out.

Page 737 of 1334


Subscribe The Xpress Video Course & Mock Test Package for Bank & Insurance Exams
If there are any suggestions/ errors in our PDFs Feel Free to contact us via this email: admin@exampundit.in
Ultra Practice Bundle PDF
SBI Clerk/ RBI Assist. Mains – Reasoning
Final arrangement:

Blood relation:

31. a
32. e
33. e
34. b
35. b
SOLUTION (36-40):
Explanation in detail (Completely modified)
1. A sits one of the three seats from right end.
2. There is only one person sit between C and A, who
is granddaughter of C. 5. The number of person sits to the right of C is same
3. Three people sit between E’s wife and C, who buy as left to the E, who buys clock.
Bottle. 6. B sits exactly between E’s wife and C
4. E’s wife sits on the odd place value from right to
left but not extreme right end.

Page 738 of 1334


Subscribe The Xpress Video Course & Mock Test Package for Bank & Insurance Exams
If there are any suggestions/ errors in our PDFs Feel Free to contact us via this email: admin@exampundit.in
Ultra Practice Bundle PDF
SBI Clerk/ RBI Assist. Mains – Reasoning

13. G’s daughter buys a bag. B is unmarried

7. G is mother of B, who buys Mug 14. D is father of A.


8. B is brother of of H, who married C.
9. G has only two children.
10. The one who buy cup sits immediate right to the
one who buy desk.
11. E’s wife does not buy cup.

15. F and the one, who buys Pen, are not neighbors.

16. F, who sits immediate right to her father.


17. The one, who buys pen and the one, who buy desk
are couples.
12. D sits immediate left to the one who buys brush Final arrangement:

Page 739 of 1334


Subscribe The Xpress Video Course & Mock Test Package for Bank & Insurance Exams
If there are any suggestions/ errors in our PDFs Feel Free to contact us via this email: admin@exampundit.in
Ultra Practice Bundle PDF
SBI Clerk/ RBI Assist. Mains – Reasoning

4. F is not married to D.
Blood relation: 5. E is grandson of A.
6. Mother of B sits second to the left of her.

36. a
37. c 7. C sits at extreme end.
38. b 8. C likes maths.
39. d 9. C sits second to the left of B.
40. e 10. B sits second to the left of F.
SOLUTION (41-45): 11. F sits second to the right of G.
Explanation in detail: 12. Not more than three persons face south direction.
1. F who is sister of B sits second to the right of A’s CASE1:
sister.
2. D is son in law of C and father of E.
3. A is father of B.

CASE2:

Page 740 of 1334


Subscribe The Xpress Video Course & Mock Test Package for Bank & Insurance Exams
If there are any suggestions/ errors in our PDFs Feel Free to contact us via this email: admin@exampundit.in
Ultra Practice Bundle PDF
SBI Clerk/ RBI Assist. Mains – Reasoning
CASE1:

13. E does not sit left of F.


14. A, who faces opposite direction of C, sits to the
right of E. Blood relation:
15. C and E face opposite direction with respect to
each other.
16. D is not an immediate neighbor of C and faces the
same direction as F faces.
So CASE2 is cancelled out.

CASE1: Final arrangement:

CASE2:

41.d
42.a
43.c
44.d
17. The one who likes English sits second to the right 45.b
of the one who sits second to the right of the one SOLUTION (46-50):
who likes politics. Explanation in detail:
18. A likes reasoning. 1. X’s sister is sitting at the extreme right end.
19. The one who likes economics sits second to the 2. U is mother of only two daughters and is sitting
right of the one who likes history. second to the right of her husband.
Page 741 of 1334
Subscribe The Xpress Video Course & Mock Test Package for Bank & Insurance Exams
If there are any suggestions/ errors in our PDFs Feel Free to contact us via this email: admin@exampundit.in
Ultra Practice Bundle PDF
SBI Clerk/ RBI Assist. Mains – Reasoning
3. V is married male.
4. V’s father in law is sitting to the immediate right
of V.

12. V’s sister in law sits only with V.


So CASE2 is cancelled out
Final arrangement:

5. X is a male.
6. Z is not daughter of U.

Blood relation:

7. Z is sitting at the extreme right end.


8. U is sitting second to the right of X.
9. X is sitting to the immediate right of V.
10. Y is sitting to the immediate right of X.
11. Two persons sit between both the Y and W.
46.b
CASE1: 47.d
48.a
49.a
50.e
CASE2:

Download Seating Arrangement Practice Questions PDF

Get More Reasoning Practice Questions PDF

Page 742 of 1334


Subscribe The Xpress Video Course & Mock Test Package for Bank & Insurance Exams
If there are any suggestions/ errors in our PDFs Feel Free to contact us via this email: admin@exampundit.in
Ultra Practice Bundle PDF
SBI Clerk/ RBI Assist. Mains – Reasoning
Seating Circle with Blood Relation
Directions (1-5): Answer the questions based on the (a) sister
information given below: (b) Niece
P, Q, R, S, T, U, V and W are sitting around a circle facing (c) Daughter
the centre but not necessarily in the same order. Each of (d) Sister-in-law
them has a relationship with P.V is sitting second to the (e) Cannot be determined
left of father of P. U is immediate neighbor of P. S, mother 5. Who is the husband of V?
of P is sitting opposite to the sister of P. Q is sitting to the (a) Brother of Q
immediate right of wife of P. T is sitting to the immediate (b) Father of Q
left of sister of P. T who is a male is sitting second to the (c) Grandfather of R
right of mother of R. Brother of P is sitting third to right (d) Son of W
of Q. Daughter of P is sitting to third to right of sister of (e) Father of P
P. P is sitting second to the right of daughter of T. V has 6. How is S related to R?
two children in different genders. (a) sister
2. What is the position of R with respect to her (b) Aunt
grandmother? (c) Mother
(a) Third to right (d) Mother-in-law
(b) Second to left (e) Grandmother
(c) Third to right Directions (6-10): Answer the questions based on the
(d) Immediate left information given below:
(e) Cannot be determined L, M, N, O, P, Q, R and S are sitting around a circle facing
3. What is the position of P with respect to her the centre but not necessarily in the same order. S’s
sister? husband is sitting second to right of M. Q is sitting second
(a) Second to right to the left of O’s daughter. Q is the sister of R. S’s husband
(b) Immediate left is not an immediate neighbor of Q. Only one person is
(c) Third to right sitting between Q and L. Similarly there is only one person
(d) Second to left sitting between P and S’s mother. No female sits to the
(e) Immediate right immediate side of M. L is father of R. S’s mother is sitting
4. How is R related to U? to the immediate right of O, who is S’s brother. Only one
Page 743 of 1334
Subscribe The Xpress Video Course & Mock Test Package for Bank & Insurance Exams
If there are any suggestions/ errors in our PDFs Feel Free to contact us via this email: admin@exampundit.in
Ultra Practice Bundle PDF
SBI Clerk/ RBI Assist. Mains – Reasoning
person sits between S and R. P is not an immediate (b) Aunt
neighbor of R, who is the mother of N. (c) Mother
6. What is the position of P with respect to L? (d) Mother-in-law
(a) Third to right (e) Cannot be determined
(b) Second to left Directions (11-15): Answer the questions based on the
(c) Third to right information given below:
(d) Fourth to right Eight family members J, K, L, M, N, O, P and Q are sitting
(e) Cannot be determined around circular table facing the centre, but not necessarily
8. What is the position of R with respect to her in the same order. There are four males and four females.
sister? Q is third to the right of K who is the son of J. N is an
(a) Second to right immediate neighbor of Q and second to the left of his son.
(b) Immediate left O, who is married to K and is third to the right of N’s son.
(c) Third to right L and her mother J are immediate neighbor of each-other.
(d) Second to left L is mother of M. P is female but she is not daughter of L
(e) Immediate right and Q. P is in third generation. O is not immediate
9. How is S related to O? neighbor of her husband’s sister. N is son-in-law of Q and
(a) sister is not immediate neighbor of M.
(b) Niece 11. Who is to the immediate left of father of L?
(c) Daughter (a) M
(d) Sister-in-law (b) N
(e) Cannot be determined (c) K
10. Who is the husband of S? (d) J
(a) brother of Q (e) None of these
(b) Father of P 12. Who sits second to the left of grandson of J?
(c) Father of Q (a) O
(d) Son of P (b) L
(e) Cannot be determined (c) Q
11. How is S related to R? (d) N
(a) sister (e) None of these

Page 744 of 1334


Subscribe The Xpress Video Course & Mock Test Package for Bank & Insurance Exams
If there are any suggestions/ errors in our PDFs Feel Free to contact us via this email: admin@exampundit.in
Ultra Practice Bundle PDF
SBI Clerk/ RBI Assist. Mains – Reasoning
13. Who is sitting third to the left of N? brother V sits to the immediate left to the Z’s mother. Only
(a) O one person sits between Y and Z.
(b) Q 31. What is the position of U with respect to W?
(c) L (a) Third to left
(d) P (b) Second to left
(e) None of these (c) Third to right
14. Who is sitting third to the right of daughter of K? (d) Fourth to right
(a) O (e) Cannot be determined
(b) J 32. What is the position of Z with respect to father of
(c) M Y?
(d) L (a) Second to right
(e) None of these (b) Immediate left
15. What is relation between P with respect to O? (c) Third to right
(a) Son (d) Second to left
(b) Uncle (e) Immediate right
(c) Daughter 33. How is X related to V?
(d) father (a) sister
(e) None of these (b) Niece
Directions (16-20): Answer the questions based on the (c) Daughter
information given below: (d) Sister-in-law
Eight members of a family are S, T, U, V, W, X, Y and Z. (e) Cannot be determined
All are sitting around a circular table facing the centre, but 34. Who is the spouse of S?
not necessarily in the same order. Y is not an immediate (a) Y
neighbor of W.T sits second to the left of Z’s husband. Y (b) T
is the mother of U. Only one person sits between Z’s (c) W
mother and W. No female is an immediate neighbor of T. (d) Z
V’s daughter sits second to the right of X. X is the sister (e) Cannot be determined
of Y. X is not an immediate neighbor of Z’s husband. Only 35. How is Z related to U?
one person sits between S and X. S is the father of Y. Z’s (a) sister

Page 745 of 1334


Subscribe The Xpress Video Course & Mock Test Package for Bank & Insurance Exams
If there are any suggestions/ errors in our PDFs Feel Free to contact us via this email: admin@exampundit.in
Ultra Practice Bundle PDF
SBI Clerk/ RBI Assist. Mains – Reasoning
(b) Aunt (d) G
(c) Grandmother (e) C
(d) Mother-in-law 22. Who sits second to the left of grandson of A?
(e) Cannot be determined (a) C
Directions (21-25): Answer the questions based on the (b) E
information given below: (c) G
Eight family members A, B, C, D, E, F, G and H are (d) D
sitting around circular table facing the centre, but not (e) None of these
necessarily in the same order. Among them four are 23. Who is sitting third to the left of the one who likes
male and four are females. There are three husbands, Virat?
three wives, two sons and two daughters. Each family (a) A
members like different cricket players i.e. Rohit, Virat, (b) D
Dhawan , Pant, Dhoni, Hardik, Jadeja and Rahul. H (c) F
likes Dhoni and married to the one who likes Hardik. A (d) H
and B cannot sit adjacent to each other. A is the father (e) None of these
of C and F, who is a male and likes Hardik. All females 24. Who is sitting second to the right of daughter of
sit together. D likes Rahul and is sitting second to the G?
left of her father. D is the daughter of C, who is second (a) H
to the right of her son. E is mother in law of H. No (b) B
male like Dhawan. The one who likes Jadeja is married (c) A
to the one who likes Pant and neither A and B likes Pant. (d) G
E likes Dhawan and is sitting between the one who likes (e) None of these
Virat and the one who likes Jadeja. G is the father of B 25. What is relation between F with respect to B?
and is sitting between two male members. (a) Son
21. Who is to the immediate left of the one who likes (b) Uncle
Dhoni? (c) Daughter
(a) D (d) Father
(b) E (e) None of these
(c) F

Page 746 of 1334


Subscribe The Xpress Video Course & Mock Test Package for Bank & Insurance Exams
If there are any suggestions/ errors in our PDFs Feel Free to contact us via this email: admin@exampundit.in
Ultra Practice Bundle PDF
SBI Clerk/ RBI Assist. Mains – Reasoning
Directions (26-30): Answer the questions based on the (b) Third to right
information given below: (c) Immediate left
Eight members S, T, U, V, W, X, Y and Z of a family are (d) Second to left
sitting around a circular table facing to the centre and there (e) Immediate right
are three married couples in the family. Each one of them 28. How is Y related to U?
likes different colors viz. Red, Blue, Green, Yellow, (a) sister
Orange, White, Black and Pink but not necessarily in the (b) Niece
same order. (c) Daughter
Z is the only sister – in – law of S. X is an immediate (d) Sister-in-law
neighbor of her aunt Z who does not sit next to V. X and (e) Cannot be determined
Y are cousins. U does not like Pink and Black. T likes 29.Who is the husband of one who likes white
white and is daughter – in –law of U. X and Y sit next to colour?
each other. Y is third to the left of V. The one who likes (a) Who likes green colour
white sits between the persons who likes Black and Pink (b) Who likes black colour
respectively. S who is the father of X sits to the immediate (c) Who likes orange colour
left of the person who likes Red. V’s husband and V’s son (d) Who likes yellow colour
sit next to V. Green is not likes by Y’s father. Y does not (e) Cannot be determined
like Green and Black. V is the mother of S and W and sits 30. How is Z related to U?
second to the left of W. The one, who likes blue sits (a) sister
between Y and the one, who likes orange. (b) Daughter-in-law
26. What is the position of U with respect to T? (c) Grandmother
(a) Third to right (d) mother-in-law
(b) Second to left (e) Cannot be determined
(c) Third to left Directions (31-35) : Answer the questions based on the
(d) Fourth to right information given below:
(e) Cannot be determined J, K, L, M, N, O, P and Q are eight family members sitting
27. What is the position of Z with respect to father around a circular table, facing the centre, but not
of one who likes pink? necessarily in the same order. Each member likes a
(a) Second to right different color viz. Red, Maroon, White, Green, Blue,

Page 747 of 1334


Subscribe The Xpress Video Course & Mock Test Package for Bank & Insurance Exams
If there are any suggestions/ errors in our PDFs Feel Free to contact us via this email: admin@exampundit.in
Ultra Practice Bundle PDF
SBI Clerk/ RBI Assist. Mains – Reasoning
Yellow, Orange, and Black, but not necessarily in the (c) Q
same order. M sits on the immediate left of her brother, (d) L
who sits in front of his father. K has two sons and one of (e) O
the son’s like Green color. Q sits in front of her mother- 34. Which colour N likes?
in-law who likes Black color. N’s son likes Orange color. (a) Red
O sits between the person who like Maroon color and the (b) Orange
person who like Green color and her favorite color is Blue. (c) Green
L and his sister-in-law like Green and White colors (d) Maroon
respectively. J is the head of the family (i.e. he is oldest (e) White
person of the family) and sits third to the left of his wife 35. How many persons sit between L and M from left
K. P has two sisters and sits second to the right of his of L?
grandfather, who likes Red color. Only two members are (a) One
in first generation. (b) Four
31. Who sits to the immediate right of the one who likes (c) Six
red? (d) Two
(a) M (e) Three
(b) Q Directions (36-40): Answer the questions based on the
(c) O information given below:
(d) N There are eight persons J, K, L, M, Q, R, S, and T are
(e) K sitting around a circular table facing the center but not
32. Who among the following likes Yellow colour? necessarily in the same order. Each of them has a
(a) M relationship with M. There are only two married couples.
(b) J The daughter of M is sitting second to the left of the
(c) O brother of M. J is sitting to the immediate right of R. The
(d) L son of M is sitting second to the right of the wife of M.
(e) K Three persons are sitting between the son of M and the
33. Who sits third to the left of J? brother of M. K is sitting to the immediate right of the
(a) M sister of M. Two persons are sitting between K and the
(b) N father of M. Two persons are sitting between Q and L. M

Page 748 of 1334


Subscribe The Xpress Video Course & Mock Test Package for Bank & Insurance Exams
If there are any suggestions/ errors in our PDFs Feel Free to contact us via this email: admin@exampundit.in
Ultra Practice Bundle PDF
SBI Clerk/ RBI Assist. Mains – Reasoning
is sitting second to the left of Q. Three persons are sitting (c) R
between L and the wife of M. T is sitting second to the (d) M
right of the mother of M.R is neither son nor wife of M. (e) T
The sister of M is sitting second to the left of R. 40. How many persons sit between brother-in-law and
36. What is the position of T with respect to the son of father-in-law of K, when counted from left of brother
K? in law of K?
(a) Immediate right (a) One
(b) Second to left (b) Four
(c) Third to right (c) Six
(d) Immediate left (d) Three
(e) Cannot be determined (e) Two
37. Four of the following five are alike in a certain way Directions (41-45): Answer the questions based on the
based on the given arrangement and hence form a information given below:
group. Which is the one that doesn’t belong to that Seven members A, B, C, D, E, F and G of a family are
group? sitting around a circular table facing to the centre but not
(a) TQ necessarily in the same order. They likes different singers
(b) JS Sonu, Arijit, Daler, Daljit, Honey, Badshah and Tony
(c) TR but not necessarily in the same order. There are four
(d) MR generations and three married couple. E and F belong to
(e) KL the same gender. D’s son likes Daljit whose daughter is C,
38. Who is daughter-in-law of T? who likes Honey. No spouse siting together. The father of
(a) Q who likes Badshah is not sitting near to any male person.
(b) K The son in law of who likes Daljit is not an immediate
(c) S neighbor of who likes Badshah and Daljit. F is un married
(d) M female who likes Badshah. A, the eldest person in the
(e) T family likes Arjit but the wife of A does not likes Daler.
39. Who is second to the right of daughter of R? G, who is son in law of who likes Daler, likes Tony. The
(a) J person who likes Sonu is sitting second to the right of
(b) Q father of the person who likes Honey. The one who likes

Page 749 of 1334


Subscribe The Xpress Video Course & Mock Test Package for Bank & Insurance Exams
If there are any suggestions/ errors in our PDFs Feel Free to contact us via this email: admin@exampundit.in
Ultra Practice Bundle PDF
SBI Clerk/ RBI Assist. Mains – Reasoning
Daljit is second to the right of the person who is unmarried 45. Which one of the following statements is true
female. F is daughter of the person who likes Tony. according to the above mentioned arrangement?
41. Who is sitting third to the right of B? (k) G is to the immediate left of who likes sonu .
(a) C (l) F sits second to the right of who likes Tony.
(b) A (m) B sits second to right of who likes Honey.
(c) F (n) E sits fourth to the right of who likes Badshah.
(d) G (o) None of these
(e) None of these Directions (46-50) : Answer the questions based on the
42. What is the position of A with respect to daughter information given below:
of G? Eight family members A, B, C, D, E, F, G and H are sitting
(a) Immediate right around circular table facing the centre, but not necessarily
(b) Immediate left in the same order, they are closely related to A. A’s father
(c) Third to the right in law sits second to the right of F. Only one person sit
(d) Third to the left between A’s son and D. H sits to the immediate right of
(e) None of these A’s son. A’s husband and H are sitting opposite to each
43. What is F’s position with respect to her other. A sits to the immediate left of E. F sits to the
grandmother? immediate right of G. A’s father in law and A’s son in law
(k) Third to the right are sitting opposite to each other. C sits to the second to
(l) Second to the right the left of A’s daughter. A’s daughter in law sits third to
(m) Immediate right the right to the A’s mother in law. D sits third to the right
(n) Second to the left of E. Only one person sits between A’s husband and B.
(o) Third to the left 46. Who is to the immediate left of H?
44. Who is sitting third to the right of who likes Sonu? (a) B
(a) F (b) C
(b) B (c) D
(c) D (d) G
(d) C (e) None of these
(e) G 47. Who is daughter-in-law of H?
(a) C

Page 750 of 1334


Subscribe The Xpress Video Course & Mock Test Package for Bank & Insurance Exams
If there are any suggestions/ errors in our PDFs Feel Free to contact us via this email: admin@exampundit.in
Ultra Practice Bundle PDF
SBI Clerk/ RBI Assist. Mains – Reasoning
(b) E (a) B and G
(c) G (b) C and D
(d) A (c) G and H
(e) None of these (d) B and A
48. Who is sitting third to the left of wife of E? (e) None of these
(a) A 50. Who is sitting fourth to the right of G?
(b) G (a) E
(c) D (b) D
(d) F (c) F
(e) None of these (d) B
49. Who are the daughter-in-law and son – in law of (e) C
A?

Seating Circle with Blood Relation – Answer and Explanation


SOLUTION (1-5):
Explanation in detail:
19. S, mother of P is sitting opposite to the sister of P.
20. Daughter of P is sitting to third to right of sister of P.
21. T is sitting to the immediate left of sister of P.
22. Q is sitting to the immediate right of wife of P.
CASE1:

CASE3:

CASE2:

Page 751 of 1334


Subscribe The Xpress Video Course & Mock Test Package for Bank & Insurance Exams
If there are any suggestions/ errors in our PDFs Feel Free to contact us via this email: admin@exampundit.in
Ultra Practice Bundle PDF
SBI Clerk/ RBI Assist. Mains – Reasoning

23. Brother of P is sitting third to right of Q.


24. Next we have only 1 position left for placing P in all 3
CASE1:
arrangements as
CASE1:

CASE2:

CASE2:

CASE3:

CASE3:

Page 752 of 1334


Subscribe The Xpress Video Course & Mock Test Package for Bank & Insurance Exams
If there are any suggestions/ errors in our PDFs Feel Free to contact us via this email: admin@exampundit.in
Ultra Practice Bundle PDF
SBI Clerk/ RBI Assist. Mains – Reasoning

Blood relation:

25. P is sitting second to the right of daughter of T.


26. T who is a male is sitting second to the right of mother
of R.
1. d
27. V is sitting second to the left of father of P. U is
2. a
immediate neighbor of P.
3. b
28. V has two children in different genders.
4. b
So final arrangement
5. e
Case-2 & 3 gets rejected as these both cases were not
satisfied with 8th statement (as we know P is a male
SOLUTION (6-10):
member)
Explanation in detail: (completely modified)
In case-1, P’s sister, P’s brother and P’s daughter were
seated apart from Q. This implies Q must be son of P
and the blood relation is shown below,

Page 753 of 1334


Subscribe The Xpress Video Course & Mock Test Package for Bank & Insurance Exams
If there are any suggestions/ errors in our PDFs Feel Free to contact us via this email: admin@exampundit.in
Ultra Practice Bundle PDF
SBI Clerk/ RBI Assist. Mains – Reasoning

Case 2 rejected as there is no place for P.

Page 754 of 1334


Subscribe The Xpress Video Course & Mock Test Package for Bank & Insurance Exams
If there are any suggestions/ errors in our PDFs Feel Free to contact us via this email: admin@exampundit.in
Ultra Practice Bundle PDF
SBI Clerk/ RBI Assist. Mains – Reasoning
17. O, who is married to K and is third to the right of
N’s son.
18. L is mother of M. P is female but she is not
daughter of L and Q. P is in third generation.
CASE1:

CASE2:

6. d
7. b
8. a
9. c
10. c
SOLUTION(11-15):
Explanation in detail:
15. Q is third to the right of K who is the son of J.
16. N is an immediate neighbour of Q and second to
the left of his son.

Page 755 of 1334


Subscribe The Xpress Video Course & Mock Test Package for Bank & Insurance Exams
If there are any suggestions/ errors in our PDFs Feel Free to contact us via this email: admin@exampundit.in
Ultra Practice Bundle PDF
SBI Clerk/ RBI Assist. Mains – Reasoning

19. P is female but she is not daughter of L and Q.


20. P is in third generation.
21. O is not immediate neighbour of her husband’s
sister.
22. N is son-in-law of Q and is not immediate
neighbour of M.
23. L and her mother J are immediate neighbour of
each-other.
24. O is not immediate neighbour of her husband’s In case 2 there is no such space for J and L.
sister. N is son-in-law of Q and is not immediate So CASE2 is cancelled out.
neighbour of M. Final arrangement:
CASE1:

CASE2: Blood relation:

Page 756 of 1334


Subscribe The Xpress Video Course & Mock Test Package for Bank & Insurance Exams
If there are any suggestions/ errors in our PDFs Feel Free to contact us via this email: admin@exampundit.in
Ultra Practice Bundle PDF
SBI Clerk/ RBI Assist. Mains – Reasoning
CASE2:

11. b
12. d
13. a
14. c
31. S is the father of Y.
15. c
32. Z’s brother V sits to the immediate left to the Z’s
SOLUTION(16-20):
mother.
Explanation in detail:
33. Only one person sits between Z’s mother and W.
25. T sits second to the left of Z’s husband.
34. Only one person sit between Y and Z.
26. No female is an immediate neighbor of T.
35. Y is the mother of U.
27. V’s daughter sits second to the right of X.
36. Y is not an immediate neighbor of W.
28. X is the sister of Y.
29. X is not an immediate neighbor of Z’s husband.
30. Only one person sits between S and X.
CASE1:

From above family tree CASE1 is cancelled out.


CASE2:

Page 757 of 1334


Subscribe The Xpress Video Course & Mock Test Package for Bank & Insurance Exams
If there are any suggestions/ errors in our PDFs Feel Free to contact us via this email: admin@exampundit.in
Ultra Practice Bundle PDF
SBI Clerk/ RBI Assist. Mains – Reasoning
19. d
20. c
SOLUTION (21-25):
Explanation in detail: (completely modified)
30. H likes Dhoni and married to the one who likes
Hardik.
31. A is the father of C and F, who is a male and
likes Hardik.
32. G is the father of B and is sitting between two
male members.
CASE2 is final arrangement: Figure modified

Blood relation:

33. E is mother in law of H.


34. D is the daughter of C, who is second to the right
of her son.

16. C
17. C
18. b

Page 758 of 1334


Subscribe The Xpress Video Course & Mock Test Package for Bank & Insurance Exams
If there are any suggestions/ errors in our PDFs Feel Free to contact us via this email: admin@exampundit.in
Ultra Practice Bundle PDF
SBI Clerk/ RBI Assist. Mains – Reasoning
39. A and B cannot sit adjacent to each other.
Final arrangement:

35. The one who likes Jadeja is married to the one


who likes Pant and neither A and B likes Pant
36. E likes Dhawan and is sitting between the one
who likes virat and the one who likes Jadeja.
37. D likes Rahul and is sitting second to the left of
21. e
her father. No male like Dhawan.
22. c
38. All females sit together.
23. a
Blood relation: (figure modified)
24. d
As there are three married couples therefore G must
25. b
married to C (also to satisfy 7th statement)
SOLUTION (26-30):
Explanation in detail:
36. S who is the father of X sits to the immediate left
of the person who likes Red.
37. X is an immediate neighbour of her aunt Z who
does not sit next to V.
38. Y is third to the left of V.
39. X and Y sit next to each other.
40. V is the mother of S and W and sits second to the
left of W.
41. Z is the only sister – in – law of S.

Page 759 of 1334


Subscribe The Xpress Video Course & Mock Test Package for Bank & Insurance Exams
If there are any suggestions/ errors in our PDFs Feel Free to contact us via this email: admin@exampundit.in
Ultra Practice Bundle PDF
SBI Clerk/ RBI Assist. Mains – Reasoning

Blood relation:

42. T likes white and is daughter – in –law of U.


43. The one who likes blue sits between Y and the one Final arrangement:
who likes orange.
44. The one who likes white sits between the persons
who likes Black and Pink respectively.
45. V’s husband and son sit next to her.
46. Green is not likes by Y’s father. Y does not like
Green and Black.
47. U does not like Pink and Black.
48. X and Y are cousins. 26. a
27. E
28. e
29. b
30. B
SOLUTION (31-35):
Explanation in detail: (completely modified)

Page 760 of 1334


Subscribe The Xpress Video Course & Mock Test Package for Bank & Insurance Exams
If there are any suggestions/ errors in our PDFs Feel Free to contact us via this email: admin@exampundit.in
Ultra Practice Bundle PDF
SBI Clerk/ RBI Assist. Mains – Reasoning

Final arrangement:

Page 761 of 1334


Subscribe The Xpress Video Course & Mock Test Package for Bank & Insurance Exams
If there are any suggestions/ errors in our PDFs Feel Free to contact us via this email: admin@exampundit.in
Ultra Practice Bundle PDF
SBI Clerk/ RBI Assist. Mains – Reasoning

3. Three persons are sitting between L and the wife


of M.
4. The son of M sits second to the right of the wife of
M.
31. a
32. A
33. e
34. d
35. d
SOLUTION (36-40):
Explanation in detail:
1. M sits second to the left of Q.
2. Two persons are sitting between Q and L.

Page 762 of 1334


Subscribe The Xpress Video Course & Mock Test Package for Bank & Insurance Exams
If there are any suggestions/ errors in our PDFs Feel Free to contact us via this email: admin@exampundit.in
Ultra Practice Bundle PDF
SBI Clerk/ RBI Assist. Mains – Reasoning
5. Three persons are sitting between the son of M and
the brother of M.
6. The daughter of M sits second to the left of the
brother of M.

9. The sister of M is sitting second to the left of R.


10. K sits to the immediate right of the sister of M.

7. J sits to the immediate right of R.


8. R is neither son nor wife of M.

Page 763 of 1334


Subscribe The Xpress Video Course & Mock Test Package for Bank & Insurance Exams
If there are any suggestions/ errors in our PDFs Feel Free to contact us via this email: admin@exampundit.in
Ultra Practice Bundle PDF
SBI Clerk/ RBI Assist. Mains – Reasoning

36. a
37. c
So there is no space for K in CASE2 so CASE2 is 38. b
cancelled out. 39. c
11. Two persons are sitting between K and the father 40. e
of M. SOLUTION (41-45):
12. T sits second to the right of the mother of M. Explanation in detail:
26. F is unmarried female who likes Badshah.
27. E and F belongs to the same gender.
28. G who is son in law of who likes Daler, likes Tony.
29. F is daughter of the person who likes Tony.
30. D’s son likes Daljit whose daughter is C, who likes
Honey.

31. The father of who likes Badshah is not sitting near


to any male person.

Page 764 of 1334


Subscribe The Xpress Video Course & Mock Test Package for Bank & Insurance Exams
If there are any suggestions/ errors in our PDFs Feel Free to contact us via this email: admin@exampundit.in
Ultra Practice Bundle PDF
SBI Clerk/ RBI Assist. Mains – Reasoning
32. The son in law of who likes Daljit is not an Blood relation:
immediate neighbor of who likes Badshah and
Daljit.
33. A, the eldest person in the family likes Arjit but
the wife of A does not likes Daler.
34. The person who likes Sonu is sitting second to the
right of father of the person who likes Honey.
35. The one who likes Daljit is second to the right of
Final arrangement:
the person who is unmarried female.

36. No spouse siting together.


37. G is not sitting near to any male person.
38. G is not an immediate neighbor F and B. 41. d
39. D is sitting second to the right of B. 42. a
40. B is second to the right to the F. 43. c
44. a
45. b
SOLUTION(46-50):
Explanation in detail:
20. A sits to the immediate left of E.
21. D sits third to the right of E.
22. Only one person sit between A’s son and D.
23. H sits to the immediate right of A’s son.

Page 765 of 1334


Subscribe The Xpress Video Course & Mock Test Package for Bank & Insurance Exams
If there are any suggestions/ errors in our PDFs Feel Free to contact us via this email: admin@exampundit.in
Ultra Practice Bundle PDF
SBI Clerk/ RBI Assist. Mains – Reasoning

CASE1:

CASE2:

CASE2:

27. C sits to the second to the left of A’s daughter.

24. A’s husband and H are sitting opposite to each So CASE1 is cancelled out
other. CASE1:
25. F sits to the immediate right of G.
26. Only one person sits between A’s husband and B.

CASE1:

CASE2:

Page 766 of 1334


Subscribe The Xpress Video Course & Mock Test Package for Bank & Insurance Exams
If there are any suggestions/ errors in our PDFs Feel Free to contact us via this email: admin@exampundit.in
Ultra Practice Bundle PDF
SBI Clerk/ RBI Assist. Mains – Reasoning
Blood relation:

Final
arrangement:
28. A’s daughter in law sits third to the right to the A’s
mother in law.
29. A’s father in law sits second to the right of F.
30. A’s father in law and A’s son in law are sitting
opposite to each other.

46. b
47. d
48. a
49. a
50. e

Download Seating Arrangement Practice Questions PDF

Get More Reasoning Practice Questions PDF

Page 767 of 1334


Subscribe The Xpress Video Course & Mock Test Package for Bank & Insurance Exams
If there are any suggestions/ errors in our PDFs Feel Free to contact us via this email: admin@exampundit.in
Ultra Practice Bundle PDF
SBI Clerk/ RBI Assist. Mains – Reasoning
Seating Linear with Distance
Directions: Study the following information and c. U, B
answer the questions following. d. P, F
Sports Council meeting is going on in a school about e. A, Q
the football matches which will be conducted. 6 2) How far is U from A?
members from two teams are sitting facing each a. 16 m
other around a rectangular table of dimensions (15m b. 225 m
x 4m). Team 1 is facing South side. All the members c. √241 m
are at equidistant points. d. 241 m
i) P, Q, R, S, T and U constitute of Team 1. A, B, C, e. 15 m
D, E and F constitute Team 2. 3) If E and R interchange their positions, then
ii) P and U are at extreme ends of the table and are how far is E from U?
15m apart. a. 7 m
iii) R sits 6m to the East of P and 9m to the West of b. 9 m
U. c. 6 m
iv) T is not an immediate neighbor of R, but sits d. 8 m
exactly opposite to D, which is 12m to the East of A. e. 5 m
v) Q sits 5m to the North East of A and 5m to the 4) If E gets up from his position and walks
North West of B. towards Q, What will be the shortest possible
vi) C is not an immediate neighbor of D. distance covered by E?(neglecting practical
vii) F sits to √52 m South West of S and is 6m to the cases)
left of E. a. √52 m
1) Who among the following sits at the extreme b. 10 m
end of the table? c. 6 m
a. P, T d. 9 m
b. T, A e. None of these

Page 768 of 1334


Subscribe The Xpress Video Course & Mock Test Package for Bank & Insurance Exams
If there are any suggestions/ errors in our PDFs Feel Free to contact us via this email: admin@exampundit.in
Ultra Practice Bundle PDF
SBI Clerk/ RBI Assist. Mains – Reasoning
5) If T and C leaves the meeting, and Q changes right of his original place and sits to the immediate
his seat such that, Q is now 5m to North West of left of E, after the movement. C sits at extreme right
F, then, what is the distance between Q and B? end after movement. A never sits adjacent to D or C.
a. √52 m G moves to the one of the seats to his right. C sits at
b. 36 m a distance of twice of the distance of E, who does not
c. 6 m sit adjacent to D. The one, who is 45 meters away, is
d. 4 m not an immediate neighbour of D. D is adjacent to F
e. 3 m only after rearrangement.
Directions 6-10: Answer the questions based on 6) Who among the following sits third to the right
the information given below: of the one is 125 meters away after
Eight persons A, B, C, D, E, F, G and H are sitting rearrangement?
in a row facing north direction. Each of them sits in a. H
a row with certain distance from one other viz. 5, 10, b. G
15, 35, 45, 75, 90 and 125. They can be shifted as c. C
per the given information and create two d. B
arrangements – before and after arrangements. e. None of these
The number of persons between C and F, before 7) Who among the following sits third to the left
rearrangement, are three less than the number of of H, after rearrangement?
persons between C and F, after rearrangement. a. F
B, who is 10 meters away from the neighbor, sits to b. The one, who is 15 meters away
the immediate left of C, before rearrangement. c. The one, who is 10 meters away
Distance of F is 25 times the distance of the d. E
immediate left of A. F never sits adjacent to B. G his e. None of these
15 meters away and sits to the immediate right of H, 8) Who among the following sits at the extreme
after movement. Three persons sit to the right of one, right end, before rearrangement?
who is 125 meters away, before rearrangement. D, a. H
who is 75 meters away, shifted towards the fifth to b. F
the left of his original position. G sits adjacent to c. G
neither C nor D, after rearrangement. B moves to the d. D

Page 769 of 1334


Subscribe The Xpress Video Course & Mock Test Package for Bank & Insurance Exams
If there are any suggestions/ errors in our PDFs Feel Free to contact us via this email: admin@exampundit.in
Ultra Practice Bundle PDF
SBI Clerk/ RBI Assist. Mains – Reasoning
e. None of these W is sitting adjacent to the person, who has 13 m
9) Who among the following sits to the immediate distance to his left. Two persons are sitting between
left of the one, who sits at 45 meters away before R and the person, who has 13m distance to his left.
arrangement? Three persons are sitting between P and T who is
a. H immediate neighbor of W. S is sitting immediately
b. E right of V who sits fourth from the right end. One
c. G person is sitting between the persons, who have 9m
d. C distance to his left and who have 13m distance to his
e. None of these left. 9. One of the persons sitting at the extreme end
10) What is the total distance to the left of G after has 5m distance to his left. Two persons are sitting
rearrangement? between U and the person has 9m distance to his left.
a. 940m Two persons are sitting between W and V and W sits
b. 950m to the right of V. Distance between Q and U is 12m.
c.960m Distance to the left of W is 2m less than the distance
d. 955m to the left V.
e. None of these @U #R %S
Directions: Answer the questions based on the 11) What is the sum of distance between T and V?
information given below: a. 20
Eight persons, P, Q, R, S, T, U, V and W are sitting b. 25
in a straight row. All of them are facing the north. c. 17
Each person has different distance to their left. d. 18
@A – One of the persons sitting adjacent to A is 16m e. None of these
away 12) How many persons are sitting between P and
# A – Distance to the left of A is twice the distance Q?
to the left of the person sitting 2nd to the right of A. a. 2
% A- Difference in the distance to the left of A and b. 3
the distance to the left of the person sitting at c. 4
extreme right end is 3m d. 1
e. None of these

Page 770 of 1334


Subscribe The Xpress Video Course & Mock Test Package for Bank & Insurance Exams
If there are any suggestions/ errors in our PDFs Feel Free to contact us via this email: admin@exampundit.in
Ultra Practice Bundle PDF
SBI Clerk/ RBI Assist. Mains – Reasoning
13) __ and ___ are sitting adjacent to R. Distance between I and F is 10m. Distance between
a. P, V K and G is 2m more than the distance between I and
b. P, S F. K is the immediate neighbor of G but doesn’t sit
c. S, V at the extreme end. H is sitting 2nd to the right of D.
d. T, Q F is sitting 2nd to the right of H. E is sitting adjacent
e. Cannot be determined to B and the distance between them is 16m. F is
14) How many persons are sitting in the left of V? sitting 2nd to the left of K. Distance between D and
a. 3 F is 30m. Distance between A and G is equal to the
b. 4 distance between H and I. G is sitting 3rd to the left
c. 2 of E. The bottommost floor is numbered 1 and the
d. 1 topmost floor is numbered 9.
e. None of these 2nd eldest child lives on floor 8. Child, who lives on
15) What is the total distance between W and S? floor 7, is seven years younger than I. Four children
a. 20 live above the 13 years old child. I is not 16 years
b. 25 old. D lives three floors below I. Eldest child lives
c. 16 two floors above H, who is not the youngest. H
d. 18 doesn’t live adjacent to D’s floor. More than two
e. None of these children live between eldest child and E, who is 10
Directions: Read the following information and years old. B lives three floors above C. Neither A nor
answer the questions given below. B is 8 years old. F lives immediately below the 8
A certain number of children are sitting in a row years old child. A lives above G. F is elder than B
facing north. The length of row is 121 m. The but younger than D.
distance is in increasing order from left end starting 16) What is the position of I with respect to H?
from 6m, 7m, 8m and so on. They also live on a 9 a. 3rd to the right
storey building. The bottommost floor is numbered b. 4th to the right
1 and the topmost floor is numbered 9. Age of each c. 2nd to the left
of these children is different and the ages are 6, 8, 9, d. 5th to the right
10, 12, 13, 14, 16 and 18. e. None of these
17) How many persons are sitting in the row?

Page 771 of 1334


Subscribe The Xpress Video Course & Mock Test Package for Bank & Insurance Exams
If there are any suggestions/ errors in our PDFs Feel Free to contact us via this email: admin@exampundit.in
Ultra Practice Bundle PDF
SBI Clerk/ RBI Assist. Mains – Reasoning
a. 11 parent has three or more than three children. Every
b. 9 person sits at different position with respect to each
c. 14 other having different number of chocolates. There
d. 10 is no single parent and two married couples in the
e. 7 family.
18) What is the position of A with respect to I? V sits 12m west to the only son of P. U is one of the
a. 3rd to the left two daughters of R and has 5 chocolates less than V.
b. 4th to the right Mother of S sits 3m west to the person, who is 10m
c. 2nd to the left south of S, who is not married. S is not in the 3rd
d. 5th to the right generation but has 2nd highest number of
e. None of these chocolates. P sits 4m north to the person who has 2nd
19) How many children live between F and 9 lowest number of chocolates. Q has more chocolates
years old child? than only R. T is not a female. The person who sits
a. Four 10m north of U is U’s sister. T has more chocolates
b. Five than P but less than U. The person with 3rd highest
c. Two number of chocolates sits 5m south west of T. R does
d. Six not belong to the 1st generation. T is not in the same
e. None of these generation as P. T is not the son of P. Q is not in 3rd
20) Who lives immediately above the 12 years old generation. Q and R have same gender. One of the
child? person sits exactly to the west of P. U doesn’t have
a. B highest number of chocolates.
b. C 21) What is the direction of the person who has
c. A 4th lowest number of chocolates with respect to
d. H the sister of U?
e. None of these a. North east
Directions: Answer the questions based on the b. South east
information given below: c. North west
There are seven members P, Q, R, S, T, U and V in d. South west
a family consist of three generations. None of the e. None of these.

Page 772 of 1334


Subscribe The Xpress Video Course & Mock Test Package for Bank & Insurance Exams
If there are any suggestions/ errors in our PDFs Feel Free to contact us via this email: admin@exampundit.in
Ultra Practice Bundle PDF
SBI Clerk/ RBI Assist. Mains – Reasoning
22) If difference between the number of c. T
chocolates with U and the father of S is 8 and P d. R
has 9 chocolates, how many chocolates does V e. Q
have? Directions: Answer the questions based on the
a. 20 information given below:
b. 17 10 persons A, B, C, D, E, P, Q, R, S and T sit in two
c. 22 parallel rows in such a way that persons sitting in
d. 23 row 1 faces in south while persons sitting in row 2
e. None of these. faces in north. Persons of Row 1 exactly face the
23) What is the shortest distance between the another person of row 2. Every person have different
point which is 6m south of the person with highest age i.e. 20, 24, 27, 31, 32, 36, 37, 39, 42 and 45.
number of chocolates and the point which is 4m Distance between any two adjacent persons is equal.
north of the person with least chocolates? The distance between the two parallel rows is 10m.
a. 12m 36 years old faces the person who sits 2nd to the right
b. 14m of B. T sits 2nd to the left of 5th eldest person.
c. 17m C sits to the immediate right of E and faces north. A
d. 13m faces north. A faces the person, who sits to the
e. None of these. immediate left of 2nd eldest person. Q sits to the
24) What is the direction of the person with 3rd immediate right of the person, who is 31 years old.
highest number of files with respect to T’s P sits 2nd to the right of R. Two persons sit between
brother-in-law? T and the person, whose age is a prime number.
a. North-east R is 27 years old, faces South and sits opposite to the
b. South-east one, who sits to the immediate right of D. 3rd
c. South-west youngest person sits to the immediate right of 37
d. North-west years old person. D is 2nd youngest person.
e. None of these. By making the above arrangement, every person
25) Who is the father of U? moves in a certain direction according to the
a. S following conditions given below-
b. P

Page 773 of 1334


Subscribe The Xpress Video Course & Mock Test Package for Bank & Insurance Exams
If there are any suggestions/ errors in our PDFs Feel Free to contact us via this email: admin@exampundit.in
Ultra Practice Bundle PDF
SBI Clerk/ RBI Assist. Mains – Reasoning
1. E, who is 8 years older than D, moves 18m 28) What is the shortest distance between the
straight, then turns left of 12m. He walks another 8m eldest and the 2nd youngest person after the 3rd
towards south and meets the person, whose age is replacement?
less than 36 years. He exchanges his seat with that a. 18m
person. b. Between 18m and 21m
2. After that, the person who sits to the immediate c. Between 21m and 24m.
right of E, turns right and walks for 3m to turn left d. Between 15m and 18m
and walks another 15m. After that he again turns left e. None of these.
and covers 9m to turn left and walks 5m and meets 29) After the 1st replacement, what is the distance
the person, who is elder than him. He exchanges his between P and A?
seat with him. a. 12m
3. After that, C, who is not the youngest person, turns b. 18m
back and moves 4m to turn right. He walks 18m and c. Between 18m and 23m
turns right to walk another 14m. He meets a person d. 6m
with whom he exchanges his seat. e. None of these.
26) What is the position of the youngest person 30) After the 2nd replacement, what is the
with respect to P after all the replacements? distance between C and Q?
a. 4th to the left a. 12m
b. Immediate right b. 18m
c. 2nd to the right c. Between 18m and 23m
d. 3rd to the left d. 6m
e. None of these. e. None of these.
27) Who among the following doesn’t form a Directions: Answer the questions based on the
group after the 3rd replacement? information given below.
a. B Directions: Answer the questions based on the
b. P information given below.
c. E Six persons P, Q, R, S, T and U sit in a row facing
d. T north. The persons are sitting from left to right such
e. A that there is a gap of 5m between two adjacent

Page 774 of 1334


Subscribe The Xpress Video Course & Mock Test Package for Bank & Insurance Exams
If there are any suggestions/ errors in our PDFs Feel Free to contact us via this email: admin@exampundit.in
Ultra Practice Bundle PDF
SBI Clerk/ RBI Assist. Mains – Reasoning
persons. The distance between leftmost and d. 17m
rightmost person is 25m. Four persons move from e. 23m
their seats. Each of them has different number of 33) Who among the following has 9 pens?
pens from 5 to 10. No person moves beyond the a. T
leftmost and rightmost person. They are standing b. U
according to increasing order of number of pens c. S
from left to right. d. Q
R moves 5m in the north and walks 15m in either e. Cannot be determined
west or east direction. 34) Who is in the north of Q after movement?
P has 2 pens lesser than R but more than that of Q. a. P
S has highest number of pens and after movement he b. T
stands 10m from east of T. c. R
P moves 10m in north and then moves 10m in the d. S
east. e. None of these
S can only walk in south direction. 35) How many pens does S have?
T walks 8m in south and then moves 5m in east or a. 5
west direction. b. 10
31) Who among the following has lowest number c. 6
of pens? d. 7
a. U e. None of these
b. T Directions: Answer the questions based on the
c. Q information given below.
d. R Eight persons L, M, N, O, P, Q, R and S are sitting
e. P at random places with some distance from each
32) What is the shortest distance between the other. Each of them has different number of boxes.
final position of P and T? L is sitting 12m north to the person, who has the
a. 10m second lowest number of boxes. P has lesser boxes
b. 8m than one of the persons sitting in the south west of P.
c. 16m Q has more boxes than at least two persons. R is

Page 775 of 1334


Subscribe The Xpress Video Course & Mock Test Package for Bank & Insurance Exams
If there are any suggestions/ errors in our PDFs Feel Free to contact us via this email: admin@exampundit.in
Ultra Practice Bundle PDF
SBI Clerk/ RBI Assist. Mains – Reasoning
sitting 6m east to Q, who is sitting 11m north to P. L 39) What is the direction of the person, who has
is sitting north west to P and has less number of 3rd lowest number of boxes with respect to O?
boxes than P. O is sitting 8m west to P. N is sitting a. North
6m east to M. R is sitting 25m north to S, who has b. South west
the highest number of boxes. L has more boxes than c. North east
O, who has more boxes than Q. N is sitting 10m in d. South
the south of O. N has more boxes than O. e. None of these
36) What is the shortest distance between M and 40) How many persons have more boxes than the
the person, who has the lowest number of boxes? person, who is sitting south east to M?
a. 28m a. Three
b. 26m b. Zero
c. 29m c. Four
d. 19m d. Six
e. None of these e. None of these
37) If Q has 39 boxes, then what is the possible Directions: Read the given information carefully
number of boxes with M? and answer the questions.
a. 40 A certain number of persons are sitting in a straight
b. 45 row. All of them are facing north. Six of them have
c. 30 six different cars among, Audi, Baleno, Porsche,
d. 41 Bolero, Swift and Rexton.
e. Cannot be determined Person, who has Rexton, is sitting 3rd from the left
38) How many persons have less number of boxes end. One person is sitting between B and S and
than the person, who is sitting North West to the distance between both of them is 20m. Two persons
person, who has 2nd highest number of boxes? sit between the persons, who have Rexton and
a. 4 Baleno. Four persons are sitting between S and the
b. 3 person, who has Baleno. M is sitting 4th to the right
c. 1 of B. Person, who has Swift is sitting at the right end.
d. 5 Five persons are sitting between R, who has Porsche
e. 2 and the person, who has Swift. Five persons are

Page 776 of 1334


Subscribe The Xpress Video Course & Mock Test Package for Bank & Insurance Exams
If there are any suggestions/ errors in our PDFs Feel Free to contact us via this email: admin@exampundit.in
Ultra Practice Bundle PDF
SBI Clerk/ RBI Assist. Mains – Reasoning
sitting between R and D. Two persons are sitting d. 19
between K and the person, who has Audi. K is sitting e. 16
2nd to the left of D and distance between them is 44) What is the total distance between S and K?
20m. Two persons are sitting between M and the a. 90m
person, who has Audi. Number of persons sitting b. 80m
between M and R is a prime number. Distance c. 100m
between M and K is 50m. Number of persons sitting d. 110m
between D and the person, who has Baleno is less e. None of these
than 10. The number of persons sitting in the left of 45) How far is B from D?
K is equal to the number of persons sitting in the a. 90m
right of the person, who has Bolero. Distance b. 80m
between B and M is 30m. c. 100m
41) How many persons are sitting in the row? d. 110m
a. 25 e. None of these
b. 27 Directions: Answer the questions based on the
c. 24 information given below.
d. 31 Seven candidates P, Q, R, S, T, U and V like
e. None of these different colors and stand at different places. Each of
42) Who has Bolero? them appeared for PO Mains exam and scored
a. B different marks (in integers).
b. S One of the candidates likes Pink. Three candidates
c. M score more than S, who stands 6m north of Q. The
d. D candidate who likes Violet and sits 7m east of S,
e. None of these scores less than S. Candidate who scores the 2nd
43) If 7 persons are sitting between D and H, then lowest scores 97 marks. The highest scorer stands
how many persons are sitting between H and S? 8m east of T, who likes Red. P scores more than S
a. 21 and sits 6m south of T. More than two candidates
b. 20 score less than T. Difference between the marks of S
c. 25 and the one who likes Violet is less than the

Page 777 of 1334


Subscribe The Xpress Video Course & Mock Test Package for Bank & Insurance Exams
If there are any suggestions/ errors in our PDFs Feel Free to contact us via this email: admin@exampundit.in
Ultra Practice Bundle PDF
SBI Clerk/ RBI Assist. Mains – Reasoning
difference of marks of S and Q. R doesn’t score the V then what is the difference between the marks
highest marks. Neither U nor V likes Violet. U of highest scorer and T if the difference between
stands 9m south of R, who scores 105 marks. V the marks of the candidate who likes Yellow and
scores 3 marks less than U. The candidate, who T is two marks more than the difference between
stands at extreme east, likes Green and sits at a the 2nd and 3rd highest scorer?
distance of 6m from the person, who likes Blue. a. 6 marks
Person who stands at the extreme south likes neither b. 8 marks
Magenta nor Yellow. Candidate who likes Yellow c. 11 marks
scores more than the one, who likes Magenta. d. 13 marks
46) How many marks does T score? e. None of these
a. 96 49) The person, who stands in the north east of Q,
b. 84 likes ____ color.
c. 94 a. Red
d. 125 b. Pink
e. 105 c. Violet
47) What is the shortest distance between the d. Magenta
candidate who likes Green and the one who likes e. None of these
Magenta? 50) Who likes Yellow color?
a. Between 12m and 17m. a. S
b. Between 17m and 21m. b. R
c. Between 21m and 24m c. Q
d. Less than 12m d. U
e. More than 24m. e. V
48) If the difference between the marks of P and
U is 44 marks and T scores 53 marks more than

Seating Linear with Distance - Answer and Explanation


Answers: 1-5 3) B
1) D 4) A
2) C 5) A
Page 778 of 1334
Subscribe The Xpress Video Course & Mock Test Package for Bank & Insurance Exams
If there are any suggestions/ errors in our PDFs Feel Free to contact us via this email: admin@exampundit.in
Ultra Practice Bundle PDF
SBI Clerk/ RBI Assist. Mains – Reasoning
Solution: 1-5 Answers:
i) P, Q, R, S, T and U constitute of Team 1. A, B, C, 6. b
D, E and F constitute Team 2. 7. e
ii) P and U are at extreme ends of the table and are 8. d
15m apart. 9. e
iii) R sits 6m to the East of P and 9m to the West of 10. d
U. Solutions: 6-10
1. B, who is 10 meters away from the neighbor, sits
to the immediate left of C, before rearrangement.
2. F distance is 25 times the distance of the
immediate left of A. F never sits adjacent to B.
iv) T So, F and A have 125 and 5 meters respectively.
is not an immediate neighbor of R, but sits exactly 3. G is 15 meters away and sits immediate right of
opposite to D, which is 12m to the East of A. H, after movement.
v) Q sits 5m to the North East of A and 5m to the 4. Three persons sit to the right of one, who is 125
North West of B. meters away, before rearrangement.
vi) C is not an immediate neighbor of D. 5. D, who is 75 meters away, shifted fifth to the left
vii) F sits to √52 m South West of S and is 6m to the of his original position.
left of E. 6. C distance is 2 times than that of E.
So, C and E are 90 and 45 meters away respectively.
So, there are three cases,

Before arrangement B(10) C F(125) D

After arrangement D F(125) H G B C

Page 779 of 1334


Subscribe The Xpress Video Course & Mock Test Package for Bank & Insurance Exams
If there are any suggestions/ errors in our PDFs Feel Free to contact us via this email: admin@exampundit.in
Ultra Practice Bundle PDF
SBI Clerk/ RBI Assist. Mains – Reasoning

Case 1

Before arrangement G B(10) C(90) A F(125) D(75)

After arrangement A D(75) F(125) H G B E C

Case 2

Before arrangement B(10) C(90) G A(5) F(125) D

After arrangement A(5) F(125) D(75) G(15) B C

Case 3
7. The one, who is 45 meters away, does not sit adjacent to D.
8. C sits at extreme right end only after movement.
9. D is adjacent to F only after rearrangement.
10. B moves to the right of his original place and sits immediate left of E, after the movement.

Before
arrangement B(10) C(90) G(15) A(5) F(125) E(45) H(35) D(75)

After
arrangement A(5) F(125) D(75) H(35) G(15) B(10) E(45) C(90)

So, case 1 is rejected. 12) D


11. A never sits adjacent to D or C. 13) A
So, case 2 is rejected. 14) B
The final arrangement is shown below: 15) A
Answer: Solutions 11-15
11) C
Page 780 of 1334
Subscribe The Xpress Video Course & Mock Test Package for Bank & Insurance Exams
If there are any suggestions/ errors in our PDFs Feel Free to contact us via this email: admin@exampundit.in
Ultra Practice Bundle PDF
SBI Clerk/ RBI Assist. Mains – Reasoning
1. S is sitting immediately right of V who sits fourth
from the right end.
2. Two persons are sitting between W and V and W
#R – Distance to the left of R is twice the distance to
sits to the right of V.
the left of the person sitting 2nd to the right of R.
3. W is sitting adjacent to the person, who has 13 m
%S - Difference in the distance to the left of S and
distance to his left.
the distance to the left of the person sitting at
extreme right end is 3m
10. Distance between Q and U is 12m.

4. Three persons are sitting between P and T who is


immediate neighbor of W.
5. One person is sitting between the persons, who Answers:

have 9m distance to his left and who have 13m 16) A

distance to his left. 17) B


18) B
19) B

6. Two persons are sitting between R and the person, 20) D

who has 13m distance to his left. Solutions: 16-20


The length of the table is 121 m as given. As we

7. Two persons are sitting between U and the person know, the distance between the adjacent persons is

has 9m distance to his left. in increasing order from 6m, 7m and so on. By
adding 6m to 16m, it sum up to 121m as per
question.
1. Distance between I and F is 10m.
8. Distance to the left of W is 2m less than the
2. Distance between K and G is 2m more than the
distance to the left V.
distance between I and F.
9. One of the persons sitting at the extreme end has
Distance between K and G = 10 + 2= 12m
5m distance to his left.
3. F is sitting 2nd to the left of K.
@U- One of the persons sitting adjacent to U is 16m
Here, we have two cases-
away

Page 781 of 1334


Subscribe The Xpress Video Course & Mock Test Package for Bank & Insurance Exams
If there are any suggestions/ errors in our PDFs Feel Free to contact us via this email: admin@exampundit.in
Ultra Practice Bundle PDF
SBI Clerk/ RBI Assist. Mains – Reasoning
8 16

7 6

4. H is sitting 2nd to the right of D. 5 I 13

5. F is sitting 2nd to the right of H. 4


6. Distance between D and F is 30m.
3
Hence, Case-2 is eliminated because no distance
between D and F sums up to make distance of 30 m 2

14. D lives three floors below I, so D lives on floor


7. E is sitting adjacent to B and the distance between
2.
them is 16m.
15. Eldest child lives two floors above H, who is not
8. G is sitting 3rd to the left of E.
the youngest.
9. Distance between A and G is equal to the distance
16. H doesn’t live adjacent to D’s floor, so 18 years
between H and I.
old child lives on floor 6 and H lives on floor 4.
The final arrangement:
17. More than two children live between eldest child
and E, who is 10 years old, so E lives on floor 1.
18. B lives three floors above C, so B either lives on
10. 2nd eldest child lives on floor 8.
floor 9 or floor 6.
11. Child, who lives on floor 7, is seven years
Case I: B lives on floor 9:
younger than I.
Floors Children Ages
12. I is not 16 years old, so I must be 13 years old.
13. Four children live above the 13 years old child, 9 B
so
8 16
Floors Children Ages
7 6
9

Page 782 of 1334


Subscribe The Xpress Video Course & Mock Test Package for Bank & Insurance Exams
If there are any suggestions/ errors in our PDFs Feel Free to contact us via this email: admin@exampundit.in
Ultra Practice Bundle PDF
SBI Clerk/ RBI Assist. Mains – Reasoning
6 C 18 22. F is elder than B but younger than D, so F is 12
years old, B is 9 years old and D is 14 years old.
5 I 13
The final table is given below:
4 H Floors Children Ages

3 9 B 9

2 D 8 A 16

1 E 10 7 G 6
Case II: B lives on floor 6:
6 C 18
Floors Children Ages
5 I 13
9
4 H 8
8 16
3 F 12
7 6
2 D 14
6 B 18
1 E 10
5 I 13
Answers:
4 H 21. B
22. C
3 C
23. A
2 D 24. C
25. C
1 E 10
Solutions: 21-25
19. F lives immediately below the 8 years old child.
As per the given instructions, first we form the blood
20. Neither A nor B is 8 years old.
relation among the family members-
21. A lives above G, this is not possible in case II, so
1. U is one of the two daughters of R and has 5
case II is rejected.
chocolates less than V. R is not in 1st generation.
2. V sits 12m west of the only son of P, so

Page 783 of 1334


Subscribe The Xpress Video Course & Mock Test Package for Bank & Insurance Exams
If there are any suggestions/ errors in our PDFs Feel Free to contact us via this email: admin@exampundit.in
Ultra Practice Bundle PDF
SBI Clerk/ RBI Assist. Mains – Reasoning
Case 1 – P may have one son and one daughter.
(1a case) – R is a son of P.
(1b case) – R is a daughter of P.
Case 2 – P may have only one son.
Case 1a: R is male member with one sister.
Let the daughter of P is denoted as 1.
Family Tree:
3. Q is not in 3rd generation. Q and R have same
gender.
4. S is not in the 3rd generation but has 2nd highest
number of chocolates Mother of S sits 3m west of
the person, who is 10m south of S, who is not
married.
5. T is not in the same generation as P. T is not the
Case 1b – R is the daughter of P having one brother. son of P. T is not a female member.
Family Tree: By the above conditions, Case 1a and case 2 are
eliminated.
Family Tree:

Case 2: R is the only son of P.


Family Tree:
Now, we solve the other part of the question means
direction sense and number of chocolates they have.
6. V sits 12m west to the only son of P means V sits
12m west of S.

Page 784 of 1334


Subscribe The Xpress Video Course & Mock Test Package for Bank & Insurance Exams
If there are any suggestions/ errors in our PDFs Feel Free to contact us via this email: admin@exampundit.in
Ultra Practice Bundle PDF
SBI Clerk/ RBI Assist. Mains – Reasoning
7. Mother of S sits 3m west to the person, who is 11. T has more chocolates than P but less than U.
10m south of S, who is not married. 12. U doesn’t have highest number of chocolates.
Mother of S is Q. Number of Chocolates:-
8. S is not in the 3rd generation but has 2nd highest V>S>U>T>P>Q>R
number of chocolates. 12. The person with 3rd highest number of
9. P sits 4m north to the person who has 2nd lowest chocolates sits 5m south west of T.
number of chocolates. Q has more chocolates than 13. Remaining position to the 10m south of S is
only R allotted to R.
Distribution of number of chocolates- 14. One of the persons sits exactly in the west of P.
___ > S > ____ > ____ > _____ > Q > R It means T sits west of P.
Direction Diagram: Distance between T and P can be calculated by
Pythagorean Theorem-
(Hypotenuse) 2 = (Height) 2 + (Base) 2
(5) 2= (4) 2 + (Base) 2
Base = 3m
Distance between T and P = (Distance between V
and S) – (Base + distance between Q and R)
= 12m – 6m = 6m
10. The person who sits 10m north of U is U’s sister.
Direction Diagram:

Page 785 of 1334


Subscribe The Xpress Video Course & Mock Test Package for Bank & Insurance Exams
If there are any suggestions/ errors in our PDFs Feel Free to contact us via this email: admin@exampundit.in
Ultra Practice Bundle PDF
SBI Clerk/ RBI Assist. Mains – Reasoning
Row 2 T 36 31 / 37

4. R is 27 years old, faces south and faces the person


who sits to the immediate right of D.
5. P sits 2nd to the right of R
6. 3rd youngest person sits immediate right of 37 year
old person. (3rd youngest person means 27 years old
i.e. R)
7. Q sits immediate right of the person, who is 31

V>S>U>T>P>Q>R years old.

Answers: Case I:

26. D Row P
27. B 1 Q (31) 36 R(27) T(37)
28. D
Row
29. A
2 B D
30. B
Case 2(a):
Solutions: 26-30
1. As 36 years old faces the person, who sits 2nd to Row
the right of B. 1 P R(27) B(37)
2. T sits 2nd to the left of 5th eldest person. (5th eldest
Row
person is 36 years old.)
2 T D(36) 31 Q
3. Two persons sit between T and the person, whose
Case 2(b):
age is a prime number.
Case 1- Row 1 P R(27) (37) B

Row 1 31/37 36 T Row 2 T D 36 31 Q

Row 2 B 8. C sits immediate right of E and faces north.


9. E is not 31 years old. A faces north.
Case 2:-
In case 2a and 2b, C, E and A doesn’t face North
Row 1 B
direction. So these cases are eliminated.

Page 786 of 1334


Subscribe The Xpress Video Course & Mock Test Package for Bank & Insurance Exams
If there are any suggestions/ errors in our PDFs Feel Free to contact us via this email: admin@exampundit.in
Ultra Practice Bundle PDF
SBI Clerk/ RBI Assist. Mains – Reasoning
10. A faces the person, who sits immediate left of
2nd eldest person (42 years old).
11. D is the second youngest person.
Case I: After that the person who sits immediate right of E,
Ro Q(4 P(3 R(2 T(3 turn right and walks for 3m to turn left and walks
w1 2) 1) 36 7) 7) another 15m. After that he again turns left and covers
9m to turn left and walks 5m and meets the person,
Ro D(2
who is elder than him. He exchanges his seat with
w2 B A 4) E C
him.
12. Remaining S is 36 years old. A is elder than Q, it means A must be 45 years old.

After the arrangement:


After 2nd replacement:
E, who is 8 years elder than D, moves 18m straight,
then turns left and move another 12m to turn left
again. He walks another 8m and meets the person,
whose age is less than 36 years. He exchanges his
seat with that person.
Age of E becomes 24 + 8 = 32 years old. After that, C, who is not the youngest person, turns
back and moves 4m to turn right. He walks 18m and
turn to walk another 14m. He meets a person with
whom he exchanges his seat.
C is not the youngest among all, it means C is 39
years old and remaining B becomes 20 years old.
After 1st replacement:

Page 787 of 1334


Subscribe The Xpress Video Course & Mock Test Package for Bank & Insurance Exams
If there are any suggestions/ errors in our PDFs Feel Free to contact us via this email: admin@exampundit.in
Ultra Practice Bundle PDF
SBI Clerk/ RBI Assist. Mains – Reasoning
So, P must have 6 pens and R must have 8 pens.
Also, then T can have 7 or 9 pens. So, we have two
possibilities,

After 3rd replacement:


Final arrangement

Or

Answers:
31. C
32. E
33. E
34. C
35. B
Solutions 31-35
1. R moves 5m in the north and walks 15m in either
west or east direction.
2. P has 2 pens lesser than R but more than that of Q.
3. P moves 10m in north and then 10m in the east.
Answers: 36-40
So, P can have either 6 or 7 pens.
36. C
4. T walks 8m in south and then 5m in east or west
37. C
direction.
38. A
5. If P has 7 pens then T must have 8 pens but then
39. C
S cannot be 10m away in the west of T.
40. B
Page 788 of 1334
Subscribe The Xpress Video Course & Mock Test Package for Bank & Insurance Exams
If there are any suggestions/ errors in our PDFs Feel Free to contact us via this email: admin@exampundit.in
Ultra Practice Bundle PDF
SBI Clerk/ RBI Assist. Mains – Reasoning
Solutions: 36-40
1. V, who secures better marks than S and U and is
6m to the west of U.
2. P secures better marks than only one, who is 2m
to the east of him.
3. R is 6m west of T. T secures better marks than R
but not S.
So, R must secure the worst marks.
4. T is 7m north of one, who secures better marks
than three persons only.
So, U must be the one, who secures better marks than
three persons only. Answers: 36-40
5. Number of persons, who score more marks than S 36. C
are not equal to the number of the persons, who score 37. C
less marks than S. 38. A
6. S is 6m to the east of one, who secures highest 39. C
marks. 40. B
Therefore, Q secures highest marks. So, we have, Solutions: 36-40
Q>V>S>U>T>P>R 1. L is sitting 12m north to the person, who has the
second lowest number of boxes.
2. R is sitting 25m north to S, who has the highest
number of boxes.
3. L has more boxes than O, who has more boxes
than Q.
4. Q has more boxes than at least two persons.
5. R is sitting 6m east to Q, who is sitting 11m north
to P.
6. L is sitting north west to P and has less number of
boxes than P.

Page 789 of 1334


Subscribe The Xpress Video Course & Mock Test Package for Bank & Insurance Exams
If there are any suggestions/ errors in our PDFs Feel Free to contact us via this email: admin@exampundit.in
Ultra Practice Bundle PDF
SBI Clerk/ RBI Assist. Mains – Reasoning
7. O is sitting 8m west to P. N is sitting 6m east to 1. Person, who has Rexton, is sitting 3rd from the left
M. end.
8. N has more boxes than O and is sitting 10m in the 2. One person is sitting between B and S and distance
south of O, so M must have the 2nd lowest number of between both of them is 20m.
boxes. 3. Two persons sit between the persons, who have
The final arrangement is as follows: Rexton and Baleno.
4. Four persons are sitting between S and the person,
who has Baleno.
5. M is sitting 4th to the right of B.
The following are the possible cases from the left
end.
Case 1:

9. P has lesser boxes than one of the persons sitting Case 2a:
in the south west of P.
10. P doesn’t have the lowest number of boxes.
The final ranking arrangement is as follows:
S>N>P>L>O>Q>M>R
Case 2b:
Answers:
41. b
42. d
43. a 6. Person, who has Swift is sitting at the right end.
44. c 7. Five persons are sitting between R, who has
45. a Porsche and the person, who has Swift.
Solutions 41-45 8. Five persons are sitting between R and D.
Page 790 of 1334
Subscribe The Xpress Video Course & Mock Test Package for Bank & Insurance Exams
If there are any suggestions/ errors in our PDFs Feel Free to contact us via this email: admin@exampundit.in
Ultra Practice Bundle PDF
SBI Clerk/ RBI Assist. Mains – Reasoning
9. Two persons are sitting between K and the person, Case 5 is not possible because Number of persons
who has Audi. sitting between M and R is a prime number.
10. K is sitting 2nd to the left of D and distance The final arrangement can be the combination of
between them is 20m. case 1 and case 4.
The following are the possible cases from the right Final Arrangement:-
end.
Case 3:

Answers:
46. c
47. a
Case 4:
48. b
49. c
50. c
Solutions 46-50
1. As three candidates score more than S, who stands
11. Two persons are sitting between M and the 6m north of Q.
person, who has Audi. 2. The candidate who likes Violet and sits 7m east of
12. Persons sitting between D and the person, who S, scores less than S.
has Baleno is less than 10, so case 2a and 2b are not S > Violet
possible. 3. Candidate who scores the 2nd lowest scores 97
13. Distance between M and K is 50m. marks, so
By combining the cases from the left end and the
right end, we only combine case 1 and case 4 to
satisfy every condition.
13. Persons sitting in the left of K are equal to the
persons sitting in the right of the person, who has
Bolero.
There are three cases for the person who likes Violet
14. Distance between B and M is 30m.
color.
Page 791 of 1334
Subscribe The Xpress Video Course & Mock Test Package for Bank & Insurance Exams
If there are any suggestions/ errors in our PDFs Feel Free to contact us via this email: admin@exampundit.in
Ultra Practice Bundle PDF
SBI Clerk/ RBI Assist. Mains – Reasoning
Case I: Q > P/T > T/P > S > __ > (97) > (Violet)
__ > __ > __ > S > (Violet) > (97) > __ 8. U stands 9m south of R, who scores 105 marks.
Case II: By this, R must scores third lowest number.
__ > __ > __ > S > __ > (97)(Violet) > __ 9. Neither U nor V likes Violet, so case II and case
Case III: III are rejected.
__ > __ > __ > S > __ > (97) > (Violet) 10. V scores 3 marks less than U.
4. The highest scorer stands 8m east of T who likes V secures 97 -3 = 94 marks.
Red. Case I:
5. P scores more than S and sits 6m south of T. Q > P/T > T/P > S > R(Violet) > U(97) > V(94)
6. More than two candidates score less than T. R
doesn’t score the highest marks.
It means, T scores more than S to satisfy the above
condition
P / T > S.
T cannot score highest marks as per the rule 4.
7. Difference between the marks of S and the one
who likes Violet is less than the difference of marks
of S and Q, so Q scores the highest marks.
11. The candidate, who stands at extreme east, likes
Green and sits at a distance of 6m from the person,
who likes Blue.
To make Green color extreme east, the remaining V
must like Green color and sits at 6m east of U.
12. Person who stands at the extreme south likes
neither Magenta nor Yellow, so P likes Pink.
Case I: 13. Candidate who likes Yellow scores more than the
Q > P/T > T/P > S > (Violet) > (97) > __ one, who lives Magenta, so Q likes Yellow and S
Case II: likes Magenta.
Q > P/T > T/P > S > __ > (97) (Violet) > __ The final ranking and direction diagram is given
Case III: below:

Page 792 of 1334


Subscribe The Xpress Video Course & Mock Test Package for Bank & Insurance Exams
If there are any suggestions/ errors in our PDFs Feel Free to contact us via this email: admin@exampundit.in
Ultra Practice Bundle PDF
SBI Clerk/ RBI Assist. Mains – Reasoning
Q(Yellow)>P(Pink)/T(Red) >T(Red)/P(Pink)
>S(Magenta) >R(105)(Violet)>U(97)(Blue)>
V(94)(Green)

Download Seating Arrangement Practice Questions PDF

Get More Reasoning Practice Questions PDF


Seating Unknown Number of Person
(Direction 1-5): Study the following information likes White and the person who likes Orange. Less than
carefully and answer the given questions: three persons sit between the person, who likes Orange
Certain number of persons sit in a square table facing and the person who likes Red but at least one person will
inside the table. Not Less than eleven persons sit in a table sit between these two persons. L sits immediate left of W
and not more than two persons sit on the same side of the and sits alone in the side
table. Also, Atleast two of the corner of the table is vacant. 1) How many persons are sitting in a table?
M and S are immediate neighbours and M likes Black. A) 14
Both of them sit on the same side of the table. W who likes B) 11
Green sits diagonally opposite to the person who likes C) 12
Red. W is neither an immediate neighbour of M nor S. G D) 15
sits second to the right of S. Three persons sit between G E) 17
and R who likes Blue. P faces A who sits to the immediate 2) What is the square of the total number of persons
left of R but does not sit at the corner of the table. Person sit between W and the person likes Red?
who likes Orange faces the person who likes Grey. P A) 25
neither likes Grey nor Red. L who likes White faces the B) 36
person who likes Yellow but they do not sit at the corner C) 16
of the table. Two persons sit between the person whom D) 49
Page 793 of 1334
Subscribe The Xpress Video Course & Mock Test Package for Bank & Insurance Exams
If there are any suggestions/ errors in our PDFs Feel Free to contact us via this email: admin@exampundit.in
Ultra Practice Bundle PDF
SBI Clerk/ RBI Assist. Mains – Reasoning
E) None of these E who sits at the corner of the table faces the person whose
3)Which of the following statement is/are true? age is 50. K whose age is 40 sits third right to the person
A) P faces the person who likes Grey whose age is 50. Person whose age is 29 faces K. Only
B) R does not like Blue one person sits between D whose age is 30 and K but not
C) S sits fourth to the right of the person who likes Green on the same side of side of the table. T whose age is 32
D) Both A and B sits to the immediate left of G who sits at the corner of the
E) Both A and C table. No one sits between D and B. Person whose age is
4) Who among the following sits tenth to the right of 8 and the person whose age is 36 are immediate
R? neighbours of each other’s. Neither E nor L are of 8years.
A) Person likes White X whose age is a prime number sits second to the left of L
B) Person likes Green on the same side of the table and second to the right of E.
C) Person likes Grey Only one person sits between L and V whose age is 25 and
D) None does not faces S. Person whose age is 15 sits fourth to the
E) Person likes Orange right of the person whose age is 36. Less than three person
5) Who among the following sits exactly between M sits between B and X.
and L? 6) What will be the square root of product of ages of
A) Person likes Orange both K and L?
B) Person likes Grey A) 10√5
C) Person likes Blue B) 10√6
D) Both A and B C) 10√7
E) Both A and C D) 10√3
(Direction 6-10): Study the following information E) None of these
carefully and answer the given questions: 7) Which of the following statement is/are true?
Certain number of persons sit in a rectangular table facing A) Age of X is 32
inside the table. Less than sixteen persons sit in a table. B) More than four persons sit between E and K
More than three persons sit on the longer side of the table C) B faces the person whose age is 25
and less than three but at least one person sit on the shorter D) Both A and B
side of the table. Also, Atleast two of the corner of the E) None
table is not vacant. 8) How many persons are sitting in a row?

Page 794 of 1334


Subscribe The Xpress Video Course & Mock Test Package for Bank & Insurance Exams
If there are any suggestions/ errors in our PDFs Feel Free to contact us via this email: admin@exampundit.in
Ultra Practice Bundle PDF
SBI Clerk/ RBI Assist. Mains – Reasoning
A)15 planted Peepal tree. L sits second to the left of Y. Person
B) 16 who planted Banyan tree sits two places away from the
C) 13 person who is planted Palm tree who sits two places away
D) 14 from the person planted Peepal tree. Q does not plant
E) 17 Banyan tree. G who, planted Oak tree sits to the
9) Who among the following sits sixth to the right of immediate right of the person who is planted Mango tree.
D? P sits exactly between Q and D who is planted Banyan
A) Person whose age is 15 tree. Person who planted Pine tree and the person who
B) Person who sits at the corner of the table planted Elm tree are immediate neighbours of each other.
C) Person who sits third to the left of T Neither T nor S planted Pine tree. S who planted Neem
D) Both A and C tree sits between D and the person who planted Palm tree.
E) Both A and B Number of persons sits to the right of Y is one more than
10) What is the cube of the total number of persons sit the number of persons sits to the left of F.
between E and G when counted clockwise from E? 11) Who among the following sits exactly between L
A) 343 and person who planted Pine tree?
B) 125 A) Person who planted Mango tree
C) 126 B) Person who planted Oak tree
D) 261 C) G
E) 216 D) D
E) Both B and C
(Direction 11-15): Study the following information 2) What is the cube of the total number of persons sit
carefully and answer the given questions: between T and Y?
Certain number of persons are sitting in a linear row A) 521
facing north. Less than eighteen persons sit in a row. B) 512
F sits at the end of the row and planted a Maple tree. Three C) 343
persons sit between Q and F. Person who planted Mango D) 216
tree sits third to the right of Q. T sits second to the left of E) 125
the person who planted Mango tree. Six person sits 13) Who among the following sits fifth from the right
between the person who planted Mango tree and Y who end?

Page 795 of 1334


Subscribe The Xpress Video Course & Mock Test Package for Bank & Insurance Exams
If there are any suggestions/ errors in our PDFs Feel Free to contact us via this email: admin@exampundit.in
Ultra Practice Bundle PDF
SBI Clerk/ RBI Assist. Mains – Reasoning
A) Person who planted Neem tree
B) Person who sits between D and L
C) Person who sits third to the left of Y
D) All A, B and C
E) Person who sits second to the right of G
14) How many persons are sitting in a table?
A) Square of three
B) 14
C) Square of four
D) 13
Three persons sit between P and Q who likes One plus and
E) 15
they are sitting at point 3 and point 4. R sits to the
15) Which of the following statement is/are true?
immediate left of Q. G sits third to left of A who likes
A) Six persons sit between P and F
Realme and who sits sixth to the right of Q. K and A are
B) Q T are not an immediate neighbours
immediate neighbours but not on the same side. C who sits
C) D sits exactly between L and P
third to the right of K likes Vivo but both does not sit at
D) Both A and B
the corner of table. H sits second to the right of C. R and
E) Both A and C
H are not sitting on the same side of the table. W who likes
(Direction 16-20): Study the following information
Apple sits third to the right of the person who likes
carefully and answer the given questions:
Realme. Two persons sit between H and R who likes Jio.
Person who likes Poco sits third to the right of the person
Certain number of persons are sitting in a Pentagon table
who likes Nokia. Six persons sit between the person who
representing the corner as 12345 as per diagram given. All
likes Oppo and the person who likes MI when counted
are facing inside the table. Less than eighteen persons sit
from the left of the person who likes MI.
in a table. Number of persons sitting on each of the longer
16) Which of the following statement is not correct?
sides are equal and same will be applied for the shorter
A) Q-One Plus
sides as well. Corners also seated by persons.
B) A- Realme
C) R-Jio
D) G-Oppo
E) W-Apple
Page 796 of 1334
Subscribe The Xpress Video Course & Mock Test Package for Bank & Insurance Exams
If there are any suggestions/ errors in our PDFs Feel Free to contact us via this email: admin@exampundit.in
Ultra Practice Bundle PDF
SBI Clerk/ RBI Assist. Mains – Reasoning
17) How many persons are sitting in a table? Certain number of persons are sitting on a W shaped table
A) 4×3 facing inside the table. Less than eighteen persons sit in a
B) 4×4 table.
C) 3×5 G who likes Kiwi sits at the centre of the table. P and B
D) 8×2 are an immediate neighbours of G. Except G no one sits
E) Both B and D at the corner of the table. Both P and B are sitting alone
18) What is the position of C with respect to P? on a side of the table. Only one person sits between O and
A) Sixth to the right P but not on the same side of the table. Two persons sit
B) Seventh to the right between B and M who likes Mango but on different sides
C) Tenth to the left of the table. Person, who likes Mango is not an immediate
D) Both A and B neighbour of the person, who likes Grapes. H sits second
E) Both A and C to the left of the person who likes Mango. R who is the
19) Which of the following statement is/are not true? neighbour of H sits two places away from the person who
A) G does not like MI likes Apple. Person who likes Papaya faces the person
B) C likes Oppo who likes Apple. Only one person sits between the person
C) A does not sit at the corner of the table who likes Grapes and the person who likes Papaya. L who
D) Both A and B likes Malta and the person who likes Litchi are an
E) None is true immediate neighbours of each other. Person who likes
20) What will be the square of total number of persons Banana sits to the immediate left of the person who likes
sit between K and Q when counted from the left of Q? Orange but on the different side of the table. R who likes
A) 64 Guava and the person who likes Orange are an immediate
B) 49 neighbours. S who likes Dragon Fruit sits three places
C) 25 away from the person who likes Litchi, who sits alone on
D) 100 one of the sides of the table.
E) 121 21) How many persons are sitting in a table?
(Direction 21-25): Study the following information A) 2² + 3²
carefully and answer the given questions: B) 2³ + 2²
C) 2² + 3³
D) 3² + 2³

Page 797 of 1334


Subscribe The Xpress Video Course & Mock Test Package for Bank & Insurance Exams
If there are any suggestions/ errors in our PDFs Feel Free to contact us via this email: admin@exampundit.in
Ultra Practice Bundle PDF
SBI Clerk/ RBI Assist. Mains – Reasoning
E) 5 + 3² (Direction 26-30): Study the following information
22) Four of the five are alike in a certain way and hence carefully and answer the given questions:
forms a group. Which of the following does not belong Certain number of persons are sitting on a M shaped table
to that group? facing towards the center. Less than Eighteen persons sit
A) F faces the person who likes Papaya on a table. Number of persons on each longer side are
B) L faces the person who likes Orange equal and same will be applied for the shorter sides as
C) R faces the person who likes Litchi well.
D) S faces the person who likes Mango A who likes Audi sits at the centre of the table. K sits two
E) O faces the person who likes Guava places away from A and sits at the corner of the table. Y
23) What is the Cube of the total number of persons sit sits to the immediate left of A and likes Honda. Two
between O and M? persons sit between K and a person who like Jaguar. C sits
A) 216 to the immediate left of the person who likes Jaguar and
B) 225 faces the person who likes Alto. Two persons sit between
C) 125 A and E who likes Toyota. E do not sit on that side of the
D) 64 table on which the person who likes Jaguar sits. No one sit
E) 343 to the right of E on the same side of the table. I who likes
24) Who among the following sits third to the right of BMW faces the person who sits to the immediate right of
O? the person who likes Hyundai. Person who likes Hyundai
A) Person who likes litchi and a person who likes Alto do not sit on the same side of
B) L the table. D faces the person who sits between the person
C) Person who likes Kiwi who likes Alto and S. Either K or S likes Maruti. Person
D) B who likes Ertiga does not share the side with the person
E) None of these who likes Toyota.
25) Which of the following statement is/are not true? 26) Who among the following sits fifth to the right of
A) B does not like Banana E?
B) S and the person who likes Grapes are neighbours A) No one
C) T faces the person who likes Mango B) Person likes Ertiga
D) Both A and B C) A
E) Both A and C D) Person likes Hyundai

Page 798 of 1334


Subscribe The Xpress Video Course & Mock Test Package for Bank & Insurance Exams
If there are any suggestions/ errors in our PDFs Feel Free to contact us via this email: admin@exampundit.in
Ultra Practice Bundle PDF
SBI Clerk/ RBI Assist. Mains – Reasoning
E) D (Direction 31-35): Study the following information
27) How many persons are sitting in a table? carefully and answer the given questions:
A) √144 Certain number of persons sit in a linear row. All are
B) √196 facing in the north direction. Less than eighteen persons
C) √169 sit in a row.
D) √225 L sits second to the right of N. Two persons sit between L
E) √121 and K who likes Avataar. S who sits at the right end of the
28) What is the square of the total number of persons row sits three places away from the person who likes
sit between I and D? Dangal. Only one person sits between the person who
A) 9² likes 1920 and the person who likes Swadesh and the one,
B) 8² who likes Swadesh will sit to the left of L. T sits to the
C) 7² immediate left of the person who likes Dangal. P who
D) 6² likes Sultan sits exactly between M and T but not
E) 5² immediate. No one sit between the person who likes
29) Which of the following statement is/are true? Avataar and M. Q sits third to the right of the person who
A) S does not like Maruti likes Sultan. Person who likes 1917 sits exactly between
B) K likes Ertiga the person who likes 1920 and T. M neither likes 1920 nor
C) T faces the person likes Hyundai likes Joker. Person who likes Joker sits fourth to the left
D) Both A and B of the person who likes Lion. No one sits to the right of
E) Both B and C the person who likes Lion. Number of persons sits to the
left of the person who likes 1920 is one less than the
30) Who among the following sits exactly between C number of persons sits to the right of the person who likes
and the person who likes Alto? Dangal.
A) Person who likes Ertiga 31) Which of the following statement is/are not true?
B) K A) K sits exactly between N and the person likes Joker
C) Y B) M likes 1920
D) Person who likes Audi C) P sits third to the left of the person who likes Dangal
E) Person who likes Honda D) Both A and B
E) Both A and C

Page 799 of 1334


Subscribe The Xpress Video Course & Mock Test Package for Bank & Insurance Exams
If there are any suggestions/ errors in our PDFs Feel Free to contact us via this email: admin@exampundit.in
Ultra Practice Bundle PDF
SBI Clerk/ RBI Assist. Mains – Reasoning
32) What is the cube of the total number of persons sit Certain number of persons sit in a E shaped table. All are
between N and S? facing in the north direction. Less than Eighteen persons
A) 4096 sit in a table. Each person sits only at the horizontal sides
B) 3375 of the table. Number of persons sit on the middle side of
C) 1278 the table is less than the number of persons sit on the upper
D) 1331 and lower side of the table.
E) 2197 H sits fourth to the right of A who was born in June and
33) How many persons are sitting in a table? both are not sitting on the longer side of the table. B who
A) 5² - 2³ was born in October sits to the south of F who is not an
B) 3² + 6 immediate neighbour of H. Only one person sits between
C) 2³ + 2² B and D but not on the same side of the table. D sits at the
D) 3³ - 2² left end of the lower side of the table. E sits to the east of
E) 4² - 2² D and born in May. B does not share the side with A.
34) Who among the following sits eighth to the left of Three persons sit between the person who was born in
P? October and the person who was born in August but on
A) L the different sides. Three persons sit between D and C who
B) M was born in March. E sits to the south of F and to the
C) Person who likes Swadesh south-east of B. Person who was born in August and the
D) K person who was born in April are sitting on the upper side
E) Person who likes Avatar of the table. G who was born in January sits to the north-
35) Which among the following pair sits at the end of east of the person who was born in July who sits neither
the row? at the upper side nor at the middle side of the table.
A) L and the person who likes Lion 36) Who among the following sits to the south-west of
B) S and the person who likes Swadesh G?
C) N and the person who likes Lion A) Person who was born in July
D) Both A and B B) D
E) Both B and C C) Person who was born in October
(Direction 36-40): Study the following information D) Both A and B
carefully and answer the given questions: E) Both A and C

Page 800 of 1334


Subscribe The Xpress Video Course & Mock Test Package for Bank & Insurance Exams
If there are any suggestions/ errors in our PDFs Feel Free to contact us via this email: admin@exampundit.in
Ultra Practice Bundle PDF
SBI Clerk/ RBI Assist. Mains – Reasoning
37) How many persons are sitting in a table? (Direction 41-45): Study the following information
A) 3² + √16 carefully and answer the given questions:
B) 2³ + 6 Certain number of persons sit in a circular table facing
C) 7 + 3² inside the table. Less than eighteen persons sit in a table.
D) 2⁴ - 2² Not more than two known persons sit adjacent to each
E) None of these other.
38) Which of the following statement is/are not true? R sits second to the left of E and likes Pasta. Only one
A) E sits to the south of G person sit between the person likes Idli and the person
B) B sits to the north-west of the person who was born in likes Pizza who sits four places away from R. G likes
July Maggie. L sits second to the left of the person who likes
C) D sits to the south-west of H Maggie who sits three places away from the person who
D) Both A and B likes Idli. L who is not an immediate neighbour of R sits
E) Both A and C to the left of E who likes Idli. Only one person sits
39) What is the square of the total number of persons between L and the person who likes Dosa. Person who
sit between H and D? likes Samosa sits second to the left of the person who likes
A) 64 Dosa. Person who likes Samosa is not an immediate
B) 36 neighbour of E. Q sits three places away from K. K neither
C) 25 likes Pizza nor likes Dosa. Seven persons sit between the
D) 49 person who likes Maggie and the person who likes
E) 81 Samosa when counted anticlockwise from the person who
40) What is the square root of the total number of likes Maggie. Q sits seventh to the left of L who likes
persons sit between C and the person who was born in Burger. P sits fifth to the right of Q.
April? 41) Who among the following sits seventh to the right
A) √12 of R?
B) √11 A) Person who likes Pizza
C) √7 B) Person who likes Dosa
D) √5 C) Person who likes Samosa
E) √9 D) Person who likes Idli
E) Person who likes Maggie

Page 801 of 1334


Subscribe The Xpress Video Course & Mock Test Package for Bank & Insurance Exams
If there are any suggestions/ errors in our PDFs Feel Free to contact us via this email: admin@exampundit.in
Ultra Practice Bundle PDF
SBI Clerk/ RBI Assist. Mains – Reasoning
42)Which of the following statement is/are true? (Direction 41-45): Study the following information
A) G sits to the immediate right of the person who likes carefully and answer the given questions:
Pasta Certain number of persons sit on a L shaped table facing
B) Four persons sit between E and L inside the table. Less than Eighteen persons sit in a table.
C) P does not like Dosa Persons sit on the vertical side are north-south of each
D) Both A and B other and the persons sit on the horizontal side are east-
E) Both A and C west of each other.
43) How many persons sit between K and the person P sits to the north of G who works in RBI but not
who likes Burger when counted anticlockwise from the immediate north. Less than four persons but at least one
person who likes Burger? person sits to the north of P. Three persons sit between N
A) 3² and T who works in BOI and also sits to the immediate
B) 2² south of P. H sits at the extreme west of the table. G sits
C) 3³ - 2³ second to the left of H. G and N are not an immediate
D) 2³ neighbours of each other. M sits to the west of R but not
E) None of these immediate west. R sits second from the right end of the
44) What is cube of the number of persons sit between table. Three persons sit between H and R who works in
P and E? SBI. More than eight persons sit between N and M who
A) 215 sits sixth to the right of the person who works in BOI.
B) 125 Person who works in UCO sits second to the left of the
C) 343 person who works in PNB who sits fourth from the right
D) 216 end. Person who works in UBI sits three places away to
E) 261 the north of the person who works in OBC who also sits
45) How many persons are sitting in a table? three places away to the north of the person who works in
A) 3 × 4 RBI.
B) 7 × 2 46) Which of the following statement is/are not true?
C) 4 × 2 A) N sits at the top of the table.
D) 5 × 3 B) M works in SBI
E) 2 × 8 C) Less than four persons sit between T and the person
who works in PNB

Page 802 of 1334


Subscribe The Xpress Video Course & Mock Test Package for Bank & Insurance Exams
If there are any suggestions/ errors in our PDFs Feel Free to contact us via this email: admin@exampundit.in
Ultra Practice Bundle PDF
SBI Clerk/ RBI Assist. Mains – Reasoning
D) Both A and B E) 125
E) Both B and C 49) How many persons are sitting in a table?
47) Who among the following sits to the north-west of A) 12
R? B) 13
A) Person who works in RBI C) 14
B) Person who works in PNB D) 15
C) Person who works in OBC E) 16
D) Person who works in UCO 50) Which of the following statement is/are true?
E) Both A and C A) H does not work in PNB
48) What is the cube of the total number of persons sit B) G sits to the north-east of M
between P and H? C) P does not work in OBC
A) 27 D) None is true
B) 343 E) Both A and C
C) 216
D) 64

Seating Unknown Number of Person – Answer and Explanation


Answers (1-5):
1) C
2) A
3) E
4) B
5) E
Solutions (1-5):
STEP I: M sits to the immediate neighbour of S and likes CASE II:
Black. Both of them sit on the same side of the table.
As per these statements, there will be two possible cases
and the arrangement will look like this:
CASE I:

Page 803 of 1334


Subscribe The Xpress Video Course & Mock Test Package for Bank & Insurance Exams
If there are any suggestions/ errors in our PDFs Feel Free to contact us via this email: admin@exampundit.in
Ultra Practice Bundle PDF
SBI Clerk/ RBI Assist. Mains – Reasoning

STEP II: W who likes Green sits diagonally opposite to


the person who likes Red. W is neither an immediate
CASE II (A):
neighbour of M nor S.
As per these statements, CASE I and CASE II will get
split into one more case each and the arrangement will
look like this:
CASE I:

STEP III: G sits second to the right of S. Three persons


sit between G and R who likes Blue. P faces A who sits to
the immediate left of R but does not sit at the corner of the
table.
As per these statements, CASE II and CASE II (A) will
CASE I (A):
get eliminated and we will continue with CASE I and
CASE I (A) and the arrangement will look like this:
CASE I:

CASE II:

Page 804 of 1334


Subscribe The Xpress Video Course & Mock Test Package for Bank & Insurance Exams
If there are any suggestions/ errors in our PDFs Feel Free to contact us via this email: admin@exampundit.in
Ultra Practice Bundle PDF
SBI Clerk/ RBI Assist. Mains – Reasoning

STEP IV: Person who likes Orange faces the person who
likes Grey. P neither likes Grey nor Red.
CASE I (A):
As per this statement, the arrangement will look like this:
CASE I:

CASE II:

CASE I (A):

CASE II (A):

Page 805 of 1334


Subscribe The Xpress Video Course & Mock Test Package for Bank & Insurance Exams
If there are any suggestions/ errors in our PDFs Feel Free to contact us via this email: admin@exampundit.in
Ultra Practice Bundle PDF
SBI Clerk/ RBI Assist. Mains – Reasoning
STEP V: L who likes White faces the person who likes
Yellow but they do not sit at the corner of the table.
As per this statement, the arrangement will look like this:
CASE I:

CASE I (A):

CASE I (A):

Answers (6-10):
6) B
7) E
8) A
9) D
STEP VI: Two persons sit between the person whom 10) E
likes White and the person who likes Orange. Less than Solutions (6-10):
three persons sit between the person, who likes Orange STEP I: E who sits at the corner of the table faces the
and the person who likes Red but at least one person will person whose age is 50. K whose age is 40 sits third to the
sit between these two persons. right of the person whose age is 50.
As per these statements, CASE I (A) will get eliminated As per this statement, the arrangement will look like this:
and the final arrangement will look like this:
CASE I:

Page 806 of 1334


Subscribe The Xpress Video Course & Mock Test Package for Bank & Insurance Exams
If there are any suggestions/ errors in our PDFs Feel Free to contact us via this email: admin@exampundit.in
Ultra Practice Bundle PDF
SBI Clerk/ RBI Assist. Mains – Reasoning

STEP II: Person whose age is 29 faces K. Only one CASE III:
person sits between D whose age is 30 and K but not on
the same side of side of the table.
As per this statement, the arrangement will look like this:

CASE IV:

STEP III: T whose age is 32 sits to the immediate left of


G who sits at the corner of the table. No one sits between
D and B.
As per these statements, there will be three more possible STEP IV: X whose age is a prime number sits second to
cases and the arrangement will look like this: the left of L on the same side of the table and second to
CASE I: the right of E.
As per this statement, CASE III and CASE IV will get
eliminated and we will continue with CASE I and CASE
II and the arrangement will look like this:
CASE I:

CASE II:

Page 807 of 1334


Subscribe The Xpress Video Course & Mock Test Package for Bank & Insurance Exams
If there are any suggestions/ errors in our PDFs Feel Free to contact us via this email: admin@exampundit.in
Ultra Practice Bundle PDF
SBI Clerk/ RBI Assist. Mains – Reasoning

CASE II:
CASE II:

CASE III:

STEP VI: Person whose age is 8 and the person whose


age is 36 are immediate neighbours of each other’s.
Neither E nor L are of 8years. Person whose age is 15 sits
fourth to the right of the person whose age is 36. Less than
three person sits between B and X.

CASE IV: As per these statements, CASE II will get eliminated and
the final arrangement will look like this:
CASE I:

STEP V: Only one person sits between L and V whose


age is 25 and does not faces S.
As per this statement, the arrangement will look like this: CASE II:
CASE I:

Page 808 of 1334


Subscribe The Xpress Video Course & Mock Test Package for Bank & Insurance Exams
If there are any suggestions/ errors in our PDFs Feel Free to contact us via this email: admin@exampundit.in
Ultra Practice Bundle PDF
SBI Clerk/ RBI Assist. Mains – Reasoning
As per these statements, the arrangement will look like
this:
CASE I:

Answers (11-15): CASE II:


11) E
12) B
13) D
14) C
15) A STEP III: Six person sits between the person who planted
Mango tree and Y who planted Peepal tree. L sits second
Solutions (11-15):
STEP I: F sits at the end of the row and planted a Maple to the left of Y.
As per these statements, the arrangement will look like
tree. Three persons sit between Q and F.
this:
As per this statement, there will be two possible cases and
CASE I:
the arrangement will look like this:
CASE I:

CASE II:

CASE II:

STEP IV: Person who planted Banyan tree sits two places
away from the person who is planted Palm tree who sits
two places away from the person planted Peepal tree. Q
STEP II: Person who planted Mango tree sits third to the does not plant Banyan tree.
right of Q. T sits second to the left of the person who As per these statements, the arrangement will look like
planted Mango tree. this:
CASE I:
Page 809 of 1334
Subscribe The Xpress Video Course & Mock Test Package for Bank & Insurance Exams
If there are any suggestions/ errors in our PDFs Feel Free to contact us via this email: admin@exampundit.in
Ultra Practice Bundle PDF
SBI Clerk/ RBI Assist. Mains – Reasoning
As per these statements, the final arrangement will look
like this:
CASE I:
CASE II:

Answers (16-20):
STEP V: G who, planted Oak tree sits to the immediate 16) D
right of the person who is planted Mango tree. P sits 17) E
exactly between Q and D who is planted Banyan tree. 18) E
As per these statements, CASE II will get eliminated and 19) E
we will continue with CASE I and the arrangement will 20) A
look like this: Solutions (16-20):
CASE I: STEP I: Three persons sit between P and Q who likes One
plus and they are sitting at point 3 and point 4. R sits to
the immediate left of Q.
As per these statements, there will be two possible cases
CASE II:
and the arrangement will look like this:
CASE I:

STEP VI: Person who planted Pine tree and the person
who planted Elm tree are immediate neighbours of each
other. Neither T nor S planted Pine tree. S who planted
Neem tree sits between D and the person who planted
Palm tree. Number of persons sits to the right of Y is one CASE II:
more than the number of persons sits to the left of F.

Page 810 of 1334


Subscribe The Xpress Video Course & Mock Test Package for Bank & Insurance Exams
If there are any suggestions/ errors in our PDFs Feel Free to contact us via this email: admin@exampundit.in
Ultra Practice Bundle PDF
SBI Clerk/ RBI Assist. Mains – Reasoning
STEP III: W who likes Apple sits third to the right of the
person who likes Realme. C who sits third to the right of
K likes Vivo but both does not sit at the corner of table. H
sits second to the right of C.
As per these statements, CASE I will get eliminated and
we will continue with CASE II and the arrangement will
look like this:
STEP II: G sits third to left of A who likes Realme and CASE I:
who sits sixth to the right of Q. K and A are immediate
neighbours but not on the same side.
As per these statements, the arrangement will look like
this:
CASE I:

CASE II:

CASE II:

STEP IV: R and H are not sitting on the same side of the
table. Two persons sit between H and R who likes Jio.

Page 811 of 1334


Subscribe The Xpress Video Course & Mock Test Package for Bank & Insurance Exams
If there are any suggestions/ errors in our PDFs Feel Free to contact us via this email: admin@exampundit.in
Ultra Practice Bundle PDF
SBI Clerk/ RBI Assist. Mains – Reasoning
As per these statements, the arrangement will look like 24) C
this: 25) E
CASE II: Solutions (21-25):
STEP I: G who likes Kiwi sits at the centre of the table.
P and B are an immediate neighbours of G.
As per these statements, there will be two possible cases
and the arrangement will look like this:
CASE I:

STEP V: Person who likes Poco sits third to the right of


the person who likes Nokia. Six persons sit between the
person who likes Oppo and the person who likes MI when
CASE II:
counted from the left of the person who likes MI.
As per these statements, the final arrangement will look
like this:
CASE II:

STEP II: Except G no one sits at the vertex of the table.


Both P and B are sitting alone on a side of the table. Only
one person sits between O and P but not on the same side
of the table.
As per these statements, the arrangement will look like
this:
CASE I:
Answers (21-25):
21) B
22) C
23) A
Page 812 of 1334
Subscribe The Xpress Video Course & Mock Test Package for Bank & Insurance Exams
If there are any suggestions/ errors in our PDFs Feel Free to contact us via this email: admin@exampundit.in
Ultra Practice Bundle PDF
SBI Clerk/ RBI Assist. Mains – Reasoning

CASE II: CASE II:

STEP III: Two persons sit between B and M who likes


Mango but on different sides of the table. Person, who CASE II (A):
likes Mango is not an immediate neighbour of the person,
who likes Grapes.
As per these statements, CASE I and CASE II will
further get spilt into one more case each and the
arrangement will look like this:
CASE I:

STEP IV: H sits second to the left of the person who likes
Mango. R who is the neighbour of H sits two places away
from the person who likes Apple. Person who likes
Papaya faces the person who likes Apple.
As per these statements, CASE II (A) and CASE I (A)
CASE I (A): will get eliminated and we will continue with CASE I and
CASE II and the arrangement will look like this:
CASE I:

Page 813 of 1334


Subscribe The Xpress Video Course & Mock Test Package for Bank & Insurance Exams
If there are any suggestions/ errors in our PDFs Feel Free to contact us via this email: admin@exampundit.in
Ultra Practice Bundle PDF
SBI Clerk/ RBI Assist. Mains – Reasoning
Malta and the person who likes Litchi are an immediate
neighbours of each other.
As per these statements, the arrangement will look like
this:
CASE I:
CASE I (A):

CASE II:

CASE II:

STEP VI: S who likes Dragon Fruit sits three places away
from the person who likes Litchi, who sits alone on one of
CASE II (A):
the sides of the table. Person who likes Banana sits to the
immediate left of the person who likes Orange but on the
different side of the table. R who likes Guava and the
person who likes Orange are an immediate neighbours.
As per these statements, CASE II will get eliminated and
we will continue with CASE I and the final arrangement
will look like this:
CASE I:
STEP V: Only one person sits between the person who
likes Grapes and the person who likes Papaya. L who likes

Page 814 of 1334


Subscribe The Xpress Video Course & Mock Test Package for Bank & Insurance Exams
If there are any suggestions/ errors in our PDFs Feel Free to contact us via this email: admin@exampundit.in
Ultra Practice Bundle PDF
SBI Clerk/ RBI Assist. Mains – Reasoning

CASE II:

CASE II:

Answers (26-30):
26) B
27) C
28) A
STEP II: Y sits to the immediate left of A and likes
29) E
Honda. Two persons sit between K and a person who like
30) D
Jaguar. C sits to the immediate left of the person who likes
Solutions (26-30):
Jaguar and faces the person who likes Alto.
STEP I: A who likes Audi sits at the centre of the table.
As per these statements, the arrangement will look like
K sits two places away from A and sits at the corner of the
this:
table.
CASE I:
As per these statements, there will be two possible cases
and the arrangement will look like this:
CASE I:

CASE II:
Page 815 of 1334
Subscribe The Xpress Video Course & Mock Test Package for Bank & Insurance Exams
If there are any suggestions/ errors in our PDFs Feel Free to contact us via this email: admin@exampundit.in
Ultra Practice Bundle PDF
SBI Clerk/ RBI Assist. Mains – Reasoning
STEP IV: No one sit to the right of E on the same side of
the table. I who likes BMW faces the person who sits to
the immediate right of the person who likes Hyundai.
Person who likes Hyundai and a person who likes Alto do
not sit on the same side of the table.
As per these statements, CASE II will get eliminated and
we will continue with CASE I and the arrangement will
STEP III: Two persons sit between A and E who likes
look like this:
Toyota. E do not sit on that side of the table on which the
CASE I:
person who likes Jaguar sits.
As per these statements, the arrangement will look like
this:
CASE I:

CASE II:

CASE II:

STEP V: D faces the person who sits between the person


who likes Alto and S. Either K or S likes Maruti.
As per these statements, the arrangement will look like
this:
CASE I:

Page 816 of 1334


Subscribe The Xpress Video Course & Mock Test Package for Bank & Insurance Exams
If there are any suggestions/ errors in our PDFs Feel Free to contact us via this email: admin@exampundit.in
Ultra Practice Bundle PDF
SBI Clerk/ RBI Assist. Mains – Reasoning

STEP II: T sits to the immediate left of the person who


likes Dangal. P who likes Sultan sits exactly between M
and T but not immediate. No one sit between the person
who likes Avataar and M.

STEP VI: Person who likes Ertiga does not share the side As per these statements, the arrangement will look like

with the person who likes Toyota. this:

As per these statements, the final arrangement will look


like this:
CASE I:
STEP III: Q sits third to the right of the person who likes
Sultan. L sits second to the right of N. Two persons sit
between L and K who likes Avataar.
As per these statements, the arrangement will look like
this:

Answers (31-35):
STEP IV: No one sits to the right of the person who likes
31) B
Lion. Person who likes Joker sits fourth to the left of the
32) E
person who likes Lion.
33) B
As per these statements, the arrangement will look like
34) C
this:
35) E
Solutions (31-35):
STEP I: S who sits at the right end of the row sits three
places away from the person who likes Dangal. STEP V: Person who likes 1917 sits exactly between the

As per this statement, the arrangement will look like this: person who likes 1920 and T. M neither likes 1920 nor
likes Joker. Number of persons sits to the left of the person

Page 817 of 1334


Subscribe The Xpress Video Course & Mock Test Package for Bank & Insurance Exams
If there are any suggestions/ errors in our PDFs Feel Free to contact us via this email: admin@exampundit.in
Ultra Practice Bundle PDF
SBI Clerk/ RBI Assist. Mains – Reasoning
who likes 1920 is one less than the number of persons sits
to the right of the person who likes Dangal.
As per these statements, the arrangement will look like
this:

STEP VI: Only one person sits between the person who
likes 1920 and the person who likes Swadesh and the one,
who likes Swadesh will sit to the left of L. CASE II:
As per this statement, the final arrangement will look like
this:

Answers (36-40):
36) D
37) A
38) B
STEP II: D sits at the left end of the lower side of the
39) D
table. Only one person sits between B and D but not on
40) B
the same side of the table. B who was born in October sits
Solutions (36-40):
to the south of F who is not an immediate neighbour of H.
STEP I: H sits fourth to the right of A who was born in
As per these statements, CASE II will get eliminated and
June and both are not sitting on the longer side of the table.
we will continue with CASE I and CASE I will further
As per this statement, there will be two possible cases and
split into one more case and the arrangement will look like
the arrangement will look like this:
this:
CASE I:
CASE I:

Page 818 of 1334


Subscribe The Xpress Video Course & Mock Test Package for Bank & Insurance Exams
If there are any suggestions/ errors in our PDFs Feel Free to contact us via this email: admin@exampundit.in
Ultra Practice Bundle PDF
SBI Clerk/ RBI Assist. Mains – Reasoning
the east of D and born in May. E sits to the south of F and
to the south-east of B.
As per these statements, the arrangement will look like
this:
CASE I:

CASE I (A):

CASE I (A):

CASE II:

STEP IV: Three persons sit between the person who was
born in October and the person who was born in August
but on the different sides. Person who was born in August
STEP III: Three persons sit between D and C who was and the person who was born in April are sitting on the
born in March. B does not share the side with A. E sits to upper side of the table.

Page 819 of 1334


Subscribe The Xpress Video Course & Mock Test Package for Bank & Insurance Exams
If there are any suggestions/ errors in our PDFs Feel Free to contact us via this email: admin@exampundit.in
Ultra Practice Bundle PDF
SBI Clerk/ RBI Assist. Mains – Reasoning
As per these statements, the arrangement will look like
this:
CASE I:

CASE I (A):

CASE I (A):

Answers (41-45):
41) C
STEP V: G who was born in January sits to the north-east
42) B
of the person who was born in July who sits neither at the
43) E
upper side nor at the middle side of the table.
44) D
As per this statement, CASE I (A) will get eliminated and
45) B
we will continue with CASE I and the final arrangement
Solutions (41-45):
will look like this:
STEP I: R sits second to the left of E and likes Pasta. Only
CASE I:
one person sit between the person likes Idli and the person
likes Pizza who sits four places away from R.
As per this statement, the arrangement will look like this:
Page 820 of 1334
Subscribe The Xpress Video Course & Mock Test Package for Bank & Insurance Exams
If there are any suggestions/ errors in our PDFs Feel Free to contact us via this email: admin@exampundit.in
Ultra Practice Bundle PDF
SBI Clerk/ RBI Assist. Mains – Reasoning
CASE I:

STEP II: G likes Maggie. L sits second to the left of the


CASE II:
person who likes Maggie who sits three places away from
the person who likes Idli. L who is not an immediate
neighbour of R sits to the left of E who likes Idli.
As per these statements, CASE I, CASE II and CASE
III will get eliminated and we will continue with CASE
IV and the arrangement will look like this:
CASE I:

CASE III:

CASE II:

CASE IV:

Page 821 of 1334


Subscribe The Xpress Video Course & Mock Test Package for Bank & Insurance Exams
If there are any suggestions/ errors in our PDFs Feel Free to contact us via this email: admin@exampundit.in
Ultra Practice Bundle PDF
SBI Clerk/ RBI Assist. Mains – Reasoning

CASE III:
STEP IV: Seven persons sit between the person who likes
Maggie and the person who likes Samosa when counted
anticlockwise from the person who likes Maggie.
As per this statement, the arrangement will look like this:
CASE IV:

CASE IV:

STEP V: Q sits three places away from K. K neither likes


Pizza nor likes Dosa. Q sits seventh to the left of L who
likes Burger. P sits fifth to the right of Q.
As per these statements, the final arrangement will look
STEP III: Only one person sits between L and the person
like this:
who likes Dosa. Person who likes Samosa sits second to
the left of the person who likes Dosa. Person who likes
Samosa is not an immediate neighbour of E.
As per this statement, the arrangement will look like this:
CASE IV:
Page 822 of 1334
Subscribe The Xpress Video Course & Mock Test Package for Bank & Insurance Exams
If there are any suggestions/ errors in our PDFs Feel Free to contact us via this email: admin@exampundit.in
Ultra Practice Bundle PDF
SBI Clerk/ RBI Assist. Mains – Reasoning

CASE III:
Answers (46-50):
46) E
47) E
48) D
49) C
50) A
Solutions (46-50):
STEP I: P sits to the north of G who works in RBI but not STEP II: Three persons sit between N and T who works
immediate north. Less than four persons but at least one in BOI and also sits to the immediate south of P.
person sits to the north of P. As per these statements, the arrangement will look like
As per these statements, there are three possible cases and this:
the arrangement will look like this: CASE I:
CASE I:

CASE II:
CASE II:
Page 823 of 1334
Subscribe The Xpress Video Course & Mock Test Package for Bank & Insurance Exams
If there are any suggestions/ errors in our PDFs Feel Free to contact us via this email: admin@exampundit.in
Ultra Practice Bundle PDF
SBI Clerk/ RBI Assist. Mains – Reasoning

CASE II:
CASE III:

CASE III:

STEP III: H sits at the extreme west of the table. G sits


second to the left of H. G and N are not an immediate
neighbours of each other.
As per these statements, CASE II and CASE III will get
eliminated and we will continue with CASE I and the
arrangement will look like this:
CASE I:
STEP IV: M sits to the west of R but not immediate west.
R sits second from the right end of the table. Three persons
sit between H and R who works in SBI.
As per these statements, the arrangement will look like
this:
Page 824 of 1334
Subscribe The Xpress Video Course & Mock Test Package for Bank & Insurance Exams
If there are any suggestions/ errors in our PDFs Feel Free to contact us via this email: admin@exampundit.in
Ultra Practice Bundle PDF
SBI Clerk/ RBI Assist. Mains – Reasoning
CASE I:

STEP VI: Person who works in UBI sits three places


STEP V: More than eight persons sit between N and M
away to the north of the person who works in OBC who
who sits sixth to the right of the person who works in BOI.
also sits three places away to the north of the person who
Person who works in UCO sits second to the left of the
works in RBI.
person who works in PNB who sits fourth from the right
As per this statement, the final arrangement will look like
end.
this:
As per these statements, the arrangement will look like
CASE I:
this:
CASE I:

Download Seating Arrangement Practice Questions PDF

Get More Reasoning Practice Questions PDF

Page 825 of 1334


Subscribe The Xpress Video Course & Mock Test Package for Bank & Insurance Exams
If there are any suggestions/ errors in our PDFs Feel Free to contact us via this email: admin@exampundit.in
Ultra Practice Bundle PDF
SBI Clerk/ RBI Assist. Mains – Reasoning
Seating 3 Parallel Row
Direction (1-5): Study the following information sits opposite to the one who sits immediate left of H.
carefully and answer the questions given below. R does not sit at any of the extreme ends. Both the
Ten persons T, A, R, H, C, N, V, L, Z and Y are sitting immediate neighbours of R are facing opposite
in two parallel rows containing five people each, in direction. R faces north. The one who sits second to
such a way that there is an equal distance between the left of R does not sit opposite to H. H does not sit
adjacent persons. In row 2, Person name starts with opposite to V. V likes Mysore pak. The one who likes
second half of the English alphabetical series are Laddu sits opposite to the one who sits second to the
seated and some of them are facing north and some of left of Y. The one who likes Rasgulla and the one who
them are south. In row 1, Z and the Person name likes Peda are immediate neighbours. A does not like
starts with first half of the English alphabetical series Peda. L does not like Kulfi. C and V face same
are seated and some of them are facing north and direction which is opposite to Y. Number of persons
some of them are facing south. The seats were sits right of N is same as the number of persons sits
arranged in such a way that the Row-1 is placed right of Z. N does not sit at any of the extreme ends.
behind of the Row-2. The seats of row 1 is south of C and A faces opposite direction. More than two
the seats of row 2 .Each one of them likes different persons sit between the one who likes Rasmali and
sweets Kulfi, Barfi, Gulab jamun, Laddu, Rasmali, the person name starts with the vowel.
Peda, Rasgulla, Kaju katli, Mysore pak and Kheer. 1) Which of the following person likes Peda?
All the above information is not necessarily in the a) C
same order. b) V
C sits second to the left of the one who likes Rasmali. c) Z
The one who sits opposite to C sits immediate left of d) L
T. The one who likes Barfi sits immediate left of N. e) None of these
Z and N does not sit opposite to each other. The one 2) How many persons are facing north and south
who likes Gulab jamun sits opposite to the one who respectively?
sits second to the left of L. The one who likes Kheer a) 3, 7

Page 826 of 1334


Subscribe The Xpress Video Course & Mock Test Package for Bank & Insurance Exams
If there are any suggestions/ errors in our PDFs Feel Free to contact us via this email: admin@exampundit.in
Ultra Practice Bundle PDF
SBI Clerk/ RBI Assist. Mains – Reasoning
b) 6, 4 There are nine people namely – O, P, Q, R, S, T, U,
c) 5, 5 V and W sitting in a row in such a way that some are
d) Cannot be determined facing north while other sits facing south. Each likes
e) None of these different laptop brand viz. LG, Acer, Lenovo, Sony,
3) Which of the following person sits second to the HP, Apple, Dell, Toshiba and Asus, the distance
left of Y? between each adjacent is different viz. 28, 18, 15, 8,
a) V 32, 12, 35 and 13 meter. All the information is not
b) The one who likes Barfi necessary to be in the same order.
c) T Q sits third to the right of W, either of them sits at
d) The one who likes Gulab jamun end of the row. The distance between the one who
e) None of these likes Sony and one who likes Dell is 36m. R, who
4) Four of the following five are like in a certain likes LG, sits third to the right of V and V sits
way and hence they form a group. Which one of immediate neighbor of Q. Distance between P and Q
the following does not belong to that group? is 60m. The number of the persons sitting right of Q
a) The one who likes Kulfi is one more than the number of the persons sit left of
b) The one who likes Rasgulla the one who likes Apple. The persons sitting at the
c) The one who likes Rasmali end of the row are facing in opposite direction. T who
d) The one who likes Barfi doesn’t like Acer sits at a gap of two from the person
e) The one who likes Laddu who likes Asus. The distance between O and U is
5) If V is related to Peda, T is related to Laddu, in 31m. At least four persons sit between the one who
the same way C is related to which of the likes HP and O, who doesn’t like Apple. One who
following? likes Acer sits immediate right of W, who sits facing
a) Rasmali south and W neither likes HP nor likes sony. The
b) Kulfi distance between T and S is more than 30m. U is
c) Rasgulla neither sits at the end nor likes HP, sits fourth to the
d) Kheer right of T, who is facing south. The distance between
e) None of these R and P is 28m. S sits immediate left of the one who
Directions (6-10): Study the following information likes Dell. The persons sitting adjacent to the one who
carefully and answer the below questions. likes Dell sits facing in the same direction. The

Page 827 of 1334


Subscribe The Xpress Video Course & Mock Test Package for Bank & Insurance Exams
If there are any suggestions/ errors in our PDFs Feel Free to contact us via this email: admin@exampundit.in
Ultra Practice Bundle PDF
SBI Clerk/ RBI Assist. Mains – Reasoning
distance between S and W is not the smallest. The a) One who likes Apple.
One, who likes Asus, sits at a gap of two persons from b) V
the one who likes Lenovo. Neither T nor P likes c) One who sits fourth from left end.
Lenovo. d) One who likes Lenovo.
6.What is the position of the one who likes Dell e) Can’t be determined.
with respect to U? 10.What is the distance between the one who sits
a) Third to right second from right end and the one who likes
b) Second to left Toshiba?
c) Fourth to right a) 47 meter
d) Immediate left b) 12 meter
e) Can’t be determined. c) 28 meter
7.What is the distance between the one who likes d) 20 meter
Sony and the one who likes Apple? e) 8 meter
a) 47 meter Directions (11 – 15): Study the following
b) 60 meter information and answer the questions below:
c) 41 meter There are eight persons i.e. A, B, C, D, E, F, G and H
d) 43 meter are sitting around two parallel rows such that five of
e) Can’t be determined. them faces north direction and three of them faces
8.Which of the following statement is true? south direction. They all like different colours i.e. Not
a) Only three persons sit between R and the one more than Four persons sit in a row. The seats in one
who likes Dell. row are exactly faces the seats in other row in equal
b) One who likes Acer sits fourth to the left of P. distance.
c) R sits immediate neighbor of the one who likes F sits second to the right of A. The one who likes
Asus. pink colour sits on the left of C. F does not sits
d) T sits second to the right of the one who likes opposite to the one who likes Pink colour. E sits
Sony. second to the right of G, who does not like Pink
e) None of the given statement is true. colour. Either F or A sits at the end of the row. C, who
9.Who among the following sits immediate right of likes Black Faces H.ie. both C and H faces each other.
the one who likes LG? F and C faces same direction. C is not the immediate

Page 828 of 1334


Subscribe The Xpress Video Course & Mock Test Package for Bank & Insurance Exams
If there are any suggestions/ errors in our PDFs Feel Free to contact us via this email: admin@exampundit.in
Ultra Practice Bundle PDF
SBI Clerk/ RBI Assist. Mains – Reasoning
neighbour of F and does not sits at the end of the row. c) Pink
G sits opposite to the one who likes Brown colour. A d) Yellow
does not like brown colour. No one sits on the right e) None of these
of B. B and A face same direction. Two persons sit 14.Which among the following pair faces opposite
between the one who likes Blue and the one who likes direction to each other?
Red colour. B does not like Blue color and red colour. a) H and G
D does not sits opposite to the one who likes Blue b) D and E
colour and D does not like Red colour. A and E faces c) A and C
opposite direction but E & A do not face each other. d) B and A
G does not like yellow colour. One of the persons who e) None of these
faces north likes Green colour. H and G does not like 15.Which group of people in the following faces
Green colour and either one of them likes White south direction?
color. a) H,E and G
11.Who among the following likes Red colour? b) G, E and A
a) B c) A, B and C
b) E d) C, D and F
c) A e) None of these
d) D Directions (16-20): Study the given information
e) None of these carefully and answer the following questions.
12.Who among the following sits second to the left Twelve persons are sitting in two parallel rows, Row
of D? 1 and Row 2. A, B, C, D, E, and F are sitting in row
a) The one who likes Brown colour 1 and facing South direction. X, Y, Z, J, K, and M are
b) The one who likes Black colour sitting in row 2 and facing North direction. Persons in
c) The one who likes Yellow colour row 1 faces the persons in row 2 in equal
d) The one who likes white colour distance.They have different ages viz, 15, 20, 25, 28,
e) None of these 30, 36, 39, 49, 50, 63, 64, and 65. The persons whose
13.H likes which of the following colour? ages are even numbers are not sit adjacent to each
a) White other and the persons whose ages are odd numbers
b) Blue are not sit adjacent to each other.

Page 829 of 1334


Subscribe The Xpress Video Course & Mock Test Package for Bank & Insurance Exams
If there are any suggestions/ errors in our PDFs Feel Free to contact us via this email: admin@exampundit.in
Ultra Practice Bundle PDF
SBI Clerk/ RBI Assist. Mains – Reasoning
A is elder than J. K is 63 years old and sits third 18.Who sits second to the left of the one who sits
from one of the extreme ends of the row. F’s age is opposite to J?
twice that of Y and sits opposite to K. There are two a) The one who is the eldest person
persons sit between F and B who is the youngest b) The one who is 49 years old
person among the group. Y sits opposite to the one c) The one who is youngest person
whose age is 64 years. C’s age is multiple of 5 and D d) None of those given as option
sits immediate left of C. D is not 64 years old. X is 20 e) D
years younger than F and sits opposite to C. The ages 19.Four of the five among the following are similar
of A and J are square numbers below 50. M sits in such a way to form a group, which one of the
somewhere to the left of Y. A sits second to the left following doesn’t belongs to the group?
of the one who is opposite to M. The number of a) 15
persons sits right of E is one less than that of the b) 28
number of persons sits left of the one whose age is 36. c) 39
The one whose age is 65 sits immediate right of the d) 50
one whose age is 20 years old. Y is not the eldest e) 20
person. Z is elder than J. Y is younger than M. 20.What is the sum of the ages of M, E and Z?
16.What will be the age of Z? a) 121
a) 25 b) 131
b) 49 c) 148
c) 36 d) 139
d) 39 e) None of those given as options
e) None of those given as options (Directions 21–25): Study the following
17.How many people younger than Y? information carefully and answer the questions
a) One given below it.
b) Two Fourteen persons are sitting in two parallel lines
c) Three facing each other on sixteen chairs eight in each row
d) More than three in such a way that A, B, C, D, E, F, and G are sitting
e) Y is the youngest person in row 2 facing south and P, Q, R, S, T, U, and V are

Page 830 of 1334


Subscribe The Xpress Video Course & Mock Test Package for Bank & Insurance Exams
If there are any suggestions/ errors in our PDFs Feel Free to contact us via this email: admin@exampundit.in
Ultra Practice Bundle PDF
SBI Clerk/ RBI Assist. Mains – Reasoning
sitting in row 1 facing north. each row contains one 23.How many person sits between S and one who
Vacant chair . faces G?
R and S sit adjacent to each other. Vacant places a) One
are not opposite to each other. B sits third to the right b) Four
of G, who is facing the one who sits immediate left of c) Two
V. Only one person sits between E and B. Either of B d) None
or V sits at the end of the row. A sits third to the right e) Three
of C, who faces U. Only three-persons sit between U 24.If E and F interchange their position then, who
and P, who sits immediate left of Q. S neither faces sits second to right of E?
B nor faces the one who sits immediate neighbor of a) A
G. T sits second to the right of U, who doesn’t sit b) C
immediate neighbor of S. D sits second to the left of c) G
F, who is neither sit at the extreme end nor faces P. d) B
Only two persons sits between V and R.C doesn’t sit e) None of these
at either end of the row. 25.Which of the given statement is not true for G?
21.How many person sits between T and vacant a) C sits second to right of G
chair in row1? b) Only three persons sit between G and one who
a) Two sits facing Q
b) Four c) G sits third to the left of the one who sits facing
c) One R
d) None d) G sits facing the one who sits immediate left of
e) Three V
22.What is the position of A with respect to D? e) All the given statements are true
a) Fourth to the left Direction (26 - 30): Answer the questions on the
b) Sixth to the right basis of the information given below .
c) Third to the right A conference is conducted in a bank headquarters.
d) Fifth to the left Seven Managers are sitting in a linear row but some
e) Second to the left of them are facing south and some of them are facing
north. Managers are sitting in a consecutive English

Page 831 of 1334


Subscribe The Xpress Video Course & Mock Test Package for Bank & Insurance Exams
If there are any suggestions/ errors in our PDFs Feel Free to contact us via this email: admin@exampundit.in
Ultra Practice Bundle PDF
SBI Clerk/ RBI Assist. Mains – Reasoning
alphabetical order from left to right. Each one of them c) 5
represents different sectors from one to seven. d) 7
The manager who represents sixth sector is sitting e) 6
second to the right of H. Managers sit at the extreme 27. Who among the following person sits between
ends face opposite in the direction( if one is facing J and L and that person represents which sector?
south then other is facing north). Manager of fourth a) K,2
sector is towards left of the manager of sixth sector. b) M ,2
Either the manager of the third sector or the manager c) I, 4
of the fourth sector (none of them is K) sits towards d) H, 5
the immediate left of the manager of the sixth sector. e) K, 3
Manager represents the second sector sit at a gap of 28. How many persons are facing north direction?
one manager towards the right of the manager a) 3
represents fifth sector. The fourth sector manager sits b) 2
second to the right of the manager from the third c) 4
sector, who faces south. The immediate neighbours of d) 1
the second sector manager face the same direction. e) 5
The manager of seventh sector is not next to third 29. Which one of the following managers
sector manager. The manager of seventh sector is to represents the fifth sector?
the immediate left of the manager of second sector, a) M
who is at a gap of three from the manager of the fourth b) G
sector. The manager who is immediate right to the c) H
manager of fifth sector is facing north. Neither the d) I
second sector manager nor the third sector manager is e) J
M. Only one vowel is used in the alphabetical series. 30. Who among the following pair is sitting at the
L does not sit extreme ends. extreme ends?
26. Which one of the following sector does I a) GH
represents? b) ML
a) 1 c) KM
b) 3 d) MG

Page 832 of 1334


Subscribe The Xpress Video Course & Mock Test Package for Bank & Insurance Exams
If there are any suggestions/ errors in our PDFs Feel Free to contact us via this email: admin@exampundit.in
Ultra Practice Bundle PDF
SBI Clerk/ RBI Assist. Mains – Reasoning
e) JK b) Second to the left
Directions (31-35): Study the following c) Third to the right
information carefully to answer the given d) Third to the left
questions. e) None of these
Eight people A, B, C, D, E, F, G, and H are sitting in 32.Who amongst the following person’s age is 14?
a straight line with equal distances between each a) A
other, but not necessarily in the same order. Some of b) B
them are facing north and some of them are facing c) Other than those given as options
south. All have different ages 14, 16, 17, 19, 21, 23, d) D
26, 31, but not in the same order. e) E
A sits at one of the extreme ends. Only three 33.How many people face south as per the given
people sit between A and G. E sits exactly between A arrangement?
and G. H sits third to the right of E. B is an immediate a) Two
neighbour of H and faces south. C sits second to the b) Four
right of F. C is not an immediate neighbour of G. c) Three
Immediate neighbours of G face opposite directions d) More than four
(i.e. if one neighbour faces North then the other e) One
neighbour faces south and Vice-Versa). A and D face 34.Four of the following five are alike in a certain
the same direction as E Faces (if E faces north then A way based upon their seating arrangement and so
and D also face North and Vice-Versa). Both the form a group. Which of the following does not
immediate neighbours of E face south. Three people belongs to the group?
sit between B and 16 yr old. 23 yr old sits immediate a) EC
left of B. Three persons sit between 26 yr and C. The b) AF
difference of the ages of neighbors of C is 15. Neither c) BF
B nor C is the youngest. B is elder than C. The d) CG
difference of the ages of neighbors of G is 7. e) DG
Immediate neighbours of F faces opposite direction. 35.Who amongst the following person sits at the
31.What is the position of C with respect to G? extreme end of the row?
a) Second to the right a) 21 yr old

Page 833 of 1334


Subscribe The Xpress Video Course & Mock Test Package for Bank & Insurance Exams
If there are any suggestions/ errors in our PDFs Feel Free to contact us via this email: admin@exampundit.in
Ultra Practice Bundle PDF
SBI Clerk/ RBI Assist. Mains – Reasoning
b) C Only four persons are standing between David and
c) 23 yr old Teena. David stands at the left ends of the line. James
d) E stands second to the left of Nancy, who is not an
e) Other than those given as options immediate neighbor of Teena. Sekar is the immediate
Directions (36-40): Study the following neighbor of Rani. Only one person is standing
information carefully and answer the questions between Guna and Rani.
given below Similarly, Eight persons-Latha, Sony, Sam, Peter,
Anu, Ram, Ragu and Renu are standing in a line-3
FR. All of them are facing south (then point F is
considered as right end and point R is left end) and
the distance between them are increasing in multiples
of 4 from the right end of the line (i.e. if the first
person is standing at the right end of the line at point
F and the remaining will be standing at the distance
as follows-4m, 8m,12m…. from point F).
Only three persons stand between Anu and Sony.
In the given figure the five-line segments 1,2,3,4 and Latha and Sony are not immediate neighbours. Sony
5 are PY, WS, FR, LQ, and TP respectively. The stands at extreme left end of the line. Four persons
lengths of the line are 40m, 40m, 34m, 50m, and 30m stand between Sam and Ragu. Two persons stand
respectively. between Latha and Anu. Renu stands second to the
Point W, Point L, and Point T are at the Extreme left right of Ragu. Ram stands third to the left of Peter.
end of the line WS, LQ, and TP. 36.In the line1- PY, what is the distance between
Eight Persons- Reka, Guna, Nancy, Teena, David, James and Rani?
Sekar, Rani and James are standing in a line 1-PY. All a) 20m
of them are facing north and the distance between b) 25m
them is increasing in multiples of 5 from the left end c) 15m
(i.e. if the first person is standing at the left end of the d) 10m
line at point P and the remaining will be stand at the e) None of these
distance as follows-5m, 10m,15m…. from point P.

Page 834 of 1334


Subscribe The Xpress Video Course & Mock Test Package for Bank & Insurance Exams
If there are any suggestions/ errors in our PDFs Feel Free to contact us via this email: admin@exampundit.in
Ultra Practice Bundle PDF
SBI Clerk/ RBI Assist. Mains – Reasoning
37.In the line 1-PY, if all the people stand in 40. In line PY, Rani and James are related to each
multiples of 8m in the same order and the rest of other in such a way, Reka and Teena related.
them moves to line 4 - LQ and stands in multiples Following the same way in line FR which pair is
of 5 from the left end in the same order , then related to each other?
What is the distance between Sekar and point Q? a) Latha and Ragu
a) 10m b) Sam and Sony
b) 20m c) Peter and Ram
c) 30m d) Anu and Ragu
d) 45m e) Latha and Renu
e) None of these Directions (41 -45): Study the following
38.If the line-3 FR is joined to line 5-TP to form a information carefully and answer the given
new straight-line FP in such a way that distance Questions below.
between point R and T is 4m. Point T is to the right Ten people are sitting in two parallel rows. Ram,
of point R. If the people in line-3 FR are made to Ravi, Rohit, Ranjit and Rakesh are sitting in row 1
stand in multiples of 7 to the newly formed line FP, facing South. Sandeep, Shukul, Sameer, Sarvesh and
then how far is Anu from point P? Sanjay are sitting in row 2 facing north. The person
a) 40m sitting in row 1 exactly faces the person sitting in row
b) 54m 2. They like five different cars, Benz, BMW, Audi,
c) 47m Toyotto and Swift. Exactly two people like the same
d) 63m Car and the persons who like the same Car don’t sit
e) None of these in the same row.
39.If all the people in line 1-PY are made to stand Shukul sits at one of the extreme ends. There is only
in line 2-WS in multiples of 3, then What is the one person sits between Shukul and the one who faces
distance between Guna and point S? the person who like Benz Car. Ranjit sits to the
a) 30m immediate right of the person who like Benz Car.
b) 24m There is only one person sits between Shukul and
c) 28m Sameer. Sameer likes Benz Car. Number of persons
d) 23m who sits to the right of Sameer is same as that of
e) None of these number of persons who sits to the left of Rohit. The

Page 835 of 1334


Subscribe The Xpress Video Course & Mock Test Package for Bank & Insurance Exams
If there are any suggestions/ errors in our PDFs Feel Free to contact us via this email: admin@exampundit.in
Ultra Practice Bundle PDF
SBI Clerk/ RBI Assist. Mains – Reasoning
person who likes Toyotto Car sits immediate left of a) Rohit
Sameer. Two people sit between Sanjay and Sandeep. b) Ranjit
Sanjay sits somewhere to the left of Sandeep. Sanjay c) Sandeep
faces the one who like Toyotto Car. One of the people d) Ravi
who likes SwiftCar is an immediate neighbour of the e) Sameer
one who like Audi Car in row 2. Rakesh sits left of 44.Which of the following statement is/are
Ram. Rakesh is an immediate neighbour of Ravi. correct?
Ravi faces the person who sits to the immediate right i) Rakesh and Sanjay like same Car
of the person who like Swift Car. Sarvesh and Ravi ii) Sameer sits second to the right of Shukul
doesn’t like Swift Car. There is only one person sits iii) Rohit is an immediate neighbour of Rakesh and
between Ranjit and the one who like BMW Car. Ravi
41.Who among the following like BMW Car? a) Only (ii)
a) Ranjit b) Only (i)
b) The one who faces Benz Car c) Only (i) and (ii)
c) The one who sits immediate left of Sarvesh d) Only (ii) and (iii)
d) Sameer e) None of the statement is correct
e) None of these 45.What is the position of Sanjay with respect to
42.In a certain way, Ranjit is related to Sameer, the one who like Swift Car?
Sandeep is related to Rohit, in the same way a) Third to the left
Sarvesh is related to who among them following b) Second to the right
person? c) Third to the right
a) The one who sits immediate left of Rakesh d) Immediate left
b) Ranjit e) None of these
c) The one who like BMW Car Directions (46 – 50): Read the information given
d) The one who sits immediate right of Ranjit below and answer the questions that follow.
e) None of these There are 14 people from A to N as in alphabetical
43.Four of the five among the following are similar series were sitting in 2 rows, 7 in each. There are
in such a way to form a group, which one of them equal number of boys and girls. Boys are sitting in
doesn’t belongs to the group? one row and girls in other. Boys face south and girls

Page 836 of 1334


Subscribe The Xpress Video Course & Mock Test Package for Bank & Insurance Exams
If there are any suggestions/ errors in our PDFs Feel Free to contact us via this email: admin@exampundit.in
Ultra Practice Bundle PDF
SBI Clerk/ RBI Assist. Mains – Reasoning
face north. The person sitting in row 1 exactly faces c) I
the person sitting in row 2.They sit based on their d) L
ranks of JEE from right to left in each row in e) None of these
increasing order of their ranks (i.e. 1st rank at the right 48.Who is sitting right of J and is also opposite to
end of the row to 7th rank at the left end of the row). H?
Note: There are 1-7 ranks in the row facing north and a) E
1-7 ranks in the row facing south. b) C
Four couples are sitting opposite to each other; AB, c) I
EF, IJ and MN. A secured 1st among boys and M d) L
came last among boys. E and I came 3rd and e) None
5th among boys while 2nd and 6th was secured by L 49. Which of the following pair of persons are
and D among girls. C sits next to A and H sits next to opposite to each other?
J. K came last but one. a) EH
46. Who secured 3rd rank among girls? b) CK
a) F c) AM
b) H d) GH
c) I e) None of these
d) J 50. Who sits second to the right of F?
e) None of these a) B
47. Who sits second to the right of the one who sits b) H
opposite to H? c) L
a) E d) J
b) C e) N

Seating 3 Parallel Row – Answer and Explanation


ANSWER AND EXPLANATION: 4.Answer: e
Direction (1-5): 5.Answer: d
1. Answer: c
2.Answer: b
3.Answer: b
Page 837 of 1334
Subscribe The Xpress Video Course & Mock Test Package for Bank & Insurance Exams
If there are any suggestions/ errors in our PDFs Feel Free to contact us via this email: admin@exampundit.in
Ultra Practice Bundle PDF
SBI Clerk/ RBI Assist. Mains – Reasoning
of the extreme end. Z and N does not sit opposite to
each other.
It is not possible to place N in third place in row 2,
because Number of persons sits right of N is same as

Explanation: the number of persons sits right of Z.

In row 1, Person name starts with first half of the N can be placed only in one place. But N can either

English alphabetical series are seated. A, H, C, L and faces north or south.

Z are seated.
In row 2, Person name starts with second half of the
English alphabetical series are seated. T, R, N, V and
Y are seated.
C sits second to the left of the one who likes Rasmali.
The one who sits opposite to the one who likes Gulab
More than two persons sit between the one who likes
jamun sits second to the left of L.
Rasmali and the person name starts with the vowel.
Case 1(a):
Only one person name starts with a vowel ie. A. There
are two cases.

Case 1:
The one who sits opposite to C sits immediate left of
T.
The one who likes Kheer sits opposite to the one who
sits immediate left of H. R does not sit at any of the
extreme end. Both the immediate neighbours of R are
facing opposite direction. R faces north. The one who
sits second to the left of R does not sit opposite to H.
The one who likes Barfi sits immediate left of N.
Number of persons sits right of N is same as the
number of persons sits right of Z. N does not sit at any

Page 838 of 1334


Subscribe The Xpress Video Course & Mock Test Package for Bank & Insurance Exams
If there are any suggestions/ errors in our PDFs Feel Free to contact us via this email: admin@exampundit.in
Ultra Practice Bundle PDF
SBI Clerk/ RBI Assist. Mains – Reasoning
of the extreme end. Z and N does not sit opposite to
each other.
It is not possible to place N in third place in row 2,
because Number of persons sits right of N is same as
the number of persons sits right of Z.
N can be placed only in one place. But N can either
Case 1(b): faces north or south.

The one who likes Kheer sits opposite to the one who The one who likes Gulab jamun sits opposite to the
sits immediate left of H. one who sits second to the left of L.
Case 2(a):

Case 2:
The one who likes Kheer sits opposite to the one who
The one who sits opposite to C sits immediate left of
sits immediate left of H.
T.

The one who likes Barfi sits immediate left of N.


Number of persons sit right of N is same as the
number of persons sits right of Z. N does not sit at any

Case 2(b):
Page 839 of 1334
Subscribe The Xpress Video Course & Mock Test Package for Bank & Insurance Exams
If there are any suggestions/ errors in our PDFs Feel Free to contact us via this email: admin@exampundit.in
Ultra Practice Bundle PDF
SBI Clerk/ RBI Assist. Mains – Reasoning

Directions (6-10):
The one who likes Kheer sits opposite to the one who 6. Answer: c
sits immediate left of H. 7. Answer: a
8. Answer: e
9. Answer: e
10. Answer: b

R does not sit at any of the extreme end. Both the


immediate neighbour of R are facing opposite
direction. Explanation:
We have:
• Q sits third to the right of W, either of them sits
at end of the row and the one who likes Acer sits

R faces north. The one who sits second to the left of immediate right of W, who sits facing south that

R does not sit opposite to H. H does not sit opposite means we have two possibilities, in case (1) Q sits at

to V. V likes Mysore pak. The one who likes Laddu left end and in case (2) W sits at extreme right end.
• R, who likes LG sits third to right of V and V
sits opposite to the one who sits second to the left of
Y. The one who likes Rasgulla and the one who likes sits immediate neighbor of Q, that means in case (1)

Peda are immediate neighbours. A does not like Peda. V sits facing north, in case (2a) V sits fourth to the

L does not like Kulfi. C and V face same direction right of W and sits facing south, in case (2b) V sits

which is opposite to Y. C and A faces opposite second to right of W and sits facing south.

direction. Based on above given information, we get:

Case 2b i) B

Page 840 of 1334


Subscribe The Xpress Video Course & Mock Test Package for Bank & Insurance Exams
If there are any suggestions/ errors in our PDFs Feel Free to contact us via this email: admin@exampundit.in
Ultra Practice Bundle PDF
SBI Clerk/ RBI Assist. Mains – Reasoning
of T, who sits facing south, this condition is not
satisfied so rejected.
Again, we have:
• Distance between P and Q is 60m and distance
between R and P is 28m, that means we have only two
possible conditions for distance to be 28m either no
Again, we have: one sits between PR and distance is 28m or only one
• T who doesn’t like Acer sits at a gap of two person sits between PR and distance must be sum of
from the person who likes Asus and U, who neither 15m and 13m, thus in case (1) P sits immediate right
sits at end nor he likes HP, sits fourth to right of T, of T and distance between P and R is 28m, in case
who sits facing south, that means in case (1) U likes (2a) as P not likes Lenovo thus, P likes Asus and
Acer and T sits facing south and third from right end, distance between PR is sum of 15m and 13m.
in case (2a) T sits third from right end facing south • Number of person sitting right of Q is one more
and case (2b) is not valid as U can’t be placed at end than number of person left of the one who likes
of the row. Apple, that means in case (1) Q sits facing north and
• One who likes Asus sits at a gap of two persons the one who likes Apple sits second from right end
from the one who likes Lenovo, neither T nor P likes facing north, in case (2a) Q sits facing north and U
Lenovo, that means in case (1) Q likes Lenovo and in likes Apple sits facing north and in case (2c) Q sits
case (2a) one who likes Lenovo sits at left end. facing north and T likes Apple sits facing south.
Based on above given information, we get: Based on above given information, we have:

Incase 2b, T who doesn’t like Acer sits at a gap of


two from the person who likes Asus and U, who
neither sits at end nor he likes HP, sits fourth to right
Again, we have:
Page 841 of 1334
Subscribe The Xpress Video Course & Mock Test Package for Bank & Insurance Exams
If there are any suggestions/ errors in our PDFs Feel Free to contact us via this email: admin@exampundit.in
Ultra Practice Bundle PDF
SBI Clerk/ RBI Assist. Mains – Reasoning
• At least four persons sit between the one who
likes HP and O, who doesn’t like Apple, that means
in case (1) O sits at right end and V likes HP, Distance
between O and U is 31m, that means case (1) is not
valid. in case (2a) O likes Lenovo and Q likes
HP,since S sits immediate left of the one who likes
Dell,the only option to fix S is immediate left of T,
hence T likes dell, in case (2c) O likes Lenovo and Q Incase 1, At least four persons sits between the one

likes HP since W neither likes HP nor likes sony.. As who likes HP and O, who doesn’t

distance between PR is sum of (13 and 15m) and like Apple, Distance between O and U is 31m, this

distance between OU is sum of (18m and 13m), thus condition is not satisfied so rejected.

distance between RU must be 13m and distance Incase 2c, S sits immediate left of one who

between OR and UP are respectively 18m and 15m in likes Dell, this condition is not

both cases 2a and 2c. satisfied so rejected.

• Person sitting at the end of row are facing in Again, we have:

opposite direction, which means O must sits facing • Distance between P and Q is 60m, as only one

north. S sits immediate left of the one who likes Dell person sits between P and Q, thus only possible

,that means in case (2a) T likes Dell and case (2c) is combination is sum of (28 & 32m). Distance between

not valid. S and W is not smallest, that means distance between

• In case 2a ,Person sitting adjacent to one who SW is not 8 and distance between one who likes Sony

likes Dell sits facing in same direction, which means and one who likes Dell is 36m, as distance between

S sits facing north. The distance between the one who VQ is either 32m or 28m, that means only possible

likes Sony and one who likes Dell is 36m. W neither combination for 36m is (28, 8), thus distance between

likes HP nor likes sony.hence V likes Sony and W VQ and QT are 28m and 8m respectively.

likes Toshiba. Based on above given information, we • Distance between T and S is more than 30m,

have: which means distance between TS must be 35m.


Based on above given information we get final
arrangements as follow:

Page 842 of 1334


Subscribe The Xpress Video Course & Mock Test Package for Bank & Insurance Exams
If there are any suggestions/ errors in our PDFs Feel Free to contact us via this email: admin@exampundit.in
Ultra Practice Bundle PDF
SBI Clerk/ RBI Assist. Mains – Reasoning

Directions (11 – 15):


11. Answer: c
12. Answer: c
13. Answer: d
5) E sits second to the right of G, who does not like
14. Answer: b
Pink colour. G sits opposite to the one who likes
15. Answer: a
Brown colour. A does not like brown colour.
Explanation:
1) F sits second to the right of A. Either F or A sits at
the end of the row.
2) C, who likes Black Faces H ie. both C and H faces
each other.
3) F and C faces same direction. C is not the
immediate neighbour of F and does not sits at the end
of the row.
4) The one who likes pink colour sits on the left of C.
F does not sits opposite to the one who likes Pink
colour.

6) No one sits on the right of B. B and A face same


direction.
7) Two person sits between the one who likes Blue
and Red colour.
Page 843 of 1334
Subscribe The Xpress Video Course & Mock Test Package for Bank & Insurance Exams
If there are any suggestions/ errors in our PDFs Feel Free to contact us via this email: admin@exampundit.in
Ultra Practice Bundle PDF
SBI Clerk/ RBI Assist. Mains – Reasoning
8) B does not like Blue color and red colour. So, case
3, case 4, case-7 and case-8 are eliminated.
9) D does not sits opposite to the one who likes Blue
colour and D does not likes Red colour.
As only three persons faces south direction so case-6
also get eliminated.

So the final arrangement is:

Directions (16-20):

16. Answer: d
17. Answer: b
10) A and E faces opposite direction but E does not
18. Answer: a
face A.
19. Answer: d
So case 1 , case-5 are eliminated.
20. Answer: b
11) G does not like yellow colour. One of the person
Explanation:
who faces north likes Green colour. One of the person
• K is 63 years old and sits third from one of the
like White color.
extreme ends of the row.

Page 844 of 1334


Subscribe The Xpress Video Course & Mock Test Package for Bank & Insurance Exams
If there are any suggestions/ errors in our PDFs Feel Free to contact us via this email: admin@exampundit.in
Ultra Practice Bundle PDF
SBI Clerk/ RBI Assist. Mains – Reasoning
• F’s age is twice that of Y and sits opposite to
K.we got two possibilities for F and Y’s age
combination . i.e (F,Y=50,25) and (F,Y= 30,15)
respectively.
• There are two persons sit between F and B who • M sits somewhere to the left of Y. From this
is the youngest person among the group.since B is the statement case 1 is eliminated.
youngest person , B age is 15, then F and Y’s age must • X is 20 year younger than F and sits opposite to
be the combination (50,25)respectively. C. Form that X’s age is 30years since F’s age is
• Y sits opposite to the one whose age is 64 years. 50.hence in case 2 and case 4 ,C’s age is 20 years .A
• The persons whose ages are even numbers are sits second to the left of the one who sits opposite to
not sit adjacent to each other and the persons whose M. In case 2 and case 4 The persons whose ages are
ages are odd numbers are not sit adjacent to each even numbers are not sit adjacent to each other and
other. By considering all the conditions above, we got the persons whose ages are odd numbers are not sit
4 possibilities, adjacent to each other.these conditions are not
satisfied hence case 2 and case 4 are eliminated.

• C’s age is multiple of 5 i.e. (20/30/65 years in


remaining numbers )and D sits immediate left of C.
• D is not 64 years old.hence to satisfy the
condition The persons whose ages are even numbers
are not sit adjacent to each other and the persons
whose ages are odd numbers are not sit adjacent to
each other, in case 1 and case 3, C must be 65 years
• The number of person sits right of E is one less
old and in case 2and case 4 C’s age must be either 20
than that of number of person sits left of the one
or 30 years.
whose age is 36.E should be sits second from the right
Page 845 of 1334
Subscribe The Xpress Video Course & Mock Test Package for Bank & Insurance Exams
If there are any suggestions/ errors in our PDFs Feel Free to contact us via this email: admin@exampundit.in
Ultra Practice Bundle PDF
SBI Clerk/ RBI Assist. Mains – Reasoning
end in row 1.So we can fix the position of the one 23. Answer: c
whose age is 36 at immediate right of Y. 24. Answer: d
• The ages of A and J are square numbers below 25. Answer: e
50.So it should be either 25 or 36 or 49.Y ‘s age is 25 Explanation:
years. Hence A and J’s age must be either 36 or 49. We have:
• A is elder than J. • B sits third to the right of G, who sits facing the
• From the above two statements J should be 36 one who sits immediate left of V and either of B or V
years old and A should be 49 years old. sits at end of the row, which means we have two
• The one whose age is 65 sits immediate right of possibilities for B and V. In case (1) B sits at extreme
the one whose age is 20 years old. From this statement right end of row 2 and in case (2) V sits at extreme
D is 20 years old. right end of row 1.
• Z elder than J. So Z Should be 39 years old • A sits third to the right of C, who sits facing U
• Y is younger than M. since Y’s age is 25 years and Only three person sits between U and P, who sits
, M should be 28 years old. immediate left of Q and C doesn’t sit at either end of
Based on the above conditions, the final arrangement the row, that means in case (1) A must sit immediate
can be done as, right of G and in case (2) we have two different
possibility of A thus, in case (2a) A sits second to
right of B and in case (2b) A sits immediate right of
B. We got more possibilities for case (1), case (2a)
and case (2b).
Based on above given information we have:

Direction(21-25):

21. Answer: d
22. Answer: b Again, we have:
Page 846 of 1334
Subscribe The Xpress Video Course & Mock Test Package for Bank & Insurance Exams
If there are any suggestions/ errors in our PDFs Feel Free to contact us via this email: admin@exampundit.in
Ultra Practice Bundle PDF
SBI Clerk/ RBI Assist. Mains – Reasoning
• Only one person sits between E and B that • T sits second to the right of U, who doesn’t sit
means case (1)A, case 1B and case 1C are not valid,. immediate neighbor of S, as S sits immediate
D sits second to left of F, who neither sits at end nor neighbor of R, that means T sits immediate left of V.
sits facing P, by this condition all the cases of 2B are Based on above given information, we have final
eliminated. in case (2a) E must sits at extreme right arrangement as follow:
end of row 2 and vacant place is immediate left of A
.Based on above given information, we have:

Direction (26 - 30):

26. Answer: b
27. Answer: a
28. Answer: c
29. Answer: a
All possibilities of Case (1) are not valid as no such
30. Answer: d
place available for E to sit at a gap of one from B
Explanation:
and all possibilities of case (2b) are not valid as no
One who is sitting second to the right of H is in sixth
such place available for D to sits second to left of
sector.
F.
Either the manager of the third sector or the manager
Again, we have:
of the fourth sector (none of them is K) sits towards
• R and S sits adjacent to each other. S neither
the immediate left of the manager of the sixth sector.
sits facing B nor sits facing the one who sits
Case 1
immediate neighbor of G, as Vacant place are neither
When H faces north
adjacent nor facing to each other and Only two
persons sit between V and R, that means R must sits
facing B.

Page 847 of 1334


Subscribe The Xpress Video Course & Mock Test Package for Bank & Insurance Exams
If there are any suggestions/ errors in our PDFs Feel Free to contact us via this email: admin@exampundit.in
Ultra Practice Bundle PDF
SBI Clerk/ RBI Assist. Mains – Reasoning
Case 2 :-
H face south

Case 2A is eliminated because, Manager of fourth


sector is towards left of the manager of sixth
sector.
Case 1C and case 1D are eliminated because, Either The Sector manager who is immediate right to the
the manager of the third sector or the manager of the fifth sector manager faces north.
fourth sector (none of them is K) sits towards the The immediate neighbours of the second sector
immediate left of the manager of the sixth sector. manager faces in the same direction
Fourth sector manager sits second to the right to the The one, who is sitting at extreme ends, face in
third sector manager who face south. opposite directions. Manager represents the second
Case 1 sector sit at a gap of one manager towards the right of
the manager represents fifth sector. The manager of
seventh sector is not next to third sector manager. The
manager of seventh sector is to the immediate left of
the manager of second sector, who is at a gap of three
from the manager of the fourth sector. Only one

Case 2 vowel is used in the alphabetical series.


CASE 1:

Page 848 of 1334


Subscribe The Xpress Video Course & Mock Test Package for Bank & Insurance Exams
If there are any suggestions/ errors in our PDFs Feel Free to contact us via this email: admin@exampundit.in
Ultra Practice Bundle PDF
SBI Clerk/ RBI Assist. Mains – Reasoning
Directions (31-35):

Case 1B is eliminated because The one, who is


sitting at extreme ends, face in opposite directions. 31. Answer: c

Case 2: 32. Answer: c


33. Answer: b
34. Answer: a
35. Answer: c
Explanation:
A sits at one of the extreme ends of the line. Only
three people sit between A and G. E sits exactly
between A and G. H sits third to the right of E.

Case 2b, 2c, 2d are eliminated because Only one


vowel is used in the alphabetical series. two more
reasons are ,The immediate neighbours of the second B is an immediate neighbour of H and faces

sector manager faces in the same direction(case south.C sits second to the right of F. C is not an

2c,case 2d). The manager of seventh sector is not next immediate neighbour of G. Immediate neighbours of

to third sector manager(case 2d 1st possibility). G face opposite directions (i.e. if one neighbour faces

So the final arrangement is, North then the other neighbour faces south and Vice-
Versa). A and D face the same direction as E face (i.e)
if E faces north then A and D also face North and

Page 849 of 1334


Subscribe The Xpress Video Course & Mock Test Package for Bank & Insurance Exams
If there are any suggestions/ errors in our PDFs Feel Free to contact us via this email: admin@exampundit.in
Ultra Practice Bundle PDF
SBI Clerk/ RBI Assist. Mains – Reasoning
Vice-Versa). Both the immediate neighbours of All of them are facing north and the distance between
E face south. them is increasing in multiples of 5 from the left end
(i.e. if the first person is standing at the left end of the
line at point P and the remaining will be stand at the
distance as follows-5m, 10m,15m…. from point P).

Only four persons are standing between David and


Teena. James stands second to the left of Nancy, who
is not an immediate neighbor of Teena. David stands
at the left ends of the line. Sekar is the immediate
Incase 2, both the immediate neighbours of E face
neighbor of Rani. Only one person is standing
south, this condition is not satisfied so rejected.
between Guna and Rani.
Three people sit between B and 16 yr old. 23 yr old
sits immediate left of B. Three persons sit between 26
yr and C. The difference of the ages of neighbours of
C is 15. Neither B nor C is the youngest. B is elder For Line FR:
than C. The difference of the ages of neighbours of G All of them are facing south (then point F is
is 7. Immediate neighbours of F faces opposite considered as right end and point R is left end) and
direction. the distance between them are increasing in multiples
of 4 from the right end of the line(point F) (i.e. if the
first person is standing at the right end of the line at
point F and the remaining will be standing at the
distance as follows-4m, 8m,12m…. from point F).

Directions (36-40):
Only three persons stand between Anu and Sony.
Explanation:
Two persons stand between Latha and Anu. Latha and
For line PY:
Sony are not immediate neighbours. Sony stands at
extreme left end of the line. Four persons stand
Page 850 of 1334
Subscribe The Xpress Video Course & Mock Test Package for Bank & Insurance Exams
If there are any suggestions/ errors in our PDFs Feel Free to contact us via this email: admin@exampundit.in
Ultra Practice Bundle PDF
SBI Clerk/ RBI Assist. Mains – Reasoning
between Sam and Ragu. Renu stands second to the
right of Ragu. Ram stands third to the left of Peter.

36.Answer: b
37.Answer: d

41.Answer: c
42.Answer: d
43.Answer: d
44.Answer: b
45.Answer: a
Explanation:
38.Answer: (c) • Shukul sits at one of the extreme ends.
• There is only one person sits between Shukul
and the one who faces the person who like Benz Car.
• Ranjit sits to the immediate right of the person
who like Benz Car.
• There is one person sits between Shukul and
39.Answer: (c)
Sameer.
• Sameer like Benz Car.

40. Answer: (c)

• Number of persons who sits to the right of


Sameer is same as that of number of persons who sits
to the left of Rohit.
• The person who likes Toyotto Car sits
immediate left of Sameer.
Directions (41 -45):

Page 851 of 1334


Subscribe The Xpress Video Course & Mock Test Package for Bank & Insurance Exams
If there are any suggestions/ errors in our PDFs Feel Free to contact us via this email: admin@exampundit.in
Ultra Practice Bundle PDF
SBI Clerk/ RBI Assist. Mains – Reasoning

• Two people sit between Sanjay and Sandeep.


• Sanjay sits somewhere to the left of Sandeep.
• Sanjay faces the one who like Toyotto Car.
46. Answer: d
47. Answer: b
48. Answer: e
• . One of the people who like Swift Car is an 49. Answer: d
immediate neighbour of the one who like Audi Car in 50. Answer: d
row 2. Explanation:
• Rakesh sits left of Ram. There are 14 people sitting in 2 rows, 7 in each. A to
• Rakesh is an immediate neighbour of Ravi. N are 14 people.
• Ravi faces the person who sits to the immediate ⇒ Boys in one row and girls in other.
right of the person who like Swift Car. ⇒ Boys face south and girls face north.
• Sarvesh and Ravi doesn’t like Swift Car. ⇒ They sit based on their ranks of JEE from right to
• There is only one person sits between Ranjit left in each row in increasing order of their ranks (i.e.
and the one who like BMW Car. Case 1 doesn’t 1st rank at the right end of the row to 7th rank at the
satisfy above condition(Sarvesh and Ravi doesn’t like left end of the row). The person sitting in row 1
Swift Car.). So this case is eliminated. And Case 2 exactly faces the person sitting in row 2.So, the
will be the final arrangement. diagram would be something like this:

Direction (46 – 50):

Page 852 of 1334


Subscribe The Xpress Video Course & Mock Test Package for Bank & Insurance Exams
If there are any suggestions/ errors in our PDFs Feel Free to contact us via this email: admin@exampundit.in
Ultra Practice Bundle PDF
SBI Clerk/ RBI Assist. Mains – Reasoning
1)4 couples, sitting opposite to each other; AB, EF,
IJ and MN. A secured 1stamong boys and M came
last among boys.
2)E and I came 3rd and 5th among boys while
2nd and 6th was secured by L and D among girls.
So, let us look at the diagram now:

C sits next to A and H sits next to J.


So, let us look at the diagram now:

K came last but one.


So, we can conclude that G has scored median
among the boys.

Page 853 of 1334


Subscribe The Xpress Video Course & Mock Test Package for Bank & Insurance Exams
If there are any suggestions/ errors in our PDFs Feel Free to contact us via this email: admin@exampundit.in
Ultra Practice Bundle PDF
SBI Clerk/ RBI Assist. Mains – Reasoning
Seating Concentric Diagrams
Directions (1-5): Study the following information and (d) YP
Answer the questions below: (e) XQ
12 persons M, N, O, P, Q, R, U, V, W, X, Y and Z are 4. If all the persons are arranged in clock wise
sitting in tow concentric hexagons with six people in direction from M how many persons does not
hexagon. M, N, O, P, Q and R sitting in outer hexagon change their position?(Excluding M)
faces centre while U, V, W, X, Y and Z are sitting in inner (a) None
hexagon faces outside. No one sit in the middle of any (b) 4
sides of hexagon. Q sits 2nd to the left of N. One of the (c) 3
immediate neighbors of W faces N. M and N are not (d) 2
immediate neighbor of each other. One person sits (e) 1
between M and R. U faces the one who sit immediate left 5. Who among the following sits 3rd to the right of the
of N. O and P are immediate neighbor of each other. The person who faces Q?
person who faces P is an immediate neighbor of W. There (a) W
is only one person between Z and the person faces P. Y is (b) U
2nd to the left of Z. Z and V opposite to each other. X is an (c) Z
immediate neighbor of V. (d) V
1. Who among the following faces R? (e) Y
(a) X
(b) Y Directions (6-10): Study the following information and
(c) Z Answer the questions below:
(d) U Eight friends E, F, G, H, T, U, V and W are sitting in two
(e) V squares in such a manner that each member of the inner
2. Who among the following are immediate neighbor square sits exactly opposite the member of the outer
of M? square. No one sit at any corner. The members sitting in
(a) RO the outer square are T, U, V and W and all of them are
(b) RQ facing towards the centre while the members of the inner
(c) OP square are E, F, G and H and they are facing away from
(d) QP the centre. Each of them likes a different city, viz Delhi,
(e) QO Shimla, Mumbai, Chennai, Raipur, Hyderabad, Kolkata
3. Four of the following five are alike in a certain and Bhopal, but not necessarily in the same order.
way based on the given information and so form a H likes neither Shimla nor Raipur and faces V, who likes
group. Find the one which does not belong to the neither Bhopal nor Kolkata. The person who likes
group? Hyderabad faces the person who likes Delhi. E, who likes
(a) UR Chennai, faces the immediate neighbor of the person who
(b) ZO likes Mumbai. V sits second to the left of W. The person
(c) VM
Page 854 of 1334
Subscribe The Xpress Video Course & Mock Test Package for Bank & Insurance Exams
If there are any suggestions/ errors in our PDFs Feel Free to contact us via this email: admin@exampundit.in
Ultra Practice Bundle PDF
SBI Clerk/ RBI Assist. Mains – Reasoning
like Raipur and Delhi are in Separate Square. The person (a) H
who likes Bhopal sits on the immediate left of W. U, who (b) W
does not like Mumbai, does not face E. The persons who (c) F
like Bhopal and Kolkata are immediate neighbors, and one (d) G
of them faces F, who likes Shimla. The persons who like (e) E
Hyderabad and Chennai sit in the same Square but they
are not immediate neighbors. Directions (11-15): Study the following information and
6. Who among the following like Mumbai? Answer the questions below:
(a) U
(b) F Twelve persons U, V, W, X, Y, Z, I, J, K, L, M and N are
(c) G sitting in the two Hexagonal table. All of them are sitting
(d) T with equal distance from each other and six persons were
(e) W sitting in each the inner Hexagon and the outer Hexagon.
7. Who among the following faces the one who like The persons sitting in the one hexagon is exactly sits
Chennai? opposite to the persons sitting in the another hexagon and
(a) The one who like Kolkata some of the are facing centre of the hexagon while some
(b) V are facing opposite to the centre of the Hexagon. All the
(c) The one who like Bhopal persons sit at corner of the sides of hexagon. Note: A is
(d) T opposite to B means they are in different hexagon.
(e) The one who like Hyderabad L sits opposite to the one who sits second to the right of Z
8. Who among the following are immediate neighbors and both of them are not sitting in the same hexagon. Z
of the one who like Delhi? and the one who sits in the outer hexagon facing each
(a) U and W other. U sits opposite to the person who sits to the
(b) F and G immediate left of X, who is an immediate neighbor of L.
(c) W and T U faces the same direction as X, who is not an immediate
(d) W and The one who like Mumbai neighbor of the one who faces Z. M sits opposite to J,
(e) T and The one who like Bhopal both are faces same direction and does not sit in the same
9. Who faces the one who sits 2nd to the left of the one table. J sits second to the right of N, who sits to the
who like Shimla? immediate left of V. N and V are facing the same direction
(a) U as X. J does not sit in the same hexagon as U. K and W
(b) W are immediate neighbors and faces the same direction. Y
(c) T sits opposite to the person, who sits second to the left of
(d) V W. Immediate neighbors of K faces opposite directions (if
(e) E one faces centre, then another one faces away from the
10. Four of the following five are alike in a certain centre of the table). Equal number of persons is facing the
way based on the given information and so form a same directions in the inner hexagon. The immediate
group. Find the one which does not belong to the neighbors of the one who faces the person I are facing the
group? opposite direction as W.
Page 855 of 1334
Subscribe The Xpress Video Course & Mock Test Package for Bank & Insurance Exams
If there are any suggestions/ errors in our PDFs Feel Free to contact us via this email: admin@exampundit.in
Ultra Practice Bundle PDF
SBI Clerk/ RBI Assist. Mains – Reasoning
11. Who among the following person is sitting third to the In a ground there are 2 benches in rectangle form. One
left of U? rectangle is inside another. Twelve people went to the
(a) M ground and sit on the benches i.e on two different
(b) Z rectangles-one inside another. T, D, R, A, S and P are in
(c) K the outer rectangle facing inward. E, K, U, M, H and N are
(d) W sitting in the inner rectangle facing outward. They all are
(e) None of these sitting in such a way that in each rectangle four persons
12. Who among the following person is sitting second to are sitting in the middle of the sides and two persons are
right of the one who faces I? sitting on diagonally opposite corners. Each friend in the
(a) J inner rectangle is facing another friend of the outer
(b) N rectangle.
(c) L T sits on any of the corner. There are exactly two persons
(d) V sitting between T and S. H sits second to the left of E. A
(e) None of these is facing N and D is not opposite R in the outer rectangle.
13. How many person sit between the person who is R is on the immediate left of the one who is facing E.
opposite to U and the person who is opposite to W? Neither H nor E faces either S or T. S is not sitting adjacent
(a) 2 to A. K is not facing T. Between K and M there are as
(b) 1 many persons as between R and P.
(c) 3 16. Who among the following sits third to the right of R?
(d) Cannot be determined (a) S
(e) None of these (b) A
14. Four of the following are alike in a certain way so (c) T
form a group which of the following does not belong (d) P
to the group? (e) None of these
(a) L, W 17. What is the position of H with respect to M?
(b) Z. V (a) Second to the left
(c) U,Y (b) Immediate right
(d) I, X (c) Fourth to the left
(e) N, K (d) Immediate left
15. Who sits 4th to the left of the one who sits immediate (e) Second to the right
right of K? 18. If ‘N’ is related to ‘H’ and ‘K’ is related to ‘E’ in a
(a) W certain way, then ‘U’ is related to which of the
(b) V
following?
(c) U
(d) I (a) M
(e) M (b) U
Directions (16-20): Study the following information and (c) H
Answer the questions below: (d) E
(e) None of these
Page 856 of 1334
Subscribe The Xpress Video Course & Mock Test Package for Bank & Insurance Exams
If there are any suggestions/ errors in our PDFs Feel Free to contact us via this email: admin@exampundit.in
Ultra Practice Bundle PDF
SBI Clerk/ RBI Assist. Mains – Reasoning
19. Who among the following person sits opposite to K
in outer Square?
(a) P
(b) A
(c) D
(d) E
(e) None of these
20. What is the position of T with respect to D?
(a) Third to the right
(b) Second to the left
(c) Third to the left
(d) Immediate right
(e) None of these J sits third to left of both B and the one who likes Armani
Perfume in the straight line. B is an immediate neighbour
Directions (21-25): Study the following information of W and sits in the extreme end of the line. The person,
and answer the questions given below. who faces K in the circle, likes Fogg Perfumes in the
Eight people viz. C, V, B, U, D, K, W and J are sitting in straight line. U sits immediate right of K in the circle. W
two circles in equal numbers facing each other as shown sits second from the extreme end of the line and likes the
in the fig. below. The people sitting in the outer circle Perfume liked by J in the circle. The person who likes
faces towards the centre of the circle and the people sitting Denver faces the person who sits immediate left of C in
in the inner circle faces outside the centre of the circle. the circle. J faces the person who sits to the immediate left
Each of them likes different Perfumes viz. Gucci, Denver, in the straight line. The person who likes Armani Perfume
Engage, Wild Stone, Boss, Fogg, Park Avenue and faces the person who likes Wild Stone Perfume. W is not
Armani. All the information is not necessary to be in the an immediate neighbour of J, who likes Engage Perfume
same order. Also, they were made to sit in a straight line, in the circle. W and C don’t face each other or sits in the
the people sitting in the inner circle faces south and like same circle. B faces the person who sits immediate left of
same Perfume. The persons sitting in the outer circle faces D, who likes Boss Perfume. K and V face each other
north and likes different Perfume. neither of them likes Denver Perfume. D sits second to the
left of the one who likes Armani Perfume in the straight
line. C neither likes Armani or Park Avenue Perfume
while sitting in the circle. The person, who likes Gucci
Perfume, sits second to the left of the one who likes Fogg
Perfume in the straight line. J doesn’t like Denver Perfume
in the straight line.
21. What is the position of K with respect to U in the
Straight line?
(a) Second to the right
(b) Fourth to the right

Page 857 of 1334


Subscribe The Xpress Video Course & Mock Test Package for Bank & Insurance Exams
If there are any suggestions/ errors in our PDFs Feel Free to contact us via this email: admin@exampundit.in
Ultra Practice Bundle PDF
SBI Clerk/ RBI Assist. Mains – Reasoning
(c) Third to the left Directions (26-30): Study the following information
(d) Fourth to the left and answer the questions given below
(e) None of these
22. Which of the following statement is true regarding the Twelve persons M, N, O, P, Q, R, S, T, U, V, W and X
circular arrangement? were seated in rectangular shaped concentric walls as
(a) C and J face each other shown in Figure
(b) The one who likes Engage Perfume is an
immediate neighbour of one who likes Denver
Perfume.
(c) U likes Park Avenue Perfume and sits to the
immediate left of C
(d) The one who likes Fogg Perfume and sits to the
immediate left of V
(e) All are true
23. Who among the following likes Wild Stone Perfume
in the straight line?
(a) C
(b) The one who sits third to the right of J
(c) W
(d) J
(e) Cannot be determined
24. Four of the following five are alike in a certain way There were 12 boxes and only one person was seated on
thus forms a group with respect to circular each box, the persons were seated facing outside the
Arrangement. Find the one which doesn’t belong to centre. There were 6 box kept on the corners and 6 were
the group? kept along the side of the walls. The Box was numbered
(a) B – Fogg 1-12 as shown in the figure. Any person seated on the
(b) J – Boss corner of a wall can see the neighbouring persons seated
(c) U – Denver along the 2 neighbouring walls or the neighbouring
(d) D – Wild Stone corners (only when nobody is there along the
(e) V – Gucci neighbouring side of the wall). For example, person on
25. How many Person sits between the one who like chair 8 can see person on chair 9 and 7 but not 10.
Armani perfume and the one who like Engage in Similarly, any person seated on the side of the wall can
Straight Line? see the persons seated on the neighbouring corners of the
(a) 3 wall. For example, person on chair 3 can see the persons
(b) 4 on chair 2 and 4 only.
(c) 5
(d) 6 M's seat number was thrice as that of N's seat number. M
(e) 2 could not see anybody. O was 3rd to the right of N who

Page 858 of 1334


Subscribe The Xpress Video Course & Mock Test Package for Bank & Insurance Exams
If there are any suggestions/ errors in our PDFs Feel Free to contact us via this email: admin@exampundit.in
Ultra Practice Bundle PDF
SBI Clerk/ RBI Assist. Mains – Reasoning
was seated on one of the corners. P could see Q and W. R (d) M
was seated 3rd to the left of S. Sum of the seat numbers of (e) N
S and Q was equal to the seat number of T. T could not Directions (31-35): Study the following information
see anybody. U's seat number was one third of the V’s seat and answer the questions given below
number. W was seated third to the left of U. Six Cricketer P, Q, R, S, T and U seated around a
hexagonal Cricket Ground which was built in a way that
26. In which box number X is seated? there were 2 hexagons (inner and outer). The positions
(a) 11 were marked as shown below:
(b) 12
(c) 10
(d) 8
(e) 7
27. Four of the following five are alike in a certain way
thus forms a group. Find the one which doesn’t
belong to the group?
(a) U
(b) N
(c) P
(d) Q
(e) S Initially they were seated along the inner Ground faces
28. Total sum of the seat number of W and U is equal to outside the centre. The following information is known
whom? about their initial arrangement:
(a) 12 1. Two persons sitting between U and S whose seat
(b) 11 number was an even value
(c) 10 2. Q's seat number was one third as that of P’s seat
(d) 9 number.
(e) 8 3. Q was seated to the immediate right of T.
29. Who among the following sits 5th to the right of the T had a dice which had the numbers 1-6. Each person
one whose seat number is 7? rolled the dice only once alphabetically i.e. A rolled the
(a) R dice followed by B and so on. They changed positions as
(b) N per the number they rolled.
(c) U Conditions for their movements were as follows:
(d) O 1. The person 1st to roll an even value moved to position
(e) S 6 of the outer Ground. The person 2nd to roll an even value
30. Which of the followings sits at middle of the walls? shifted to his immediate Right. The person 3rd to roll an
(a) P even value moved to position 4 of the outer Ground.
(b) W 2. The person 1st to roll an odd value moved to position 1
(c) Q of the outer ground. The person 2nd to roll an odd value
Page 859 of 1334
Subscribe The Xpress Video Course & Mock Test Package for Bank & Insurance Exams
If there are any suggestions/ errors in our PDFs Feel Free to contact us via this email: admin@exampundit.in
Ultra Practice Bundle PDF
SBI Clerk/ RBI Assist. Mains – Reasoning
shifted to his immediate left. The person 3rd to roll an odd (b) R
value moved to position 3 of the outer ground. (c) Q
The numbers rolled by them were as follows: (d) U
1. P-5 (e) P
2. Q-3 35. If Q shift to the outer ground in position 2nd Then
3. R-6 Which of the following are the neighbours of Q?
4. S-1 (a) S, U
5. T-2 (b) S, P
6. U-4 (c) R, P
NOTE: If any required position is already taken by (d) U, P
another person during the movements, then the person (e) None
(who was to move) remains at his original position. Directions (36-40): Study the following information
31. After both movement how many Cricketer are in outer and answer the questions given below
hexagon?
(a) 1 Nine persons P, Q, R, S, T, U, V, W and X were seated
(b) 2 along two concentric triangular. There were 3 persons
(c) 3 seated along the inner smaller park such that there was 1
(d) 4 person at each of the corner and 6 were seated along the
(e) 5 outer larger park such that there was 1 person at each of
the corner and one person along each of the side. The
32. In final arrangement how many person don’t change persons seated along the outer park were seated facing
their position? towards the centre and the persons seated along the Inner
(a) None Park were seated facing away from the centre. The seats
(b) 5 were numbered as shown in the figure:
(c) 4
(d) 3
(e) 2
33. If the person sits at outer ground faces outside then
who sits third to the right of R?
(a) S
(b) T
(c) Q
(d) U
(e) P • P was seated to the Immediate right of the one whose
34. Four of the following five are alike in a certain way seat number was thrice as that of U's seat number
thus forms a group. Find the one which doesn’t belong • T's seat number was twice as that of S's seat number.
to the group? S was seated along one of the sides.
• R and T were seated at consecutive seat numbers.
(a) S

Page 860 of 1334


Subscribe The Xpress Video Course & Mock Test Package for Bank & Insurance Exams
If there are any suggestions/ errors in our PDFs Feel Free to contact us via this email: admin@exampundit.in
Ultra Practice Bundle PDF
SBI Clerk/ RBI Assist. Mains – Reasoning
• X's seat number was more than that of Q's seat 40. Who sits 4th to the right of the person whose seat
number. number is 6?
• Difference between R's seat number and Q's seat
number was 2. (a) S
• W was seated at one of the corners but was not (b) V
neighboring S. (c) Q
(d) U
36. In which seat number W is seated?
(e) P
(a) 9
Directions (41-45): Study the following information
(b) 4
(c) 5 and answer the questions given below
(d) 6
Sixteen persons are sitting around two circular tables as
(e) 7
one is inscribed in another one. All of them are facing
towards the center. Eight persons i.e. M, N, O, P, Q, R, S
37. Who faces the person who sits at seat number 7?
and T are sitting around the inner circular table. Eight
(a) X persons i.e. E, F, G, H, I, J, K and L are sitting around the
(b) P outer circular table.
(c) Q
Note- The persons of outer circular table are sitting
(d) U
exactly behind the persons sitting around the inner circular
(e) R
table. The persons of inner circular table are sitting exactly
38. Which of the following sits at outer triangle? ahead the persons sitting around the outer circular table.

(a) W P sits 3rd to the right of the one who sits ahead of E. L is
(b) X
(c) T
(d) R
(e) None of these

39. Which seat number is immediate neighbor of V?

(a) 1, 4
(b) 4, 6
(c) 2, 4
(d) 7, 9
(e) 5, 4

Page 861 of 1334


Subscribe The Xpress Video Course & Mock Test Package for Bank & Insurance Exams
If there are any suggestions/ errors in our PDFs Feel Free to contact us via this email: admin@exampundit.in
Ultra Practice Bundle PDF
SBI Clerk/ RBI Assist. Mains – Reasoning
not an immediate neighbor of J. F sits second to the left of (c) G
E. M is not an immediate neighbor of O. Q sits neither (d) I
ahead of E nor F. K sits 2nd to the left of L. Q and S are (e) J
immediate neighbors of T. G sits immediate right of E. R
sits immediate right of the person, who sits ahead of F. H 45. The person who is ahead of I is between which of the
sits behind of O. J sit 2nd to the right of H. following persons?

41. Who sits 5th to the right of the one who is behind M? (a) Q, M
(b) J, H
(a) L (c) T, S
(b) K (d) O, Q
(c) F (e) M, N
(d) E
Directions (46-50): Study the following information
(e) G and answer the questions given below
42. Who among the following seated opposite to N in
same circle? There are nine persons P, Q, M, R, S, T, U, V, W and X
sitting on the corners of two different polygons of which
(a) S one is square and other is pentagon. Both the polygon is
(b) M concentric such that Square is inside the pentagon. All the
(c) O persons are facing inside the polygon. S is sitting third to
(d) Q the right of X. X is exactly ahead or behind of R. W is
(e) T sitting third to the left of R. T is the immediate neighbour
of R. P is not the immediate neighbour of U. V sits on the
43. How many people are seated between the person who immediate left of Q. U is neither the immediate neighbour
is immediate left of the person who faces T and E (in of T nor X.
clock wise direction from E)?
46. Who among the following sits second to the right of
(a) 2 U after arrangement?
(b) 4
(c) 5 (a) Q
(d) 6 (b) R
(e) 3 (c) P
(d) T
44. Which of the following person behind the person who (e) V
sit 4th to the left of S?

(a) E
(b) H

Page 862 of 1334


Subscribe The Xpress Video Course & Mock Test Package for Bank & Insurance Exams
If there are any suggestions/ errors in our PDFs Feel Free to contact us via this email: admin@exampundit.in
Ultra Practice Bundle PDF
SBI Clerk/ RBI Assist. Mains – Reasoning
47. How many persons are sitting between R and W in 49. If all the persons sitting in outer polygon faces outside
final arrangement when counted to the right of W. than which of the following sitting second to the right
of T?
(a) 1
(b) 4 (a) W
(c) 3 (b) P
(d) 2 (c) U
(e) 5 (d) R
(e) None of these
48. Which of the following pair is immediate neighbor of
X? 50. If the entire person in outer polygon arranged in
alphabetical order from P in clock wise direction then
(a) S, Q how many person change their position? (Except P)
(b) R, Q
(c) U, W (a) 4
(d) None of these (b) 1
(e) S, V (c) 2
(d) 3
(e) None of these

Seating Concentric Diagrams – Answer and Explanation

Direction (1-5)
Explanation in detail:
25. Q sits 2nd to the left of N.
26. One of the immediate neighbors of W faces N.
27. U faces the one who sit immediate left of N

28. The person who faces P is an immediate neighbor of


W.

Page 863 of 1334


Subscribe The Xpress Video Course & Mock Test Package for Bank & Insurance Exams
If there are any suggestions/ errors in our PDFs Feel Free to contact us via this email: admin@exampundit.in
Ultra Practice Bundle PDF
SBI Clerk/ RBI Assist. Mains – Reasoning
29. There is only one person between Z and the person Final Arrangement
faces P
30. Z and V are opposite to each other.
31. Y is 2nd to the left of Z.
32. X is an immediate neighbor of V.

1. D
2. d
3. b
4. e
33. O and P is immediate neighbor of each other. 5. a
34. One person sits between M and R.
35. M and N is not immediate neighbor of each other. Direction (6-10)

Explanation in detail:

1. H likes neither Shimla nor Raipur and faces V.


2. V sits second to the left of W.
3. The person who likes Bhopal sits on the immediate left
of W.
4. The Persons who like Bhopal and Kolkata are
immediate neighbors.

Page 864 of 1334


Subscribe The Xpress Video Course & Mock Test Package for Bank & Insurance Exams
If there are any suggestions/ errors in our PDFs Feel Free to contact us via this email: admin@exampundit.in
Ultra Practice Bundle PDF
SBI Clerk/ RBI Assist. Mains – Reasoning
6. The person who likes Hyderabad faces the person who
likes Delhi.
7. E, who likes Chennai, faces the immediate neighbor
of the person who likes Mumbai.
8. The persons who like Hyderabad and Chennai sit in
the same Square but they are not immediate neighbors.
9. U, who does not like Mumbai, does not face E.

CASE 1

5. The persons who like Bhopal and Kolkata are


immediate neighbors, and one of them faces F, who
likes Shimla. (Here Two case for F)
CASE 1

CASE 2

CASE 2

10. H likes neither Shimla nor Raipur.


11. The person like Raipur and Delhi are in Separate
Square.

CASE 2 was eliminated (H not like Raipur)

Page 865 of 1334


Subscribe The Xpress Video Course & Mock Test Package for Bank & Insurance Exams
If there are any suggestions/ errors in our PDFs Feel Free to contact us via this email: admin@exampundit.in
Ultra Practice Bundle PDF
SBI Clerk/ RBI Assist. Mains – Reasoning
CASE1 7. a

8. e

9. c

10. b

Direction (11-15)

Explanation in detail:

1. L sits opposite to the one who sits second to the right


of Z and both of them are not sitting in the same
12. U does not like Mumbai. hexagon.
2. Z and the one who sits in the outer hexagon facing
each other. (From here it is clear that Z is in Inner
hexagon and faces outside)

Final Arrangement

3. U sits opposite to the person who sits to the


immediate left of X, who is an immediate neighbor
of L.
4. U faces the same direction as X, who is not an
immediate neighbor of the one who faces Z.

Two case for X

CASE 1

6. d

Page 866 of 1334


Subscribe The Xpress Video Course & Mock Test Package for Bank & Insurance Exams
If there are any suggestions/ errors in our PDFs Feel Free to contact us via this email: admin@exampundit.in
Ultra Practice Bundle PDF
SBI Clerk/ RBI Assist. Mains – Reasoning

CASE2 9. K and W are immediate neighbors and face the same


direction.
10. Y sits opposite to the person, who sits second to the
left of W.
11. Immediate neighbors of K faces opposite directions

CASE 2

5. M sits opposite of J, both are faces same direction


and does not sit in the same table.
6. J sits second to the right of N, who sits to the
immediate left of V.
7. N and V are facing the same direction as X.
8. J does not sit in the same hexagon as U.

CASE 1eliminated (M is immediate neighbor of Z. V


does not faces same direction as X)

CASE 2

Note: It is given in inner hexagon equal number of


person faces in and out. M and J faces outside

Page 867 of 1334


Subscribe The Xpress Video Course & Mock Test Package for Bank & Insurance Exams
If there are any suggestions/ errors in our PDFs Feel Free to contact us via this email: admin@exampundit.in
Ultra Practice Bundle PDF
SBI Clerk/ RBI Assist. Mains – Reasoning
12. Equal number of persons is facing the same Direction (16-20)
directions in the inner hexagon.
Explanation in detail:
1. T sits on any of the corner.
2. There are exactly two persons sitting between T and
S.
3. Four persons are sitting in the middle of the sides and
two persons are sitting on diagonally opposite
corners.
From the above information it clear diagram is like
this shown below.

13. The immediate neighbors of the one who faces the


person I are facing the opposite direction as W.

Final Arrangement

4. H sits second to the left of E.


5. Neither H nor E faces either S or T.
6. R is on the immediate left of the one who is facing E.
Two case for E
H and E Only sit at middle of the sides.
CASE 1

11. c
12. b
CASE 2
13. d (either 1 or 3 persons sit between them)
14. e (both are facing in same direction)
15. a
Page 868 of 1334
Subscribe The Xpress Video Course & Mock Test Package for Bank & Insurance Exams
If there are any suggestions/ errors in our PDFs Feel Free to contact us via this email: admin@exampundit.in
Ultra Practice Bundle PDF
SBI Clerk/ RBI Assist. Mains – Reasoning
CASE 2

7. A is facing N
8. S is not sitting adjacent to A.
From here case 1 is eliminated as (no place for A)
CASE 2

Final Arrangement

9. D is not opposite R in the outer rectangle.

CASE 2
16.d

17.e

18.a

19.E

20.b

Direction (21-25)
10. K is not facing T.
11. Between K and M there are as many persons as Explanation in detail:
between R and P.

Page 869 of 1334


Subscribe The Xpress Video Course & Mock Test Package for Bank & Insurance Exams
If there are any suggestions/ errors in our PDFs Feel Free to contact us via this email: admin@exampundit.in
Ultra Practice Bundle PDF
SBI Clerk/ RBI Assist. Mains – Reasoning
1. B is an immediate neighbour of W and sits in the
extreme end of the line.
2. W sits second from the extreme end of the line and
likes the Perfume liked by J in the circle.
3. J sits third to left of both B and the one who likes
Armani Perfume in the straight line.
4. W is not an immediate neighbour of J, who likes
Engage Perfume in the circle.

From here it is clear that W and J faces North direction


as per given instruction. Because perfumes liked by W
in circle and line are different. (The persons sitting in
the outer circle faces north and likes different Perfume)
5. D sits second to the left of the one who likes Armani
Line Arrangement
Perfume in the straight line.
6. U sits immediate right of K in the circle.
CASE 1
7. K and V faces each other

From here 2 case for K in circle

Line Arrangement

CASE 1
CASE 2

CASE 2
Circle Arrangement

Page 870 of 1334


Subscribe The Xpress Video Course & Mock Test Package for Bank & Insurance Exams
If there are any suggestions/ errors in our PDFs Feel Free to contact us via this email: admin@exampundit.in
Ultra Practice Bundle PDF
SBI Clerk/ RBI Assist. Mains – Reasoning
Circle Arrangement 8. The person who likes Denver faces the person who
sits immediate left of C in the circle.
CASE 1 9. J faces the person who sits to the immediate left in
the straight line.
10. B faces the person who sits immediate left of D, who
likes Boss Perfume

From we check the CASE 1 and CASE 2 of circle

CASE 1(J faces B which is wrong because B is not


neighbour of J in CASE 1 Line arrangement)

CASE 2

CASE 2 (The person who likes Denver faces the


person who sits immediate left of C in the circle)

For CASE 1 Line Arrangement (B facing south so


it is on inner circle)

Here it clear that CASE 1 and CASE 2 of Circle


wrong and B is in Outer circle.

CASE 1 of Line arrangement also eliminated

Page 871 of 1334


Subscribe The Xpress Video Course & Mock Test Package for Bank & Insurance Exams
If there are any suggestions/ errors in our PDFs Feel Free to contact us via this email: admin@exampundit.in
Ultra Practice Bundle PDF
SBI Clerk/ RBI Assist. Mains – Reasoning
For CASE 2 of Line Arrangement 16. C neither likes Armani or Park Avenue Perfume while
sitting in the circle.

Line Arrangement

CASE 2

11. The person who likes Denver faces the person who
sits immediate left of C in the circle.
12. J faces the person who sits to the immediate left in
the straight line.
13. B faces the person who sits immediate left of D, who Circle arrangement
likes Boss Perfume
CASE 1

CASE 2 of Circle Arrangement Eliminated (D is


immediate left of J so J have to face D but in CASE 2
of circle arrangement J faces U)

CASE 1

17. C neither likes Armani or Park Avenue Perfume


while sitting in the circle

14. The person, who faces K in the circle, likes Fogg


Perfumes in the straight line.
15. The person who likes Armani Perfume faces the
person who likes Wild Stone Perfume.
Page 872 of 1334
Subscribe The Xpress Video Course & Mock Test Package for Bank & Insurance Exams
If there are any suggestions/ errors in our PDFs Feel Free to contact us via this email: admin@exampundit.in
Ultra Practice Bundle PDF
SBI Clerk/ RBI Assist. Mains – Reasoning
Circle Final Arrangement

Line Final Arrangement

21.B

22.c

18. The person, who faces K in the circle, likes Fogg 23.d
Perfumes in the straight line.
19. The person, who likes Gucci Perfume, sits second to 24.a
the left of the one who likes Fogg Perfume in the
straight line. 25.b

CASE 2 Direction (26-30)

Explanation in detail: (1 case is missing, refer


attachement)

1. M's seat number was thrice as that of N's seat


number.
2. M could not see anybody
20. W sits second from the extreme end of the line and 3. O was 3rd to the right of N who was seated on one of
likes the Perfume liked by J in the circle the corners.
21. J doesn’t like Denver Perfume in the straight line.

CASE 2

Page 873 of 1334


Subscribe The Xpress Video Course & Mock Test Package for Bank & Insurance Exams
If there are any suggestions/ errors in our PDFs Feel Free to contact us via this email: admin@exampundit.in
Ultra Practice Bundle PDF
SBI Clerk/ RBI Assist. Mains – Reasoning

Final Arrangement
4. U's seat number was one third of the V’s seat
number.
5. W third to the left of U.
Now two case for U and V

U= 1, 2 not possible because we cannot place W.

26.A

27.a

28.d

29.E

30.D
6. Sum of the seat numbers of S and Q was equal to the
seat number of T. Direction (31-35)
7. T could not see anybody.
8. R was seated 3rd to the left of S. Explanation in detail:
9. P could see Q and W.
The following information is known about their initial
arrangement:
Page 874 of 1334
Subscribe The Xpress Video Course & Mock Test Package for Bank & Insurance Exams
If there are any suggestions/ errors in our PDFs Feel Free to contact us via this email: admin@exampundit.in
Ultra Practice Bundle PDF
SBI Clerk/ RBI Assist. Mains – Reasoning
1. Q's seat number was one third as that of P’s seat If any required position is already taken by another person
number. during the movements, then the person (who was to move)
2. Q was seated to the immediate right of T. remains at his original position.
3. Two persons sitting between U and S whose seat Condition for movement
number was an even value. 4. The person 1st to roll an even value moved to position
First we solve the inner Hexagon arrangement 6 of the outer Ground. The person 2nd to roll an even
value shifted to his immediate Right. The person 3rd to
roll an even value moved to position 4 of the outer
Ground.
First we move R then T and then U
From above table it is clear that 4 is not possible for S Because fist number of even rolled is 6 than 2 and last
4 in alphabetical order.
because U is not opposite to S. so S = 2, Q = 1, P = 3
New arrangement after First movement (Even value)
and T = 6
Final Inner arrangement is

T cannot move from their position as per note is given


Now we have to move cricketers of inner hexagon as 5. The person 1st to roll an odd value moved to position
per given condition 1 and 2. 1 of the outer ground. The person 2nd to roll an odd
value shifted to his immediate left. The person 3rd to
The numbers rolled by them were as follows:
roll an odd value moved to position 3 of the outer
1. P-5
ground.
2. Q-3
3. R-6 First we move P then Q and then S
4. S-1 Because fist number of Odd rolled is 5 than 3 and
5. T-2 last 1 in alphabetical order
6. U-4
Each person rolled the dice only once alphabetically i.e. A New arrangement after Second movement (odd
rolled the dice followed by B and so on. They changed value)
positions as per the number they rolled
Page 875 of 1334
Subscribe The Xpress Video Course & Mock Test Package for Bank & Insurance Exams
If there are any suggestions/ errors in our PDFs Feel Free to contact us via this email: admin@exampundit.in
Ultra Practice Bundle PDF
SBI Clerk/ RBI Assist. Mains – Reasoning

31. d

32. A

33. a

34. c

35. b

Summary table (newly formed by myself)


Direction (36-40)
P=5 (1st odd) P moves position 1 of outer table
Q=3 (2nd odd) Q moves to the immediate left Explanation in detail
R=6 (1st even) R moves position 6 of outer table
S=1 (3rd odd) S moves position 3 of outer table 1. T's seat number was twice as that of S's seat number.
T=2 (2nd even) T moves to the immediate right S was seated along one of the sides
U=4 (3rd even) U moves position 4 of outer table
Two CASE for S
Final Arrangement (figure modified)

CASE 1

Note: No one remains at original position since the


condition is given during movements i.e. during
CASE 2
movements no one occupied others place.
Page 876 of 1334
Subscribe The Xpress Video Course & Mock Test Package for Bank & Insurance Exams
If there are any suggestions/ errors in our PDFs Feel Free to contact us via this email: admin@exampundit.in
Ultra Practice Bundle PDF
SBI Clerk/ RBI Assist. Mains – Reasoning

2. P was seated to the Immediate right of the one whose


seat number was thrice as that of U's seat number.
CASE 2b
CASE1

CASE 2c

For CASE 2

3. R and T were seated at consecutive seat numbers.


4. X's seat number was more than that of Q's seat
CASE 2a number.

Page 877 of 1334


Subscribe The Xpress Video Course & Mock Test Package for Bank & Insurance Exams
If there are any suggestions/ errors in our PDFs Feel Free to contact us via this email: admin@exampundit.in
Ultra Practice Bundle PDF
SBI Clerk/ RBI Assist. Mains – Reasoning
5. Difference between R's seat number and Q's seat
number was 2.

From here Case 1 Eliminated (R=5 so the difference


between R and Q is 2 not possible)

CASE 2b Eliminated (R and Q combination are 9 and


7 here difference is 2 but X seat number always less
the Q)

CASE 2c eliminated (R=9 so the difference between R


and Q is 2 not possible)

CASE 2a R =7, Q =5 ➔R –Q = 7-5 => 2


Final Arrangement

6. W was seated at one of the corners but was not


neighboring S
7. X's seat number was more than that of Q's seat
number.

CASE 2a 36. a

37. d

38. b

39. c

40. e

Direction (41-45)

Page 878 of 1334


Subscribe The Xpress Video Course & Mock Test Package for Bank & Insurance Exams
If there are any suggestions/ errors in our PDFs Feel Free to contact us via this email: admin@exampundit.in
Ultra Practice Bundle PDF
SBI Clerk/ RBI Assist. Mains – Reasoning
Explanation in detail 8. K sits 2nd to the left of L.
9. Q and S are immediate neighbors of T.
1. P sits 3rd to the right of the one who sits ahead of E
2. F sits second to the left of E CASE 1
3. G sits immediate right of E.
4. R sits immediate right of the person, who sits ahead of
F.

CASE 2

5. H sits behind of O.
6. J sit 2nd to the right of H.
Two cases for H
CASE 1

10. M is not an immediate neighbor of O.

CASE 2 was eliminated (M is not an immediate neighbor


of O)

CASE 1
CASE 2

7. L is not an immediate neighbor of J. Final Arrangement


Page 879 of 1334
Subscribe The Xpress Video Course & Mock Test Package for Bank & Insurance Exams
If there are any suggestions/ errors in our PDFs Feel Free to contact us via this email: admin@exampundit.in
Ultra Practice Bundle PDF
SBI Clerk/ RBI Assist. Mains – Reasoning

41.b
2. X is exactly ahead or behind of R.
42. E 3. S is sitting third to the right of X.
4. T is the immediate neighbor of R.
43. e
Now here two case for T in CASE 1
44. b
CASE 1a
45. d

Direction (46-50)

Explanation in detail

1. W is sitting third to the left of R.

Two case for R (either R is in Pentagon or in square).

CASE 1

CASE 1b

CASE 2

Page 880 of 1334


Subscribe The Xpress Video Course & Mock Test Package for Bank & Insurance Exams
If there are any suggestions/ errors in our PDFs Feel Free to contact us via this email: admin@exampundit.in
Ultra Practice Bundle PDF
SBI Clerk/ RBI Assist. Mains – Reasoning

CASE 2 Final Arrangement

5. P is not the immediate neighbour of U.


6. V sits on the immediate left of Q.
7. U is neither the immediate neighbour of T nor X.
46.c
From here Case 1a eliminated (If we fix the position of
V and Q then P and U always together which is not
47.d
possible according to statements given)
48.e
Case 2 also eliminated (In this case U is always
neighbour of either X or T which is not possible) 49.a

CASE 1b 50.A

Download Seating Arrangement Practice Questions PDF

Get More Reasoning Practice Questions PDF


Page 881 of 1334
Subscribe The Xpress Video Course & Mock Test Package for Bank & Insurance Exams
If there are any suggestions/ errors in our PDFs Feel Free to contact us via this email: admin@exampundit.in
Ultra Practice Bundle PDF
SBI Clerk/ RBI Assist. Mains – Reasoning

Seating Movement Based


Directions (1-5): Answer the following questions based Then they shifted to table in which there are eight vacant
on the information given below: seats and the position of vacant seats are numbered from
Ten persons A, B, C, D, E, F, G, H, I and J are sitting 1 to 8 in clockwise direction.
around a rectangular table which consists of 10 seats but
not necessarily in the same order. All are facing outside
the table. Two person sit at the longer side of the table and
one person sit at the shorter side and four persons are
sitting at corner of the table. F,, who is sitting at corner is
sitting between C and E who is the neighbor of B. J and G
are neighbors but none of them is sitting near to B. D is
sitting at a gap of three places from I, who sits second to Conditions:
the left of F. A is sitting second to the left of J. 1. The first person who chooses yellow color ball, moves
Now they start playing a game where each of them has to to position 3 while second person moves to position-5,
choose a ball of different colors i.e. Yellow, Black, White facing the same direction as current direction.
and Green. All of them chooses a ball and changes their 2. If the ball is Black colour, the person who chooses first
places according to the given conditions. moves to position 4 while the second person moves to
position-6.facing the opposite direction of current
direction.

Page 882 of 1334


Subscribe The Xpress Video Course & Mock Test Package for Bank & Insurance Exams
If there are any suggestions/ errors in our PDFs Feel Free to contact us via this email: admin@exampundit.in
Ultra Practice Bundle PDF
SBI Clerk/ RBI Assist. Mains – Reasoning
3. If the ball is white colour, the person who chooses first (a) B
moves to position 1 while the second person moves to (b) G
position 7 facing the same direction of current direction. (c) C
4. If the ball is green colour, the person who chooses first (d) H
moves to position 2 while second person moves to (e) None of these
position 8, facing the opposite direction of current 3. After all persons changed their places based on the
direction. above conditions, then how many persons are facing
Chosen: away from the center?
1. D chooses a ball of green colour. (p) Two
2. B chooses a ball of green colour. (q) One
3. H chooses a ball of yellow colour. (r) Five
4. A chooses a ball of yellow colour. (s) Three
5. F chooses a ball of Black colour. (t) None of these
6. G chooses a ball of Black colour. 4. After all persons changed their places based on the
7. C chooses a ball of white colour. above conditions, who sits exactly between the C and
8. E chooses a ball of white colour. E from left of E?
All the person choose the ball in a same order as given (a) B
above. (b) D
1. After all persons changed their places based on the (c) G
above conditions, how many persons are sit between C (d) F
and F? (e) None of these
(a) Two 5. Who sits to the immediate right of F in initial
(b) Four condition?
(c) One (a) J
(d) Three A
(e) None of these (b) E
2. After all persons changed their places based on the (c) H
above conditions, then who among the following is (d) None of these
sitting to the immediate right of D?

Page 883 of 1334


Subscribe The Xpress Video Course & Mock Test Package for Bank & Insurance Exams
If there are any suggestions/ errors in our PDFs Feel Free to contact us via this email: admin@exampundit.in
Ultra Practice Bundle PDF
SBI Clerk/ RBI Assist. Mains – Reasoning
Directions (6-10): Answer the questions based on the
information given below
There are eight persons Deval, Anurag, Prashant, Hemant,
Abhinav, Abhishek, Satyam and Ravi are sitting around a
circular table having eight seats. Four of them are facing
the center while four are facing opposite to the center.
Anurag sits third to the right of Satyam. Hemant is second
to the left of Anurag. Ravi is adjacent to Satyam but not
opposite to Anurag. Abhinav and Ravi faces in same
Conditions:
direction. Abhinav is immediate left of Hemant but not
1. If the box containing a 2-numbered book, the person
adjacent to Anurag. Hemant and Abhishek face in same
who choses first moves to Position-6 facing same
direction. Prashant faces away from the center but does
direction of current direction, then the second person who
not sit immediate right of Hemant. Abhishek does not sit
chooses the box containing either 1 or 3-numbered book
adjacent to Anurag. Deval and Satyam faces in same
moves to Position-4, facing the same direction of current
direction.
direction.
Now they start playing a game where each of them has to
2. If the box containing a 7-numbered book, person
choose a box containing different numbered book. It can
remains on the same position facing opposite direction.
be an odd or even i.e. 1, 2, 3, 4, 6 and 8. Two separate
3. If the box containing 8-numbered book, the person who
boxes each containing either a 7-numberd or 10-numbered
choses first moves to Position 5 facing same direction,
books were also placed along with these boxes. Prashant
then the second person, who chooses the box containing
chooses a box and changes his place according to the
6-numbered moves to Position 3, facing same direction of
given conditions. Similarly, others choose boxes
current direction.
according to the conditions given below:
4. If the box containing a 10-numbered book, person
Then they shift to square table where there are eight seats
remains on the same position facing same direction of
and the position of seats are numbered from 1 to 8 in anti-
current direction.
clockwise direction.
Chosen:
1. Prashant chooses a box containing an even numbered
book other than 4, 6 and 8 and it is also a prime number.
2. Abhinav chooses a box containing 10-numbered book.
Page 884 of 1334
Subscribe The Xpress Video Course & Mock Test Package for Bank & Insurance Exams
If there are any suggestions/ errors in our PDFs Feel Free to contact us via this email: admin@exampundit.in
Ultra Practice Bundle PDF
SBI Clerk/ RBI Assist. Mains – Reasoning
3. Abhishek chooses a box containing an even numbered 8. After all persons changed their places based on the
book other than 2, 4 and 6. above conditions, then how many persons facing away
4. Hemant chooses a box containing 7-numbered book. from the center in square table?
5. Satyam chooses a box containing odd numbered book (a) Two
but less than 5. (b) One
6. Anurag chooses a box containing 7-numbered book. (c) Five
7. Deval chooses a box containing 10-numbered book. (d)Three
8. Ravi chooses a box containing an even numbered other (e) None of these
than 2, 4, 8 and 10. 9. After changing their places based on the above
All the person chooses the boxes in the same sequence conditions, who sits second to the right of Satyam?
as given above. (a) Abhishek
6. After changing their places based on the above (b) Ravi
conditions, how many persons are sit in the circular (c) Deval
table? (d) Prashant
(a) Two (e) None of these
(b) Four 10. Who sits immediate right of Abhishek in initial
(c) One condition?
(d) Three (a) Prashant
(e) None of these Abhinav
7. After changing their places based on the above (b) Deval
conditions, then who among the following is sitting to (c) Anurag
the immediate right of Deval? (d) None of these
(a) Anurag Directions (11-15): Study the information carefully and
(b) Abhinav answer the questions given below.
(c) Prashant Eight people – P, Q, R, S, T, U, V and W are sitting around
(d) Hemant a circular table facing the center, but not necessarily in the
(e) None of these same order. All of them have cars of different companies
i.e. Ford, TATA, Renault, Maruti, Kia, Toyota, BMW and
Audi.

Page 885 of 1334


Subscribe The Xpress Video Course & Mock Test Package for Bank & Insurance Exams
If there are any suggestions/ errors in our PDFs Feel Free to contact us via this email: admin@exampundit.in
Ultra Practice Bundle PDF
SBI Clerk/ RBI Assist. Mains – Reasoning
Only one person is sitting between T and the one who has Conditions:
Maruti car. W neither has Toyota nor Maruti car. The one 1. If someone rolled a dice and outcome is 4, the person
who has a Ford car is sitting exactly opposite to the one who rolled the dice first moves to position 3, then the
who has a BMW car. Two persons are sitting between V second person who gets the same outcome moves to
and the one who has a Ford car. V does not have a Renault position 7.
car. Three persons are sitting between Q and the one who 2. If someone rolled a dice and outcome is 1, the person
has an Audi car. Q neither has Renault nor BMW car. R who rolled the dice first moves to position 2, then the
neither has Maruti nor Audi car. P is sitting third to the left second person who gets the same outcome moves to
of Q. The one who has TATA car is sitting fifth to the left position 8.
of W. The one who has a Toyota car is not an immediate 3. If someone rolled a dice and outcome is 6, the person
neighbour of Q. One person is sitting between R and the who rolled the dice first moves to position 1, then the
one who has a BMW car. The person who has Renault car second person who gets the same outcome moves to
is sitting third to the right of R. U is sitting fourth to the position 5.
left of the one who has Renault car. 4. If someone rolled a dice and outcome is 3, the person
Each of them rolled a dice only once. After each roll, there who rolled the dice first moves to position 4, then the
were some movements based on certain rules. Then they second person who gets the same outcome moves to
shifted to a table in which there are eight seats and the position 6.
position of seats are numbered from 1 to 8 in clockwise Chosen:
direction. 1. The person who have Toyota rolled a dice and outcome
is 1.
2. The person who have TATA rolled a dice and outcome
is 3.
3. The person who have Ford rolled a dice and outcome is
4.
4. The person who have Kia rolled a dice and outcome is
6.
5. The person who have Maruti rolled a dice and outcome
is 4.

Page 886 of 1334


Subscribe The Xpress Video Course & Mock Test Package for Bank & Insurance Exams
If there are any suggestions/ errors in our PDFs Feel Free to contact us via this email: admin@exampundit.in
Ultra Practice Bundle PDF
SBI Clerk/ RBI Assist. Mains – Reasoning
6. The person who have Renault rolled a dice and outcome (e) None of these
is 3. 14. After all persons changed their places based on the
7. The person who have BMW rolled a dice and outcome above conditions, who sits on the shortest side of the
is 1. table?
8. The person who have Audi rolled a dice and outcome is (a) T
6. (b) Q
All the person rolled the dice in the same sequence as (c) W
given above. (d) P
11. After all persons changed their places based on the (e) None of these
above conditions, how many persons are sit between W 15. In initial condition R has which of the following
and T from anticlockwise direction of W? car?
(a) Two (a) Kia
(b) Four (b) Ford
(c) One (c) Toyoto
(d) Three (d) Maruti
(e) None of these (e) None of these
12. After changing their places based on the above Directions (16-20): Study the following information
conditions, then who among the following sits on carefully and answer the givenquestions:
position-5? Eight people A, B, C, D, E, F, G and H are sitting in a
(a) T circular table. Some of them are facing center and some of
(b) V them facing outside the center but not necessarily in the
(c) Q sameorder.
(d) P C sits second to the right of F. One person sits between C
(e) None of these and D. E sits third to the right of G. E face opposite
13. In initial arrangement who has Maruti car? direction of C. E is not an immediate neighbor of D and F.
(a) W G sits second to the left of A, who sits second to the right
(b) P of B. B and A are not immediate neighbors of D. H and D
(c) V sit immediate right to each other. C faces the same
(d)S direction as A. E faces the center.

Page 887 of 1334


Subscribe The Xpress Video Course & Mock Test Package for Bank & Insurance Exams
If there are any suggestions/ errors in our PDFs Feel Free to contact us via this email: admin@exampundit.in
Ultra Practice Bundle PDF
SBI Clerk/ RBI Assist. Mains – Reasoning
They decided to give test of reasoning subject. They 16. After all persons changed their places based on the
obtain different rank in test. Each person obtain different above conditions, who sits fourth to the left of E?
rank from 1-8 such that A got 1 rank and the rest of the (a) D
numbers were given to the persons in the clockwise (b) F
direction from A (in ascending order). Then they shift to (c) H
linear table in which there are eight seats and all of them (d) A
are facing north direction. (e) None of these
17. After changing their places based on the above
conditions, then who got 7th rank?
(a) A
(b) H
Conditions:
(c) C
1. The person who got 6 rank sits third from extreme
(d) G
left end.
(e) None of these
2. The person who got 2 rank sits third to the right of
18. In initial arrangement who among the following
D.
faces away from centre?
3. The person who got 5 rank sits fourth to the left of
(a) H
F.
(b) B
4. The person who got 3 rank sits at extreme end but
(c) F
not an immediate neighbor of H.
(d) G
5. The person who got 7 rank sits fourth to the left of
(e) None of these
B.
19. After all persons changed their places based on the
6. The person who got 1 rank sits immediate left of
above conditions, who among the following sits second
F.
from right side of the linear table?
7. The person who got 8 rank sits second to the left
(a) D
of D.
(b) G
8. The person who got 4 rank sits second to the right
(c) H
of A.
(d) E
All the person got the rank in the same sequence as
(e) None of these
given above.
Page 888 of 1334
Subscribe The Xpress Video Course & Mock Test Package for Bank & Insurance Exams
If there are any suggestions/ errors in our PDFs Feel Free to contact us via this email: admin@exampundit.in
Ultra Practice Bundle PDF
SBI Clerk/ RBI Assist. Mains – Reasoning
20. After all persons changed their places based on the Then they shift to table on which there are eight seats and
above conditions, how many persons are sit between G the position of seats are numbered from 1 to 8 in clockwise
and E? direction which is given below:
(a) Two
(b) Four
(c) One
(d) Three
(e) None of these
Directions (21-25): Read the following information
and answer the questions given:
Eight persons S, T, U, V, W, X, Y and Z - are sitting
Conditions:
around a rectangular table in such a way that four of them
1. If the ball has 2 printed on it, the person who
are sitting at four corners of the table while the other four
chooses first moves to position 3 facing the
sit in the middle of the sides. Those who sit in the middle
opposite direction of current direction.
of the sides are facing outside the center while those who
2. The person who chooses the ball which has either
sit at the four corners faces towards the center. There are
5 or 9 printed on it then, person moves to position-
only two persons sit between X and Z. S sits second to the
5 first then to position 7 facing the same direction.
right of X. Z is not an immediate neighbor of S. V sits on
3. If the ball has 7 printed on it, then person moves to
the middle side of the table. U sits immediate left to the V.
position-6 facing the same direction.
T sits immediate right to the Y. W sits second to the left
4. If the ball has 3 printed on it, the person who
of T. Y sits one the corner side of the table.
chooses first moves to position 4 facing the same
Now they start playing a game where each of them has to
direction of current direction.
choose a ball from a bag full of balls on which different
5. The person who chooses the ball which has 6
numbers are printed. It can be an odd or an even number
printed on it moves to position-2 facing the same
i.e. 1, 2, 3, 4, 5, 6, 7 and 9. S chooses a ball and changes
direction.
his place according to the given conditions. Similarly,
6. The person who chooses the ball which has 1
others choose balls given in the condition below:
printed on it moves to position-1 facing the same
direction.

Page 889 of 1334


Subscribe The Xpress Video Course & Mock Test Package for Bank & Insurance Exams
If there are any suggestions/ errors in our PDFs Feel Free to contact us via this email: admin@exampundit.in
Ultra Practice Bundle PDF
SBI Clerk/ RBI Assist. Mains – Reasoning
7. The person who chooses the ball which has 4 (b) second to the right
printed on, moves to position-8 facing the same (c) third to the left
direction. (d) Immediate left
Chosen: (e) None of these
1. W chooses a ball on which the Even number are printed 23. In initial arrangement who among the following
other than 4 and 6 is printed. sits third to the right of X?
2. X chooses a ball on which 7 is printed. (a) V
3. The person immediate right of Y chooses a ball on (b) Z
which 5 are printed. (c) W
4. Y chooses a ball on which 3 is printed. (d) Y
5. Z chooses a ball on which Even number are printed (e) None of these
other than 2 and 6 is printed. 24. After all persons changed their places based on the
6. U chooses a ball on which 1 is printed. above conditions, who among the following sits second
7. V chooses a ball on which Even number are printed from right side of the linear table?
other than 2 and 4 is printed. (a) D
8. S chooses a ball on which 9 is printed. (b) G
All the person chooses the ball in the same sequence as (c) E
given above. (d) H
21. After all persons changed their places based on the (e) None of these
above conditions, who sits second to the left of X? 25. After all persons changed their places based on the
(a) Y above conditions, how many persons are sit between G
(b) V and E?
(c) W (a) Two
(d) Z (b) Four
(e) None of these (c) One
22. After changing their places based on the above (d) Three
conditions, What is the position of Y with respect to (e) None of these
W? Directions (26-30) : Answer the questions based on the
(a) Fourth to the left information given below:

Page 890 of 1334


Subscribe The Xpress Video Course & Mock Test Package for Bank & Insurance Exams
If there are any suggestions/ errors in our PDFs Feel Free to contact us via this email: admin@exampundit.in
Ultra Practice Bundle PDF
SBI Clerk/ RBI Assist. Mains – Reasoning
Eight friends – A, B, C, D, E, F, G and H are sitting around person, who also draw heart moves to position 6 facing
a rectangular table, but not necessarily in the same order, the same direction
five of them are facing outside and sitting adjacent to each 3. If the card drawn is diamond, the person who draws first
other. G does not face outside and sits middle side of the moves to position 1 facing same direction, then the second
table. B who is facing center is seventh to the left of E. person, who also draw diamond moves to position 4 and
There are four people sitting between G and H and all of direction remains the same
them are facing outside. H faces outside the table. D is 4. If the card drawn is club, the person who draws first
sitting to the immediate left of C. G and A have one person moves to position 2 facing same direction, then the second
between them. C is not the immediate neighbour of G and person, who also draw club moves to position 7 and
A. direction remains the same.
Now they start playing cards game. They shuffled a pack Draws:
of cards. Then they shifted to table in which there are eight 1. B draws queen of Club
vacant seats and the position of vacant seats are numbered 2. A draws Jack of Club
from 1 to 8 in anti-clockwise direction. 3. E draws 2 of Spade
4. C draws 7 of Spade
5. D draws a King of Diamond
6. H draws 8 of Diamond
7. G draws 6 of Heart
8. F draws Ace of Heart
All the person draws the card in the same sequence as
given above.
26. After all persons changed their places based on the
Conditions: above conditions, who sits sixth to the left of B?
1. If the card drawn is spade, the person who draws first (a) H
moves to position 5 facing same direction of current (b) A
direction, then the second person who also draw spade (c) D
moves to position 8facing same direction of the current (d) F
direction (e) None of these
2. If card drawn is heart, the person who draws first moves
to position 3 facing same direction, then the second
Page 891 of 1334
Subscribe The Xpress Video Course & Mock Test Package for Bank & Insurance Exams
If there are any suggestions/ errors in our PDFs Feel Free to contact us via this email: admin@exampundit.in
Ultra Practice Bundle PDF
SBI Clerk/ RBI Assist. Mains – Reasoning
27. After changing their places based on the above (d) Three
conditions, What is the position of G with respect to A? (e) None of these
(a) Fourth to the left Directions (31-35): Answer the questions based on the
(b) second to the right information given below:
(c) third to the left There are eight persons Virat, Dhoni, Rohit, Dhawan,
(d) Immediate left Suresh, Mohit, Devang and Hemant are sitting around a
(e) None of these circular table having eight seats. All of them are facing the
28. In initial arrangement four of the following five are center. Each of them likes different subjects i.e. History,
alike in a certain way and hence form a group. Which Reasoning, Quant, GA, Computer, English, Economics
of the following does not belong to the group?? and Geography but not necessarily in the same order.
(a) G Virat sits third to the right of the person who likes
(b) A Geography. Virat does not like Quant. Suresh is not an
(c) C immediate neighbour of Hemant. Rohit sits on the
(d) D immediate right of the person who likes Quant. Neither
(e) F Virat nor Hemant likes Computer or Reasoning. The
29. After all persons changed their places based on the person who likes Computer is not an immediate neighbour
above conditions, who among the following sits second of the person who likes Geography. The person who likes
to the right of C? History sits second to the left of Suresh. The person who
(a) B likes History is an immediate neighbour of both the
(b) G persons who like English and Economics. Person who
(c) H likes Economics sits third to the right of Dhoni. Dhoni
(d) F does not like Computer. Mohit is not an immediate
(e) None of these neighbour of Virat. Devang is not an immediate neighbour
30. After all persons changed their places based on the of the person who likes History. Only two people sit
above conditions, how many persons are sit between D between the person who likes Geography and Hemant.
and E? The persons who like Computer and the Reasoning are
(a) Two immediate neighbours of each other.
(b) Four Now they start playing a game where each of them has to
(c) One choose a ball from a bag full of balls on which different

Page 892 of 1334


Subscribe The Xpress Video Course & Mock Test Package for Bank & Insurance Exams
If there are any suggestions/ errors in our PDFs Feel Free to contact us via this email: admin@exampundit.in
Ultra Practice Bundle PDF
SBI Clerk/ RBI Assist. Mains – Reasoning
alphabets are printed. It can be a Vowel or a Consonant the same direction, then the person who chooses the ball
viz. E, B, G, I, C, O, D and F. Suresh chooses a ball and which has B printed on it moves to position-1 facing the
changes his place according to the given conditions. same direction.
Similarly others choose balls given in the condition 4. If the box contains I, person remains on the same
below: position facing same direction of current direction.
Then they shift to square table in which there are eight Chosen:
vacant seats and the position of vacant seats are numbered 1. The person who likes Quant chooses a ball on which
from 1 to 8 in anti-clockwise direction. the vowel other than O and I is printed.
2. Dhoni chooses a ball on which F is printed.
3. The person immediate right of Virat chooses a ball on
which the consonant other than B, C and F is printed.
4. Rohit chooses a ball on which C is printed.
5. The person who likes History chooses a ball on which
the vowel other than E and O is printed.
6. Virat chooses a ball on which F is printed.
7. Mohit chooses a ball on which the vowel other than E
and I is printed.
8. The person who likes Computer chooses a ball on which
Conditions:
D is printed.
1. If the ball has E printed on it, the person who chooses
All the person chooses the balls in the same sequence
first moves to position 3 facing the opposite direction of
as given above.
current direction and the person who chooses the ball
31. After all persons changed their places based on the
which has either G or D printed on it moves to position-5
above conditions, who sits on the position-7 on square
first then to position 7 facing the same direction.
table?
2. If the ball has F printed on it, the person remains on the
(a) Devang
same position facing opposite direction.
(b) Mohit
3. If the ball has C printed on it, the person who chooses
(c) Suresh
first moves to position 4 facing the same direction of
(d) Rohit
current direction, then the second person who chooses the
(e) None of these
ball which has O printed on it moves to position-2 facing

Page 893 of 1334


Subscribe The Xpress Video Course & Mock Test Package for Bank & Insurance Exams
If there are any suggestions/ errors in our PDFs Feel Free to contact us via this email: admin@exampundit.in
Ultra Practice Bundle PDF
SBI Clerk/ RBI Assist. Mains – Reasoning
32. After changing their places based on the above (e) None of these
conditions, what is the position of Mohit with respect Directions (36-40): Answer the questions based on the
to who likes Quant? information given below:
(a) Fourth to the left Eight persons J, K, L, M, N, O, P and Q are seating in a
(b) second to the right row, some are facing north and some are facing south but
(c) third to the left not necessarily in the same order. J sits second to the left
(d) Immediate right of L. O sits second to the right of J. No one sits to the left
(e) None of these of O. As many persons sit to the right of J as sit to the left
33. In initial arrangement who among the following of K. Q doesn’t face south. N sits second to the left of the
sits second to the right of who likes GA? one who sits third to the right of P. Q is not an immediate
(a) Hemant neighbour of P. N and M face opposite direction with
(b) Rohit respect to each other. M face same direction as Q faces.
(c) Dhoni Now they start playing cards game. They shuffled a pack
(d) Dhawan of cards. Then they shift to table in which there are eight
(e) None of these seats and the position of seats are numbered from 1 to 8 in
34. After all persons changed their places based on the clockwise direction.
above conditions, who among the following sits third
to left of Dhawan?
(a) Devang
(b) Hemant
(c) Suresh
(d) Mohit
(e) None of these
35. After all persons changed their places based on the
above conditions, who sits in the circular table?
(a) Dhoni
(b) Rohit
(c) Mohit
(d) Devang

Page 894 of 1334


Subscribe The Xpress Video Course & Mock Test Package for Bank & Insurance Exams
If there are any suggestions/ errors in our PDFs Feel Free to contact us via this email: admin@exampundit.in
Ultra Practice Bundle PDF
SBI Clerk/ RBI Assist. Mains – Reasoning
1. If card drawn is queen of spade, the person who draws 36. After all persons changed their places based on the
moves to position 3. above conditions, how many persons sit between Q and
2. If card drawn is queen of diamond, the person who N?
draws moves to position 7. (a) One
3. If card drawn is queen of club, the person who draws (b) Two
moves to position 5. (c) Three
4. If card drawn is queen of Heart, the person who draws (d) Four
moves to position 4. (e) None of these
5. If card drawn is king of Spade, the person who draws 37. After changing their places based on the above
moves to position 1. conditions, who sits two place away from P in
6. If card drawn is king of Diamond, the person who draws clockwise direction?
moves to position 8. (a) J
7. If card drawn is king of Club, the person who draws (b) Q
moves to position 6. (c) L
8. If card drawn is king of Heart, the person who draws (d) O
moves to position 2. (e) None of these
Draws: 38. In initial arrangement who among the following
1. K draws queen of Club sits second to the left of Q?
2. J draws king of Club (a) N
3. N draws queen of Spade (b) L
4. L draws king of Spade (c) J
5. M draws King of Diamond (d) M
6. Q draws queen of Diamond (e) None of these
7. P draws king of Heart 39. After all persons changed their places based on the
8. O draws queen of Heart above conditions, who among the following sits
All the person draws the card in the same sequence as immediate next to N?
given above. (a) M, P
(b) L, M
(c) J, K

Page 895 of 1334


Subscribe The Xpress Video Course & Mock Test Package for Bank & Insurance Exams
If there are any suggestions/ errors in our PDFs Feel Free to contact us via this email: admin@exampundit.in
Ultra Practice Bundle PDF
SBI Clerk/ RBI Assist. Mains – Reasoning
(d) O, P
(e) None of these
40. After all persons changed their places based on the
above conditions, who sits on the position-5 on table?
(a) K
(b) M
(c) J
(d) Q
(e) None of these
Directions (41-45) : Answer the questions based on the
Conditions:
information given below:
1. If the card drawn is spade, the person who draws first
There are eight friends Ankit, Ravi, Rajan, Rahul, Piyush,
moves to position 5 facing opposite direction of current
Satyam, Sanjay and Kartik are sitting around a circular
direction, then the second person who also draw spade
table on which some facing towards the center and some
moves to position 6, similarly 7 and 8.
are facing outside the center but not necessarily in the
2. If card drawn is heart, person remains on the same
same order. Ankit sits second to left of Rajan and Piyush
position facing same direction.
sits third to right of Ankit. only three persons are sitting
3. If the card drawn is diamond, the person who draws first
between Piyush and Ravi. Piyush facing opposite
moves to Position 1 facing same direction, then the second
direction as Kartik. Kartik sits immediate right of Rajan.
person, who also draw diamond moves to Position 2,
Rahul sits second to right of Satyam who sits third to the
similarly Position 3.
left of Kartik. Ankit and Rajan are facing opposite
4. If card drawn is club, person remains on the same
direction. Rajan sits third to right of Sanjay and both
position facing opposite direction of current direction.
facing same direction. Rahul and Ravi are facing same
Draws:
direction as facing Satyam. Piyush facing inside the circle.
1. Ravi draws queen of Club
Now they start playing cards game. They shuffled a pack
2. Ankit draws Jack of Club
of cards. Then they shift to square table in which there are
3. Piyush draws 3 of Spade
eight seats and the position of seats are numbered from 1
4. Rahul draws a King of Diamond
to 8 in anti-clockwise direction.
5. Satyam draws Ace of Heart
6. Sanjay draws 9 of Heart

Page 896 of 1334


Subscribe The Xpress Video Course & Mock Test Package for Bank & Insurance Exams
If there are any suggestions/ errors in our PDFs Feel Free to contact us via this email: admin@exampundit.in
Ultra Practice Bundle PDF
SBI Clerk/ RBI Assist. Mains – Reasoning
7. Kartik draws 7 of Diamond 44. After all persons changed their places based on the
8. Rajan draws 4 of Spade above conditions, who among the following is sitting at
All the person chooses the card in the same squence as the Position 2 of the square?
given above. (a) Ravi
41. After all persons changed their places based on the (b) Satyam
above conditions, how many persons are remains in (c) Sanjay
the circle? (d) Kartik
(a) Two (e) None of these
(b) Four 45. After all persons changed their places based on the
(c) One above conditions, then who among the following is
(d) Three sitting to the immediate left of Piyush?
(e) None of these (a) Sanjay
42. After all persons changed their places based on the Rajan
above conditions, then who among the following is (b) Rajan
sitting to the immediate right of Satyam? (c) Piyush
(a) Sanjay (d) None of these
(b) Rajan Directions (46-50) : Answer the questions based on the
(c) No One information given below:
(d) Piyush There are eight persons P, Q, R, S, T, U, V and W are
(e) None of these sitting around a circular table four of them are facing the
43. After all persons changed their places based on the center and other four are facing away from the center.
above conditions, then how many persons facing away They are working in different banks i.e. BOB, Canara
from the center? bank, Axis bank, Hdfc bank, SBI, ICICI bank, Syndicate
(a)Two bank and PNB bank but not necessarily in the same order.
(b) One T is sitting between the person who works in Axis bank
(c) Five and the one who works in PNB bank. U faces the center
(d) Three and sits third to the right of R. The person who works in
(e) None of these Hdfc and SBI banks are facing same direction. S works in
Axis bank and faces the person who works in ICICI bank.

Page 897 of 1334


Subscribe The Xpress Video Course & Mock Test Package for Bank & Insurance Exams
If there are any suggestions/ errors in our PDFs Feel Free to contact us via this email: admin@exampundit.in
Ultra Practice Bundle PDF
SBI Clerk/ RBI Assist. Mains – Reasoning
V sits third to the right of Q, who works in BOB bank. P 2. If the card drawn is heart, the person who draws first
works in in SBI and R works in Canara bank. The person moves to position 2, then the second person who also draw
who works in Syndicate bank is facing outside the center heart moves to position 4.
and an immediate neighbor of who works in PNB bank. 3. If the card drawn is diamond, the person who draws first
W is an immediate neighbor of the person who works in moves to position 1, then the second person who also draw
SBI and Syndicate bank. T sits to immediate left of S. diamond moves to position 5.
Now they start playing cards game. They shuffled a pack 4. If the card drawn is Club, the person who draws first
of cards. Then they shift to table in which there are eight moves to position 7, then the second person who also draw
seats and the position of seats are numbered from 1 to 8 in Club moves to position 8.
anticlockwise direction. Draws:
1. P draws ace of Club
2. R draws king of Spade.
3. S draws ace of Diamond
4. T draws queen of Heart
5. V draws king of Diamond
6. W draws queen of Spade
7. Q draws ace of Club
8. U draws king of Heart
All the person chooses the card in the same sequence
as given above.
46. After all persons changed their places based on the
above conditions, how many persons sit between W
and T?
Conditions: (a) Two
1. If the card drawn is spade, the person who draws first (b) Four
moves to position 3, then the second person who also draw (c) One
spade moves to position 6. (d) Three
(e) None of these

Page 898 of 1334


Subscribe The Xpress Video Course & Mock Test Package for Bank & Insurance Exams
If there are any suggestions/ errors in our PDFs Feel Free to contact us via this email: admin@exampundit.in
Ultra Practice Bundle PDF
SBI Clerk/ RBI Assist. Mains – Reasoning
47. In initial arrangement who among the following (e) None of these
sits second to the right of who works in HDFC bank? SOLUTION (1-5):
(a) T Explanation in detail:
(b) W 1. F, who is sitting at corner is sitting between C and
(c) V E.
(d) S 2. E is the neighbor of B.
(e) None of these 3. D is sitting at a gap of three places from I.
48. After all persons changed their places based on the 4. I sits second to the left of F.
above conditions, then how many persons sit between
S and V?
(a) Three
(b) One
(c) Five
(d) Two
(e) None of these
49. In initial arrangement who among the following
sits opposite to the one who works in ICICI?
5. A is sitting second to the left of J.
(a) T
6. J and G are neighbours but none of them is sitting
(b) V
near B.
(c) W
(d) S
(e) None of these
50. After all persons changed their places based on the
above conditions, then who among the following sits on
position-7?
(a) T
(b) Q
(c) P
Movement based on following conditions:
(d) W
1. D chooses a ball of green colour.
Page 899 of 1334
Subscribe The Xpress Video Course & Mock Test Package for Bank & Insurance Exams
If there are any suggestions/ errors in our PDFs Feel Free to contact us via this email: admin@exampundit.in
Ultra Practice Bundle PDF
SBI Clerk/ RBI Assist. Mains – Reasoning
2. B chooses a ball of green colour. If the ball is Black colour, the person who
If the ball is green colour, the person who chooses first moves to position 4 facing the
chooses first moves to position 2 facing the opposite direction of current direction, then the
opposite direction of current direction, then the second person who chooses the moves to
second person who chooses the moves to position-6.
position-8 first.

3. H chooses a ball of yellow colour.


7. C chooses a ball of white colour.
4. A chooses a ball of yellow colour.
8. E chooses a ball of white colour.
If the ball is yellow colour the person who
If the ball is white colour, the person who
chooses first moves to position 3 facing the same
chooses first moves to position 1 facing the same
direction of current direction, then the second
direction of current direction, then the second
person who chooses the moves to position-5.
person who chooses the moves to position-7.

5. F chooses a ball of Black colour.


6. G chooses a ball of Black colour.

Page 900 of 1334


Subscribe The Xpress Video Course & Mock Test Package for Bank & Insurance Exams
If there are any suggestions/ errors in our PDFs Feel Free to contact us via this email: admin@exampundit.in
Ultra Practice Bundle PDF
SBI Clerk/ RBI Assist. Mains – Reasoning
1. d
2. c
3. e
4. d
5. c

SOLUTION (6-10):
Explanation in detail:
1. Anurag sits third to the right of Satyam.
6. Abhinav and Ravi faces in same direction.
2. Ravi is adjacent to Satyam but not opposite to Anurag.
7. Prashant faces away from the centre but does not sit
3. Hemant is second to the left of Anurag.
immediate right of Hemant.
4. Abhinav is immediate left of Hemant but not adjacent
8. Abhishek does not sit adjacent to Anurag.
to Anurag.
9. Deval and Satyam faces in same direction.
5. Hemant and Abhishek faces in same direction.
10. Hemant and Abhishek faces in same direction.

Page 901 of 1334


Subscribe The Xpress Video Course & Mock Test Package for Bank & Insurance Exams
If there are any suggestions/ errors in our PDFs Feel Free to contact us via this email: admin@exampundit.in
Ultra Practice Bundle PDF
SBI Clerk/ RBI Assist. Mains – Reasoning
(Condition - 4: If the box containing a 10-numbered book,
person remains on the same position facing same direction
of current direction.)

So there is no direction satisfy for Ravi and Abhinav


in CASE 2 so CASE 2 is cancelled out.

3. Abhishek chooses a box containing an even numbered


book other than 2, 4 and 6.
(Condition - 3: If the box containing 8-numbered book,
the person who choses first moves to position 5 facing
Movement based on following conditions:
same direction)
1. Prashant chooses a box containing an even numbered
4. Hemant chooses a box containing 7-numbered book.
book other than 4, 6 and 8 and it is also a prime number.
(Condition - 2: If the box containing a 7-numbered book,
(Condition - 1: If the box containing a 2-numbered book,
person remains on the same position facing opposite
the person who choses first moves to position 6 facing
direction.)
same direction of current direction)
2. Abhinav chooses a box containing 10-numbered book.
Page 902 of 1334
Subscribe The Xpress Video Course & Mock Test Package for Bank & Insurance Exams
If there are any suggestions/ errors in our PDFs Feel Free to contact us via this email: admin@exampundit.in
Ultra Practice Bundle PDF
SBI Clerk/ RBI Assist. Mains – Reasoning

7. Deval
5. Satyam chooses a box containing odd numbered book
chooses a box containing 10-numbered book.
but less than 5.
(Condition - 4: If the box containing a 10-numbered book,
(Condition - 1: The second person who chooses the box
person remains on the same position facing same direction
containing either 1 or 3-numbered book moves to position
of current direction)
4 facing same direction of current direction)
8. Ravi chooses a box containing an even numbered other
6. Anurag chooses a box containing 7-numbered book.
than 2, 4, 8 and 10.
(Condition - 2: If the box containing a 7-numbered book,
(Condition - 3: The second person, who chooses the box
person remains on the same position facing opposite
containing 6-numbered moves to position 3 facing same
direction)
direction of current direction)
Final arrangement:
Page 903 of 1334
Subscribe The Xpress Video Course & Mock Test Package for Bank & Insurance Exams
If there are any suggestions/ errors in our PDFs Feel Free to contact us via this email: admin@exampundit.in
Ultra Practice Bundle PDF
SBI Clerk/ RBI Assist. Mains – Reasoning
16. U is sitting fourth to the left of the one who has
Renault car.
17. One person is sitting between R and the one who
has a BMW car.
18. The one who has a Ford car is sitting exactly
opposite to the one who has a BMW car.

6.
b
7. d
8. a
9. d
10. a 19. Two persons are sitting between V and the one
SOLUTION(11-15): who has a Ford car.
Explanation in detail: 20. V does not have a Renault car.
15. The person who has Renault car is sitting third to
the right of R.
Page 904 of 1334
Subscribe The Xpress Video Course & Mock Test Package for Bank & Insurance Exams
If there are any suggestions/ errors in our PDFs Feel Free to contact us via this email: admin@exampundit.in
Ultra Practice Bundle PDF
SBI Clerk/ RBI Assist. Mains – Reasoning

There is no such space for Q and Audi in CASE 1 so


case 1 is cancelled out.
23. R neither has Maruti nor Audi car.
24. P is sitting third to the left of Q.

21. Three persons are sitting between Q and the one


who has a Audi car.
22. Q neither has Renault nor BMW car.

25. The one who has TATA car is sitting fifth to the
left of W.
26. Only one person is sitting between T and the one
who has Maruti car.
27. W neither has Toyota nor Maruti car.

Page 905 of 1334


Subscribe The Xpress Video Course & Mock Test Package for Bank & Insurance Exams
If there are any suggestions/ errors in our PDFs Feel Free to contact us via this email: admin@exampundit.in
Ultra Practice Bundle PDF
SBI Clerk/ RBI Assist. Mains – Reasoning
28. The one who has a Toyota car is not an immediate
neighbour of Q.

Movement based on following


conditions:
1. The person who have Toyota rolled a dice and
outcome is 1.
3. The person who have Ford rolled a dice and
(Condition: If someone rolled a dice and outcome
outcome is 4.
is 1, the person who chooses first moves to
(Condition: If someone rolled a dice and outcome
position 2, then the second person who chooses the
is 4, the person who chooses first moves to
ball moves to position 8.)
position 3, then the second person who chooses the
ball moves to position 7.)
2. The person who have TATA rolled a dice and
4. The person who have Kia rolled a dice and
outcome is 3.
outcome is 6.
(Condition: If someone rolled a dice and outcome
(Condition: If someone rolled a dice and outcome
is 3, the person who chooses first moves to
is 6, the person who chooses first moves to
position 4, then the second person who chooses the
position 1, then the second person who chooses the
ball moves to position 6.)
ball moves to position 5.)

Page 906 of 1334


Subscribe The Xpress Video Course & Mock Test Package for Bank & Insurance Exams
If there are any suggestions/ errors in our PDFs Feel Free to contact us via this email: admin@exampundit.in
Ultra Practice Bundle PDF
SBI Clerk/ RBI Assist. Mains – Reasoning

5. The person who have Maruti rolled a dice and


outcome is 4.
7. The person who have Audi rolled a dice and
(Condition: If someone rolled a dice and outcome
outcome is 6.
is 4, the person who chooses first moves to
(Condition: If someone rolled a dice and outcome
position 3, then the second person who chooses the
is 6, the person who chooses first moves to
ball moves to position 7.)
position 1, then the second person who chooses the
6. The person who have Renault rolled a dice and
ball moves to position 5.)
outcome is 3.
8. The person who have BMW rolled a dice and
(Condition: If someone rolled a dice and outcome
outcome is 1.
is 3, the person who chooses first moves to
(Condition: If someone rolled a dice and outcome
position 4, then the second person who chooses the
is 1, the person who chooses first moves to
ball moves to position 6.)
position 2, then the second person who chooses the
ball moves to position 8.)

Page 907 of 1334


Subscribe The Xpress Video Course & Mock Test Package for Bank & Insurance Exams
If there are any suggestions/ errors in our PDFs Feel Free to contact us via this email: admin@exampundit.in
Ultra Practice Bundle PDF
SBI Clerk/ RBI Assist. Mains – Reasoning

11. a
12. d
13. a
14. d
15. c
SOLUTION(16-20):
3. E is not immediate neighbor of D and F.
Explanation in detail:
1. C sits second to the right of F.
2. One person sits between C and D.

Page 908 of 1334


Subscribe The Xpress Video Course & Mock Test Package for Bank & Insurance Exams
If there are any suggestions/ errors in our PDFs Feel Free to contact us via this email: admin@exampundit.in
Ultra Practice Bundle PDF
SBI Clerk/ RBI Assist. Mains – Reasoning

10. E face opposite direction of C.


So CASE2 is cancelled out.
4. E sits third to the right of G.
So Rank of all persons are in ascending order from
5. G sits second to the left of A, who sits second to
A in clockwise direction and in ascending order.
the right of B.
6. B and A are not immediate neighbors of D.
7. H and D sit immediate right to each other.
8. C faces the same direction as A.
9. E faces towards the center.

Page 909 of 1334


Subscribe The Xpress Video Course & Mock Test Package for Bank & Insurance Exams
If there are any suggestions/ errors in our PDFs Feel Free to contact us via this email: admin@exampundit.in
Ultra Practice Bundle PDF
SBI Clerk/ RBI Assist. Mains – Reasoning
8. The person who got 4 rank sits second to the right
of A.

16. a
17. d
18. a
19. d

Movement based on following conditions: 20. a

1. The person who got 6 rank sits third from extreme SOLUTION(21-25):

left end. Explanation in detail:

2. The person who got 2 rank sits third to right of D. 1. Y sits one the corner side of the table.
2. T sits immediate right to the Y.
3. W sits second to the left of T.

3. The person who got 5 rank sits fourth to the left of


F.
4. The person who got 3 rank sits at extreme end but
not an immediate neighbor of H.

4. V sits on the middle side of the table.

5. The person who got 7 rank sits fourth to the left of 5. U sits immediate left to the V.

B. 6. S sits second to the right of X.

6. The person who got 1 rank sits immediate left of 7. There are only two persons sit between X and Z.

F. 8. Z is not an immediate neighbor of S.

7. The person who got 8 rank sits second to left of D.

Page 910 of 1334


Subscribe The Xpress Video Course & Mock Test Package for Bank & Insurance Exams
If there are any suggestions/ errors in our PDFs Feel Free to contact us via this email: admin@exampundit.in
Ultra Practice Bundle PDF
SBI Clerk/ RBI Assist. Mains – Reasoning
(Condition: The person who chooses the ball which
has either 5 or 9 printed on it moves to position-5 first
then to position 7 facing the same direction.)
4. Y chooses a ball on which 3 is printed.
(Condition: If the ball has 3 printed on it, the person
who chooses first moves to position 4 facing the same
direction of current direction.)

Movement based on following conditions:


1. W chooses a ball on which the even number is
printed other than 4 and 6 is printed.
(Condition: If the ball has 2 printed on it, the person
who chooses first moves to position 3 facing the
opposite direction of current direction.)
2. X chooses a ball on which 7 is printed.
(Condition: If the ball has 7 printed on it, the person
moves to position-6 facing the same direction.) 5. Z chooses a ball on which Even number are
printed other than 2 and 6 is printed.
(Condition: The person who chooses the ball which
has 4 printed on, moves to position-8 facing the
same direction.)
6. U chooses a ball on which 1 is printed.
(Condition: The person who chooses the ball which
has 1 printed on it moves to position-1 facing the
same direction.)

3. The person immediate right of Y chooses a ball


on which 5 are printed.
Page 911 of 1334
Subscribe The Xpress Video Course & Mock Test Package for Bank & Insurance Exams
If there are any suggestions/ errors in our PDFs Feel Free to contact us via this email: admin@exampundit.in
Ultra Practice Bundle PDF
SBI Clerk/ RBI Assist. Mains – Reasoning
22. d
23. a
24. c
25. a
SOLUTION(26-30)
Explanation in detail:
1. G does not face outside and sits middle side of the
table.
2. There are four people sitting between G and H and
7. V chooses a ball on which Even number are all of them are facing outside.
printed other than 2 and 4 is printed. 3. H faces outside the table.
(Condition: The person who chooses the ball
which has 6 printed on it moves to position-2
facing the same direction.)
8. S chooses a ball on which 9 is printed.
(Condition: The person who chooses the ball
which has either 5 or 9 printed on it moves to
position-5 first then to position-7 facing the
same direction.)

4. B who is facing inside is seventh to the left of E.


5. There are only one person between G and A.

21. d
Page 912 of 1334
Subscribe The Xpress Video Course & Mock Test Package for Bank & Insurance Exams
If there are any suggestions/ errors in our PDFs Feel Free to contact us via this email: admin@exampundit.in
Ultra Practice Bundle PDF
SBI Clerk/ RBI Assist. Mains – Reasoning

C is not the immediate neighbour of G and A.


So case 1 is cancelled out.

6. D is sitting to the immediate left of C.

Movement based on following conditions:


1. B draws queen of Club
2. A draws Jack of Club
(Condition: If the card drawn is club, the person who
draws first moves to position 2 facing same direction, then
the second person, who also draw club moves to position
7 and direction remains the same)

Page 913 of 1334


Subscribe The Xpress Video Course & Mock Test Package for Bank & Insurance Exams
If there are any suggestions/ errors in our PDFs Feel Free to contact us via this email: admin@exampundit.in
Ultra Practice Bundle PDF
SBI Clerk/ RBI Assist. Mains – Reasoning
5. D draws a King of Diamond
6. H draws 8 of Diamond
(Condition: If the card drawn is diamond, the person who
draws first moves to position 1 facing same direction, then
the second person, who also draw diamond moves
position 4 and direction remains the same)

E draws
2 of Spade
3. C draws 7 of Spade
(Condition: If the card drawn is spade, the person who
draws first moves to position 5 facing same direction
of current direction, then the second person who also
draw spade moves to position 8facing same direction
of the current direction) 7. G draws 6 of Heart
8. F draws Ace of Heart
(Condition: If card drawn is heart, the person who draws
first moves to position 3 facing same direction, then the
second person, who also draw heart moves to position 6
and direction remains the same)

Page 914 of 1334


Subscribe The Xpress Video Course & Mock Test Package for Bank & Insurance Exams
If there are any suggestions/ errors in our PDFs Feel Free to contact us via this email: admin@exampundit.in
Ultra Practice Bundle PDF
SBI Clerk/ RBI Assist. Mains – Reasoning
There can be two cases according to the given
information:

26. a
27. a
28. d
29. d
30. d
SOLUTION (31-35):
Explanation in detail:
6. Suresh is not an immediate neighbour of Hemant.
1. Virat sits third to the right of the person who likes
7. The person who likes History sits second to the left of
Geography.
Suresh.
2. Only two people sit between the person who likes
8. The person who likes History is an immediate
Geography and Hemant.
neighbour of the both persons who like English and
3. The person who likes Computer is not an immediate
Economics.
neighbour of the person who likes Geography.
4. Neither Virat nor Hemant likes Computer or Reasoning.
5. The persons who like Computer and the Reasoning are
immediate neighbours of each other.

Page 915 of 1334


Subscribe The Xpress Video Course & Mock Test Package for Bank & Insurance Exams
If there are any suggestions/ errors in our PDFs Feel Free to contact us via this email: admin@exampundit.in
Ultra Practice Bundle PDF
SBI Clerk/ RBI Assist. Mains – Reasoning
There is no such space for Suresh in CASE1 so CASE1
is cancelled out.
9. Person who likes Economics sits third to the right of
Dhoni.
10. Dhoni does not like Computer.

11. Mohit is not an immediate neighbour of Virat.


12. Devang is not an immediate neighbour of the person
who likes History.
13. Virat does not like Quant.

Page 916 of 1334


Subscribe The Xpress Video Course & Mock Test Package for Bank & Insurance Exams
If there are any suggestions/ errors in our PDFs Feel Free to contact us via this email: admin@exampundit.in
Ultra Practice Bundle PDF
SBI Clerk/ RBI Assist. Mains – Reasoning
14. Rohit sits on the immediate right of the person who
likes Quant.

Movement based on following conditions:


1. The person who likes Quant i.e. Suresh chooses a ball
in which the vowel other than O and I is printed.
(Condition – 1: If the ball has E printed on it, the person
who chooses first moves to position 3 facing the opposite
direction of current direction.)
2. Dhoni chooses a ball in which F is printed.
(Condition – 2: If the ball has F printed on it, the person
remains on the same position facing opposite direction.)
3. The person immediate right of Virat i.e. Dhawan

There is no such space for Quant and Rohit in CASE chooses a ball in which the consonant other than B, C and

2a So CASE 2a is cancelled out. F is printed.


(Condition – 1: the second person who chooses the ball
which has either G or D printed on it moves to position-5
facing the same direction)

Page 917 of 1334


Subscribe The Xpress Video Course & Mock Test Package for Bank & Insurance Exams
If there are any suggestions/ errors in our PDFs Feel Free to contact us via this email: admin@exampundit.in
Ultra Practice Bundle PDF
SBI Clerk/ RBI Assist. Mains – Reasoning

4. Rohit chooses a ball on which C is printed.


(Condition – 3: If the ball has C printed on it, the person
who chooses first moves to position 4 facing the same
7. Mohit chooses a ball in which the vowel other than E
direction of current direction)
and I is printed.
5. The person who likes History i.e. Hemant chooses a ball
(Condition – 3: second person who chooses the ball which
on which the vowel other than E and O is printed.
has O printed on it moves to position-2 facing the same
(Condition – 4: If the box contains I, person remains at the
direction)
same position facing same direction)
8. The person who likes Computer i.e. Devang chooses a
6. Virat chooses a ball in which F is printed.
ball in which D is printed.
(Condition – 2: If the ball has F printed on it, the person
remains on the same position facing opposite direction.)

Page 918 of 1334


Subscribe The Xpress Video Course & Mock Test Package for Bank & Insurance Exams
If there are any suggestions/ errors in our PDFs Feel Free to contact us via this email: admin@exampundit.in
Ultra Practice Bundle PDF
SBI Clerk/ RBI Assist. Mains – Reasoning
(Condition – 1: the third person who chooses the ball 35. a
which has either G or D printed on it moves to position-7 SOLUTION (36-40):
facing the same direction) Explanation in detail:
Final arrangement: 1. J sits second to the left of L.
2. O sits second to the right of J.
3. No one sits to the left of O.
4. As many persons sit to the right of J as sit to the
left of K.

5. N sits second to the left of the one who sits third to


the right of P.
6. Q is not an immediate neighbour of P.
7. N and M face opposite direction with respect to
each other.
8. M face same direction as Q faces.

31. a
32. d
33. a
34. d

Page 919 of 1334


Subscribe The Xpress Video Course & Mock Test Package for Bank & Insurance Exams
If there are any suggestions/ errors in our PDFs Feel Free to contact us via this email: admin@exampundit.in
Ultra Practice Bundle PDF
SBI Clerk/ RBI Assist. Mains – Reasoning

3. N draws queen of Spade


(Condition: If card drawn is queen of spade, the
person who draws moves to position 3.)
4. L draws king of Spade
9. Q doesn’t face south
(Condition: If card drawn is king of Spade, the
So case 2 is cancelled out.
person who draws moves to position 1.)

Movement based on following conditions:


1. K draws queen of Club.
(Condition: If card drawn is queen of club, the
person who draws moves to position 5.)
2. J draws king of Club
(Condition: If card drawn is king of Club, the
person who draws moves to position 6.)

5. M draws King of Diamond


(Condition: If card drawn is king of Diamond, the
person who draws moves to position 8.)
6. Q draws queen of Diamond
(Condition: If card drawn is queen of diamond, the
person who draws moves to position 7.)

Page 920 of 1334


Subscribe The Xpress Video Course & Mock Test Package for Bank & Insurance Exams
If there are any suggestions/ errors in our PDFs Feel Free to contact us via this email: admin@exampundit.in
Ultra Practice Bundle PDF
SBI Clerk/ RBI Assist. Mains – Reasoning
37. d
38. a
39. d
40. a
SOLUTION (41-45):
Explanation in detail:
1. Ankit and Rajan are facing opposite direction.
2. Ankit sits second to left of Rajan and Piyush sits
third to right of Ankit

7. P draws king of Heart


(Condition: If card drawn is king of Heart, the
person who draws moves to position 2.)
8. O draws queen of Heart
(Condition: If card drawn is queen of Heart, the
person who draws moves to position 4.)

3. Only three persons are sitting between Piyush and


Ravi.
4. Kartik sits immediate right of Rajan.
36. c

Page 921 of 1334


Subscribe The Xpress Video Course & Mock Test Package for Bank & Insurance Exams
If there are any suggestions/ errors in our PDFs Feel Free to contact us via this email: admin@exampundit.in
Ultra Practice Bundle PDF
SBI Clerk/ RBI Assist. Mains – Reasoning
5. Rahul sits second to right of Satyam who sits third
to the left of Kartik.
6. Rajan sits third to right of Sanjay and both facing
same direction.
7. Rahul and Ravi are facing same direction as
Satyam.
8. Piyush facing opposite direction as Kartik.

Movement based on following conditions:


1. Ravi draws queen of Club,
2. Ankit draws Jack of Club.
(Condition: If card drawn is club, person remains on the
same position facing opposite direction of current
direction.)

9. Piyush facing inside the circle.


So case 2 is eliminated.

Page 922 of 1334


Subscribe The Xpress Video Course & Mock Test Package for Bank & Insurance Exams
If there are any suggestions/ errors in our PDFs Feel Free to contact us via this email: admin@exampundit.in
Ultra Practice Bundle PDF
SBI Clerk/ RBI Assist. Mains – Reasoning

3. Piyush draws 3 of Spade,


(Condition: If the card drawn is spade, the person who
draws first moves to position 5 facing opposite direction 4. Rahul draws a King of Diamond.
of current direction.) (Condition: If the card drawn is diamond, the person who
draws first moves to Position 1 facing same direction.)

Page 923 of 1334


Subscribe The Xpress Video Course & Mock Test Package for Bank & Insurance Exams
If there are any suggestions/ errors in our PDFs Feel Free to contact us via this email: admin@exampundit.in
Ultra Practice Bundle PDF
SBI Clerk/ RBI Assist. Mains – Reasoning

7. Kartik draws 7 of Diamond.


(Condition: If the card drawn is diamond, the person who
5. Satyam draws Ace of Heart,
draws first moves to corner 1 facing same direction, then
6. Sanjay draws 9 of Heart.
the second person who also draw diamond moves to
(Condition: If card drawn is heart, person remains on the
corner 2, similarly 3 and 4.)
same position facing same direction.)

Page 924 of 1334


Subscribe The Xpress Video Course & Mock Test Package for Bank & Insurance Exams
If there are any suggestions/ errors in our PDFs Feel Free to contact us via this email: admin@exampundit.in
Ultra Practice Bundle PDF
SBI Clerk/ RBI Assist. Mains – Reasoning

8. Rajan draws 4 of Spade.


(Condition: If the card drawn is spade, the person who 41. b
draws first moves to position 5 facing opposite direction 42. c
of current direction, then the second person who also draw 43. e
spade moves to position 6, similarly 7 and 8.) 44. d
45. c
SOLUTION (46-50):
Explanation in detail:
11. The person who works in Hdfc and SBI banks are
facing same direction.
12. T sits to immediate left of S.
13. T is sitting between the person who works in Axis
bank and the one who works in PNB bank.
14. S works in Axis bank and faces the person who
works in ICICI bank.

Page 925 of 1334


Subscribe The Xpress Video Course & Mock Test Package for Bank & Insurance Exams
If there are any suggestions/ errors in our PDFs Feel Free to contact us via this email: admin@exampundit.in
Ultra Practice Bundle PDF
SBI Clerk/ RBI Assist. Mains – Reasoning
20. W is an immediate neighbor of the person who
works in SBI and Syndicate bank.

Movement based on following


conditions:
1. P draws ace of Club
(Condition: If the card drawn is Club, the person
who draws first moves to position 7, then the
second person who also draw Club moves to
position 8.)
2. R draws king of Spade.
In
(Condition: If the card drawn is spade, the person
CASE1 S faces who works in ICICI bank so CASE1 is
who draws first moves to position 3, then the
cancelled out.
second person who also draw spade moves to
15. U faces the center and sits third to the right of R.
position 6.)
16. V sits third to the right of Q, who works in BOB
bank.
17. P works in in SBI.
18. R works in Canara bank.
19. The person who works in Syndicate bank is facing
outside the center and an immediate neighbor of
who works in PNB bank.
Page 926 of 1334
Subscribe The Xpress Video Course & Mock Test Package for Bank & Insurance Exams
If there are any suggestions/ errors in our PDFs Feel Free to contact us via this email: admin@exampundit.in
Ultra Practice Bundle PDF
SBI Clerk/ RBI Assist. Mains – Reasoning
(Condition: If the card drawn is diamond, the
person who draws first moves to position 1, then
the second person who also draw diamond moves
to position 5.)
6. W draws queen of Spade
(Condition: If the card drawn is spade, the person
who draws first moves to position 3, then the
second person who also draw spade moves to
position 6.)

3. S draws an Ace of Diamond


(Condition: If the card drawn is diamond, the
person who draws first moves to position 1, then
the second person who also draw diamond moves
to position 5.)
4. T draws Queen of Heart
(Condition: If the card drawn is heart, the person
who draws first moves to position 2, then the
second person who also draw heart moves to
position 4.)
7. Q draws Ace of Club
(Condition: If the card drawn is Club, the person
who draws first moves to position 7, then the
second person who also draw Club moves to
position 8.)
8. U draws king of Heart
(Condition: If the card drawn is heart, the person
who draws first moves to position 2, then the
second person who also draw heart moves to
position 4.)
5. V draws King of Diamond
Page 927 of 1334
Subscribe The Xpress Video Course & Mock Test Package for Bank & Insurance Exams
If there are any suggestions/ errors in our PDFs Feel Free to contact us via this email: admin@exampundit.in
Ultra Practice Bundle PDF
SBI Clerk/ RBI Assist. Mains – Reasoning
46. d
47. c
48. a
49. d
50. c

Download Seating Arrangement Practice Questions PDF

Get More Reasoning Practice Questions PDF


Puzzle Income Based
Question:
Direction 1-5: Read the following information carefully to
@1 B means one person is heavier than B.
answer the question that follows: 1@ B means one person is lighter than B.
Six persons J, K, L, M, N and P are sitting in a straight row
@2K, L~>K, J~< N, P > M, N < L, M > J
facing north direction in such a way that there is an equal
At least two persons are lighter than J.
distance between the adjacent persons. 1) If the weight of J is 67kg and difference between t
L sits second to the right of J. At least five persons sit to the
weights of J and L is 23kg, what is the probable weight
right of J. K sits to the immediate left of M who sits to the
M?
immediate left of N. P sits immediate right of J. a. 65kg
The six persons J, K, L, M, N and P have different weights
b. 34kg
also. c. 44kg
A < B means A is lighter than B, similarly, A ~< B means A
d. 70kg
is not lighter than B. e. None of these
A > B means A is heavier than B, similarly, A ~> B means A
2) Who sits third to the right of P?
is not heavier than B. a. L

Page 928 of 1334


Subscribe The Xpress Video Course & Mock Test Package for Bank & Insurance Exams
If there are any suggestions/ errors in our PDFs Feel Free to contact us via this email: admin@exampundit.in
Ultra Practice Bundle PDF
SBI Clerk/ RBI Assist. Mains – Reasoning
b. K and W. T is sitting adjacent to W. U is sitting 2nd to the
c. M left of Z. Y is not sitting adjacent to S. Y is sitting in the
d. N right of Z. Not more than 2 persons are sitting between U
e. None of these and T.
3) Who is the third heaviest person? The Eights students S, T, U, V, W, X, Y and Z also score
a. L different marks in a class test.
b. K A > B means A scores less than B, similarly, A ~> B
c. M means A doesn’t score less than B.
d. N A < B means A scores more than B, similarly, A ~< B
e. None of these means A doesn’t score more than B.
4) If the weight of K is 63kg what is the possible 1@ B means one student scores more than B.
weight of L? @1 B means one student scores less than B.
a. 65 kg 1@W, @2V, T < Z, Y < U, T~<U, Z~< S, U < S, Y~>W
b. 64 kg Less than two students score less than X.
c. 62 kg 6) How many persons are sitting to the left of V?
d. 67 kg a. 5
e. None of these b. 2
5) How many persons are lighter than N? c. 6
a. One d. 4
b. Two e. 3
c. Three 7) If S scores 23 marks and the difference between
d. Four marks of S and V is 7 and the difference between
e. None marks of V and U is 17 then what could be the score
Directions 6-10: Answer the questions based on the of the highest scorer?
information given below: a. 35
Eight persons, S, T, U, V, W, X, Y and Z are sitting in a b. 27
straight row. All of them are facing the north. c. 24
Three persons are sitting between S and V, who is sitting d. 20
2nd to the right of X. Two persons are sitting between X e. None of these

Page 929 of 1334


Subscribe The Xpress Video Course & Mock Test Package for Bank & Insurance Exams
If there are any suggestions/ errors in our PDFs Feel Free to contact us via this email: admin@exampundit.in
Ultra Practice Bundle PDF
SBI Clerk/ RBI Assist. Mains – Reasoning
8) How many students score more than X but less G sits to the right of F, who faces south. Both A and E
than T if Z doesn’t score the 2nd lowest and More are facing the same direction. F doesn’t sit to the left of
than three students score more than S.? E.
a. 3 These Seven persons have different weights also.
b. 2 P#Q means P is heavier than Q.
c. 4 P ~#Q means P is not heavier than Q.
d. 1 P#2 means two persons are heavier than P.
e. None of these P2# means two persons are lighter than P.
9) How many persons are sitting between Z and T? A #2, B 2#, A # C
a. 4 D and F are lighter than C. G is not lighter than E. D is
b. 2 not the lightest person.
c. 3 11) How many persons sit between A and E?
d. 5 a. Two
e. None of these b. Four
10) Who is sitting immediate right of Y? c. Three
a. S d. One
b. U e. None of these
c. W 12) Which among the following statement(s) is/are
d. Either (a) or (b) true?
e. None of these a. G sits 3rd to the left of B.
Directions 11-15 : Answer the questions based on the b. A sits to the immediate left of F.
information given below: c. B sits 2nd to the right of C.
Seven persons A, B, C, D, E, F and G are sitting in a d. E sits 2nd to the right of D.
straight row. Some of them face north while others face e. None is true.
south direction. 13) What is the position of D with respect to B?
C sits 3rd to the left of D, who faces north. B sits to the a. 4th to the right
immediate right of C. A sits 3rd to the left of B, who faces b. 2nd to the right
north. Person, who sits to the immediate left of G, faces c. 3rd to the left
south. E sits neither adjacent to A nor at any extreme end. d. 2nd to the left

Page 930 of 1334


Subscribe The Xpress Video Course & Mock Test Package for Bank & Insurance Exams
If there are any suggestions/ errors in our PDFs Feel Free to contact us via this email: admin@exampundit.in
Ultra Practice Bundle PDF
SBI Clerk/ RBI Assist. Mains – Reasoning
e. None of these a. Q
14) How many persons are lighter than C? b. M
a. Two c. N
b. Five d. T
c. One e. None of these
d. Three 17) Who among the following sits second to the right
e. None of these of M?
15) Who among the following is the heaviest? a. R
a. E b. Q
b. C c. S
c. G d. T
d. D e. None of these
e. None of these 18) What is the difference between the age of S and
Directions 16-20 : Answer the questions based on the T?
information given below. a. 15 years
Eight persons M, N, O, P, Q, R, S and T are sitting around b. 5 years
the circular table equidistant from each other facing c. 30 years
towards the center. Each of them is different years old d. 20 years
among 5, 10, 15, 20, 25, 30, 35 and 40. Some of the e. None of these
statements are coded as, 1 9) What is the age of S?
A +2 B means A is 2nd to the right of B. a. 15 years
A 2+ B means A is 2nd to the left of B. b. 5 years
A % B means A’s age is twice that of B. c. 30 years
T is not elder than that of N. R sits third to the right of O, d. 20 years
who is 5 years old. M sits to the immediate right of P. N e. None of these
sits to the immediate left of one, who is 10 years old. N 20) Who sits third to the right of P?
is not opposite to O or M. M is not 35 years old. a. R
T 2+ O, N % Q, S % P, (15yrs old) +2 S b. Q
16) Who among the following is the eldest? c. S

Page 931 of 1334


Subscribe The Xpress Video Course & Mock Test Package for Bank & Insurance Exams
If there are any suggestions/ errors in our PDFs Feel Free to contact us via this email: admin@exampundit.in
Ultra Practice Bundle PDF
SBI Clerk/ RBI Assist. Mains – Reasoning
d. T b. O
e. None of these c. Either (a) or (b)
Directions 21-25: Answer the questions based on the d. P
information given below: e. None of these
Seven persons, J, K, L, M, N, O and P are sitting in a 23) What is the position of N with respect to O?
straight row. The number of persons facing the north is a. 4th to the right
more than those who are facing the south. b. 2nd to the left
L is sitting 2nd to the right of J and both of them are c. 4th to the left
facing the same direction. O is sitting 3rd to the left of L. d. 3rd to the right
Immediate neighbors of O are facing the opposite e. None of these
direction of O. M is sitting adjacent to O. Two persons 24) How many persons are sitting in the right of J?
are sitting between M and P. N is sitting 2nd to the left a. 4
of P and both of them are facing the same direction. K is b. 2
sitting adjacent to N. Persons sitting at the extreme ends c. Either (a) or (b)
are facing the opposite directions. d. 3
These seven persons are of different ages. e. None of these
A + B means A is older than B. 25) How many persons are sitting between L and M?
A @ B means A is older than only B. a. 1
A%B means A is younger than B. b. 3
K @ J; L+M; L%N c. 2
O is older than N. M is not the 3rd youngest person. d. None of these
21) How many persons are older than P? e. 4
a. 5 Direction 26-30: Study the following information
b. 3 carefully to answer the given questions.
c. 2 Eight persons viz. Anand, Aman, Abhinav, Atul, Anish,
d. 4 Ashish, Ankit and Abhishek played a match and scored
e. None of these different runs.
22) Who is the 2nd oldest person? A#B means A always scored more than B.
a. N A&B means A always scored less than B.

Page 932 of 1334


Subscribe The Xpress Video Course & Mock Test Package for Bank & Insurance Exams
If there are any suggestions/ errors in our PDFs Feel Free to contact us via this email: admin@exampundit.in
Ultra Practice Bundle PDF
SBI Clerk/ RBI Assist. Mains – Reasoning
A@ means A scored highest runs. a. 77
A^ means A scored lowest runs. b. 64
A3$ means A scored third highest runs. c. 82
A3! means A scored third lowest runs. d. 56
A#*B means A has more runs than only B. e. None of these
A &*B means A has less runs than only B. 29) Who is A in A2!?
A%B means A is not equal to B. a. Atul
Further it is given that: b. Anand
i) Aman &*1 ii) Anand #* (Atul, Ashish) iii) Abhinav & c. Ashish
Abhishek iv) Ankit # Abhishek v) Abhishek 4$ vi) Ankit d. Aman
# Anish vii) The 2$ scored 80 runs. e. Can’t be determined
26) In a rematch among bottom three, if Ashish 30) How many people are present between 2$ and 3!
#*Anand, who has scored the highest among the scorers?
three? a. 2
a. Anand b. 3
b. Ashish c. 1
c. Atul d. 4
d. Ankit e. None of these
e. Aman Directions 31-35: Answer the questions based on the
27) What is the minimum run, Abhishek has to score information given below.
to achieve the second rank if the score of Abhishek is Seven persons P, Q, R, S, T, U and V are sitting in the
65? horizontal row but not in the same order. Some of them
a. 14 are facing towards the north while some are facing
b. 13 towards the south.
c. 15 At most one person sitting to the left of P, who faces
d. 16 towards the north direction,. Only one person sits
e. Can’t be determined between P and V. T sits second to the left of V and both
28) If Abhishek scored 65 runs, what can be the score are facing the same direction. R sits second to the right
of Anish? of U and neither of them is an immediate neighbor of P.

Page 933 of 1334


Subscribe The Xpress Video Course & Mock Test Package for Bank & Insurance Exams
If there are any suggestions/ errors in our PDFs Feel Free to contact us via this email: admin@exampundit.in
Ultra Practice Bundle PDF
SBI Clerk/ RBI Assist. Mains – Reasoning
R sits to the immediate left of Q. S and R face the same 34) Who among the following sits second to the left of
direction but opposite to U. T?
These Seven persons have different heights also. a. Q
A @ B means A is taller than B. b. V
A ~@ B means A is not taller than B. c. P
A 2# means two persons are taller than A. d. S
A 2@ means two persons are shorter than A. e. None of these
P @ R, P 2#, T 2@ 35) How many persons are facing south direction?
S is taller than Q, who is taller than R. U is taller than V a. Three
but not as much as R. b. One
31) Who among the following is the tallest? c. Four
a. Q d. Five
b. P e. None of these
c. S Directions 36-40: Answer the questions based on the
d. T information given below.
e. None of these Eight persons P, Q, R, S, T, U, V and W are sitting
32) How many persons are taller than U? around the circular table facing towards the center. Each
a. Three of them has different number of books from 1 to 8. Some
b. Four of the statements are coded as below,
c. Six A % B means A sits third to the right of B.
d. Five A # B means A sits second to the right of B.
e. None of these A $ B means A has double the number of books than B.
33) Who sits at the right end of the row? T sits adjacent to P. The one, who has 6 books, sits to the
a. R immediate left of W. Q, who has 5 books, sits adjacent to
b. P S. Neither R nor W has highest number of books. U sits
c. S adjacent to one, who has 1 book. U does not have 6 books
d. Q and is not opposite to S. R has more books than W. V is
e. None of these not opposite to T.
P % W, S # T, W $ S , R $ U

Page 934 of 1334


Subscribe The Xpress Video Course & Mock Test Package for Bank & Insurance Exams
If there are any suggestions/ errors in our PDFs Feel Free to contact us via this email: admin@exampundit.in
Ultra Practice Bundle PDF
SBI Clerk/ RBI Assist. Mains – Reasoning
36) Who among the following has 7 books? e. None of these
a. T Direction 41- 45: Study the following information
b. S carefully to answer the given questions.
c. U Eight persons A, B, C, D, E, F, G and H are sitting around
d. P the rectangular table facing away from the center. Four
e. Cannot be determined of them sit at the corner of the table and remaining are
37) Who sits second to the left of R? sits at the middle of the edges. Each of them has different
a. S number of coins among 5, 15, 20, 26, 30, 35, 40 and 71.
b. Q Some of the statements are coded in the following
c. P manner,
d. V P $ Q means P sits third to the left of Q.
e. None of these P % Q means P has twice the number of coins than that
38) How many books does R have? of Q.
a. 4 D sits opposite to the one, who sits adjacent to C.
b. 7 D sits one of the longer side of the table and has a coin
c. 6 which is multiple of 6.
d. 2 H is adjacent to C who sits at the corner.
e. None of these A sits to the immediate right of E, who has 71 coins.
39) Who sits to the immediate left of V? E and C are not adjacent to each other.
a. P A and D are not adjacent to each other.
b. U F has 26 coins and does not sit adjacent to D.
c. T H has more coins than A. E sits at corner.
d. Q The one, who has 30 balls, sits adjacent to B.
e. None of these The person, who has 20 balls, does not sit adjacent to D.
40) Who among the following has 1 book? G$H,B%G,D%C
a. R
b. T 41) Who among the following has 35 balls?
c. U a. B
d. V b. H

Page 935 of 1334


Subscribe The Xpress Video Course & Mock Test Package for Bank & Insurance Exams
If there are any suggestions/ errors in our PDFs Feel Free to contact us via this email: admin@exampundit.in
Ultra Practice Bundle PDF
SBI Clerk/ RBI Assist. Mains – Reasoning
c. D Directions 46-50: Answer the questions based on the
d. A information given below.
e. None of these Seven persons A, B, C, D, E, F and G are sitting in the
42) Who sits opposite to H? straight horizontal row. Some of them are facing north
a. The one, who has 20 balls and some are facing south. Not more than two persons
b. The one, who has 15 balls facing in the same direction are sitting together. The
c. B extreme ends are considered with respect to north
d. A direction.
e. C C sits second from one of the ends of the row and is
facing towards south.
43 ) Who sits second to the left of C? A sits to the immediate right of C and both of them are
a. D facing opposite directions with respect to each other.
b. G B sits second to the right of A. Only two persons are
c. E sitting between B and E.
d. F G sits to the immediate right of E and second to the right
e. None of these of D, who faces north direction.
44) Who among the following has 5 coins? C and B face opposite directions with respect to each
a. A other. F and G face same direction.
b. C These seven persons have different weights
c. D A @ B means A is not lighter than B.
d. B A # B means A is not heavier than B.
e. E C @ B, D @ B, F # B, E @ G, G # F, A # B
45) Who among the following has 40 coins? D is not the heaviest and E is lighter than at least 4
a. A persons. A is not the lightest.
b. C 46) If the weights of A and E are 34kg and 23kg
c. D respectively. What can be the weight of D?
d. B a. 30kg
e. E b. 28kg
c. 20kg

Page 936 of 1334


Subscribe The Xpress Video Course & Mock Test Package for Bank & Insurance Exams
If there are any suggestions/ errors in our PDFs Feel Free to contact us via this email: admin@exampundit.in
Ultra Practice Bundle PDF
SBI Clerk/ RBI Assist. Mains – Reasoning
d. 40kg
e. None of these Solution:
47) ___ is the 3rd heaviest while G is the ___ person. Answers 1-5
a. B, 4th lightest 1. d
b. A, 5th heaviest 2. c
c. D, 5th lightest 3. b
d. B, lightest 4. c
e. None of these 5. e
48) Who among the following sits at the extreme right Solutions 1-5
end of the row? 1. L sits second to the right of J.
a. A 2. At least five persons sit to the right of J.
b. B
c. C
d. G
e. None of these 3. K sits to the immediate left of M who sits to the
49) Who among the following sits second to the left of immediate left of N.
D? 4. P sits immediate right of J.
a. B
b. A
c. C
d. E @2K means two persons are heavier than K.
e. Can’t be determined L~>K means L is not heavier than K.
50) How many persons in the row facing south J ~< N means J is not lighter than N.
direction? N < L means N is lighter than L.
a. Five P > M means P is heavier than M.
b. One At least two persons are lighter than J.
c. Two M > J means M is heavier than J,
d. Four so the arrangement can be given as:
e. Three P>M>K>J>L>N
Answers:
Page 937 of 1334
Subscribe The Xpress Video Course & Mock Test Package for Bank & Insurance Exams
If there are any suggestions/ errors in our PDFs Feel Free to contact us via this email: admin@exampundit.in
Ultra Practice Bundle PDF
SBI Clerk/ RBI Assist. Mains – Reasoning
6. c more than S. U < S means U scores more than S. Y~>W
7. a means Y doesn’t score less than W, so
8. b Y > W > U > T/S > S/T > V > X/Z > Z/X
9. a Answers:
10. e 11. c
Solutions 6-10 12. e
1 Three persons are sitting between S and V, who is 13. b
sitting 2nd to the right of X. 14. d
2. Two persons are sitting between X and W. 15. c
3. T is sitting adjacent to W. Solutions 11-15
4. U is sitting 2nd to the left of Z. 1. C sits 3rd to the left of D, who faces north.
5. Y is not sitting adjacent to S. 2. B sits to the immediate right of C.
Case 1: 3. A sits 3rd to the left of B, who faces north, so
Case I:

Case 2:

6. Not more than 2 persons are sitting between U and T.


7. Y is sitting to the right of Z.
The final arrangement is as follows: Case II:
4. E neither sits adjacent to A nor sits at any extreme
ends,
5. G sits to the right of F, who faces south.
1@W means one student scores more than W. Less than
6. Person, who sits to the immediate left of G, faces
two students score less than X. @2V means two students
south.
score less than V. T < Z means T scores more than Z. Y
7. F doesn’t sit to the left of E. so case I is rejected.
< U means Y scores more than U. T~< U means T
8. A and E face the same direction.
doesn’t score more than U. Z~<S means Z doesn’t score
Page 938 of 1334
Subscribe The Xpress Video Course & Mock Test Package for Bank & Insurance Exams
If there are any suggestions/ errors in our PDFs Feel Free to contact us via this email: admin@exampundit.in
Ultra Practice Bundle PDF
SBI Clerk/ RBI Assist. Mains – Reasoning
The final seating arrangement is given below: left of one, who is 10 years old. N is not opposite to O or
M. So, we have,
Case 1:

A#2: Two persons are heavier than A.


B 2#: Two persons are lighter than B.
A# C: A is heavier than C.
D and F are lighter than C. G is not lighter than E. D is
not the lightest person So, we have,
G>E>A>C>B>D>F
Answers:
16. C
17. B
18. D
19. C
Case 2:
20. B
Solutions 16-20
Decoding:
T 2+ O: T sits second to the left of O.
N % Q: N’s age is twice that of Q.
S % P: S’s age is twice that of P.
(15yrs old) +2 S: 15 years old person sits second to the
right of S.
Starting point: Here, we have three statements
associated with O. So, we must fix the position of O and
then proceed forward accordingly.
Clues: R sits third to the right of O, who is 5 years old.
M sits to the immediate right of P. N sits to the immediate Clues: 15 years old person sits second to the right of S.
Case 1:

Page 939 of 1334


Subscribe The Xpress Video Course & Mock Test Package for Bank & Insurance Exams
If there are any suggestions/ errors in our PDFs Feel Free to contact us via this email: admin@exampundit.in
Ultra Practice Bundle PDF
SBI Clerk/ RBI Assist. Mains – Reasoning

Answers:
21. D

Case 2: 22. A
23. C
24. A
25. B
Solutions 21-25
1. L is sitting 2nd to the right of J and both of them are
facing the same direction, either north or south.
2. O is sitting 3rd to the left of L.
3. Immediate neighbors of O are facing the opposite
direction of O.
4. M is sitting adjacent to O.
Clue: T is not elder than that of N. 5. Two persons are sitting between M and P.
Inference: Case 1 is rejected as T cannot be 35 years old. 6. N is sitting 2nd to the left of P and both of them are
facing the same direction.
7. K is sitting adjacent to N.

Page 940 of 1334


Subscribe The Xpress Video Course & Mock Test Package for Bank & Insurance Exams
If there are any suggestions/ errors in our PDFs Feel Free to contact us via this email: admin@exampundit.in
Ultra Practice Bundle PDF
SBI Clerk/ RBI Assist. Mains – Reasoning
Case 1: When both L and J are facing north. Persons ii) Ankit # Abhishek → Ankit scored more than
sitting at the extreme ends are facing the opposite Abhishek.
direction, so this case is not possible. iii) Abhinav & Abhishek → Abhinav scored less than
Abhishek.
From the above statements, we can conclude;
Ankit > Abhishek > Abhinav.
Case 2: When both L and J are facing south. iv) Aman &*1 means Aman scored less than only one
The final arrangement is as follows: person.
From the above statement, we conclude that Aman is the
second highest scorer.
v) Abhishek 4$ → Abhishek is 4th highest scorer.
K @ J means K is older than only J. L+M; L%N means
The order becomes:
L is older than M but younger than N.
Ankit > Aman > _ > Abhishek > Abhinav > Anand >
O is older than N. M is not the 3rd youngest person.
Atul/Ashish > Ashish/Atul
K is older than only J, L is older than M but younger than
vi) Ankit # Anish → Ankit scores more than Anish.
N.
Therefore, the final order becomes:
The final arrangement is as follows:
Ankit > Aman > Anish > Abhishek > Abhinav > Anand
O>N>L>M>P>K>J
> Atul/Ashish > Ashish/Atul
Answers:
vii) The one who is 2$ scored 80 runs → The person who
26. C
scored 2nd highest score 80 runs i.e. Aman scored 80
27. D
runs.
28. A
A2! means A has the second lowest score.
29. E
From the above arrangement, we can see that, it can be
30. B
either Atul or Ashish.
Solution 26-30
Answers:
i) Anand #* (Atul, Ashish) → Anand scored more than
31. C
only Atul and Ashish.
32. D
From the above statement, we can conclude;
33. D
_ > _ > _ > _ > _ > Anand > Atul/Ashish > Ashish/Atul
34. A

Page 941 of 1334


Subscribe The Xpress Video Course & Mock Test Package for Bank & Insurance Exams
If there are any suggestions/ errors in our PDFs Feel Free to contact us via this email: admin@exampundit.in
Ultra Practice Bundle PDF
SBI Clerk/ RBI Assist. Mains – Reasoning
35. C 36. E
Solutions 31-35 37. B
1. At most one person sitting to the left of P, who faces 38. C
towards the north direction so, either P sits at the extreme 39. B
end or second from the left end. 40. D
2. Only one person sits between P and V. T sits second Solutions 36-40
to the left of V and both are facing the same direction, 1. Now, after decoding, P % W - P sits third to the
which means V faces the south direction. right of W.
2. S # T - S sits second to the right of T.
3. W $ S - W has double number of books than S.
4. R $ U - R has double number of books than U.
5. T sits adjacent to P.
6. The one, who has 6 books, sits to the immediate
3. R sits second to the right of U and neither of them is left of W.
an immediate neighbor of P. R sits immediate left of Q 7. Q, who has 5 books sits adjacent to S.
which is not possible in case II so, case II is invalid. 8. Neither R nor W has highest number of books.
4. S and R faces the same direction but opposite to U 9. U sits adjacent to one, who has 1 book.
which means both S and R faces south. 10. U does not have 6 books and is not opposite to S.
The final arrangement is as follows: 11. R has more books than W. So, we have, two
cases,
Case 1:

P @ R: P is taller than R.
P 2#: Two persons are taller than P.
T 2@: Two persons are shorter than T.
S is taller than Q, who is taller than R. U is taller than V
but not as much as R.
. So, we have,
S>Q>P>R>T>U>V
Answers 36-40:
Page 942 of 1334
Subscribe The Xpress Video Course & Mock Test Package for Bank & Insurance Exams
If there are any suggestions/ errors in our PDFs Feel Free to contact us via this email: admin@exampundit.in
Ultra Practice Bundle PDF
SBI Clerk/ RBI Assist. Mains – Reasoning
Answers 41-45:
41. B
42. D
43. B
44. A
45. D
Solutions 41-45
1. Now, after decoding, G $ H - G sits third to the left of
H.
2. B % G - B has double the number of coins than G.
Case 2:
3. D % C- D has double the number of coins than C.
Now, D sits opposite to one, who sits adjacent to C.
D sits one of the longer side of the table and has a coin
which is multiple of 6.
H is adjacent to C who sits at the corner.
A sits immediate right of E, who has 71 coins.
E and C are not adjacent to each other.
A and D are not adjacent to each other.
F has 26 coins and does not sit adjacent to D.

Now, V is not opposite to T. So, case 2 is rejected. So, H has more coins than A.

we have, The one, who has 30 balls, sits adjacent to B.


E sits at corner.
So, D has 30 balls. So, we have
Now, the person, who has 20 balls, does not sit adjacent
to D. So the final arrangement becomes,

Page 943 of 1334


Subscribe The Xpress Video Course & Mock Test Package for Bank & Insurance Exams
If there are any suggestions/ errors in our PDFs Feel Free to contact us via this email: admin@exampundit.in
Ultra Practice Bundle PDF
SBI Clerk/ RBI Assist. Mains – Reasoning
Case I: When C sits second from the left end.

Case II: When C sits second from the right end of the

row.
Answers 46-50
F and G face the same direction.
46. D
So, in case II if F and G both face north or south then the
47. D
number of persons facing same direction is more than
48. D
two.
49. A
So, case II is invalid.
50. E
So, final arrangement is as follows:
Solutions 46-50
C sits second from one of the ends of the row and is
facing towards south.
A sits to the immediate right of C and both of them are
Now let’s decode their weights
facing opposite direction with respect to each other.
As C @ B means C is not lighter than B.
B sits second to the right of A.
D @ B means D is not lighter than B.
Only two persons are sitting between B and E.
F # B means F is not heavier than B.
G sits to the immediate right of E and second to the right
E @ G means E is not lighter than G.
of D, who faces north direction.
G # F means G is not heavier than F.
C and B face opposite direction with respect to each
A # B means A not heavier than B, so
other.
C > D > B > A/F > F/A > E > G
Or C > D > B > F/A > E > A/F > G

Page 944 of 1334


Subscribe The Xpress Video Course & Mock Test Package for Bank & Insurance Exams
If there are any suggestions/ errors in our PDFs Feel Free to contact us via this email: admin@exampundit.in
Ultra Practice Bundle PDF
SBI Clerk/ RBI Assist. Mains – Reasoning
Inequality
Directions (1-5): Study the following information and III. M>H
answer the question:
3. Statement:
a) Only I follows
D>C≥B<A; X≤U<S=L; C<U<L
b) Only III follows
Conclusion:
c) Either II or III and I follows
I. B<S
d) Either I or III follows
II. A≥S
e) Either I or III and II follows
III. S>A
1. Statement:
4. Statement:
P≥V=S<B≤K; K=R<L≥H<C≤T
2<6≥9<5; 7<3=8≥6; 1>3>4
Conclusion:
Conclusion:
I. P≥R
I. 5>8
II. S<H
II. 2<4
III. R>P
III. 1>9
2. Statement:
5. Statement:
U<S=R≥Q>P;
E≤I<R≤Q=S;
S>K>H;
M=H≥Y<O=R<T;
K<L≤M>N
X=D≤Q>F
Conclusion:
Conclusion:
I. P<K
I. H≤X
II. Q≤M
Page 945 of 1334
Subscribe The Xpress Video Course & Mock Test Package for Bank & Insurance Exams
If there are any suggestions/ errors in our PDFs Feel Free to contact us via this email: admin@exampundit.in
Ultra Practice Bundle PDF
SBI Clerk/ RBI Assist. Mains – Reasoning
II. E<S III. S<P

III. X<H 8. Statement:

Directions (6 - 10): Study the following information M≤Q<T=F≥H; I≤Y>X=T<N; S≥L≥Y≤R<K


and answer the question:
Conclusion:
a) Only III follows
I. K≥I
b) Only I follows
II. M<K
c) Only I and either II or III follows
III. S>H
d) None follows
9. Statement:
e) Both II and III follows
D≥G>J≤O=V; C<E≤J>Z; R>E=B<A
6. Statement:
Conclusion:
@≥$>#≤₩; €>£=$≥!<%; &≤!≤¥>?
I. D>C
Conclusion:
II. R≤O
I. %>&
III. O<R
II. $>@
10. Statement:
III. ¥≥£
S=N≤R>P<Q; F<M<P=T≤Y; H<T<R≥X
7. Statement:
Conclusion:
G<R≤P=Y>T; X>Q≤P>Z; U≤L≥Q>N=S
I. H≥P
Conclusion:
II. T<H
I. R<L
III. F<S
II. G<T

Page 946 of 1334


Subscribe The Xpress Video Course & Mock Test Package for Bank & Insurance Exams
If there are any suggestions/ errors in our PDFs Feel Free to contact us via this email: admin@exampundit.in
Ultra Practice Bundle PDF
SBI Clerk/ RBI Assist. Mains – Reasoning
Directions (11 - 15): Study the following information U>S≥T>W<I; E>S≥Q>R; A=L>E≥Y>X
and answer the question:
Conclusion:
a) Only II follows
I. R<Y
b) Only I and either II or III follows
II. W≤A
c) Both I and III follows
III. A<W
d) None follows
14. Statement:
e) Either II or III follows
J≥H=G>L>P<A; E<Y≤Q<B≤U;
11. Statement: M>N<H=W≥Q

L≥K>H=N>A≤V; R≥D=N≤O≥I Conclusion:

Conclusion: I. E<J

I. L>A II. U>L

II. R>H III. Y≤G

III. R=H 15. Statement:

12. Statement: B≥A≥C≥D; Q≤C≥P; W=B>E

R>Q>P>I=T; L<M≤U; U>P≥F=B Conclusion:

Conclusion: I. Q<E

I. T<F II. D≤W

II. U≥B III. P≤D

III. R>L Directions (16 - 20): Study the following information


and answer the question:
13. Statement:
a) Only I follows
Page 947 of 1334
Subscribe The Xpress Video Course & Mock Test Package for Bank & Insurance Exams
If there are any suggestions/ errors in our PDFs Feel Free to contact us via this email: admin@exampundit.in
Ultra Practice Bundle PDF
SBI Clerk/ RBI Assist. Mains – Reasoning
b) Only II follows II. Q≤Z

c) Both I and II follows III. Z>Q

d) Only I and either II or III follows 19. Statement:

e) Either II or III follows O=P≤D<H>Z; X>M≥U=P<E; U=B<I

16. Statement: Conclusion:

C<D<E>H≤L=K≤O; P>T=I≤H≥M I. X>O

Conclusion: II. E>I

I. O≥I III. H<O

II. M≥T 20. Statement:

III. P>M N≥E>M=V>L≤P; W≥D=V≤T≥R

17. Statement: Conclusion:

Q<R<W≤V≥T; M=W<A<B≥U≥E I. N≥R

Conclusion: II. W≥M

I. M<B III. P<N

II. V>Q Directions (21 - 25): Study the following information


and answer the question:
III. Q<U
A%B means "A is either smaller than or equal to B"
18. Statement:
A@B means "A is smaller than B"
F>G≥J≤M=N; Q≤Y<X>Z=M; X<P
A&B means "A is equal to B"
Conclusion:
A$B means "A is either greater than or equal to B"
I. N<P

Page 948 of 1334


Subscribe The Xpress Video Course & Mock Test Package for Bank & Insurance Exams
If there are any suggestions/ errors in our PDFs Feel Free to contact us via this email: admin@exampundit.in
Ultra Practice Bundle PDF
SBI Clerk/ RBI Assist. Mains – Reasoning
A*B means "A is greater than B" Conclusion:

a) Only I follows I. G@D

b) Only III follows II. E$H

c) Only III and either I or II follows III. F@B

d) Either I or II follows 24. Statement:

e) All follows K&L, L*M, M$N, N@O, O%P, P*Q

21. Statement: Conclusion:

M*N$P; Q&R@P; N%T I. N@Q

Conclusion: II. M*Q

I. R*N III. L*N

II. T$M 25. Statement:

III. Q@T Q$R@S; S%T@U; U&V*W

22. Statement: Conclusion:

G*H$I; I@J&K; K@L%M I. W@U

Conclusion: II. T*R

I. G*J III. S@V

II. J$G Directions (26 - 30): Study the following information


and answer the question:
III. I@M
A×B means "A is either smaller than or equal to B"
23. Statement:
A-B means "A is smaller than B"
B@D, D*E, E$F, F&G, G*H

Page 949 of 1334


Subscribe The Xpress Video Course & Mock Test Package for Bank & Insurance Exams
If there are any suggestions/ errors in our PDFs Feel Free to contact us via this email: admin@exampundit.in
Ultra Practice Bundle PDF
SBI Clerk/ RBI Assist. Mains – Reasoning
A#B means "A is equal to B" 28. Statement:

A÷B means "A is either greater than or equal to B" M×N, O÷P, P#Q, R÷S, Q+R, N÷O

A+B means "A is greater than B" Conclusion:

a) Both II and III follows I. S-P

b) Only III follows II. M×Q

c) Both I and III follows III. Q-M

d) Only I and either II or III follows 29. Statement:

e) All follows T-U÷V; V#W+Y; Y×Z#L

26. Statement: Conclusion:

J+K, O×P, N#O, M-N, L×M, K÷L I. T+Z

Conclusion: II. Y-U

I. J-M III. L÷Y

II. P÷L 30. Statement:

III. N+L G-H×I; I#J+K; K÷L#M

27. Statement: Conclusion:

A#B, C×D, B-C, E+F, F÷G, D÷E I. J÷H

Conclusion: II. M-I

I. D+A III. J+L

II. E×C Directions (31 - 35): Study the following information


and answer the question:
III. G-D

Page 950 of 1334


Subscribe The Xpress Video Course & Mock Test Package for Bank & Insurance Exams
If there are any suggestions/ errors in our PDFs Feel Free to contact us via this email: admin@exampundit.in
Ultra Practice Bundle PDF
SBI Clerk/ RBI Assist. Mains – Reasoning
A!B means "A is neither smaller than nor equal to B" I. A^Y

A&B means "A is not smaller than B" II. A%Y

A%B means "A is neither smaller than nor greater III. Y!M
than B"
33. Statement:
A^B means "A is neither greater than nor equal to B"
T%U&V; V!X&N; E#O^M#N
A#B means "A is not greater than B"
Conclusion:
a) Only I follows
I. E!U
b) Only II follows
II. E#U
c) Only III and either a or b
III. V^E
d) Either II or III or I follows
34. Statement:
e) None follows
O%H&M#B#A; O#X^Y
31. Statement:
Conclusion:
M&H!O; B!A%L#X%O
I. A!Y
Conclusion:
II. A^Y
I. M!A
III. A%Y
II. M%A
35. Statement:
III. H#B
L&Y; X%N#M^O; O^H%Y
32. Statement:
Conclusion:
M^A%L#I; Y%X&I; H&O!M
I. L!X
Conclusion:
II. X!O

Page 951 of 1334


Subscribe The Xpress Video Course & Mock Test Package for Bank & Insurance Exams
If there are any suggestions/ errors in our PDFs Feel Free to contact us via this email: admin@exampundit.in
Ultra Practice Bundle PDF
SBI Clerk/ RBI Assist. Mains – Reasoning
III. L&N 37. Statement:

Directions (36 - 40): Study the following information E$F₩G$H; H&T₩U¥V


and answer the question:
Conclusion:
A€B means "A is neither smaller than nor equal to B"
I. H€E
A&B means "A is not smaller than B"
II. T₩V
A¥B means "A is neither smaller than nor greater than
III. E¥H
B"
38. Statement:
A₩B means "A is neither greater than nor equal to
B" I¥J&K; K€L$Q₩R¥S

A$B means "A is not greater than B" Conclusion:

a) Only II follows I. I&R

b) Both I and II follows II. I₩R

c) Either I or II follows III. L€I

d) All follows 39. Statement:

e) None follows M$N₩O€P¥Q; Q€R&S₩T

36. Statement: Conclusion:

A€C&D$Y; Y₩B¥Z&X I. M₩Q

Conclusion: II. P€S

I. Z€Y III. N₩R

II. D₩A 40. Statement:

III. B&X G¥H$I₩J; J¥K&L¥M&N

Page 952 of 1334


Subscribe The Xpress Video Course & Mock Test Package for Bank & Insurance Exams
If there are any suggestions/ errors in our PDFs Feel Free to contact us via this email: admin@exampundit.in
Ultra Practice Bundle PDF
SBI Clerk/ RBI Assist. Mains – Reasoning
Conclusion: (43-44): Find the appropriate statement for the given
conclusion
I. M$K
43. Conclusion: A>D, E≤B
II. J€G
a) A>B≥D<C<E
III. L₩I
b) A>C≥D<E<B
Directions (41 - 42): Study the following information
and answer the question: c) A<B=D≤C≤E

41. If "+" means subtraction, "-" means division, d) A>B≥D≥E<C


"×" means addition and "÷" means
e) None of these
multiplication, then 20÷4-8×2÷3 is
44. Conclusion: U>Z, V<Y
a) 2
a) X≥U>Y≥V<Z
b) 1
b) U>Y>Z≤X<V
c) 15
c) U>Y≥Z>X≥V
d) 16
d) Y<Z<V<U≥X
e) None of the above
e) Both c and d
42. If "+" and "-" are interchanged, then which of
45. Which of the following expressions is true, if
the following is false?
A≥D>C<B=E≥F?
a) 4-3+2=5
a) F<B
b) 9+6-7=10
b) A>B
c) 2-2+3=1
c) A>C
d) 4-1+3=2
d) A≥C
e) All are true
e) E<C
Page 953 of 1334
Subscribe The Xpress Video Course & Mock Test Package for Bank & Insurance Exams
If there are any suggestions/ errors in our PDFs Feel Free to contact us via this email: admin@exampundit.in
Ultra Practice Bundle PDF
SBI Clerk/ RBI Assist. Mains – Reasoning
46. If the expression R<Y≤H>Q, H≤Z and Y<K are e) =
true, which of the following
(49-50): Which of the should be placed in the blank
Conclusion will be definitely false? spaces respectively ( from left to right)?

a) R<K 49. If Q<R is true, then __<__≤ __>__

b) Y≤Z a) QTSR

c) R>Q b) TQSR

d) R>H c) RTQS

e) None of these d) STRQ

(47-48): Which of the following symbols should replace e) None of the above
the question mark
50. If E>U is definitely true, then
47. If T<S, K>Q are true, K>S≥V?Y≥T?Q=V E__R__N__U__M

a) ≥, = a) >, =, ≤, =

b) >, ≤ b) <, >, =, ≥

c) >, > c) ≥, >, =, >

d) ≤, > d) ≥, =, ≥, <

e) Either a or c e) None of the above

48. If Q≥X, N>R are true, Q≥T=R? X≤F<N ANSWERS:

a) > 1. d

b) ≥ I. False P≥R
II. False
c) ≤
III. False R>P so either I or III follows
d) Either a or b 2. b
Page 954 of 1334
Subscribe The Xpress Video Course & Mock Test Package for Bank & Insurance Exams
If there are any suggestions/ errors in our PDFs Feel Free to contact us via this email: admin@exampundit.in
Ultra Practice Bundle PDF
SBI Clerk/ RBI Assist. Mains – Reasoning
I. False I. FALSE
II. False II. TRUE M<K
III. True M>H III. TRUE S>H
3. c 9. c

I. True B<S I. TRUE D>C


II. False A≥S II. FALSE R≤O
III. False A<S so either II or III and I III. FALSE O<R So either II or III and I
follows follows
4. b 10. d

I. False I. FALSE
II. False II. FALSE
III. True 1>9 III. FALSE
5. e 11. b

I. False H≤X I. True L>A


II. True E<S II. False R>H Here statement
III. False H>X so either I or III and II (R≥H=N=D)
follows III. False R=H So either II or III and I
6. b follows
12. d
I. True %>&
II. False I. False
III. False II. False
7. a III. False
13. d
I. False
II. False I. False
III. True S<P II. False W ≤ A
8. e III. False A<W
14. c
Page 955 of 1334
Subscribe The Xpress Video Course & Mock Test Package for Bank & Insurance Exams
If there are any suggestions/ errors in our PDFs Feel Free to contact us via this email: admin@exampundit.in
Ultra Practice Bundle PDF
SBI Clerk/ RBI Assist. Mains – Reasoning
I. True E<J II. True W≥M
II. False III. False
III. True Y≤G so both I and III follows 21. b
15. a
I. False
I. False II. False
II. True D≤W III. True Q<T
III. False 22. c
16. a
I. False G>J
I. True O≥J II. False J≥G
II. False III. True I<M so either I or II and III
III. False follows
17. c 23. a

I. True M<B I. True G<D


II. True V>Q II. False
III. False III. False
18. d 24. b

I. True N<P I. False


II. False Q≤Z II. False
III. False Z>Q so I and either II or III III. True L>N
follows 25. e
19. a
I. True W<U
I. True X>O II. True T>R
II. False III. True S<V
III. False 26. b
20. b
I. False
I. False II. False

Page 956 of 1334


Subscribe The Xpress Video Course & Mock Test Package for Bank & Insurance Exams
If there are any suggestions/ errors in our PDFs Feel Free to contact us via this email: admin@exampundit.in
Ultra Practice Bundle PDF
SBI Clerk/ RBI Assist. Mains – Reasoning
III. True N>L III. True Y>M so either I or II and III
27. c follows
33. e
I. True D>A
II. False I. False
III. True G<D II. False
28. d III. False
34. d
I. True S<P
II. False M≤Q I. False A>Y
III. False Q<M so I and Either II or III II. False A<Y
follows III. False A=Y Here all the three
29. a inequalities are compared.so it should
be
I. False
Either I or II or III
II. True Y<U
follows
III. True L≥Y
30. e 35. a

I. True J≥H I. True L>X


II. True M<I II. False
III. True J>L III. False
31. a 36. d

I. True M>A I. True Z>Y


II. False II. True D<A
III. False III. True B≥X
32. c 37. b

I. False A<Y Here A≤Y I. True H>E


II. False A=Y II. True T<V
III. False

Page 957 of 1334


Subscribe The Xpress Video Course & Mock Test Package for Bank & Insurance Exams
If there are any suggestions/ errors in our PDFs Feel Free to contact us via this email: admin@exampundit.in
Ultra Practice Bundle PDF
SBI Clerk/ RBI Assist. Mains – Reasoning
38. c 45. c

I. False I≥R R<Y≤H(R>H is definitely false)


II. False I<R
46. d C<B=E(E>C)
III. False so either I or II follows
39. a 47. c

I. False K>S≥V>Y≥T>Q=V (T<S, K>Q)


II. True P>S
48. e
III. False
40. b Q≥F, N>R (Q≥T=R=X≤F<N)

I. True M≤K 49. d


II. True J>G
S<T≤R>Q (Q<R)
III. False
41. d . 20×4÷8+2×3=16 50. c

42. e E≥R>N=U≥M (E>U)

4+3-2=5

9-6+7=10

2+2-3=1

4+1-3=2

Hence all are true

43. d

A>B≥D (A>D), E≤D≤B (E≤B)

44. c

U>Y≥Z (U>Z), Y≥Z>X≥V (V<Y)

Page 958 of 1334


Subscribe The Xpress Video Course & Mock Test Package for Bank & Insurance Exams
If there are any suggestions/ errors in our PDFs Feel Free to contact us via this email: admin@exampundit.in
Ultra Practice Bundle PDF
SBI Clerk/ RBI Assist. Mains – Reasoning
Syllogism
Directions ( 1-5): Study the following information and Conclusion:
answer the questions below: I. All Sun being Star is a possibility
a) Only II follows II. Some Stars are Sun
b) Either I or II and III follows III. Some Planets are Earth
c) Both I and II follows 4.Statement:
d) Both II and III follows All Mathematics are Economics
e) None follows Only a few Mathematics are Chemistry
1. Statement: No Chemistry is Physics
Only a few Genes are DNA Conclusion:
Only DNA are RNA I. All Chemistry are Economics
No RNA is Protein II. Some Economics are Chemistry
Conclusion: III. All Mathematics being Physics is a possibility
I. No Gene is Protein 5.Statement:
II. Some proteins are Genes Some twos are threes
III. Some Genes are not DNA No four is ten
2. Statement: Only a few tens are threes
Some A are B Conclusion:
Only a few B are C I. No three is four
No A is D II. Some twos are four is a possibility
Conclusion: III. Some threes are twos
I.At least some A are C Directions ( 6 - 10): Study the following informations
II. All B are C is a possibility and answer the questions below:
III. No D is C a) Only I follows
3.Statement: b) Only II follows
All Planets are Stars c) Only III follows
Some Stars are Earth d) Both I and III follows
Only a few Suns are Planets e) Either II or III and I follows

Page 959 of 1334


Subscribe The Xpress Video Course & Mock Test Package for Bank & Insurance Exams
If there are any suggestions/ errors in our PDFs Feel Free to contact us via this email: admin@exampundit.in
Ultra Practice Bundle PDF
SBI Clerk/ RBI Assist. Mains – Reasoning
6.Statement: I. All pens can be table
Some keyboards are mouse II. All pencils being tables is a possibility
Only a few dashboards are pianos III. Some notebooks are tables
All pianos are mouse 10.Statement:
Conclusion: No black is blue
I. All mouse being dashboard is a possibility Some black are red
II. No keyboard is piano Only a few blues are orange
III. Some keyboards are piano Conclusion:
7.Statement: I. All red can never be blue
Some dairy milks are KitKat II. Some reds are blues
Only a few KitKat are milky bar III. All orange being black is a possibility
All dairy milk are snickers Directions (11 - 15): Study the following information
Conclusion: and answer the questions below:
I. All KitKat can never be milky bar a) Only I follows
II. Some dairy milks are milky bar b) Only II follows
III. Some sneakers are KitKat c) Both I and II follows
8. Statement: d) All follows
Only a few radios are TVs e) Either I or III and II follows
Only a few laptops are cables 11.Statement:
All cables are TVs Some internet are intranet
Conclusion: Some intranet are not files
I. All laptop can never be TVs Only a few files are documents
II. Some radio being cable is a possibility Conclusion:
III. Some radios are laptops I. Some files are not intranet
9.Statement: II. All documents being intranet is a possibility
Some pens are pencils III. All files are internet
Only a few pencils are tables 12. Statement:
All tables are notebooks Only a few Roses are sunflower
Conclusion: Only a few sunflowers are jasmine

Page 960 of 1334


Subscribe The Xpress Video Course & Mock Test Package for Bank & Insurance Exams
If there are any suggestions/ errors in our PDFs Feel Free to contact us via this email: admin@exampundit.in
Ultra Practice Bundle PDF
SBI Clerk/ RBI Assist. Mains – Reasoning
Some sunflowers are not lilies II. All tables can never be windows
Conclusion: III. Some benches are windows
I. Some roses can be jasmine Directions ( 16 - 20): Study the following information
II. Some roses are jasmine and answer the questions below:
III. Some lilies are rose a) Only III follows
13. Statement: b) Both I and II follows
Some editors are not directors c) Both I and III follows
All directors are producers d) Either I or II and III follows
No producer is screenplay e) All I, II, III follows
Only a few screenplays are actors 16.Statement:
Conclusion: Only a few cars are van
I .All producer can be editor All van are jeep
II. No director is screenplay No jeep is bus
III. Some actors are editors Conclusion:
14. Statement: I. Some buses are car
Only GMs are PMs II. No bus is car
Only a few CMs are president III. All jeep are car is a possibility
No president is PM 17. Statement:
Conclusion: Some fruits are jam
I. Some GMs are presidents Only a few jams are butter
II. All CM being GM is a possibility All butter are gee
III. No GM is president Conclusion:
15. Statement: I. All gee can be jam
Some doors are tables II. Some butter are not jam
Only tables are benches III. All butter are fruit is a possibility
Only a few tables are windows 18. Statement:
No door is clock Some apples are mango
Conclusion: No mango is papaya
I. No bench is window Only a few papaya are kiwi

Page 961 of 1334


Subscribe The Xpress Video Course & Mock Test Package for Bank & Insurance Exams
If there are any suggestions/ errors in our PDFs Feel Free to contact us via this email: admin@exampundit.in
Ultra Practice Bundle PDF
SBI Clerk/ RBI Assist. Mains – Reasoning
Conclusion: All bats are football
I. Some papayas are not kiwi Only a few footballs are cricket
II. All mangoes are kiwi is a possibility Conclusion:
III. Some apples are not kiwi I. Some toy can never be cricket
19. Statement: II. All cricket can be bat is a possibility
All balloon are ball III. Some cricket are not football
Some balloons are ribbon 22. Statement:
Only a few ribbons are stick Only a few mobile are tablet
Conclusion: Some tablets are laptop
I. Some ball is stick No laptop is palmtop
II. Some sticks are not ribbon Conclusion:
III. All balloon are stick is a possibility I. Some tablets are not mobile
20. Statement: II. All palmtop is mobile is a possibility
All chairs are table III. Some tablet is not palmtop
No table is desk 23. Statement:
Only a few desks are pen All markers are eraser
Conclusion: Some eraser are pencil
I. Some pens are not chair Only a few pencils are scale
II. Some chairs are not desk Conclusion:
III. Some pens are chair is a possibility I. All scale are eraser is a possibility
Directions (21 - 25): Study the following information II. Some scale are not marker
and answer the questions below: III. All markers are pen
a) Only I follows 24. Statement:
b) Only II follows No fan is AC
c) Either I or III and II follows Some AC is cooler
d) Both II and III follows Only a few cooler is air cooler
e) All I, II, III follows Conclusion:
21. Statement: I. Some coolers are not fan
No toy is bat II. All air cooler are cooler is a possibility

Page 962 of 1334


Subscribe The Xpress Video Course & Mock Test Package for Bank & Insurance Exams
If there are any suggestions/ errors in our PDFs Feel Free to contact us via this email: admin@exampundit.in
Ultra Practice Bundle PDF
SBI Clerk/ RBI Assist. Mains – Reasoning
III. All AC are air cooler is a possibility I. Some B are C
25. Statement: II. No D is A is a possibility
Some TVs are radio III. All C are D is a possibility
Only a few radio are speakers 28. Statement:
All speakers are projector Some A are not B
Conclusion: No B is C
I. Some speakers are TVs Only a few B is D
II. All projector is TV is a possibility Conclusion:
III. No TV is speaker I. Some D are not C
Directions (26 - 30): Study the following information II. Some D are A is a possibility
and answer the questions below: III. All A are B is a possibility
a) Only II follows 29. Statement:
b) Only III follows All A are B
c) Either I or III follows No B is C
d) Both I and II follows Some C are not D
e) All I, II and III follows Conclusion:
26. Statement: I. Some D are not A
Some A are not B II. All C is A is a possibility
Some B are C III. All D are A
Only a few C are D 30. Statement:
Conclusion: No A is B
I. Some B are not D Some B are not C
II. All A are D is a possibility Only a few B are D
III. All A are B is a possibility Conclusion:
27. Statement: I. No A is C
All A are B II. Some D are not B
Only a few B are C III. All C are both B & D is a possibility
Some C are not D Directions (31 - 35): Study the following information
Conclusion: and answer the questions below:

Page 963 of 1334


Subscribe The Xpress Video Course & Mock Test Package for Bank & Insurance Exams
If there are any suggestions/ errors in our PDFs Feel Free to contact us via this email: admin@exampundit.in
Ultra Practice Bundle PDF
SBI Clerk/ RBI Assist. Mains – Reasoning
a) Only I follows 34. Statement:
b) Both II and III follows All B are A
c) Both I and II follows Some B are not C
d) Both I and III follows Only a few C are D
e) Either II or III and I follows Conclusion:
31. Statement: I. Some A are D is a possibility
All A are B II. Some D are not C
Some B are not C III. All D are B is a possibility
Only a few C are D 35. Statement:
Conclusion: Some A are not B
I. All A are C is a possibility Some B are not C
II. Some C are not D Only a few B are D
III. All B can be C Conclusion:
32. Statement: I. All A are C is a possibility
No A is B II. All D are B
Some A are C III. Some D are not B
Some C are B Directions (36 - 40): Study the following information
Conclusion: and answer the questions below:
I. All C are B a) Either I or II follows
II. Some B can never be A b) Both I and II follows
III. Some C are not A c) Both II and III follows
33. Statement: d) All I, II and III follows
Some A are not B e) Only III follows
Only a few B are C 36. Statement:
All C are D All A are B
Conclusion: No A is C
I. All D are A is a possibility Some C are not D
II. Some A are C Conclusion:
III. All A are B is a possibility I. All B are C is a possibility

Page 964 of 1334


Subscribe The Xpress Video Course & Mock Test Package for Bank & Insurance Exams
If there are any suggestions/ errors in our PDFs Feel Free to contact us via this email: admin@exampundit.in
Ultra Practice Bundle PDF
SBI Clerk/ RBI Assist. Mains – Reasoning
II. All D are B is a possibility Only C are D
III. Some B are A Conclusion:
37. Statement:-- I. Some C are not B
Only A is B II. All C are D is a possibility
Some A is C III. All C are A is a possibility
Some C is not D Directions ( 41 - 46): Study the following information
Conclusion: and answer the questions below:
I. All D are C a) Either I or III follows
II. Some D are not C b) Only II follows
III. All Aare B is a possibility c) Both I and II follows
38. Statement: d) Both II and III follows
Only C is B e) All I, II and III follows
Only a few C is A 41. Statement:
Some A are not D Only a few A is B
Conclusion: Only a few C is B
I. All C are D is a possibility Some C are not D
II. Some A is not C Conclusion:
III. Some A are D is a possibility I. Some A is C
39. Statement: II. All C are B is a possibility
Only a few A is B III. No C is A
Only B is C 42. Statement:
Some B are not D Only few Tiger are Lion
Conclusion: Only Lion are Cheetah
I. Some A are not B Some Lion are Elephant
II. Some D are A is a possibility Conclusion:
III. All C are A is a possibility I . Some Elephant are Tiger
40. Statement: II. Some Tiger can be Cheetah
All A are B III. No Tiger is Elephant
Some B are not C 43. Statement:

Page 965 of 1334


Subscribe The Xpress Video Course & Mock Test Package for Bank & Insurance Exams
If there are any suggestions/ errors in our PDFs Feel Free to contact us via this email: admin@exampundit.in
Ultra Practice Bundle PDF
SBI Clerk/ RBI Assist. Mains – Reasoning
All Twitter are Facebook II. Some Delhi can be Chennai
Only a few Facebook are Orkut III. Some Delhi can never be a Chennai
Only Orkut are Whatsapp Directions ( 47 - 50): Study the following information
Conclusion: and answer the questions below:
I. All Orkut is Twitter is a possibility Which of the conclusion definitely does not follow (also
II. No Facebook is Whatsapp not a possibility of occurrence)
III.Some Facebook are not Orkut 47. Statement:
44. Statement: Some leaves are roots
Some Google are Opera Only a few roots are branches
Only Opera are UC Some leaves are not trees
Only a few Opera are Yahoo Conclusion:
Conclusion: a) Some trees are branches
I. Some Yahoo are not Opera b) Some leaves are roots
II. No Google is UC c) Some roots are not branches
III. All Opera are UC is a possibility d) All leaves are branches
45. Statement: e) All roots can be branches
Only Mouse is a Desktop 48. Statement:
Only a few Mouse is CPU Some kings are queen
No CPU is a Camera Only a few emperors are king
Conclusion: No queen is princess
I. No Desk is Camera Conclusion:
II. Some Desk is not a CPU a) Some queen are emperor
III. All Mouse can be a Camera b) Some princess are king
46. Statement: c) All emperor being queen is a possibility
No Chennai is a Bangalore d) All king being princess is a possibility
Only Bangalore are Hyderabad e) All kings are queen
Only a few Bangalore are Delhi 49. Statement:
Conclusion: Some countries are not continent
I. No Delhi is Hyderabad Only villages are continent

Page 966 of 1334


Subscribe The Xpress Video Course & Mock Test Package for Bank & Insurance Exams
If there are any suggestions/ errors in our PDFs Feel Free to contact us via this email: admin@exampundit.in
Ultra Practice Bundle PDF
SBI Clerk/ RBI Assist. Mains – Reasoning
Only a few countries are towns Some consonants are vowels
Conclusion: Only a few letters are consonants
a) All countries being villages is a possibility Conclusion:
b) Some towns are villages a) Some symbols are not consonants
c) All continents are towns b) Some vowels are letters
d) All countries can never be towns c) All letters are consonants is a possibility
e) Some villages are not countries d) All symbols are letters
50. Statement: e) Some symbols are vowels
All letters are symbols

Syllogism – Answers and Explanation


3. c

4. a

1. b

5. d
2. e

6. e

Page 967 of 1334


Subscribe The Xpress Video Course & Mock Test Package for Bank & Insurance Exams
If there are any suggestions/ errors in our PDFs Feel Free to contact us via this email: admin@exampundit.in
Ultra Practice Bundle PDF
SBI Clerk/ RBI Assist. Mains – Reasoning
10. a

11. b

7. d

12. a

8. b

13. c

14. e
9. d

15. c

Page 968 of 1334


Subscribe The Xpress Video Course & Mock Test Package for Bank & Insurance Exams
If there are any suggestions/ errors in our PDFs Feel Free to contact us via this email: admin@exampundit.in
Ultra Practice Bundle PDF
SBI Clerk/ RBI Assist. Mains – Reasoning

21. b
16. d

22. d

17. c

23. a

18. b
24. e

19. a
25. c

20. e

Page 969 of 1334


Subscribe The Xpress Video Course & Mock Test Package for Bank & Insurance Exams
If there are any suggestions/ errors in our PDFs Feel Free to contact us via this email: admin@exampundit.in
Ultra Practice Bundle PDF
SBI Clerk/ RBI Assist. Mains – Reasoning
26. a

27. d
32. b

28. d

33. a

29. c

34. d

30. b

35. e
31. c

Page 970 of 1334


Subscribe The Xpress Video Course & Mock Test Package for Bank & Insurance Exams
If there are any suggestions/ errors in our PDFs Feel Free to contact us via this email: admin@exampundit.in
Ultra Practice Bundle PDF
SBI Clerk/ RBI Assist. Mains – Reasoning

40. b

36. c

41. a
37. a

38. e
42. a

39. b

43. d

Page 971 of 1334


Subscribe The Xpress Video Course & Mock Test Package for Bank & Insurance Exams
If there are any suggestions/ errors in our PDFs Feel Free to contact us via this email: admin@exampundit.in
Ultra Practice Bundle PDF
SBI Clerk/ RBI Assist. Mains – Reasoning

44. b

48. d

45. c
49. c

46. e

50. c

47. e

Page 972 of 1334


Subscribe The Xpress Video Course & Mock Test Package for Bank & Insurance Exams
If there are any suggestions/ errors in our PDFs Feel Free to contact us via this email: admin@exampundit.in
Ultra Practice Bundle PDF
SBI Clerk/ RBI Assist. Mains – Reasoning
Coded Direction Part I
Direction (1-5): Read the following information c) West
carefully and answer the given questions. d) North-East
X * Y means X is to the left of Y at a distance of 7m. e) East
X # Y means X is to the south direction of Y at a distance 4). E # F * H %G * I then find the distance between E and
of 5m. H?
X @ Y means X is to the right of Y at a distance of 3m. a) 1)√72 m
X % Y means X is to the north direction of Y at a distance b) 2)9 m
of 9m. c) 3)3√2 m
In each of the following questions initially, all persons are d) 4)√74 m
facing north. e) 5)None of these
1). B @ D % V # H @ K, then in which direction is K with 5). R#M @ N * O % P # Q, then Q in which direction with
respect to D? respect to R?
a) North a) South
b) East b) North-West
c) South-west c) West
d) North-West d) North-East
e) None of these e) East
2). M % N, Q * M, N % G, L * Q then find the minimum Direction (6-10): In certain coding language, the
distance between Q and N (approx)? directions are coded as per below conditions.
a) √ 12 m J@K means – J is North of K
b) √ 130 m J%K means – J is South of K
c) √ 81 m J#K means – J is East of K
d) √ 49 m J $ K means – J is West of K
e) √ 100 m J@#K means- J is North East K
3). S @ T * R % M * U, then T is in which direction with The distance between two point is either 8 m or 11 m.
respect to U? @ means the distance between the two points is 8m
a) South % means the distance between the two points is either
b) North-West 11m.
Page 973 of 1334
Subscribe The Xpress Video Course & Mock Test Package for Bank & Insurance Exams
If there are any suggestions/ errors in our PDFs Feel Free to contact us via this email: admin@exampundit.in
Ultra Practice Bundle PDF
SBI Clerk/ RBI Assist. Mains – Reasoning
JK > LM Means the distance between point J and K is c) North-west
greater than that of point L and M. d) North-East
Example: J @ K means J is north of K and the distance e) None of these
between J and K is 8m. 10) which of following statement is true?
A#B, C%B, D@E, F#E, D $ C, F@G, H#G a) D@C
BC=GH, EF>FG,DC<AB=GH b) A%$ G
6). In which direction A with respect to G? c) C@$G
a) @ d) H$C
b) % e) None of these
c) @# Direction (11-14): Read the following information
d) %$ carefully to answer the questions that follow. The
e) None of these questions are based on following coding formats:
7) What is the shortest distance between Point D and Point # – North
F? @ – South
a) √100 m % – East
b) √64 m $ – West
c) √121 m ! – 4m
d) √185m & – 3m
e) None of these Examples: A@B means A is South of B, A#$B means A
8). Akshay travels from point D to F and F to H.Then what is North-West of B, A@$!B means A is South-West of B
is the shortest distance he travelled through this journey? at a distance of 4m .
a) √270m Conditions given are as:
b) √121 m I. P#!Q
c) √185 m II. P#$R
d) either a or b III. R%&Q
e) None of these IV. R@!S
9) If I is 11m east of D then I is in which direction of F? V. U%&R
a) North VI. U#!W
b) East VII. W@$&X

Page 974 of 1334


Subscribe The Xpress Video Course & Mock Test Package for Bank & Insurance Exams
If there are any suggestions/ errors in our PDFs Feel Free to contact us via this email: admin@exampundit.in
Ultra Practice Bundle PDF
SBI Clerk/ RBI Assist. Mains – Reasoning
11) W is in which direction with respect to R. &5 – North(10m)
a) #% @3 – South(6m)
b) #$ * 4– East(8m)
c) @$ % 2– West(4m)
d) @% Examples: A@3B means A is 6m South of B.
e) None of these A&%6B means A is 12m North-West of B.
12) If W%!Z then distance between Z and S is? A@*4B means A is 8m South-West of B.
a) 6m Conditions given are as:
b) 8m I. S%6V
c) 4m II. V@%2K
d) 5m III. R@3V
e) 9m IV. B&%5T
13) P is in with direction with respect to W and what is the V. R@%Y
distance between P and W is? VI. T*4K
a) North-East(10m) VII. V%1Y
b) North-East(8m) 15) B is in which direction with respect to V.
c) South-East(6m) a) @%
d) North-West(10m) b) &%
e) None of these c) &*
14) If Y#&U then Y is in with direction with respect to S d) @%
? e) None of these
a) North-East 16). R is in which direction of T ? distance between Y and
b) North-East R is?
c) South-East a) North-East 6m
d) North-West b) South-West √ 40m
e) Data inadequate c) South East √ 4m
Direction(15-18): Read the following information d) East-√ 40m
carefully to answer the questions that follow. The e) West-√ 36m
questions are based on following coding formats:

Page 975 of 1334


Subscribe The Xpress Video Course & Mock Test Package for Bank & Insurance Exams
If there are any suggestions/ errors in our PDFs Feel Free to contact us via this email: admin@exampundit.in
Ultra Practice Bundle PDF
SBI Clerk/ RBI Assist. Mains – Reasoning
17) Y is in with direction with respect to S and what is the Triangle formed by connecting point Z,U and T should be
distance between S and Y is? equilateral triangle.
a) 1)East(10m) 19). Z is in which direction with respect to Y; what is
b) 2)West(12m) distance between point Z and point Y ?
c) 3)East(12m) a) ^!15m
d) 4)West(10m) b) $*17m
e) 5) None of these c) ^*15m
18). Which of the following pair is incorrect? d) $*19m
a) V is South-west of K e) None of these
b) T is South-east of B 20). which of following pair has equal distance between
c) S is West of Y them?
d) T is East of R a) XH-SM
e) None of these b) TM-ZT
Direction (19-22): Read the following information c) ZX-XH
carefully and answer the given questions. d) TM-HX
J^K means J is to the north of K e) None of these
at a distance of either 4m or 9m 21). If Amar want to reach at point U which is in North-
J*K means J is to the west direction of K West of T ; from Point X then what distance should he
at a distance of either 3m or 12m travel?
J $ K means J is to the south direction of K a) 12m
at a distance of either 4m or 9m b) 9m
J!K means J is to the east direction of K at c) 23m
a distance of either 3m or 12m d) 20m
J^! K means J is to the north-east of K e) 21m
J$! K means J is to the south-east direction of K. 22). M is in which direction with respect to U ?
X$Z, Y!X, Z*T, S$*T, S*M$T, H$X, a) ^!
(SM<XH),(TM=XH),(XY=TZ) b) $*
Triangle formed by connecting point X,Y and Z should be c) ^*
right angled triangle. d) $!

Page 976 of 1334


Subscribe The Xpress Video Course & Mock Test Package for Bank & Insurance Exams
If there are any suggestions/ errors in our PDFs Feel Free to contact us via this email: admin@exampundit.in
Ultra Practice Bundle PDF
SBI Clerk/ RBI Assist. Mains – Reasoning
e) None of these Direction (26-30): Read the following information
Direction : (23-25) Read the following information carefully and answer the questions given below it:
carefully and answer the questions given below it: Nine people – A, B, C, D, E, F, G, H and I were stood at
Study the following information carefully and answer the some distance from each other in a grocery area. C was 18
questions given below. m $ to that of B. A was 8 m % of that of B and H was 14
P%R (10)- P is 12m in south of R m # of that of A. G was 10 m %@ of that of H while D
P$R (15)- P is 17m in north of R was 20 m # of that of G and F was 10 m % of that of G. I
P#R (22)- P is 24m in west of R was situated just in the middle of B and C while E was just
P&R (14)- P is 16m in east of R in the middle of G and D.
P%Q (22), P#S (5), Q&U(15), U$W (28), Y&P (15) Here, % means North, # means South, $ means East and
23) Q is in which direction with respect to Y? @ means West
a) North %$ means North-East ,#@ means south-West.
b) West For example,
c) North-west X%$Y means Y is to the North-East of X
d) North-East Y%$X means X is to the North-East of Y
e) South-west 26). What is the direction of I with respect to F?
24) What is the distance between Q and S? a) South-East
a) 27 b) South –west
b) 22 c) North
c) 25 d) North – west
d) 17 e) South
e) 24 27).What is the shortest distance between person B and
25) What is the direction of W with respect to S? person G?
a) North a) 12m
b) South-East b) 10m
c) West c) 9m
d) East d) 11m
e) South-West e) None of these

Page 977 of 1334


Subscribe The Xpress Video Course & Mock Test Package for Bank & Insurance Exams
If there are any suggestions/ errors in our PDFs Feel Free to contact us via this email: admin@exampundit.in
Ultra Practice Bundle PDF
SBI Clerk/ RBI Assist. Mains – Reasoning
28). Which of the following persons stand in the straight F is 6m%E.G is 5m$F. H*GI.J is4m@H. G is 12m % I.
line? 31). Amar moves from E to H, then H to G,then G to F
a) G-A-B ;what is the total distance covered by the Amar?
b) C-H-I a) 10m
c) G-E-D b) 12m
d) A-B-H c) 16m
e) Both 3 and 4 d) 18m
29)What is the direction of E with respect to I? e) None of these
a) South 32). If there is a bus stand X at 14m west to G , then if a
b) South – west Akshay moves from G to X, then Akshay is standing at
c) North – west what distance from point B?
d) North a) 1)12m
e) None of these b) 2) 6m
30)What is the difference between the sum of distance of c) 3) 5m
FG+GE and BI+GB ? d) 4) 11m
a) 17 e) 5) None of these
b) 03 33). If Sunil moves to school Z from point F; where Z is
c) 14 13m towards east from H, then at what distance from F?
d) 13 a) 8m
e) None of these b) 5m
Direction (31-35): Study the information below and c) 3m
answer the following questions d) 10m
Y % Z – Y is north of Z. e) None of these
Y # Z – Y is south of Z. 34). In which direction is H with respect to A?
Y @ Z – Y is east of Z. a) North
Y$ Z – Y is west of Z. b) East
X * YZ – X is midpoint of vertical straight line YZ. c) North-East
X! YZ – X is midpoint of horizontal straight line YZ. d) North-West
A is 8m@B. C!AB. D is 5m%C. E is 10m@D. e) South West

Page 978 of 1334


Subscribe The Xpress Video Course & Mock Test Package for Bank & Insurance Exams
If there are any suggestions/ errors in our PDFs Feel Free to contact us via this email: admin@exampundit.in
Ultra Practice Bundle PDF
SBI Clerk/ RBI Assist. Mains – Reasoning
35) If Bus travels from point I to point H and then to point e) None of these
D,the D to point C and again towards point B;what is total 38). What is the sum of distance between E-F ,D-B and E-
distance covered by bus? I?
a) 16m a) 14m
b) 11m b) 18m
c) 10m c) 12m
d) 25m d) 8m
e) None of these e) None of these
Direction (36-39): Study the information below and 39)D is in which direction to G?
answer the questions. a) North
G & H (98m) – G is 77m south of H. b) North-East
G % H (65m) – G is 43m east of H. c) South-East
G @ H (113m) – G is 91m north of H. d) North-West
G # H (66m) – G is 44m west of H. e) None of these
G&# H (49m)-G is 27m South-west of H. Direction (40-43): Study the following information
A @ B (25m), C # D (38m), B % C (42m), E % D (52m), carefully and answer the questions given below:
G # F(64m), G & H (37m), I & E (28m), F @ I (30m) The symbols @, © , π, $, % and # are used with the
,W@F(39m) following meanings illustrated.
36). What is the direction of B with respect to I ? D$C means D is south C.(4m/6m)
a) North D#C means D is west C.(3m/7m)
b) North-East D@C means D is east C .(4m/6m)
c) South-West D%C means D is north C.(3m/7m)
d) North-West DπC means D is south-east C.(4m/6m)
e) None of these D©C means D is south-west C.(3m/7m)
37). What is the direction of A with respect to W? M @ L, N % M, O π N, P % O, Q @ P and R $ Q.
a) North QR > PQ, PQ=LM, NO > PQ, NM >QR , NM=PO
b) North-East 40) What is the distance between N and O ?
c) South-West a) 3m
d) North-West b) 4m

Page 979 of 1334


Subscribe The Xpress Video Course & Mock Test Package for Bank & Insurance Exams
If there are any suggestions/ errors in our PDFs Feel Free to contact us via this email: admin@exampundit.in
Ultra Practice Bundle PDF
SBI Clerk/ RBI Assist. Mains – Reasoning
c) 6m C$D - D is in the west direction of C.
d) 7m E£CD- E is the mid-point of CD horizontally.
e) None of these C#$D-D is in south-west direction of C.
41). In which direction point O is with respect to point R? S$24m B. P#5mS. K@25mB.L$20mK. Q#10mL.
a) South F$13mQ. B&16mE.
b) East D#15mF.
c) North-East e.g. C#$8mD means D is 8m in south-west direction of C.
d) South-West 44) What is the direction of point K with respect to S?
e) None of these a) #&
42). what is total distance between point N and point R? b) @&
a) 21m c) #$
b) 18m d) @$
c) 27m e) None of these
d) 22m 45) What is the shortest distance and direction of point D
e) 23m with respect to E?
Direction (43): If S is 10m to the south-East of point a) 16m, #$
Q;then R is in which direction with respect to point S b) 15m, @&
and what is shortest distance between point S and c) 17m, #$
point R ? d) 25m, $&
a) South (12m) e) None of these
b) North-West (27m) 46) What is the difference of distance between point D-E
c) South-East (36m) and Point E-Q + Q-L ?
d) North a) 7m
e) West-(8m) b) 9m
Direction (44-46): Study the following information and c) 5m
answer the given questions: d) 6m
C#D - D is in the south direction of C. e) None of these
C@D -D is in the north direction of C. Direction (47-50): Read the following information
C&D - D is in the east direction of C. carefully and answer the questions that follow:

Page 980 of 1334


Subscribe The Xpress Video Course & Mock Test Package for Bank & Insurance Exams
If there are any suggestions/ errors in our PDFs Feel Free to contact us via this email: admin@exampundit.in
Ultra Practice Bundle PDF
SBI Clerk/ RBI Assist. Mains – Reasoning
E% F means E is to the right of F at a distance of 6m. c) South
E @ F means E is to the left of F at a distance of 7m. d) West
E $ F means E is to the north of F at a distance of 8m. e) North-West
E # F means E is to the south of F at a distance of 9m. 49) E $ F @ G # H, then H is in which direction with
E * F means E is to the east of F at a distance of 10m. respect to E and what is distance between H and E?
E ! F means E is to the west of F at a distance of 11m. a) North, 1m
All people are facing North direction. b) South, 7m
Q.47)E % F # C ! D, then D is in which direction with c) North-East,√50m
respect to E? d) East, √43m
a) North e) West,√8m
b) North-East 50) P ! R * S # Q @ T, Then T is in which direction with
c) North-West respect to P ?
d) South-West a) North
e) South b) South-West
48) A # B % C # D, then D is in which direction with c) West
respect to A? d) East
a) North e) North-East
b) North-East

Coded Direction Part I – Answer and Explanation


1. C 2. B

3. B
Page 981 of 1334
Subscribe The Xpress Video Course & Mock Test Package for Bank & Insurance Exams
If there are any suggestions/ errors in our PDFs Feel Free to contact us via this email: admin@exampundit.in
Ultra Practice Bundle PDF
SBI Clerk/ RBI Assist. Mains – Reasoning

4. D

6.C
7. D
8. A
9. A
5. D 10. C
Solution (11-14):

Solution (6-10): 11.D


Page 982 of 1334
Subscribe The Xpress Video Course & Mock Test Package for Bank & Insurance Exams
If there are any suggestions/ errors in our PDFs Feel Free to contact us via this email: admin@exampundit.in
Ultra Practice Bundle PDF
SBI Clerk/ RBI Assist. Mains – Reasoning
12.B 19. C
13.D 20.D
14.C 21.E
Solution(15-18): 22.D
Solution(23-25):

15.C
16.B 23. D

17.A 24. C

18.D 25. E

Solution(19-22): Solution(26-30):

Page 983 of 1334


Subscribe The Xpress Video Course & Mock Test Package for Bank & Insurance Exams
If there are any suggestions/ errors in our PDFs Feel Free to contact us via this email: admin@exampundit.in
Ultra Practice Bundle PDF
SBI Clerk/ RBI Assist. Mains – Reasoning
26.A 36.B
27.E 37.C
28.E 38.C
29.B 39.C
30.B Solution(40-43):
Solution(31-35):

40.C
41.D
31.C
42.E
32.D
43.E
33.D
Solution(44-47):
34.C
35.D
Solution(36-39):

44.C
45.E
46.D
Page 984 of 1334
Subscribe The Xpress Video Course & Mock Test Package for Bank & Insurance Exams
If there are any suggestions/ errors in our PDFs Feel Free to contact us via this email: admin@exampundit.in
Ultra Practice Bundle PDF
SBI Clerk/ RBI Assist. Mains – Reasoning
47.B 49. C

48.E

50.E

Coded Blood Relation Part I


Direction (1-5): Study the following information 1. If C @ D * E & F % G $ H # D, then how H is related
carefully and answer the given questions: F?
S#T – S is the son of T. a) Brother
S@T – T is the child of S. b) Uncle
S$T – S is elder than T. c) Father in law
S*T– S is the husband of T. d) Brother in law
S©T – S is the parent of T. e) Cannot be determined
S&T– T is the daughter-in-law of S. 2. If S * M @R $ N # S, the age of M is 35 years and of
S%T– S is the wife of T. N is 18 years then what is the probable age of R?
a) 17 years
Page 985 of 1334
Subscribe The Xpress Video Course & Mock Test Package for Bank & Insurance Exams
If there are any suggestions/ errors in our PDFs Feel Free to contact us via this email: admin@exampundit.in
Ultra Practice Bundle PDF
SBI Clerk/ RBI Assist. Mains – Reasoning
b) 19 years X&Y– X is younger to Y
c) 20 years X$Y– X is brother of Y
d) 08 year X*Y– X is wife of Y
e) Both b and c X#Y– X is sister-in-law of Y
3. If W%F @ G * J & K % H, then how W is related X!Y- X is son in law of Y.
to J? X^Y-X is not the sibling of Y
a) Father in law X*F$D&Q$F; E@X; F% D*C!B; W#D; A*K@W
b) grandmother in law C^W
c) Daughter in law 6.If the age of F is 45 years and age of Q is 40 years, so
d) Mother in law what can be the age of D?
e) Cannot be determined a) 44 years
4. If C % D © E * A © B and the age of C is 70 years, b) 43 years
then what could be the age of E? c) 47 years
a) 77 years d) 39years
b) 75 years e) None of these
c) 40 years 7. If G@D then how X is related to G?
d) 80 years a) Uncle
e) 72 years b) Grandmother
5. If X * Y © Z $ U # M and X#R@M, then how X is c) sister-in-law
related to U ? d) Aunt
a) Grandfather e) None of these
b) Uncle 8. How C is related to F?
c) Aunty a) Bother in law
d) Sister b) Sister in law
e) None of these c) Brother
Direction (6-9): Study the following information and d) Uncle
answer the given questions: e) cannot be determined
X@Y– X is the child of Y. 9. Which of the following pair is depicting correct
X%Y– X is elder to Y relation?

Page 986 of 1334


Subscribe The Xpress Video Course & Mock Test Package for Bank & Insurance Exams
If there are any suggestions/ errors in our PDFs Feel Free to contact us via this email: admin@exampundit.in
Ultra Practice Bundle PDF
SBI Clerk/ RBI Assist. Mains – Reasoning
a) D*F d) D is father of M
b) K@B e) None of these
c) W*F Direction (13-16): . Study the following information
d) Q@B and answer the given questions:
e) All are correct G©I– G is the parent of I
Direction (10-12): Following questions are based on G%I– G is elder to I
the information provided below: G&I– G is younger to I
A × B means B is mother of A G$I– G is brother of I
A – B means B is brother of A G*I– G is wife of I
A + B means B is sister of A G#I– G is sister-in-law of I
A ÷ B means B is father of A A © G; J * O $ L $ G; G© N & M; J© M; T # L,T ©N
10. If the expression M × N + R ÷ T is true, then which 13. How is J related to A?
of the following is true? a) Uncle
a) M is son of R b) Brother-in-law
b) N is aunt of T c) daughter-in-law
c) M is granddaughter of T d) Aunt
d) T is father of N e) None of these
e) None of these 14. If K is the wife of L then how is K related to N?
11. Which of the following means ‘Q’ is brother of ‘T’? a) Aunt
a) Y ÷ Q – T + R b) brother-in-law
b) T ÷ Q – Y- R c) sister-in-law
c) T × M – R + Q d) Uncle
d) T + M ÷ Y + Q e) None of these
e) None of these 15. If N is the brother of R, then how is R related to A?
12. If the expression M – Q – D ÷ F + C, then which of a) Daughter
the following is true? b) Son-in -law
a) C is daughter of F c) Grand son
b) F is niece of C d) Grand-daughter
c) C is sister of F e) cannot be determined

Page 987 of 1334


Subscribe The Xpress Video Course & Mock Test Package for Bank & Insurance Exams
If there are any suggestions/ errors in our PDFs Feel Free to contact us via this email: admin@exampundit.in
Ultra Practice Bundle PDF
SBI Clerk/ RBI Assist. Mains – Reasoning
16. If , the age of M is 18 years and age of T is 33 years, a) H
so what can be the age of N? b) S
a) 19 years c) R
b) 13 years d) Q
c) 30 years e) None of the above
d) 35 years 19 If Z^G$ E is true statement then G is related to R
e) 36 years is?
Direction (17-20) Study the following information a) grandfather
carefully to answer the given questions. b) grandson
B= M means B is the sister of M. c) Son
B * M means B is the husband of M. d) grand daughter
B ? M means B is the brother of M. e) grandmother
B $ M means B is the wife of M. 20. If C is the maternal uncle of R then Y is related to
B @ M means B is the son of M. C is?
B x M means B is the daughter of M. a) brother
B ! M means B is the son in law of M. b) uncle
B ^ M means B is the daughter in law of M. c) Sister in law
B + M means B is the nephew of M. d) Cannot be determined
B - M means B is the niece of M. e) Brother in law
T$R@Z$Y?W$X Direction (21-22 ) Study the following information
Q@S@WxE carefully to answer the given questions.
H^F^X If ‘P $ Q’ means that P is the Father of Q,
17 Which of the following is not correct? ‘P ÷ Q’ means that P is the sister of Q,
a) R is the wife of T . ‘P + Q’ means that P is the son of Q,
b) F is the son of W. ‘P = Q’ means that P is the brother of Q,
c) W is the brother of Y. 21. Which of the following means that C is the sister of
d) Q and H are siblings. D?
e) All are incorrect a) C =P ÷ D
18 Who among the following is grandsonof X? b) P + D ÷ C

Page 988 of 1334


Subscribe The Xpress Video Course & Mock Test Package for Bank & Insurance Exams
If there are any suggestions/ errors in our PDFs Feel Free to contact us via this email: admin@exampundit.in
Ultra Practice Bundle PDF
SBI Clerk/ RBI Assist. Mains – Reasoning
c) D ÷ P = C e) None of these
d) D= C ÷ P 25. M * H @ D * K, represents what relation of K with
e) None of these respect to M?
22. Which of the following is definitely true about C + a) Nephew
A ÷ B? b) Father
a) B is the uncle of C c) Brother
b) A is the son of C d) Son
c) B is the husband of A e) None of these
d) C is the nephew of B Direction (26-28): Study the following information
e) None of these carefully and answer the questions which follow–
Direction (23-25): These questions are based on the T @ S means T is parent of S (either mother or father)
following information. T # S means T is sister of S
Z is the mother of X is represented by X@Z T $ S means S is grandchild of T
Z is the Wife of X is represented by X $ Z T % S means T is brother of S
Z is the sister of X is represented by X # Z T & S means T is the son-in-law of S
Z is the son of X is represented by X * Z T ! S means T is the daughter-in-law of S
23. If F # J * T $ R @ L, then which of the following is T * S means T is the wife of S
definitely true? T © S means S is the mother of T
a) L is the brother of F There are some members in a family having three
b) F is the sister of L generation. The relation between the different
c) F is the brother of J members of the family are defined as follows:
d) L is the mother in law of T U@G#J
e) None of these U&I
24. Which of the following indicates the relationship K !U
‘R’ is the daughter of T? F%J*C
a) R # F * B$ T E©G
b) F * B $ T@R H@E#L
c) T @ B # R * F 26. If M is the only child of C, then how is M related to
d) T @ B # F * R F?

Page 989 of 1334


Subscribe The Xpress Video Course & Mock Test Package for Bank & Insurance Exams
If there are any suggestions/ errors in our PDFs Feel Free to contact us via this email: admin@exampundit.in
Ultra Practice Bundle PDF
SBI Clerk/ RBI Assist. Mains – Reasoning
a) Niece C%P ;
b) Nephew D &E *F;
c) Uncle J + E;
d) Aunt 29) How I is related to J?
e) cannot be determined a) daughter
27. If F has only one son, then how is J related to F’s b) Granddaughter
son? c) Mother in law
a) sister d) Grandmother
b) mother in law e) None of these
c) aunt 30) If S=P, S*V then how V is related to P?
d) Uncle a) Niece
e) None of these b) Nephew
28 which of the following pair is correct? c) Brother
a) J!U d) Sister
b) G$K e) Cannot be determined
c) L#E 31). If D*L, R+F such that L is married person, then
d) J*C how R is related to E?
e) All are correct a) Son in law
Directions (29-31): Study the following information b) Mother in law
carefully and answer the questions given below. c) Daughter in law
A!C means ‘C is sister in law of A’ d) Father in law
A*C means ‘C is child of A’ e) Either a or c
A£C means ‘C is mother of A’ Directions (32–34): Study the following information
A$C means ‘C is father in law of A’ carefully and answer the questions given below it.
A=C means ‘C is siblings of A’ 4^ 3 means 4 is mother of 3.
A&C means ‘C is married to A’ 4# 3 means 4 is son of 3.
C%A means ‘A is son in law of C’ 3&4 means 3 is father of 4.
A+C means ‘C is brother in law of A’ 4%3means 4 is brother of 3.
C*D=J$K*P£I; 3+ 4 means 3 is daughter of 4.

Page 990 of 1334


Subscribe The Xpress Video Course & Mock Test Package for Bank & Insurance Exams
If there are any suggestions/ errors in our PDFs Feel Free to contact us via this email: admin@exampundit.in
Ultra Practice Bundle PDF
SBI Clerk/ RBI Assist. Mains – Reasoning
4@3 means 4 is sister of 3. X$R’ means ‘R is the brother-in-law of X’
1+2^3; 1@4; ‘X*R’ means ‘R is either nephew or niece of X’
5&3@7&6 . 35)If S ~ K & G % U & W ~ Q , if K has only sister
32). In the given expression below how 4 is related to 3 then , then which of following will correctly depicts
a) Brother correct relation between S and G ?
b) Grandmother a) S%G
c) Sister b) G!S
d) Mother-in-law c) G~S
e) Either a or c d) S ! G
33) In the given expression below which of the e) Both b and d
following statement is true? 36)If E £ F, M& E ~ G * H ~Q, then how Q is related
a) 7 is maternal uncle of 4. to F, if G has only one nephew?
b) 1 is granddaughter of 5. a) Son
c) 7 is sister of 2. b) Granddaughter
d) 2 is grandfather of 3. c) Cannot be determined
e) All the given statements are false. d) Grandson
34) How 5 is related to 4? e) None of these
a) Brother 37). If S ~ T & Q $ R & M ~ Q, then how Q is related
b) Mother to R?
c) Father a) Niece
d) Mother-in-law b) Nephew
e) Either 1 or 3 c) Cannot be determined
Directions (35-37): Study the following information d) brother in law
and answer the questions given below: e) sister in law
‘X~R’ means ‘X and R are siblings’ Directions (38-41): Study the following information
‘X&R’ means ‘R and X are married couple’ and answer the given questions.
‘X£R’ means ‘X is the only son of R’ If
‘X%R’ means ‘X is mother in law of R’ *10 $ means * is mother of $
‘X!R’ means ‘X is the sister-in-law of R’ * 8 $ means * is sister of $

Page 991 of 1334


Subscribe The Xpress Video Course & Mock Test Package for Bank & Insurance Exams
If there are any suggestions/ errors in our PDFs Feel Free to contact us via this email: admin@exampundit.in
Ultra Practice Bundle PDF
SBI Clerk/ RBI Assist. Mains – Reasoning
* 9 $ means * is father of $ d) sister in law
* 7 $ means * is brother of $ e) None of these
* 6 $ means * is wife of $ Directions (42-44): Study the following information
* 5 $ means * is son of $ and answer the given questions.
38) If # 10 % 5 $ 9 & 6 @ then how % is related to X @ Y means X is wife of Y
@? X © Y means X is sister of Y
a) Son in law X % Y means X is son of Y
b) Brother in law X # Y means Y is mother of X
c) Sister in law X $ Y means Y is father of X
d) Cannot be determined X + Y means X is daughter of Y
e) None of these X =Y means X is nephew of Y
39) If # 5 + 9 = 8 > 10 % then how % is related to = ? X *Y means X is niece of Y
a) Sister in law I%D#E©F+G@H;
b) nephew D * F;
c) Brother in law I $ J# K
d) niece 42. How is I is related to F ?
e) cannot be determined a) Son
40) If $ 8 % 7 ^ 6 + 9 *_ ^ then which of the following b) Nephew
number should be in between * and ^? to prove that * c) Sister’s grandson
is nephew of % . d) Sister’s grand daughter
a) 9 e) None of these
b) 5 43. If L is son of G then how L is related to J ?
c) 7 a) Uncle
d) 6 b) Nephew
e) 10 c) brother in law
41) If !7@9#5&then how & is related to ! ? d) Mother-in-law’s brother-in-law
a) Son in law e) Mother-in-law’s brother
b) Brother in law
c) Daughter in law

Page 992 of 1334


Subscribe The Xpress Video Course & Mock Test Package for Bank & Insurance Exams
If there are any suggestions/ errors in our PDFs Feel Free to contact us via this email: admin@exampundit.in
Ultra Practice Bundle PDF
SBI Clerk/ RBI Assist. Mains – Reasoning
44. If pointing towards the person G says this person 46. What should come in the place of question mark,
is only son of my grandfather’s only grandchild, then to establish that A is the niece of F in the expression
how F is related to that person ? ‘A_ B $ C ᴨ D % E _F’ in same order
a) sister a) ᴨ , ^
b) nephew b) %, @
c) brother in law c) ᴨ, #
d) Aunt d) !,#
e) cannot be determined e) ᴨ, ᴨ
Directions (45-48): Study the following information 47. In the expression ‘A $ S ᴨ D @ F ᴨ G % H ’, how
carefully and answer the questions given below. is H related to A?
A # B means A is the father of B a) Grand Mother-in-law
A + B means A is the mother of B b) Brother-in-law
A $ B means A is the husband of B c) Grand Father-in-law
A %B means A is the wife of B d) Sister-in-law
A @B means A is the brother of B e) Son-in-law
A ^B means A is the sister of B 48. What should come in the place of question mark,
A ! B means A is the son of B to establish that Q and Y is sibling in the expression Q
A ᴨ B means A is the daughter of B ᴨ W _E ! R # T _Y in same order such that E is not the
A * B means A is the aunt of B father of Y.
45. Which of the following statements is not true if the a) $, %
expression b) @, *
‘ T ! M ^ X ᴨ N % K # W $ Y’ is definitely true? c) $, *
a) T is nephew of W d) Cannot be determined
b) Y is aunt of T e) None of these
c) N is grandmother of T Directions (49-50): Study the following information
d) X is daughter-in-law of M carefully and answer the questions given below.
e) All are correct A is West of B means A is the father of B
A is North of B means A is the mother of B
A is south B means A is the husband of B

Page 993 of 1334


Subscribe The Xpress Video Course & Mock Test Package for Bank & Insurance Exams
If there are any suggestions/ errors in our PDFs Feel Free to contact us via this email: admin@exampundit.in
Ultra Practice Bundle PDF
SBI Clerk/ RBI Assist. Mains – Reasoning
A is East B means A is the wife of B c) Grandson
Draw a family tree for following statements: d) Cannot be determined
A is west S while I is north of D; e) None of these
D is in the east A; 50. Which of following statement is correct?
S is in south of G; a) A is north of D
G is in north of H. b) I is east of D
49. In the expression how is H related to A? c) G is east of S
a) Grand daughter d) A is west of G
b) daughter e) All are correct

Coded Blood Relation Part I - Answer and Explanation


1) D
H is the brother of G ,so he is brother in law for F.

Page 994 of 1334


Subscribe The Xpress Video Course & Mock Test Package for Bank & Insurance Exams
If there are any suggestions/ errors in our PDFs Feel Free to contact us via this email: admin@exampundit.in
Ultra Practice Bundle PDF
SBI Clerk/ RBI Assist. Mains – Reasoning

2) E (R>N i.e. N’s age is 18 years and then R’s age can
be 19 or 20 years)

5) B

3) D

Solution :( 6-9) (figure modified)

4 ). C 6) D
D should be elder than B as D is grand parent of B. F is elder to D and D is younger to Q means age of D
should be less than F and Q.

Page 995 of 1334


Subscribe The Xpress Video Course & Mock Test Package for Bank & Insurance Exams
If there are any suggestions/ errors in our PDFs Feel Free to contact us via this email: admin@exampundit.in
Ultra Practice Bundle PDF
SBI Clerk/ RBI Assist. Mains – Reasoning
7) D 12). C
G @D means G is the child of D and X is wife of F, who
is uncle of G. Therefore wife of uncle (X) is aunt for G.
8) A
9) D
10) D
Solution: (13-16)

11) A (figure modified)

13) C
14) A
L is uncle of N, so if K is wife of L then she will aunt
of N.
15) E
If N is the brother R, then we can’t say about gender
of R, so how R is related to A is cannot be determined.
16) B
As N is younger to M, his/her age should be less than
M.
Solution :(17-20)

Page 996 of 1334


Subscribe The Xpress Video Course & Mock Test Package for Bank & Insurance Exams
If there are any suggestions/ errors in our PDFs Feel Free to contact us via this email: admin@exampundit.in
Ultra Practice Bundle PDF
SBI Clerk/ RBI Assist. Mains – Reasoning

22) D

17) E
18) D
19) E
Z^G$ E means Z is daughter in law of G; G is wife of 23) D (figure modified)
E. We know Z is mother of R. So mother in law of
mother is grandmother. Therefore G is grandmother
of R.
20) E
If C is the maternal uncle of R means C is brother of
Z, and Z is wife of Y; so Y is brother in law of C. 24) E
21) D (i) Figure modified

Page 997 of 1334


Subscribe The Xpress Video Course & Mock Test Package for Bank & Insurance Exams
If there are any suggestions/ errors in our PDFs Feel Free to contact us via this email: admin@exampundit.in
Ultra Practice Bundle PDF
SBI Clerk/ RBI Assist. Mains – Reasoning
Solution (29-31):

25) D

29) C
30) E
Gender of V is not confirmed so we cannot determine
relation between V and P.
31) E
Solution(26-28): (figure modified) Gender of R is not known so we cannot determine how R
is related to E.
Solution (32-34):

26) E
As gender of M is not confirmed so we can not determine
how M is related to F. 32) E
27) C We don’t know exact gender of 4 but he/she is brother or
28) D sister of 1. So for 3 , 4 can be either brother and sister

Page 998 of 1334


Subscribe The Xpress Video Course & Mock Test Package for Bank & Insurance Exams
If there are any suggestions/ errors in our PDFs Feel Free to contact us via this email: admin@exampundit.in
Ultra Practice Bundle PDF
SBI Clerk/ RBI Assist. Mains – Reasoning
33) E
34) C
35) E
If K has only one sister implies that S is sister of K
.Therefore G is sister in law to S and S is sister in law to
39) E
G.
Gender of % is not known ,so we cannot
determine the relation how % is related with
respect to =.

36) C
H is either nephew or niece of G i.e. gender of H is not
known. Q’s gender is also not known even if G has only
one nephew. Therefore Q is either grandson or 40) Option: B
granddaughter to F. Thus we can’t determine the relation. In order to prove that * is nephew of % . The gender
of * should be male. By putting 2nd option symbol( 5)
in between * and ^ we can prove that * is nephew of %.
41) D

37) C
Solution (42-44): (figure modified)
Gender of Q is not known so we cannot determine
relation of Q with respect to R.

38) B (figure modified)


Page 999 of 1334
Subscribe The Xpress Video Course & Mock Test Package for Bank & Insurance Exams
If there are any suggestions/ errors in our PDFs Feel Free to contact us via this email: admin@exampundit.in
Ultra Practice Bundle PDF
SBI Clerk/ RBI Assist. Mains – Reasoning
So by using Symbols in option C (ᴨ, #).we can prove A is
the niece of F.

42) C Solution: I is grandson of E, who is sister of F.


47) C
Therefore I is sister’s grandson.
43) E Solution: If L is son of G and then L is brother
of E, who is mother-in-law of J. Therefore L is mother-
in-law’s brother to J.
44) A
Pointing towards person G says he is only son of my
grandfather’s only grandchild that implies that person
is son of G. Therefore that person is E and F is sister of
48) B (figure modified)
him.
Among options, to establish that Q and Y is
45) D
sibling we have to prove that Y is niece/nephew of T and
Q is sister of Y. symbols in option b are proving this.
Note: E is not the father of Y. Therefore W must
be the father of Y so that Q and Y are sibilings.

46) C (figure modified)


Among given options, to establish that A is the niece of F
,A should be daughter of B and F should be sibling of C.

Page 1000 of 1334


Subscribe The Xpress Video Course & Mock Test Package for Bank & Insurance Exams
If there are any suggestions/ errors in our PDFs Feel Free to contact us via this email: admin@exampundit.in
Ultra Practice Bundle PDF
SBI Clerk/ RBI Assist. Mains – Reasoning

49) D
Solution(49-50) (figure modified) Gender of H is not known .So we cannot be
determined relation of A with H.
50) C
G is east of S means G is wife of S. from family tree
we can prove this.

Alpha Numeric New Pattern


Directions 1-5: Answer the questions based on the letters starting from the 11th letter of A-Z alphabetical
sequence of numbers, alphabets and symbols given. series is replaced by the second digit of its represented
number (for e.g. L = 12 so L is represented as 2) and
A2ZRQ#Y$%U4*W1%D9S@T8
other letters will remain same.
There are certain operations which are to be applied, and
3) Symbols ‘%’ is replaced by a symbol ‘$’
then mark your answer accordingly.
1) How many total numbers are there in the series?
1) If each letter in A-Z alphabetical series is represented
by number 1-26 respectively, then only even number in a. 3
the series is replaced by the alphabet as per its
b. 4
represented number.
c. 6
2) If each letter in A-Z alphabetical series is represented
by number 1-26 respectively, then only 10 consecutive d. 7

Page 1001 of 1334


Subscribe The Xpress Video Course & Mock Test Package for Bank & Insurance Exams
If there are any suggestions/ errors in our PDFs Feel Free to contact us via this email: admin@exampundit.in
Ultra Practice Bundle PDF
SBI Clerk/ RBI Assist. Mains – Reasoning
e. 8. c. Two

2) How many such alphabets are there in the series d. Three


which are followed by a number?
e. Four
a. One
5) What is the sum of numbers are there in the series?
b. Two
a. 34
c. Three
b. 32
d. Four
c. 33
e. Five
d. 35
3)How many such symbol/s is/are there in the series
e. None of these
which is/are preceded by a number but followed by
an alphabet? Directions 6-10: Answer the questions based on the
sequence of numbers, alphabets and symbols given.
a. One
1#W$4S$D&*27!AH4%8@!53L9F@Q
b. Two
$6<
c. Three There are certain operations which are to be applied, and

d. Four then mark your answer accordingly.

1) If each letter in A-Z alphabetical series is represented


e. Five
by number 1–26 respectively, then each number in the
4) How many pairs of numbers/alphabets/symbols are series is replaced by the alphabet as per its represented
there in the final series? (if there is more two number, number.
symbol or letter is repeated they also considered as
2) If each letter in A-Z alphabetical series is represented
one count)
by number 1–26 respectively, then only first nine letters
a. None of A-Z alphabetical series is replaced by its represented
number and other letters will remain same.
b. One

Page 1002 of 1334


Subscribe The Xpress Video Course & Mock Test Package for Bank & Insurance Exams
If there are any suggestions/ errors in our PDFs Feel Free to contact us via this email: admin@exampundit.in
Ultra Practice Bundle PDF
SBI Clerk/ RBI Assist. Mains – Reasoning
3) Symbols ‘$’ and ‘!’ are replaced by a symbol ‘<’. a. Eleven

6) How many such numbers are there in the series b. Eight


which are preceded by a symbol?
c. Seventeen
a. None
d. Fourteen
b. One
e. Ten
c. Two
10) How many such numbers/alphabets are there in
d. Three the series which are immediately preceded and
followed by a symbol ‘<’?
e. More than three
a. None
7) How many such alphabets are there in the series
which are followed by an alphabet only? b. One

a. 0 c. Two

b. 4 d. Three

c. 5 e. Four

d. 7 Direction 11-15: Read the following information


carefully and answer the question asked below
e. 8
7M4$GU*5%@X3#S&29∆I8O1J©6VP
8) How many alphabets are there in the series?
^
a. 8
Step 1: The letters which are immediately preceded by
b. 10 number are to be arranged in the alphabetical order
c. 4 immediately after the last element of the series.

d. 14 Step 2: The number which is immediately followed by


another number and immediately preceded by the symbol
e. 15
are to be arranged in descending order immediately after
9) How many numbers are there in the series? the last element of the series.

Page 1003 of 1334


Subscribe The Xpress Video Course & Mock Test Package for Bank & Insurance Exams
If there are any suggestions/ errors in our PDFs Feel Free to contact us via this email: admin@exampundit.in
Ultra Practice Bundle PDF
SBI Clerk/ RBI Assist. Mains – Reasoning
Step 3: The symbol which is immediately preceded by a b. &
letter is to be interchange with the element that
c. 7
immediately followed that symbol.
d. ∆
Note – Step 2 is performed after completion of step 1 and
step 3 is to be performed after completion of step 2. e. 2

11) What is the sum of the element which is sixth 14) If half of the series get reversed from starting

from the left in step 3 and eighth from the right in after last step which of the following will be eighth

step 2? element from the left end?

a. Addition of element is not possible a. %

b. 13 b. 5

c. 11 c. @

d. 10 d. *

e. 12 e. U

12) How many letters are there between the fourth 15) If all the alphabets are arranged in alphabetical

element from the right and twelfth element from the order on the same places where alphabets placed

right after completion of all the steps? after the last step which of the following element is
seventh to the left of seventeenth from the left end?
a. One
a. X
b. Two
b. O
c. Three
c. S
d. Four
d. P
e. More than four
e. J
13) Which of the following is seventh to the right of
eight from the left in step 2? 16-20) Directions: Study the following information to
answer the question given below:
a. 9
Page 1004 of 1334
Subscribe The Xpress Video Course & Mock Test Package for Bank & Insurance Exams
If there are any suggestions/ errors in our PDFs Feel Free to contact us via this email: admin@exampundit.in
Ultra Practice Bundle PDF
SBI Clerk/ RBI Assist. Mains – Reasoning
@M36P%9KTQ5C$8A#7DS*H2WZ c. Three

Step: 1 – Those Numbers which are immediately d. None


preceded by the symbol and immediately followed by an
e. More than three
Alphabet are written at the right end in ascending order.
18) Which of the following element is 9th from the
Step: 2 – After completing the step -1, interchange the
right end in step 2?
Odd number with the previous element in the series to
form the step -2 a. $

Step: 3 – After completing the step - 2, Alphabets which b. A


are immediately followed by a symbol are written
c. S
between sixth and seventh element from the right end; in
alphabetical order from left to right. d. *

16) Which of the following element is seventh to the e. C


right of the element which is Tenth from the left end
19) How many Symbols are immediately followed and
in step -3?
immediately preceded by an alphabet in step -2?
a. Z
a. One
b. P
b. Two
c. D
c. Three
d. W
d. None
e. H
e. More than three

20) Which element is 2th to the right of 6nd element


17) How many numbers are immediately followed from the right end after step 3?
and immediately preceded by an alphabet in step -3?
a. 2
a. One
b. W
b. Two
c. 9
Page 1005 of 1334
Subscribe The Xpress Video Course & Mock Test Package for Bank & Insurance Exams
If there are any suggestions/ errors in our PDFs Feel Free to contact us via this email: admin@exampundit.in
Ultra Practice Bundle PDF
SBI Clerk/ RBI Assist. Mains – Reasoning
d. 7 e. *

e. 8 22) How many elements are there to right of ‘^’ in


step 4?
Direction 21-25: Read the information carefully and
answer the questions. a. 6

Input: (3 G & I 4 D 7 # U X 2 1 * A 8 H 6 & 9 ^ M P ! b. 8


# 5) c. 1

Step 1: If the number is followed by a symbol then it d. 5


interchanged with that symbol.
e. 9
Step 2: After completing step 1, all even numbers are
23) How many symbols are there which are followed
written between 10th element and 11th element from left,
by vowels in step1?
arranged in ascending order.
a. Three
Step 3: After completing step 2, the symbols preceded by
b. One
a number and succeeded by a letter is written between
11th element and 12th element from the right end. c. Four

Step 4: After completing step 3, the odd numbers d. None


followed by a letter are written at the beginning of the e. Two
series in the reverse order of the given series.
24) Which element is 3rd to the right of the element
21) In step 3, which of the following element is which is 4th to the left of the letter which immediate
between 8th element and 10th element from the right right of P in step 3?
end?
a. !
a. H
b. #
b. ^ c. P

c. M d. M

d. 1 e. 9
Page 1006 of 1334
Subscribe The Xpress Video Course & Mock Test Package for Bank & Insurance Exams
If there are any suggestions/ errors in our PDFs Feel Free to contact us via this email: admin@exampundit.in
Ultra Practice Bundle PDF
SBI Clerk/ RBI Assist. Mains – Reasoning
25) In step 4, how many symbols are preceded by 27) How many symbols are succeeded by consonant
Symbols? in step 2?

a. Three a. 4

b. None b. 7

c. Four c. 3

d. One d. 1

e. Two e. none

26-30) Direction: Read the information carefully and 28) In step 3, how many consonants are preceded by a
answer the questions. symbol and not followed by a number?

Input: & 1 B * 5 4 9 Z & 3 T 0 2 # 1 C W 2 7 * ! 6 K # 3 a. 6


@21D^5
b. 4
Step 1: The Consonants preceded by a number are
c. 3
interchanged with each other.
d. 2
Step 2: After completing step 1, the symbols which are
followed by odd numbers are written at the first, e. 0

according to their occurrence in the series. 29) Which element is at the beginning in the step 2?

Step 3: After completing step 2, even numbers are a. ^


multiplied by 3.
b. B
26) In step 3, which element is 8th from the left end?
c. !
a. 9
d. *
b. 12
e. Z
c. Z
30) How many letters are succeeded by or preceded
d. 5 by a number in step 1?

e. & a. 9
Page 1007 of 1334
Subscribe The Xpress Video Course & Mock Test Package for Bank & Insurance Exams
If there are any suggestions/ errors in our PDFs Feel Free to contact us via this email: admin@exampundit.in
Ultra Practice Bundle PDF
SBI Clerk/ RBI Assist. Mains – Reasoning
b. 10 a. 6

c. 7 b. 7

d. 8 c. 3

e. 4 d. 1

31-35) Direction: Read the information carefully and e. none


answer the questions.
33) In step 3, how many numbers are preceded by a
Input: 2 R * 7 8 E $ G 2 # 4 9 L % K 1 & A W? P + Q symbol?
@8F6
a. 6
Step 1: If the Symbol is followed by an even number
b. 4
than position of both will be interchanged.
c. 3
Step 2: After completing step 1, all consonants will be
d. 2
changed to immediate succeeding letter in English
alphabet series. e. 1

Step 3: After completing step 2, the vowels present in the 34) How many elements between the elements which
series should be changed to their immediate preceding 4th to the right and 3rd to the left after step 2?
letter in English alphabet series. a. 18

31) In step 3, which element is 6th from the left end? b. 19

a. 7 c. 20

b. $ d. 21

c. D e. 17

d. H 35) How many consonants are succeeded by and

e. None of these preceded by a symbol in step 1?

32) How many symbols are preceded by consonant in a. 2

step 3? b. 3
Page 1008 of 1334
Subscribe The Xpress Video Course & Mock Test Package for Bank & Insurance Exams
If there are any suggestions/ errors in our PDFs Feel Free to contact us via this email: admin@exampundit.in
Ultra Practice Bundle PDF
SBI Clerk/ RBI Assist. Mains – Reasoning
c. 5 e. None of these

d. 6 37) How many elements are there to right of 4th

e. 4 consonant from the left in step -2?

Answer-B a. 16

36-40) Direction: Read the information carefully and b. 18

answer the questions. c. 17

Input: 5 K A 7 6 3 2 @ T 8 € V 7 % B 3 E G $ M P © d. 15
QT%3UB65$#
e. 21
Step 1: All the numbers which are preceded by
38) How many symbols are there which are followed
consonants should be written after the final element of
by even numbers in step1?
the series of the given input in the ascending order.
a. Three
Step 2: After completing step 1, all Vowels should be
changed to immediate succeeding alphabet b. One

c. Four
Step 3: After completing step 2, all the symbols which
are immediately followed and succeeding by consonants d. None
should be written before the 1st element of the series.
e. Two
` 39) Which element is 6th to the right of the element
36) After step 3, which element is between 3rd which is 7th to the left in step 3?
element and 7th element from the right end? a. 2

a. $ b. @

b. # c. T

c. 3 d. V

d. B e. B

Page 1009 of 1334


Subscribe The Xpress Video Course & Mock Test Package for Bank & Insurance Exams
If there are any suggestions/ errors in our PDFs Feel Free to contact us via this email: admin@exampundit.in
Ultra Practice Bundle PDF
SBI Clerk/ RBI Assist. Mains – Reasoning
40) In step 2, how many symbols are preceded by 41) What is the sum of the element which is fourth
Consonants? from the right in step 2 and 10th from the left in step
3?
a. Three
a. 9
b. None
b. 13
c. Five
c. 11
d. One
d. 10
e. Two
e. 12
41-45) Direction: Read the information carefully and
answer the questions. 42) How many letters are there between the sixth
element from the right and thirteenth element from
¥R6#JK4@5L7*1MN$JH£G9©5Z%TK
the left in step 3?
&
a. Eight
Step 1: The letters which are immediately preceded by
symbols are to be arranged in the alphabetical order b. Six
immediately after the 3rd element from the left of the
c. Seven
series
d. Five
Step 2: The number which is immediately followed by
symbol is to arrange after the end of the series in the e. Four

ascending order. 43) Which of the following is 10th to the right of 3rd

Step 3: The symbol which is immediately preceded by an from the left in step 2?

alphabet whose position is even in the English a. K


rd
alphabetical order is to be placed after the last 3
b. &
element of the series from the right.
c. 7
Note – Step 2 is performed after completion of step 1 and
step 3 is to be performed after completion of step 2. d. %

e. 1

Page 1010 of 1334


Subscribe The Xpress Video Course & Mock Test Package for Bank & Insurance Exams
If there are any suggestions/ errors in our PDFs Feel Free to contact us via this email: admin@exampundit.in
Ultra Practice Bundle PDF
SBI Clerk/ RBI Assist. Mains – Reasoning
44) How many such symbols are there in the step -1 series is replaced by the alphabet as per its represented
each of which are immediately preceded by a number number.
and followed by an alphabet?
2) If each letter in A-Z alphabetical series is represented
a. One by number 1-26 respectively, the letters whose position

b. Two is a prime number in the English alphabet series are to be


replaced with their represented number.
c. Three
3) Symbols are replaced by immediate preceding element
d. Four

e. More than four

45) If all the symbols in the above arrangement are 46) How many such numbers are there in the above

dropped, then which of the following will be the tenth arrangement each of which are immediately preceded

from the right end in step -3 of the arrangement? by a consonant and followed by a number in after the
final arrangement?
a. N
a. Four
b. M
b. Two
c. H

d. 5 c. One

e. Z d. Three

46-50) Direction: Read the information carefully and e. More than three
answer the questions.
47) If all the symbols in the odd number from the
K@R5$U6&T£H8©I¥OP3€1 final arrangement are dropped, then which of the

There are certain operations which are to be applied, and following will be the tenth from the right end?

then mark your answer accordingly. a. K

1) If each letter in A-Z alphabetical series is represented b. R


by number 1-26 respectively, then only vowels in the
c. 8

Page 1011 of 1334


Subscribe The Xpress Video Course & Mock Test Package for Bank & Insurance Exams
If there are any suggestions/ errors in our PDFs Feel Free to contact us via this email: admin@exampundit.in
Ultra Practice Bundle PDF
SBI Clerk/ RBI Assist. Mains – Reasoning
d. I b. Six

e. None of these c. Five

48) How many such consonants is/are there in the d. Three


series which is/are proceeded by a even number but
e. Four
not followed by an alphabet?
50) What is the sum of numbers are there in the
a. One
series?
b. Two
a. 94
c. Three
b. 92
d. Four
c. 93
e. Five
d. 90
49) How many pairs of numbers/alphabets are there
e. 95
in the series?

a. Seven

Alpha Numeric New Pattern - Answer and Explanation


Solution 1-5

Page 1012 of 1334


Subscribe The Xpress Video Course & Mock Test Package for Bank & Insurance Exams
If there are any suggestions/ errors in our PDFs Feel Free to contact us via this email: admin@exampundit.in
Ultra Practice Bundle PDF
SBI Clerk/ RBI Assist. Mains – Reasoning
A2ZRQ#Y$%U4*W1%D9S@T8

10 Consecutive letters starting from 11th letter = K, L, M, N, O, P, Q, R, S, T.

K = 1, L = 2, M = 3, N = 4, O = 5, P = 6, Q = 7, R = 8, S = 9, T = 0

Series A 2 Z R Q # Y $ % U 4 * W 1 % D 9 S @ T 8

Condition 1 B D H

Condition 2 8 7 9 0

Condition 3 $ $

Final A B Z 8 7 # Y $ $ U D * W 1 $ D 9 9 @ 0 H

Final Series: A B Z 8 7 # Y $ $ U D * W 1 $ D 9 9 @ 0 H

1. C
2. C
3. B
4. D
5. A

Solution 6-10

Consider the series given:

Series 1 # W $ 4 S $ D & * 2 7 ! A H 4 % 8 @ ! 5 3 L 9 F @ Q $ 6 <

Condi A D B G D H E C I F
tion 1

Condi 1 4 4 1 8 4 8 5 3 9 6 6
tion 2

Page 1013 of 1334


Subscribe The Xpress Video Course & Mock Test Package for Bank & Insurance Exams
If there are any suggestions/ errors in our PDFs Feel Free to contact us via this email: admin@exampundit.in
Ultra Practice Bundle PDF
SBI Clerk/ RBI Assist. Mains – Reasoning
Condi < < < < <
tion 3

Final 1 # W < 4 S < 4 & * 2 7 < 1 8 4 % 8 @ < 5 3 L 9 6 @ Q < 6 <

Final Series: are to be arranged in descending order immediately after


the last element of the series.
1#W<4S<4&*27<184%8@<53L96@Q
<6< Step 3: 7 4 $ G U 5 * % @ X 3 # S 9 & ∆ I 8 1 © 6 P J ^
MOV2
6. E
7. A Step 3: The symbol which is immediately preceded by a
8. C letter is to be interchange with the element that
9. D immediately followed that symbol.
10. B
If all the alphabets are arranged in alphabetical order
Solutions 11-15
New series: 7 4 $ G I 5 * % @ J 3 # M 9 & ∆ O 8 1 © 6
Given series: 7 M 4 $ G U * 5 % @ X 3 # S & 2 9 ∆ I 8 PS^UVX2
O1J©6VP^
11. C
Step 1: The letters which are immediately preceded by 12. B
number are to be arranged in the alphabetical order 13. A
immediately after the last element of the series. 14. D
Step 1: 7 4 $ G U * 5 % @ X 3 # S & 2 9 ∆ I 8 1 © 6 P ^ 15. E
JMOV

Step 2: 7 4 $ G U * 5 % @ X 3 # S & 9 ∆ I 8 1 © 6 P ^ J Solution: 16-20

MOV2 Given Input: @ M 3 6 P % 9 K T Q 5 C $ 8 A # 7 D S


*H2WZ
Step 2: The number which is immediately followed by
another number and immediately preceded by the symbol
Page 1014 of 1334
Subscribe The Xpress Video Course & Mock Test Package for Bank & Insurance Exams
If there are any suggestions/ errors in our PDFs Feel Free to contact us via this email: admin@exampundit.in
Ultra Practice Bundle PDF
SBI Clerk/ RBI Assist. Mains – Reasoning
Step: 1 – Those Numbers which are immediately Step 1: If the number is followed by a symbol then it
preceded by the symbol and immediately followed by an interchanged with that symbol.
Alphabet are written at the right end in ascending order.
Step 1: (3 G & I 4 D # 7 U X 2 * 1 A 8 H & 6 ^ 9 M P !
Step 1: @ M 3 6 P % K T Q 5 C $ A # D S * H 2 W Z # 5)
789
Step 2: After completing step 1, all even numbers are
Step: 2 – After completing the step -1, interchange the written between 10th element and 11th element, arranged
Odd number with the previous element in the series to in ascending order.
form the step -2
(3 G & I D # 7 U X 2 4 6 8 * 1 A H & ^9 M P ! # 5)
Step 2: @ 3 M 6 P % K T 5 Q C $ A # D S * H 2 W 7 Z
98 Step 3: There are no such symbols their series remains
the same as in step 2.
Step: 3 – After completing the step - 2, Alphabets which
are immediately followed by a symbol are written (3 G & I D # 7 U X 2 4 6 8 * 1 A H & ^ 9 M P ! # 5)
between sixth and seventh element from the right end; in
Step 4: After completing step 3, the odd numbers
alphabetical order from left to right.
followed by a letter are written at the beginning of the
Final series:- series in the reverse order of the given series.

Step 3: @ 3 M 6 % K T 5 Q $ # D * H A C P S 2 W 7 Z
9173(G&ID#UX24 68 *AH&^MP!#5
98
21. A
16. B
22. D
17. C
23. B
18. C
24. C
19. E
25. E
20. D
Solution 26-30

Solution 21-25 Step 1: The Consonants preceded by a number are


interchanged with each other.
Given Input: (3 G & I 4 D 7 # U X 2 1 * A 8 H 6 & 9 ^
M P ! # 5)

Page 1015 of 1334


Subscribe The Xpress Video Course & Mock Test Package for Bank & Insurance Exams
If there are any suggestions/ errors in our PDFs Feel Free to contact us via this email: admin@exampundit.in
Ultra Practice Bundle PDF
SBI Clerk/ RBI Assist. Mains – Reasoning
Input: & B 1 * 5 4 Z 9 & T 3 0 2 # C 1 W 2 7 * ! K 6 # 3 Step 2: After completing step 1, all consonants will be
@ 2 D1 ^ 5 changed to immediate succeeding letter in English
alphabet series.
Step 2: After completing step 1, the symbols followed by
an odd number are written at the first, according to their Input: 2 S * 7 8 E $ H 2 4# 9 M % L 1 2 & X? Q + R 8
occurrence in the series. @G6

Input: * # ^ & B 1 5 4 Z 9 & T 3 0 2 # C 1 W 2 7 * ! K 6 Step 3: After completing step 2, the vowels present in the
3@2D15 series should be changed to their immediate preceding
letter in English alphabet series.
Step 3: After completing step 2, even numbers are
multiplied by 3. Input: 2 S * 7 8 D $ H 2 4# 9 M % L 1 2 & X? Q + R 8
@G6
Input:* # ^ & B 1 5 12 Z 9 & T 3 0 6 # C 1 W 6 7 * ! K
18 3 @ 6 D 1 5 31. C
32. A
26. B 33. D
27. A 34. C
28. E 35. A
29. D
Solutions 36-40
30. C
Input: 5 K A 7 6 3 2 @ T 8 € V 7 % B 3 E G $ M P ©
Solution 31-35
QT%3UB65$#
Input: 2 R * 7 8 E $ G 2 # 4 9 L % K 1 & 2 W? P + Q @
Step 1: All the numbers which are preceded by
8F6
consonants should be written after the final element of
Step 1: If the Symbol is followed by an even number the series of the given input in the ascending order.
than position of both will be interchanged.
Input: 5 K A 7 6 3 2 @ T € V % B E G $ M P © Q T %
Input: 2 R * 7 8 E $ G 2 4# 9 L % K 1 2 & W? P + Q 8 3UB5$#3678
@F6
Step 2: After completing step 1, all Vowels should be
changed to immediate succeeding alphabet

Page 1016 of 1334


Subscribe The Xpress Video Course & Mock Test Package for Bank & Insurance Exams
If there are any suggestions/ errors in our PDFs Feel Free to contact us via this email: admin@exampundit.in
Ultra Practice Bundle PDF
SBI Clerk/ RBI Assist. Mains – Reasoning
Input: 5 K B 7 6 3 2 @ T € V % B F G $ M P © Q T % Ste 3: The symbol which is immediately preceded by an
3VB5$#3678 alphabet whose position is even in the English
alphabetical order is to be placed after the last 3rd
Step 3: After completing step 2, all the symbols which
element of the series from the right.
are immediately followed and preceded by consonants
should be written before the 1st element of the series. This is the final series:

Input: € % $ © 5 K B 7 6 3 2 @ T V B F G M P Q T % ¥#GJJRTK@5L1MNH©5ZK&46*$£%
3VB5$#3678 79

36. B 41. A
37. E 42. E
38. D 43. E
39. C 44. A
40. C 45. B

Solutions 41-45 Solution 46-50

¥R6#JK4@5L7*1MN$JH£G9©5Z%T Given Series: - K @ R 5 $ U 6 & T £ H 8 © I ¥ O P 3 € 1

K&

Step 1: The letters which are immediately preceded by


symbols are to be arranged in the alphabetical order
immediately after the 3rd element from the left of the
series

¥6#GJJRTK4@5L7*1MN$H£9©5Z%K
&

Step 2: The number which is immediately followed by


symbol is to be arrange after the end of the series in the
ascending order.

¥#GJJRTK@5L*1MN$H£©5Z%K&46
79
Page 1017 of 1334
Subscribe The Xpress Video Course & Mock Test Package for Bank & Insurance Exams
If there are any suggestions/ errors in our PDFs Feel Free to contact us via this email: admin@exampundit.in
Ultra Practice Bundle PDF
SBI Clerk/ RBI Assist. Mains – Reasoning
Seri K @R 5 $ U 6 & T £ H 8 © I ¥ O P 3 € 1
es

Con 2 9
diti 1 1
on 5
1

Con 1
diti 1
on
2

Con 1 5 6 8 9 3
diti 1 T
on
3

Fin 1 1 R 5 5 2 6 6 T T H 8 8 9 9 1 P 3 3 1
al 1 1 1 5

48. A

Final Series: 11 11 R 5 5 21 6 6 T T H 8 8 9 9 15 P 3 3 1 49. A

46. D 50. D

47. B

Download Seating Arrangement Practice Questions PDF


Get More Reasoning Practice Questions PDF

Page 1018 of 1334


Subscribe The Xpress Video Course & Mock Test Package for Bank & Insurance Exams
If there are any suggestions/ errors in our PDFs Feel Free to contact us via this email: admin@exampundit.in
Ultra Practice Bundle PDF
SBI Clerk/ RBI Assist. Mains – Reasoning
Alphabetical Series
Directions (1-5): Following questions are based on (b) One
five words given below:
(c) Two
RICH PICK FLAT SOUL ROAR (d) Three

(Note: The words formed after performing the given (e) None
operations may or may not be meaningful English
4. How many letters are there in the English
words.)
alphabetical series between the second letter of the
1. If all the letters in each of the words are arranged
word which is second from the right and the second
alphabetically (within the word), how many words
letter of the word which is second from the left of the
will remain unchanged?
given words?
(a) One
(a) Two
(b) Two
(b) Five
(c) None
(c) Six
(d) Three
(d) Nine
(e) Four
(e) Three
2. If the given words are arranged in the order as
they would appear in a dictionary from left to right, 5. If in each of the given words each of the consonants
which of the following will be second from the right? is changed to previous letter and each vowel is
changed to next letter in the English alphabetical
(a) SOUL
series, in how many words thus formed will no vowels
(b) PICK appear?

(c) RICH (a) None

(d) FLAT (b) One

(e) ROAR (c) Two

3. If last letter in each of the words is changed to next (d) More than three
alphabet in the English alphabetical order, how many
(e) Three
words having two vowels (same or different vowels)
will be formed?

(a) Four
Page 1019 of 1334
Subscribe The Xpress Video Course & Mock Test Package for Bank & Insurance Exams
If there are any suggestions/ errors in our PDFs Feel Free to contact us via this email: admin@exampundit.in
Ultra Practice Bundle PDF
SBI Clerk/ RBI Assist. Mains – Reasoning
Directions (6-10): Following questions are based on (e) Four
five words given below:
TUB HUB MUG PAR FOR 9. How many letters are there in the English
alphabetical series between the first letter of the word
6. If the positions of the first and the third alphabets which is third from the right and the second letter of
of each words are interchanged, which of the the word which is first from the left of the given
following would form meaningful words with the new words?
arrangement? (a) Two
(a) TUB
(b) Five
(b) HUB
(c) Six
(c) MUG and TUB
(d) Nine
(d) PAR and TUB
(e) Seven
(e) FOR and HUB
10. If in each of the given words each of the
7. If the given words are arranged in the order as consonants is changed to previous letter and each
they would appear in a dictionary from right to left, vowel is changed to next letter in the English
which of the following will be second from the left? alphabetical series, in how many words thus formed
(a) FOR will no vowels appear?

(b) HUB (a) One

(c) MUG (b) More than three

(d) PAR (c) Two

(e) TUB (d) Three

8. If first letter in each of the words is changed to (e) None


previous alphabet in the English alphabetical order, Directions (11-15): Following questions are based on
how many words having two vowels (same or five words given below:
different vowels) will be formed?
LEAK PEST CHOK TENT LOKI
(a) None
(The new words formed after performing the mentioned
(b) One operations may or may not necessarily be meaningful
(c) Two English words.)

(d) Three
Page 1020 of 1334
Subscribe The Xpress Video Course & Mock Test Package for Bank & Insurance Exams
If there are any suggestions/ errors in our PDFs Feel Free to contact us via this email: admin@exampundit.in
Ultra Practice Bundle PDF
SBI Clerk/ RBI Assist. Mains – Reasoning
11. If the fourth alphabet of each of the words is 14. How many letters are there in the English
changed to the next alphabet in the English alphabetical series between the first letter of the word
alphabetical series, which of the following has exactly which is second from the right and the second letter
one vowel? of the word which is first from the left of the given
words?
(a) CHOK
(a) Twelve
(b) LOKI
(b) Fifteen
(c) PEST
(c) Ten
(d) LEAK
(d) Fourteen
(e) Both CHOK and LOKI
(e) Three
12. If the given words are arranged in the order as
they would appear in a dictionary from left to right, 15. If in each of the given words each of the
which of the following will be fourth from the right? consonants is changed to previous letter and each
vowel is changed to next letter in the English
(a) LEAK alphabetical series, in how many words thus formed
(b) LOKI will no vowels appear?

(c) PEST (a) None

(d) TENT (b) One

(e) CHOK (c) Two

13. If first alphabet in each of the words is changed to (d) Three


previous alphabet in the English alphabetical order, (e) More than three
how many words having two vowels (same or
different vowels) will be formed? Directions (16-20): Following questions are based on
five words given below:
(a) None
LAPE CUTE CARE SOON HIDE
(b) One
(The new words formed after performing the mentioned
(c) Two operations may or may not necessarily be meaningful
(d) Three English words.)

(e) Four 16. If the third letter of each of the words is changed
to the next alphabet in the English alphabetical series,
which of the following has exactly one vowel?
Page 1021 of 1334
Subscribe The Xpress Video Course & Mock Test Package for Bank & Insurance Exams
If there are any suggestions/ errors in our PDFs Feel Free to contact us via this email: admin@exampundit.in
Ultra Practice Bundle PDF
SBI Clerk/ RBI Assist. Mains – Reasoning
(a) LAPE (a) Two

(b) HIDE (b) Five

(c) CUTE (c) Six

(d) CARE (d) None

(e) SOON (e) Three

17. If the given words are arranged in the order as 20. If first letter in each of the words is replaced to
they would appear in a dictionary from left to right, ‘B’ then how many words will be formed meaningful?
which of the following will be second from the left?
(a) None
(a) CUTE
(b) One
(b) LAPE
(c) Two
(c) SOON
(d) Three
(d) HIDE
(e) More than three
(e) CARE
Directions (21-25): Following questions are based on
18. If in each of the given words, each of the five words given below:
consonants is changed to previous letter in the
English alphabetical series, in how many words thus TAPE LATE DONE FOOT SUIT
formed will at least three vowel appear? (The new words formed after performing the mentioned
(a) None operations may or may not necessarily be meaningful
English words.)
(b) One
21. If the positions of the first and the third alphabets
(c) Two of each words are interchanged, how many words
form meaningful words with the new arrangement?
(d) Three
(a) One
(e) More than three
(b) Two
19. How many letters are there in the English
alphabetical series between the third letter of the (c) Three
word which is third from the right and the first letter
of the word which is first from the left of the given (d) Four
words? (e) None

Page 1022 of 1334


Subscribe The Xpress Video Course & Mock Test Package for Bank & Insurance Exams
If there are any suggestions/ errors in our PDFs Feel Free to contact us via this email: admin@exampundit.in
Ultra Practice Bundle PDF
SBI Clerk/ RBI Assist. Mains – Reasoning
22. If the given words are arranged in the order as (e) Four
they would appear in a dictionary from left to right,
which of the following will be second from the right? 25. If first consonent in each of the words is changed
to next alphabet in the English alphabetical order,
(a) LATE which word having Three different vowels will be
formed?
(b) SUIT
(a) TAPE
(c) TAPE
(b) LATE
(d) FOOT
(c) DONE
(e) DONE
(d) SUIT
23. If in each of the given words, each of the
consonants is changed to previous letter and each (e) FOOT
vowel is changed to next letter in the English
alphabetical series, in how many words thus formed Directions (26-30): Following questions are based on
will at least one vowel appear? five words given below:

LEFT POST QUIZ PAID MORE


(a) None
(The new words formed after performing the mentioned
(b) Two operations may or may not necessarily be meaningful
English words.)
(c) One
26. If the positions of the first and the third alphabets
(d) Three
of each words are interchanged, which of the
(e) More than three following would form meaningful words with the new
arrangement?
24. How many letters are there in the English
alphabetical series between the second letter of the (a) QUIZ
word which is second from the right and the first
(b) MORE
letter of the word which is first from the left of the
given words? (c) Both LEFT and MORE

(a) Two (d) PAID

(b) Five (e) Both POST and QUIZ

(c) Six 27. If the given words are arranged in the order as
they would appear in a dictionary from left to right,
(d) None
which of the following will be fourth from the left?
Page 1023 of 1334
Subscribe The Xpress Video Course & Mock Test Package for Bank & Insurance Exams
If there are any suggestions/ errors in our PDFs Feel Free to contact us via this email: admin@exampundit.in
Ultra Practice Bundle PDF
SBI Clerk/ RBI Assist. Mains – Reasoning
(a) POST 30. If second alphabet in each of the words is changed
to next alphabet in the English alphabetical order,
(b) MORE how many words having two vowels will be formed?
(c) QUIZ (a) None
(d) LEFT (b) One
(e) PAID (c) Two
28. If in each of the given words, each of the (d) Three
consonants is changed to previous letter and each
vowel is changed to next letter in the English (e) More than three
alphabetical series, in how many words thus formed
will at least one vowel appear? Directions (31-35): Following questions are based on
five words given below:
(a) None
SORT ABLE BOND DUKE GIFT
(b) One
(The new words formed after performing the mentioned
(c) Two operations may or may not necessarily be meaningful
English words.)
(d) Three
31. If the positions of the first and the second letter of
(e) More than three each words are interchanged, which of the following
29. How many letters are there in the English would not start from vowel in the new arrangement?
alphabetical series between the third letter of the (a) GIFT
word which is second from the right and the first
letter of the word which is second from the left of the (b) DUKE
given words?
(c) ABLE
(a) Two
(d) SORT
(b) Five
(e) BOND
(c) Six
32. If the given words are arranged in the order as
(d) None they would appear in a dictionary from right to left,
which of the following will remains on the same
(e) Three position in the new arrangement?

(a) SORT

Page 1024 of 1334


Subscribe The Xpress Video Course & Mock Test Package for Bank & Insurance Exams
If there are any suggestions/ errors in our PDFs Feel Free to contact us via this email: admin@exampundit.in
Ultra Practice Bundle PDF
SBI Clerk/ RBI Assist. Mains – Reasoning
(b) GIFT (b) Three

(c) DUKE (c) Two

(d) ABLE (d) One

(e) BOND (e) More than three

33. If in each of the given words, each of the Directions (36-40): Following questions are based on
consonants is changed to previous letter and each five words given below:
vowel is changed to next letter in the English
alphabetical series, in how many words thus formed FUEL HARM ITEM KNOW LACK
will at least one vowel appear? (The new words formed after performing the mentioned
(a) None operations may or may not necessarily be meaningful
English words.)
(b) One
36. If the positions of the first and the third alphabets
(c) Two of each words are interchanged, which of the
following would starts from consonant in the new
(d) Three arrangement?
(e) More than three (a) KNOW
34. How many letters are there in the English (b) KNOW
alphabetical series between the first letter of the word
which is first from the right and the first letter of the (c) Both HARM and KNOW
word which is first from the left of the given words?
(d) LACK
(a) Eleven
(e) Both HARM and LACK
(b) Five
37. If the given words are arranged in the order as
(c) Six they would appear in a dictionary from right to left,
which of the following will be third from the right?
(d) None
(a) HARM
(e) Twelve
(b) FUEL
35. If third alphabet in each of the words is changed
to next alphabet in the English alphabetical order, (c) ITEM
how many words having two vowels will be formed?
(d) KNOW
(a) None
(e) LACK
Page 1025 of 1334
Subscribe The Xpress Video Course & Mock Test Package for Bank & Insurance Exams
If there are any suggestions/ errors in our PDFs Feel Free to contact us via this email: admin@exampundit.in
Ultra Practice Bundle PDF
SBI Clerk/ RBI Assist. Mains – Reasoning
38. If in each of the given words, each of the (e) ITEM
consonants is changed to previous letter and each
vowel is changed to next letter in the English Directions (41-45): Following questions are based on
alphabetical series, in how many words thus formed five words given below:
will at least one vowel appear? MAIL GROW COST HOUR LOVE
(a) None (The new words formed after performing the mentioned
(b) One operations may or may not necessarily be meaningful
English words.)
(c) Two
41. If the positions of the first and the third alphabets
(d) Three of each words are interchanged, which of the
following would form meaningful words with the new
(e) More than three arrangement?
39. How many letters are there in the English (a) HOUR
alphabetical series between the third letter of the
word which is second from the right and the first (b) MAIL
letter of the word which is second from the left of the
given words? (c) COST

(a) Two (d) LOVE

(b) Five (e) GROW

(c) Six 42. If the given words are arranged in the order as
they would appear in a dictionary from left to right,
(d) None which of the following will be second from the left?

(e) Three (a) GROW

40. If first alphabet in each of the words is changed to (b) HOUR


next alphabet in the English alphabetical order,
which of the following will start with vowels in the (c) LOVE
new arrangement? (d) COST
(a) KNOW (e) MAIL
(b) HARM 43. If in each of the given words, each of the
(c) LACK consonants is changed to previous letter and each
vowel is changed to next letter in the English
(d) FUEL
Page 1026 of 1334
Subscribe The Xpress Video Course & Mock Test Package for Bank & Insurance Exams
If there are any suggestions/ errors in our PDFs Feel Free to contact us via this email: admin@exampundit.in
Ultra Practice Bundle PDF
SBI Clerk/ RBI Assist. Mains – Reasoning
alphabetical series, in how many words thus formed Directions (46-50): Following questions are based on
will at least one vowel appear? five words given below:

(a) None ACID SIDE POOL DESK BOAT

(b) One (The new words formed after performing the mentioned
operations may or may not necessarily be meaningful
(c) Two English words.)
(d) Three 46. If the positions of the first and the second
(e) More than three alphabets of each words are interchanged, which of
the following would form meaningful words with the
44. How many letters are there in the English new arrangement?
alphabetical series between the third letter of the
word which is third from the right and the first letter (a) DESK
of the word which is first from the right of the given (b) POOL
words?
(c) Both ACID and SIDE
(a) Two
(d) None
(b) Five
(e) Both DESK and BOAT
(c) Six
47. If the given words are arranged in the order as
(d) None they would appear in a dictionary from left to right,
(e) Three which of the following will be fourth from the left?

45. If First alphabet in each of the words is changed (a) POOL


to next alphabet in the English alphabetical order, (b) ACID
which of the following word start with vowels in the
new arrangement? (c) DESK

(a) MAIL (d) BOAT

(b) HOUR (e) SIDE

(c) LOVE 48. If in each of the given words, each of the


consonants is changed to previous letter and each
(d) COST vowel is changed to next letter in the English
(e) GROW alphabetical series, in how many words thus formed
will at least one vowel appear?

Page 1027 of 1334


Subscribe The Xpress Video Course & Mock Test Package for Bank & Insurance Exams
If there are any suggestions/ errors in our PDFs Feel Free to contact us via this email: admin@exampundit.in
Ultra Practice Bundle PDF
SBI Clerk/ RBI Assist. Mains – Reasoning
(a) None (c) Four

(b) One (d) eleven

(c) Two (e) Three

(d) Three 50. If second alphabet in each of the words is changed


to next alphabet in the English alphabetical order,
(e) More than three how many words having no vowels will be formed?
49. How many letters are there in the English (a) None
alphabetical series between the third letter of the
word which is third from the left and the forth letter (b) One
of the word which is first from the left of the given
words? (c) Two

(a) Ten (d) Three

(b) Five (e) More than three

Alphabetical Series– Answer and Explanation

SOLUTIONS(1-5) RICH⇒CHIR, PICK⇒CIKP, FLAT⇒AFLT,


S1. Ans. (c) SOUL⇒LOSU, ROAR⇒AORR

Hence, none such word remain unchanged


Page 1028 of 1334
Subscribe The Xpress Video Course & Mock Test Package for Bank & Insurance Exams
If there are any suggestions/ errors in our PDFs Feel Free to contact us via this email: admin@exampundit.in
Ultra Practice Bundle PDF
SBI Clerk/ RBI Assist. Mains – Reasoning
S2. Ans. (e) TUB PAR MUG HUB FOR

FLAT PICK RICH ROAR SOUL S8. Ans. (c)

S3. Ans. (a) SUB GUB LUG OAR EOR

RICI PICL FLAU SOUM ROAS Hence, two word has two vowel if the first letter of all
the words is changed to the next letter of the English
Hence, four word has two vowel if the last letter of all alphabet.
the words is changed to the next letter of the English
alphabet. S9. Ans. (e)

S4. Ans. (b) TUB HUB MUG PAR FOR

RICH PICK FLAT SOUL ROAR Hence, seven word has between M and U in the English
alphabet.
Hence, five word has between O and I in the English
alphabet. MNOPQRSTU

IJKLMNO S10. Ans. (a)

S5. Ans. (e) SVA GVA LVF OBQ EPQ

QJBG OJBJ EKBS RPVK QPBQ Hence, There are four words which has one vowel.

Hence, Only OJBJ and EKBS has one vowel. SOLUTIONS (11-15)

SOLUTIONS (6-10)
S11. Ans. (e)

LEAK⇒LEAL, PEST ⇒PESU, CHOK ⇒CHOL, TENT


S6. Ans. (c)
⇒TENU, LOKI ⇒LOKJ
TUB⇒BUT, HUB⇒BUH, MUG⇒GUM, PAR⇒RAP,
FOR⇒ROF S12. Ans. (a)

S7. Ans. (d)


Page 1029 of 1334
Subscribe The Xpress Video Course & Mock Test Package for Bank & Insurance Exams
If there are any suggestions/ errors in our PDFs Feel Free to contact us via this email: admin@exampundit.in
Ultra Practice Bundle PDF
SBI Clerk/ RBI Assist. Mains – Reasoning
CHOK LEAK LOKI PEST TENT CARE CUTE HIDE LAPE SOON

S13. Ans. (d) S18. Ans. (b)

KEAK OEST BHOK SENT KOKI KAOE BUSE BAQE ROOM GICE

Hence, Three word has two vowel if the first letter of all Hence, One word has atleast three vowel.
the words is changed to the next letter of the English
alphabet. S19. Ans. (b)

S14. Ans. (d) LAPE CUTE CARE SOON HIDE

LEAK PEST CHOK TENT LOKI Hence, five word has between L and R in the English
alphabet.
Hence, Fourteen word has between E and T in the
English alphabet. LMNOPQR

EFGHIJKLMNOPQRST S20. Ans. (c)

S15. Ans. (e) BAPE BUTE BARE BOON BIDE

KFBJ OFRS BGPJ SFMS KPJJ Hence, There are two meaningful words i.e. Bare and
Boon.

Hence, There are four words which has NO vowel.


SOLUTIONS (21-25)

SOLUTIONS (16-20)
S21. Ans. (c)

S16. Ans. (e) TAPE ⇒PATE, LATE ⇒TALE, DONE ⇒NODE,


FOOT ⇒OOFT, SUIT ⇒IUST
LAPE CUTE CARE SOON HIDE

LAPE⇒LAQE, CUTE ⇒CUUE, CARE ⇒CASE, S22. Ans. (b)


SOON ⇒SOPN, HIDE ⇒HIEE
DONE FOOT LATE SUIT TAPE
S17. Ans. (a)

S23. Ans. (b)

Page 1030 of 1334


Subscribe The Xpress Video Course & Mock Test Package for Bank & Insurance Exams
If there are any suggestions/ errors in our PDFs Feel Free to contact us via this email: admin@exampundit.in
Ultra Practice Bundle PDF
SBI Clerk/ RBI Assist. Mains – Reasoning
SBOF KBSF CPMF EPPS RVJS
S28. Ans. (d)
Hence, Two word has atleast One vowel.
LEFT POST QUIZ PAID MORE

S24. Ans. (e) KFES OPRS PVJY OBJC LPQF

TAPE LATE DONE FOOT SUIT Hence, three word has atleast one vowel.

Hence, FOUR word has between O and T in the English


S29. Ans. (c)
alphabet.

LEFT POST QUIZ PAID MORE


OPQRST
Hence, Six words has between I and P in the English
S25. Ans. (a) alphabet.

TAPE LATE DONE FOOT SUIT IJKLMNOP

UAPE MATE EONE GOOT TUIT


S30. Ans. (a)

LEFT POST QUIZ PAID MORE


SOLUTIONS (26-30)
LFFT PPST QVIZ PBID MPRE

S26. Ans. (c) SOLUTIONS (31-35)

LEFT POST QUIZ PAID MORE


S31. Ans. (c)

SORT ABLE BOND DUKE GIFT


LEFT ⇒FELT, POST ⇒ SOPT, QUIZ ⇒IUQZ, PAID
⇒IAPD, MORE ⇒ROME SORT⇒ OSRT, ABLE⇒ BALE, BOND⇒ OBND,
DUKE ⇒ UDKE, GIFT⇒ IGFT
S27. Ans. (a)
S32. Ans. (a)
LEFT POST QUIZ PAID MORE
SORT ABLE BOND DUKE GIFT
LEFT MORE PAID POST QUIZ
SORT GIFT DUKE BOND ABLE

Page 1031 of 1334


Subscribe The Xpress Video Course & Mock Test Package for Bank & Insurance Exams
If there are any suggestions/ errors in our PDFs Feel Free to contact us via this email: admin@exampundit.in
Ultra Practice Bundle PDF
SBI Clerk/ RBI Assist. Mains – Reasoning
Hence, only SORT will remains on same position. LACK KNOW ITEM HARM FUEL

S38. Ans. (b)


S33. Ans. (d)

FUEL HARM ITEM KNOW LACK


SORT ABLE BOND DUKE GIFT
EVFK GBQL JSFL JMPV KBBJ
RPQS BAKF APMC CVJF FJES

S39. Ans. (C)


Hence, three word has atleast one vowel.

FUEL HARM ITEM KNOW LACK


S34. Ans. (a)
Hence, Six word has between H and O in the English
SORT ABLE BOND DUKE GIFT alphabet.

Hence, Eleven word has between S and G in the English


HIJKLMNO
alphabet.

S40. Ans. (b)


GHIJKLMNOPQRS

FUEL HARM ITEM KNOW LACK


S35. Ans. (b)
GUEL IARM JTEM LNOW MACK
SORT ABLE BOND DUKE GIFT
SOLUTIONS (41-45)
SOST ABME BOOD DULE GIGT

Hence, THREE word has two vowel. S41. Ans. (d)

MAIL GROW COST HOUR LOVE


SOLUTIONS (36-40)
MAIL ⇒ IAML, GROW ⇒ ORGW, COST ⇒ SOCT,
HOUR ⇒ UOHR, LOVE ⇒ VOLE
S36. Ans. (e)

FUEL ⇒ EUFL, HARM ⇒ RAHM, ITEM ⇒ ETIM, S42. Ans. (a)


KNOW ⇒ ONKW, LACK ⇒ CALK
COST GROW HOUR LOVE MAIL
S37. Ans. (c)
S43. Ans. (b)
Page 1032 of 1334
Subscribe The Xpress Video Course & Mock Test Package for Bank & Insurance Exams
If there are any suggestions/ errors in our PDFs Feel Free to contact us via this email: admin@exampundit.in
Ultra Practice Bundle PDF
SBI Clerk/ RBI Assist. Mains – Reasoning
MAIL GROW COST HOUR LOVE S47. Ans. (a)

LBJK FQPV BPRS GPVQ KPUF


ACID SIDE POOL DESK BOAT

S44. Ans. (C) ACID BOAT DESK POOL SIDE

MAIL GROW COST HOUR LOVE


S48. Ans. (c)
Hence, Six word has between S and L in the English
alphabet. ACID SIDE POOL DESK BOAT

BBJC RJCF OPPK CFRJ APBS


LMNOPQRS
S49. Ans. (a)
S45. Ans. (b)
ACID SIDE POOL DESK BOAT
MAIL GROW COST HOUR LOVE
Hence, Ten words has between D and O in the English
NAIL HROW DOST IOUR MOVE alphabet.

SOLUTIONS (46-50)
DEFGHIJKLMNO

S46. Ans. (d) S50. Ans. (b)

ACID SIDE POOL DESK BOAT


ACID SIDE POOL DESK BOAT
ACID ⇒ CAID, SIDE ⇒ ISDE, POOL ⇒ OPOL, DESK
ADID SJDE PPOL DFSK BPAT
⇒ EDSK, BOAT ⇒ OBAT

Download Seating Arrangement Practice Questions PDF


Get More Reasoning Practice Questions PDF
Coding Decoding Number Symbol-Based Condition
Direction 1-5: In each question below is given a group the following coding system and the steps given and
of numbers / symbols followed by five combinations mark the number of that combination as your
of letter codes numbered (1), (2), (3), (4) and (5). You answer:
have to find out which of the combinations correctly
represents the group of numbers / symbols based on
Page 1033 of 1334
Subscribe The Xpress Video Course & Mock Test Package for Bank & Insurance Exams
If there are any suggestions/ errors in our PDFs Feel Free to contact us via this email: admin@exampundit.in
Ultra Practice Bundle PDF
SBI Clerk/ RBI Assist. Mains – Reasoning
Numb ^ 2 ) 5 # ( 1 8 * 3 7 6 % @ 4 c. BOSDWAH
ers /
Symb d. HBSHWAO
ols
e. BHGDWAH
Letter Z D G T Y W H L A O F P R B X
s Code 3) What is the code for 56^8%#@?

Steps to be followed as per the given order: a. LPSZRYT

(1) All the numbers are to be sorted in descending order b. LZPTRYB


and then written in the place of numbers only, i.e., the
c. TPLSRYB
place of symbols should not be shifted or changed.
d. LZPRTYB
(2) In the series obtained after the step one if it contains
any number followed by the symbol then the places of e. TPSTRYB
that number and the symbol should be interchanged.
4) What is the code for ^4(*68)?
(3) If a number is followed by another number then add
the two numbers to obtain the new series thus formed. a. ZXWAPLG

The new series which will be formed by following the b. ZWLAPGX


above steps will be then coded according to the table
c. HXSAPLZ
values given above.
d. ZXWSPLG
1) What is the code for 47%3@#1?
e. GXWSPLZ
a. FRXOBYH

b. FROXSYH
5) What is the code for 2*31% 5@?
c. FRXBOYH
a. ARLDBH
d. XFRXBYX
b. ALRBDH
e. FROBSYH
c. ALRDBH

d. ALRDHB
2) What is the code for 1@)2(*3?
e. None of these
a. BOGWDAH
. Direction 6-10 : In each question below is given a
b. BOGHWAO
group of numbers / symbols followed by five

Page 1034 of 1334


Subscribe The Xpress Video Course & Mock Test Package for Bank & Insurance Exams
If there are any suggestions/ errors in our PDFs Feel Free to contact us via this email: admin@exampundit.in
Ultra Practice Bundle PDF
SBI Clerk/ RBI Assist. Mains – Reasoning
combinations of letter codes numbered (1), (2), (3), (4) a. GPATW
and (5). You have to find out which of the
combinations correctly represents the group of b. GPAUW
numbers / symbols based on the following coding c. GPAUV
system and the steps given and mark the number of
that combination as your answer: d. GQAUW

Numbers / $ 6 @ 8 % > 7 2 + * 4 e.9GPOUW


< # &
Symbols
Letters Code C F N Q W U M P A O G P H B I
8) What is the code for 86&*4?
Steps to be followed as per the given order:
a. GFIPO
(1) All the numbers are to be sorted in ascending order
and then written in the place of numbers only b. GEIQO
(2) In the series obtained after the step one if it contains c. GFIQN
number preceded by symbol than both should
interchange d. GFIQO

(3) If odd number is followed by even number then e. None of these


subtract the lower number from higher number to obtain
9) What is the code for +49<@?
the new series thus formed.
a. None of these
The new series which will be formed by following the
above steps will be then coded according to the table b. GAPHN
values given above.
c. GAPGN
6) What is the code for 72*6@?
d. GAOHN
a. PEMON
e. GAPHJ
b. PFNON
10) What is the code for 68$%2?
c. PFMUN
a. PECQW
d. PFMON
b. PFDQW
e. None of these
c. PFCQV

d. PFCQW
7)What is the code for 49+>%?
e. None of these
Page 1035 of 1334
Subscribe The Xpress Video Course & Mock Test Package for Bank & Insurance Exams
If there are any suggestions/ errors in our PDFs Feel Free to contact us via this email: admin@exampundit.in
Ultra Practice Bundle PDF
SBI Clerk/ RBI Assist. Mains – Reasoning
11) What is the code for I#>O$^?

Direction 11-15: In each question below is given a a. P9RFW6


group of numbers / symbols followed by five
combinations of letter codes numbered (1), (2), (3), (4) b. PRF9W6
and (5). You have to find out which of the c. RPF96W
combinations correctly represents the group of
numbers / symbols based on the following coding d. PR9FW6
system and the steps given and mark the number of
e. PRF96W
that combination as your answer:
12) What is the code for U@E+%?
Lette * & + A @ I # > O $ ^ U = E %
rs / a. KHD53
Symb
ols b. KD35H
Lette S N H 1 K 9 P R 6 F W _ M 5 D c. KDH59
rs _
Code d. K5HD9
Steps to be followed as per the given order: e. None of these
(1) The vowel which comes first in English alphabet 13) What is the code for A#IO^=?
series will place at the extreme left end and the vowel
which come last in English alphabet series will placed at a. 1PM9W6
the extreme right end.
b. 1PWM96
(2) In series obtained after the step one if a vowel whose
c. 1WM9P6
numerical position is a perfect square in English alphabet
series is immediately followed by symbol than that d. 196WM
vowel should place immediately right to the symbol
placed third to the right of the element placed at the e. None of these
extreme left end after step one. 14) What is the code for +U>*EO?
(3) In the series obtained after the step two if a vowel a. H3R5S6
whose numerical position is a prime number is followed
by symbol then the position of that will be changed to the b. HR6S53
immediate left of the vowel placed at extreme right end
c. HRS563
The new series which will be formed by following the
d. HR5S63
above steps will be then coded according to the table
values given above. e. None of these
Page 1036 of 1334
Subscribe The Xpress Video Course & Mock Test Package for Bank & Insurance Exams
If there are any suggestions/ errors in our PDFs Feel Free to contact us via this email: admin@exampundit.in
Ultra Practice Bundle PDF
SBI Clerk/ RBI Assist. Mains – Reasoning
15) What is the code for &+A@I%? b. PCUFG

a. NH1KD9 c. PDUGG

b. 1NH9DK d. PCMGG

c. NH1K9D e. None of these

d. 1NHK9D 17) What is the code for 5#6+23?

e. None of these a. TVBBXR

16-20) In each question below is given a group of b. TVNBXR


numbers / symbols followed by five combinations of
letter codes numbered (1), (2), (3), (4) and (5). You c. TVVBCR
have to find out which of the combinations correctly d. TVVNCR
represents the group of numbers / symbols based on
the following coding system and the conditions and e. None of these
mark the number of that combination as your
18) What is the code for 3*5@#&?
answer:
a. R5SOVG
Number /
5 7 6 3 @ # + * 8 % 1 $ & 4 2
Symbols b. RSOOVG
Letter c. RSTOVG
T A N R O V B S M P C U G L X
Code
d. RSTTVG
Conditions:
e. None of these
(1) If the third element is a prime number and the last
element is a symbol then that prime number is to be used 19) What is the code for $@+5#*?
as the code for the symbol.
a. OUTBVS
(2) If there are no odd numbers then the codes of the
third and the fourth elements have to be interchanged. b. UBOTSV
(3) If an even number is immediately preceded by c. UOBTVS
symbols then the even number is to be coded as its next
number/symbol. d. OUSTBV

16) What is the code for %1$8&? e. None of these

a. PCIGG 20) What is the code for 2#837+?

a. VXAMMB
Page 1037 of 1334
Subscribe The Xpress Video Course & Mock Test Package for Bank & Insurance Exams
If there are any suggestions/ errors in our PDFs Feel Free to contact us via this email: admin@exampundit.in
Ultra Practice Bundle PDF
SBI Clerk/ RBI Assist. Mains – Reasoning
b. VXMBAB c. $88!@$88

c. XBVMBA d. !88&@$88

d. XVBMAB e. None of these

e. None of these 22) What can be the code of the word “between”?

21-25) In each question below is given a group of a. 3#!5##=


numbers / symbols followed by five combinations of
letter codes numbered (1), (2), (3), (4) and (5). You b. 3#!#5#=
have to find out which of the combinations correctly c. 3#!##5=
represents the group of numbers / symbols based on
the following coding system and the conditions and d. 3#!5=#5
mark the number of that combination as your
e. None of these
answer:
23) What can be the code of the word “superwoman”?
Letter o u a s f e r n p h c b t w
m a. @%*#857+&=
Coded
Symbol/Number 7 % & @ $ # 8 = + * < > 3 ! 5 b. @%#*857+&=

Conditions: c. @%#*758+&=
(1) If the seventh alphabet is a vowel and the fourth d. @%#*758&+=
alphabet is a consonant then all the consonants should be
changed to next vowel and rest alphabet should be coded e. None of these
as the given symbol

(2) If there are more than five consonants then the codes
of the third and the fourth letter have to be interchanged. 24) What can be the code of the word “Counterman”?

(3) If second and last letter is consonant then the code of a. >7=!%#8+&=
vowel present in the word will be changed to code of last
consonant b. >7=%!#8+&=

(Note: If more than one condition apply then conditions c. >7%=!#8+&=


will be apply one by one) d. >=%7!#8+&=
21) What is the code for ‘transfer’? e. None of these
a. @88!@$88

b. #88!@$88 25) What can be the code of the word “Perfume”?


Page 1038 of 1334
Subscribe The Xpress Video Course & Mock Test Package for Bank & Insurance Exams
If there are any suggestions/ errors in our PDFs Feel Free to contact us via this email: admin@exampundit.in
Ultra Practice Bundle PDF
SBI Clerk/ RBI Assist. Mains – Reasoning
a. *#8*%*# b. %@!2

b. *#8%%*# c. !%2@

c. *8#*%*# d. %!2×

d. *#**%*# e. %!2%

e. None of these 27) What is the code for EKUY?

26-30) In each question below is given a group of a. ×22%


numbers / symbols followed by five combinations of
letter codes numbered (1), (2), (3), (4) and (5). You b. @22×
have to find out which of the combinations correctly c. ×!!%
represents the group of numbers / symbols based on
the following coding system and the conditions and d. ×2!%
mark the number of that combination as your
e. None of these
answer:
28) What is the code for SKETJ?
Letters J T I
K S Y U O E A a. 2!××2
Codes % % b. 2!@@2
! 2 X @ ! 2 X @
c. @2!!×

Conditions: d. 2@!@2
i)If the first element is a vowel then the codes of first and
e. !2×@!
last elements are to be interchanged.
ii)If the last element is vowel then code of last and first 29) What is the code for TSIA?
to be coded as the code of the last digit.
iii)If the first and last element is consonant then the a. @2%×
codes of both the element are to be coded as the code of
b. @%2×
the first element.
iv)If two elements with same codes are there in the c. @2%@
arrangement then both should be coded as the code of the
next(succeeding) elements given in the above series. d. 2%@×
Note- More than one condition may be applied. e. 2%@2
26) What is the code of IKST?

a. ×!2% 30) What is the code for KIOY?


Page 1039 of 1334
Subscribe The Xpress Video Course & Mock Test Package for Bank & Insurance Exams
If there are any suggestions/ errors in our PDFs Feel Free to contact us via this email: admin@exampundit.in
Ultra Practice Bundle PDF
SBI Clerk/ RBI Assist. Mains – Reasoning
a. !%2! c. ©@4© ©&2©

b. !2%! d. $@4© ©&2©

c. !2%@ e. None of these

d. !2@% 32) What can be the code of ‘ODD WEATHER’?

e. None of these a. *3@ ©#@86#©

31-35) In each question below is given a group of b. *2* ©#@86#©


numbers / symbols followed by five combinations of
letter codes numbered (1), (2), (3), (4) and (5). You c. *3* ©#@68#©
have to find out which of the combinations correctly d. *3* ©#@8#6©
represents the group of numbers / symbols based on
the following coding system and the conditions and e. None of these
mark the number of that combination as your
33) What can be the code of ‘OLIVE BOARD’?
answer:
a. &%11# ©&@6©
Letters A I E O U
b. &1%1# ©&6@©
Codes # $ % @ &
c. &1%1# ©@&6©
Rest alphabets of English alphabetical series should be
written as 1-8 (for ex- B is coded as 1, C-2………..K-8) d. &1%1# ©&@6©
and again those numbers get repeated (for ex- L-1, M-
2……..so on). e. None of these
Conditions:
34) What can be the code of ‘NEVER GIVEUP’?
(i) If both first and last letter of a word is consonant then
the codes of both the letters are coded as ‘©’. a. ©#1#© ©%1$#©

(ii) If first letter of a word is vowel and last letter is b. ©#1#© ©%#1$©
consonant then both are to be coded as *.
c. ©##1© ©%1#$©
(If the word does not satisfy the conditions given above
then the letters of that word are to be coded as per the d. ©#1#© ©%1#$©
directions given above) e. None of these
31) What can be the code of ‘CAPS LOCK’? 35) What can be the code of ‘KEYBOARD?
a. ©@4© ©2&© a. ©$41@6#©
b. ©4@© ©&2© b. ©$41@#6©
Page 1040 of 1334
Subscribe The Xpress Video Course & Mock Test Package for Bank & Insurance Exams
If there are any suggestions/ errors in our PDFs Feel Free to contact us via this email: admin@exampundit.in
Ultra Practice Bundle PDF
SBI Clerk/ RBI Assist. Mains – Reasoning
c. ©$41@©6# d. µ96@$5µ

d. ©$14@#6© e. None of these

e. None of these 37) What can be the code of RFKOSH?

36-40) In each question below is given a group of a. 7558$5


numbers / symbols followed by five combinations of
letter codes numbered (1), (2), (3), (4) and (5). You b. 758$55
have to find out which of the combinations correctly c. 57585$
represents the group of numbers / symbols based on
the following coding system and the conditions and d. 7585$5
mark the number of that combination as your
e. None of these
answer: Note: More than one condition may apply.
38) What can be the code of NIHONE?
Letter R G F A P Q U N E I K J S O H
a. $@$8$@
Digit/Sy # 2 7 µ % 3 & 9 1 @ 5 © 6 8 $
mbol b. $1@8$@
Conditions: c. $1$8$@
(i) If first letter is vowel and last letter is consonant then
both are coded with the code of the consonant. d. $1$7$@
(ii) If both the 2nd letter and the last letter is vowel, then
e. None of these
their codes are to be interchanged.
(iii) If the second letter is a consonant and the 2nd last 39) What can be the code of HSJEQP?
letter is a vowel, then both are to be coded as the code
for the vowel. a. 6©1@%©
(iv) If both 1st and fifth letter is consonant then both are
b. 6©1©$©
coded as the code of third letter.
(v) If only one condition is applied among the above c. 6©1©%6
given, then the code of first letter is interchanged with
code of second letter and third letter code interchanged d. 6©1©%©
with 4th letter and so on after that applied condition. e. None of these
36) What can be the code of ANSHIKA? 40) What can be the code of AROHIS?
a. µ96$5@µ a. 6@8$@6
b. µ96$5@$ b. 6#8$#6
c. µ9@6$5µ c. 6@6$#6
Page 1041 of 1334
Subscribe The Xpress Video Course & Mock Test Package for Bank & Insurance Exams
If there are any suggestions/ errors in our PDFs Feel Free to contact us via this email: admin@exampundit.in
Ultra Practice Bundle PDF
SBI Clerk/ RBI Assist. Mains – Reasoning
d. 6@8$#6 e. None of these

e. None of these

41-45) In each of the questions given below, a group of 42) What can be the code of SMZPQ?
letters is given followed by four combinations of
symbols/numbers (A), (B), (C) and (D). You have to find a. 1@387
out which of the four combinations correctly represents b. &3@8&
the group of letters based on the symbol/number codes
and the conditions given below. If none of the four c. 7@381
combinations represents the group of digits correctly,
d. &83@&
give (E) ‘None of these’ as the answer.
e. None of these
Letter K Z M Q A B S E D P I L X O C U
43) What can be the code of OPDCM?
Digit/S 2 3 @ 7 % # 1 6 0 8 5 ! 4 + $ 9
ymbol a. @08%3

b. @08^3
Conditions: c. @80^@
(a) If the first and fourth letter of the word is vowel, then d. 3%80+
both are coded as first letter.
e. None of these
(b) If the first letter is vowel and last letter is consonant,
then both are coded as last letter. 44) What can be the code of BELXU?

(c) If the both first and last letter of the word is vowel, a. #4!69
then the codes for both letters will be interchanged.
b. #6!49
(d) If the both first and last letter of the word are
c. 6#!46
consonant, then both are coded as &.
d. #6!94
41) What can be the code of ALXBC?
e. None of these
a. $!4#$
45) What can be the code of IBXCU?
b. %!4#$
a. 5#4^9
c. $!#4$
b. 94#^5
d. $4#!$
c. 59#4^
Page 1042 of 1334
Subscribe The Xpress Video Course & Mock Test Package for Bank & Insurance Exams
If there are any suggestions/ errors in our PDFs Feel Free to contact us via this email: admin@exampundit.in
Ultra Practice Bundle PDF
SBI Clerk/ RBI Assist. Mains – Reasoning
d. 9#4^5 d. 7==@=

e. None of these e. None of these

Direction 46-50: In each question below is given a 47) What is the code for POWERFUL?
group of numbers / symbols followed by five
combinations of letter codes numbered (1), (2), (3), (4) a. @955%>19
and (5). You have to find out which of the b. 5955%>19
combinations correctly represents the group of
numbers / symbols based on the following coding c. 5955>%19
system and the steps given and mark the number of
d. $955>%1@
that combination as your answer:
e. None of these
Numb # 7 3 & * + = @ 5 9 % $ 1 ! >
ers / 48) What is the code for PARTY?
Symb
ols a. $7>&*
Letter Z A I T Y W H L E O F P U C R b. 7$>&*
s
Code c. $7&&*
Conditions d. $7&>*
1) The letters which are immediately followed and e. None of these
immediately preceded by a Vowel are coded as code of
the vowel which is immediately followed by the letter. 49) What is the code for WHITE?

2) The words which contains more than one vowel their a. +3=55
first and last letter should be coded as code of the 1st and
b. +=355
2nd vowel according to English alphabetical series
respectively c. 5==5=

3) If the third and last letter of the given word is vowel, d. 5=353
then both are coded as code of Second letter.
e. None of these
46) What is the code for WHALE?
50) What is the code for LUCIFER?
a. +7=@5
a. 5133555
b. +=7@5
b. 5133353
c. 7=7@5
c. 5135553
Page 1043 of 1334
Subscribe The Xpress Video Course & Mock Test Package for Bank & Insurance Exams
If there are any suggestions/ errors in our PDFs Feel Free to contact us via this email: admin@exampundit.in
Ultra Practice Bundle PDF
SBI Clerk/ RBI Assist. Mains – Reasoning
d. 5133553 e. None of these

Coding Decoding Number Symbol-Based Condition – Answer and


Explanation

Solution 1 Solution 4
(i) Descending order arrangement of numbers (i) Descending order arrangement of numbers ^8(*64).
74%3@#1. (ii) Interchanging places of number followed by the
(ii) Interchanging places of number followed by the symbol ^(8*6)4.
symbol 7%4@3#1. (iii) No number is followed by another number;
(iii) No number is followed by another number; therefore, the series obtained is ^(8*6)4.
therefore, the series obtained is 7%4@3#1. Hence, ^(8*6)4 will be coded as ZWLAPGX.
Hence, 7%4@3#1 will be coded as FRXBOYH. Solution 5
Solution 2 (i) Descending order arrangement of numbers 5*32%1@
(i) Descending order arrangement of numbers 3@)2(*1. (ii) Interchanging places of number followed by the
(ii) Interchanging places of number followed by the symbol *53%2@1
symbol @3)(2*1. (iii) one number is followed by another number;
(iii) No number is followed by another number; therefore, the series obtained is *8%2@1
therefore, the series obtained is @3)(2*1. Hence, *8%2@1 will be coded as ALRDBH
Hence, @3)(2*1 will be coded as BOGWDAH. Solution 6
Solution 3 (i) Ascending order arrangement of numbers 26*7@
(i) Descending order arrangement of numbers 86^5%#@. (ii) Interchanging places of number preceded by the
(ii) Interchanging places of number followed by the symbol 267*@
symbol 8^6%5#@. (iii) No odd number is followed by even number,
(iii) No number is followed by another number; therefore, the series obtained is 267*@
therefore, the series obtained is 8^6%5#@. Hence, 267*@ will be coded as PFMON
Hence, 8^6%5#@ will be coded as LZPRTYB.
Solution 7

Page 1044 of 1334


Subscribe The Xpress Video Course & Mock Test Package for Bank & Insurance Exams
If there are any suggestions/ errors in our PDFs Feel Free to contact us via this email: admin@exampundit.in
Ultra Practice Bundle PDF
SBI Clerk/ RBI Assist. Mains – Reasoning
(i) Ascending order arrangement of numbers 49+>% placed third to the right of the element placed at the
(ii) Interchanging places of number preceded by the extreme left end.
symbol 49+>% As I is the vowel whose numerical position is a perfect
(iii) No odd number is followed by even number, square.
therefore, the series obtained is 49+>% I#>$^O #>$I^O
Hence, 49+>%will be coded as GPAUW. (iii) After step-2 if a vowel whose numerical position is a
Solution 8 prime number is followed by symbol then the position of
(i) Ascending order arrangement of numbers 46&*8 that vowel will be changed to the immediate left of the
(ii) Interchanging places of number preceded by the vowel placed at extreme right end.
symbol 46&8* As in the series we got after step -2 has no vowels whose
(iii) No odd number is followed by even number, numerical position is a prime number so no change will
therefore, the series obtained is 46&8* occur in the series after step -2
Hence, 46&8*will be coded as GFIQO Now, the new series which will be formed by following
Solution 9 the above steps will be then coded according to the table
(i) Ascending order arrangement of numbers +49<@ values given
(ii) Interchanging places of number preceded by the Hence, #>$I^O will be coded as PRF9W6
symbol 4+9<@.
(iii) No odd number is followed by even number,
therefore, the series obtained is 4+9<@. Solution 12
Hence, 4+9<@.will be coded as GAPHN (i) Vowel which comes first in English alphabet series
will place at the extreme left end and the vowel which
Solution 10 come last will placed at the extreme right end.
(i) Ascending order arrangement of numbers 26$%8 U@E+% E@+%U
(ii) Interchanging places of number preceded by the (ii) After step–1 if a vowel whose numerical position is a
symbol 26$8% perfect square is immediately followed by symbol than
(iii) No odd number is followed by even number, that vowel should place immediately right to the element
therefore, the series obtained is 26$8% placed third to the right of the element placed at the
Hence, 26$8% will be coded as PFCQW extreme left end.
As there will be no change as no element is present
Solution 11 whose numerical position is a perfect square.
(i) Vowel which comes first in English alphabet series E@+%U
will place at the extreme left end and the vowel which (iii) After step-2 if a vowel whose numerical position is a
come last will placed at the extreme right end. prime number is followed by symbol then the position of
I#>O$^ I#>$^O that vowel will be changed to the immediate left of the
(ii) After step–1 if a vowel whose numerical position is a vowel placed at extreme right end.
perfect square is immediately followed by symbol than As in the series we got after step -2 has one vowel whose
that vowel should place immediately right to the element numerical position is a prime number.
@+%EU
Page 1045 of 1334
Subscribe The Xpress Video Course & Mock Test Package for Bank & Insurance Exams
If there are any suggestions/ errors in our PDFs Feel Free to contact us via this email: admin@exampundit.in
Ultra Practice Bundle PDF
SBI Clerk/ RBI Assist. Mains – Reasoning
Now, the new series which will be formed by following +U>*EO E+>*OU
the above steps will be then coded according to the table (ii) After step–1 if a vowel whose numerical position is a
values given perfect square is immediately followed by symbol than
The code of U is not given but if we see the pattern of that vowel should place immediately right to the element
vowels, they are coded as the sum of digits of their placed third to the right of the element placed at the
numerical position. So U is coded as 21= 2+1= 3 extreme left end.
Hence, U@E+% will be coded as KHD53 As there will be no change as no element is present
whose numerical position is a perfect square
Solution 13 E+>*OU
(i) Vowel which comes first in English alphabet series (iii) After step-2 if a vowel whose numerical position is a
will place at the extreme left end and the vowel which prime number is followed by symbol then the position of
come last will placed at the extreme right end. that vowel will be changed to the immediate left of the
A#IO^= A#I^=O vowel placed at extreme right end.
(ii) After step–1 if a vowel whose numerical position is a As in the series we got after step -2 has one vowel whose
perfect square is immediately followed by symbol than numerical position is a prime number i.e. E
that vowel should place immediately right to the element E+>*OU +>*EOU
placed third to the right of the element placed at the Now, the new series which will be formed by following
extreme left end. the above steps will be then coded according to the table
As I is the vowel whose numerical position is a perfect values given
square. The code of U is not given but if we see the pattern of
A#I^=O A#^=IO vowels, they are coded as the sum of digits of their
(iii) After step-2 if a vowel whose numerical position is a numerical position. So U is coded as 21= 2+1= 3
prime number is followed by symbol then the position of Hence, +>*EOU will be coded as HRS563
that vowel will be changed to the immediate left of the
vowel placed at extreme right end. Solution 15
As in the series we got after step -2 has no vowel whose (i) Vowel which comes first in English alphabet series
numerical position is a prime number. will place at the extreme left end and the vowel which
A#^=IO come last will placed at the extreme right end.
Now, the new series which will be formed by following &+A@I% A&+@%I
the above steps will be then coded according to the table (ii) After step–1 if a vowel whose numerical position is a
values given perfect square is immediately followed by symbol than
Hence, A#IO^= will be coded as 1PWM96 that vowel should place immediately right to the element
placed third to the right of the element placed at the
extreme left end.
Solution 14 As I is the vowel whose numerical position is a perfect
(i) Vowel which comes first in English alphabet series square.
will place at the extreme left end and the vowel which A&+@%I A&+@I%
come last will placed at the extreme right end.
Page 1046 of 1334
Subscribe The Xpress Video Course & Mock Test Package for Bank & Insurance Exams
If there are any suggestions/ errors in our PDFs Feel Free to contact us via this email: admin@exampundit.in
Ultra Practice Bundle PDF
SBI Clerk/ RBI Assist. Mains – Reasoning
(iii) After step-2 if a vowel whose numerical position is a Two odd numbers are present in the series
prime number is followed by symbol then the position of c) If an even number is immediately preceded by
that vowel will be changed to the immediate left of the symbols then the even number is to be coded as its next
vowel placed at extreme right end. number/symbol.
As in the series we got after step -2 has no vowel whose Hence 6 is preceded by symbol so 6 will be coded as B
numerical position is a prime number Code for 5#6+23 is TVBBXR
Now, the new series which will be formed by following
the above steps will be then coded according to the table
values given Solution 18
Hence, A&+@I% will be coded as 1NHK9D a) If Third element is a prime number and the last
element is a symbol than prime number is to be used as
Solution 16 the code for the symbol.
a) If Third element is a prime number and the last 3*5@#&
element is a symbol than prime number is to be used as There are two prime numbers and the third element is a
the code for the symbol. prime number.
%1$8& So 5 is coded as code for @ i.e. O
There are no prime numbers b) If there are no odd numbers then the codes of the third
b) If there are no odd numbers then the codes of the third and the fourth elements have to be interchanged.
and the fourth elements have to be interchanged. 3*O@#&
%1$8& One odd number is present in the series
1 is a odd number present in the series c) If an even number is immediately preceded by
c) If an even number is immediately preceded by symbols then the even number is to be coded as its next
symbols then the even number is to be coded as its next number/symbol.
number/symbol. No even number is present
Hence 8 is preceded by symbol so 8 will be coded as G Code for 3*5@#& is RSOOVG
Code for %1$8& is PCUGG

Solution 19
Solution 17 a) If Third element is a prime number and the last
a) If Third element is a prime number and the last element is a symbol than prime number is to be used as
element is a symbol than prime number is to be used as the code for the symbol.
the code for the symbol. $@+5#*
5#6+23 There is one prime numbers and but the third element is
There are three prime numbers but not the third element not a prime number.
is a prime number. b) If there are no odd numbers then the codes of the third
b) If there are no odd numbers then the codes of the third and the fourth elements have to be interchanged.
and the fourth elements have to be interchanged. $@+5#*
5#6+23 One odd number is present in the series
Page 1047 of 1334
Subscribe The Xpress Video Course & Mock Test Package for Bank & Insurance Exams
If there are any suggestions/ errors in our PDFs Feel Free to contact us via this email: admin@exampundit.in
Ultra Practice Bundle PDF
SBI Clerk/ RBI Assist. Mains – Reasoning
c) If an even number is immediately preceded by Hence, this condition follows
symbols then the even number is to be coded as its next transfer’ - !88&@$88
number/symbol. Solution 22
No even number is present (1) If the seventh alphabet is a vowel and the fourth
Code for $@+5#* is UOBTVS alphabet is a consonant then codes of both will be same
as code of 1st letter of the given word
Solution 20 Hence, this condition does not follow.
a) If Third element is a prime number and the last b) If there are more than five consonants then the codes
element is a symbol than prime number is to be used as of the third and the fourth letter have to be interchanged.
the code for the symbol. Hence, this condition does not follow
2#387+ c) If second and last letter is consonant then the code of
There is three prime numbers and the third element is a vowel present in the word will be changed to code of last
prime number. consonant
So 3 is coded as the code of + Hence, this condition does not follow
b) If there are no odd numbers then the codes of the third Therefore the code be will same as code given in table.
and the fourth elements have to be interchanged. between – 3#!5##=
2#387+ Solution 23
Two odd numbers is present in the series (1) If the seventh alphabet is a vowel and the fourth
c) If an even number is immediately preceded by alphabet is a consonant then codes of both will be same
symbols then the even number is to be coded as its next as code of 1st letter of the given word
number/symbol. Hence, this condition does not follow
Even number is not preceded by symbol b) If there are more than six consonants then the codes of
Code for 2#387+is XVBMAB the third and the fourth letter have to be interchanged.
Hence, this condition follows
Solution 21 “superwoman” - @%#*857+&=
(1) If the seventh alphabet is a vowel and the fourth c) If second and last letter is consonant then the code of
alphabet is a consonant then codes of both will be same vowel present in the word will be changed to code of last
as code of 1st letter of the given word consonant
Hence, this condition follows Hence, this condition does not follow
Therefore the code be will same as code given in table.
‘transfer’ - !8&!@$!8 superwoman” - @%#*857+&=
b) If there are more than five consonants then the codes Solution 24
of the third and the fourth letter have to be interchanged. (1) If the seventh alphabet is a vowel and the fourth
Hence, this condition follows alphabet is a consonant then codes of both will be same
transfer’ - !8!&@$!8 as code of 1st letter of the given word
c) If second and last letter is consonant then the code of Hence, this condition does not follow.
vowel present in the word will be changed to code of last b) If there are more than six consonants then the codes of
consonant the third and the fourth letter have to be interchanged.
Page 1048 of 1334
Subscribe The Xpress Video Course & Mock Test Package for Bank & Insurance Exams
If there are any suggestions/ errors in our PDFs Feel Free to contact us via this email: admin@exampundit.in
Ultra Practice Bundle PDF
SBI Clerk/ RBI Assist. Mains – Reasoning
Hence, this condition follows This condition does not follows
Counterman - >7=%!#8+&= Hence, Code of IKST - ×!2%
c) If second and last letter is consonant then the code of Solution 27
vowel present in the word will be changed to code of last Conditions:
consonant i)If the first element is a vowel then the codes of first and
Hence, this condition does not follow last elements are to be interchanged.
Hence, ‘Counterman’ is coded as - >7=%!#8+&= This condition follows
Solution 25 EKUY- @!!×
(1) If the seventh alphabet is a vowel and the fourth ii)If the last element is vowel then code of last and first
alphabet is a consonant then codes of both will be same to be coded as the code of the last digit.
as code of 1st letter of the given word This condition does not follows
iii)If the first and last element is consonant then the
Hence, this condition follows. codes of both the element are to be coded as the code of
“Perfume” - *#8*%*# the first element.
b) If there are more than six consonants then the codes of This condition does not follows
the third and the fourth letter have to be interchanged. iv)If two elements with same codes are there in the
Hence, this condition does not follow arrangement then both should be coded as the code of the
c) If second and last letter is consonant then the code of next(succeeding) elements given in the above series.
vowel present in the word will be changed to code of last This condition follows
consonant Hence, Code of IKST - @22×
Hence, this condition does not follow Solution 28
Therefore the code of “Perfume” will be - *#8*%*# Conditions:
Solution 26 i)If the first element is a vowel then the codes of first and
Conditions: last elements are to be interchanged.
i)If the first element is a vowel then the codes of first and This condition does not follows
last elements are to be interchanged. ii)If the last element is vowel then code of last and first
This condition follows to be coded as the code of the last digit.
IKST - ×!2% This condition does not follows
ii)If the last element is vowel then code of last and first iii)If the first and last element is consonant then the
to be coded as the code of the last digit. codes of both the element are to be coded as the code of
This condition does not follows the first element.
iii)If the first and last element is consonant then the This condition follows
codes of both the element are to be coded as the code of Code of SKETJ - 2!EX%
the first element. iv)If two elements with same codes are there in the
This condition does not follows arrangement then both should be coded as the code of the
iv)If two elements with same codes are there in the next(succeeding) elements given in the above series.
arrangement then both should be coded as the code of the This condition follows
next(succeeding) elements given in the above series. Hence, Code of SKETJ - 2!@@2
Page 1049 of 1334
Subscribe The Xpress Video Course & Mock Test Package for Bank & Insurance Exams
If there are any suggestions/ errors in our PDFs Feel Free to contact us via this email: admin@exampundit.in
Ultra Practice Bundle PDF
SBI Clerk/ RBI Assist. Mains – Reasoning
Solution 29 F-4, G-5, H-6, J-7, K-8, L-1, M-2, N-3, P-4, Q-5, R-6, S-
Conditions: 7, T-8, V-1, W-2, X-3, Y-4, Z-5.
i) If the first element is a vowel then the codes of first Letters A I E O U
and last elements are to be interchanged. Codes # $ % @ &
This condition does not follows
ii) If the last element is vowel then code of last and first (i) If both first and last letter of a word is consonant then
to be coded as the code of the last digit. the codes of both the letters are coded as ‘©’.
This condition follows This condition follow
TSIA - @2%@ ‘‘CAPS LOCK’ is- ‘©@4© ©&2©’
iii) If the first and last element is consonant then the (ii) If first letter of a word is vowel and last letter is
codes of both the element are to be coded as the code of consonant then both are to be coded as *
the first element. This condition does not follow
This condition does not follows iv) If two elements with Hence, the code for ‘CAPS LOCK’ is- ‘©@4© ©&2©’
same codes are there in the arrangement then both should Solution 32
be coded as the code of the next(succeeding) elements In this new pattern coding decoding each letter, except
given in the above series. vowel, is assigned a number from 1-8 So, B-1, C-2, D-3,
This condition does not follow F-4, G-5, H-6, J-7, K-8, L-1, M-2, N-3, P-4, Q-5, R-6, S-
Solution 30 7, T-8, V-1, W-2, X-3, Y-4, Z-5.
Conditions: Letters A I E O U
i) If the first element is a vowel then the codes of first
Codes # $ % @ &
and last elements are to be interchanged.
This condition does not follows
(i) If both first and last letter of a word is consonant then
ii) If the last element is vowel then code of last and first
the codes of both the letters are coded as ‘©’.
to be coded as the code of the last digit.
This condition follow
This condition does not follows
‘ODD WEATHER’ is- ‘@33 ©#@86#©’
(ii) If first letter of a word is vowel and last letter is
iii) If the first and last element is consonant then the
consonant then both are to be coded as *
codes of both the element are to be coded as the code of
This condition follow
the first element.
Hence, the code for ‘ODD WEATHER’ is - ‘*3*
This condition follows
©#@86#©’
KIOY - !%2!
Solution 33
iv) If two elements with same codes are there in the
In this new pattern coding decoding each letter, except
arrangement then both should be coded as the code of the
vowel, is assigned a number from 1-8 So, B-1, C-2, D-3,
next(succeeding) elements given in the above series.
F-4, G-5, H-6, J-7, K-8, L-1, M-2, N-3, P-4, Q-5, R-6, S-
This condition does not follow
7, T-8, V-1, W-2, X-3, Y-4, Z-5.
Solution 31
Letters A I E O U
In this new pattern coding decoding each letter, except
vowel, is assigned a number from 1-8 So, B-1, C-2, D-3, Codes # $ % @ &

Page 1050 of 1334


Subscribe The Xpress Video Course & Mock Test Package for Bank & Insurance Exams
If there are any suggestions/ errors in our PDFs Feel Free to contact us via this email: admin@exampundit.in
Ultra Practice Bundle PDF
SBI Clerk/ RBI Assist. Mains – Reasoning
‘KEYBOARD – ©$41@#6©
(i) If both first and last letter of a word is consonant then (ii) If first letter of a word is vowel and last letter is
the codes of both the letters are coded as ‘©’. consonant then both are to be coded as *
This condition follow This condition does not follow
OLIVE BOARD’ - ‘&1%1# ©&@6©’. Hence, the code for KEYBOARD is – ©$41@#6©
(ii) If first letter of a word is vowel and last letter is
consonant then both are to be coded as * Solution 36
This condition does not follow (i) If first letter is vowel and last letter is consonant then
Hence, the code for OLIVE BOARD’ is - ‘&1%1# both are coded with the code of the consonant.
©&@6©’. This condition does not follow
Solution 34
In this new pattern coding decoding each letter, except (ii) If both the 2nd letter and the last letter is vowel, then
vowel, is assigned a number from 1-8 So, B-1, C-2, D-3, their codes are to be interchanged.
F-4, G-5, H-6, J-7, K-8, L-1, M-2, N-3, P-4, Q-5, R-6, S- This condition does not follow
7, T-8, V-1, W-2, X-3, Y-4, Z-5.
Letters A I E O U (iii) If the second letter is a consonant and the 2nd last
Codes # $ % @ & letter is a vowel, then both are to be coded as the code
for the vowel.
(i) If both first and last letter of a word is consonant then This condition does not follow
the codes of both the letters are coded as ‘©’.
This condition follow (iv) If both 1st and fifth letter is consonant then both are
‘NEVER GIVEUP’ is– ‘©#1#© ©%1#$©’ coded as the code of third letter.
(ii) If first letter of a word is vowel and last letter is This condition does not follow
consonant then both are to be coded as *.
This condition does not follow (v) If only one condition is applied among the above
Hence, the code for ‘NEVER GIVEUP’ is– ‘©#1#© given, then the code of first letter is interchanged with
©%1#$©’ code of second letter and third letter code interchanged
Solution 35 with 4th letter and so on after that applied condition.
In this new pattern coding decoding each letter, except This condition does not follow
vowel, is assigned a number from 1-8 So, B-1, C-2, D-3, Hence, the code for ANSHIKA will be µ96$@5µ
F-4, G-5, H-6, J-7, K-8, L-1, M-2, N-3, P-4, Q-5, R-6, S-
7, T-8, V-1, W-2, X-3, Y-4, Z-5. Solution 37
Letters A I E O U (i) If first letter is vowel and last letter is consonant then
both are coded with the code of the consonant.
Codes # $ % @ &
This condition does not follow

(i) If both first and last letter of a word is consonant then


(ii) If both the 2nd letter and the last letter is vowel, then
the codes of both the letters are coded as ‘©’.
their codes are to be interchanged.
This condition follow
Page 1051 of 1334
Subscribe The Xpress Video Course & Mock Test Package for Bank & Insurance Exams
If there are any suggestions/ errors in our PDFs Feel Free to contact us via this email: admin@exampundit.in
Ultra Practice Bundle PDF
SBI Clerk/ RBI Assist. Mains – Reasoning
This condition does not follow code of second letter and third letter code interchanged
with 4th letter and so on after that applied condition.
(iii) If the second letter is a consonant and the 2nd last This condition does not follow
letter is a vowel, then both are to be coded as the code Hence, the code for NIHONE will be $1$8$@
for the vowel.
This condition does not follow Solution 39
(i) If first letter is vowel and last letter is consonant then
(iv) If both 1st and fifth letter is consonant then both are both are coded with the code of the consonant.
coded as the code of third letter. This condition does not follow
This condition follow
RFKOSH- 57585$ (ii) If both the 2nd letter and the last letter is vowel, then
(v) If only one condition is applied among the above their codes are to be interchanged.
given, then the code of first letter is interchanged with This condition does not follow
code of second letter and third letter code interchanged
with 4th letter and so on after that applied condition. (iii) If the second letter is a consonant and the 2nd last
This condition follow letter is a vowel, then both are to be coded as the code
Hence, the code for RFKOSH will be 7585$5 for the vowel.
This condition does not follow
Solution 38
(i) If first letter is vowel and last letter is consonant then (iv) If both 1st and fifth letter is consonant then both are
both are coded with the code of the consonant. coded as the code of third letter.
This condition does not follow This condition follow
HSJEQP - ©6©9©%
(ii) If both the 2nd letter and the last letter is vowel, then (v) If only one condition is applied among the above
their codes are to be interchanged. given, then the code of first letter is interchanged with
This condition follow code of second letter and third letter code interchanged
NIHONE -91$89@ with 4th letter and so on after that applied condition.
(iii) If the second letter is a consonant and the 2nd last This condition follow
letter is a vowel, then both are to be coded as the code Hence, the code for HSJEQP will be 6©1©%©
for the vowel. Solution 40
This condition does not follow (i) If first letter is vowel and last letter is consonant then
both are coded with the code of the consonant.
(iv) If both 1st and fifth letter is consonant then both are This condition follow
coded as the code of third letter. AROHIS – 6#8$@6
This condition follow (ii) If both the 2nd letter and the last letter is vowel, then
NIHONE -$1$8$@ their codes are to be interchanged.
(v) If only one condition is applied among the above This condition does not follow
given, then the code of first letter is interchanged with
Page 1052 of 1334
Subscribe The Xpress Video Course & Mock Test Package for Bank & Insurance Exams
If there are any suggestions/ errors in our PDFs Feel Free to contact us via this email: admin@exampundit.in
Ultra Practice Bundle PDF
SBI Clerk/ RBI Assist. Mains – Reasoning
(c) If the both first and last letter of the word is vowel,
(iii) If the second letter is a consonant and the 2nd last then the codes for both letters will be interchanged.
letter is a vowel, then both are to be coded as the code This condition does not follow
for the vowel. (d) If the both first and last letter of the word are
This condition follow consonant, then both are coded as &.
AROHIS – 6@8$@6 This condition follow
(iv) If both 1st and fifth letter is consonant then both are Hence, the code for SMZPQ will be &@38&
coded as the code of third letter. Solution 43
This condition does not follow (a) If the first and fourth letter of the word is vowel, then
(v) If only one condition is applied among the above both are coded as first letter.
given, then the code of first letter is interchanged with This condition does not follow
code of second letter and third letter code interchanged (b) If the first letter is vowel and last letter is consonant,
with 4th letter and so on after that applied condition. then both are coded as last letter.
This condition does not follow This condition follow
Hence, the code for AROHIS will be 6@8$@6 OPDCM - @80^@
Solution 41 (c) If the both first and last letter of the word is vowel,
(a) If the first and fourth letter of the word is vowel, then then the codes for both letters will be interchanged.
both are coded as first letter. This condition does not follow
This condition does not follow (d) If the both first and last letter of the word are
(b) If the first letter is vowel and last letter is consonant, consonant, then both are coded as &.
then both are coded as last letter. This condition does not follow
This condition follow Hence, the code for OPDCM will be @80^@
ALXBC - $!4#$ Solution 44
(c) If the both first and last letter of the word is vowel, (a) If the first and fourth letter of the word is vowel, then
then the codes for both letters will be interchanged. both are coded as first letter.
This condition does not follow This condition does not follow
(d) If the both first and last letter of the word are (b) If the first letter is vowel and last letter is consonant,
consonant, then both are coded as &. then both are coded as last letter.
This condition does not follow This condition does not follow
Hence, the code for ALXBC will be $!4#$ (c) If the both first and last letter of the word is vowel,
Solution 42 then the codes for both letters will be interchanged.
(a) If the first and fourth letter of the word is vowel, then This condition does not follow
both are coded as first letter. (d) If the both first and last letter of the word are
This condition does not follow consonant, then both are coded as &.
(b) If the first letter is vowel and last letter is consonant, This condition does not follow
then both are coded as last letter. Hence, the code for BELXU will be #6!49
This condition does not follow Solution 45

Page 1053 of 1334


Subscribe The Xpress Video Course & Mock Test Package for Bank & Insurance Exams
If there are any suggestions/ errors in our PDFs Feel Free to contact us via this email: admin@exampundit.in
Ultra Practice Bundle PDF
SBI Clerk/ RBI Assist. Mains – Reasoning
(a) If the first and fourth letter of the word is vowel, then 2nd vowel according to English alphabetical series
both are coded as first letter. respectively
This condition does not follow This condition follow
(b) If the first letter is vowel and last letter is consonant, POWERFUL - 5955>%19
then both are coded as last letter. 3) If the third and last letter of the given word is vowel,
This condition does not follow then both are coded as code of Second letter.
(c) If the both first and last letter of the word is vowel, This condition does not follow
then the codes for both letters will be interchanged. Hence, the code for POWERFUL will be 5955>%19
This condition follow Solution 48
IBXCU - 9#4^5 1) The letters which are immediately followed and
(d) If the both first and last letter of the word are immediately preceded by a Vowel are coded as code of
consonant, then both are coded as &. the vowel which is immediately followed by the letter.
This condition does not follow This condition does not follow
Hence, the code for IBXCU will be 9#4^5 2) The words which contains more than one vowel their
Solution 46 first and last letter should be coded as code of the 1st and
1) The letters which are immediately followed and 2nd vowel according to English alphabetical series
immediately preceded by a Vowel are coded as code of respectively
the vowel which is immediately followed by the letter. This condition does not follow
This condition follow 3) If the third and last letter of the given word is vowel,
WHALE - +=7@5 then both are coded as code of Second letter.
2) The words which contains more than one vowel their This condition does not follow
first and last letter should be coded as code of the 1st and Hence, the code for PARTY will be $7>&*
2nd vowel according to English alphabetical series Solution 49
respectively 1) The letters which are immediately followed and
This condition follow immediately preceded by a Vowel are coded as code of
WHALE - 7=7@5 the vowel which is immediately followed by the letter.
3) If the third and last letter of the given word is vowel, This condition follow
then both are coded as code of Second letter. WHITE - +=355
This condition follow 2) The words which contains more than one vowel their
Hence, the code for WHALE will be 7==@= first and last letter should be coded as code of the 1st and
Solution 47 2nd vowel according to English alphabetical series
1) The letters which are immediately followed and respectively
immediately preceded by a Vowel are coded as code of This condition follow
the vowel which is immediately followed by the letter. WHITE - 5=353
This condition follow 3) If the third and last letter of the given word is vowel,
POWERFUL - $955>%1@ then both are coded as code of Second letter.
2) The words which contains more than one vowel their This condition follow
first and last letter should be coded as code of the 1st and Hence, the code for WHITE will be 5==5=
Page 1054 of 1334
Subscribe The Xpress Video Course & Mock Test Package for Bank & Insurance Exams
If there are any suggestions/ errors in our PDFs Feel Free to contact us via this email: admin@exampundit.in
Ultra Practice Bundle PDF
SBI Clerk/ RBI Assist. Mains – Reasoning
Solution 50 2nd vowel according to English alphabetical series
1) The letters which are immediately followed and respectively
immediately preceded by a Vowel are coded as code of This condition follow
the vowel which is immediately followed by the letter. LUCIFER - 5133553
This condition follow 3) If the third and last letter of the given word is vowel,
LUCIFER - @13355> then both are coded as code of Second letter.
2) The words which contains more than one vowel their This condition does not follow
first and last letter should be coded as code of the 1st and Hence, the code for LUCIFER will be 5133553

Download Seating Arrangement Practice Questions PDF


Get More Reasoning Practice Questions PDF

Coding & Decoding New Pattern


Directions (1-5): Study the following information “code society among series” is coded as “3U, 18F, 15O,
carefully and answer the questions given below: 4E”

In a code language, “number synchronous you lenovo” is coded as “14W,


14V, 21P, 13F”
“postal watch english paint” is coded as “7T, 9O, 20D,
19B” “left north peru house” is coded as “6G, 18S, 21T, 18U”

1 How is the word ‘emotion’ coded?


Page 1055 of 1334
Subscribe The Xpress Video Course & Mock Test Package for Bank & Insurance Exams
If there are any suggestions/ errors in our PDFs Feel Free to contact us via this email: admin@exampundit.in
Ultra Practice Bundle PDF
SBI Clerk/ RBI Assist. Mains – Reasoning
a. 14P a. 5F 19F

b. 15O b. 6G 18F

c. 16P c. 6H 18H

d. 15P d. 6O 19S

e. None of these e. None of these

2) 18S’ is the code for which among the following? Directions (6-10): Study the following information
carefully and answer the questions given below:
a. care
In a code language,
b. cope
“honour public clear vendor” is coded as “4B, 3N, 4N,
c. drain 3E”
d. duke “watch wrapped optimist question” is coded as “5A, 4E,
e. None of these 4T, 5T”

3) Which among the following is correctly matched? “alphabets used Microsoft commerce” is coded as “2E,
6C, 6P, 5M”
a. capture – 17S
“page curiously women” is coded as “5R, 3M, 2G”
b. cricket – 10F
6) How is the word ‘Plastics’ coded?
c. hopeful – 15G
a. 5S
d. table – 2M
b. 5N
e. None of these
c. 5A
4) How is the word ‘every’ coded?
d. 6A
a. 5R
e. None of these
b. 22F
7) What is the code for ‘2A’?
c. 5S
a. Neat
d. 6R
b. Plant
e. None of these
c. Sister
5) How is the phrase “soft corner” coded?
Page 1056 of 1334
Subscribe The Xpress Video Course & Mock Test Package for Bank & Insurance Exams
If there are any suggestions/ errors in our PDFs Feel Free to contact us via this email: admin@exampundit.in
Ultra Practice Bundle PDF
SBI Clerk/ RBI Assist. Mains – Reasoning
d. Please ‘ROYAL BORE PACE AIMED’ is coded as ‘2G18 1F5
18N1 16G3’
e. None of these
‘OLIVER CLAIM READING BOUND’ is coded as
8) Which among the following is correctly matched? ‘2F14 3O9 18I14 15T5’
a. move – 3O ‘FLIGHT HANDLE TOAST STAIRS’ is coded as
b. arrogant – 5R ‘20V19 6V8 19U18 8G12’

c. pencil – 3N ‘TRINK ZONED LIGHT OFFERS’ is coded as ‘26F5


12V8 20M14 15U18’
d. mobile – 3C
11) What is the code for ‘BROAD’?
e. None of these
a. 6F4
9) What is the code for ‘LIGHT’?
b. 2F1
a. 4K
c. 1B2
b. 5J
d. 15D3
c. 4G
e. None of these
d. 4O
12) ‘PROPER’ is coded as?
e. None on these
a. 16O19
10) What is the code for 3O?
b. 2T19
a. finger
c. 16T5
b. frooti
d. 15C20
c. four
e. None of these
d. race
13) ‘SEPARATE’ is coded as?
e. None of these
a. 15I1
Directions (11-15): Study the following information
carefully and answer the questions given below: b. 19V5

In a certain language, c. 1A20

d. 19G20

Page 1057 of 1334


Subscribe The Xpress Video Course & Mock Test Package for Bank & Insurance Exams
If there are any suggestions/ errors in our PDFs Feel Free to contact us via this email: admin@exampundit.in
Ultra Practice Bundle PDF
SBI Clerk/ RBI Assist. Mains – Reasoning
e. None of these b. P24M

14) What is the code for ‘BALANCE’? c. Z12S

a. 6G4 d. Q16M

b. 2G3 e. None of these

c. 1B2 17) What is the code for the word ‘Origami’?

d. 5C3 a. D7S

e. None of these b. L9R

15) ‘PROTEST’ is coded as? c. E11T

a. 16O19 d. M12B

b. 12A19 e. None of these

c. 16V19 18) How is ‘smart student’ coded?

d. 15C20 a. J10G, R7H

e. None of these b. J11G, J7H

Directions (16-20): Study the following information c. H7W, H9S


carefully and answer the given questions.
d. H6W, H9S
In a certain code language,
e. None of these
“pages are blank” is coded as “K7J, Z5W, Y7S”
19) How is the word “Philosophy” coded as?
“chocolate tastes sweet” is coded as “X11Y, G8J, H7J”
a. K20B
“dress colour is pink” is coded as “W7X, X8Z, R4N,
K6S” b. K12M

“confidence and peace” is coded as “X12H, Z5S, K7H” c. L30B

“knowledge gives wisdom” is coded as “P11L, T7J, d. L20B


D8T” e. None of these
16) What is the code for the word ‘Ambivalent’? 20) What is the code for Peace?
a. Z24L a. X12H
Page 1058 of 1334
Subscribe The Xpress Video Course & Mock Test Package for Bank & Insurance Exams
If there are any suggestions/ errors in our PDFs Feel Free to contact us via this email: admin@exampundit.in
Ultra Practice Bundle PDF
SBI Clerk/ RBI Assist. Mains – Reasoning
b. Z5S e. *s6

c. K7H

d. L20B 23) How would the sentence ‘Atmaram was a waste’


be coded?
e. None of these
a. @s3 !a1 @m7 !e5
Directions (21-25): Study the information below and
answer the following question. b. !s3 @a1 !m7 @e5

In a certain code language, c. !s3 @a1 @m7 !e5

‘The rank wombat waves’ is written as ‘%e3 $k4 @t6 d. @s3 !a1 !m7 @e5
@s5’.
e. @s3 !a1 @m7 ie5
‘An alcoholic advertises rank’ is written as ‘!n2 !c9 !s10
$k4’. 24) How would the sentence ‘toast was toasted to
toaster’ be coded?
‘Another terrifying viewpoint sleeps’ is written as ‘!r7
%g10 *t9 (s6’. a. %t5 @s3 %d7 %o2 %r7

‘Nidhi sleeps at nine’ is written as ‘^i5 (s6 !t2 ^e4’. b. @t5 @s3 %d7 %o2 %r7

21) How would the word ‘an’ be coded? c. %t5 @s3 @d7 %o2 %r7

a. !n2 d. %t5 @s3 %d7 @o2 %r7

b. !a2 e. @t5 @s3 @d7 @o2 %r7

c. *n2 25) How would the sentence ‘Nidhi Sleeps Alcoholic’


be coded?
d. (n2
a. !i5 ^s6 (c9
e. ^a2
b. ^i5 !s6 (c9
22) How would the word ‘sleeps’ be coded?
c. ^i5 (s6 !c9
a. (t6
d. (i5 ^s6 !c9
b. (s6
e. ^i5 (s6 (c9
c. ^t6
Directions (26-30): Study the information below and
d. (e6 answer the following question.

Page 1059 of 1334


Subscribe The Xpress Video Course & Mock Test Package for Bank & Insurance Exams
If there are any suggestions/ errors in our PDFs Feel Free to contact us via this email: admin@exampundit.in
Ultra Practice Bundle PDF
SBI Clerk/ RBI Assist. Mains – Reasoning
In a certain code of language, d. Ability

‘one apple daily please’ is written as ‘A$ A@ B% B@’ e. None of these

‘doctor advised help better’ is written as ‘B% B& A^ 29) What is the code of ‘America Africa’?
B%’
a. A^ B%
‘almost all treat well’ is written as ‘A% B@ A$ B&’
b. A^ A%
‘pollute airs thus rivers’ is written as ‘B^ A& B& B%’
c. A@ A*
26) Which of the following is the code for ‘Rain Starts
Even’? d. B% B^

a. B& B% A& e. None of these

b. A* A@ A& 30) What is the code of “SCHOOL”?

c. B& B% A@ a. B$

d. B% B* A$ b. A&

e. None of these c. A^

27) The code ‘B% B&’ represents which of the d. B%


following words? e. None of these
a. Must Done Directions (31-35) Study the information below and
b. Patent Skip answer the following question.

c. Hunger Over In a code language,

d. Engine Start “larson pink pride green” is coded as “S25E, S25F,


J16O, B36P”
e. None of these
“pack trozen in hospital” is coded as “O4J, P64B, B16D,
28) The code ‘A%’ is represents which of the S36F”
following word?
“priority for flags hoping” is coded as “P9P, M25H,
a. Apple S64U, P36O”

b. Train “finance run fest process” is coded as “V9V, J49D,


S49T, F16T”
c. Orange
31) How is the word ‘prominent’ coded?
Page 1060 of 1334
Subscribe The Xpress Video Course & Mock Test Package for Bank & Insurance Exams
If there are any suggestions/ errors in our PDFs Feel Free to contact us via this email: admin@exampundit.in
Ultra Practice Bundle PDF
SBI Clerk/ RBI Assist. Mains – Reasoning
a. R81N 35) How is the phrase “green fruits” coded?

b. S64O a. R16F G25T

c. R36N b. H25O S25U

d. S81O c. S25F S36U

e. None of these. d. R64P N16K

32) ‘B16U’ is the code for which among the e. None of these
following?
Directions (36-40) Study the information below and
a. fate answer the following question.

b. card In a certain code of language,

c. feared ‘much needed helps required’ is written as ‘14@G 9%R


19$C 15&C’
d. next
‘climate lazy worker tests’ is written as ‘13@X 24&Q
e. None of these 4!D 21%R’
33) Which among the following is correctly matched? ‘awards given with today’ is written as ‘2&R 8%M
a. sight – J16I 24@G 21%X’

b. main – B16J ‘ready steady quiz decline’ is written as ‘19%X 5!D


18@Y 20&X’
c. source – P36C
36) How will the word ‘SCHOOL’ be coded as?
d. mate – B16F
a. 20!K
e. None of these.
b. 20@M
34) How is the word ‘scraped’ coded?
c. 20&N
a. D36F
d. 20&K
b. S49Q
e. None of these
c. T49F
37) How will ‘TRAIN LEFT STATION’ be coded as?
d. D49F
a. 21%M 12@S 19!M
e. None of these
b. 21&M 12!S 19!M
Page 1061 of 1334
Subscribe The Xpress Video Course & Mock Test Package for Bank & Insurance Exams
If there are any suggestions/ errors in our PDFs Feel Free to contact us via this email: admin@exampundit.in
Ultra Practice Bundle PDF
SBI Clerk/ RBI Assist. Mains – Reasoning
c. 21%M 13@S 20!M Directions (41-45) Study the information below and
answer the following question.
d. 21%M 13@S 19!M
In a code language,
e. None of these
“random angry values alphabetical” is coded as “F6, L2,
38) The code ‘3&C’ is coded from which of the F16, E19”
following word?
“physics mystery an bowl” is coded as “G19, D24, B2,
a. Beggar G4”
b. Behind “again question revive copy” is coded as “D17, F23,
c. Bihar E10, H16”

d. Besides “fifteen wrongly sole replay” is coded as “G6, D13, F2,


G13”
e. None of these
41) How is the word ‘society’ coded?
39) If code for the word ‘something better’ is ‘20#F
3&Q’ in the same language then what would be the a. G20
code of ‘knowledge gain’? b. F19
a. 12#D 8@M c. G21
b. 11#D 7@M d. T7
c. 12%D 6&M e. None of these
d. 10$G 6%M 42) ‘E20’ is the code for which among the following?
e. 12@M 8#H a. common
40) Which words are coded as ‘2$C 14&X’? b. kiosk
a. Assault and Youth c. filler
b. Rebel and Define d. fleask
c. Assigned and Memory e. None of these
d. Dress and Kites 43) Which among the following is correctly matched?
e. None of these a. daily – E11

b. margin - G10
Page 1062 of 1334
Subscribe The Xpress Video Course & Mock Test Package for Bank & Insurance Exams
If there are any suggestions/ errors in our PDFs Feel Free to contact us via this email: admin@exampundit.in
Ultra Practice Bundle PDF
SBI Clerk/ RBI Assist. Mains – Reasoning
c. single - F12 46) How is the word ‘Resident’ coded?

d. flicker – G6 a. F#

e. None of these b. E@

44) How is the word ‘weighty’ coded? c. E$

a. G21 d. F@

b. E23 e. None of these.

c. F24 47) Which word can be coded as ‘F$’?

d. G20 a. Movie

e. None of these b. Promo

45) How is the phrase “coming fast” coded? c. Demo

a. F13 D19 d. Prompt

b. E15 D20 e. Seating

c. F15 D20 48) Which among the following is correctly matched?

d. F16 S21 a. Shirt – I$

e. None of these b. Trousers – S$

Directions: (46-50) Study the information below and c. Maintain – B#


answer the following question.
d. Rose – P$
In a code language,
e. None of these
“Gujarat warm picnic founder” is coded as “V@, J$,
P@, B#” 49) What is the code for “bring document”?

“orange pretend slower right” is coded as “J#, S$, S@, a. S# P@


M$” b. P# S@
“hope summer search funny” is coded as “V$, F$, V#, c. S@ O@
P$”
d. S# P$
“rainy season pen smooth” is coded as “B$, N$, F#, F@”
e. None on these
Page 1063 of 1334
Subscribe The Xpress Video Course & Mock Test Package for Bank & Insurance Exams
If there are any suggestions/ errors in our PDFs Feel Free to contact us via this email: admin@exampundit.in
Ultra Practice Bundle PDF
SBI Clerk/ RBI Assist. Mains – Reasoning
50) What is the code for “summer” ? c. V#

a. V$ d. P$

b. F$ e. None of these

Coding & Decoding New Pattern – Answer and Explanation


Solution (1-5) The letter in each code is the 3rd letter of the word (from
the left end).
The number in the code is the value (in alphabetical
series) of the 3rd letter from the left end of each of the Example:
words.
‘honour’
The letter in the code is the immediate succeeding letter
(in alphabetical series) of 2nd last letter of each of the The given word has 3 consonants.
words. 3rd letter of the word (from the left end) is ‘N’.
Example: Therefore, the code for ‘honour’ is ‘3N’.
‘postal’ 6. D
3rd letter from the left end is S, whose value in 7. A
alphabetical series is ‘19’.
8. B
2nd last letter of the given word is A and its immediate
succeeding letter (in alphabetical series) is B. 9. C

Therefore, the code for ‘postal’ is ‘19B’ 10. B

1. D Solution (11-15)

2. A The coding language used to code the given sentences


should be decoded in three steps.
3. D
Step I: The first element of the code is the alphabetical
4. C position of 1st element of the word.
5. B Step II: The second element of the code is the 2nd
Solution (6-10) succeeding alphabet as per English Alphabetical order of
the last letter of the word.
The number in each code is the number of consonants in
the word. Step III: The third element is the alphabetic position of
the 2nd last element of the word.
Page 1064 of 1334
Subscribe The Xpress Video Course & Mock Test Package for Bank & Insurance Exams
If there are any suggestions/ errors in our PDFs Feel Free to contact us via this email: admin@exampundit.in
Ultra Practice Bundle PDF
SBI Clerk/ RBI Assist. Mains – Reasoning
For example, 19. B

‘ROYAL’ is coded as ‘18N1’ 20. C

First element R =18 Solution (21-25)

Last word + 2 = N Logic:

2nd last element A = 1 1) The first letter is a coded symbol of the first letter.

11. B 2) The second letter is the last letter of the word.

12. C 3) The third letter is the number of letters in the word.

13. D E.g. *y2 = my

14. B * represents m

15. C ‘y’ is last letter

Solution (16-20) 2 is number of letters

1. The first element coding: The reverse alphabet of the 21). A


first word.
22). B
2. The second element coding: Total number of letters in
the word + 2 23). B

3. The third element coding: The second last alphabet of 24). A


the word from the beginning is taken and + 5 forward 25). C
alphabet is taken.
Solutions (26-30)
For Example
For the first symbol – if the given word starts with vowel
‘pages’ will be coded as follows, then write A and if the word starts with consonant then
Pages → P (reverse of K), 7 (total number of letters + 2), write B.
J (E + 5) For the second symbol- If total number of alphabets is 3
Hence, 'Pages' is coded as “K7J”. then use symbol - $

16. C If total number of alphabets are 4 use symbol - &

17. B If total number of alphabets are 5 use symbol - @

18. C If total number of alphabets are 6 use symbol - %


Page 1065 of 1334
Subscribe The Xpress Video Course & Mock Test Package for Bank & Insurance Exams
If there are any suggestions/ errors in our PDFs Feel Free to contact us via this email: admin@exampundit.in
Ultra Practice Bundle PDF
SBI Clerk/ RBI Assist. Mains – Reasoning
If total number of alphabets are 7 use symbol - ^ 32). A

Such as in the words Treat and Advised, the word Treat 33). B
is start with consonant and total number of alphabets are
5 so the code of word Treat is B@ same as the code for 34). D
Advised is A^. 35). C
26). A Solutions (36-40)
27). B For the symbols – Symbols are coded according to the
28). C number of letters in each word

29). B Such as, if there are 4 letters in the word, use - @

30). D If 5, use – %

Solutions (31-35) If 6, use - &

The first letter in the code is the immediate succeeding If 7, use - !


letter (in alphabetical series) of the 2nd letter (from the If 8, use - $
left end) of the word.
For the number, +1 in the place value of first letter of
The number between the letters is the square of the each word.
number of letters in the word.
For the letter, -1 in the last letter of the word.
The last letter in the code is the immediate succeeding
letter (in alphabetical series) of the 2nd letter (from the For example the code of the word CLIMATE is 4!D
right end) of the word. because C+1 = 4, there are 7 letters in the word so use
symbol !, last letter E-1 = D so, the code is 4!D.
Example: ‘larson’
36). D
2nd letter from the left end of the word is ‘A’, and its
immediate succeeding letter in alphabetical series is ‘B’. 37). C

Number of letters in the word is 6, so the number 38). B


between the letters is 36.
39). A
2nd letter from the right end of the word is ‘O’, so the its
40). C
immediate succeeding letter in alphabetical series is ‘P’.
Solution (41-45)
Therefore, the code for ‘larson’ is ‘B36P’

31). D
Page 1066 of 1334
Subscribe The Xpress Video Course & Mock Test Package for Bank & Insurance Exams
If there are any suggestions/ errors in our PDFs Feel Free to contact us via this email: admin@exampundit.in
Ultra Practice Bundle PDF
SBI Clerk/ RBI Assist. Mains – Reasoning
The letter in the code is the letter corresponding to the In each code, the letter is the immediate succeeding letter
number of letters in the word (Ex. A for 1, B for 2 and so (in alphabetical series) of the 2nd letter from the left end
on). of the word.

The number in the code is 1 more than the number There are three symbols used for coding:
representation (in alphabetical series) of the 2nd last
letter of the word (from the left end). a. If there is only one vowel in the word, the symbol used
is ‘#’.
Example:
b. If there are two vowels in the word, the symbol used is
‘random’ ‘$’.

The word has 6 letters, so the letter in its code will be c. If there are three vowels in the word, the symbol used
‘F’. is ‘@’.

2nd last letter of the word is ‘O’, its corresponding Example:


number in alphabetical series is 15, so the number in the
code will be (15 + 1) = 16. ‘Gujarat’

Therefore, the code for ‘random’ is ‘F16’ 2nd letter from the left end is ‘U’. Its immediate
succeeding letter in the alphabetical series is ‘V’.
41). C
There are three vowels in the word, so ‘@’ is used as
42). B symbol.

43). A Therefore, the code for ‘Gujarat’ is ‘V@’.

44). A 46). D

45). C 47). C

Solution (46-50) 48). D

49). A

50). A

Download Seating Arrangement Practice Questions PDF


Get More Reasoning Practice Questions PDF

Page 1067 of 1334


Subscribe The Xpress Video Course & Mock Test Package for Bank & Insurance Exams
If there are any suggestions/ errors in our PDFs Feel Free to contact us via this email: admin@exampundit.in
Ultra Practice Bundle PDF
SBI Clerk/ RBI Assist. Mains – Reasoning
Coding Decoding Number Condition Based
Directions (1-5): In every question two rows are given 2) What is the resultant of second row if Y is the
and to find out the resultant of a particular row you resultant of first row?
need to follow the following steps: 7 6 8 9
Step 1: If an even number is followed by prime number 3 12 Y 13
then the resultant will be the sum of two numbers. a) 5
Step 2: If an even number is followed by odd number then b) 3
the resultant will be the difference between the two c) 8
numbers. d) 9
Step 3: If an odd number is followed by an odd number e) None of these
then the resultant will division of bigger number by 3) What is the sum of resultants of both the rows?
smaller number among them. 9 17 7 4
Step 4: If an even number is followed by an even 2 5 19 6
composite number then the resultant will be the higher a) 60
number of them b) 42
Step 5: If odd number is followed by even number then c) 28
the resultant will be the multiplication of both numbers. d) 70
Step 6: If odd number is followed by prime number then e) None of these.
the resultant will be the lower number among them. 4) What is the LCM of resultant of both the rows?
1). what is the difference in resultant of both the 11 3 2 9
rows? 4 5 2 3
12 3 8 7 a) 1
2 4 6 9 b) 8
c) 35
a) 110 d) 14
b) 111 e) None of these
c) 124
d) 123 5) What is the product of resultant of both the rows?
e) None of these
Page 1068 of 1334
Subscribe The Xpress Video Course & Mock Test Package for Bank & Insurance Exams
If there are any suggestions/ errors in our PDFs Feel Free to contact us via this email: admin@exampundit.in
Ultra Practice Bundle PDF
SBI Clerk/ RBI Assist. Mains – Reasoning
2 7 5 8 121 7 12
7 6 3 13 42 8 17

a) 52 a) 116
b) 104 b) 112
c) 106 c) 114
d) 5 d) 110
e) None of these e) None of these
Directions (6-10): There are two rows given and to 7) Find the product of the resultant of the given two
find out the resultant of a particular row you need to rows.
follow the following steps: – 60 08 13
Step 1: If an even number is followed by an even (non- 42 3 28
prime) number then the resultant will be the difference a) 56
between numbers. b) 72
Step 2: If an even number is followed by an odd (prime) c) 580
number then the resultant will be the division of higher d) 290
number by lower number. e) None of these
Step 3: If an odd number is followed by another odd 8) Find the sum of the resultant of the given two rows.
number (but not a cube or square) then the resultant will 46 23 36
be the addition of both the numbers. 65 4 5
Step 4: If an odd number is followed by an even number a) 82
(but not a cube number) then the resultant comes by b) 83
multiplying the numbers. c) 85
Step 5: If an odd number is followed by a cube number d) 86
then the resultant will be the difference of the cube e) None of these
number and the odd number.
6). Find the difference between the resultant of the 9) What is the resultant of the second row, if X is the
first and second row. resultant of the first row?
09 27 03

Page 1069 of 1334


Subscribe The Xpress Video Course & Mock Test Package for Bank & Insurance Exams
If there are any suggestions/ errors in our PDFs Feel Free to contact us via this email: admin@exampundit.in
Ultra Practice Bundle PDF
SBI Clerk/ RBI Assist. Mains – Reasoning
12 04 X Step 4: If an odd number is followed by an even number
a) 6 then the resultant will be higher number between them.
b) 8 Step 5: If an even number is followed by a prime number
c) 2 then the resultant will be lower number between them.
d) 4 11) Find the difference between the resultant of the
e) None of these first and second row.
10) Find the sum of the resultant of the given two 08 16 12
rows. 3 8 17
a) 6 7 36 03
b) 8 28 6 11
a) 74 c) 4
b) 86 d) 2
c) 94 e) None of these
d) 68 12) Find the product of the resultant of the given two
e) None of these rows.
Directions (11-15): There are two rows given and to 8 5 3
find out the resultant of a particular row you need to 7 14 37
follow the following steps: – a) 196
Step 1: If an odd number is followed by an odd (non- b) 210
prime) number then the resultant will be the addition of c) 280
the numbers. d) 220
Step 2: If an even number is e) None of these
9 23 18
followed by an even (non- 13) Find the sum of the resultant of the given two
45 58 8
prime) number then the rows.
resultant will be the subtraction of lower number from a) 252
higher number. b) 253
Step 3: If an odd number is followed by another odd c) 50
number (prime number) then the resultant will be d) 257
multiplication of both the numbers. e) None of these

Page 1070 of 1334


Subscribe The Xpress Video Course & Mock Test Package for Bank & Insurance Exams
If there are any suggestions/ errors in our PDFs Feel Free to contact us via this email: admin@exampundit.in
Ultra Practice Bundle PDF
SBI Clerk/ RBI Assist. Mains – Reasoning
14) What is the resultant of the second row, if X is the Step 4: If square number is followed by square number
resultant of the first row? then the resultant will be higher number between them.
27 33 2 Step 5: If all of the condition mentioned above does not
22 7 X fulfill then resultant number will be addition of those
a) 10 number.
b) 13 Note: we will take 1 as a cube number
c) 07 16) Find the answer of division of resultant of the row
d) 02 two by resultant of row one.
e) None of these 1 16 8
15) Find the sum of the resultant of the given two 3 13 9
rows. a) 144
37 86 31 b) 16
18 6 23 c) 1
a) 43 d) 2
b) 55 e) None of these
c) 265 17) Find the difference of the resultant of the given
d) 235 two rows.
e) None of these 27 8 4
Directions (16-20): There are two rows given and to 16 9 125
find out the resultant of a particular row you need to a) 109
follow the following steps: – b) 70
Step 1: If a square number is followed by cube number c) 28
then the resultant will be the subtraction of lower number d) 39
from higher number. e) None of these
Step 2: If cube number is followed by cube number then 18) Find the sum of the resultant of the given two
the resultant will be the addition of the numbers. rows.
Step 3: If a cube number is followed by square number 9 27 81
then the resultant will be multiplication of both the 1 8 25
numbers. a) 99

Page 1071 of 1334


Subscribe The Xpress Video Course & Mock Test Package for Bank & Insurance Exams
If there are any suggestions/ errors in our PDFs Feel Free to contact us via this email: admin@exampundit.in
Ultra Practice Bundle PDF
SBI Clerk/ RBI Assist. Mains – Reasoning
b) 116 Step 2: If an even number is followed by an even number
c) 124 then the resultant will be the division of both the numbers
d) 115 such that higher number is divided by lower number.
e) None of these Step 3: If an odd number is followed by an odd number
19) What is the difference resultant of the both row, if then the resultant will be addition of both the numbers.
X is the resultant of the first row? Step 4: If an odd number is followed by an even number
216 4 1 then the resultant will be the subtraction of lower number
81 125 X from higher number.
a) 55 Step 5: If all of the condition mentioned above does not
b) 99 fulfill then resultant number will be addition of those
c) 44 number.
d) 11 21) Find the answer of addition of resultant of the
e) None of these row one and row two.
20) Find the remainder of the division resultant of the
given two rows, when highest number is divided by 16 8 2
lowest number. 6 5 3
a) 1 100 121 125
16 4 23 b) 19
a) 3 c) 91
b) 5 d) 90
c) 6 e) None of these
d) 4 22) Find the difference of the resultant of the given
e) None of these two rows.
Directions (21-25): There are two rows given and to 25 9 2
find out the resultant of a particular row you need to 6 18 12
follow the following steps: – a) 26
Step 1: If an even number is followed by an odd number b) 9
then the resultant will be the multiplication of both the c) 8
numbers. d) 17

Page 1072 of 1334


Subscribe The Xpress Video Course & Mock Test Package for Bank & Insurance Exams
If there are any suggestions/ errors in our PDFs Feel Free to contact us via this email: admin@exampundit.in
Ultra Practice Bundle PDF
SBI Clerk/ RBI Assist. Mains – Reasoning
e) None of these Directions (26-30): There are two rows given and to
23) Find the product of the resultant of the given two find out the resultant of a particular row you need to
rows. follow the following steps: –
4 25 10 Step 1: If the number is multiple of 2 is followed by the
11 13 15 number is multiple of 3 then the resultant will be the
a) 240 subtraction of both the numbers.
b) 390 Step 2: If the number is multiple of 2 is followed by the
c) 3600 number is multiple of 2 then the resultant will be the
d) 3900 addition of both the numbers.
e) None of these Step 3: If the number is multiple of 3 is followed by the
24) What is the remainder of division resultant of the number is multiple of 2 then the resultant will be the
row 1 by resultant of row 2, if X is the resultant of the subtraction of both the numbers.
second row? Step 4: If the number is multiple of 3 is followed by the
36 4 X number is multiple of 3 then the resultant will be the
17 8 14 addition of both the numbers.
a) 3 Step 5: If the number is a multiple of both 2 and 3 then
b) 4 the concerned number will be taken as number is multiple
c) 5 of 2.
d) 1 26) Find the answer of addition of resultant of the
e) None of these row one and row two.
25) Find the difference of resultant of the given two
4 6 8 rows.
15 9 4 89 121 5
24 4 67
a) 138 a) 06
b) 348 b) 36
c) 558 c) 28
d) 448 d) 46
e) None of these e) None of these

Page 1073 of 1334


Subscribe The Xpress Video Course & Mock Test Package for Bank & Insurance Exams
If there are any suggestions/ errors in our PDFs Feel Free to contact us via this email: admin@exampundit.in
Ultra Practice Bundle PDF
SBI Clerk/ RBI Assist. Mains – Reasoning
27) Find the difference of the resultant of the given 30) Find the difference of the resultant of the given
two rows. two rows.
16 22 26 12 16 102
21 27 2 3 9 21
a) 64 a) 133
b) 14 b) 130
c) 41 c) 142
d) 50 d) 109
e) None of these e) None of these
28) Find the sum of the resultant of the given two Directions (31-35): In every question two rows are
rows. given and to find out the resultant of a particular row
15 33 39 you need to follow the following steps:
33 24 51
Rules:
a) 21
b) 69 i) If an odd number is followed by a two digit odd number
c) 75 then they are to be added.
d) 57
ii) If an odd number is followed by a two digit even
e) None of these
number then the lower number is to be subtracted from the
higher number.
29) What is the multiplication resultant of the both
row? iii) If an even number is followed by a number which is a

2 10 9 perfect cube of a number then the second number is to be

6 3 14 multiplied by the first number.

iv) If an even number is followed by a two digit odd


a. 14
b) 11 number but not a perfect cube then the lower number is to

c) 33 be subtracted from the higher number.

d) 2 v) If an odd number is followed by a one digit odd number


e) None of these then we will take preceded odd number as a resultant.

Page 1074 of 1334


Subscribe The Xpress Video Course & Mock Test Package for Bank & Insurance Exams
If there are any suggestions/ errors in our PDFs Feel Free to contact us via this email: admin@exampundit.in
Ultra Practice Bundle PDF
SBI Clerk/ RBI Assist. Mains – Reasoning
vi) If an even number is followed by a one digit even e) None of these
number then we will take followed even number as a 33) What is the difference between resultants of both
resultant. the rows?

Note-1: If none of the conditions satisfied, then 4 27 13


resultant between the two numbers is the lowest
7 5 19
number between them.
a) 60

Note-2: If more than one conditions satisfied, then all b) 69

the conditions are processed and the resultant will be c) 68

the highest number among results. d) 70


e) None of these.
31) What is the difference in resultant of both the
rows? 34) What is the sum of resultant of both the rows?

2 3 7
11 23 27
21 40 16
36 9 22
a) 1
b) 4 a) 918

c) 3 b) 922
d) 2 c) 923
e) None of these
d) 914
32) What is the resultant of second row if Y is half of
the resultant of first row? e) None of these

35) What is the remainder of division of resultant of


7 6 8
row 1 by resultant of row 2?
12 13 Y
a) 25 10 64 5
16 21 3
b) 23
a) 5
c) 28
b) 10
d) 9 c) 1
Page 1075 of 1334
Subscribe The Xpress Video Course & Mock Test Package for Bank & Insurance Exams
If there are any suggestions/ errors in our PDFs Feel Free to contact us via this email: admin@exampundit.in
Ultra Practice Bundle PDF
SBI Clerk/ RBI Assist. Mains – Reasoning
d) 0 c) 6
e) None of these d) 2
e) None of these
Directions (36-40): In every question two rows are
37) What is difference of the resultant of second row
given and to find out the resultant of a particular row
and first row; if Z is half of the resultant of second
you need to follow the following steps:
row?
Rules:
15 7 Z
i) If a number is divisible by followed number then first 42 7 18
number is to be divided by second number. a) 76
b) 28
ii) If a number is not divisible by followed number then
c) 10
the smaller number is to be subtracted from the larger
d) 32
number.
e) None of these
iii) If a number is a followed by its multiple numbers then
38) What is the sum of resultants of both the rows?
they are to be multiplied.
10 100 90
iv) If a number is followed by a number which is square
24 16 64
of that number then a number is to be subtracted from the
a) 81
square number.
b) 57
Note: If more than one conditions satisfied, then all the c) 56
conditions are processed and the resultant will be the d) 49
lowest number among results. e) None of these.

36) What is double of the difference in resultant of 39) What is the square of product of resultant of both
both the rows? the rows?

4 16 6 41 11 5
20 5 3 36 18 4
a) 3 a) 12
b) 1 b) 122
Page 1076 of 1334
Subscribe The Xpress Video Course & Mock Test Package for Bank & Insurance Exams
If there are any suggestions/ errors in our PDFs Feel Free to contact us via this email: admin@exampundit.in
Ultra Practice Bundle PDF
SBI Clerk/ RBI Assist. Mains – Reasoning
c) 124 v) If none of the above condition followed then simply add
d) 144 both the numbers.
e) None of these
41) What is square of the difference in resultant of
40) What is the remainder of division of resultant of both the rows?
row 1 by resultant of row 2?
4 25 29
7 14 49
6 27 729
a) 5 a) 58
6 19 13
b) 10 b) 27
c) 1 c) 961
d) 0 d) 729
e) None of these e) None of these
Directions (41-45): In every question two rows are 42) What is the difference between the resultant of
given and to find out the resultant of a particular row second row and Z; if Z is the resultant of first row?
you need to follow the following steps:
5 125 4
Rules: 7 49 z
a) 124
i) If a number is followed by its square number then
b) 131
second number is to be divided by first number.
c) 8
ii) If a number is followed by its cube number then the d) 7
smaller number is to be subtracted from the larger number. e) None of these

iii) If a number is a followed by a square number (not of 43) What is the double of remainder of division of
preceded number) then they are to be added. resultants of row 1 by resultant of the row 2?

iv) If a number is followed by a cube number (not of 2 9 81


preceded number) then the second number is taken as 12 144 16
resultant. a) 16
b) 12

Page 1077 of 1334


Subscribe The Xpress Video Course & Mock Test Package for Bank & Insurance Exams
If there are any suggestions/ errors in our PDFs Feel Free to contact us via this email: admin@exampundit.in
Ultra Practice Bundle PDF
SBI Clerk/ RBI Assist. Mains – Reasoning
c) 24 Rules:
d) 18
i) If an odd square number is followed by an even number
e) None of these.
then both are added.
44) What is the difference between resultant of both
ii) If an even square number is followed by an odd number
the rows?
then the smaller number is to be subtracted from the larger
8 48 49 number.
36 216 100
iii) If an odd cube number is a followed by even number,
a) 120
then we will take cube number as a resultant.
b) 145
c) 237 iv) If an even cube number is followed by odd number,
d) 211 then the odd number is taken as resultant.
e) None of these
v) If none of the above condition followed then simply do
45) What is the sum of resultant of row 1 and resultant multiplication of numbers.
of row 2?
Note: We will take 1 as an Odd cube number.
11 1131 49
46) What is the difference between the resultant of
2 4 8
both the rows?
a) 1165
b) 1169 81 6 2

c) 1171 16 25 14
d) 1181 a) 174
e) None of these b) 153
c) 151
d) 23
e) None of these

Directions (46-50): In every question two rows are 47) What is the double of sum of the resultant of second

given and to find out the resultant of a particular row row and the value of Z, if the value of Z is the resultant

you need to follow the following steps: of row 1?

Page 1078 of 1334


Subscribe The Xpress Video Course & Mock Test Package for Bank & Insurance Exams
If there are any suggestions/ errors in our PDFs Feel Free to contact us via this email: admin@exampundit.in
Ultra Practice Bundle PDF
SBI Clerk/ RBI Assist. Mains – Reasoning
49) What is the difference between resultant of both
the rows?
1 34 7
121 48 Z 512 125 74
a) 185 49 56 121
b) 177 a) 12380
c) 370 b) 12280
d) 354 c) 12680
e) None of these d) 12580
e) None of these
48) What is the double of difference between resultants
of row 1 by resultant of the row 2? 50) What is the sum of resultant of row 1 and resultant
of row 2?
216 27 84
64 15 15 11 07 49
a) 1344 343 4 56
b) 1408 a) 4106
c) 1426 b) 4116
d) 1436 c) 4216
e) None of these. d) 4316
e) None of these

Page 1079 of 1334


Subscribe The Xpress Video Course & Mock Test Package for Bank & Insurance Exams
If there are any suggestions/ errors in our PDFs Feel Free to contact us via this email: admin@exampundit.in
Ultra Practice Bundle PDF
SBI Clerk/ RBI Assist. Mains – Reasoning
Coding Decoding Number Condition Based – Answer and Explanation
).C Therefore Y in row 2 is 33.

Solution: In row 2,

In row 1, According to step 5, multiplication (3, 12) is 36.

According to step 1, addition of (12, 3) is 15. Then step 2, difference between (36, 33) is 3.

Then step 2, quotient of (15, 8) is 120. Then step 6, smaller number (3, 13) is 3.

Then step 1, 120+7= 127 Resultant of row 2 is 3.

Resultant of row 1 is 127. 3). D

In row 2, Solution:

According to step 4, bigger number among them (4, 2) is In row 1,


4.
According to step 6, smaller of (9, 17) is 9.
Then step 4, bigger number among them (4, 6) is 6.
Then step 6, smaller number (9, 7) is 7.
Then step 2, difference of (9-6,) is 3.
Then step 5, multiplication (7, 4) = 28
Resultant of row 2 is 3.
Resultant of row 1 is 28.
Difference in resultants is 127-3 = 124.
In row 2,
2). B
According to step 1, sum of (2, 5) is 7.
Solution:
Then step 6, lower number (7, 19) is 7.
In row 1,
Then step 5, multiplication (7, 6) is 42.
According to step 5, multiplication of (6, 7) is 42.
Resultant of row 2 is 3.
Then step 4, bigger number (42, 8) is 42.
So sum of resultant of both rows is 42+28=70
Then step 2, 42-9= 33
4). C
Resultant of row 1 is 33.
Solution:

Page 1080 of 1334


Subscribe The Xpress Video Course & Mock Test Package for Bank & Insurance Exams
If there are any suggestions/ errors in our PDFs Feel Free to contact us via this email: admin@exampundit.in
Ultra Practice Bundle PDF
SBI Clerk/ RBI Assist. Mains – Reasoning
In row 1, Resultant of row 2 is 13.

According to step 6, lower number (11, 3) is 3. Product of resultant of both rows is 40*13= 520

Then step 6, lower number (3, 2) is 2. 6). C


Solution:
Then step 2, difference (9,2)=7
In row 1,
Resultant of row 1 is 7.
According to step 3, 121+7=128
In row 2,
Then step 1, 128-12=116.
According to step 1, sum (4, 5) is 9.
Resultant of row 1 is 116.
Then step 6, lower number (9, 2) is 2.
In row 2,
Then step 1, sum (2, 3) is 5.
According to step 1, 42-8 =34
Resultant of row 2 is 5.
Then step 2, 34/17=2.
LCM of resultant of both rows is 35 (7*5=35).
Resultant of row 2 is 2.
5). E
Difference in resultants is 116-2= 114.
Solution:
7). A
In row 1,
Solution:
According to step 1, sum of (2, 7) is 9.
In row 1,
Then step 6, lower number (9, 5) is 5.
According to step 1, 60-8=52
Then step 5, Multiplication (5, 8) is 40
Then step 2, 52/13=4
Resultant of row 1 is 40
Resultant of row 1 is 4.
In row 2,
In row 2,
According to step 5, multiplication (7, 6) is 42.
According to step 2, 42/3 =14.
Then step 1, sum (42, 3) is 45.
Then step 1, 14-28=14.
Then step 6, smaller number (45, 13) is 13.
Resultant of row 2 is 14.
Page 1081 of 1334
Subscribe The Xpress Video Course & Mock Test Package for Bank & Insurance Exams
If there are any suggestions/ errors in our PDFs Feel Free to contact us via this email: admin@exampundit.in
Ultra Practice Bundle PDF
SBI Clerk/ RBI Assist. Mains – Reasoning
Product of resultants of row 4*14=56 Solution:

8). D In row 1,

Solution: According to step 4, 7*36=252


In row 1,
Then step 2, 252/3=84.
According to step 2, 46/23=2
Resultant of row 1 is 84.
Then step 1, 2-36 =34.
In row 2,
Resultant of row 1 is 34.
According to step 1, 28-6=22
In row 2,
Then step 2, 22/11=2
According to step 4, 65*4=260
Resultant of row 2 is 2.
Then step 2,260/5=52
Addition of resultant of both row is 84+2=86
Resultant of row 2 is 52.
11). C
So addition of resultant of both rows is 34+52=86 Solution:
In row 1,
9). C
Solution: According to step 2, 16-8=8

In row 1, Then step 2, 12-8 =4.

According to step 5, difference 9-27=18 Resultant of row 1 is 4.

Then step 2, 18/3=6 In row According to step 4, 3, 8 higher numbers is 8.

Resultant of row 1 is 6 i.e. X=6 Then step 5, 8, 17 lower number is 8

In row 2, Resultant of row 2 is 8.

According to step 1, difference, 12-4=8 So difference of resultant of both rows is 8-4=4.

Then step 1, difference, 8-6=2 12). B


Solution:
Resultant of row 2 is 2
In row 1,
10). B

Page 1082 of 1334


Subscribe The Xpress Video Course & Mock Test Package for Bank & Insurance Exams
If there are any suggestions/ errors in our PDFs Feel Free to contact us via this email: admin@exampundit.in
Ultra Practice Bundle PDF
SBI Clerk/ RBI Assist. Mains – Reasoning
According to step 5, (8, 5) lower number is 5 Resultant of row 1 is 2.

Then step 3, 5*3=15. In row 2, X is 2

Resultant of row 1 is 15. According to step 5, (22, 7) lower number is 7.

In row According to step 4, (7, 14) higher number is 14. Then step 4, (7, 2) so higher number is 7

Then step 5, (14 37) lower number is 14. Resultant of row 2 is 7.

Resultant of row 2 is 14. So resultant of 2nd row is 7.

So multiplication of resultant of both rows is 15). A


14*15=210. Solution:
In row 1,
13). D
Solution: According to step 4, (37 86) higher number is 86
In row 1,
Then step 5, (86 31) lower number is 31
According to step 3, 9*23=207
Resultant of row 1 is 31.
Then step 4, (207, 18) higher number is 207
In row 2
Resultant of row 1 is 207.
According to step 2, 18-6=12
In row 2
Then step 5, (12, 23) lower number is 12
According to step 4, (45, 58) higher number is 58.
Resultant of row 2 is 12.
Then step 2, 58-8=50
So addition of resultant of both rows is 12+31=43.
Resultant of row 2 is 50.
16). D
So addition of resultant of both rows is 207+50=257 Solution:
In row 1,
14). C
Solution: According to step 3, 1*16=16
In row 1,
Then step 1, subtraction 16-8=8
According to step 1, 27+33=60
Resultant of row 1 is 8.
Then step 5, (60, 2) lower number is 2
Page 1083 of 1334
Subscribe The Xpress Video Course & Mock Test Package for Bank & Insurance Exams
If there are any suggestions/ errors in our PDFs Feel Free to contact us via this email: admin@exampundit.in
Ultra Practice Bundle PDF
SBI Clerk/ RBI Assist. Mains – Reasoning
In row 2 Then step 4, (9, 25) higher number is 25

According to step 5, 3+13=16 Resultant of row 2 is 25.

Then step 4, (16, 9) higher number is 16 So addition of resultant of both rows is 99+25=124

Resultant of row 2 is 16. 19). C


Solution:
So row2/row1=16/8=2
In row 1,
17). B
According to step 3, 216*4=864
Solution:
In row 1, Then step 5,864+1=865

According to step 2, 27+8=35 Resultant of row 1 is 865.

Then step 5, 35+4=39 In row 2, X is 865

Resultant of row 1 is 39. According to step 1, 125-81=44

In row According to step 4, (16, 9) higher number is 16. Then step 5, 44+865=909

Then step 1,125-16=109. Resultant of row 2 is 909.

Resultant of row 2 is 109. Difference resultant of both rows, 909-865=44

So difference of resultant of both rows is 109-39=70. 20).A


Solution:
18). C
In row 1,
Solution:
In row 1, According to step 4, (100, 121) higher number is 121

According to step 1, 27-8=18 Then step 1,125-121=4

Then step 5, 18+81=99 Resultant of row 1 is 4.

Resultant of row 1 is 99. In row 2

In row 2 According to step 4, (16, 4) higher number is 16

According to step 2, 1+8=9 Then step 5, 16+23=39

Page 1084 of 1334


Subscribe The Xpress Video Course & Mock Test Package for Bank & Insurance Exams
If there are any suggestions/ errors in our PDFs Feel Free to contact us via this email: admin@exampundit.in
Ultra Practice Bundle PDF
SBI Clerk/ RBI Assist. Mains – Reasoning
Resultant of row 2 is 39. Resultant of row 2 is 9.

So division of resultant of row 2 by row 1 is 39/4=9 So difference between resultant of row 1 and row 2 is
times exactly divided & 3 is remainder. 17-9=8

21). C 23). C
Solution: Solution:
In row 1, In row 1,

According to step 2, 16/8=2 According to step 1, 25*4=100

Then step 2, 2/2=1 Then step 2, 100/10=10

Resultant of row 1 is 1. Resultant of row 1 is 10.

In row 2 In row 2

According to step 1, 6*5=30 According to step 3, 11+13=24

Then step 1, 30*3=90 Then step 1, 24*15=360

Resultant of row 2 is 90. Resultant of row 2 is 360.

So addition of resultant of row 1 and row 2 is So product of resultant of row 1 and row 2 is
1+90=91 10*360=3600

22). C 24). B
Solution: Solution:
In row 1, In row 2,

According to step 3, 25+9=34 According to step 4, 17-8=9

Then step 2, 34/2=17 Then step 4, 14-9=5

Resultant of row 1 is 17. Resultant of row 2 is 5.

In row 2 In row 1

According to step 2, 18/6=3 X is 5.

Then step 4, 12-3=9 According to step 2, 36/4=9

Page 1085 of 1334


Subscribe The Xpress Video Course & Mock Test Package for Bank & Insurance Exams
If there are any suggestions/ errors in our PDFs Feel Free to contact us via this email: admin@exampundit.in
Ultra Practice Bundle PDF
SBI Clerk/ RBI Assist. Mains – Reasoning
Then step 3, 9+5=14 In row 2

Resultant of row 1 is 14. According to step 4, 15+9=24

So division of resultant of row 1 by row 2 is 14/5=2 According to step 5 we will take 24 as multiple of 2.
times exactly divided and 4 remainder.
Then step 2, 24+4=28
25). E
Resultant of row 2 is 28
Solution:
So addition of resultant of row 1 and row 2 is
In row 1,
28+18=46
According to step 3, 89+121=210
27). B
Then step 1, 210*5=1050
Solution:
Resultant of row 1 is 1050. In row 1

In row 2 According to step 2, 22+16=38

According to step 2, 24/4=6 Then step 2, 38+26=64

Then step 1, 6*67=402 Resultant of row 1 is 64.

Resultant of row 2 is 402. In row 2

So difference of resultant of row 1 and row 2 is 1050- According to step 4, 21+27=48


402=648
According to step 5, we will take 48 as a multiple of 2.
26). D
Then step 2, 48+2=50
Solution:
Resultant of row 2 is 50
In row 1
So difference between resultant of row 1 and row 2 is
, Here 6 is multiple of 2 and 3, so according to step 5 we
64-50=14
will take it as multiple of 2.
28). B
According to step 2, 4+6=10
Solution:
Then step 2, 10+8=18
In row 1
Resultant of row 1 is 18.
According to step 4, 15+33=48

Page 1086 of 1334


Subscribe The Xpress Video Course & Mock Test Package for Bank & Insurance Exams
If there are any suggestions/ errors in our PDFs Feel Free to contact us via this email: admin@exampundit.in
Ultra Practice Bundle PDF
SBI Clerk/ RBI Assist. Mains – Reasoning
According to step 5, we will take 48 as a multiple of 2. Solution:
In row 1
Then step 1, 48-39=9.
According to step 5, we will take 12 as a multiple of 2.
Resultant of row 1 is 9.
According to step 2, 16+12=28
In row 2
According to step 5, we will take 102 as a multiple of 2.
According to step 5, we will take 24 as a multiple of 2.
Then step 1, 102+28=130.
According to step 3, 33-24=9
Resultant of row 1 is 130.
Then step 4, 9 +51=60
In row 2
Resultant of row 2 is 60
According to step 4, 3+9=12
So sum of resultant of row 1 and row 2 is 9+60=69
According to step 5, we will take 12 as a multiple of 2.
29). C
Solution: Then step 1, 21-12=9
In row 1
Resultant of row 2 is 9
According to step 2, 10+2=12
The difference between row one and row two is 130-
According to step 5, we will take 12 as a multiple of 2. 9=121
31). A
Then step 1, 12-9=3.
Solution:
Resultant of row 1 is 3.
In row 1
In row 2
(2, 3) no conditions satisfied, lowest number is 2.
According to step 5, 6 will take as multiple of 2.
(2, 7) no conditions satisfied, lowest number is 2.
According to step 1, 6-3=3
Resultant of row 1 is 2.
Then step 3, 14-3=11
In row 2
Resultant of row 2 is 11
According to step 2, 40-21=19
So multiplication of both rows result is 3*11=33
According to step 2, 19-16=3
30). E
Resultant of row 2 is 3.
Page 1087 of 1334
Subscribe The Xpress Video Course & Mock Test Package for Bank & Insurance Exams
If there are any suggestions/ errors in our PDFs Feel Free to contact us via this email: admin@exampundit.in
Ultra Practice Bundle PDF
SBI Clerk/ RBI Assist. Mains – Reasoning
Difference between row 1 and row 2 is 3-2=1 Resultant of row 2 is 26.
32). B
Difference between Resultant of row 1 and row 2 is
Solution:
95-26=69
In row 1
34). E
(7, 6) no conditions satisfied, lowest number is 6. Solution:
In row 1
Two conditions satisfied as follows (6, 8)
According to step 1, 11+23=34
According to step 3, 6*8=48
According to step 3, 27*34=918
According to step 6, (6, 8), followed number is 8.
Resultant of row 1 is 918.
Among two conditions 48 is the highest result.
In row 2
Resultant of row 1 is 48.
(36, 9) no conditions satisfied, lowest number is 9
In row 2
According to step 2, 22-9=13
According to step 4, 13-12=1
Resultant of row 2 is 13
Y is half of Resultant of row 1, 48/2=24
Sum of Resultant of row 1 and row 2 is 918+13=931
According to step 2, 24-1=23
35). D
Resultant of row 2 is 23.
Solution:
33). B In row 1
Solution:
According to step 4, 64*10=640
In row 1
(640, 5) no conditions satisfied, lowest number is 5
According to step 3, 27*4=108
Resultant of row 1 is 5.
According to step 4,108-13=95
In row 2
Resultant of row 1 is 95.
According to step 4, 21-16=5
In row 2
According to step 5, (5, 3) preceded number is 5.
According to step 5, (7, 5) preceded number is 7.
Resultant of row 2 is 5.
According to step 1, 7+19=26
Page 1088 of 1334
Subscribe The Xpress Video Course & Mock Test Package for Bank & Insurance Exams
If there are any suggestions/ errors in our PDFs Feel Free to contact us via this email: admin@exampundit.in
Ultra Practice Bundle PDF
SBI Clerk/ RBI Assist. Mains – Reasoning
Division of Resultant of row 1 and row 2 is 5/5=1 times According to step ii, 18-6=12
exactly divided.
According to step iii, 6*18=108
So remainder is 0.
Among two conditions, 12 is the lowest result.
36). D
Solution: Resultant of row 2 is 12.
In row 1 In row 1
Three conditions satisfied as follows (4, 16) Z is half of resultant of row 2, 12/2=6
According to step ii, 16-4=12 According to step ii, 15-7=8
According to step iii, 16*4=64 According to step ii, 8-6=2
According to step iv, 16-4=12 Resultant of row 1 is 2.
Among three conditions, 12 is the lowest result. Difference between row 2 and row 1 is 12-2=10
According to step i, 12/6=2.
38). B
Resultant of row 1 is 2. Solution:

In row 2 In row 1

According to step i, 20/5=4 Three conditions satisfied as follows (10, 100)

According to step ii, 4-3=1 According to step ii, 100-10=90

Resultant of row 2 is 1. According to step iii, 10*100=1000

Difference of resultant of row 1 and row 2 is 2-1=1 According to step iv, 100-10=90

So double of 1 is 1*2=2 Among three conditions, 90 is the lowest result.


37). C
Two conditions satisfied as follows (90, 90)
Solution:
In row 2 According to step i, 90/90=1

According to step i, 42/7=6 According to step iii, 90*90=8100

Two conditions satisfied as follows (6, 18) Among two conditions, 1 is the lowest result.

Resultant of row 1 is 1
Page 1089 of 1334
Subscribe The Xpress Video Course & Mock Test Package for Bank & Insurance Exams
If there are any suggestions/ errors in our PDFs Feel Free to contact us via this email: admin@exampundit.in
Ultra Practice Bundle PDF
SBI Clerk/ RBI Assist. Mains – Reasoning
In row 2 product of resultant of row 1 and row 2 is 6*2=12
Square of product 12 is 144.
According to step ii, 24-16=8
40).D
Three conditions satisfied as follows (8, 64)
Solution:
According to step ii, 64-8=56 In row 1

According to step iii, 8*64=512 Two conditions satisfied as follows (7, 14)

According to step iv, 64-8=56 According to step ii, 14-7=7

Among three conditions, 56 is the lowest result. According to step iii, 7*14=98

Resultant of row 2 is 56. Among two conditions, 7 is the lowest result.

Sum of the resultant of row 1 and row 2 is 1+56=57. Again three conditions satisfied as follows (7, 49)
39). D
According to step ii, 49-7=42
Solution:
According to step iii, 7*49=343
In row 1
According to step iv, 49-7=42
According to step ii, 41-11=30
Among three conditions, 42 is the lowest result.
According to step i, 30/5=6
Resultant of row 1 is 42
Resultant of row 1 is 6.
In row 2
In row 2
According to step ii, 19-6=13
According to step i, 36/18=2
Two conditions satisfied as follows (13, 13)
Three conditions satisfied as follows (2, 4)
According to step i, 13/13=1
According to step ii, 4-2=2
According to step iii, 13*13=169
According to step iii, 2*4=8
Among two conditions, 1 is the lowest result.
According to step iv, 4-2=2
Resultant of row 2 is 1.
Among three conditions, 2 is the lowest result.
Division of Resultant of row 1 and row 2 is 42/1=42
Resultant of row 2 is 2.
times exactly divided.
Page 1090 of 1334
Subscribe The Xpress Video Course & Mock Test Package for Bank & Insurance Exams
If there are any suggestions/ errors in our PDFs Feel Free to contact us via this email: admin@exampundit.in
Ultra Practice Bundle PDF
SBI Clerk/ RBI Assist. Mains – Reasoning
So remainder is 0. Resultant of row 2 is 131.
41). C
Difference between row 2 and Z is 131-124=7
Solution:
In row 1 43). A
Solution:
According to step iii, 25+4=29
In row 1
According to step v, 29+29=58
According to step iii, 2+9=11
Resultant of row 1 is 58.
According to step iii, 11+81=92
In row 2
Resultant of row 1 is 92
According to step iv, 27
In row 2
According to step i, 729/27=27.
According to step i, 144/12=12
Resultant of row 2 is 27.
According to step iii, 12+16=28
Difference between resultant of row 1 and row 2 is 58-
Resultant of row 2 is 28.
27=31
So square of this is 961. Division of resultant of row 1 and row 2 92/24= 3 times

42). D exactly divided

Solution: Remainder is 8.

In row 1 Double of remainder is 8*2=16

According to step ii, 125-5=120 44). D


Solution:
According to step iii, 120+4=124
In row 1
Resultant of row 2 is 124.
According to step v, 8+48=56
In row 2
According to step iii, 56+49=105
Z is resultant of row 1, 124
Resultant of row 1 is 105
According to step i, 49/7=7
In row 2
According to step v, 124+7=131
According to step iv, 216

Page 1091 of 1334


Subscribe The Xpress Video Course & Mock Test Package for Bank & Insurance Exams
If there are any suggestions/ errors in our PDFs Feel Free to contact us via this email: admin@exampundit.in
Ultra Practice Bundle PDF
SBI Clerk/ RBI Assist. Mains – Reasoning
According to step iv, 216+100=316 Resultant of row 2 is 23.

Resultant of row 2 is 316. Difference between Resultant of row 1 and row 2 is 174-
23=151
Difference between resultant of row 2 and row 1 is
47). C
316-105=211
Solution:
45). E
In row 1
Solution:
In row 1 According to step iii, (1 34) resultant is 1

According to step v, 1131+11=1142 According to step v, 1+7=8

According to step iii, 1142+49=1191 Resultant of row 1 is 8.

Resultant of row 1 is 1191 In row 2

In row 2 Z is of resultant of row 1, 8

According to step i, 4/2=2 According to step i, 121+48=169

According to step ii, 8-2=6 According to step i, 169+8=177

Resultant of row 2 is 6. Resultant of row 2 is 177.

Sum of resultant of row 1 and row 2 is 1191+6=1197 Addition of row 2 and Z is 8+177=185
46). C
Double of addition of row 2 and Z is 370.
Solution:
48).E
In row 1
Solution:
According to step i, 81+6=87 In row 1
According to step v, 87*2=174 According to step iv, (216, 27) resultant is 27
Resultant of row 1 is 174. According to step iii, (27, 84) resultant is 27
In row 2 Resultant of row 1 is 27.
According to step ii, 25-16=9 In row 2
According to step i, 9+14=23 According to step ii, 64-15=49

Page 1092 of 1334


Subscribe The Xpress Video Course & Mock Test Package for Bank & Insurance Exams
If there are any suggestions/ errors in our PDFs Feel Free to contact us via this email: admin@exampundit.in
Ultra Practice Bundle PDF
SBI Clerk/ RBI Assist. Mains – Reasoning
According to step v, 49*15=735 Resultant of row 2 is 12705

Resultant of row 2 is 735. Difference between resultant of row 2 and row 1 is


12705-125=12580
Difference between resultant of row 1 and row 2 is 735-
50). B
27=708
Solution:
Double of difference between resultant of row 1 and row
In row 1
2 is 1416.
According to step v, 11*7=77
49). D
Solution: According to step v, 77*49=3773

In row 1 Resultant of row 1 is 3773.

According to step iv, (512, 125) resultant is 125 In row 2

According to step iii, (125, 74) resultant is 125 According to step iii, (343, 4) resultant is 343

Resultant of row 1 is 125. According to step iii, (343, 56) resultant is 343

In row 2 Resultant of row 2 is 343

According to step i, 49+56=105 Sum of resultant of row 1 and row 2 is

According to step v, 105*121=12705 3773+343=4116.

Download Seating Arrangement Practice Questions PDF


Get More Reasoning Practice Questions PDF
Coding Decoding Symbol Alphabet Based
(Direction 1-5): Study the following information
carefully and answer the given questions: In a coded language:

In alphabetical series A-Z each letter except vowels is It Is High Time Now: ^14 ^12 12^1012 14^2% 4&18
assigned a different number starting from multiple of 2
(for ex- B is coded as 2, C-4, L-18) and again those Besides the above example, following operations are to be
numbers get repeated (for ex- M-2, N-4 and so on). Also, applied for coding the words given in the question below:
each vowel is coded with different symbol viz. $, %, ^, &
and * respectively.
Page 1093 of 1334
Subscribe The Xpress Video Course & Mock Test Package for Bank & Insurance Exams
If there are any suggestions/ errors in our PDFs Feel Free to contact us via this email: admin@exampundit.in
Ultra Practice Bundle PDF
SBI Clerk/ RBI Assist. Mains – Reasoning
I. If the first letter of the word is consonant and fourth b) 8101010*18$
letter is vowel then the code of the consonant will be the c) 1010101*18
same for the vowel. d) Both A and C
e) None of These
II. If the last two letters are vowels then the code ending
with 0 will be replaced with * 4) What will be the code of SHIPPING?
a) !12^66^4!
III. If the second letter of the word is vowel and fourth b) 121212^^^^^10
letter is consonant then both are to be coded as the code of c) 1212^^^^^10
the third letter. d) Both A and C
e) Cannot Be Determined
IV. If both first and the second letter are consonant then
the code which are in single digit will be replaced with the 5) What can be the code of TAJIKISTAN?
code of the third letter. a) 14$14^16^1214$4
b) !$14^16^1214$!
V. If first letter of the word is consonant and the last letter c) +$14^16^1214$$
of the word is also consonant then both are to be coded as d) ^$14^16^1214$4
“!” e) Both B and D

If more than one condition applies for a word then, both (Direction 6-10): Study the following information
the codes will be the answer carefully and answer the given questions:

1) What can be the code of PILOT? In alphabetical series A-Z each letter except vowels is
a) &^18&14 assigned a different number starting from multiple of 3
b) 618^14& (for ex- B is coded as 3, C-6, L-27) and again those
c) !^18&! numbers get repeated (for ex- M-3, N-6 and so on). Also,
d) Both A and C each vowel is coded with different symbol viz. @, #, $, %,
e) Cannot Be Determined & respectively.

2) What will be the code of INDIA? In a coded language:


a) ^46^$
b) $46^^ Corona Is Killing World: 6%15%6@ $18 24$2727$15
c) $^46$ 27%15279
d) Either A or B
e) Both A and B Besides the above example, following operations are to be
applied for coding the words given in the question below:
3) What can be the code of FORMULA?
a) 8202020*18$
Page 1094 of 1334
Subscribe The Xpress Video Course & Mock Test Package for Bank & Insurance Exams
If there are any suggestions/ errors in our PDFs Feel Free to contact us via this email: admin@exampundit.in
Ultra Practice Bundle PDF
SBI Clerk/ RBI Assist. Mains – Reasoning
I. If the first letter of the word is vowel and fourth letter is b) #12&$**#6*
consonant then the code of the both the letters will be the c) ##&#93#621
code of the vowel d) Both B and C
e) Cannot Be Determined
II. If the last two letters are consonant then the odd number
code will be replaced with * 9) What will be the code of EAST?
a) #@18#
III. If the second letter of the word is consonant and fourth b) #@1825
letter is vowel then both are to be coded as the code of the c) #@18*
first letter. d) #@1821
e) A, C and D
IV. If both first and the second letter are vowel then the
code which are in single digit will be replaced with the 10) What will be the code of AFRICA?
code of the fourth letter. a) @@15@6@
b) Both A and C
V. If first letter of the word is vowel and the last letter of c) 81215$68
the word is also vowel then both are to be coded as “8” d) Cannot Be Determined
e) None of These
If more than one condition applies for a word then, both
the codes will be the answer (Direction 11-15): Study the following information
carefully and answer the given questions:
6) What will be the code of ANDHERI?
a) @69@#15$ In alphabetical series A-Z each letter except vowels is
b) 86918#158 assigned a different number starting from multiple of 2
c) Both A and B (for ex- B is coded as 2, C-4, L-18) and again those
d) @69@#15$ numbers get repeated (for ex- M-2, N-4 and so on). Also,
e) Cannot Be Determined each vowel is coded with different symbol viz. $, %, ^, &
and * respectively.

7) What will be the code of REPLACEMENT? In a coded language:


a) *#**@6#*#6*
b) @#**@6#*#6* It Is High Time Now: ^14 ^12 12^1012 14^2% 4&18
c) !#**@6#*#6*
d) *#**@6#*#6@ Besides the above example, following operations are to be
e) Cannot Be Determined applied for coding the words given in the question below:

8) What will be the code of EQUIPMENT? I. If first letter of the word is consonant and last letter is
a) #12&$**#6@ vowel then the codes of both of them will be interchanged.
Page 1095 of 1334
Subscribe The Xpress Video Course & Mock Test Package for Bank & Insurance Exams
If there are any suggestions/ errors in our PDFs Feel Free to contact us via this email: admin@exampundit.in
Ultra Practice Bundle PDF
SBI Clerk/ RBI Assist. Mains – Reasoning
14) What will be the code of SANITIZER?
II. If both first and last letter of the word are vowel then a) ^$4^^^6%^
the single digit code of the word will be replaced with “*”. b) 2$4%14%24%10
c) 4$4%14%24%10
III. If first letter of the word is vowel and last letter is d) Both A and E
consonant then both are to be coded as the code of first e) 12%4%14%6%10
letter.
15) What will be the code of CONGO?
IV. If both first and last letter are consonant then the code a) 4%410%
with two digits is replaced with “^”. b) &&4104
c) Both A and B
V. If the middle letter of the word is consonant, then all d) Cannot Be Determined
the vowels present in the word will be coded as “%” e) None of These

If more than one condition applies for a word then, both (Direction 16-20): Study the following information
the codes will be the answer carefully and answer the given questions:

11) What can be the code of BOTTLE? In alphabetical series A-Z each letter except vowels is
a) 2&141418% assigned a different number starting from multiple of 3
b) %&1414182 (for ex- B is coded as 3, C-6, L-27) and again those
c) %*141418 numbers get repeated (for ex- M-3, N-6 and so on). Also,
d) Both A and C each vowel is coded with different symbol viz. @, #, $, %,
e) Cannot Be Determined & respectively.

12) What will be the code of AMERICA? In a coded language:


a) $2%10^4$
b) $*%10^*$ Corona Is Killing World: 6%15%6@ $18 24$2727$15
c) %2%10%4% 27%15279
d) Both B and C
e) Both A and C Besides the above example, following operations are to be
applied for coding the words given in the question below:
13) What will be the code of AMAZON?
a) $2$6&$ I. If first letter of the word is vowel and last letter is
b) &2$6&$ consonant then the codes of both of them will be code of
c) ! 2$6&$ the third letter.
d) (2$6&$
e) None of These II. If both first and last letter of the word are vowel then the
odd digit code of the word will be replaced with “9”.
Page 1096 of 1334
Subscribe The Xpress Video Course & Mock Test Package for Bank & Insurance Exams
If there are any suggestions/ errors in our PDFs Feel Free to contact us via this email: admin@exampundit.in
Ultra Practice Bundle PDF
SBI Clerk/ RBI Assist. Mains – Reasoning
d) #$999#@
III. If first letter of the word is consonant and last letter is e) #999$#@
vowel then both are to be coded as the code of fourth letter.
20) What will be the code of PATHOLOGY?
IV. If both first and the second last letter are vowels then a) S#@S#S#%S#%S#S#
the code with even numbered digits is replaced with “0”. b) S#@S#S#%S#%S##S
c) S#@S#S#%S#%S%S#
V. If the middle letter of the word is vowel, then all the d) S#@S#S#%S#%%#S#
consonants present in the word will be coded as “S#” e) S#%S#S#%S#%S#S#

If more than one condition applies for a word then, both the (Direction 21-25): Study the following information
codes will be the answer carefully and answer the given questions:

16) What will be the code of ENGAGEMENT? In alphabetical series A-Z each letter except vowels is
a) 15615@15#3#615 assigned a different number starting with two-digit odd
b) 21615@15#3#615 number (for ex- B is coded as 11, C-13, L-27) and again
c) 18615@15#3#615 those numbers get repeated (for ex- M-11, N-13 and so
d) 12615@15#3#615 on). Also, each vowel is coded with different symbol viz.
e) 24615@15#3#615 #, $, @, % and & respectively.

17) What will be the code of ESTONIA? In a coded language:


a) #S#S#%S#$@
b) #189%6$@ Use Masks And Sanitizer: &21$ 11#212521 #1315
c) #021%0$@ 21#13@23@17$19
d) All A, B and C
e) Cannot Be Determined Besides the above example, following operations are to be
applied for coding the words given in the question below:
18) What will be the code of MIDDLE?
a) 9$99279 I. If the first letter of the word is vowel and fifth letter is
b) 9999279 consonant then the code of the consonant will be the same
c) 2799999 for the both the letters
d) 9279999
e) None of These II. If the last two letters are vowels then the odd number
code will be replaced with “P”
19) What will be the code of ERITREA?
a) #9$99#@ III. If the third letter of the word is consonant and last
b) #15$2115#@ letter is vowel then both are to be coded as the code of the
c) Both A and B second letter.
Page 1097 of 1334
Subscribe The Xpress Video Course & Mock Test Package for Bank & Insurance Exams
If there are any suggestions/ errors in our PDFs Feel Free to contact us via this email: admin@exampundit.in
Ultra Practice Bundle PDF
SBI Clerk/ RBI Assist. Mains – Reasoning

IV. If both first and the second letter are vowel then the 25) What will be the code of ANTARCTICA?
code which are in single digit will be replaced with the a) 191323#191323@13#
code of the third letter. b) #1313#191323@1313
c) 11323#191323@131
V. If first letter of the word is vowel and the last letter of d) All A, B and C
the word is also vowel then both are to be coded as “1” e) Both A and B

If more than one condition applies for a word then, both (Direction 26-30): Study the following information
the codes will be the answer carefully and answer the given questions:

21) What will be the code of ODESSA? In alphabetical series A-Z each letter except vowels is
a) 2115$2121# assigned a different number starting with two-digit even
b) 115$21211 number (for ex- B is coded as 10, C-12, L-26) and again
c) Both A and B those numbers get repeated (for ex- M-10, N-12 and so
d) Cannot Be Determined on). Also, each vowel is coded with different symbol viz.
e) 2115$21213 &, @, %, # and $ respectively.

22) What will be the code of ASIA? In a coded language:


a) #21@#
b) #P@# Use Disinfectant Sprays: $20@
c) 121@1 14%20%1216@1222&1222 201418&1220
d) Both B and C
e) None of These Besides the above example, following operations are to be
applied for coding the words given in the question below:
23) What will be the code of MAPUTO?
a) #11#&23# I. If the first letter of the word is consonant and fifth letter
b) ##11&23# is vowel then the code of the vowel will be the code for
c) 11##&23# the both the letters
d) 11##23&#
e) 11##23#& II. If the last two letters are consonant then the even
number code will be replaced with “A”
24) What will be the code of AUTO?
a) #&&& III. If the third letter of the word is vowel and last letter is
b) 1&231 consonant then both are to be coded as the code of the first
c) #&#& letter.
d) #&23%
e) A, B and D
Page 1098 of 1334
Subscribe The Xpress Video Course & Mock Test Package for Bank & Insurance Exams
If there are any suggestions/ errors in our PDFs Feel Free to contact us via this email: admin@exampundit.in
Ultra Practice Bundle PDF
SBI Clerk/ RBI Assist. Mains – Reasoning
IV. If both first and the second letter are consonant then
the code which are ending with 0 will be replaced with the 30) What is the code of TABLET?
code of the second letter. a) &@1026@22
b) @&1026@22
V. If first letter of the word is consonant and the last letter c) 2&1026@2
of the word is also consonant then both are to be coded as d) Both B and C
“2” e) Both A and B

If more than one condition applies for a word then, both (Direction 31-35): Study the following information
the codes will be the answer carefully and answer the given questions:

26) What is the code of CONVERSATION? In alphabetical series A-Z each letter except vowels is
a) @#1224@1820&22%#12 assigned a different number starting with two-digit odd
b) #@1224@1820&22%#12 number (for ex- B is coded as 11, C-13, L-27) and again
c) 2#1224@1820&22%#2 those numbers get repeated (for ex- M-11, N-13 and so
d) Both A and C on). Also, each vowel is coded with different symbol viz.
e) Both A and B #, $, @, % and & respectively.

27) What is the code of PRICING? In a coded language:


a) AA%A%AA
b) 14181412%1214 Use Masks And Sanitizer: &21$ 11#212521 #1315
c) 1418%12%1218 21#13@23@17$19
d) 218%12%122
e) All of the Above Besides the above example, following operations are to be
applied for coding the words given in the question below:
28) What is the code of SHARJAH?
a) 2020&1822&20 I. If first letter of the word is vowel and last letter is
b) 0220201822&20 consonant then the codes of both of them will be of code
c) 2020201822&20 of the fourth letter.
d) 220&1822&2
e) A, C and D II. If both first and last letter of the word are consonants
then the even digit code of the word will be replaced with
29) What is the code of FREIGHT? “6”.
a) All of the given options
b) 1618@%181822 III. If first letter of the word is consonant and last letter is
c) AA@%AAA vowel then both are to be coded as the code of second
d) 161816%182016 letter.
e) 218@%18202
Page 1099 of 1334
Subscribe The Xpress Video Course & Mock Test Package for Bank & Insurance Exams
If there are any suggestions/ errors in our PDFs Feel Free to contact us via this email: admin@exampundit.in
Ultra Practice Bundle PDF
SBI Clerk/ RBI Assist. Mains – Reasoning
IV. If both first and last letter are vowels then the code a) 11%%25@1319
with odd numbered digits is replaced with “3”. b) 11XX25X1319
c) 11X25XX1319
V. If the middle letter of the word is consonant, then all d) Both A and B
the vowels present in the word will be coded as “X” e) Both A and C

If more than one condition applies for a word then, both (Direction 36-40): Study the following information
the codes will be the answer carefully and answer the given questions:

31) What will be the code of OCTOBER? In alphabetical series A-Z each letter except vowels is
a) %1323%11$% assigned a different number starting with first five letters
b) 13%23%11$% will have the code of first five multiple of 11 (for ex- B is
c) %323%11$% coded as 11, C-22, D-33) and next four letters will have
d) %123%11$% the code of first four multiple of 13 and then again next
e) %133%11$% four letters will have the code of first four multiple of 17
and after that next five letters will have the code of first
32) What will be the code of CLAVAM? five multiple of 19 and at last next four letters will have
a) 1237#25#11 the code of first four multiple of 21 and at last Z will be
b) 3127#25#11 coded as 1. Also, each vowel is coded with different
c) 1372#25#11 symbol viz. &, @, %, # and $ respectively.
d) 1273#25#11
e) 1327#25#11 In a coded language:

33) What will be the code of QUOTE? Get Your Test: 26@95 84#$57 95@7695
a) 17&%23$
b) &&%23& Besides the above example, following operations are to be
c) &%%23& applied for coding the words given in the question below:
d) &&%32&
e) &&%3&2 I. If the first letter of the word is consonant and fifth letter
is vowel then the code of the vowel will be the code for
34) What will be the code of AUSTRALIA? the both the letters
a) XX212319X27XX
b) #21&2319#27@# II. If the last two letters are consonant then the even
c) &#212319#27@# number code will be replaced with “A”
d) #&333#3@#
e) Both A and D III. If the third letter of the word is vowel and last letter is
consonant then both are to be coded as the code of the first
35) What will be the code of BOOKING? letter.
Page 1100 of 1334
Subscribe The Xpress Video Course & Mock Test Package for Bank & Insurance Exams
If there are any suggestions/ errors in our PDFs Feel Free to contact us via this email: admin@exampundit.in
Ultra Practice Bundle PDF
SBI Clerk/ RBI Assist. Mains – Reasoning
e) 223&57126@
IV. If both first and the second letter are consonant then
the code which are ending with 0 will be replaced with the 40) What will be the code of COMMODITY?
code of the second letter. a) ##5151#33%9584
b) A#5151#33%95A
V. If first letter of the word is consonant and the last letter c) 2#5151#33%952
of the word is also consonant then both are to be coded as d) A, B, C
“2” e) None of These

If more than one condition applies for a word then, both (Direction 41-45): Study the following information
the codes will be the answer carefully and answer the given questions:

36) What will be the code of CAREER? In alphabetical series A-Z each letter except vowels is
a) @&57@@57 assigned a different number starting with two-digit even
b) 2&57@@2 number (for ex- B is coded as 10, C-12, L-26) and again
c) Both A and B those numbers get repeated (for ex- M-10, N-12 and so
d) 257&@@2 on). Also, each vowel is coded with different symbol viz.
e) Both A and D &, @, %, # and $ respectively.

37) What will be the code of PRESS? In a coded language:


a) 1957@AA
b) 1957197619 Use Disinfectant Sprays: $20@
c) 1957@7676 14%20%1216@1222&1222 201418&1220
d) 257@762
e) All of the Above Besides the above example, following operations are to be
applied for coding the words given in the question below:
38) What will be the code of PRODUCT?
a) $57#33$2295 If more than one condition applies for a word then, both
b) 1957#33$A95 the codes will be the answer
c) 19571933$2219
d) 1957#33$2295 I. If first letter of the word is vowel and last letter is
e) All of the Above consonant then the codes of both of them will be coded as
per the code second letter.
39) What will be the code of CHARGE?
a) 2239&5726@ II. If both first and last letter of the word are consonants
b) 3922&5726@ then the even digit codes of the word will be replaced with
c) 2293&5726@ “9”.
d) 239&25726@
Page 1101 of 1334
Subscribe The Xpress Video Course & Mock Test Package for Bank & Insurance Exams
If there are any suggestions/ errors in our PDFs Feel Free to contact us via this email: admin@exampundit.in
Ultra Practice Bundle PDF
SBI Clerk/ RBI Assist. Mains – Reasoning
III. If first letter of the word is consonant and last letter is e) #22%%%
vowel then both are to be coded as the code of third letter.
45) What will be the code of LOOKOUT?
IV. If both first and last letter are vowels then the code a) 9##9#$9
with even numbered digits is replaced with “2”. b) 26BB24BB22
c) Both A and B
V. If the middle letter of the word is consonant, then all d) Both A and E
the vowels present in the word will be coded as “B” e) 28BB24BB22

If more than one condition applies for a word then, both (Direction 46-50): Study the following information
the codes will be the answer carefully and answer the given questions:

41) What will be the code of ASHDOD? In alphabetical series A-Z each letter except vowels is
a) 20202014#20 assigned a different number starting with first five letters
b) 20202214#18 will have the code of first five multiple of 11 (for ex- B is
c) 20202214#16 coded as 11, C-22, D-33) and next four letters will have
d) 20202214#14 the code of first four multiple of 13 and then again next
e) 20202214#12 four letters will have the code of first four multiple of 17
and after that next five letters will have the code of first
42) What will be the code of QUALITY? five multiple of 19 and at last next four letters will have
a) 9$&9%99 the code of first four multiple of 21 and at last Z will be
b) 16BB26B2212 coded as 1. Also, each vowel is coded with different
c) Both A and B symbol viz. &, @, %, # and $ respectively.
d) 16BB26B2210
e) Both B and D In a coded language:

43) What will be the code of CLASSMATE? Get Your Test: 26@95 84#$57 95@7695
a) 26&&202010&22&
b) &26&202010&22& Besides the above example, following operations are to be
c) 1226B202010B22B applied for coding the words given in the question below:
d) Both B and C
e) Both A and B I. If first letter of the word is vowel and last letter is
consonant then the codes of both of them will be code of
44) What will be the code of OFFICE? the second letter.
a) ##2%2@
b) #2#%2@ II. If both first and last letter of the word are consonants
c) #22%2@ then the even digit codes of the word will be replaced with
d) #22%2% “9”.
Page 1102 of 1334
Subscribe The Xpress Video Course & Mock Test Package for Bank & Insurance Exams
If there are any suggestions/ errors in our PDFs Feel Free to contact us via this email: admin@exampundit.in
Ultra Practice Bundle PDF
SBI Clerk/ RBI Assist. Mains – Reasoning
d) 90@5751%9&9
III. If first letter of the word is consonant and last letter is e) 99@5751%9&9
vowel then both are to be coded as the code of third letter.
48) What is the code of FREE?
IV. If both first and last letter are vowels then the code a) $57@@
with even numbered digits is replaced with “2”. b) %57@@
c) *57@@
V. If the middle letter of the word is consonant, then all d) ^57@@
the vowels present in the word will be coded as “B” e) @57@@

If more than one condition applies for a word then, both 49) What is the code of AUSTRIA?
the codes will be the answer a) &$29557%&
b) BB769557BB
46) What is the code of EXPORT? c) Both A and B
a) 636319#5763 d) AB209557BB
b) 663319#5763 e) Both A and D
c) 636919#5763
d) 633319#5763 50) What is the code of CALLING?
e) 666319#5763 a) 8&99%99
b) 9&99%99
47) What is the code of TERMINAL? c) 22B3434B6826
a) 95@5751119&9 d) Both B and C
b) 95@5751%969 e) 9999%99
c) 95@5751%9&9

Page 1103 of 1334


Subscribe The Xpress Video Course & Mock Test Package for Bank & Insurance Exams
If there are any suggestions/ errors in our PDFs Feel Free to contact us via this email: admin@exampundit.in
Ultra Practice Bundle PDF
SBI Clerk/ RBI Assist. Mains – Reasoning
Coding Decoding Symbol Alphabet Based – Answer and Explanation
Answers (1-5):

1) D
2) A
3) B
4) C
5) E

Solutions (1-5):

1)
Code for consonant and vowels are:
Consonant B C D F G H J K L M N P Q R S T V W X Y Z
Code 2 4 6 8 10 12 14 16 18 2 4 6 8 10 12 14 16 18 2 4 6

Vowel A E I O U
Code $ % ^ & *
Code of “PILOT” will be 6^18&14

Now condition number I and V will be applied to word PILOT and after that the code of the word PILOT will be
&^18&14 and !^18&!

2)
Code for consonant and vowels are:
Consonant B C D F G H J K L M N P Q R S T V W X Y Z

Code 2 4 6 8 10 12 14 16 18 2 4 6 8 10 12 14 16 18 2 4 6

Vowel A E I O U
Code $ % ^ & *
Code of “INDIA” will be ^46^$

Now condition number II will be applied to word INDIA but in the code of the word there is no code ending with 0 so,
code of the word INDIA will be ^46^$

Page 1104 of 1334


Subscribe The Xpress Video Course & Mock Test Package for Bank & Insurance Exams
If there are any suggestions/ errors in our PDFs Feel Free to contact us via this email: admin@exampundit.in
Ultra Practice Bundle PDF
SBI Clerk/ RBI Assist. Mains – Reasoning
3)
Code for consonant and vowels are:
Consonant B C D F G H J K L M N P Q R S T V W X Y Z

Code 2 4 6 8 10 12 14 16 18 2 4 6 8 10 12 14 16 18 2 4 6

Vowel A E I O U
Code $ % ^ & *
Code of “FORMULA” will be 8&102*18$

Now condition number III will be applied to word FORMULA and after that the code of the word FORMULA will
be 8101010*18$

4)
Code for consonant and vowels are:
Consonant B C D F G H J K L M N P Q R S T V W X Y Z

Code 2 4 6 8 10 12 14 16 18 2 4 6 8 10 12 14 16 18 2 4 6

Vowel A E I O U
Code $ % ^ & *
Code of “SHIPPING” will be 1212^66^410

Now condition number IV and V will be applied to word SHIPPING and after that the code of the word
SHIPPING will be !12^66^4! and 1212^^^^^10

5)
Code for consonant and vowels are:
Consonant B C D F G H J K L M N P Q R S T V W X Y Z

Code 2 4 6 8 10 12 14 16 18 2 4 6 8 10 12 14 16 18 2 4 6

Vowel A E I O U

Page 1105 of 1334


Subscribe The Xpress Video Course & Mock Test Package for Bank & Insurance Exams
If there are any suggestions/ errors in our PDFs Feel Free to contact us via this email: admin@exampundit.in
Ultra Practice Bundle PDF
SBI Clerk/ RBI Assist. Mains – Reasoning
Code $ % ^ & *
Code of “TAJIKISTAN” will be 14$14^16^1214$4

Now condition number V and I will be applied to word TAJIKISTAN and after that the code of the word
TAJIKISTAN will be 14$14^16^1214$4 and ^$14^16^1214$4
Answers (6-10):
6) C
7) A
8) D
9) E
10) B
Solutions (6-10):

6)
Code for consonant and vowels are:
Consonant B C D F G H J K L M N P Q R S T V W X Y Z

Code 3 6 9 12 15 18 21 24 27 3 6 9 12 15 18 21 24 27 3 6 9

Vowel A E I O U
Code @ # $ % &
Code of “ANDHERI” will be @6918#15$

Now condition number I and V will be applied to word ANDHERI and after that the code of the word ANDHERI
will be @69@#15$ and 86918#158

7)
Code for consonant and vowels are:
Consonant B C D F G H J K L M N P Q R S T V W X Y Z

Code 3 6 9 12 15 18 21 24 27 3 6 9 12 15 18 21 24 27 3 6 9

Vowel A E I O U
Page 1106 of 1334
Subscribe The Xpress Video Course & Mock Test Package for Bank & Insurance Exams
If there are any suggestions/ errors in our PDFs Feel Free to contact us via this email: admin@exampundit.in
Ultra Practice Bundle PDF
SBI Clerk/ RBI Assist. Mains – Reasoning
Code @ # $ % &
Code of “REPLACEMENT” will be 15#927@6#3#621

Now condition number II will be applied to word REPLACEMENT and after that the code of the word
REPLACEMENT will be *#**@6#*#6*

8)
Code for consonant and vowels are:
Consonant B C D F G H J K L M N P Q R S T V W X Y Z

Code 3 6 9 12 15 18 21 24 27 3 6 9 12 15 18 21 24 27 3 6 9

Vowel A E I O U
Code @ # $ % &
Code of “EQUIPMENT” will be #12&$93621

Now condition number III and II will be applied to word EQUIPMENT and after that the code of the word
EQUIPMENT will be #12&$**#6* and ##&#93#621

9)
Code for consonant and vowels are:
Consonant B C D F G H J K L M N P Q R S T V W X Y Z

Code 3 6 9 12 15 18 21 24 27 3 6 9 12 15 18 21 24 27 3 6 9

Vowel A E I O U
Code @ # $ % &
Code of “EAST” will be #@1821

Now condition number IV, II and I will be applied to word EAST and after that the code of the word EAST will
be #@18#, #@18* and #@18#

10)
Code for consonant and vowels are:
Page 1107 of 1334
Subscribe The Xpress Video Course & Mock Test Package for Bank & Insurance Exams
If there are any suggestions/ errors in our PDFs Feel Free to contact us via this email: admin@exampundit.in
Ultra Practice Bundle PDF
SBI Clerk/ RBI Assist. Mains – Reasoning
Consonant B C D F G H J K L M N P Q R S T V W X Y Z

Code 3 6 9 12 15 18 21 24 27 3 6 9 12 15 18 21 24 27 3 6 9

Vowel A E I O U
Code @ # $ % &
Code of “AFRICA” will be @1215$6@

Now condition number V and III will be applied to word AFRICA and after that the code of the word AFRICA
will be @@15@6@ and 81215$68

Answers (11-15):

11) B
12) D
13) A
14) D
15) C

Solutions (11-15):

11) A
Code for consonant and vowels are:
Consonant B C D F G H J K L M N P Q R S T V W X Y Z

Code 2 4 6 8 10 12 14 16 18 2 4 6 8 10 12 14 16 18 2 4 6

Vowel A E I O U
Code $ % ^ & *
Code of “BOTTLE” will be 2&141418%

Now condition number I will be applied to word BOTTLE and after that the code of the word BOTTLE will be
%&1414182

Page 1108 of 1334


Subscribe The Xpress Video Course & Mock Test Package for Bank & Insurance Exams
If there are any suggestions/ errors in our PDFs Feel Free to contact us via this email: admin@exampundit.in
Ultra Practice Bundle PDF
SBI Clerk/ RBI Assist. Mains – Reasoning
12)
Code for consonant and vowels are:
Consonant B C D F G H J K L M N P Q R S T V W X Y Z
Code 2 4 6 8 10 12 14 16 18 2 4 6 8 10 12 14 16 18 2 4 6

Vowel A E I O U
Code $ % ^ & *
Code of “AMERICA” will be $2%10^4$

Now condition number II and V will be applied to word AMERICA and after that the code of the word
AMERICA will be $*%10^*$ and %2%10%4%

13)
Code for consonant and vowels are:
Consonant B C D F G H J K L M N P Q R S T V W X Y Z

Code 2 4 6 8 10 12 14 16 18 2 4 6 8 10 12 14 16 18 2 4 6

Vowel A E I O U
Code $ % ^ & *
Code of “AMAZON” will be $2$6&4

Now condition number III will be applied to word AMAZON and after that the code of the word AMAZON will
be $2$6&$

14)
Code for consonant and vowels are:
Consonant B C D F G H J K L M N P Q R S T V W X Y Z

Code 2 4 6 8 10 12 14 16 18 2 4 6 8 10 12 14 16 18 2 4 6

Vowel A E I O U
Code $ % ^ & *
Page 1109 of 1334
Subscribe The Xpress Video Course & Mock Test Package for Bank & Insurance Exams
If there are any suggestions/ errors in our PDFs Feel Free to contact us via this email: admin@exampundit.in
Ultra Practice Bundle PDF
SBI Clerk/ RBI Assist. Mains – Reasoning
Code of “SANITIZER” will be 12$4^14^6%10

Now condition number IV and V will be applied to word SANITIZER and after that the code of the word
SANITIZER will be ^$4^^^6%^ and 12%4%14%6%10

15)
Code for consonant and vowels are:
Consonant B C D F G H J K L M N P Q R S T V W X Y Z

Code 2 4 6 8 10 12 14 16 18 2 4 6 8 10 12 14 16 18 2 4 6

Vowel A E I O U
Code $ % ^ & *
Code of “CONGO” will be 4&410&

Now condition number I and V will be applied to word CONGO and after that the code of the word CONGO
will be 4%410% and &&4104

Answers (16-20):

16) A
17) D
18) A
19) C
20) A

Solutions (16-20):

16)
Code for consonant and vowels are:
Consonant B C D F G H J K L M N P Q R S T V W X Y Z

Page 1110 of 1334


Subscribe The Xpress Video Course & Mock Test Package for Bank & Insurance Exams
If there are any suggestions/ errors in our PDFs Feel Free to contact us via this email: admin@exampundit.in
Ultra Practice Bundle PDF
SBI Clerk/ RBI Assist. Mains – Reasoning
Code 3 6 9 12 15 18 21 24 27 3 6 9 12 15 18 21 24 27 3 6 9

Vowel A E I O U
Code @ # $ % &
Code of “ENGAGEMENT” will be #615@15#3#621

Now condition number I will be applied to word ENGAGEMENT and after that the code of the word
ENGAGEMENT will be 15615@15#3#615

17)
Code for consonant and vowels are:
Consonant B C D F G H J K L M N P Q R S T V W X Y Z
Code 3 6 9 12 15 18 21 24 27 3 6 9 12 15 18 21 24 27 3 6 9

Vowel A E I O U
Code @ # $ % &
Code of “ESTONIA” will be #1821%6$@

Now condition number II, IV and V will be applied to word ESTONIA and after that the code of the word
ESTONIA will be #S#S#%S#$@, #189%6$@ and #021%0$@

18)
Code for consonant and vowels are:
Consonant B C D F G H J K L M N P Q R S T V W X Y Z
Code 3 6 9 12 15 18 21 24 27 3 6 9 12 15 18 21 24 27 3 6 9

Vowel A E I O U
Code @ # $ % &
Code of “MIDDLE” will be 3$9927#

Now condition number III will be applied to word MIDDLE and after that the code of the word MIDDLE will
be 9$99279

19)

Page 1111 of 1334


Subscribe The Xpress Video Course & Mock Test Package for Bank & Insurance Exams
If there are any suggestions/ errors in our PDFs Feel Free to contact us via this email: admin@exampundit.in
Ultra Practice Bundle PDF
SBI Clerk/ RBI Assist. Mains – Reasoning
Code for consonant and vowels are:
Consonant B C D F G H J K L M N P Q R S T V W X Y Z

Code 3 6 9 12 15 18 21 24 27 3 6 9 12 15 18 21 24 27 3 6 9

Vowel A E I O U
Code @ # $ % &
Code of “ERITREA” will be #15$2115#@

Now condition number IV and II will be applied to word ERITREA and after that the code of the word ERITREA
will be #9$99#@ and #15$2115#@

20)
Code for consonant and vowels are:
Consonant B C D F G H J K L M N P Q R S T V W X Y Z
Code 3 6 9 12 15 18 21 24 27 3 6 9 12 15 18 21 24 27 3 6 9

Vowel A E I O U
Code @ # $ % &
Code of “PATHOLOGY” will be 9@2118%27%156

Now condition number V will be applied to word PATHOLOGY and after that the code of the word
PATHOLOGY will be S#@S#S#%S#%S#S#

Answers (21-25):

21) C
22) D
23) C
24) E
25) D

Solutions (21-25):

Page 1112 of 1334


Subscribe The Xpress Video Course & Mock Test Package for Bank & Insurance Exams
If there are any suggestions/ errors in our PDFs Feel Free to contact us via this email: admin@exampundit.in
Ultra Practice Bundle PDF
SBI Clerk/ RBI Assist. Mains – Reasoning
21)
Code for consonant and vowels are:
Consonants B C D F G H J K L M N P Q R S T V W X Y Z

Code 11 13 15 17 19 21 23 25 27 11 13 15 17 19 21 23 25 27 11 13 15

Vowel A E I O U
Code # $ @ % &
Code of “ODESSA” will be %15$2121#

Now condition number I and V will be applied to word ODESSA and after that the code of the word ODESSA
will be 2115$2121# and 115$21211

22)
Code for consonant and vowels are:
Consonants B C D F G H J K L M N P Q R S T V W X Y Z

Code 11 13 15 17 19 21 23 25 27 11 13 15 17 19 21 23 25 27 11 13 15

Vowel A E I O U
Code # $ @ % &
Code of “ASIA” will be #21@#

Now condition number II and V will be applied to word ASIA and after that the code of the word ASIA will be
#P@# and 121@1

23)
Code for consonant and vowels are:
Consonants B C D F G H J K L M N P Q R S T V W X Y Z

Code 11 13 15 17 19 21 23 25 27 11 13 15 17 19 21 23 25 27 11 13 15

Vowel A E I O U
Code # $ @ % &
Code of “MAPUTO” will be 11#15&23%
Page 1113 of 1334
Subscribe The Xpress Video Course & Mock Test Package for Bank & Insurance Exams
If there are any suggestions/ errors in our PDFs Feel Free to contact us via this email: admin@exampundit.in
Ultra Practice Bundle PDF
SBI Clerk/ RBI Assist. Mains – Reasoning

Now condition number III will be applied to word MAPUTO and after that the code of the word MAPUTO will
be 11##&23#

24)
Code for consonant and vowels are:
Consonants B C D F G H J K L M N P Q R S T V W X Y Z

Code 11 13 15 17 19 21 23 25 27 11 13 15 17 19 21 23 25 27 11 13 15

Vowel A E I O U
Code # $ @ % &
Code of “AUTO” will be #&23%

Now condition number III, IV and V will be applied to word AUTO and after that the code of the word AUTO
will be #&&&, 1&231 and #&23%

25)
Code for consonant and vowels are:
Consonants B C D F G H J K L M N P Q R S T V W X Y Z

Code 11 13 15 17 19 21 23 25 27 11 13 15 17 19 21 23 25 27 11 13 15

Vowel A E I O U
Code # $ @ % &
Code of “ANTARCTICA” will be #1323#191323@13#

Now condition number I, III and V will be applied to word ANTARCTICA and after that the code of the word
ANTARCTICA will be 191323#191323@13#, #1313#191323@1313 and 11323#191323@131

Answers (26-30):

26) D
27) E
28) E
29) A
Page 1114 of 1334
Subscribe The Xpress Video Course & Mock Test Package for Bank & Insurance Exams
If there are any suggestions/ errors in our PDFs Feel Free to contact us via this email: admin@exampundit.in
Ultra Practice Bundle PDF
SBI Clerk/ RBI Assist. Mains – Reasoning
30) D

Solutions (26-30):

26)
Consonants B C D F G H J K L M N P Q R S T V W X Y Z

Code 10 12 14 16 18 20 22 24 26 10 12 14 16 18 20 22 24 26 10 12 14

Vowel A E I O U
Code & @ % # $
Code of “CONVERSATION” will be 12#1224@1820&22%#12

Now condition number I and V will be applied to word CONVERSATION and after that the code of the word
CONVERSATION will be @#1224@1820&22%#12 and 2#1224@1820&22%#2

27)
Consonants B C D F G H J K L M N P Q R S T V W X Y Z

Code 10 12 14 16 18 20 22 24 26 10 12 14 16 18 20 22 24 26 10 12 14

Vowel A E I O U
Code & @ % # $
Code of “PRICING” will be 1418%12%1218

Now condition number II, III, IV and V will be applied to word PRICING and after that the code of the word
PRICING will be AA%A%AA, 14181412%1214, 1418%12%1218 and 218%12%122

28)
Consonants B C D F G H J K L M N P Q R S T V W X Y Z

Code 10 12 14 16 18 20 22 24 26 10 12 14 16 18 20 22 24 26 10 12 14

Vowel A E I O U
Code & @ % # $
Page 1115 of 1334
Subscribe The Xpress Video Course & Mock Test Package for Bank & Insurance Exams
If there are any suggestions/ errors in our PDFs Feel Free to contact us via this email: admin@exampundit.in
Ultra Practice Bundle PDF
SBI Clerk/ RBI Assist. Mains – Reasoning
Code of “SHARJAH” will be 2020&1822&20

Now condition number III, IV and V will be applied to word SHARJAH and after that the code of the word
SHARJAH will be 2020&1822&20, 2020201822&20 and 220&1822&2

29)
Consonants B C D F G H J K L M N P Q R S T V W X Y Z

Code 10 12 14 16 18 20 22 24 26 10 12 14 16 18 20 22 24 26 10 12 14

Vowel A E I O U
Code & @ % # $
Code of “FREIGHT” will be 1618@%182022

Now condition number II, III, IV and V will be applied to word FREIGHT and after that the code of the word
FREIGHT will be 1618@%181822, AA@%AAA, 161816%182016 and 218@%18202

30)
Consonants B C D F G H J K L M N P Q R S T V W X Y Z

Code 10 12 14 16 18 20 22 24 26 10 12 14 16 18 20 22 24 26 10 12 14

Vowel A E I O U
Code & @ % # $
Code of “TABLET” will be 22&1026@22

Now condition number I and V will be applied to word TABLET and after that the code of the word TABLET
will be @&1026@22 and 2&1026@2

Answers (31-35):

31) A
32) E
33) B
34) E
35) D
Page 1116 of 1334
Subscribe The Xpress Video Course & Mock Test Package for Bank & Insurance Exams
If there are any suggestions/ errors in our PDFs Feel Free to contact us via this email: admin@exampundit.in
Ultra Practice Bundle PDF
SBI Clerk/ RBI Assist. Mains – Reasoning

Solutions (31-35):

31)
Consonants B C D F G H J K L M N P Q R S T V W X Y Z

Code 11 13 15 17 19 21 23 25 27 11 13 15 17 19 21 23 25 27 11 13 15

Vowel A E I O U
Code # $ @ % &
Code of “OCTOBER” will be %1323%11$19

Now condition number I will be applied to word OCTOBER and after that the code of the word OCTOBER will
be %1323%11$%

32)
Consonants B C D F G H J K L M N P Q R S T V W X Y Z

Code 11 13 15 17 19 21 23 25 27 11 13 15 17 19 21 23 25 27 11 13 15

Vowel A E I O U
Code # $ @ % &
Code of “CLAVAM” will be 1327#25#11

Now condition number II will be applied to word CLAVAM and after that the code of the word CLAVAM will
be 1327#25#11

33)
Consonants B C D F G H J K L M N P Q R S T V W X Y Z

Code 11 13 15 17 19 21 23 25 27 11 13 15 17 19 21 23 25 27 11 13 15

Vowel A E I O U
Code # $ @ % &
Code of “QUOTE” will be 17&%23$
Page 1117 of 1334
Subscribe The Xpress Video Course & Mock Test Package for Bank & Insurance Exams
If there are any suggestions/ errors in our PDFs Feel Free to contact us via this email: admin@exampundit.in
Ultra Practice Bundle PDF
SBI Clerk/ RBI Assist. Mains – Reasoning

Now condition number III will be applied to word QUOTE and after that the code of the word QUOTE will be
&&%23&

34)
Consonants B C D F G H J K L M N P Q R S T V W X Y Z

Code 11 13 15 17 19 21 23 25 27 11 13 15 17 19 21 23 25 27 11 13 15

Vowel A E I O U
Code # $ @ % &
Code of “AUSTRALIA” will be #&212319#27@#

Now condition number II and V will be applied to word AUSTRALIA and after that the code of the word
AUSTRALIA will be XX212319X27XX and #&333#3@#

35)
Consonants B C D F G H J K L M N P Q R S T V W X Y Z

Code 11 13 15 17 19 21 23 25 27 11 13 15 17 19 21 23 25 27 11 13 15

Vowel A E I O U
Code # $ @ % &
Code of “BOOKING” will be 11%%25@1319

Now condition number II and V will be applied to word BOOKING and after that the code of the word
BOOKING will be 11%%25@1319 and 11XX25X1319

Answers (36-40):

36) C
37) E
38) E
39) A
40) D

Page 1118 of 1334


Subscribe The Xpress Video Course & Mock Test Package for Bank & Insurance Exams
If there are any suggestions/ errors in our PDFs Feel Free to contact us via this email: admin@exampundit.in
Ultra Practice Bundle PDF
SBI Clerk/ RBI Assist. Mains – Reasoning
Solutions (36-40):

36)
Consonants B C D F G H J K L M N P Q R S T V W X Y Z

Code 11 22 33 13 26 39 52 17 34 51 68 19 38 57 76 95 21 42 63 84 1

Vowel A E I O U
Code & @ % # $
Code of “CAREER” will be 22&57@@57

Now condition number I and V will be applied to word CAREER and after that the code of the word CAREER
will be @&57@@57 and 2&57@@2

37)
Consonants B C D F G H J K L M N P Q R S T V W X Y Z

Code 11 22 33 13 26 39 52 17 34 51 68 19 38 57 76 95 21 42 63 84 1

Vowel A E I O U
Code & @ % # $
Code of “PRESS” will be 1957@7676

Now condition number II, III, IV and V will be applied to word PRESS and after that the code of the word
PRESS will be 1957@AA, 1957197619, 1957@7676 and 257@762

38)
Consonants B C D F G H J K L M N P Q R S T V W X Y Z

Code 11 22 33 13 26 39 52 17 34 51 68 19 38 57 76 95 21 42 63 84 1

Vowel A E I O U
Code & @ % # $
Code of “PRODUCT” will be 1957#33$2295

Page 1119 of 1334


Subscribe The Xpress Video Course & Mock Test Package for Bank & Insurance Exams
If there are any suggestions/ errors in our PDFs Feel Free to contact us via this email: admin@exampundit.in
Ultra Practice Bundle PDF
SBI Clerk/ RBI Assist. Mains – Reasoning
Now condition number I, II, III, IV and V will be applied to word PRODUCT and after that the code of the word
PRODUCT will be $57#33$2295, 1957#33$A95, 19571933$2219 and 1957#33$2295

39)
Consonants B C D F G H J K L M N P Q R S T V W X Y Z

Code 11 22 33 13 26 39 52 17 34 51 68 19 38 57 76 95 21 42 63 84 1

Vowel A E I O U
Code & @ % # $
Code of “CHARGE” will be 2239&5726@

Now condition number IV will be applied to word CHARGE and after that the code of the word CHARGE will
be 2239&5726@

40)
Consonants B C D F G H J K L M N P Q R S T V W X Y Z

Code 11 22 33 13 26 39 52 17 34 51 68 19 38 57 76 95 21 42 63 84 1

Vowel A E I O U
Code & @ % # $
Code of “COMMODITY” will be 22#5151#33%9584

Now condition number I, II and V will be applied to word COMMODITY and after that the code of the word
COMMODITY will be ##5151#33%9584, A#5151#33%95A and 2#5151#33%952

Answers (41-45):

41) A
42) C
43) D
44) C
45) C

Solutions (41-45):
Page 1120 of 1334
Subscribe The Xpress Video Course & Mock Test Package for Bank & Insurance Exams
If there are any suggestions/ errors in our PDFs Feel Free to contact us via this email: admin@exampundit.in
Ultra Practice Bundle PDF
SBI Clerk/ RBI Assist. Mains – Reasoning

41)
Consonants B C D F G H J K L M N P Q R S T V W X Y Z

Code 10 12 14 16 18 20 22 24 26 10 12 14 16 18 20 22 24 26 10 12 14

Vowel A E I O U
Code & @ % # $
Code of “ASHDOD” will be &202014#14

Now condition number I will be applied to word ASHDOD and after that the code of the word ASHDOD will be
20202014#20

42)
Consonants B C D F G H J K L M N P Q R S T V W X Y Z

Code 10 12 14 16 18 20 22 24 26 10 12 14 16 18 20 22 24 26 10 12 14

Vowel A E I O U
Code & @ % # $
Code of “QUALITY” will be 16$&26%2212

Now condition number II and V will be applied to word QUALITY and after that the code of the word QUALITY
will be 9$&9%99 and 16BB26B2212

43)
Consonants B C D F G H J K L M N P Q R S T V W X Y Z

Code 10 12 14 16 18 20 22 24 26 10 12 14 16 18 20 22 24 26 10 12 14

Vowel A E I O U
Code & @ % # $
Code of “CLASSMATE” will be 1226&202010&22@

Page 1121 of 1334


Subscribe The Xpress Video Course & Mock Test Package for Bank & Insurance Exams
If there are any suggestions/ errors in our PDFs Feel Free to contact us via this email: admin@exampundit.in
Ultra Practice Bundle PDF
SBI Clerk/ RBI Assist. Mains – Reasoning
Now condition number III and V will be applied to word CLASSMATE and after that the code of the word
CLASSMATE will be &26&202010&22& and 1226B202010B22B

44)
Consonants B C D F G H J K L M N P Q R S T V W X Y Z

Code 10 12 14 16 18 20 22 24 26 10 12 14 16 18 20 22 24 26 10 12 14

Vowel A E I O U
Code & @ % # $
Code of “OFFICE” will be #1616%12@

Now condition number IV will be applied to word OFFICE and after that the code of the word OFFICE will be
#22%2@

45)
Consonants B C D F G H J K L M N P Q R S T V W X Y Z

Code 10 12 14 16 18 20 22 24 26 10 12 14 16 18 20 22 24 26 10 12 14

Vowel A E I O U
Code & @ % # $
Code of “LOOKOUT” will be 26##24#$22

Now condition number II and V will be applied to word OFFICE and after that the code of the word OFFICE
will be 9##9#$9 and 26BB24BB22

Answers (46-50):

46) A
47) C
48) E
49) C
50) D

Solutions (46-50):
Page 1122 of 1334
Subscribe The Xpress Video Course & Mock Test Package for Bank & Insurance Exams
If there are any suggestions/ errors in our PDFs Feel Free to contact us via this email: admin@exampundit.in
Ultra Practice Bundle PDF
SBI Clerk/ RBI Assist. Mains – Reasoning

46)

Consonants B C D F G H J K L M N P Q R S T V W X Y Z

Code 11 22 33 13 26 39 52 17 34 51 68 19 38 57 76 95 21 42 63 84 1

Vowel A E I O U
Code & @ % # $
Code of “EXPORT” will be @6319#5795

Now condition number I will be applied to word EXPORT and after that the code of the word EXPORT will be
636319#5763

47)
Consonants B C D F G H J K L M N P Q R S T V W X Y Z

Code 11 22 33 13 26 39 52 17 34 51 68 19 38 57 76 95 21 42 63 84 1

Vowel A E I O U
Code & @ % # $
Code of “TERMINAL” will be 95@5751%68&34

Now condition number II will be applied to word TERMINAL and after that the code of the word TERMINAL
will be 95@5751%9&9

48)
Consonants B C D F G H J K L M N P Q R S T V W X Y Z

Code 11 22 33 13 26 39 52 17 34 51 68 19 38 57 76 95 21 42 63 84 1

Vowel A E I O U
Code & @ % # $
Code of “FREE” will be 1357@@

Page 1123 of 1334


Subscribe The Xpress Video Course & Mock Test Package for Bank & Insurance Exams
If there are any suggestions/ errors in our PDFs Feel Free to contact us via this email: admin@exampundit.in
Ultra Practice Bundle PDF
SBI Clerk/ RBI Assist. Mains – Reasoning
Now condition number III will be applied to word FREE and after that the code of the word FREE will be
@57@@

49)
Consonants B C D F G H J K L M N P Q R S T V W X Y Z

Code 11 22 33 13 26 39 52 17 34 51 68 19 38 57 76 95 21 42 63 84 1

Vowel A E I O U
Code & @ % # $
Code of “AUSTRIA” will be &$769557%&

Now condition number IV and V will be applied to word AUSTRIA and after that the code of the word AUSTRIA
will be &$29557%& and BB769557BB

50)
Consonants B C D F G H J K L M N P Q R S T V W X Y Z

Code 11 22 33 13 26 39 52 17 34 51 68 19 38 57 76 95 21 42 63 84 1

Vowel A E I O U
Code & @ % # $
Code of “CALLING” will be 22&3434%6826

Now condition number II and V will be applied to word CALLING and after that the code of the word CALLING
will be 9&99%99 and 22B3434B6826

Page 1124 of 1334


Subscribe The Xpress Video Course & Mock Test Package for Bank & Insurance Exams
If there are any suggestions/ errors in our PDFs Feel Free to contact us via this email: admin@exampundit.in
Ultra Practice Bundle PDF
SBI Clerk/ RBI Assist. Mains – Reasoning

Box Type Coding Decoding


Direction (1-5): In a certain code language the words (d) qaka
are written in the form of numbers which are specific
to a certain code. These codes can be traced from the (e) None of these
table with the respective number. For e.g. the code for 3. What is the code for ‘orange’?
the number ‘10’ will be ‘ka’, code for the number ‘01’
will be ‘so’ and so on. (a) go
0 1 2 (b) ma
0 go so zo
1 ka la pa (c) ta
2 ma ta qa
(d) zo

‘wait party shock maid’ is coded as ‘22000211’ (e) None of these

‘wait solid master soft’ is coded as ‘10220120’ 4. What is the code for ‘shock’ in the given code
language?
‘shock solid try soft’ is coded as ‘02101220’
(a) zo
‘soft orange try maid ’is coded as ‘20211211’
(b) go
1. What is the code for ‘master’?
(c) qa
(a) ka
(d) ma
(b) la
(e) None of these
(c) qa
5. What is the code for ‘wait solid in the given code
(d) so language?
(e) None of these (a) lata

2. Which of the following may be the code for ‘soft (b) maso
try’?
(c) gozo
(a) pa ma
(d) qaka
(b) qa la
(e) None of these
(c) sola

Page 1125 of 1334


Subscribe The Xpress Video Course & Mock Test Package for Bank & Insurance Exams
If there are any suggestions/ errors in our PDFs Feel Free to contact us via this email: admin@exampundit.in
Ultra Practice Bundle PDF
SBI Clerk/ RBI Assist. Mains – Reasoning
Direction (6-10): In a certain code language the words (b) wn ma
are written in the form of numbers which are specific
to a certain code. These codes can be traced from the (c) ja ra
table with the respective number. For e.g. the code for (d) xu yw
the number ‘10’ will be ‘yw’, code for the number ‘01’
will be ‘ba’ and so on. (e) None of these

8. What is the code for ‘fool’in the given code


language?

(a) ma
0 1 2 3
0 pa ba ta ja (b) wn
1 yw ra na ma (c) la
2 la xu ka wn
(d) yw

(e) None of these


‘fool and best gain pins’ is coded as ‘20 12 01 23 00’
9. What is the code for ‘pins’in the given code
‘loud joker flower pins’ is coded as ‘21 03 13 00’ language?
‘red and flower money’ is coded as ‘22 12 13 10’ (a) wn

‘fool pen solid red joker’ is coded as ‘20 11 02 22 03’ (b) ta

6. What is the code for ‘joker’in the given code (c) ba


language?
(d) pa
(a) xu (e) None of these
(b) ra 10. If ‘loudgain’is coded as ‘xuwn’ then what is the
(c) wn code for ‘best’?
(a) ba
(d) ja
(b) yw
(e) None of these
(c) xu
7. What is the code for ‘money flower’in the given
code language? (d) ta
(a) ma yw
(e) None of these

Page 1126 of 1334


Subscribe The Xpress Video Course & Mock Test Package for Bank & Insurance Exams
If there are any suggestions/ errors in our PDFs Feel Free to contact us via this email: admin@exampundit.in
Ultra Practice Bundle PDF
SBI Clerk/ RBI Assist. Mains – Reasoning
Direction (11-15): In a certain code language the (e) Can’t be determine
words are written in the form of numbers which are
specific to a certain code. These codes can be traced 13. What is the code for ‘fail’in the given code
from the table with the respective number. For e.g. language?
the code for the number ‘10’ will be ‘sw’, code for the
number ‘01’ will be ‘bx’ and so on. (a) sw

(b) mq
0 1 2 3
0 xz bx mz mq (c) mz
1 sw rp kw za (d) bx
2 pq ax ta na
(e) None of these

14. What is the code for ‘like people’ in the given code
like rock real people retail’ is coded as ‘00 21 11 13 23’ language?
‘day detail allow people’ is coded as ’10 20 23 22’ (a) mqsq

‘fail retail day actual’ is coded as ’02 21 22 01’ (b) naxz

‘like virtual suit fail allow’ is coded as ’00 12 03 02 20' (c) pqax

11. What is the code for ‘retail’in the given code (d) narp
language?
(e) None of these
(a) ax
15. If ‘rock’ is coded as ‘rp’ then what is the code for
(b) sw ‘real’?
(a) xz
(c) kw
(b) mq
(d) za
(c) sw
(e) None of these
(d) za
12. Which of the following may be the code for
‘suit’? (e) None of these
(a) za
Direction (16-20): In a certain code language the words
(b) xz are written in the form of numbers which are specific
to a certain code. These codes can be traced from the
(c) bx table with the respective number. For e.g. the code for
(d) mq
Page 1127 of 1334
Subscribe The Xpress Video Course & Mock Test Package for Bank & Insurance Exams
If there are any suggestions/ errors in our PDFs Feel Free to contact us via this email: admin@exampundit.in
Ultra Practice Bundle PDF
SBI Clerk/ RBI Assist. Mains – Reasoning
the number ‘10’ will be ‘ba’, code for the number ‘01’ 18. What is the code for ‘floor’in the given code
will be ‘sa’ and so on. language?

0 1 2 3 (a) ma
0 la sa ka pa
(b) ra
1 ba ma na ra
2 ta fa ha qa (c) ha

(d) ta

‘dance luxary loose win award’ is coded as ’23 10 01 02 (e) Can’t be determine
21’
19. What is the code for ‘luxarymeeting’ in the given
‘plus hope award meeting’ is coded as ’00 03 21 11’ code language?
(a) ha fa
‘flat luxary meeting round’ is coded as ’13 10 11 12’
(b) ba ma
‘dance show floorflathope’ is coded as ’23 20 22 13 03’
(c) raba
16. What is the code for ‘hope’in the given code
language? (d) tama

(a) ka (e) None of these

(b) qa 20. If ‘loose’ is coded as ‘sa’ then what is the code for
‘win’?
(c) fa
(a) la
(d) la
(b) pa
(e) None of these
(c) ba
17. What is the code for ‘award’in the given code
(d) ka
language?
(e) None of these
(a) la
Direction (21-25): In a certain code language the
(b) ba
words are written in the form of numbers which are
(c) sa specific to a certain code. These codes can be traced
from the table with the respective number. For e.g.
(d) fa the code for the number ‘10’ will be ‘ea’, code for the
number ‘01’ will be ‘ba’ and so on.
(e) None of these
Page 1128 of 1334
Subscribe The Xpress Video Course & Mock Test Package for Bank & Insurance Exams
If there are any suggestions/ errors in our PDFs Feel Free to contact us via this email: admin@exampundit.in
Ultra Practice Bundle PDF
SBI Clerk/ RBI Assist. Mains – Reasoning
0 1 2 3
0 aa ba ca da (a) ba

1 ea fa ga ha (b) ja
2 ia ja ka la
(c) ea

(d) ha
‘ground pitch ball free cricket’ is coded as 02 13 22 23
00’ (e) Can’t be determine

‘wide out cricket decision’is coded as ’21 20 00 10’ 24. What is the code for ‘pitch’ in the given code
language?
‘umpire pitch decision play’ is coded as ’03 13 10 01’ (a) ha

‘ground final match umpire out’ is coded as ’02 11 12 03 (b) ga


20’
(c) ia
21. What is the code for ‘ground’in the given code
language? (d) la

(a) la (e) None of these

(b) fa 25. If ‘final’ is coded as ‘fa’ then what is the code for
‘match’?
(c) ca (a) ka
(d) ba (b) ea
(e) None of these (c) aa
22. What is the code for ‘decision’in the given code (d) ga
language?
(a) aa (e) None of these

(b) fa Direction (26-30): In a certain code language the


words are written in the form of numbers which are
(c) ha specific to a certain code. These codes can be traced
from the table with the respective number. For e.g.
(d) la
the code for the number ‘10’ will be ‘dq’, code for the
(e) None of these number ‘01’ will be ‘bq’ and so on.

23. What is the code for ‘wide’in the given code 0 1 2


language? 0 aq bq cq
Page 1129 of 1334
Subscribe The Xpress Video Course & Mock Test Package for Bank & Insurance Exams
If there are any suggestions/ errors in our PDFs Feel Free to contact us via this email: admin@exampundit.in
Ultra Practice Bundle PDF
SBI Clerk/ RBI Assist. Mains – Reasoning
1 dq eq fq (d) fq eq
2 gq hq iq
(e) None of these

‘lack down hunter defer’ is coded as ‘22 00 02 11’ 29.What is the code for ‘mankind’in the given code
language?
‘hunter sound tension surface’ is coded as ‘02 10 12 20’ (a) hq
‘lack sound mankind surface’ is coded as ‘10 22 01 20’ (b) eq
‘surface often tension defer’is coded as ‘20 21 12 11’ (c) bq
26. What is the code for ‘lack’in the given code (d) dq
language?
(e) None of these
(a) cq
30. What is the code for ‘hunter’ in the given code
(b) aq language?
(c) dq
(a) gq
(d) bq
(b) cq
(e) iq
(c) hq
27. What is the code for ‘surface’in the given code
(d) bq
language?
(a) gq (e) None of these
(b) hq Direction (31-35): In a certain code language the
words are written in the form of numbers which are
(c) eq
specific to a certain code. These codes can be traced
(d) cq from the table with the respective number. For e.g.
the code for the number ‘10’ will be ‘jm’, code for the
(e) None of these number ‘01’ will be ‘dm’ and so on.
28. What is the code for ‘tension defer’in the given
code language?
(a) aq dq 0 1 2 3

(b) gq cq 0 am dm fm hm
1 jm km om qm
(c) dq bq
2 rm tm um zm

Page 1130 of 1334


Subscribe The Xpress Video Course & Mock Test Package for Bank & Insurance Exams
If there are any suggestions/ errors in our PDFs Feel Free to contact us via this email: admin@exampundit.in
Ultra Practice Bundle PDF
SBI Clerk/ RBI Assist. Mains – Reasoning
(c) rm

‘supplying incentive turn stay Higher’ is coded as ‘20 12 (d) zm


01 23 00’
(e) None of these
‘industries squeeze sector Higher’ is coded as ‘21 03 13
00’ 34. What is the code for ‘higherfirms’ in the given
code language?
‘unorganized incentive sector firms’ is coded as ‘22 12 (a) rmqm
13 10’
(b) zmam
‘supplying out in unorganized squeeze’ is coded as ‘20
11 02 22 03’ (c) fmkm

31. What is the code for ‘squeeze’in the given code (d) amjm
language? (e) None of these
(a) dm 35. If ‘stay’ is coded as ‘dm’ then what is the code for
(b) rm ‘turn’?
(a) hm
(c) zm
(b) km
(d) hm
(c) zm
(e) None of these
(d) om
32. What is the code for ‘unorganized sector’in the
given code language?‘? (e) None of these
(a) um qm Direction (36-40): In a certain code language the
(b) hmzm words are written in the form of numbers which are
specific to a certain code. These codes can be traced
(c) tmrm from the table with the respective number. For e.g.
the code for the number ‘10’ will be ‘hw’, code for the
(d) dmhm
number ‘01’ will be ‘dw’ and so on.
(e) None of these
0 1 2 3
33. What is the code for ‘industries’in the given code 0 aw dw fw gw
language?
1 hw jw kw mw
(a) tm
2 nw ow qw sw
(b) qm

Page 1131 of 1334


Subscribe The Xpress Video Course & Mock Test Package for Bank & Insurance Exams
If there are any suggestions/ errors in our PDFs Feel Free to contact us via this email: admin@exampundit.in
Ultra Practice Bundle PDF
SBI Clerk/ RBI Assist. Mains – Reasoning
‘should so that clerk subject’ is coded as ‘00 21 11 13 (d) fw
23’
(e) None of these
‘mark preparation selection clerk’ is coded as ‘10 20 23
22’ 39. What is the code for ‘that’ in the given code
language?
‘soon subject mark exam’ is coded as ‘02 21 22 01’ (a) sw

‘should utilize keep soon selection’ is coded as ‘00 12 03 (b) jw


02 20'
(c) ow
36. What is the code for ‘subject’in the given code
language? (d) mw

(a) mw (e) Can’t be determine

(b) gw 40. If ‘keep’ is coded as ‘gw’ then what is the code for
‘utilize’?
(c) nw (a) jw

(d) ow (b) kw
(e) None of these (c) fw
37. Which of the following may be the code for (d) dw
‘examsoon’?
(a) owsw (e) None of these

(b) jwmw Direction (41-45): In a certain code language the


words are written in the form of numbers which are
(c) gwdw specific to a certain code. These codes can be traced
from the table with the respective number. For e.g.
(d) dwfw
the code for the number ‘10’ will be ‘ee’, code for the
(e) None of these number ‘01’ will be ‘bb’ and so on.

38. What is the code for ‘preparation’in the given 0 1 2 3


code language? 0 aa bb cc dd
1 ee ff gg hh
(a) hw
2 mm nn qq ss
(b) ow
‘rural affect economy growth capital’ is coded as ‘23 20
(c) jw 22 13 03’

Page 1132 of 1334


Subscribe The Xpress Video Course & Mock Test Package for Bank & Insurance Exams
If there are any suggestions/ errors in our PDFs Feel Free to contact us via this email: admin@exampundit.in
Ultra Practice Bundle PDF
SBI Clerk/ RBI Assist. Mains – Reasoning
‘Rise capital urban country’ is coded as ‘00 03 21 11’ (e) None of these

‘rural enterprises do both urban’ is coded as ‘23 10 01 44. What is the code for ‘enterprises’in the given code
02 21’ language?
(a) bb
‘growth enterprises country large’ is coded as ‘13 10 11
12’ (b) dd

41. What is the code for ‘rural’in the given code (c) ee
language?
(d) ff
(a) qq
(e) None of these
(b) mm
45. If ‘affect’ is coded as ‘mm’ then what is the code
(c) ss for ‘economy’?
(a) gg
(d) dd
(b) cc
(e) None of these
(c) dd
42. What is the code for ‘growth’in the given code
language? (d) bb
(a) qq
(e) None of these
(b) ee
Direction (46-50): In a certain code language the
(c) mm words are written in the form of numbers which are
specific to a certain code. These codes can be traced
(d) hh from the table with the respective number. For e.g.
(e) None of these the code for the number ‘10’ will be ‘so’, code for the
number ‘01’ will be ‘aq’ and so on.

43. What is the code for ‘capital’in the given code 0 1 2 3


language? 0 zy aq lo mo
1 so go ho to
(a) dd
2 mh gz sb ak
(b) mm

(c) ee ‘fund sober monetary latest’ is coded as ’03 13 10 01’


(d) qq

Page 1133 of 1334


Subscribe The Xpress Video Course & Mock Test Package for Bank & Insurance Exams
If there are any suggestions/ errors in our PDFs Feel Free to contact us via this email: admin@exampundit.in
Ultra Practice Bundle PDF
SBI Clerk/ RBI Assist. Mains – Reasoning
‘across sober purpose bank protracted’ is coded as 02 13 48. What is the code for ‘monetary’in the given code
22 23 00’ language?

‘annual raise protracted monetary’ is coded as ’21 20 00 (a) zy


10’
(b) ho
‘across board latin fund raise’ is coded as ’02 11 12 03
20’ (c) so

46. What is the code for ‘sober’in the given code (d) to
language?
(e) None of these
(a) mh
49. What is the code for ‘protracted’ in the given code
(b) ho language?
(a) aq
(c) go
(b) zy
(d) to
(c) lo
(e) None of these
(d) go
47. What is the code for ‘annual raise’in the given
code language? (e) None of these

(a) mhgz 50. If ‘purpose’ is coded as ‘sb’ then what is the code
for ‘bank’?
(b) zy lo (a) mh
(c) aqmh (b) aq
(d) ho to (c) ak
(e) None of these (d) so

(e) None of these

Box Type Coding Decoding– Answer and Explanation


Word Code Number
Explanation in detail: wait qa 22
Party go 00
Page 1134 of 1334
Subscribe The Xpress Video Course & Mock Test Package for Bank & Insurance Exams
If there are any suggestions/ errors in our PDFs Feel Free to contact us via this email: admin@exampundit.in
Ultra Practice Bundle PDF
SBI Clerk/ RBI Assist. Mains – Reasoning
shock zo 02 SOLUTION(11-15):
maid la 11
Solid ka 10 Explanation in detail:
Master so 01
Soft ma 20 Word Code Number
Try pa 12 like xz 00
orange ta 21 actual bx 01
1.d fail mz 02
Virtual/suit mq/kw 03/12
2.a detail sw 10
Rock/real rp/za 11/13
3.c Virtual/suit mq/kw 12/03
Rock/real rp/za 13/11
4.a allow pq 20
retail ax 21
5.d day ta 22
people na 23
SOLUTION(6-10):

Explanation in detail:
11.a

12.e
Word Code Number
13.c
pins pa 00
Gain/best wn/ba 23/01 14.b
Solid/pen ta/ra 02/11
joker ja 03 15.d
money yw 10
Solid/pen ta/ra 02/11 SOLUTION(16-20):
and na 12
flower ma 13 Explanation in detail:
fool la 20
loud xu 21 Word Code Number
red ka 22 Plus la 00
Gain/best wn/ba 23/01 Loose/win sa/ka 01/02
6.d Loose/win ka/sa 02/01
Hope pa 03
7.a luxary ba 10
meeting ma 11
8.c round na 12
flat ra 13
9.d Show/floor ta/ha 20/22
award fa 21
10.a
Page 1135 of 1334
Subscribe The Xpress Video Course & Mock Test Package for Bank & Insurance Exams
If there are any suggestions/ errors in our PDFs Feel Free to contact us via this email: admin@exampundit.in
Ultra Practice Bundle PDF
SBI Clerk/ RBI Assist. Mains – Reasoning
Show/floor ha/ta 22/20 Explanation in detail:
dance qa 23
Word Code Number
lack iq 22
16.e Down aq 00
hunter cq 02
17.d defer eq 11
Sound dq 10
18.e Mankind bq 01
Surface gq 20
19.b
Tension fq 12
20.d often hq 21

SOLUTION(21-25):
26.e
Explanation in detail:
27.a
Word Code Numbe
r 28.d
cricket aa 00 29.c
play ba 01
ground ca 02 30.b
umpire da 03
decision ea 10 SOLUTION(31-35):
final/match fa/ga 11/12
final/match fa/ga 12/11 Explanation in detail:

pitch ha 13 Word Code Number


out ia 20 Higher am 00
wide ja 21 Stay/turn dm/zm 23/01
ball/free ka/la 22/23 In/out fm/km 02/11
ball/free ka/la 23/22 squeeze hm 03
21.c firms jm 10
In/out km/fm 02/11
22.e incentive om 12
sector qm 13
23.b
supplying rm 20
24.a industries tm 21
unorganized um 22
25.d Stay/turn zm/dm 23/01
31.d
SOLUTION(26-30):
32.a
Page 1136 of 1334
Subscribe The Xpress Video Course & Mock Test Package for Bank & Insurance Exams
If there are any suggestions/ errors in our PDFs Feel Free to contact us via this email: admin@exampundit.in
Ultra Practice Bundle PDF
SBI Clerk/ RBI Assist. Mains – Reasoning
33.a country ff 11
large gg 12
34.d growth hh 13
Affect/economy mm/qq 20/22
35.c urban nn 21
SOLUTION(36-40): Affect/economy qq/mm 22/20
rural ss 23
Explanation in detail:

Word Code Number 41.c


should aw 00
42.d
exam dw 01
soon fw 02 43.a
Utilize/keep gw/kw 03/12
preparation hw 10 44.c
So/that jw/mw 11/13
Utilize/keep gw/kw 12/03 45.e
So/that jw/mw 13/11
selection nw 20
subject ow 21
mark qw 22 SOLUTION(46-50):
clerk sw 23
Explanation in detail:

Word Code Numbe


36.d
r
37.d protracted zy 00
latest aq 01
38.a across lo 02
fund mo 03
39.e monetary so 10
board/latin ho/go 11/12
40.b
board/latin go/ho 12/11
SOLUTION(41-45): sober to 13
raise mh 20
Explanation in detail: annual gz 21
purpose/bank sb/ak 22/23
Word Code Number
purpose/bank sb/ak 23/22
Rise aa 00
Do /both bb/cc 01/02
Do /both cc/bb 02/01 46.d
Capital dd 03
enterprises ee 10 47.a
Page 1137 of 1334
Subscribe The Xpress Video Course & Mock Test Package for Bank & Insurance Exams
If there are any suggestions/ errors in our PDFs Feel Free to contact us via this email: admin@exampundit.in
Ultra Practice Bundle PDF
SBI Clerk/ RBI Assist. Mains – Reasoning
48.c 50.c

49.b

Download Seating Arrangement Practice Questions PDF


Get More Reasoning Practice Questions PDF
Input- Output Part 1
Step 4 is the last step of the rearrangement. As per the
Directions 1-5: Study the given information carefully rules followed in the above steps, find out the answers to
and answer the Direction: Study the following each of the following questions.
information carefully and answer the questions given
below. Input: 13 99 42 19 66 87 32 70

A number arrangement machine, when given an input, 1) Which of the following element is between 3* and
rearranges it following a particular rule. The following is &3 in the last step?
the illustration of the input and steps of the arrangement. a. None

Condition: b. 6*

i) If an odd number comes at the even-numbered c. 7*


position, then the first digit of the number is changed to
d. &3
&.
e. &9
ii) If an Even number comes at the odd-numbered
position, then the Second digits of the number changed 2) Which of the following steps are not used in the
to *. rearrangement?

a. 4* 3* &9 &3 &9 &7 7* 6*


Input: 34 25 56 98 11 77 68 83
b. 19 &3 4* 66 3* 7* &9 &7
Step 1: 11 34 25 56 77 68 83 98
c. 13 42 19 66 87 32 7* &9
Step 2: 25 &1 3* 56 77 68 9* &3
d. 19 4* &3 3* 66 7* &9 &7
Step 3: 3* &5 &1 56 6* 9* &3 &7
e. None of these
Step 4: 5* 3* &5 &1 9* &3 &7 6*
Page 1138 of 1334
Subscribe The Xpress Video Course & Mock Test Package for Bank & Insurance Exams
If there are any suggestions/ errors in our PDFs Feel Free to contact us via this email: admin@exampundit.in
Ultra Practice Bundle PDF
SBI Clerk/ RBI Assist. Mains – Reasoning
3) Which of the following element is immediately following a particular pattern in each step. The following
preceded by 4* in step 2? is an illustration of input and rearrangement.

a. 19 Input: glitter situation younger contain visiting hostel


ginger
b. 66

c. &3 Step I: younger contain visiting ginger situation glitter


hostel
d. 6*
Step II: 41 25 27 14 55
e. None 14 20

4) In step 3, which element is second to the right of Step III: 66 52 41 69 69


the number which is third from the left end? 34
a. 6* Step IV: 19 12 20 30 22
b. &5
Step V: 7 -8 -10 8
c. 77
Step VI: -56 80 -80
d. 9*
Step VII: 24 0
e. 56
Step VIII: 24
5) Which among the following is the last step of the
rearrangement?

a. 4* 3* &6 &3 &7 &9 7* 6* Input: biscuit decide hideout silent essential author
gesture
b. 4* 3* &9 &3 &9 &6 7* 7*
6) Which element is third to the left in step IV?
c. 4* 3* &3 &9 &3 &7 7* 6*

d. 4* 3* &9 &3 &9 &7 7* 6* a. 21

e. None of these b. 32

Directions: Study the following information carefully c. 24


and answer the given questions:
d. 120
A word and number arrangement device when given an
input line of words and numbers rearranges them e. None of these

Page 1139 of 1334


Subscribe The Xpress Video Course & Mock Test Package for Bank & Insurance Exams
If there are any suggestions/ errors in our PDFs Feel Free to contact us via this email: admin@exampundit.in
Ultra Practice Bundle PDF
SBI Clerk/ RBI Assist. Mains – Reasoning
7) What is the sum of the all the digits of the number b) 46
in final step?
c) 43
a) 04
d) 44
b) 06
e) None of these
c) 13
Directions: Study the following information carefully
d) 12 and answer the given questions:

e) None of these A word and number arrangement device when given an


input line of words and numbers rearranges them
8) Which element is fifth from the right in step 1? following a particular pattern in each step. The following
is an illustration of input and rearrangement.
a) Author
Input: Three person bought prime laptops
b) 89
Step I: 25 75 315 45 15
c) Silent
Step II: 34 64 204 54 04
d) Hideout
Step III: 04 34 54 64 204
e) None of these
Step IV: 30 20 10 140
9) Which step number is the following output?
Step V: 3 2 1 5
‘21 32 24 12 17’
Step VI: X Y Z V
a) Step I
Step VI is the last step of the given input
b) Step II
Input: seven persons attend eight meeting
c) Step IV
11) What is the position of ‘54’ in the step-3?
d) Step III
a) fourth from the left
e) Step V
b) second from the left
10) What is sum of all the numbers of penultimate
step? c) fourth from the right
a) 49 d) third from the left
Page 1140 of 1334
Subscribe The Xpress Video Course & Mock Test Package for Bank & Insurance Exams
If there are any suggestions/ errors in our PDFs Feel Free to contact us via this email: admin@exampundit.in
Ultra Practice Bundle PDF
SBI Clerk/ RBI Assist. Mains – Reasoning
e) second from the right a) 2

12) What is the sum of all the numbers in step 5? b) None

a) 16 c) 1

b) 15 d) 3

c) 10 e) 4

d) 11 Directions: Study the following information carefully


and answer the given questions:
e) 12
A word and number arrangement device when given an
13) How many elements are there between ‘334’ and input line of words and numbers rearranges them
‘270’ in step 3? following a particular pattern in each step. The following
is an illustration of input and rearrangement.
a) Two
Input: articulate reticent incorrigible adroit martinet
b) One
Step I: adroit articulate incorrigible martinet reticent
c) Three
Step II: 6 10 12 8 8
d) None
Step III: 60 120 96 64
e) There is no such step
Step IV: -60 24 32
14) Which step number is the following output?
Step V: -36 56
30 20 10 270
Step VI: 20
a) Step I
Step VII: T
b) Step II
Input: drag gorgeous energetic wonder harper
c) Step IV
16) Which of the following represents position of “24”
d) Step III
in Step IV of the new input?
e) Step V
a) Second from the left
15) How many vowels are presents in the last step of
b) Third from the right
the given input?
Page 1141 of 1334
Subscribe The Xpress Video Course & Mock Test Package for Bank & Insurance Exams
If there are any suggestions/ errors in our PDFs Feel Free to contact us via this email: admin@exampundit.in
Ultra Practice Bundle PDF
SBI Clerk/ RBI Assist. Mains – Reasoning
c) Third from the left e) Step V

d) First from the left 20) What is difference between fifth number from left
and fifth number from right in Step II?
e) First from the right
a) 5
17) Which number would be at third position from
left in Step IV? b) 6

a) 24 c) 3

b) -36 d) 4

c) -24 e) 2

d) 12 Directions 21-25: Directions: A word and number


arrangement machine when given an input line of
e) 36 words and numbers rearranges then following a
particular rule in each step. The following is an
18) Which alphabet appears in the last step of the
illustration of an input rearrangement.
output?

a) L Input: Current True 28 30 Given 24 Situation 48


Triangles 52
b) M
Step I: Triangles Current True 28 30 Given Situation 48
c) X 52 24

d) Q Step II: Triangles Situation Current True 30 Given 48 52


24 28
e) J
Step III: Triangles Situation True Current Given 48 52
19) Which step number is the following output?
24 28 30
“36 72 48 36”
Step IV: Triangles Situation True Current Given 52 24
a) Step I 28 30 48

b) Step II Step V: Triangles Situation True Current Given 24 28 30


48 52
c) Step IV
Step VI: 18 09 18 21 09 24 28 30 48 52
d) Step III
Page 1142 of 1334
Subscribe The Xpress Video Course & Mock Test Package for Bank & Insurance Exams
If there are any suggestions/ errors in our PDFs Feel Free to contact us via this email: admin@exampundit.in
Ultra Practice Bundle PDF
SBI Clerk/ RBI Assist. Mains – Reasoning
Step VII: 09 09 09 03 09 06 10 03 12 07 c. Figure

Step VIII: 03 03 06 07 09 09 09 09 10 12 d. Medicine

e. None of these
Step VIII is the output of the above input
24) What is the sum of first four numbers from left
Input: Nest Dinner 26 Figure 78 34 38 Medicine Summer end in last step?
56
a. 12
21) In the Step V, which element will be sixth to the
b. 18
right of seventh element from the right end for the
input given below? c. 14

a. 78 d. 10

b. Nest e. 20

25) Which element is third to the left of third element


c. 56
from the right end in step III?
d. 38 a. 26

e. None of these b. 78

22) What would be the sum of all the numbers in the c. Medicine
penultimate step? d. Dinner
a. 95 e. None of these
b. 83
26-30 Directions: Study the following information
c. 75 carefully and Directions: Read the given information
carefully and answer the questions.
d. 88
A three digit number arrangement machine when given
e. None of these
an input line of numbers rearranges them following a
23) Which number is second to the right of third particular rule in each step. The following is an
element from left end in step IV? illustration of input and rearrangement.

a. 78 Input: 347 736 479 238 547 632


b. Nest
Step I: 256 627 568 329 456 723
Page 1143 of 1334
Subscribe The Xpress Video Course & Mock Test Package for Bank & Insurance Exams
If there are any suggestions/ errors in our PDFs Feel Free to contact us via this email: admin@exampundit.in
Ultra Practice Bundle PDF
SBI Clerk/ RBI Assist. Mains – Reasoning
Step II: 723 627 568 456 329 256 e. None of these

Step III: 237 267 568 456 239 256 28) What is the sum of all the digits of the 3rd
number from the left end in step II of the given
Step IV: 23 44 53 34 29 32 input?

Step V: 23 29 32 34 44 53 a. 15

Step V is the last step of the above input. As per rules b. 14


followed in the above steps, find out in each of the given
questions the appropriate steps for the given input. c. 16

Input for the question: d. 18

“825 392 647 528 724 438” e. None of these

26) What is the absolute difference between first two 29) What is the addition of first two numbers (from
numbers (from the left end) in Step IV of the given the left end) in Step V of the given input?
input?
a. 48
a. 4
b. 54
b. 14
c. 51
c. 11
d. 59
d. 8
e. None of these
e. None of these
30) What is the sum of all the odd numbers in the last
27) What is the sum of all the prime numbers in the step of the given input?
last step of the given input?
a. 170
a. 70
b. 205
b. 94
c. 187
c. 87
d. 199
d. 99
Page 1144 of 1334
Subscribe The Xpress Video Course & Mock Test Package for Bank & Insurance Exams
If there are any suggestions/ errors in our PDFs Feel Free to contact us via this email: admin@exampundit.in
Ultra Practice Bundle PDF
SBI Clerk/ RBI Assist. Mains – Reasoning
e. None of these Input: biscuit decide hideout silent essential author
gesture
31-35) Direction: Study the following information
carefully and answer the questions given below. 31) What is the sum of all numbers in the penultimate
step of given input?
A word and number arrangement machine, when given a
particular input, rearranges it following a particular rule. a. 82

The following is the illustration of the input and the steps b. 85


of arrangement.
c. 76
Input: glitter situation younger contain visiting hostel
ginger d. 75

Step I: ginger glitter situation younger contain visiting e. 80


hostel
32) Which word is the fifth from the left in step III of
given input?
Step II: ginger hostel glitter situation younger contain
visiting a. Author

Step III: ginger hostel contain glitter situation younger b. Gesture


visiting
c. Essential
Step IV: ginger hostel contain glitter younger situation d. Decide
visiting
e. Hideout
Step V: ginger hostel contain glitter younger visiting
situation 33) Which number is the third to the right of sixth
number from right end in step VI of given input?
Step VI: 07 08 20 20 14 22 01
a. 03
Step VII: 08 10 22 22 16 24 02 b. 20

Step VIII: 02 08 10 16 22 22 24 c. 21

And Step VIII is the last step of the rearrangement as the d. 19


desired arrangement is obtained. As per the rules e. None of the above
followed in the above steps, answer the following
questions for the given input. 34) Which of the following is the middle word in step
III of given input?
Page 1145 of 1334
Subscribe The Xpress Video Course & Mock Test Package for Bank & Insurance Exams
If there are any suggestions/ errors in our PDFs Feel Free to contact us via this email: admin@exampundit.in
Ultra Practice Bundle PDF
SBI Clerk/ RBI Assist. Mains – Reasoning
a. Silent 216 36 108 12
b. Biscuit
339 113 452 226
c. Hideout
After player codes are generated, players are arranged in
d. Gesture ascending order as per player code and the first four
players are kept in team P and last four players are kept
e. Author in team Q:
35) Which word is the third to the left of third word Work out the same for the following runs of the players:
from right end in step IV of given input?
315, 234, 195, 423, 414, 87, 162, 378
a. Biscuit
36) What is the player code of the player, who has 87
b. Gesture runs?

c. Author a. 65
d. Hideout
b. 130
e. Decide
c. 58
36-40) Direction: Study the following information
carefully and answer the questions given below. d. 105
Eight batsmen with different runs in a match are
e. None of these
arranged in two teams P and Q. Runs of each batsman
are coded as per the following examples: 37) What is the difference of the highest and the
lowest player codes in team P?
Runs Step 1 Step 2 Player codes
a. 14
342 57 171 19
b. 23
123 41 164 82
c. 19
279 93 372 186
d. 17
324 54 162 18
e. None of these
126 21 63 7
38) If a player scores 75 runs, what will be his code?
189 63 252 126
a. 150
Page 1146 of 1334
Subscribe The Xpress Video Course & Mock Test Package for Bank & Insurance Exams
If there are any suggestions/ errors in our PDFs Feel Free to contact us via this email: admin@exampundit.in
Ultra Practice Bundle PDF
SBI Clerk/ RBI Assist. Mains – Reasoning
b. 70

c. 25

d. 50

e. None of these

39) What is the sum of the 2nd highest and the lowest
player codes in Team Q?

a. 188

b. 340

c. 268
As per the rules followed in the above steps, find out in
d. 329 each of the following questions the appropriate steps for
the given input.
e. None of these
Input – 24389 21952 19863 15625 13824
40) The player with 3rd highest code (including both 10648
the teams) has how many runs?
41) Which of the following is the 3rd number from left
a. 414 in the penultimate step?

b. 378 a. 56

c. 195 b. 58

d. 87 c. 50

e. None of these d. 52

e. None of these
41-45) Directions: Study the given information to 42) What will be final step of the given input?
answer the questions based on it.
a. 2 5 3 0 2
A number arrangement machine when given an input
line of numbers rearranges them following a particular b. 2 3 5 0 2
rule. The following is an illustration of input and re-
arrangement. c. 2 5 3 2 0

d. 2 5 3 0 2

Page 1147 of 1334


Subscribe The Xpress Video Course & Mock Test Package for Bank & Insurance Exams
If there are any suggestions/ errors in our PDFs Feel Free to contact us via this email: admin@exampundit.in
Ultra Practice Bundle PDF
SBI Clerk/ RBI Assist. Mains – Reasoning
e. None of these b. 46

43) What is the multiplication of the number which is c. 55


third from left and which fifth from right in the last
step? d. 52

a. One e. 49

b Two 45) What is the sum of the elements which are 2nd
from the right end of step II and 3rd from the right in
c. Four step III?

d. Three a. 103

e. More than Three b. 104

44) Which of the following element is 3rd to the right c. 101


of the element which is 2nd from the left end in the
step II? d. 109

a. 57 e. None of these

Input Output Part 1- Answers and Explantion


1. E ii) If an Even number comes at the odd-numbered
position, then the Second digits of the number changed
2. D to *.
3. C Operation applied:
4. A Numbers are arranged in increasing order from left end
5. D and in decreasing order from right end simultaneously.

Input: 34 25 56 98 11 77 68 83

Step 1: 11 34 25 56 77 68 83 98
Solution 1-5
Step 2: 25 &1 3* 56 77 68 9* &3
Given condition:
Step 3: 3* &5 &1 56 6* 9* &3 &7
i) If an odd number comes at the even-numbered
position, then the first digit of the number is changed to Step 4: 5* 3* &5 &1 9* &3 &7 6*
&. Now for the input,

Page 1148 of 1334


Subscribe The Xpress Video Course & Mock Test Package for Bank & Insurance Exams
If there are any suggestions/ errors in our PDFs Feel Free to contact us via this email: admin@exampundit.in
Ultra Practice Bundle PDF
SBI Clerk/ RBI Assist. Mains – Reasoning
Input: 13 99 42 19 66 87 32 70 Step 6: Product of the two adjacent numbers given in
step 5.
Step 1: 13 42 19 66 87 32 7* &9
Step 7: sum of the two adjacent numbers given in step 6.
Step 2: 19 &3 4* 66 3* 7* &9 &7

Step 3: 3* &9 &3 4* 6* &9 &7 7* Step 8: 2nd number from the left is to be subtracted
from the first from the left
Step 4: 4* 3* &9 &3 &9 &7 7* 6*
Step-8 is the final step

Now given
6. c
Input: biscuit decide hideout silent essential author
7. b gesture

8. d Step I: silent author hideout biscuit gesture decide


essential
9. c
Step II: 14 37 50 39 31 19
10. d 20

Step III: 51 87 89 70 50 39
Solution 6-10: Step IV: 21 32 24 12 17
In the given illustration: Step V: -11 8 -12 05
Step I: arrange the words from the left side which comes Step VI: -88 96 -60
last in the dictionary and then the word which comes first
in the English dictionary. Now, the second last word and Step VII: 08 36
second word will be arranged and so on.
Step VIII: -24
Step 2: each word is replaced by sum of the numerical
position of vowels in the English alphabetical order.

Step 3: sum of the two adjacent numbers given in step 2. 11. d

Step 4: sum of all the digits of the two adjacent numbers 12. b
given in step 3.
13. e
Step 5: Subtraction of 2nd number from the left from 1st
and 3rd number from 2nd and so on. 14. c

Page 1149 of 1334


Subscribe The Xpress Video Course & Mock Test Package for Bank & Insurance Exams
If there are any suggestions/ errors in our PDFs Feel Free to contact us via this email: admin@exampundit.in
Ultra Practice Bundle PDF
SBI Clerk/ RBI Assist. Mains – Reasoning
15. b

Solution 11-15: 16. a

In the given illustration: 17. d

Step I: Each word is replaced by the multiplication of 18. c


numerical positions of vowels in the English alphabetical
series. 19. d

Step 2: Each odd digit of the number should be replaced 20. e


by -1 and each even digit of the number should be
Solution 16-20:
replaced by +1.
In the given illustration:
Step 3: Arrange all the numbers in the ascending order.
Step I: arrange the words in an alphabetical order.
Step 4: Difference of the two adjacent numbers given in
step 3( Subtraction of lower number from higher Step 2: each word is replaced by number of letters in that
number). word.
Step 5: Sum of the digits of the numbers given in step 4. Step 3: Product of two adjacent numbers given in step 2.
Step 6: The alphabet which have the positional value Step 4: difference between the two adjacent numbers
same as the number given in step 5. given in step 3.
Step 6 is the last step Step 5: sum of the two adjacent numbers given in step 4.
Input: Seven persons attend eight meeting Step 6: sum of the two adjacent numbers given in step 5.
Step I: 25 75 15 45 225 Step 7: the alphabet which have the positional value
same as the number given in step 6.
Step II: 34 64 04 54 334
Step 7 is the last step
Step III: 04 34 54 64 334
Now given
Step IV: 30 20 10 270
Input: drag gorgeous energetic wonder harper
Step V: 3 2 1 9
Step I: drag energetic gorgeous harper wonder
Step VI: X Y Z R
Step II: 4 9 8 6 6
Step VI is the last step of the given input

Page 1150 of 1334


Subscribe The Xpress Video Course & Mock Test Package for Bank & Insurance Exams
If there are any suggestions/ errors in our PDFs Feel Free to contact us via this email: admin@exampundit.in
Ultra Practice Bundle PDF
SBI Clerk/ RBI Assist. Mains – Reasoning
Step III: 36 72 48 36 Step II: Summer Nest Dinner Figure 78 38 Medicine 56
26 34
Step IV: -36 24 12
Step III: Summer Nest Dinner Figure 78 Medicine 56 26
Step V: -12 36 34 38
Step VI: 24
Step IV: Summer Nest Dinner Medicine Figure 78 26 34
Step VII: X 38 56

Step V: Summer Nest Dinner Medicine Figure 26 34 38


56 78
21. A
Step VI: 21 05 09 05 09 26 34 38 56 78
22. B

23. C Step VII: 03 05 09 05 09 08 07 11 11 15

24. E Step VIII: 03 05 05 07 08 09 09 11 11 15

25. B 26. D
Solution 21-25
27. B
In the illustration from Step I-V, the words are being
28. B
arranged according to highest number, which is obtained
by the sum of numbers representing the first and last 29.D
alphabet of the word, from left to right and
simultaneously the numbers are being arranged one by 30. B
one in ascending order. In Step VI, after rearrangement
of words and numbers, we write the number as the Solutions 26-30
number representing second letter of the word. In step
VII, we add the digits of the numbers. In step VIII, we The rearrangement takes place in the following ways:
arranged the numbers in increasing order from left to
In Step I, all the odd digits of each numbers are
right.
subtracted by 1 and all the even digits of each numbers
are added by 1.
Input: Nest Dinner 26 Figure 78 34 38 Medicine Summer
56 In Step II, all the numbers in step I are arranged in
decreasing order from left to right.
Step I: Summer Nest Dinner Figure 78 34 38 Medicine
In Step III, all the digits of all the numbers are arranged
56 26
in increased order (within each number)

Page 1151 of 1334


Subscribe The Xpress Video Course & Mock Test Package for Bank & Insurance Exams
If there are any suggestions/ errors in our PDFs Feel Free to contact us via this email: admin@exampundit.in
Ultra Practice Bundle PDF
SBI Clerk/ RBI Assist. Mains – Reasoning
In Step IV, the hundredth digit is added to the product of Step VIII: Numbers are arranged in ascending order from
unit digit and tenth digit of each numbers. left end.
In Step V, numbers in step IV are arranged in increasing Input: biscuit decide hideout silent essential author
order from left to right. gesture
Input: 825 392 647 528 724 438
Step I: author biscuit decide hideout silent essential
Step I: 934 283 756 439 635 529 gesture
Step II: 934 756 635 529 439 283 Step II: author decide biscuit hideout silent essential
Step III: 349 567 356 259 349 238 gesture

Step IV: 39 47 33 47 39 26 Step III: author decide silent biscuit hideout essential
gesture
Step V: 26 33 39 39 47 47
Step IV: author decide silent biscuit gesture hideout
essential
31. C
Step V: author decide silent biscuit gesture hideout
32. E
essential
33. B
Step VI: 01 04 19 03 20 05 14
34. B
Step VII: 02 06 20 04 22 06 16
35. E
Step VIII: 02 04 06 06 16 20 22
Solution 31-35

The logic followed here is:


36. C
Step I - Step V: Words are arranged in increasing order
37. A
of number of letters. If two or more words have same
number of letters then those words are arranged in 38. D
alphabetical order of first letters.
39. C
Step VI: If a word has even number of letters then it will
be replaced by the number representing first letter and if 40. C
a word has odd number of letters then it will be replaced Solution 36-40
by number representing middle letter.
As per the pattern if the runs of the players are an even
Step VII: One is added to odd numbers and two is added number then in step 1, run scored is divided by 6 then in
to even numbers. step 2, the value obtained in step 1 is multiplied by 3 and

Page 1152 of 1334


Subscribe The Xpress Video Course & Mock Test Package for Bank & Insurance Exams
If there are any suggestions/ errors in our PDFs Feel Free to contact us via this email: admin@exampundit.in
Ultra Practice Bundle PDF
SBI Clerk/ RBI Assist. Mains – Reasoning
finally in step 3, the value obtained in step 2 is divided 414 69 207 23
by 9, to obtain the player code.
If the runs of the players is an odd number then in step 1, Team Q
run scored is divided by 3 then in step 2, the value
Runs Step 1 Step 2 Player codes
obtained in step 1 is multiplied by 4 and finally in step 3,
the value obtained in step 2 is divided by 2, to obtain the
87 29 116 58
player code

Runs Step 1 Step 2 Player codes 195 65 260 130

315 105 420 210 315 105 420 210

234 39 117 13 423 141 564 282

195 65 260 130


Solution 41-45
423 141 564 282
Input: 24389 21952 19863 15625 13824 10648
414 69 207 23 Step I: All the numbers in the given input are cubes of
two digit numbers so, take the cube root of all the
87 29 116 58 numbers in step-1

162 27 81 9 Step II: Sum of first and 2nd number, 2nd and 3rd number
and 3rd and 4th number and so on from the left side of
378 63 189 21 step I

Distribution of players according to the player codes in Step III: Arrange all the numbers in the ascending order.
team P and team Q, Step IV: Difference of digit of all numbers. Lower
number is subtracted from higher number.

Team P

Runs Step 1 Step 2 Player codes

162 27 81 9

234 39 117 13

378 63 189 21

Page 1153 of 1334


Subscribe The Xpress Video Course & Mock Test Package for Bank & Insurance Exams
If there are any suggestions/ errors in our PDFs Feel Free to contact us via this email: admin@exampundit.in
Ultra Practice Bundle PDF
SBI Clerk/ RBI Assist. Mains – Reasoning
41. D

42. D

43. E

44. B

45. C

Download Seating Arrangement Practice Questions PDF


Get More Reasoning Practice Questions PDF
Input Output Part 2
Directions: Study the given information to answer the
questions based on it.

A number arrangement machine when given an input


line of numbers rearranges them following a particular
rule. The following is an illustration of input and re-
arrangement.

Page 1154 of 1334


Subscribe The Xpress Video Course & Mock Test Package for Bank & Insurance Exams
If there are any suggestions/ errors in our PDFs Feel Free to contact us via this email: admin@exampundit.in
Ultra Practice Bundle PDF
SBI Clerk/ RBI Assist. Mains – Reasoning
As per the rules followed in the above steps, find out in e. None of these
each of the following questions the appropriate steps for
the given input. 4) Which of the following element is 2nd to the right
of the element which is 6th from the right end in the
Input – 262 153 237 308 156 261 482 555 step III?
1) Which of the following is the middle number in the a. 10
last step?
b. 13
a. 26
c. 26
b. 18
d. 18
c. 10
e. None of these
d. 30
5) What is the sum of the elements which are 3rd
e. None of these from the right end of step II and III?
2) What will be resultant if 3rd element from the left a. 163
end in step II is multiplied to 2nd element from the
right end in step III? b. 174

a. 164 c. 173

b. 1510 d. 189

c. 2310 e. None of these

d. 60 Directions: Study the given information carefully and


answer the given questions. An input-output is given
e. None of these in different steps. Some logical operations are done in
each step. No logical operation is repeated in next
3) If all the numbers in step IV are arranged in the
step.
ascending order from the left to right, then how many
numbers remain unchanged considering their
original positions?

a. One

b. Two

c. Four

d. Three
Page 1155 of 1334
Subscribe The Xpress Video Course & Mock Test Package for Bank & Insurance Exams
If there are any suggestions/ errors in our PDFs Feel Free to contact us via this email: admin@exampundit.in
Ultra Practice Bundle PDF
SBI Clerk/ RBI Assist. Mains – Reasoning
7) Which of the following combinations represents the
2nd table of step III?

a. 1, 1

b. 1, 4

c. 2, 4

d. 1, 3

e. None of these

8) What is the product of 1st digit of 2nd table of step


II and 2nd digit of 3rd table of step I?

a. 20

b. 21

c. 24

As per the rules followed in the steps given above, find d. 30


out in each of the following questions the appropriate
e. None of these
step for the given input.
9) What is the difference between the 1st digit and
2nd digit of step IV of the given input?

a. 1

b. 2
6) Which of the following combinations represents the
first digit of 2nd table of step II and second digit of c. 3
first table of step III?
d. 7
a. 4, 1
e. None of these
b. 3, 2
10) What is the sum of 2nd digit of 2nd tables in step
c. 2, 1 I and 2nd digit of 2nd table of step III?
d. 5, 3 a. 5
e. None of these
b. 7

Page 1156 of 1334


Subscribe The Xpress Video Course & Mock Test Package for Bank & Insurance Exams
If there are any suggestions/ errors in our PDFs Feel Free to contact us via this email: admin@exampundit.in
Ultra Practice Bundle PDF
SBI Clerk/ RBI Assist. Mains – Reasoning
c. 6 a. 159

d. 4 b. 162

e. None of these c. 48

d. 164

Direction: Read the following given information e. None of these


carefully and answer the questions. An input-output
is given in different steps. Some mathematical 12) Which of the following number is obtained in the
operations are done in each step. No mathematical last step?
operation is repeated in next step.
a. 3

b. 27

c. 64

d. 48

e. 125

13) What is the multiplication of all numbers in


step2?

a. 250

b. 382

c. 135

d. 180
As per the rules followed in the steps given above,
find out in each of the following questions the e. 278
appropriate step for the given input.
14) Which of the following is the step 3?

a. 15, 09

b. 75,20

11) What will be the sum of the numbers in Step 1? c. 12, 25


Page 1157 of 1334
Subscribe The Xpress Video Course & Mock Test Package for Bank & Insurance Exams
If there are any suggestions/ errors in our PDFs Feel Free to contact us via this email: admin@exampundit.in
Ultra Practice Bundle PDF
SBI Clerk/ RBI Assist. Mains – Reasoning
d. 12, 36 As per the rules followed in the steps given above,
find out in each of the following questions the
e. There is no such step. appropriate step for the given input.

15) What is the sum of second block in step 1 and


second digit of a number in second block in step 2?

a. 64
16) Which of the following represent the difference
b. 73
between the first digit of the second number and
c. 72 second digit of the first number in Step 2?

d. 79 a. 9

e. 15 b. 1

Direction: Read the following given information c. 4


carefully and answer the questions. An input-output
d. 3
is given in different steps. Some mathematical
operations are done in each step. No mathematical e. 0
operation is repeated in next step.
17) If the value 8.5 is added to final output, then what
will be the resultant value?

a. 2

b. 19

c. 21.5

d. 19.5

e. 18

18) If the second digit of all blocks in Step I is halved


and then added the half numbers, what will be the
result?

a. 4.5

b. 3
Page 1158 of 1334
Subscribe The Xpress Video Course & Mock Test Package for Bank & Insurance Exams
If there are any suggestions/ errors in our PDFs Feel Free to contact us via this email: admin@exampundit.in
Ultra Practice Bundle PDF
SBI Clerk/ RBI Assist. Mains – Reasoning
c. 6

d. 4

e. 5.5

19) Find the difference of two numbers in Step 3?

a. 76

b. 8

c. 92

d. 75

e. 10
Step 4 is the last step of the rearrangement.
20) Which is the smallest number in Step 1?
As per the rules followed in the above steps, find out the
a. 10 answers to each of the following input

b. 11

c. 12
21) Find the sum of the numbers in step 1?
d. 08
a. 12
e. 06
b. 14
Directions 21-25: Study the given information
carefully and answer the given questions. c. 18
An arrangement machine, when given a particular input, d. 16
rearranges following a particular rule. The following is
the illustration of the input and the steps of the e. None of these
arrangement.
22) What is the division of number which is highest to
the lowest number that occurs in Step 3?

a. 4

b. 9

Page 1159 of 1334


Subscribe The Xpress Video Course & Mock Test Package for Bank & Insurance Exams
If there are any suggestions/ errors in our PDFs Feel Free to contact us via this email: admin@exampundit.in
Ultra Practice Bundle PDF
SBI Clerk/ RBI Assist. Mains – Reasoning
c. 16 An input-output is given in different steps. Some
mathematical operations are done in each step. No
d. 25 mathematical operation is repeated in next step.
e. 36

23) What is the final number in step 4?

a. 6

b. 5

c. 4

d. 9

e. 8

24) If the last step number is multiplied by 4 what


will be the final answer?
As per the rules followed in the steps given above, find
a. 25 out in each of the following questions the appropriate
b.38 step for the given input.

c.36

d. 32
26) Which among the following numbers are in 2nd
e. None of these table in step III of the given input?

25) If all the number of the Step-3 are added then a. 13, 15
what will be the answer?
b. 18, 19
a. 45
c. 34, 19
b.48
d. 23, 17
c.44
e. None of these.
d. 32
27) What are the letters in the final table of the given
e. None of these input?

26-30 Study the given information carefully and a. E, P


answer the given questions.
Page 1160 of 1334
Subscribe The Xpress Video Course & Mock Test Package for Bank & Insurance Exams
If there are any suggestions/ errors in our PDFs Feel Free to contact us via this email: admin@exampundit.in
Ultra Practice Bundle PDF
SBI Clerk/ RBI Assist. Mains – Reasoning
b. C, O Study the given information carefully and answer the
given question-ns.
c. E, C
An input-output is given in different steps. Some
d. P, O mathematical operations are done in each step. No
e. None of these mathematical operation is repeated in next step.

28) What are the letters in the 2nd table in step II of


the given input?

a. P, L

b. R, X

c. T, P

d. Q, L

e. None of these

29) Which letter is in the 2nd part of the 3rd table of


Step I of the given input?

a. V
As per the rules followed in the steps given above, find
b. K out in each of the following questions the appropriate
step for the given input.
c. R

d. W

e. None of these
31) Which among the following numbers are in 1st
30) If all the number of the second last step are added table of the step III?
then what will be the answer?
a. 12, 17
a. 85
b. 12, 16
b. 84
c. 13, 16
c. 88
d. 17, 16
d. 86
e. None of these
e. 82
Page 1161 of 1334
Subscribe The Xpress Video Course & Mock Test Package for Bank & Insurance Exams
If there are any suggestions/ errors in our PDFs Feel Free to contact us via this email: admin@exampundit.in
Ultra Practice Bundle PDF
SBI Clerk/ RBI Assist. Mains – Reasoning
32) Which of the following letters are in 2nd table of d. 56
the step II?
e. 52
a. W, P
Directions: Study the given information carefully and
b. R, Y answer the given questions.
c. J, D An input-output is given in different steps. Some
mathematical operations are done in each step. No
d. I, G
mathematical operation is repeated in next step but it can
e. None of these be repeated with some other mathematical operation (as
multiplication can be used with subtraction in step 1 and
33) Which of the following letters are in step IV? same can be used with addition in step 2).
a. D, E

b. T, Z

c. Y, U

d. I, O

e. D, D

34) Which is 3rd element from the left in step-2?

a. E

b. G

c. I As per the rules followed in the steps given above, find


out in each of the following questions the appropriate
d. J step for the given input.
e. None of these

35) If all the number of the second last step are added
then what will be the answer?
36) Which of the following represents the product of
a. 55 the first digit of the second value and the second digit
of the first value in step II of the given input?
b. 54

c. 58 a. 10

Page 1162 of 1334


Subscribe The Xpress Video Course & Mock Test Package for Bank & Insurance Exams
If there are any suggestions/ errors in our PDFs Feel Free to contact us via this email: admin@exampundit.in
Ultra Practice Bundle PDF
SBI Clerk/ RBI Assist. Mains – Reasoning
b. 25 d. 45

c. 2 e. None of these

d. 35 40) What is the sum of the numbers of step-3?


a. 3
e. 5
b. 6
37) Which of the following combinations represent
the second digit of the third block and the first digit c. 2
of the second block in step I of the given input?
d. 4
a. 3, 2
e. None of these
b. 2, 4
Directions: Study the given information carefully and
c. 4, 2 answer the given question.

d. 2, 2 An input-output is given in different steps. No


mathematical operation is repeated in next step but it can
e. 2, 8 be repeated with some other mathematical operation (as
multiplication can be used with subtraction in step 1 and
38) If the value ‘2’ is multiplied with the final output
same can be used with addition in step 2).
then what will be the resultant value?

a. 18

b. 16

c. 06

d. 12

e. -12

39) Find the difference of two numbers obtained in


Step II?

a. 30

b. 60 As per the rules followed in the steps given above, find


out in each of the following questions the appropriate
c. 21 step for the given input.
Page 1163 of 1334
Subscribe The Xpress Video Course & Mock Test Package for Bank & Insurance Exams
If there are any suggestions/ errors in our PDFs Feel Free to contact us via this email: admin@exampundit.in
Ultra Practice Bundle PDF
SBI Clerk/ RBI Assist. Mains – Reasoning
44) What will be the difference between the sum of
numbers in the second last step and the sum of
numbers in the second step?
41) What is the sum of inverse of the numbers
a. 39
obtained in step 3?
b. 48
a. 87
c. 46
b. 122
d. 42
c. 78
e. None of these
d. 85
45) What is the sum of the numbers in step 2?
e. 135
a. 76
42) What is the difference between the first block
obtained in step 2 and second block obtained in step b. 88
3?
c. 78
a. 12
d. 72
b. 29
e. None of these
c. 26
Directions: Study the given information carefully and
d. 14 answer the given question.
e. 21 Below are input-output, steps responsive mathematical
operations on a set of numbers according to which you
43) What will be the final result of the given input?
need to answer the following questions.
a. 26

b. 88

c. 56

d. 42

e. None of these

Page 1164 of 1334


Subscribe The Xpress Video Course & Mock Test Package for Bank & Insurance Exams
If there are any suggestions/ errors in our PDFs Feel Free to contact us via this email: admin@exampundit.in
Ultra Practice Bundle PDF
SBI Clerk/ RBI Assist. Mains – Reasoning
d. 15

e. None of these

47) How many elements in step 1(unique) are


multiples of 6?

a. 0

b. 1

c. 2

d. 3

e. None of these

48) If all the numbers in step 2 are multiplied with


each other and divided by the answer obtained in step
As per the rules followed in the steps given above, find 3, and the quotient of the same is divided by the
out in each of the following questions the appropriate number obtained in step 3, what's the remainder?
step for the given input.
a. 0

b. 1

c. 2

d. 3

e. None of these
46) Find the difference between sum of numbers
obtained in 1st step and sum of numbers obtained in 49) If the numbers 4 and 5 are reversed in step 2,
all other steps? what is the difference between the old output and new
output?
a. 25
a. 0
b. 35
b. 1
c. 20
c. 2
Page 1165 of 1334
Subscribe The Xpress Video Course & Mock Test Package for Bank & Insurance Exams
If there are any suggestions/ errors in our PDFs Feel Free to contact us via this email: admin@exampundit.in
Ultra Practice Bundle PDF
SBI Clerk/ RBI Assist. Mains – Reasoning
d. 3 b.16

e. None of these c.14

50) What is the sum of the numbers in step 2? d. 13

a.17 e. None of these

Input Output Part 2– Answer and Explanation

.a Step I: Divide all the even numbers by 2 and then


subtract 1 from the obtained numbers and divide all the
2. c odd numbers by 3 and then subtract 1 from the obtained
3. c numbers.

4. b Step II: Sum of first and 2nd number from the left side of
step I then difference of 2nd and 3rd number then sum of
5. c 3rd and 4th number and so on.

Solutions 1-5: Step III: Add all the digits of each number.

Given input is Step IV: Multiply odd number by 2 and even number by
3.
262 153 237 308 156 261 482 555

Page 1166 of 1334


Subscribe The Xpress Video Course & Mock Test Package for Bank & Insurance Exams
If there are any suggestions/ errors in our PDFs Feel Free to contact us via this email: admin@exampundit.in
Ultra Practice Bundle PDF
SBI Clerk/ RBI Assist. Mains – Reasoning
In step IV: Subtract 1st digit of 1st column with the
1st digit of 2nd column and similarly subtract the 2nd digit
of 1st column with 2nd digit of 2nd column of step III.

The final arrangement is as follows:

6. E

7. C

8. E

9. E (c)

10. E

Solutions 6-10:

Step I: First digit of first column is multiplied by the


2nd digit of 4th column and 2nd digit of 1st column is
multiplied by 1st digit of 4th column. 11. A

Similar operation is applied on the 2nd and 3rd column 12. B


with 5th and 6th columns respectively.
13. C
In step II: Addition of 1st digits of all the columns and
14. D
then multiply by 2, similarly addition of 2nd digits of all
the columns and then multiply by 2. 15. B
In step III: Square each digit in first and 2nd column and
Solutions 11-15
then add both the squared number of each column of step
II. Step 1: Add the digits within blocks. Resultant of 1st
block is summed with the resultant of 2nd block and
obtained sum is multiplied with smallest number among
them. In the same way resultant of 3rd block and 4th
Page 1167 of 1334
Subscribe The Xpress Video Course & Mock Test Package for Bank & Insurance Exams
If there are any suggestions/ errors in our PDFs Feel Free to contact us via this email: admin@exampundit.in
Ultra Practice Bundle PDF
SBI Clerk/ RBI Assist. Mains – Reasoning
block is added, resultant number is multiplied with 16. C(d)
smallest number among two and so on.
17. B
Example, consider 1st two blocks
18. C
23 and 73 19. A
Sum of digits of 23 = 5 20. D
Sum of digits of 73 = 10 Solutions 16-20
Then adding both result = 5 + 10 = 15.
Step 1: Numbers in consecutive block is added, then
taking square root of the sum.
Multiplying with smallest number among them = 15 × 5
= 75. And So on.
Example,
Step 2: 1st digits of each block of step 1 is added to a get 59 + 41 = 100
single block and the second digits of each block is added
to form a single number. 41 + 23 = 64

Step 3: Take the difference within the blocks of step 2 23 + 98 = 121 so on


and if the number is even then multiply it with 3, if it is
odd number then multiply with 4. Step 2: Number in 1st block and last block is added then
doubled. Number in 2nd block and 3rd block is added,
Step 4: Larger number is divided by Smaller number and then the number is doubled.
cubing the obtained result to get single number.
Step 3: Sum of step 2 and difference of Step 2 is taken.

Step 4: Larger number is divided by smaller number.

Page 1168 of 1334


Subscribe The Xpress Video Course & Mock Test Package for Bank & Insurance Exams
If there are any suggestions/ errors in our PDFs Feel Free to contact us via this email: admin@exampundit.in
Ultra Practice Bundle PDF
SBI Clerk/ RBI Assist. Mains – Reasoning

21. B 26. C

22. A 27. B

23. D 28. A

24. C 29. B

25. A 30. D

Solution 21-25 Solutions 26-30

Step 1: This step tells us the number of alphabets in Step I: The letter (among the two letters of 1st table)
between the two given letters. which has highest alphabetical number in the
alphabetical series is written in the 1st part of the
Step 2: The arithmetic mean of the two numbers is taken 2nd table. And the letter (among the two letters of
and the corresponding alphabet is inserted as per 4th table) which has highest alphabetical number in the
dictionary order. alphabetical series is written in the 2nd part of the
2nd table.
Step 3: This is the square of the arithmetic mean.
Similarly, the letter (among the two letters of 2nd table)
Step 4: Sum of the two numbers (9 + 36 = 45) and the
which appears last in the alphabetical series, is written in
number is added with itself up to a single digit (4+5 =9).
the 1st part of the 1st table. And the letter (among the
two letters of 3rd table) which appears last in the

Page 1169 of 1334


Subscribe The Xpress Video Course & Mock Test Package for Bank & Insurance Exams
If there are any suggestions/ errors in our PDFs Feel Free to contact us via this email: admin@exampundit.in
Ultra Practice Bundle PDF
SBI Clerk/ RBI Assist. Mains – Reasoning
alphabetical series, is written in the 2nd part of the 1st 32. D
table. Same is repeated for the remaining tables.
33. E
Step II: Write the immediately preceding 4th letters of all
the letters in the corresponding parts of the tables. 34. C

Step III: Sum of the values of the letters of 1st part of 35. C
1st table and 1st part of 2nd table is written in 1st part of
Solutions 31-35
the 1st table. Again, sum of the values of the letters of the
2nd part of 1st and 2nd part of 2nd table is written in Step I: The 2nd letter (among the two letters of 1st
2nd part of the 1st table. The same is repeated for the next table)is written in second part of second table And the
table. 2nd letter (among the two letters of 4th table) is written in
the 1st part of the 2nd table.
Step IV: The corresponding letter (in alphabetical series)
representing the difference of the numbers of both the Similarly, the 2nd letter (among the two letters of
parts of the 1st table is written in the 1st part while the 2nd table) is written in the 2nd part of the 1st table. And
corresponding letter (in alphabetical series) representing the 2nd letter (among the two letters of 3rd table) is
the difference of the numbers of both the parts of the written in the 1st part of the 1st table. Same is repeated
2nd table is written in the 2nd part of the table. for the remaining tables.

The final arrangement will be: Step II: Write each letter according to the sum of place
values (in English alphabetically series) of the letter
which appears in step 1. Example: If in step 1 first table
consists W and P whose place value is 23 and 16 so the
sum of each place value i.e. 2+3=5 which
represents E and 1+6=7 which represents G so the first
table of step II contains E and G in 1st and 2nd part
respectively and same follows on the rest of the tables.
Step III: Sum of the values of the letters of 1st part of 1st
table and 2nd part of the 2nd table and 2nd part of the
1st table and 1st part of 2nd table. Similarly, sum of the
values of the letters of 1st part of 2nd table and 2nd part of
the 3rd table and 2nd part of the 2nd table and 1st part of
3rd table.
Step IV: The corresponding letter (in alphabetical series)
of the difference of the numbers of both the parts of the
1st table is written in the 1st part while the corresponding
letter (in alphabetical series) of the difference of the
31. B
Page 1170 of 1334
Subscribe The Xpress Video Course & Mock Test Package for Bank & Insurance Exams
If there are any suggestions/ errors in our PDFs Feel Free to contact us via this email: admin@exampundit.in
Ultra Practice Bundle PDF
SBI Clerk/ RBI Assist. Mains – Reasoning
numbers of both the parts of the 2nd table is written in the (Second Block) - Multiply the second digit of the first
2nd part of the table. block with the first digit of fourth block. i.e. (3 × 2) = 6.
The final arrangement is as follows: Same procedure is applied for all the blocks.

Step II:

(First block)- Add the first digit of all numbers in step I


and then multiply by 3 and write down the value in
reverse order.

i.e. (9 + 2 + 5) = 16 × 3 = 48.

Reverse of 48 is 84.

(Second block) - Add the second digit of all numbers in


step I and then multiply by 3 and write down the value in
reverse order.

i.e. (6 + 4 + 2) = 12 × 3 = 36.

Reverse of 36 is 63.

Step III:
36. E
(First block) - Subtract the second digit of second block
37. C by the first digit of the first block.

38. D i.e. (8 - 3) = 5.

39. B (Second block)- Subtract the first digit of second block


by the second digit of the first block.
40. B
i.e. (4 - 6) = -2.
Solutions 36-40:
Step IV:
Step I:
Add both numbers.
(First Block) - Multiply the first digit of the first block
with the second digit of the fourth block. i.e. (3 × 3) = 9. i.e. [5 + (-2)] = 3.

Same procedure is applied for the given input:

Page 1171 of 1334


Subscribe The Xpress Video Course & Mock Test Package for Bank & Insurance Exams
If there are any suggestions/ errors in our PDFs Feel Free to contact us via this email: admin@exampundit.in
Ultra Practice Bundle PDF
SBI Clerk/ RBI Assist. Mains – Reasoning
Same process is to be repeated (on second digits of all
three blocks) to get the second block.

Step 3:

In this step, square of first digit of first block and square


of first digit from second block is added and then product
of first digit from both the blocks is subtracted from it
i.e. 32 + 42 - 3 × 4 = 13.

Same process is done on second digits of previous blocks


to get the second block of this step.

Step 4:
41. B
Difference between the sum of the blocks and difference
42. D of both the blocks is calculated to get the final answer i.e.
13 + 67 = 80 and difference between 67 and 13 is 54.
43. A
Finally, difference between 80 and 54 is 26.
44. C
Same procedure is applied for the given input:
45. C

Solutions 41-45:

Step 1:

Difference between the product of first digits from both


the blocks and product of second digits from both the
blocks i.e. 5 × 6 = 30 and 8 × 1 = 8.

Difference between 30 and 8 is 22.

Same process is applied in all pair of blocks.

Step 2:

In this step, first digit of each block is added and then


multiplied by 3 and the result is added to 18 i.e. (2 + 4 +
2) × 3 + 18 = 42

Page 1172 of 1334


Subscribe The Xpress Video Course & Mock Test Package for Bank & Insurance Exams
If there are any suggestions/ errors in our PDFs Feel Free to contact us via this email: admin@exampundit.in
Ultra Practice Bundle PDF
SBI Clerk/ RBI Assist. Mains – Reasoning
46. E 3 - 9 = -6 ----taken as 6

47. A 6-4=2

48. C 1 - 5 = -4 taken as 4

49. B The ouptut is obtained by adding the horizontal elements


and subtracting the sum of the vetical elements from
50. E them:

(6 + 4) - (5 + 2) = 3

Solutions 46-50:

In Step 1, the corresponding elements (digits) from circle


1 and circle 4 are added and are represented in the first
circle.(If it is greater than 9, then the digits are further
added. Example, 11 would give us 2)

4 + 6 = 10 ⇒ 1 + 0 = 1

1+2=3

3+1=4

2+3=5

And the second circle is obtained by adding the


corresponding elements from circle 2 and 3 similarly.

For obtaining step 2, we subtract both the corresponding


elements from both circles and take the positive value to
get further elements.

1 - 6 = -5 ---- taken as 5

Page 1173 of 1334


Subscribe The Xpress Video Course & Mock Test Package for Bank & Insurance Exams
If there are any suggestions/ errors in our PDFs Feel Free to contact us via this email: admin@exampundit.in
Ultra Practice Bundle PDF
SBI Clerk/ RBI Assist. Mains – Reasoning
Input Output Box Type
Directions (1-5): Study the following diagram and (ii) If element contains odd number then add 4 in the
convert it into other diagrams by implementing the number and replace each letters by their immediate
instructions which is given in each step to get next step. followed letter in the alphabetical series.
Step-2:

H6B M15K R9T


8F2 13O11 18I20
F11M X11U
6K13 24K21
T4F I7Z Y6M
20D6 9G26 25F13
For step 3: Step 3 is coded in some special pattern.
B6H K9I P3R
Step 3:
D5K V5S

F4T G1X M6Y

Replace the letters with their immediate following letter


in the alphabetical series and increase the number by 2.
As per the rules followed in the above step, find out the
appropriate steps for the given input and answer the
Step 1:
following questions:
C8I L11J Q5S
S5R K8U K8H
E7L W7T
L8Q V5S
G6U H3Y N8Z
Z1E V6H X4Z
For step 2:
(i) If element contains even number then subtract 2 from
1. Which among the following element appears in the
the number and exchange the position of the letters after
2nd column of 1st row in step 3?
replacing each by their immediate preceding letter in the
a. 21K20
alphabetical series
b. 21H11
c. 8H11
Page 1174 of 1334
Subscribe The Xpress Video Course & Mock Test Package for Bank & Insurance Exams
If there are any suggestions/ errors in our PDFs Feel Free to contact us via this email: admin@exampundit.in
Ultra Practice Bundle PDF
SBI Clerk/ RBI Assist. Mains – Reasoning
d. 17H12 e. 26D24
e. None of these Directions (6-10): Study the following diagram and
2. Which element appears in the 3rd row of 2nd convert it into other diagrams by implementing the
column in step 2 ? instructions which is given in each step to get next step.
a. U11T
b. U8K K8L D5G X4S
c. Q18L
F5M A9W
d. B7G
e. H6V G8J E3Q D4E
3. Which element appears in the 3rd column of 1st
row in step 1? Replace the letters with their immediate preceding letter
a. T7S in the alphabetical series and decrease the number by 1.
b. L10V Step 1:
c. L10I J7K C4F W3R
d. M10R
E4L Z8V
e. W7T
4. Which element appears in the 3rd column of 2nd F7I D2P C3D
row in step 3?
a. 17R12 For step 2:
b. 24K21 (i) If element contains even number then Add 2 in the
c. 2G7 number and exchange the position of the letters after
d. 8F22 replacing each by their immediate followed letter in the
e. 26D24 alphabetical series
5. Which among the following element appears in the (ii) If element contains odd number then subtract 2 from
3rd row of 3rd column in step 3? the number and exchange the place of letters.
a. 21H11
b. 8H11
c. 24K21
d. 8F22

Page 1175 of 1334


Subscribe The Xpress Video Course & Mock Test Package for Bank & Insurance Exams
If there are any suggestions/ errors in our PDFs Feel Free to contact us via this email: admin@exampundit.in
Ultra Practice Bundle PDF
SBI Clerk/ RBI Assist. Mains – Reasoning
Step-2: b. CUH
K5J G6D R1W
c. ENH
M6F W10A d. CET

I5F Q4E D1C e. HSI


For step 3: Step 3 is 8.Which element appears in the 1st column of 2nd row
coded in some special pattern. in step 2?
Step 3: a. R6F

ELK FHE ASX b. T3G


c. M5J
FNG JXB
d. C3R
EJG DRF AED e. R8H
As per the rules followed in 9.Which element appears in the 3rd column of 3rd row
the above step, find out the appropriate steps for the given in step 3?
input and answer the following questions: a. FSG
b. CUH
F5R H6U K8N
c. JPQ
S6D H7R d. CET
e. HSI
V3D N7F P9O
6.Which among the 10.Which among the following element appears in the
following element appears in the 2nd column of 3RD 1st row of 3rd column in step 3?
row in step 2? a. FSG
a. D3S b. CUH
b. R8H c. JNQ
c. D4V d. ENK
d. M10P e. HSI
e. None of these Direction (11-15): Study the following diagram and
7.Which element appears in the 1strow of 2nd column convert it into other diagrams by implementing the
in step 3? instructions which is given in each step to get next step.
a. FSG

Page 1176 of 1334


Subscribe The Xpress Video Course & Mock Test Package for Bank & Insurance Exams
If there are any suggestions/ errors in our PDFs Feel Free to contact us via this email: admin@exampundit.in
Ultra Practice Bundle PDF
SBI Clerk/ RBI Assist. Mains – Reasoning
(v) If number is both the multiple of 2 and 3 or is neither
C7C I6H K6R
a multiple of 2 nor 3, then in the cell, exchange the
A3I A3B position of first and second alphabet and don’t make any
change in number.
S4R J6I Q4L
STEP 2:
P5M I7J R4S
For Step-3: It is coded in some special pattern (modified)
B3A I3A
24G24 18F19 16F9
R6K H6I B8D
26C18 26C25

Interchange the opposite boxes to get step 1 as mentioned


8D9 17F18 10D15
in the above figure.
As per the rules followed in the above step, find out the
B8D H6I R6K
appropriate steps for the given input. And answer the
I3A B3A following questions.

R4S I7J P5M

F5R K7G H7T


For Step-2 (i) Associate each alphabet with a number.
This number is the alphabet’s position in list of alphabets L9V K8H
e.g. A – 1, B – 2 etc.
(ii) Find the sum in each cell by adding numbers D5S V6D L3E
associated with each alphabet and the number present in
cell.
11. Which element is present in of 2nd column and 3rd
(iii) If the sum is a multiple of 2, then in the cell, increase
row in step 3?
the first alphabet by 1, decrease the second alphabet by 1
a.23H14
and decrease the number by 1.
b.23E5
(iv) If the sum is a multiple of 3, then in the cell, increase
c.14B5
both the alphabets by 1 and increase the number by 1.
d.20G16
Page 1177 of 1334
Subscribe The Xpress Video Course & Mock Test Package for Bank & Insurance Exams
If there are any suggestions/ errors in our PDFs Feel Free to contact us via this email: admin@exampundit.in
Ultra Practice Bundle PDF
SBI Clerk/ RBI Assist. Mains – Reasoning
e. None of these Direction (16-20): Study the following diagram and
12. Which element is present at rightmost top corner convert it into other diagrams by implementing the
in step 2? instructions which is given in each step to get next step.
a. M2D ER4 TH6 SD4
b. V9L DS3 TD7
c. E4R
HD5 CV7 NG8
d. G7K
e. T7H
Interchange the opposite boxes to get step 1 as mentioned
13. Which element appears in the 3rd column of 3rd
in the above figure.
row in step 3?
a.23H14 NG8 CV7 HD5

b.23E5
TD7 DS3
c.14B5
d.6K5
SD4 TH6 ER4
e.9E21
14. Which element appears in the 1st column of 2nd row
in step 2? For Step-2 (i) Associate each alphabet with a number.

a. D8M This number is the alphabet’s position in list of alphabets

b. D5N e.g. A – 1, B – 2 etc.

c. L9I (ii) Find the sum in each cell by adding numbers

d. E12Z associated with each alphabet and the number present in

e. G6V cell.

15. Which of the following is not a corner element in (iii)If the sum is an even number, then in the cell, replace

step 3? the first alphabet by its succeeding letter, do not change

a.14B23 second alphabet and increase the number by 2.

b.22D9 (iv)If the sum is an odd number, then in the cell, replace

c.4E24 both the alphabets by its succeeding letter and decrease

d.7G19 the number by 2.

e.9E21

Page 1178 of 1334


Subscribe The Xpress Video Course & Mock Test Package for Bank & Insurance Exams
If there are any suggestions/ errors in our PDFs Feel Free to contact us via this email: admin@exampundit.in
Ultra Practice Bundle PDF
SBI Clerk/ RBI Assist. Mains – Reasoning
c. NO6
OH6 DV9 IE3 d. HH9
e. HD7
UE5 ES5
18. Which element appears in the 1st row of 3rd
For Step-3: TE2 UH8 FS2 It is column in step 3?
coded in a. NLJ
some special pattern (fully modified) b. ARD
OGF CUI IEC c. ERB
UEE ERE d. GCG
SEB UGH ERB e. None of these
19. What is sum of numbers present in rightmost
As per the rules followed in the above step, find out the bottom corner and leftmost upper corner in step 2?
appropriate steps for the given input. And answer the a. 8
following questions. b. 10
GD5 GH7 MN8 c. 9
ER4 LK9 d. 11

AS2 IU7 PO4 e. 7


20. Which of the following is not a corner element in

16. Which element is present in of 1st column and 3rd step 3?

row in step 3? a. POB

a. NLJ b. IUE

b. GTA c. ARD

c. OPH d. MOF

d. MOF e. GCG

e. MNG Directions (21-25): Study the following diagram and

17. Which element is present at rightmost bottom convert it into other diagrams by implementing the

corner in step 2? instructions which is given in each step to get next step.

a. MK11
b. FS2

Page 1179 of 1334


Subscribe The Xpress Video Course & Mock Test Package for Bank & Insurance Exams
If there are any suggestions/ errors in our PDFs Feel Free to contact us via this email: admin@exampundit.in
Ultra Practice Bundle PDF
SBI Clerk/ RBI Assist. Mains – Reasoning
ILW QPL TMU
For step 3: Step 3 is coded in some special pattern.
GHP GLO
Step 3:
EHZ FFQ ULR As per the rules followed in the above step, find out the
D5R O9J F6G appropriate steps for the given input and answer the
following questions:
M4V L5T
V6F U3Q L8I
C4X L2W K8H
L9H B7T

Replace the letters with their immediate previous letter in L8Y W4E C5R
the alphabetical series and increase the number by 3.
Step 1: 21.Which among the following element appears in the
1st row of 3rd column in step 3?
C8Q N12I E9F
a. UME
L7U K8S
b. JJF
B7W K5V J11G c. TLQ
d. SPO
For step 2: e. GNY
(i) If element contains prime number then subtract 1 from 22. Which element appears in the 2nd row of 2nd
the number and replace alphabet each by their immediate column in step 2 ?
preceding letter in the alphabetical series. a. V11F
(ii) If element contains non-prime number then add 2 to b. U8Q
the number and replace each letters by their immediate c. L14H
followed letter in the alphabetical series. d. J10W
Step-2: e. None of these
D10R O14J F11G 23.Which element appears in the left side lower most
corner in step 3?
K6T L10T
a. UME
A6V J4U I10F b. JJF

Page 1180 of 1334


Subscribe The Xpress Video Course & Mock Test Package for Bank & Insurance Exams
If there are any suggestions/ errors in our PDFs Feel Free to contact us via this email: admin@exampundit.in
Ultra Practice Bundle PDF
SBI Clerk/ RBI Assist. Mains – Reasoning
c. SPO Interchange the opposite boxes to get step 1 as mentioned
d. DLQ in the above figure.
e. XHF L9H Z4X V6A
24.Which element appears in the right side uppermost K3S M8R
corner in step 1?
a. T6P N9M G7T V5D
b. K11H
c. A10S For Step-2 (i) Associate each alphabet with a number.
d. V7D This number is the alphabet’s position in list of alphabets
e. B8Q e.g. A – 1, B – 2 etc.
25.Which among the following element appears (ii) Find the sum of associate number of alphabet in each
behind of JJF step 3? cell.
a. TLQ (iii) If the sum is an even number , then in the cell, increase
b. GNY the first alphabet by 1, decrease second alphabet by 1 and
c. XHF increase the number by 1.
d. ILX (iv) If the sum is an odd number, then in the cell, decrease
e. None of these both the alphabets by 1 and decrease the number by 2.
M10G A5W U4Z For Step-3: It is coded in some special pattern.
L4R L6Q 10 19 43
M7L F5S W6C 26 23
Direction (26-30): Study the following diagram and 18 20 20
convert it into other diagrams by implementing the
instructions which is given in each step to get next step. As per the rules followed in the above step, find out the
appropriate steps for the given input and answer the
V5D G7T N9M
following questions:
M8R K3S
F8H R4E B8X

V6A Z4X L9H K4S L9A

R7G K6F Z4S

Page 1181 of 1334


Subscribe The Xpress Video Course & Mock Test Package for Bank & Insurance Exams
If there are any suggestions/ errors in our PDFs Feel Free to contact us via this email: admin@exampundit.in
Ultra Practice Bundle PDF
SBI Clerk/ RBI Assist. Mains – Reasoning
c. Q5F
26. What is sum of element appears in the 1st row of 2nd d. C9W
column and 3rd row 2nd column in step 3? e. Q2D
a. 56 30. Which among the following element appears in
b. 54 front of Q2D step 2?
c. 30 a. K6F
d. 37 b. J4E
e. None of these c. K3S
d. R4E
27. Which among the following element appears in the e. F8H
2nd row of 3rd column in step 2? Directions (31-35): Study the following diagram and
a. Y1R convert it into otherdiagrams by implementing the
b. J4E instructions which is given in each step to get next step.
c. Q5F A53 N89 G76
d. K7Z
K67 C23
e. L5R
28.Which element is diagonally faced to the number K76 L98 K21
which is in the uppermost left corner in step 3?
a. 17 Exchange the element of row one to row 3.Don’t
b. 18 exchange Row 2.
c. 26 Step 1:
d. 19 K76 L98 K21
e. 5
K67 C23
9.Which element appears in the right side uppermost
corner in step 2? A53 N89 G76
a. Y1R
b. J4E
For step 2:

Page 1182 of 1334


Subscribe The Xpress Video Course & Mock Test Package for Bank & Insurance Exams
If there are any suggestions/ errors in our PDFs Feel Free to contact us via this email: admin@exampundit.in
Ultra Practice Bundle PDF
SBI Clerk/ RBI Assist. Mains – Reasoning
(i) If element contains two digit prime number then D88 V56 G76
subtract 4 from the number and replace letter by their
immediate preceding letter in the alphabetical series
31. Which among the following element appears in the
(ii) If element contains non-prime odd number then add 4
3rd column of 2nd row in step 3?
to the number and replace letter by their immediate
a. V49
followed letter in the alphabetical series.
b. D51
(iii) If element contains non-prime even number then
c. I43
divide it by 2 and replace letter by their immediate
d. H62
followed letter in the alphabetical series.
e. O101
Step-2:
32. Which element appears in right lowermost corner
L38 M49 L25 in step 2 ?
J63 B19 a. W28
b. H38
Z49 M85 H38
c. S43
d. L89
For step 3: Step 3 is coded in some special pattern.
e. K27
Step 3:
33. Which element appears in the 3rd column of 1st

O50 N62 O37 row in step 3?


a. V49
Q73 Y21
b. D51
A75 N98 S46 c. S46
d. I43
e. H62
As per the rules followed in the above step, find out the
34.Which element appears in left lowermost corner in
appropriate steps for the given input and answer the
step 1 ?
following questions:
a. D88
S17 K85 L31 b. V56

S29 T47 c. S29


d. S17

Page 1183 of 1334


Subscribe The Xpress Video Course & Mock Test Package for Bank & Insurance Exams
If there are any suggestions/ errors in our PDFs Feel Free to contact us via this email: admin@exampundit.in
Ultra Practice Bundle PDF
SBI Clerk/ RBI Assist. Mains – Reasoning
e. K85 For Step-2 (i) Associate each alphabet with a number.
35..Which 48 3 34 This number is the alphabet’s position in list of alphabets
element is 19 14 e.g. A – 1, B – 2 etc.
diagonally (ii) Find the difference between sum of the alphabet
13 12 41
faced to the position in alphabet series and center number.
number which is in the uppermost right corner in step (iii) Then interchange the positions of cells to get step 2.
3?
a. V49 Step 2:
b. D51 For Step-3: It is coded in some special pattern.
c. I43 41 12 13 E25 C9 D16
d. I31 14 19 E25 J100
e. O101 G49 C9 L144
34 3 48

Direction (36-40): Study the following diagram and As per the rules followed in the above step, find out the
convert it into other diagrams by implementing the appropriate steps for the given input. And answer the
instructions which is given in each step to get next step: following questions.
W7R B2C Z2X G6D M3C H2A

L5G D3R X4U T7Y


D5S L8H M3X
V7Z L8H O6D

36. Which is sum of the numbers present in of 2nd


Interchange the column 1 with column 3 to get step 1.Do
column and 3rd row and 1st row 3rd column in step 2?
not change column 2.
a.34
Z2X B2C W7R
b.13
D3R L5G c.57
O6D L8H V7Z d.47
e.None of these
37. Which element is present at right lowermost corner
in step 3?

Page 1184 of 1334


Subscribe The Xpress Video Course & Mock Test Package for Bank & Insurance Exams
If there are any suggestions/ errors in our PDFs Feel Free to contact us via this email: admin@exampundit.in
Ultra Practice Bundle PDF
SBI Clerk/ RBI Assist. Mains – Reasoning
a. C9 A25 J27 E121
b. G49 Z16 U343
c. K121
B32 L81 K48
d. D16
e. D49
Interchange the opposite boxes to get step 1 as mentioned
38. Which element appears in the immediately front of
in the above figure.
5 in step 2?
a. 38
K48 L81 B32
b. 41
U343 Z16
c. 12
E121 J27 A25
d. 18
For Step-2
e. None of these
(i) If the cell contains a perfect square number then take
39. According to coding for step 3,what will be the code
square root of that number and use that numbered letter in
for 345?
alphabet series. Decrease the first alphabet by 1.
a. F36
ii)If the cell contains a perfect cube number then take cube
b. L60
root of that number and use that numbered letter in
c. O225
alphabet series. Increase the first alphabet by 1.
d. L144
ii)If none of the above condition is followed then divide
e. M225
that number by 2 and use that numbered letter in alphabet
40. In which of the following is not a corner element in
series. Don’t change the first letter.
step 3?
a. I81 KX KI BP
b. C9 VG YD
c. G49 DK KC ZE
For Step-3: It is coded in some special pattern.
d. E25
19 34 36
e. None of these
25 25
Direction (41-45): Study the following diagram and
39 40 23
convert it into other diagrams by implementing the
instructions which is given in each step to get next step.

Page 1185 of 1334


Subscribe The Xpress Video Course & Mock Test Package for Bank & Insurance Exams
If there are any suggestions/ errors in our PDFs Feel Free to contact us via this email: admin@exampundit.in
Ultra Practice Bundle PDF
SBI Clerk/ RBI Assist. Mains – Reasoning
As per the rules followed in the above step, find out the a. BJ
appropriate steps for the given input. And answer the b. CI
following questions. c. VG
G125 L38 F8 d. FD
A1000 K36 e. None of the above
I512 C18 R256 45. What is sum of the number which is in the cell of
right side uppermost corner and element in 2nd column
41. Which element is present in of 2nd column and 3rd and 3rd row in step 3?
row in step 2? a.66
a. QP b.69
b. CI c.59
c. VG d.60
d. FD e.75
e. LS Direction (46-50): Study the following diagram and
42. Which element is present in front of JF in step 2? convert it into other diagrams by implementing the
a. CI instructions which is given in each step to get next step.
b. JH C4A C4D Z2X
c. QP F6G D3R
d. LS P9I S5Z K7R
e. HE
43. Which element appears in the 3rd column of 3rd Replace the letters with their immediate preceding letter
row in step 3? in the alphabetical series and decrease the number by 1.
a.41
b.25
B3Z B3C Y1W
c.36
E5F C2Q
d.39
O8H R4Y J6Q
e.None of these
44. Which element appears in the 3rd column of 2nd row
in step 2?

Page 1186 of 1334


Subscribe The Xpress Video Course & Mock Test Package for Bank & Insurance Exams
If there are any suggestions/ errors in our PDFs Feel Free to contact us via this email: admin@exampundit.in
Ultra Practice Bundle PDF
SBI Clerk/ RBI Assist. Mains – Reasoning
For Step-2 (i) Associate each alphabet with a number. a. G
This number is the alphabet’s position in list of alphabets b. I
e.g. A – 1, B – 2 etc. c. F
(ii)Interchange the column 1 with column 3 to get step d. K
1.Do not change column 2. e. None of these

Y1W B3C B3Z 47. Which of below element is not present in corner in
step 1?
C2Q E5F
a. L2C
J6Q R4Y O8H
b. K3J
(iii)Find the sum of the alphabet position in alphabet series c. O8D
and center number to obtain step 2 d. J6F
e. Q4R
49 8 31
22 16
48. Which element appears in the front of 19 in step 2?
33 47 31
a. 39
For Step-3: It is coded in some special pattern. b. 24
c. 17
d. 27
N I E
e. 22
E H
49. According to coding for step 3,what will be the code
G L E
for 289 ?
As per the rules followed in the above step, find out the a. L
appropriate steps for the given input and answer the b. S
following questions. c. T
M3D L4K P9E d. U
C8X G7H e. R
K7G V6C R5S 50. What is sum of element of 3rd column 1st row and
46. Which element is present at left lowermost corner 3rd row and 3rd column in step 2?
in step 3? a. 39

Page 1187 of 1334


Subscribe The Xpress Video Course & Mock Test Package for Bank & Insurance Exams
If there are any suggestions/ errors in our PDFs Feel Free to contact us via this email: admin@exampundit.in
Ultra Practice Bundle PDF
SBI Clerk/ RBI Assist. Mains – Reasoning
b. 42 d. 43
c. 29 e. 27

Input Output Box Type – Answer and Explanation


SOLUTIONS (1-5):

S5R K8U K8H

L8Q V5S

Z1E V6H X4Z

Replace the letters with their immediate following letter in the alphabetical series and increase the number by 2.
Step 1:

T7S L10V L10I

M10R W7T

A3F W8I Y6A

For step 2:
(i) If element contains even number then subtract 2 from the number and exchange the position of the letters after
replacing each by their immediate preceding letter in the alphabetical series
(ii) If element contains odd number then add 4 in the number and replace each letters by their immediate followed letter
in the alphabetical series.
Step-2:

U11T U8K H8K

Q8L X11U

B7G H6V Z4X

For step 3: Step 3 is coded in some special pattern.

Page 1188 of 1334


Subscribe The Xpress Video Course & Mock Test Package for Bank & Insurance Exams
If there are any suggestions/ errors in our PDFs Feel Free to contact us via this email: admin@exampundit.in
Ultra Practice Bundle PDF
SBI Clerk/ RBI Assist. Mains – Reasoning
To obtain step-3: Each alphabet is replaced by number with respect to its alphabetical position and each number is
replaced by alphabet with respect to its English alphabetical series.
Step 3:

21K20 21H11 8H11

17H12 24K21

2G7 8F22 26D24


Answers:
1) Option b
2) Option e
3) Option c
4) Option b
5) Option e
SOLUTIONS (6-10):

F5R H6U K8N

S6D H7R

V3D N7F P9O

Replace the letters with their immediate preceding letter in the alphabetical series and decrease the number by 1.
Step 1:

E4Q G5T J7M

R5C G6Q

U2C M6E O8N

For step 2:

Page 1189 of 1334


Subscribe The Xpress Video Course & Mock Test Package for Bank & Insurance Exams
If there are any suggestions/ errors in our PDFs Feel Free to contact us via this email: admin@exampundit.in
Ultra Practice Bundle PDF
SBI Clerk/ RBI Assist. Mains – Reasoning
(i) If element contains even number then Add 2 in the number and exchange the position of the letters after replacing
each by their immediate followed letter in the alphabetical series
(ii) If element contains odd number then subtract 2 from the number and exchange the place of letters.
Step-2:

R6F T3G M5J

C3R R8H

D4V F8N O10P

For step 3: First we take alphabet for middle number, then immediate following alphabet for 1st and last alphabet.
Step 3:

FSG CUH ENK

CDS HSI

DEU HGO JPQ


Answers:
6) Option e
7) Option b
8) Option d
9) Option c
10) Option d
Solution (11-15): (fully modified)
Input:

F5R K7G H7T

L9V K8H

Page 1190 of 1334


Subscribe The Xpress Video Course & Mock Test Package for Bank & Insurance Exams
If there are any suggestions/ errors in our PDFs Feel Free to contact us via this email: admin@exampundit.in
Ultra Practice Bundle PDF
SBI Clerk/ RBI Assist. Mains – Reasoning
D5S V6D L3E

Step 1:
Interchange the opposite boxes to get step 1 as mentioned in the above figure.

L3E V6D D5S

K8H L9V

H7T K7G F5R

Step 2:
For Step-2 (i) Associate each alphabet with a number. This number is the alphabet’s position in list of alphabets e.g. A
– 1, B – 2 etc.
(ii) Find the sum in each cell by adding numbers associated with each alphabet and the number present in cell.
(iii) If the sum is a multiple of 2, then in the cell, increase the first alphabet by 1, decrease the second alphabet by 1 and
decrease the number by 1.
(iv) If the sum is a multiple of 3, then in the cell, increase both the alphabets by 1 and increase the number by 1.
(v) If number is both the multiple of 2 and 3 or is neither a multiple of 2 nor 3, then in the cell, exchange the position
of first and second alphabet and don’t make any change in number.

M2D W5C E4R

L9I V9L

T7H G7K R5F


For Step 3: For all the letters, there value in the alphabetical series is written in reverse
order i.e. Z will be denoted by 1, Y will be denoted by 2 and all the numbers are replaced by their equivalent letter in
the alphabetical series (starting from A).
Step 3:

14B23 4E24 22D9

15I18 5I15

Page 1191 of 1334


Subscribe The Xpress Video Course & Mock Test Package for Bank & Insurance Exams
If there are any suggestions/ errors in our PDFs Feel Free to contact us via this email: admin@exampundit.in
Ultra Practice Bundle PDF
SBI Clerk/ RBI Assist. Mains – Reasoning
7G19 20G16 9E21

Answers:
11) Option d
12) Option c
13) Option e
14) Option c
15) Option c

SOLUTIONS 16-20:

GD5 GH7 MN8


ER4 LK9

AS2 IU7 PO4


Interchange the opposite boxes to get step 1 as mentioned in the above figure.
PO4 IU7 AS2
LK9 ER4

MN8 GH7 GD5

For Step-2 (i) Associate each alphabet with a number. This number is the alphabet’s position in list of alphabets e.g. A
– 1, B – 2 etc.
(ii) Find the sum in each cell by adding numbers associated with each alphabet and the number present in cell.
(iii)If the sum is an even number, then in the cell, replace the first alphabet by its succeeding letter, do not change
second alphabet and increase the number by 2.
(iv) If the sum is an odd number, then in the cell, replace both the alphabets by its succeeding letter and decrease the
number by 2.

Page 1192 of 1334


Subscribe The Xpress Video Course & Mock Test Package for Bank & Insurance Exams
If there are any suggestions/ errors in our PDFs Feel Free to contact us via this email: admin@exampundit.in
Ultra Practice Bundle PDF
SBI Clerk/ RBI Assist. Mains – Reasoning
QP2 JV5 BS4
MK11 FS2
For Step-3: It is coded in some special pattern. (Fully added)
NO6 HH9 HD7
For Step-3: If cell contains any vowel then do not change the vowel; replace
the other letter with its immediately preceding letter. For number take that numbered positioned alphabet in alphabet
series.
If cell does not contain any vowel then replace the both letters by its immediately preceding letter. For number take
that numbered positioned alphabet in alphabet series.
POB IUE ARD
LJK ERB
MOF GGI GCG

Answers:
16) Option d
17) Option e
18) Option b
19) Option d
20) Option b
SOLUTION 21-25:

V6F U3Q L8I

L9H B7T

L8Y W4E C5R

Replace the letters with their immediate previous letter in the alphabetical series and increase the number by 3.
Step 1:

Page 1193 of 1334


Subscribe The Xpress Video Course & Mock Test Package for Bank & Insurance Exams
If there are any suggestions/ errors in our PDFs Feel Free to contact us via this email: admin@exampundit.in
Ultra Practice Bundle PDF
SBI Clerk/ RBI Assist. Mains – Reasoning
U9E T6P K11H

K12G A10S

K11X V7D B8Q

For step 2:
(i) If element contains prime number then subtract 1 from the number and replace alphabet each by their immediate
preceding letter in the alphabetical series.
(ii) If element contains non-prime number then add 2 in the number and replace each letters by their immediate followed
letter in the alphabetical series.
Step-2:

V11F U8Q J10G

L14H B12T

J10W U6C C10R

For step 3: Exchange the position of first and third letter and replace that letter by taking reverse alphabet as per the
alphabet series i.e. A is denoted as Z, B is denoted as Y and so on.
For center number consider as a position number in alphabet series, replace it with alphabet which is two places away
from it in alphabet series.
Step 3:

UME JJF TLQ

SPO GNY

DLQ XHF ILX

Answers:
21) Option c

Page 1194 of 1334


Subscribe The Xpress Video Course & Mock Test Package for Bank & Insurance Exams
If there are any suggestions/ errors in our PDFs Feel Free to contact us via this email: admin@exampundit.in
Ultra Practice Bundle PDF
SBI Clerk/ RBI Assist. Mains – Reasoning
22) Option e
23) Option d
24) Option b
25) Option c

SOLUTIONS 26-30:
F8H R4E B8X
K4S L9A

R7G K6F Z4S

Interchange the opposite boxes to get step 1 as mentioned in the above figure.
Z4S K6F R7G
L9A K4S

B8X R4E F8H

For Step-2 (i) Associate each alphabet with a number. This number is the alphabet’s position in list of alphabets e.g. A
– 1, B – 2 etc.
(ii) Find the sum of associate number of alphabet in each cell.
(iii) If the sum is an even number , then in the cell, increase the first alphabet by 1, decrease second alphabet by 1 and
increase the number by 1.
(iv) If the sum is an odd number, then in the cell, decrease both the alphabets by 1 and decrease the number by 2.

Y2R J4E Q5F


K7Z L5R
C9W Q2D G9G

Page 1195 of 1334


Subscribe The Xpress Video Course & Mock Test Package for Bank & Insurance Exams
If there are any suggestions/ errors in our PDFs Feel Free to contact us via this email: admin@exampundit.in
Ultra Practice Bundle PDF
SBI Clerk/ RBI Assist. Mains – Reasoning
For Step-3: The middle number is subtracted from the sum of the position number of letters in alphabet series.
41 11 18
30 25
17 19 5

Answers:
26) Option c
27) Option e
28) Option e
29) Option c
30) Option b
Solutions:
Directions 31-35:

S17 K85 L31

S29 T47

D88 V56 G76

Exchange the element of row one to row 3.Don’t exchange Row 2.


Step 1:

D88 V56 G76

S29 T47

S17 K85 L31

For step 2:
(i) If element contains two digit prime number then subtract 4 from the number and replace letter by their immediate
preceding letter in the alphabetical series

Page 1196 of 1334


Subscribe The Xpress Video Course & Mock Test Package for Bank & Insurance Exams
If there are any suggestions/ errors in our PDFs Feel Free to contact us via this email: admin@exampundit.in
Ultra Practice Bundle PDF
SBI Clerk/ RBI Assist. Mains – Reasoning
(ii) If element contains non-prime odd number then add 4 in the number and replace letter by their immediate followed
letter in the alphabetical series.
(iii) If element contains non-prime even number then divide it by 2 and replace letter by their immediate followed letter
in the alphabetical series.
Step-2:

E44 W28 H38

R25 S43

R13 L89 K27

For step 3: replace letter with its reverse letter in alphabet series.
For number, add letter’s numerical position in alphabet series to number.
Step 3:

V49 D51 S46

I43 H62

I31 O101 P38

Answers:
31) Option d
32) Option e
33) Option c
34) Option d
35) Option d

Page 1197 of 1334


Subscribe The Xpress Video Course & Mock Test Package for Bank & Insurance Exams
If there are any suggestions/ errors in our PDFs Feel Free to contact us via this email: admin@exampundit.in
Ultra Practice Bundle PDF
SBI Clerk/ RBI Assist. Mains – Reasoning

SOLUTION 36-40:
G6D M3C H2A
X4U T7Y
D5S L8H M3X

Interchange the column 1 with column 3 to get step 1.Do not change column 2.
H2A M3C G6D
T7Y X4U
M3X L8H D5S

For Step-2 (i) Associate each alphabet with a number. This number is the alphabet’s position in list of alphabets e.g.
A – 1, B – 2 etc.
(ii) Find the difference between sum of the alphabet position in alphabet series and center number.
(iii) Then interchange the positions of cells to get step 2.

7 13 5
38 41

34 12 18
Step 2:

18 12 34
41 38

5 13 7

For Step-3: Do sum of digits in number, take that position alphabet in alphabet series and write a square of that sum.
I81 C9 G49
E25 K121

Page 1198 of 1334


Subscribe The Xpress Video Course & Mock Test Package for Bank & Insurance Exams
If there are any suggestions/ errors in our PDFs Feel Free to contact us via this email: admin@exampundit.in
Ultra Practice Bundle PDF
SBI Clerk/ RBI Assist. Mains – Reasoning
E25 D16 G49

Answers:
36) Option d
37) Option b
38) Option b
39) Option d

40) Option b
SOLUTION 41-45: Study the following diagram and convert it into other diagrams by implementing the
instructions which is given in each step to get next step.
G125 L38 F8
A1000 K36

I512 C18 R256

Interchange the opposite boxes to get step 1 as mentioned in the above figure.

R256 C18 I512


K36 A1000
F8 L38 G125
For Step-2
(ii) If the cell contains a perfect square number then take square root of that number and use that numbered letter in
alphabet series. Decrease the first alphabet by 1.
ii)If the cell contains a perfect cube number then take cube root of that number and use that numbered letter in alphabet
series. Increase the first alphabet by 1.
ii)If none of the above condition is followed then divide that number by 2 and use that numbered letter in alphabet
series. Don’t change the first letter.

QP CI JH

Page 1199 of 1334


Subscribe The Xpress Video Course & Mock Test Package for Bank & Insurance Exams
If there are any suggestions/ errors in our PDFs Feel Free to contact us via this email: admin@exampundit.in
Ultra Practice Bundle PDF
SBI Clerk/ RBI Assist. Mains – Reasoning
JF BJ
GB LS HE

For Step-3: Take reverse alphabet series number of the letter then do sum of them.
21 42 36
38 42
45 23 41

Answers:
41) Option e
42) Option c
43) Option a
44) Option a
45) Option c
SOLUTION 46-50:
M3D L4K P9E
C8X G7H
K7G V6C R5S

Replace the letters with their immediate preceding letter in the alphabetical series and decrease the number by 1.

L2C K3J O8D


B7W F6G
J6F U5B Q4R
For Step-2 (i) Associate each alphabet with a number. This number is the
alphabet’s position in list of alphabets e.g. A – 1, B – 2 etc.
(ii)Interchange the column 1 with column 3 to get step 1.Do not change column 2.

O8D K3J L2C


F6G B7W
Page 1200 of 1334
Subscribe The Xpress Video Course & Mock Test Package for Bank & Insurance Exams
If there are any suggestions/ errors in our PDFs Feel Free to contact us via this email: admin@exampundit.in
Ultra Practice Bundle PDF
SBI Clerk/ RBI Assist. Mains – Reasoning
Q4R U5B J6F

Step 2:

27 24 17
19 32
39 28 22
For Step-3: do sum of digits in number and add 1 in it.Take that numbered
letter of alphabet in alphabet series.

J G I
K F
M K E
Answers:
46) Option e
47) Option b
48) Option d
49) Option c

50) Option a

Critical Reasoning
1. the student's study. Most of the students use their mobile
Statement I: The university authority has instructed all even during class hours to forward SMS or keep chatting
the colleges under its jurisdiction to ban use of all phones in their phones.
inside the college premises. Mobile phones are really Statement II: Majority of the teachers of the colleges
necessary and useful. But the college authorities have signed a joint petition to the university complaining the
banned them in the colleges since they are a distraction to disturbances caused by cell phone ring-tones inside the

Page 1201 of 1334


Subscribe The Xpress Video Course & Mock Test Package for Bank & Insurance Exams
If there are any suggestions/ errors in our PDFs Feel Free to contact us via this email: admin@exampundit.in
Ultra Practice Bundle PDF
SBI Clerk/ RBI Assist. Mains – Reasoning
classrooms. They quoted that it became harmful to the 3.
institutes to maintain their peaceful environment. So the Statement I: There has been a high increase in the
college administration should ban mobiles in the campus. incidents of atrocities against women in the city during the
a. Statement I is the cause and statement II is its effect past few months. Violence against women in India refers
b. Statement II is the cause and statement I is its effect to physical or sexual violence committed against Indian
c. Both the statements I and II are independent causes women, typically by a man. Common forms of violence
d. Both the statements I and II are effects of independent against women in India include acts such as domestic
causes abuse, sexual assault, and murder.
e. Both the statements I and II are effects of some common Statement II: The police authority has been unable to nab
cause the culprits who are committing crime against women.
2. Most typically, these acts are committed by men as a result
Statement I: Rural and semi-urban areas in the country of the long-standing gender inequalities present in the
have been suffering due to load shedding for quite some country.
time. Load shedding is a way to distribute demand for a. Statement I is the cause and statement II is its effect
electrical power across multiple power sources. b. Statement II is the cause and statement I is its effect
Statement II: If the Government is not able to overcome c. Both the statements I and II are independent causes
the power crisis, load shedding will be extended even to d. Both the statements I and II are effects of independent
the urban areas. Load shedding is used to relieve stress on causes
a primary energy source when demand for electricity is e. Both the statements I and II are effects of some common
greater than the primary power source can supply. Most cause
buildings, including data centers, purchase electrical 4.
power from a utility provider. Statement I: The prices of vegetables have been
a. Statement I is the cause and statement II is its effect increased considerably during this summer. At the retail
b. Statement II is the cause and statement I is its effect level too, prices have increased sharply during the period.
c. Both the statements I and II are independent causes Prices of potatoes increased 30.5 per cent (to Rs 27.59 per
d. Both the statements I and II are effects of independent kg on an average) and tomatoes increased by 32.6 per cent
causes (to an average Rs 25.33 a kg).
e. Both the statements I and II are effects of some common Statement II: There is tremendous increase in the
cause temperature during this summer thereby damaging crops

Page 1202 of 1334


Subscribe The Xpress Video Course & Mock Test Package for Bank & Insurance Exams
If there are any suggestions/ errors in our PDFs Feel Free to contact us via this email: admin@exampundit.in
Ultra Practice Bundle PDF
SBI Clerk/ RBI Assist. Mains – Reasoning
greatly. The effect of temperature is enhanced under high d. Both the statements I and II are effects of independent
vapor pressure deficits because pollen viability (prior to causes
silk reception) is a function of pollen moisture content e. Both the statements I and II are effects of some common
which is strongly dependent on vapor pressure deficit cause
a. Statement I is the cause and statement II is its effect 6.
b. Statement II is the cause and statement I is its effect Statement I: The performance of most of the students in
c. Both the statements I and II are independent causes final exam of class X in the schools run by the
d. Both the statements I and II are effects of independent Government was excellent. At a time when large number
causes of Government schools are facing closure threat due to
e. Both the statements I and II are effects of some common dwindling student enrolment, some government schools
cause are finding favour of parents due to very good results they
5. are producing.
Statement I: There is considerable reduction in the Statement II: Many teachers of the Government schools
number of people affected by water-borne diseases in City left the school and joined private schools. Private schools
A during this rainy season. Waterborne diseases are allow for flexibility on the part of the teacher as well.
caused by drinking contaminated or dirty water. Teachers in private schools are not required to adhere to
Contaminated water can cause many types of diarrheal strict curriculum guidelines; they have some form of
diseases, including Cholera, and other serious illnesses control over course content and topics to be taught. These
such as Guinea worm disease, Typhoid, and Dysentery teachers have greater control over the choice of
Statement II: The government has opened four new civil instructional materials and textbooks to be used.
hospitals in City A in the beginning of the year to improve a. Statement I is the cause and statement II is its effect
the quality of life of people by providing better Health b. Statement II is the cause and statement I is its effect
Services. These hospitals strive to help people improve c. Both the statements I and II are independent causes
their productivity and reduce risks of diseases and injury d. Both the statements I and II are effects of independent
in a cost-effective way. causes
a. Statement I is the cause and statement II is its effect e. Both the statements I and II are effects of some common
b. Statement II is the cause and statement I is its effect cause
c. Both the statements I and II are independent causes 7.

Page 1203 of 1334


Subscribe The Xpress Video Course & Mock Test Package for Bank & Insurance Exams
If there are any suggestions/ errors in our PDFs Feel Free to contact us via this email: admin@exampundit.in
Ultra Practice Bundle PDF
SBI Clerk/ RBI Assist. Mains – Reasoning
Statement I: The farmers have decided against selling Statement II: In the past, the general public did not have
their kharif crops to the Government agencies. Kharif access to all these information related to primary
crops, monsoon crops or autumn crops are domesticated education. Primary education takes place in primary
plants like rice that are cultivated and harvested in India, school, the elementary school or first and middle school
Pakistan and Bangladesh during the Indian subcontinent's depending on the location.
monsoon season, which lasts from June to November a. Statement I is the cause and statement II is its effect
depending on the area. b. Statement II is the cause and statement I is its effect
Statement II: The Government has reduced the c. Both the statements I and II are independent causes
procurement price of kharif crops starting from last month d. Both the statements I and II are effects of independent
to the next six months. Procurement price of a commodity causes
refers to the price at which govt. procures the commodity e. Both the statements I and II are effects of some common
from producers/manufactures for maintaining the buffer cause
stock or the public distribution system. These prices are 9) Statement 1: Competition Act provides for, keeping in
announced by the govt. of India on the recommendations view of the economic development of the country, the
of the Commission for Agricultural Costs and Prices establishment of a Commission to prevent practices
before the harvest season of the crop. having adverse effect on competition, to promote and
a. Statement I is the cause and statement II is its effect sustain competition in markets, to protect the interests of
b. Statement II is the cause and statement I is its effect consumers and to ensure freedom of trade carried on by
c. Both the statements I and II are independent causes other participants in markets in India.
d. Both the statements I and II are effects of independent Statement 2: Anti–competitive agreements are void.
causes a) Statement 1 is the cause and statement 2 is its effect.
e. Both the statements I and II are effects of some common b) Statement 2 is the cause and statement 1 is its effect.
cause c) Statement 1 and statement 2 are independent effects of
8. independent causes.
Statement I: The government has decided to make all the d) Statement 1 and statement 2 are independent effects of
information related to primary education available to the a common cause.
general public. Primary education is typically the first e) Statement 1 is the cause but statement 2 is not effect
stage of formal education, coming after preschool and 10) Statement 1: Unregistered trademark proprietor has
before secondary school. no legal remedy against infringement of its trademark.

Page 1204 of 1334


Subscribe The Xpress Video Course & Mock Test Package for Bank & Insurance Exams
If there are any suggestions/ errors in our PDFs Feel Free to contact us via this email: admin@exampundit.in
Ultra Practice Bundle PDF
SBI Clerk/ RBI Assist. Mains – Reasoning
Statement 2: The registered proprietor of the trademark D. generalizing about all actions of a certain kind on the
has exclusive right to the use of the trademark in relation basis of a description of one such action
to the goods or service in respect of which the trade mark E. treating something that is probable as though it were
is registered and only registered proprietor can obtain inevitable
relief in respect of infringement of the trademark. 12. Statement :Studies show that repeated exposure to
a) Statement 1 is the cause and statement 2 is its effect. an allergen can trigger an allergic person’s defense
b) Statement 2 is the cause and statement 1 is its effect. mechanisms to the extent that eventually, even
c) Statement 1 and statement 2 are independent effects of minimal exposure to the allergen can provoke a
independent causes. stronger than normal, even life threatening reaction.
d) Statement 1 and statement 2 are independent effects of Which of the following is the best course of action for
a common cause. the process by which minimal contact with an allergen
e) Statement 1 is the cause but statement 2 is not effect can cause a major reaction?
STATEMENT AND COURSES OF ACTION A. People with unhealthy diets are more susceptible to
11.Statement : Garbage in this neighborhood probably diabetes and high blood pressure.
will not be collected until Thursday this week. Garbage B. Because minor earthquakes can weaken the existing
is usually collected here on Wednesdays, and the infrastructure in an area, a series of minor earthquakes
garbage collectors in this city are extremely reliable. cansometimes lead to more damage than a single major
However, Monday was a public holiday, and after a earthquake.
public holiday that falls on a Monday, garbage C. Peanut allergy sufferers should avoid all potential
throughout the city is supposed to be collected one day contact with peanuts.
later than usual. D. A student who fails a test can still earn a good grade if
The statement proceeds by which of the following she works hard for the rest of the semester.
course of actions? E. Drivers who speed regularly are more likely to be
A. treating several pieces of irrelevant evidence as though caught than those who speed only occasionally.
they provide support for the conclusion 13. Statement :The importance of the ozone layer to
B. indirectly establishing that one thing is likely to occur terrestrial animals is that it entirely filters out some
by directly ruling out all of the alternative possibilities wavelengths of light but lets others through. Holes in
C. providing information that allows application of a the ozone layer and the dangers associated with these
general rule to a specific case holes are well documented. However, one danger that

Page 1205 of 1334


Subscribe The Xpress Video Course & Mock Test Package for Bank & Insurance Exams
If there are any suggestions/ errors in our PDFs Feel Free to contact us via this email: admin@exampundit.in
Ultra Practice Bundle PDF
SBI Clerk/ RBI Assist. Mains – Reasoning
has not been given sufficient attention is that these A. The history of physics is characterized by a pattern of
holes could lead to severe eye damage for animals of one successful theory subsequently surpassed by another.
many species. B. Long-standing success or substantiation of a theory of
Which one of the following is most strongly supported physics is no guarantee that the theory will continue to
course of action by the statements above, if they are bedominant indefinitely.
true? C. Every theory of physics, no matter how successful, is
A. All wavelengths of sunlight that can cause eye damage eventually surpassed by one that is more successful.
are filtered out by the ozone layer where it is intact. D. Once a theory of physics is accepted, it will remain
B. Few species of animals live on a part of the earth’s dominant for centuries.
surface that is not threatened by holes in the ozone layer. E. If a long-accepted theory of physics is surpassed, it
C. Some species of animals have eyes that will not suffer must be surpassed by a theory that is equally successful.
any damage when exposed to unfiltered sunlight. 15. Statement :A free marketplace of ideas ensures
D. A single wavelength of sunlight can cause severe that all ideas get a fair hearing. Even ideas tainted with
damage to the eyes of most species of animals. prejudice and malice can prompt beneficial outcomes.
E. Some wavelengths of sunlight that cause eye damage In most countries, however, the government is
are more likely to reach the earth’s surface where there responsible for over half the information released to
areholes in the ozone layer than where there are not. the public through all media. For this reason, the
14. Statement :Newtonian physics dominated science power of governments over information needs to be
for over two centuries. It found consistently successful curtailed. Everyone grants that governments should
application, becoming one of the most highly not suppress free expression, yet governments
substantiated and accepted theories in the history of continue to construct near monopolies on the
science. Nevertheless, Einstein’s theories came to show publication and dissemination of enormous amounts of
the fundamental limits of Newtonian physics and to information.
surpass the Newtonian view in the early 1900s, giving Which one of the following most accurately expresses
rise once again to a physics that has so far enjoyed wide the course of action of the journalist’s statement?
success. A. The freedom of the marketplace of ideas is in jeopardy.
Which one of the following course of action follows B. Preserving a free marketplace of ideas is important.
from the statements above? C. The control that governments have over information
needs to be reduced.

Page 1206 of 1334


Subscribe The Xpress Video Course & Mock Test Package for Bank & Insurance Exams
If there are any suggestions/ errors in our PDFs Feel Free to contact us via this email: admin@exampundit.in
Ultra Practice Bundle PDF
SBI Clerk/ RBI Assist. Mains – Reasoning
D. Ideas that have malicious content or stem from A. A physician should not prescribe any antidepressant
questionable sources can be valuable. drug for a patient if that patient is overweight.
E. Governments have near monopolies on the B. People who are trying to lose weight should not ask
dissemination of many kinds of information. their doctors for an antidepressant drug.
16. Statement :The number of North American C. At least some patients taking antidepressant drugs gain
children who are obese—that is, who have more body weight as a result of taking them.
fat than do 85 percent of North American children D. The weight gain experienced by patients taking
their age—is steadily increasing, according to four antidepressant drugs should be attributed to lack of
major studies conducted over the past 15 years. dieting.
If the finding reported above is correct, it can be E. All patients taking antidepressant drugs should diet to
properly concluded with which of the following course maintain their weight.
of action? 18. Statement :Emissions from automobiles that burn
A. when four major studies all produce similar results, gasoline and automobiles that burn diesel fuel are
those studies must be accurate threatening the quality of life on our planet,
B. North American children have been progressively less contaminating both urban air and global atmosphere.
physically active over the past 15 years Therefore, the only effective way to reduce such
C. the number of North American children who are not emissions is to replace the conventional diesel fuel and
obese increased over the past 15 years gasoline used in automobiles with cleaner burning
D. over the past 15 years, the number of North American fuels, such as methanol, that create fewer emissions.
children who are underweight has declined Which one of the following is the course of action on
E. the incidence of obesity in North American children which the statement depends?
tends to increase as the children grow older A. Reducing the use of automobiles would not be a more
17. Statement :Most antidepressant drugs cause effective means to reduce automobile emissions than
weight gain. While dieting can help reduce the amount theuse of methanol.
of weight gained while taking such antidepressants, B. There is no fuel other than methanol that is cleaner-
some weight gain is unlikely to be preventable. burning than both diesel fuel and gasoline.
The information above most strongly supports which C. If given a choice of automobile fuels, automobile
one of the following course of action? owners would not select gasoline over methanol.

Page 1207 of 1334


Subscribe The Xpress Video Course & Mock Test Package for Bank & Insurance Exams
If there are any suggestions/ errors in our PDFs Feel Free to contact us via this email: admin@exampundit.in
Ultra Practice Bundle PDF
SBI Clerk/ RBI Assist. Mains – Reasoning
D. Automobile emissions constitute the most serious 20. Statement :A democratic society cannot exist
threat to the global environment. unless its citizens have established strong bonds of
E. At any given time there is a direct correlation between mutual trust. Such bonds are formed and strengthened
the level of urban air pollution and the level of only by participation in civic organizations, political
contaminationpresent in the global atmosphere. parties, and other groups outside the family. It is
19. Statement :In humans, ingested protein is broken obvious then that widespread reliance on movies and
down into amino acids, all of which must compete to electronic media for entertainment has an inherently
enter the brain. Subsequent ingestion of sugars leads corrosive effect on democracy.
to the production of insulin, a hormone that breaks Which one of the following is the course of action on
down the sugars and alsorids the bloodstream of which the columnist’s statement depends?
residual amino acids, except for tryptophan. A. Anyone who relies on movies and electronic media for
Tryptophan then slips into the brain uncontested and entertainment is unable to form a strong bond of
is transformed into the chemical serotonin, increasing mutualtrust with a citizen.
the brain’s serotonin level. Thus, sugars can play a B. Civic organizations cannot usefully advance their goals
major role in mood elevation, helping one to feel by using electronic media.
relaxed and anxiety-free. C. Newspapers and other forms of print media strengthen,
Which one of the following is the course of action on rather than weaken, democratic institutions.
which the statement depends? D. Relying on movies and electronic media for
A. Elevation of mood and freedom from anxiety require entertainment generally makes people less likely to
increasing the level of serotonin in the brain. participate ingroups outside their families.
B. Failure to consume foods rich in sugars results in E. People who rely on movies and electronic media for
anxiety and a lowering of mood. entertainment are generally closer to their families than
C. Serotonin can be produced naturally only if tryptophan arethose who do not.
is present in the bloodstream. Statement and inferences
D. Increasing the level of serotonin in the brain promotes 21. Statement :The Civil Service Act of 1889, also
relaxation and freedom from anxiety. known as the Pendleton Act, which created a
E. The consumption of protein-rich foods results in professional corps of administrators, was passed after
anxiety and a lowering of mood. a disappointed office-seeker assassinated President
James A. Garfield. For a hundred years, this system

Page 1208 of 1334


Subscribe The Xpress Video Course & Mock Test Package for Bank & Insurance Exams
If there are any suggestions/ errors in our PDFs Feel Free to contact us via this email: admin@exampundit.in
Ultra Practice Bundle PDF
SBI Clerk/ RBI Assist. Mains – Reasoning
has anchored American government service to (b) Bob’s Bistro is closed on Sundays, and so for Sunday
competence rather than corruption. The best way to brunch residents of Montrose would either go to Andrew’s
preserve this state in the new millennium is to maintain Eatery or venture to a neighbouring town.
the Pendleton Act as it is. (c) The profit per meal is higher, on average, at Bob’s
Inferences: Bistro than it is at Andrew’s Eatery.
(a) The methods that the Swiss and British governments (d) Bob’s Bistro attracts a large number of patrons that had
have used to prevent corruption in government service for never dined in Montrose before, and on many days Bob’s
the past one hundred years. Bistro attracts more customers than its seats.
(b) The current level of job satisfaction among (e) none of the above
government office-seekers and office-holders Which of the above, if true, provides the best inference
(c) The levels of competence and corruption in American for the given statement?
government service between 1950 and the present 22.Statement : When Bob’s Bistro opened in the town
(d) The number of Presidents assassinated since the of Montrose last year, the proprietors of Andrew’s
passage of the Pendleton Act Eatery, the only other restaurant in town, feared that
(e) none of the above their business would suffer. Surprisingly though, in
Which of the above would be the statement’s the past year the average number of meals per day
inference? served at Andrew’s Eatery has actually increased
22.Statement : When Bob’s Bistro opened in the town significantly.
of Montrose last year, the proprietors of Andrew’s Inferences:
Eatery, the only other restaurant in town, feared that (a) The meals at Andrew’s Eatery are substantially lower
their business would suffer. Surprisingly though, in in price than those offered at Bob’s Bistro.
the past year the average number of meals per day (b) Bob’s Bistro is closed on Sundays, and so for Sunday
served at Andrew’s Eatery has actually increased brunch residents of Montrose would either go to Andrew’s
significantly. Eatery or venture to a neighbouring town.
Inferences: (c) The profit per meal is higher, on average, at Bob’s
(a) The meals at Andrew’s Eatery are substantially lower Bistro than it is at Andrew’s Eatery.
in price than those offered at Bob’s Bistro. (d) Bob’s Bistro attracts a large number of patrons that had
never dined in Montrose before, and on many days Bob’s
Bistro attracts more customers than its seats.

Page 1209 of 1334


Subscribe The Xpress Video Course & Mock Test Package for Bank & Insurance Exams
If there are any suggestions/ errors in our PDFs Feel Free to contact us via this email: admin@exampundit.in
Ultra Practice Bundle PDF
SBI Clerk/ RBI Assist. Mains – Reasoning
(e) none of the above If the above statements are true, they support which of
Which of the above, if true, provides the best inference the given inferences?
for the given statement? 24. Statement : One of the most vexing problems in
23. Statement:Calorie restriction, a diet high in historiography is dating an event when the usual
nutrients but low in calories, is known to prolong the sources offer conflicting chronologies of the event.
life of rats and mice by preventing heart disease, Historians should attempt to minimize the number of
cancer, diabetes, and other diseases. A six-month study competing sources, perhaps by eliminating the less
of 48 moderately overweight people, who each reduced credible ones. Once this is achieved and several sources
their calorie intake by at least 25 percent, are left, as often happens, historians may try, though
demonstrated decreases in insulin levels and body on occasion unsuccessfully, to determine
temperature, with the greatest decrease observed in independently of the usual sources which date is more
individuals with the greatest percentage change in likely to be right.
their calorie intake. Low insulin level and body Inferences:
temperature are both consideredas signs of longevity, A. We have no plausible chronology of most of the events
partly because an earlier study by other researchers for which attempts have been made by historians
found both traits in long-lived people. todetermine the right date.
Inferences: B. Some of the events for which there are conflicting
(a) Calorie restriction produces similar results in humans chronologies and for which attempts have been made
as it does in rats and mice. byhistorians to determine the right date cannot be dated
(b) Humans who reduce their calorie intake by at least 25 reliably by historians.
percent on a long-term basis will live longer than they C. Attaching a reliable date to any event requires
wouldhave had they not done so. determining which of several conflicting chronologies is
(c) Calorie intake is directly correlated to insulin level in most likelyto be true.
moderately overweight individuals. D. Determining independently of the usual sources which
(d) Individuals with low insulin levels are healthier than of several conflicting chronologies is more likely to be
individuals with high insulin levels. rightis an ineffective way of dating events.
(e) Some individuals in the study reduced their calorie E. The soundest approach to dating an event for which the
intake by more than 25 percent. usual sources give conflicting chronologies is to

Page 1210 of 1334


Subscribe The Xpress Video Course & Mock Test Package for Bank & Insurance Exams
If there are any suggestions/ errors in our PDFs Feel Free to contact us via this email: admin@exampundit.in
Ultra Practice Bundle PDF
SBI Clerk/ RBI Assist. Mains – Reasoning
underminethe credibility of as many of these sources as construction of underground and multi-level parking
possible. spaces especially in and around market areas.
Which one of the above inferences is most strongly Inferences:
supported by the statement given above? (a) The government hopes to meet the present demand for
25. Statement: The government is set to overhaul parking space especially in the congested areas within the
annual targets for public sector lenders, ending a focus city.
on size that has long encouraged banks to initiate their (b) Some traffic from neighbouring towns is diverted to
loans and deposits at the year-end to hit growth pass through the city which creates additional traffic
objectives. Banking and government sources said that congestion.
the new targets, to be discussed at meeting with top (c) By providing additional parking space in areas in and
state bank officials, would focus on efficiency, with around the market, the government is ignoring the greater
objectives set around return on assets, or return on need for parking space in residential areas.
equity, and controlling bad debts. (d) Issues related to traffic congestion cannot be resolved
Inferences: unless the government ensures that all traffic rules are
(a) Loans and deposits of the banks will go up. obediently followed by drivers and pedestrians.
(b) Loans on which there is maximum probability of (e) none of the above
return will be encouraged. Which of the above can be inferred from the above
(c) Number of bad debts per financial year will increase. statement?
(d) Financial stability of the banks will be lost. 27. Statement: It was co-incidental that two separate
(e) none of the above teams of scientists discovered the remains of
Which of the above points can be inferred from the tyrannosauruses on two separate locations of the same
given statement? island. The first team found that the tyrannosaurus,
26. Statement: With the increase in the number of which was a female, had developed wings; however, it
vehicles in the city, the government is trying hard to was not possible for it to fly and support its weight
resolve issues related to available space for parking during its flight. The other team discovered that the
vehicles in public areas. Around one thousand new tyrannosaurus, which was a male, had no traces of
vehicles add to the present traffic situation daily. This wings and its structure was no different from what was
causes further problems of traffic congestion. In order already known to the scientists. Eventually, both the
to resolve the issues, the government has suggested the discoveries made the scientists and island dwellers

Page 1211 of 1334


Subscribe The Xpress Video Course & Mock Test Package for Bank & Insurance Exams
If there are any suggestions/ errors in our PDFs Feel Free to contact us via this email: admin@exampundit.in
Ultra Practice Bundle PDF
SBI Clerk/ RBI Assist. Mains – Reasoning
realize that the island contained many undiscovered I. It would be easier to replace or do without a person with
treasures and answers. general knowledge than a person withspecialized
Inferences: knowledge.
(a) There are certain islands where traces of dinosaurs and II. A decision that affected only one area of government
other ancient species can still be unearthed and studied to would not require Advani’s expertise.
fill in the gaps. III. Advani’s knowledge of each separate area of
(b) The history of evolution provides ample evidence to government is as profound as that of a specialist.
justify the difference between the male and female species If the statements above are true, which inferences
of a race. must also be true?
(c) It is possible that some female tyrannosauruses (a) I only
developed wings, but, whether this physical feature was (b) II Only
only specific to females is open to further discovery. (c) III only
(d) The tyrannosaurus was the only species of dinosaurs (d) II and III only
that had wings but could not fly due to their humungous (e) all I, II and III
bodyweight. 29. Statement:Sociologist and political observers are of
(e) none of the above the opinion that making educational qualification
Assuming that the above statement is true, identify mandatory for election candidates would be
which of the above can be inferred from the statement? discriminatory since that would naturally deprive
28. Statement: Advani’s role in the governments is citizens of India ofthe right to represent their people.
unique. Whereas all other state officials are specialists Institutional qualification, as they say, is different
in one area or another, Advani is the supreme from political acumen and wisdom. In a country like
generalist, the single person who can determine how India where about a quarter of 815 million eligible
various interrelated areas of government are likely to voters are illiterate, it would not be prudent to deny a
be affected by a particular course of action. As a politically aware and socially conscious individual the
consequence, Advani is the only truly indispensable election ticket just because he/she doesn’t have a
person in the government minimum qualification.
Inferences: Inferences :
I. No,mere literacy is no guarantee of political maturity of
an individual.

Page 1212 of 1334


Subscribe The Xpress Video Course & Mock Test Package for Bank & Insurance Exams
If there are any suggestions/ errors in our PDFs Feel Free to contact us via this email: admin@exampundit.in
Ultra Practice Bundle PDF
SBI Clerk/ RBI Assist. Mains – Reasoning
II. Yes, Illiterate people are less likely to make politically Which of the inferences numbered (I), (II), (III), (IV),
wiser decisions of voting for a right candidate or party. and (V) can be concluded from the fact/information
III. No, Voting is the constitutional right of every citizen. given in the statement?
Should “literacy” be the minimum criterion for (a) Only II
becoming a voter in India? (b) Only II and V
(a) Only I and II (c) Only III and IV
(b) Only III (d) Only IV
(c) Only II and III (e) Only III
(d) Only II Statement and Assumption
(e) All I, II and III 31. Statement : Publicity campaigns for endangered
30. Statement:“Poor growth in corporate tax species are unlikely to have much impact on the most
collection pulled down overall rise in gross direct tax important environmental problems, for while the ease
mobilization in the first half of the current year, ruling of attributing feelings to large mammals facilitates
out any pickup in growth in the first half of the year, evoking sympathy for them, it is more difficult to elicit
indicating that the government will find it difficult to sympathy for other kinds of organisms, such as the soil
meet its tax target for the year.” microorganisms on which large ecosystems and
Inferences: agriculture depend.
(I) Corporate tax collection rose only in the first quarter of Which one of the following is an assumption on which
the current year. the argument in the given statement depends?
(II) Industrial production slowed down in the first half of A. The most important environmental problems involve
the year. endangered species other than large mammals.
(III) The government has achieved the target of corporate B. Microorganisms cannot experience pain or have other
tax collection in comparison to previous year. feelings.
(IV) Poor tax collection in the corporate pulled down the C. Publicity campaigns for the environment are the most
economy. effective when they elicit sympathy for some organism.
(V) The government is likely to face difficulties in D. People ignore environmental problems unless they
achieving the target of collection of corporate tax due to believe the problems will affect creatures with which they
slowdown in production and the government has forecast sympathize.
a growth in corporate tax.

Page 1213 of 1334


Subscribe The Xpress Video Course & Mock Test Package for Bank & Insurance Exams
If there are any suggestions/ errors in our PDFs Feel Free to contact us via this email: admin@exampundit.in
Ultra Practice Bundle PDF
SBI Clerk/ RBI Assist. Mains – Reasoning
E. An organism can be environmentally significant only if woman. Legalizing gay marriage would be changing
it affects large ecosystems or agriculture. thousands of years of tradition. The purpose of
32. Statement: Leibniz, the seventeenth-century marriage is to procreate, and same–sex couples can’t
philosopher, published his version of calculus before have children. If we legalize gay marriage, it’s a
Newton did. But then Newton revealed his private slippery slope to polygamy, incest, and/or bestiality.
notebooks, which showed he had been using these Children raised by a mother and a father are more
ideas for at least a decade before Leibniz’s publication. emotionally well–adjusted than those raised by same–
Newton also claimed that he had disclosed these ideas sex parents etc. No concrete justification has ever been
to Leibniz in a letter shortly before Leibniz’s presented for their claims that gay marriage is wrong.
publication. Yet close examination of the letter shows Which of the following assumptions if true,
that Newton’s few cryptic remarks did not reveal strengthens the statement above?
anything important about calculus. Thus, Leibniz and a) The legal rights and benefits of heterosexual couples
Newton each independently discovered calculus. are completely unaffected by the existence of gay
Which one of the following is an assumption required marriage.
by the historian’s argument in the given statement? b) Marriage rates have been declining steadily since the
A. Leibniz did not tell anyone about calculus prior to 1970s, decades before any U.S. jurisdictions legalized gay
publishing his version of it. marriage.
B. No third person independently discovered calculus c) Long–standing tradition and unfounded fears are not
prior to Newton and Leibniz. good reasons to not legalize gay marriage.
C. Newton believed that Leibniz was able to learn d) An estimated 16,000 same–sex couples are raising
something important about calculus from his letter to him. more than 22,000 adopted children in the U.S., and these
D. Neither Newton nor Leibniz knew that the other had findings indicate that these children will likely fare no
developed a version of calculus prior to Leibniz’s differently, as a result of their family type, than those
publication. being raised by heterosexual parents.
E. Neither Newton nor Leibniz learned crucial details e) none of the above
about calculus from some third source. 34. Statement: Modern Medicine has extended our
33. Statement: Haters of gay marriage argue that gay lifespan and we can now treat disease and illness with
marriage harms the institution of traditional marriage more ease. Traditional medicine involved herbs and
since marriage has always been between a man and a plants which were not always effective since people still

Page 1214 of 1334


Subscribe The Xpress Video Course & Mock Test Package for Bank & Insurance Exams
If there are any suggestions/ errors in our PDFs Feel Free to contact us via this email: admin@exampundit.in
Ultra Practice Bundle PDF
SBI Clerk/ RBI Assist. Mains – Reasoning
died of simple illnesses such as chickenpox or even the Health care executives want to increase revenues while
common cold. Nowadays, medicine cannot always cure reducing costs. Consequently, they propose
us, but it can lessen the effectiveness of our symptoms significantly greater automation of health care. Yet,
and improve our condition. Traditional medicine is this should be rejected. Radical automation of health
called Traditional for a Reason : it is old fashioned and care would cause patients to lose trust in the system as
outdated. It is not as effective as Modern Medicine. the health care they would receive would lack the in–
Some traditional treatments work well but modern person care that studies show patients desire.
medicine just takes these traditional treatments and Which of the following assumption expresses the main
makes them more effective. Modern medicine can have point of the statement?
side effects, but if taken with regulation and not in a) Health care should not be heavily automated
overdose, modern medicine can be the most effective b) Patients desire customized in–person care
way to help yourself. c) Trends in the general economy do not apply to the
Which of the following assumptions if true, weakens health care industry
the statement that modern medicine is a better option d) Health care executives are becoming too greedy
compared to traditional medicine? e) none of the above
a) Recently there have been many law suits filed and won 36. Statement: From 1983 to 1999 total energy use in
by consumers against modern drug manufacturers, this country increased less than 10 percent. However,
because of harmful side effects of modern medicine. the use of electrical energy in this country during this
b) Modern medicine is recommended because there is same period grew by more than 50 percent, as did the
more profit for everyone involved. gross national product— the total value of all goods
c) Traditional medicine has been researched and has and services produced in the nation.
proved to work. If the statements above are true, which one of the
d) The dangerous side effects of modern medicines have following must be the assumption?
been well documented. Traditional medicines do not have A. Most of the energy used in this country in 1999 was
any harmful side effects. electrical energy.
e) none of the above B. From 1983 to 1999 there was a decline in the use of
35. Statement: Automation, the trademark of a energy other than electrical energy in this country.
modern economy, is essential to maximize a country’s
economic production while minimizing its costs.

Page 1215 of 1334


Subscribe The Xpress Video Course & Mock Test Package for Bank & Insurance Exams
If there are any suggestions/ errors in our PDFs Feel Free to contact us via this email: admin@exampundit.in
Ultra Practice Bundle PDF
SBI Clerk/ RBI Assist. Mains – Reasoning
C. From 1983 to 1999 there was an increase in the D. There are more low-crime districts than high-crime
proportion of energy use in this country that consisted of districts in city Y.
electrical energy use. E. Districts of the city from which police officers are
D. In 1999 electrical energy constituted a larger removed do not experience significant crime increases
proportion of the energy used to produce the gross shortly after the removal of those officers.
national product than did any other form of energy. 38. Statement: Although there has been great scientific
E. In 1983 the electrical energy that was produced debate for decades over global warming, most
constituted a smaller proportion of the gross national scientists now agree that human activity is causing the
product than did all other forms of energy combined. Earth’s temperature to rise. Though predictions vary,
37. Statement: To decrease the number of crimes in many global warming experts believe that average
city Y, the city’s Police Commissioner proposed taking global temperatures will rise between three and eight
some police officers from low-crime districts of the city degrees Fahrenheit during the next century. Such an
and moving them to high-crime districts of the city. His increase would cause an alarming rise in sea levels,
proposal is based on city Y crime data that show that displacing millions of people by destroying major
the number of crimes in any district of the city population centers along the world’s coastlines.
decreases when additional police officers are moved Which of the following is an assumption in support of the
into that district. above statement?
The Police Commissioner’s proposal depends on which of A. New technological developments in the next century
the following assumptions? will not divert rising seas from the world’s coastal cities.
A. City X experienced a drastic reduction in crime after B. Individuals will not become more aware of the steps
implementing a proposal similar to that proposed by the they can take to reduce the emission of greenhouse gases.
Police Commissioner of city Y. C. Rising sea levels similarly affect all coastal population
B. The severity of crimes committed in any district of the centers.
city decreases when additional police officers are moved D. Some global warming experts predict a greater than
into that district. eight degree Fahrenheit increase in global temperatures
C. The number of crimes committed in all high-crime during the next century.
districts of city Y is more than triple the number of crimes E. Human activity is the sole cause of increasing global
committed in all low-crime districts of city Y. temperatures.

Page 1216 of 1334


Subscribe The Xpress Video Course & Mock Test Package for Bank & Insurance Exams
If there are any suggestions/ errors in our PDFs Feel Free to contact us via this email: admin@exampundit.in
Ultra Practice Bundle PDF
SBI Clerk/ RBI Assist. Mains – Reasoning
39. Statement: Researchers studying the spread of the peel away when the temperature exceeds 95 degrees
Black Plague in sixteenth-century England claim that Fahrenheit, leaving the tree with fewer rings than it
certain people survived the epidemic because they would otherwise have. So only if the temperature in the
carried a genetic mutation, known as Delta-32, that is Brazilian ash’s environment never exceeds 95 degrees
known to prevent the bacteria that causes the Plague Fahrenheit will its rings be a reliable measure of the
from overtaking the immune system. To support this tree’s age.
hypothesis, the researchers tested the direct Which of the following is an assumption on which the
descendants of the residents of an English town where statement above depends?
an unusually large proportion of people survived the A. The growth of new rings in a tree is not a function of
Plague. More than half of these descendants tested levels of precipitation.
positive for the mutation Delta-32, a figure nearly B. Only the Brazilian ash loses rings because of excessive
three times higher than that found in other locations. heat.
The researchers’ hypothesis is based on which of the C. Only one day of temperatures above 95 degrees
following assumptions? Fahrenheit is needed to cause the Brazilian ash to lose a
A. Delta-32 does not prevent a carrier from contracting ring.
any disease other than the Plague. D. The internal rings of all trees are of uniform thickness.
B. The Plague is not similar to other diseases caused by E. The number of rings that will be lost when the
bacteria. temperature exceeds 95 degrees Fahrenheit is not
C. Delta-32 did not exist in its current form until the predictable.
sixteenth century. Statement and conclusions
D. No one who tested positive for Delta-32 has ever 41. Statement :Due to high jet fuel costs, airline
contracted a disease caused by bacteria. carriers are looking for new ways to increase revenues
E. The Plague does not cause genetic mutations such as and thereby counteract declining profits. Airline X has
Delta-32. proposed increasing the number of passengers that
40. Statement: The popular notion that a tree’s age can can fit on its airplanes by creating several standing
be determined by counting the number of internal rooms only “seats” in which passengers would be
rings in its trunk is generally true. However, to help propped against a padded backboard and held in place
regulate the internal temperature of the tree, the with a harness. This proposal, since it relates to
outermost layers of wood of the Brazilian ash often

Page 1217 of 1334


Subscribe The Xpress Video Course & Mock Test Package for Bank & Insurance Exams
If there are any suggestions/ errors in our PDFs Feel Free to contact us via this email: admin@exampundit.in
Ultra Practice Bundle PDF
SBI Clerk/ RBI Assist. Mains – Reasoning
passenger safety, cannot be implemented without e. none of the above
prior approval by the Aviation Administration. 43. Statement:In a game of cricket, if a player takes a
Which of the following is the exact conclusion of the position as wicketkeeper, he must also take up the role
above statement? of the first slip fielder and a motivator. If the fielder is
a. The addition of standing room only “seats” will occupying the second slip, he must occupy second slip
generate more revenue than the cost of ensuring that these and either the first slip or the motivator’s role. If he is
seats meet safety standards. a motivator, he is also a second slip fielder.
b. The Aviation Administration will approve Airline X’s If the player described above is not a motivator, which of
specific proposal. the following conclusions can be drawn?
c. The revenue generated by the addition of standing room a) The player occupies the wicket keeping position.
only “seats” is greater than the current cost of jet fuel. b) The player is a motivator and first slip fielder but is not
d. There are no safer ways in which Airline X can increase a wicket keeper.
revenues. c) The player is not wicketkeeper.
e. Passenger safety is less important than increasing d) The player is a second slip and first slip fielder.
revenue. e) none of the above
42. Statement: All the employees of Law Prep 44) Statement:News channels rarely cover local
Tutorial must enroll in the company’s medical politics thoroughly, and local political business is
insurance programme. Some, but not all of the usually conducted secretively. These factors each tend
employees’ spouses have enrolled in the same to isolate local politicians from their electorates. This
programme. No one who is not enrolled in the has the effect of reducing the chance that any
company’s medical insurance programme may enter particular act of resident participation will elicit a
the Bhopal centre. positive official response, which in turn discourages
Based on the information above, which of the following resident participation in local politics.
conclusions must be true? Which one of the following is the conclusion supported by
a. Some employees’ spouses are also employees of the the above statement?
company. a) If local politicians were less isolated from their
b. No employees’ spouses are employees of the company. electorate, acts of resident participation would be likely to
c. No employee’s spouses may enter the Bhopal Centre. elicit a positive response from them.
d. all the above

Page 1218 of 1334


Subscribe The Xpress Video Course & Mock Test Package for Bank & Insurance Exams
If there are any suggestions/ errors in our PDFs Feel Free to contact us via this email: admin@exampundit.in
Ultra Practice Bundle PDF
SBI Clerk/ RBI Assist. Mains – Reasoning
b) Local political business should be conducted less d) Those who have reported sightings of the White rhino
secretively because this would encourage resident are not experienced naturalists or foresters.
participation in local politics. e) none of the above
c) A resident’s decision as whether to participate in local 46. Statement : All the pain and suffering comes
politics is mostly dependent on the chance that the from the stupidity of humans. Their stupidity to
participation will elicit a positive official response. believe nonsense and force this nonsense on others is
d) More–frequent thorough coverage of local politics by what causes pain and suffering in the world. In the
news channels would reduce at least one source of words of the great philosopher called Voltaire,” If you
discouragement from resident participation in local believe in absurdities, you can commit atrocities”.
politics. What do you conclude from the author’s point of view
e) none of the above about a person who believes in ghosts?
45) Statement :Forester -- The recent claims that the a) The person is absurd and should change his beliefs.
White rhino is not extinct are false. The White rhino’s b) The person’s beliefs are dangerous to others.
natural habitat was taken over by deforestation, c) The author does not say anything about ghosts.
resulting in the animal’s systematic elimination from d) Voltaire was a great philosopher.
the area. Since then naturalists working in the region e) none of the above
have discovered no hard evidence of its survival, such 47. Statement : At an enormous research cost, a
as carcasses or tracks. In spite of alleged sightings of leading chemical company has developed a
the animal, the White rhino no longer exists. manufacturing process for converting wood fibers into
Which one of the following is the conclusion on which the a plastic. According to the company, this new plastic
forester’s statement depends? can be used for, among other things, the hulls of small
a) Deforestation drove the last White rhinos to starvation sailboats. But what does the company think sailboat
by chasing them from their natural habitat. hulls used to be made of ? Surely the mania for high
b) Every naturalist working in the White rhino’s natural technology can scarcely go further than this.
habitat has looked systematically for evidence of the The author’s conclusion of the manufacturing process
rhino’s survival. described in the statement is based primarily on the fact
c) The White rhino did not move and adapt to a different that ?
region in response to the loss of habitat. (a) plastic is unlikely to be durable enough for high-
quality sailboat hulls

Page 1219 of 1334


Subscribe The Xpress Video Course & Mock Test Package for Bank & Insurance Exams
If there are any suggestions/ errors in our PDFs Feel Free to contact us via this email: admin@exampundit.in
Ultra Practice Bundle PDF
SBI Clerk/ RBI Assist. Mains – Reasoning
(b) the research costs of developing the process outweigh d. Any high school senior who contributes to food, shelter,
any savings possible from the use of the plastic or clothing for himself or hisfamilyhas significant
(c) a small sailboat is not normally regarded as a high-tech financial responsibilities.
product e. The majority of high school students have no financial
(d) hulls for small sailboats can be made from wood responsibilities to their families.
without converting it into plastic 49. Statement : Students who consistently receive low
e) none of the above scores on standardized tests have also demonstrated
48. Statement : According to a recent study on lower than average levels of self-esteem. As a result,
financial roles, one-third of high school seniors say that educators believe that lowering the requirements for
they have “significant financial responsibilities.” success on standardized tests will increase students’
These responsibilities include, but are not limited to, level of self-esteem.
contributing to food, shelter, or clothing for Which of the following, if true, concludes and contradicts
themselves or their families. At the same time, a second about the point of view of the educators in the above
study demonstrates that a crisis in money management statement?
exists for high school students. According to this study, a) A person’s level of self-esteem is usually established by
80% of high school seniors have never taken a personal the time he or she begins attending school and does not
finance class even though the same percentage of change.
seniors has opened bank accounts and one-third of b) The scales for standardized tests are based on a
these account holders have bounced a check. nationwide sample tests and not on any local faction.
Which of the following conclusions can be properly c) Changing the grading scale for standardized tests will
drawn from the statements above? have no effect on the students’ ability to succeed in
a. High schools would be wise to incorporate personal college or graduate school.
finance classes into their core curricula. d) Self-esteem is a very fragile part of a person’s
b. At least one-third of high school seniors work part-time psychology and is highly susceptible to changes caused by
jobs after school. the outside faction.
c. The number of high school seniors with significant e) none of the above
financial responsibilities is greater than the number of 50. Statement :Donors are almost never offended by
seniors who have bounced a check. being asked for too much (in fact, they are usually
flattered.) And if you ask for too much, our donor can

Page 1220 of 1334


Subscribe The Xpress Video Course & Mock Test Package for Bank & Insurance Exams
If there are any suggestions/ errors in our PDFs Feel Free to contact us via this email: admin@exampundit.in
Ultra Practice Bundle PDF
SBI Clerk/ RBI Assist. Mains – Reasoning
always suggest a smaller amount. On the other hand, b) a good fund-raiser will value the worth of the donor.
donors are frequently offended by being asked for too c) it is worth the gamble to ask for large donations.
little. A common reaction is, “So, that’s all they think d) fund-raisers often think that donors are incapable of
I’m worth.” giving much.
The above statement concludes that ? e) none of the above
a) donors are usually never asked for enough.

Critical Reasoning – Answer and Explanation


1. Answer: B have resulted in a short supply of vegetables and hence an
Statement II is the cause and statement I is its effect increase in their prices.
.Clearly, the university's decision came as a sequel to the
complaint received by it from the college teachers against 5. Answer: C
use of mobile phones in the college premises. Both the statements I and II are independent causes .The
given statements are self-sufficient and depict
2. Answer: E independent events.
Both the statements I and II are effects of some common
cause 6. Answer: D
The facts given in both the statements are clearly the result Both the statements I and II are effects of independent
of acute power shortage. causes . The students of government schools performing
well in the examinations and the teachers of government
3. Answer: C schools leaving their jobs to join private schools are two
Both the statements I and II are independent causes . An separate situations that must have been triggered by
increase in the cases of atrocities on women and the police independent causes.
being unable to nab the culprits involved in the same are
independent happenings in themselves. 7. Answer: B
Statement II is the cause and statement I is its effect. The
4. Answer: B reduction in procurement price of crops must have
Statement II is the cause and statement I is its effect . instigated the farmers not to sell their produce to
Clearly, damage to crops due to high temperature may Government agencies.
Page 1221 of 1334
Subscribe The Xpress Video Course & Mock Test Package for Bank & Insurance Exams
If there are any suggestions/ errors in our PDFs Feel Free to contact us via this email: admin@exampundit.in
Ultra Practice Bundle PDF
SBI Clerk/ RBI Assist. Mains – Reasoning
elementsoccurred and the answer presents an accurate
8. Answer: B course of action of the way theauthor made his or her
Statement II is the cause and statement I is its effect . The statement, hence this answer is correct.
government must have seen the unawareness of the people
as a strong factor in the primary education programme 12. Answer :B
being not successful. The step indicated in I must, thus, A series of minor earthquakes (minor events) weaken
have been sought for as a remedy for the same. theexisting infrastructure. The weakened infrastructure
then allows a minorearthquake to do greater damage (an
9. Answer : A extreme response) than would a single major earthquake,
Since the Competition Act provides for, keeping in view hence this answer is correct.
of the economic development of the country, statement 2
becomes the effect of the cause mentioned in statement 1. 13. Answer :E
We can follow the chain of connections in thestimulus to
10. Answer : B prove this answer: the ozone layer filters some
According to statement 2, only registered proprietors have wavelengths oflight; holes in the ozone layer are
all the rightsto use the trademark in relation to the goods dangerous, but one previously overlookeddanger of the
or service, hence this becomes the cause and also, in holes is possible eye damage for many species. From
statement 1 the limitations of Unregistered trademark thesetwo statements we can infer that the holes must be
proprietors have been mentioned, this becomes the effect. letting some damagingwavelengths of light through. This
is essentially what answer choice (E)states.
11. Answer :C
Consider each piece of the statement: 14. Answer :B
“providinginformation”—a variety of information about The statement tells the story of recentphysics theories:
the garbage situation is provided.“application of a general Newtonian physics was preeminent for over two
rule”—the general rule is that “After a public holidaythat centuries,and despite widespread acknowledgment and
falls on a Monday, garbage throughout the city is confirmation it was surpassedby Einsteinian physics in the
supposed to becollected one day later than usual.” “to a early 1900s.Asshown by the case of Newtonian physics,
specific case”—the specific case isthe pickup of garbage success and substantiation is noguarantee of dominance.
this week in this neighborhood. Given that all

Page 1222 of 1334


Subscribe The Xpress Video Course & Mock Test Package for Bank & Insurance Exams
If there are any suggestions/ errors in our PDFs Feel Free to contact us via this email: admin@exampundit.in
Ultra Practice Bundle PDF
SBI Clerk/ RBI Assist. Mains – Reasoning
15. Answer :C theconditional nature of the conclusion and so the author
The course of action to this statement is the fourth assumes that thispossibility cannot occur. This answer
sentence, which begins with theconclusion indicator “For clearly shows that there are other, more effective ways
this reason...” By applying the Primary Objectivesyou ofreducing emissions and therefore thismust be the correct
should have identified this course of action while reading, answer.
and then, uponclassifying the question stem you should
have looked for a paraphrase of thissentence. Answer 19. Answer :D
choice (C) fits the bill, and is the correct answer. Thisis the correct answer. The author states that after the
action of the sugars,more serotonin enters the brain. The
16. Answer :C author then concludes that this leads toa mood elevation.
Answer choice (C) is fully supported because the Thus, we can conclude a course of action based on the
Statement providesinformation about both the number and fact in the statement that serotonin has an effect onthe
percentage of obese children. Asstated earlier, if the mood level.
stimulus provides information about both the numbersand
percentages in a situation, then you can select any 20. Answer : D
supported answerchoice that contains either numbers or The assumption that is neededtherefore, is to show that
percentages. Note the emphasis onthe word “supported.” movies and other electronic media somehow lead toa
lessening of participation in civic organizations, political
17. Answer :C parties, and othergroups outside the family. This is the
This is the correct answer. Some individuals connection made in answer choice (D),hence it is the
takingantidepressants that cause weight gain will gain correct answer.
weight even though dietingcan reduce the amount of the
gain.
21. Answer: C
18. Answer :A This would confirm or deny the inference that American
Consider the author’s position that the only effective way government service is currently not corrupt. Thestatement
to reduceemissions is fuel replacement. Wouldn’t it be only claims that this was so for the first hundred years
more effective to simply stopusing cars altogether? Of after the passage of the Pendleton Act. There is no
course this is true, but this would undermine

Page 1223 of 1334


Subscribe The Xpress Video Course & Mock Test Package for Bank & Insurance Exams
If there are any suggestions/ errors in our PDFs Feel Free to contact us via this email: admin@exampundit.in
Ultra Practice Bundle PDF
SBI Clerk/ RBI Assist. Mains – Reasoning
information about the level of corruption for the past (C) The study observed that individuals with the greatest
twenty-some years. percentage decrease intheir calorie intake demonstrated
the greatest decrease in insulin levels and
22. Answer: D bodytemperature. This shows a strong correlation
This statement explains that the number of diners in the between calorie intake and insulinlevels. However, this
town of Montrose as a whole has increased due to the correlation is not necessarily direct.
opening of Bob’s Bistro, and also explains why the (D) The study makes no reference to the health of
patronage at Andrew’s Eatery has increased. Bob’s Bistro individuals who reduce theircalorie intake. It tries to draw
has apparently brought in many out-of-towners to eat at some connection to the longevity of thoseindividuals, but
Montrose, and many of these first-time diners, when longevity is not the same as health.
turned away from Bob's Bistro, arrive at Andrew's (E) This choice is correct. Thestatement states that the
Eateryas the only alternative in town. greatest decrease in insulin levelswas observed in
individuals with the greatest percentage change in their
23. Answer : E calorieintake. This means that some individuals in the
The statement provides some specific information about study reduced their calorie intakeby a greater percentage
the effects of calorierestriction. In rats and mice, this diet than other individuals in the study. The statement
is known to prolong life by preventingdiseases. In a study alsostates that the study participants reduced their
of moderately overweight humans, insulin levels and individual calorie intakes by "atleast 25 percent." Thus,
bodytemperature decreased. one can safely infer that there were some participantswho
(A) The statement states that calorie restriction in mice reduced their calorie intake by more than 25 percent.
and rats prolongs life bypreventing diseases. The human
study had much more limited findings – thatcalorie 24. Answer : B
restriction in moderately overweight humans decreases The stimulus is a fact set and offers asolution for dating
insulin levels and body temperature. an event when the usual sources offer
(B) While the passage indicates that certain traits known conflictingchronologies: 1. Minimize the number of
to be associated withlongevity are found in moderately competing sources, possibly byeliminating the less
overweight humans who reduce their calorieintake, this is credible ones. 2. Independent of the usual
far removed from the conclusion that calorie intake will sources,determine which date is more likely to be right.
actuallyincrease a human's lifespan. By definition, historiography is the writing of history.

Page 1224 of 1334


Subscribe The Xpress Video Course & Mock Test Package for Bank & Insurance Exams
If there are any suggestions/ errors in our PDFs Feel Free to contact us via this email: admin@exampundit.in
Ultra Practice Bundle PDF
SBI Clerk/ RBI Assist. Mains – Reasoning
(A):The stimulus discusses dates where there is conflict above statement as it is given that the government is
between sources. In noway does the stimulus support looking to overhaul yearly targets and control bad debts.
answer choice (A).
(B): This isthe correct answer. As stated in the last 26. Answer: A
sentence, historians are on occasion unsuccessful in Option (a) is the best choice because it is within the
determining independently the date of an event. If the purview of the statement. It is clear that the government
usualsources offered are in conflict about the date of a plans to take these steps in the hope of meeting the
particular event and ananalysis independent of the usual requirement for parking space in the city. This can be
sources fails to confirm a date, then a datecannot be inferred from the given statement as it is in sync with the
reliably determined for the event. government’s intentions presented above.
(C): About onethirdof all test takers choose this answer. Option (b) is incorrect because it suggests that diversion
The stimulus speaks specifically ofdating an event when of traffic from neighboringareas creates further problem;
the usual sources offer conflicting chronologies. however, this cannot be justified from the statement.
Thestimulus does not discuss dating an event when there Option (c) cannot be inferred from the statement because
is no conflict ofchronologies, and most likely many dates the argument pertains to the problem of parking space in
could be set with certainty in theabsence of any conflict. public areas within the city and not residential areas.
(D): The language of the answer choice istoo strong in Hence, this is an incorrect inference.
saying that an independent determination is an ineffective Option (d) is incorrect because it cannot be inferred from
wayof dating events. the statement. The statement does not refer to the
(E): This is another tricky answer, and just under aquarter problems related to following traffic rules. The behavior
of test takers incorrectly select this answer. The answer of the drivers on the road is also not discussed in the
claims thatthe soundest approach to dating an event is to statement which is why no inference can be drawn on this
undermine the credibility of asmany of the competing basis.
sources as possible.
27. Answer: C
25. Answer: B Option (a) pertains to other islands that contain traces of
As by focusing on efficiency the changes of loan recovery dinosaurs. This cannot be inferred on the basis of the given
will increase. So, Option (b) can be inferred from the statement which refers to the discovery made on a specific
island.

Page 1225 of 1334


Subscribe The Xpress Video Course & Mock Test Package for Bank & Insurance Exams
If there are any suggestions/ errors in our PDFs Feel Free to contact us via this email: admin@exampundit.in
Ultra Practice Bundle PDF
SBI Clerk/ RBI Assist. Mains – Reasoning
Option (b) is a general statement regarding the difference which requires practical awareness or everyday awareness
in the evolution of male and female species of a race. This of everyday events. Thus, I also holds strong. Besides,
does not specifically pertain to the subject in question – theConstitution has extended the right to vote equally to
tyrannosaurus. Hence, this cannot be inferred from the all its citizens. Hence, III also holds.
statement.
Option (c) is the best choice because it refers to a 30. Answer: B
possibility based on the information given in the Only inference (II) and (V) can be concluded. Inference
statement. It refers to the sudden discovery of wings and (III) talks about the positive aspect, which is contrary to
leaves it open for others to discover its possibility in some the details given in the passage. Nothing has been directly
male species as well. and in detail mentioned about inference(IV). Hence option
Option (d) is incorrect because there is no evidence (b) is correct.
present in the statement that justifies this statement. In
fact, wings are not a common physical feature in 31. Answer : A
tyrannosaurus which is quite clear from the discovery This is a challenging problem because two of the wrong
made by the two teams. answer choices are attractive. The argument in the given
statement itself is not overly complex, but you must pay
28. Answer: B attention to the language. Consider the conclusion of the
Since Advani’s speciality is that he knows how one single argument: “Publicity campaigns for endangered species
decision affects the working of all the departments ofthe are unlikely to have much impact on the most important
government. So any decision that affects the working of environmental problems.” Ask yourself, why is it that
only one area of the government does not requirethe these campaigns are unlikely to have much impact on the
expertise of Advani. most important problems? According to the premises, the
reason is that “it is more difficult to elicit sympathy for
29. Answer: E other kinds of organisms [than large mammals].” The
Clearly, illiterate people lack will power and maturity in reasoning shows that the author believes there is a
thoughts. They may easily be misled into false convictions connection between the important problems and
or lured into temptations to vote for a particular group. So, organisms that are not large mammals. This Supporter
statement II holds. However, a person is literate does not connection is perfectly reflected in answer choice (A), the
mean that he is conscious of all political movements, correct answer.

Page 1226 of 1334


Subscribe The Xpress Video Course & Mock Test Package for Bank & Insurance Exams
If there are any suggestions/ errors in our PDFs Feel Free to contact us via this email: admin@exampundit.in
Ultra Practice Bundle PDF
SBI Clerk/ RBI Assist. Mains – Reasoning
34. Answer: D
32. Answer : E The statement here is that “In general, modern medicine
Answer choice (E): This is the correct answer. The answer is a better option compared to traditional medicine.”
can be difficult because it is somewhat similar to answer Option (a) is unrelated because instead of comparing
choice (B), which many people already eliminated by the modern medicine with traditional medicine it only talks of
time they reached this answer. Answer (E) is different law suits against modern medicine manufacturers. Option
from answer (B) because it involves learning details from (b) is unrelated since it talks about profit. Profit does not
a third source. This is important because the conclusion mean modern medicine is not better than traditional
references the independent discovery of calculus, and so medicine. Option (c) is unrelated because instead of
the author must believe that neither Newton nor Leibniz comparing modern medicine with traditional medicine, it
learned anything substantial about calculus from other just talks about
sources. This elimination of an idea that weakens the research. Option (d) weakens the statement because it says
argument is the essence of a Defender answer choice. To that most modern medicines have harmful side effects
further confirm the answer, consider the negation of this whereas traditional medicines don’t.
answer choice (“neither...nor” becomes “either...or”):
“Either Newton or Leibniz learned crucial details about 35. Answer: A
calculus from some third source.” This negated answer A. This is the main point of the statement. The author
undermines the assertion that Leibniz and Newton each indicates, “Yet, this should be rejected,” where this refers
independently discovered calculus. Consequently, this is to “significantly greater automation of health care.”
the correct answer. B. The author argues against automation since it will “lack
33. Answer: C the in-person care that studies show patients desire.” The
The statement here is that homosexuals do not have the statement is against automation, not for in-person care. In
right to marry as it affects the traditional institution of other words, “patients desire customized in-person care”
straight marriage negatively. Option (a) is unrelated is used as a premise to support the rejection of automation.
because it doesn’t address the statement which Centre’s C. Although this is a possible inference that can be made
around tradition. Option (b) does not address the by piecing together the premises, it is not the statement
statement. Option (d) only talks about the impact on that is being advocated.
children.

Page 1227 of 1334


Subscribe The Xpress Video Course & Mock Test Package for Bank & Insurance Exams
If there are any suggestions/ errors in our PDFs Feel Free to contact us via this email: admin@exampundit.in
Ultra Practice Bundle PDF
SBI Clerk/ RBI Assist. Mains – Reasoning
D. The statement says nothing about executives becoming At the same time, you should be attracted to an answer
too greedy. Automation “should be rejected” because it such as (C) because it contains only percentage
“would cause patients to lose trust in the system.” information, and as it turns out, answer choice (C) is
correct.
36. Answer: C
We are given the following facts. From 1983 to 1999 total 37. Answer: E
energy use increased less than 10%. During this same The Police Commissioner's proposal hopes to decrease the
period, the use of electrical energy grew by more than number of crimes in city Y by shifting police officers from
50%. During this same period, the gross national product low-crime to high-crime districts. His proposal is based on
(GNP) grew by more than 50%. A careful examination of data that demonstrate that crime decreases when
the second sentence reveals that there is no stated additional police officers are moved into a district.
connection between the growth of the GNP and the However, the data do not mention anything about the
increase in the use of electrical energy. If you assume that effect on the districts from which the police officers were
the use of electrical energy somehow caused the growth removed. The commissioner's plan is based on the
of the GNP, you are guilty of making an unwarranted assumption that the movement of police officers will not
causal assumption. Because there is no stated connection have any adverse effect on the low-crime districts.
between the two other than the they both grew by more (A) While it is encouraging that a similar plan worked
than 50%, any answer that attempts to connect the two is successfully in City X, this fact is certainly not essential
incorrect. for the success of the plan in City Y. The cities may be so
Answer choices (D) and (E) can both be eliminated by this different as to make the comparison meaningless.
reasoning. (B) The police commissioner's proposal is focused solely
A close analysis also reveals that answer choice (B) can on decreasing the number of crimes in city Y. The severity
be eliminated. of the crimes has no bearing on whether the
you should be very suspicious of answer choice (A) commissioner's proposal will succeed or not.
(which states that the number of electrical units used was (C) The actual numerical distinction between high and
greater) and answer choice (B) (which states that the use low-crime areas of the city is immaterial to the
of non- electrical energy declined) since they both contain commissioner's proposal.
numerical information. (D) It would be practically beneficial to the
commissioner's plan if there were more low crime than

Page 1228 of 1334


Subscribe The Xpress Video Course & Mock Test Package for Bank & Insurance Exams
If there are any suggestions/ errors in our PDFs Feel Free to contact us via this email: admin@exampundit.in
Ultra Practice Bundle PDF
SBI Clerk/ RBI Assist. Mains – Reasoning
high crime districts in city Y. This would enable the (C) The statement never suggests that all coastal
movement of police officers to every high crime district. population centers are similarly affected; this choice is too
However, this is not necessary to achieve the extreme and overreaching for the argument’s conclusion.
commissioner's goal of decreasing the total number of (D) This might be true, but it is not an assumption on
crimes in city Y. which the statement rests. Instead, this answer choice is
(E) this is the correct choice. This choice establishes that, simply an inference that might be drawn from the
in fact, the low-crime districts do NOT suffer from premises.
significant crime increases after the removal of some (E) The idea that human activity is the sole cause of global
officers-- an essential assumption upon which the warming is neither suggested nor assumed by the
commissioner's proposal depends. argument. In addition, the wording "sole cause" is too
extreme.
38. Answe : A
The statement concludes that rising sea levels caused by 39. Answer: E
global warming will destroy major coastal population The researchers claim that Delta-32 prevents its carriers
centers and displace millions of people. Any assumption from contracting the Plague. They support this claim by
in support of this conclusion would have to corroborate noting that a strikingly large percentage of descendants of
that these events will definitively take place. Plague survivors carry the mutation. We are asked to find
(A) This choice is correct. If new technological an assumption underlying the claim.
developments in the next century allow people to divert (A) The statement is specific to the relationship between
rising seas from the world’s cities (i.e., population Delta-32 and resistance to the Plague. Other diseases are
centers), cities will not be destroyed and millions of irrelevant.
people will not be displaced. Thus, a necessary (B) Again, the statement is specific to the relationship
assumption is that these technologies will not be between Delta-32 and resistance to the Plague. Other
developed. diseases are irrelevant.
(B) A simple awareness of the steps to reduce emissions (C) Delta-32 may have existed in its current form before
in no way undermines the statement’s assumption, as this the sixteenth century and the merit of the argument would
answer choice does not describe any action being taken by not change.
individuals. (D) The statement does not claim that Delta-32 prevents
all bacteria-caused disease.

Page 1229 of 1334


Subscribe The Xpress Video Course & Mock Test Package for Bank & Insurance Exams
If there are any suggestions/ errors in our PDFs Feel Free to contact us via this email: admin@exampundit.in
Ultra Practice Bundle PDF
SBI Clerk/ RBI Assist. Mains – Reasoning
(E) This choice is correct. The researchers claim that predict how many rings would be lost when the
Delta-32 prevented its carriers from contracting the temperature does exceed 95 degrees.
Plague on the basis of its presence in descendants of
Plague survivors. In order to claim that the mutation 41. Answer : B
prevented the Plague, we must assume that the Plague did The conclusion is that a company should wait until
not cause the mutation Delta-32. purchases of an old device begin to decline before
announcing a new device. The basis for this claim is that
40. Answer: E consumers stop buying the old device. We are asked to
The author concludes that one will only be able to strengthen the argument.
determine the age of a Brazilian ash by counting its rings (a) The typical drop in the price of new technology does
if the temperature in the tree's environment never exceeds not influence whether a company should wait until sales
95 degrees Fahrenheit. The author bases this conclusion of an old technology begin to decline before introducing a
on the fact that the tree loses rings when the temperature new one.
exceeds that level. However, if the number of rings lost by (b) This choice states that media outlets such as television
a Brazilian ash at high temperatures can be predicted, it and magazines often report on the planned introduction of
may be possible to determine the age of a tree even if the new devices while sales of old devices are still strong. The
temperature exceeds 95 degrees. argument requires that consumers "hear about the new
(A) The statement says nothing about precipitation. This device"; stories in the media provide a means for
answer choice is out of scope since it would require a consumers to do so. Hence B is correct.
number of other assumptions to make it relevant to the (c) If many consumers are unable to determine the
statement's conclusion. superiority of new technology, then they might be less
(B) Whether other trees share this feature is irrelevant; the swayed to wait for a new, recently announced device than
statement focuses only on the Brazilian ash. otherwise. As a result, this claim may be seen to weaken
(C) The number of days of excessive heat needed to cause the assertion.
the tree to lose rings is irrelevant. (d) The number of technology purchases per year does not
(D) The thickness of the rings is irrelevant. directly relate to this argument. The argument is about
(E) This choice is correct. The conclusion is that the rings waiting until the consumer demand declines before
will be a reliable measure only if the temperature never announcing a new technology.
exceeds 95 degrees. This is true only if there is no way to

Page 1230 of 1334


Subscribe The Xpress Video Course & Mock Test Package for Bank & Insurance Exams
If there are any suggestions/ errors in our PDFs Feel Free to contact us via this email: admin@exampundit.in
Ultra Practice Bundle PDF
SBI Clerk/ RBI Assist. Mains – Reasoning
(e) The passage makes no mention of whether the If this point was not true and the White rhinos have shifted
technologies belong to the same company or different to another location, then the conclusion will not hold
companies good. However, if this point was actually true, it would
clear any confusion and provide evidence for the
42. Answer: E foresters’ view.
The best approach for these kinds of questions is to treat
each question independently as a true/false question. In 46. Answer : B
this case, each of the three selections is false. The first You may immediately go ahead and select (d) but Voltaire
selection could be true based on the information that some is not the main topic of the passage. Please consider all
spouses have enrolled in the programme, but it is possible options before you answer. Option (a) doesn’t follow from
that spouses might be enrolled without being employees. the passage. The passage presents the idea that a person
Similarly, for b and c and hence choice (e) is the right who believes in ridiculous things or superstitious or
answer. illogical things can easily commit evil deeds against
others. So the obvious option to infer from that is (b) The
43. Answer: C person’s beliefs are dangerous to others.
This is an ‘if ……. then’ kind of argument. (a) is incorrect
because the first sentence of the argument required to be a 47. Answer : D
1st slip position to occupy the position of a wicketkeeper. The author is trying to point out the redundancy of the so-
(c) is the exact opposite of (a) and hence is the right called new invention/development. Hulls are made of
answer. wood. The new invention involves a method, which
converts wood into some kind of plastic, which is then
44. Answer : D used to make hulls of ships. The new invention is basically
We have to find a conclusion for the statement. The best lengthening and complicating the process. Thus, D is
way to do that is to ask “why” with each option. When we correct.
do this, D forms the best question that can be answered by
the given statement and that’s why it is the correct answer. 48. Answer: C
This passage relates information from two studies
45. Answer : C concerning high school seniors: the first discusses the
financial responsibilities of high school seniors, while the

Page 1231 of 1334


Subscribe The Xpress Video Course & Mock Test Package for Bank & Insurance Exams
If there are any suggestions/ errors in our PDFs Feel Free to contact us via this email: admin@exampundit.in
Ultra Practice Bundle PDF
SBI Clerk/ RBI Assist. Mains – Reasoning
second explains the coursework in finance taken by financial responsibilities. This does not mean that any
typical high school seniors. high school senior who contributes to these categories has
(a) Although it might be true that schools would be wise significant financial responsibilities.
to educate students in finance, this is an opinion; it doesn't (e) The passage states that one-third of high school seniors
necessarily need to follow from the given evidence. say that they have “significant financial responsibilities.”
(b) The fact that one-third of high school seniors claim This in no way indicates that the other two thirds have
“significant financial responsibilities” to their families “no” responsibilities. Because no information is given
does not necessarily mean that these same students work about the other two-thirds of the students, a reasoned
“part-time jobs after school.” There are many possible conclusion cannot be drawn about them.
ways that these students might earn money for their 49. Answer: A
families. If they do work, they might work on weekends The conclusion in the statement is that altering the tests so
or over the summer, for example. that students will receive higher score which will have a
(c) This choice is correct. The first study states that one- positive effect on students’ self-esteem levels. (a), if true
third of all high school seniors have significant financial contradicts this by showing that self-esteem is already set
responsibilities to their families. The second study states and cannot be changed by something like test scores.
that 80% of seniors have opened a bank account, and of
this 80%, one third has bounced a check. The number of 50. Answer: C
seniors that has bounced a check (one-third of 80%) is The conclusion is that potential donor will be flattered by
fewer than the number of seniors with significant financial requests for large donations and frequently offended by
responsibilities to their families (one-third of 100%). requests for small amounts. Therefore, it is worth the
(d) The passage states that certain high school seniors who gamble to start high. At worse, the potential donor may
contribute to the food, shelter, or clothing for themselves decrease his request. Alternatives (a), (b) and (d) are not
or their families rate themselves as having significant conclusion made in the statement.

Page 1232 of 1334


Subscribe The Xpress Video Course & Mock Test Package for Bank & Insurance Exams
If there are any suggestions/ errors in our PDFs Feel Free to contact us via this email: admin@exampundit.in
Ultra Practice Bundle PDF
SBI Clerk/ RBI Assist. Mains – Reasoning
Data Sufficiency
Directions (1-2): The question below can be answered b) J sits third to the right of B. L and W doesn’t sit
using one of the options given below. Study the together.
following information carefully and find out which of C) Two persons sit between N and G, who doesn’t sit
the following option is sufficient to answer. adjacent to W. J sits at one of the ends but not adjacent to
1) Eight persons namely – A, B, C, D, E, F, G, and H G.
are sitting around a circular table facing the center. G d) J sits second to the right of N. L doesn’t sit adjacent to
and E sit together. D sits third to the left of E. Two S.
persons sit between G and A, who sits adjacent to D. e) More than one statement is sufficient.
Who sits immediate right of H? (Directions 3–5): Below question consists of one
a) Two persons sit between B and C. B sits adjacent to H question followed by two statements you have to decide
but not adjacent to D. which of the following statement is sufficient to
b) Two persons sit between F and H. F neither sits answer.
adjacent to G nor sits adjacent to E. a) If only statement I alone is sufficient to answer the
c) B sits facing F, who sits immediate left of C. F and G question.
don’t sit together. b) If only statement II alone is sufficient to answer the
d) C sits immediate right of F, who doesn’t sits adjacent question.
to G. B sits adjacent to H, who doesn’t sits adjacent to A. c) If either statement I or statement II alone sufficient
e) Either C or D. to answer the question.
2) Nine persons namely –B, G, H, J, L, N, R, S and W d) If neither the statement I nor statement II together
are sitting in a linear row facing north. B sits fourth are sufficient to answer the question.
from the right end. W neither sits adjacent to H nor e) If both the statement I and statement II together are
sits adjacent to B. Three persons sit between B and H, sufficient to answer the question
who sits immediate right of R. Two persons sit between
S and W. 3) Six persons P, Q, R, S, T, and U are sitting around a
Who sits second to the right of L? circular table facing the center. Who sits immediate
a) G, who doesn’t sits adjacent to W, sits third places from right of Q?
N. One person sits between N and J.

Page 1233 of 1334


Subscribe The Xpress Video Course & Mock Test Package for Bank & Insurance Exams
If there are any suggestions/ errors in our PDFs Feel Free to contact us via this email: admin@exampundit.in
Ultra Practice Bundle PDF
SBI Clerk/ RBI Assist. Mains – Reasoning
I. R sits third to the left of the one who sits second to in the option is sufficient to answer the below
the left of P. Neither P nor R sits adjacent to either S questions.
or Q. 6) Seven persons namely – P, Q, R, S, T, U, and V are
II. R sits facing S, who sits second to the left of P. sitting in a circular table facing the center, then who
Neither P nor S sits adjacent to T. among the following person sits immediate left of T?
4) Five persons namely – G, H, K, L, and M were born a) S sits third to the left of U. V sits second to the right of
in different years viz.- 1992, 1989, 1985, 1981, and 1977 Q, who doesn’t sit adjacent to U. T neither sits adjacent to
but not necessarily in the same order. The age of each U nor sits adjacent to V.
person is calculated from the base year 2020. Who b) One person sits between S and R, who sits third to the
among the following person was born just before H? left of T. S neither sits adjacent to T nor sits adjacent to U.
I. Two persons were born between L and K, whose age c) Two persons are sitting between P and V, who sits third
is a multiple of 3.The age of H is not a prime number to the right of T.
and was born after G. d) The Person immediately left of T sits fourth to the right
II. H was born two persons after G and his age is a of S, who sits second to the left of R. Q and R are not
multiple of 7. Two persons were born between L and sitting together.
K, who was born before L. e) More than one statement is sufficient.
5) Six persons namely – B, D, G, H, M, and U are 7) Seven persons namely – Ankur, Shreya, Tapsi,
attending a seminar during a week starting from Kokila, Murgan, Sameer, and Navya were born in the
Monday to Saturday but not necessarily in the same same week starting from Tuesday to Monday. Ankur
order. Who attends the seminar just after D? was born on Sunday and was born three days after
I. U attends a seminar on Friday three days after B. Sameer. Who among the following person was born
Two persons are attending a seminar between G and just after Murgan?
H, both are attending a seminar after B. I. Shreya was elder than Murgan and Ankur but
II. H attends the seminar on Wednesday and attends younger than Navya. Murgan is not the youngest.
the seminar two days after M. Neither M nor H attends II. Murgan was elder than Kokila and Tapsi but
the seminar just before G. More than two persons are younger than Shreya, who is not the eldest.
attending a seminar between B and G. III. Kokila is elder than Ankur and Tapsi but younger
Directions (6-8): Study the following statement than Shreya and Murgan. Navya is the eldest.
carefully and decide which of the following statement a) Only statement I alone is sufficient

Page 1234 of 1334


Subscribe The Xpress Video Course & Mock Test Package for Bank & Insurance Exams
If there are any suggestions/ errors in our PDFs Feel Free to contact us via this email: admin@exampundit.in
Ultra Practice Bundle PDF
SBI Clerk/ RBI Assist. Mains – Reasoning
b) Either statement II or statement III alone is sufficient c) If data in either statement I or II alone sufficient to
c) Either statement I or statement II alone is sufficient answer the question.
d) None of these d) If data in both statements I and II together are not
e) Onlystatement III alone is sufficient sufficient to answer the question.
8) Statements: e) If data in both statements I and II together are
I. Some Cycles are Train. sufficient to answer the question.
II. No Bus is Auto. 9) There are five friends P, Q, R, S, and T. Each of
III. Some Autos are Train. them was born in the same month of the same year on
Conclusions: different dates viz.- 10, 17, 19, 26, and 31but not
I. Some Cycles are not Cart. necessarily in the same order. T was born on which of
II. Some Autos are not Cart. the following date?
Which of the following statements need to be added in I. S and P were not born on an even date. S was born
such a way that both the given conclusions are before P. Only two persons were born between R and Q.
definitely true? II. None of them was born after Q. One person was born
a) Some Cycles are Cart. between T and R. Either T or R was born on an even date.
b) No Cart is Auto. 10) There are seven persons namely – C, E, G, H, I, O,
c) No Train is Cart. and N in a family. Is I is the grandson of H?
d) All Buss are Cart. I. H is the father of G. E is the brother-in-law of G. E has
e) More than one statement is true. two children.
Directions (9 -10): Each of the questions below consists II. N is the son-in-law of E and not married to I. O is the
of a question and two statements numbered I and II. daughter of C. G is the maternal uncle of O. I is the sibling
You have to decide whether the data provided in the of O.
statements are sufficient to answer the questions. Direction (11-12): Below questions consists of two
a) If data in statement I alone sufficient to answer, statements I and II you have to decide which of the
while data in statement II is not sufficient to answer following statement is sufficient to answer the
the question. following question.
b) If data in statement II alone sufficient to answer, 11) Seven persons are living in a building topmost floor
while data in statement I is not sufficient to answer the is numbered as 7 and the bottom-most floor is
question. numbered as 1. Who lives on the fourth floor?

Page 1235 of 1334


Subscribe The Xpress Video Course & Mock Test Package for Bank & Insurance Exams
If there are any suggestions/ errors in our PDFs Feel Free to contact us via this email: admin@exampundit.in
Ultra Practice Bundle PDF
SBI Clerk/ RBI Assist. Mains – Reasoning
I) there are two persons live between B and A. F lives a) If data in statement I alone is sufficient to answer.
on the top floor. There are four floors between B and b) If data in statement II alone is sufficient to answer.
G, who lives on an even floor. c) If data in either statement I or statement II alone is
II) There are three floors above and below A. sufficient to answer.
a) Only I d) If data in both statements I and statement II
b) Only II together are not sufficient to answer.
c) Neither I nor II e) If data in both statements I and statement II
d) Both I and II together are sufficient to answer.
e) Either I or II 13) Six persons namely –A, B, C, D, E, and F are sitting
12) There is an uncertain number of the person sitting around a triangular table in such a way that one
in a row faces north. How many persons are there in a person sits on each side facing center and one person
row? sits on each corner facing away from the center. Who
I) Three persons sit between P and Q, who sits left of P sits immediate right of B?
and one of them sits at extreme ends. Five persons sit I. Two persons are sitting between B and A, who sits on
between P and R, who sits fourth from one of the the sides. Neither C nor F is sitting adjacent to B. One
extreme ends. Only three persons sit to the right of R. person sits between E and D.D and F are not immediate
II) Six persons sit between P and Q, who sit at the neighbors.
extreme left of the row. O sits fourth right to P. S sits II. C sits second to the left of B, who sits on the corner.
fourth left of P. Neither D nor F is sitting adjacent to C. F and A are sitting
a) Only I together but A and C are sitting together.
b) Only II 14) Which of the following statement is sufficient such
c) Neither I nor II that all three conclusions are true?
d) Both I and II Statement: Only Tea is Coffee. Only a Few tea are Juice.
e) Either I or II All Ice-crème is Juice. Few Tea are Soda.
Directions (13-17): Each of the questions below Conclusion: Some Tea is not Ice-cream. All Soda can
consists of a question followed by two statements never be Tea. Some Juice is not Cold Drink.
numbered I and II. You have to decide whether the I. Only Soda is Cold Drink.
data provided in the statements are sufficient to II. No Ice-cream is Cold Drink.
answer the questions.

Page 1236 of 1334


Subscribe The Xpress Video Course & Mock Test Package for Bank & Insurance Exams
If there are any suggestions/ errors in our PDFs Feel Free to contact us via this email: admin@exampundit.in
Ultra Practice Bundle PDF
SBI Clerk/ RBI Assist. Mains – Reasoning
15) Eight persons namely – A, B, C, D, E, F, G, and H between V and T, who neither submits a thesis on
are equally sitting around two concentric circular Wednesday nor Friday.
tables. The person in the inner circular table sits II. Three persons submit T and S, who submits the thesis
exactly facing the persons in the outer circular table. on Friday. Two persons submit a thesis between U and V,
The person facing D sit immediate right of G, who sits who neither submits thesis Tuesday nor Sunday.
facing away from the center. One person sits between Directions (18-20): Study the following statement
G and H, who sits facing E. carefully and decide which of the following statement
Who among the following persons sits immediate left in the option is sufficient to answer the below
of H? questions.
I. One person sits between B and E. C and F are facing 18) Eight persons namely – A, B, C, D, E, F, G, and H
each other. are sitting around a circular table facing the center. C
II. One person sits between A and F, who doesn’t sit sits three places away from B. Person who sits second
facing D. right of B sits facing H, who sits third to the right of F.
16) Six persons namely – B, C, D, E, F, and G were Who among the following person sits second to the
born in different years viz.- 1987, 1989, 1991, 1994, right of E?
1996, and 1999. Who among the following person was a) G and F are not sitting together. D and G are sitting
born just after B? together.
I. C was born three years after E, who was born in an even- b) One person sits between D and A, who is not sitting
numbered year. Two persons were born between E and B. adjacent to B. Persons facing A sits adjacent to E.
One person was born between F and D. c) Two persons are sitting between G and A, who doesn’t
II. Three persons were born between C and B. Two sit adjacent to B.
persons were born between G and D, who was born in d) The Person facing D sits second to the right of A, who
1987. doesn’t sit adjacent to G. B sits immediate left of G.
17) Seven Ph.D. persons namely – P, Q, R, S, T, U, and e) More than one statement is sufficient.
V submits a thesis on seven days of a week starting 19) Nine persons namely – A, B, C, D, E, F, G, H, and
from Monday to Sunday. Who among the following I are sitting in a row facing north. A sits third from the
person submits the thesis just before V? left end and sits fourth to the left of B. Two persons are
I. Q submits a thesis on Saturday and three persons submit sitting between F and H, who either sits immediate left
thesis between Q and P. Two persons are submitted thesis

Page 1237 of 1334


Subscribe The Xpress Video Course & Mock Test Package for Bank & Insurance Exams
If there are any suggestions/ errors in our PDFs Feel Free to contact us via this email: admin@exampundit.in
Ultra Practice Bundle PDF
SBI Clerk/ RBI Assist. Mains – Reasoning
of B or immediate right of A. H sits fourth to the left of provided in the statements are sufficient to answer the
E. question. Read both the statements and give answer.
Who among the following person is sitting second to 21) Six persons A, B, C, D, E and F are sitting around
the right of C? a triangular table facing centre such that three of them
a) Three persons are sitting between D and I, either of are sitting at the corners and the remaining are sitting
them sits at the end. in the middle of each side but not necessarily in the
b) Persons second to the right of C sit five places away same order. Then who sits immediate right of A?
from D. I) A sits second to the right of D and sits in the middle
c) Three persons are sitting between person adjacent to H of the table. Two persons sit between D and E.
and D, who sits at any place right of C. II) Two persons sit between E and F and sits in the
d) The number of persons between C and H is the same as middle of the table. B sits second to the right of E and
the numbers of persons between D and I. second to the left of A.
e) None of the statements is sufficient. ,m a) Only statement I alone is sufficient
20) Five boxes – A, B, C, D, and E are kept one above b) Only statement II alone is sufficient
the other. Color of each box is different viz.- Pink, c) Either statement I or statement IIis sufficient
Yellow, Blue, Red, and Black. The red box is kept three d) Both statement I and statement II is sufficient
places above box D, which is kept just above the Pink e) None of these
box. One box is kept between box E and Blue box. The 22) What is the code of ‘Corona’ in a certain code?
color of box E is neither red nor Pink. I) ‘Corona is pandemic’ is written as ‘su da tc’
Which of the box is kept just below box B? II) ‘Medicine is produced’ is written as ‘tcphra’
a) The black box is kept two places below box C. III) ‘Corona is dangerous’ is written as ‘tcsu, gi’
b) The yellow box is kept just below box C. IV) ‘No vaccination till’ is written as’ qi pnsi’
c) Box A has kept three places below the Yellow box. a) Only statement I alone is sufficient
d) The black box is kept exactly between boxes A and C. b) Only statement IV alone is sufficient
Box E is not in black color. c) Either statement I or statement II is sufficient
e) More than one statement is sufficient. d) Statements I, II and III is sufficient
Direction: (21-24): Each of the following questions e) None of these
below consists of a question and some statements are 23) Ram is 16th from the right end in the class and
given below it. You have to decide whether the data Shyam is 17th from the left end in the class. Find the

Page 1238 of 1334


Subscribe The Xpress Video Course & Mock Test Package for Bank & Insurance Exams
If there are any suggestions/ errors in our PDFs Feel Free to contact us via this email: admin@exampundit.in
Ultra Practice Bundle PDF
SBI Clerk/ RBI Assist. Mains – Reasoning
total students in a class, if ram sits to the right of data provided in the statements are sufficient to
shyam? answer the question. Read all the three statements and
I) If they interchange their position then the position give answer.
of Ram is 20th from the right end.
II) If 5 students sit between Ram and Shyam. 25. A, B, C, D and E are five persons who live on a five
a) Only statement I alone is sufficient floor building where ground floor is numbered as 1st
b) Only statement II alone is sufficient floor, the floor above it is numbered as 2nd so on till the
c) Either statement I or statement II is sufficient topmost floor is numbered as 5th but not necessarily in
d) Neither statement I nor statement II is sufficient the same order then find who among the following lives
e) None of these on the 3rd floor?
24) Six boxes are arranged in two stacks from bottom I. E lives on an odd numbered floor but not on the
to top numbered from 1-3. Each stack contains three topmost floor. D lives immediately above A’s floor.
boxes. Stack X is in the West of Stack Y. Which box is II. Two persons lives between D and E. C lives on an
placed immediate above B in the same stack? even numbered floor.
I) Box B is kept west of box F and placed immediate III. B lives above C’s floor but not immediately above.
above box A but not in the same stack. D lives above E’s floor.
II) One box is placed between box B and Box E both (a)Both II and III
are in the same stack. C is placed west of D. Box E is (b)Both I and III
placed above box B. No box is placed to the west of box (c)Both I and II
E. (d) Any of the two statements
a) Only statement I alone is sufficient (e) All I, II, III
b) Only statement II alone is sufficient
c) Either statement I or statement II is sufficient 26. M is in which direction with respect to N?
d) Neither statement I nor statement II is sufficient I. W is in the east of Z, who is in the south of Q.
e) Both statement I and statement II are sufficient II. M is in the north of W and east of Q.
III. N is in the south-west of Z.
Direction (25-26): Each of the questions below consists (a) Both I and II
of a question and three statements numbered as I, II (b) Only III
and III given below it. You have to decide whether the (c) Any two of the three

Page 1239 of 1334


Subscribe The Xpress Video Course & Mock Test Package for Bank & Insurance Exams
If there are any suggestions/ errors in our PDFs Feel Free to contact us via this email: admin@exampundit.in
Ultra Practice Bundle PDF
SBI Clerk/ RBI Assist. Mains – Reasoning
(d) All I, II and III R. Two persons go between M and L who goes to the
(e) None of these market on Monday. N doesn’t go on Sunday.
Direction (27-28): Each of the questions below consists II) M goes on Friday. Two persons go between L and
of a question and two statements numbered as I and II M. Four persons go between L and Q.
given below it. You have to decide whether the data a) Only statement I
provided in the statements are sufficient to answer the b) Only statement II
question. Read all the two statements and give answer. c) Either statement I or statement II
27) Eight persons P, Q, R, S, T, U, V and W sitting d) Neither statement I nor statement II
around a rectangular table but not in the same order. e) Both statement I and statement II
Four of them sit at the corner while remaining sit at In each of the following questions two statements are
the middle of each side of the table. All of them are given. You have to decide which of the following
facing towards the centre. Who sits third to the right statement logically sufficient to get the answer of the
of the one who faces U? following questions.
I) Three persons sit between R and T. U sits third to a) If statement I is alone sufficient.
the left of T. b) If statement II is alone sufficient.
II) Two persons sit between S and V, who sits at the c) If either statement I or II is alone sufficient.
middle of the table. Q sits second to the left of V. U sits d) If neither statement I nor II is alone sufficient.
third to the left of S. e) If both statements I and II together are sufficient.
a) Only statement I 29) Six boxes viz. - P, Q, R, S, T and U are kept one
b) Only statement II above other in the form of stack but not necessarily in
c) Either statement I or statement II the same order. Which of the following box is kept just
d) Neither statement I nor statement II above the box T?
e) Both statement I and statement II I) One box is kept between the box U and the box T. Box
28) Seven persons L, M, N, O, P, Q and R goes to the P is kept three places above box S. The box R is kept just
market on different days of week starting from above the box U. Box S and box T are not kept adjacent
Monday but not in the same order. Who goes to the to each other.
market at last? II) Two boxes are kept between S and P, which is kept
I) O goes to the market on Wednesday. Two persons just below the box U. The box Q is not kept at bottom.
go between O and P. Three persons go between P and

Page 1240 of 1334


Subscribe The Xpress Video Course & Mock Test Package for Bank & Insurance Exams
If there are any suggestions/ errors in our PDFs Feel Free to contact us via this email: admin@exampundit.in
Ultra Practice Bundle PDF
SBI Clerk/ RBI Assist. Mains – Reasoning
Box U is kept three places above the box Q. The box R is II) One person sits between A and C, who doesn’t sits
neither kept adjacent to the box P nor to box S. adjacent to B. C sits adjacent to E, who is facing D. B sits
30) Five persons namely – P, Q, R, S and T are sitting at one of the corners and third to right of A. Two persons
in a row facing north. Each person works in different sit between D and H when counted from the left of D.
bank - RBI, PNB, BOB, SBI and UBI. All the 32) Age of seven persons namely – P, Q, R, S, T, U and
information is not necessarily in the same order. Who V are different. Age of how many persons are more
sits immediate right of the one who works in RBI? than Q?
I) The one who works in PNB sits second to right of S. Q I) U is eldest and age of only one person is less than T.
sits third to left of R, who works in BOB. One person sits Age of only two person are more than P. S is younger to
between R and the one who works in PNB. S neither P and R but not as much as V and Q.
works in RBI nor works in UBI. The one who works in II) Age of only one person is more than R. Age of only
UBI sits immediate right of P. three person is less than S. Age of P is more than S and T
II) T neither works in PNB nor works in BOB. One person but not as much as U and R.
sits between the one who works in BOB and the one who Each of the questions below consists of a question and
works in PNB. The one who works in SBI sits third to left two statements numbered I and II given below it. You
of T at extreme end. The one who works in PNB sits have to decide whether the data provided in the
immediate right of P. Two persons sit between Q and R, statements are sufficient to answer the question:
who doesn’t sits adjacent to the one who works in SB a) If the data in Statement I alone is sufficient to answer
31) Eight persons namely – A, B, C, D, E, F, G and H the question, while the data in Statement II alone is not
are sitting in a square table in such a way that one sufficient to answer the question.
person sits at each corner and one person sits at middle b) If the data in Statement II alone is sufficient to answer
of each side. Person sitting at the middle of the sides the question, while the data in Statement I alone is not
are facing away from centre and persons at corner sufficient to answer the question.
facing centre. All the information is not necessarily in c) If the data either in Statement I alone or in Statement II
the same order. Who sits second to the right of G? alone is sufficient to answer the question.
I) D is facing E, who sits immediate left of C. H sits third d) If the data in both the Statements I and II together are
to right of F, who sits immediate left of A. D neither sits not sufficient to answer the question.
adjacent to H nor sits adjacent to F. One person sits e) If the data in both the Statements I and II together are
between C and A, who doesn’t sits adjacent to H. necessary to answer the question.

Page 1241 of 1334


Subscribe The Xpress Video Course & Mock Test Package for Bank & Insurance Exams
If there are any suggestions/ errors in our PDFs Feel Free to contact us via this email: admin@exampundit.in
Ultra Practice Bundle PDF
SBI Clerk/ RBI Assist. Mains – Reasoning
33) Salary of seven persons namely – A, B, C, D, E, F II). Three persons sit between Q and R, who doesn’t sits
and G are different. Salary of who among the following at the end. P sits second to left of R, who doesn’t sits
is second lowest? adjacent to S. One person sits between T and U.
I). Salary of only G is more than B. Salary of C is not 36) Six persons namely – P, Q, R, S, T and U sit in a
lowest. Salary of A is more than E and D but not as much circular table facing centre. Who sits immediate right
as B. Salary of only two persons is less than D. of R?
II). Salary of only one person is more than B. Salary of F I). T is facing R. T sits second to left of Q, who doesn’t
is lowest. Salary of E is more than D and C but not as sits adjacent to U. S neither sits adjacent to Q nor T.
much as G and B. Salary of A is not the highest. II). U is facing Q, who sits adjacent to P. R neither sits
34) Seven persons namely – P, Q, R, S, T, U and V lives adjacent to P nor sits adjacent to U.
on seven floored building marked 1 to 7 from bottom 37) In a family of six members having one married
to top but not necessarily in the same order. What is couple, who is R’s son?
the position of S from bottom? I). R is mother of Q, who is only sister of T’s father. L is
I). P lives just below Q, who doesn’t lives on topmost only brother of T and is son of J who is a female. T is a
floor. T lives on even number floor and lives three floors granddaughter of R.
away from P. R neither lives on adjacent floor of P nor II). R is brother of Q’s mother. H is mother-in-law of L’s
lives adjacent floor of Q. U lives four floors above R. brother. J is married to Q and is only son of T.
II). R lives three floors below Q, who lives on even Each of the questions below consists of a question and
numbered floor. V neither lives on adjacent floor of Q nor two statements numbered I and II given below it. You
R. T lives on adjacent floor of V but not on odd number have to decide whether the data provided in the
floor. U lives on any floor above P, who lives on odd statements are sufficient to answer the question:
number floor. a) If the data in Statement I alone is sufficient to answer
35) Six persons namely – P, Q, R, S, T and U sits in a the question, while the data in Statement II alone is not
row facing north but not necessarily in the same order. sufficient to answer the question.
Who sits immediate right of U? b) If the data in Statement II alone is sufficient to answer
I). P sits third from left end. Q neither sits adjacent to P the question, while the data in Statement I alone is not
nor sits adjacent to R. One person sits between P and R. S sufficient to answer the question.
sits immediate right of Q. Neither U nor S sits at end of c) If the data either in Statement I alone or in Statement II
the row. alone is sufficient to answer the question.

Page 1242 of 1334


Subscribe The Xpress Video Course & Mock Test Package for Bank & Insurance Exams
If there are any suggestions/ errors in our PDFs Feel Free to contact us via this email: admin@exampundit.in
Ultra Practice Bundle PDF
SBI Clerk/ RBI Assist. Mains – Reasoning
d)If the data in both the Statements I and II together are lives on an odd number floor just below E. One person
not sufficient to answer the question. lives between D and C, who doesn’t lives on adjacent floor
e) If the data in both the Statements I and II together are of E.
necessary to answer the question. II. The one who likes 2 lives just above E. At least two
38) Height of six persons namely – P, Q, R, S, T and U person lives below E. D lives two floors above C and likes
are different. Height of who among the following is 8. C does not like 6.
second tallest? 41) Six boxes namely - P, Q, R, S, T and U are kept one
I. Height of only two persons is more than Q. Height of P above another not necessary in same order. Which box
is more than R and T but not as much as U and S. is kept just below box Q?
II. Height of only two persons is less than P. Height of S I. Box U is kept third from top and is kept at a gap of two
is more than R and U but not as much as Q and T. boxes from P. Box R is neither kept adjacent to box P nor
39) Six persons namely – P, Q, R, S, T and U sits box U. Box T is kept at any place below box U but not
around a rectangular table facing center but not immediately below.
necessarily in the same order. In such a way that two II. Only one box is kept below box T, which is kept at a
persons sit at longer side and one person sits at each gap of two from box S. Box R is kept adjacent to box S
smaller side. Who is facing P? and is at a gap of one box from box U. R is placed above
I. T sits immediate left of R. U sits second to left of S, who U.
sits at smaller side. R neither sits adjacent to U nor sits 42) How “Life” is coded in the given code language?
adjacent to S. I. “Book More Life System” is coded as “go la zm
II. P sits third to right of R. One person sits between S and hq” and “Half Book Keep System” is coded as “hm dj
U. R sits immediate left of Q, who sits at one of the smaller la zm”.
side. II. “Long Day More Life” is coded as “pm xm hq
40) Five persons namely – A, B, C, D and E live on five go” and “Heavy Day Book System” is coded as “xm pz
floor building marked 1 to 5 from bottom to top but zm la”.
not necessarily in the same order. Each person likes an Each of the questions below consists of a question and
even number from 2 to 10. One number is likes by only three statements numbered I, II and III given below it.
one person. Who lives just above the one who likes 6? You have to decide whether the data provided in the
I. B, who likes 2, lives on an odd number floor. The one statements are sufficient to answer the question:
who likes 10 lives three floors below B. D, who likes 8,

Page 1243 of 1334


Subscribe The Xpress Video Course & Mock Test Package for Bank & Insurance Exams
If there are any suggestions/ errors in our PDFs Feel Free to contact us via this email: admin@exampundit.in
Ultra Practice Bundle PDF
SBI Clerk/ RBI Assist. Mains – Reasoning
43) Six persons A, B, C, D, E and F live on six floored II). Jay sits at any position right of Deep but not at the end
building marked 1 to 6 from bottom to top but not of the row.
necessarily in the same order. Each person likes III). Neither Hari nor Ravi sits adjacent to Deep.
different number viz.- 7, 11, 12, 16, 19 and 21. D lives a) Either II or III
on floor 3 and lives three floors below C. F, who likes b) Both I and II
16, lives just below C. Two persons live between the c) Either III or both I and II
one who likes 21 and B, who doesn’t like odd number. d) Only III
Three persons live between the one who likes 16 and e) Only II
21. Who lives just above the one who likes 7? 45) Eight boxes – A, B, C, D, E, F, G and H are kept
I). D neither likes 7 nor likes 11.Two persons live between one above other. Box B is kept four places below
the one who likes 11 and the one who likes 19. C which is not kept adjacent to H. One box is kept
II). A, who doesn’t likes 21, lives on any floor below the between C and E. Box E is kept three places below box
one who likes 11.At least two persons live between E and G. At least three boxes are kept between E and D. G is
the one likes 11. not kept sixth from the bottom. Which of the box is
III). E, who doesn’t likes prime number, lives two floors kept just below box F?
below the one who likes 19. The one who likes 11 lives on I). Box F is kept at any position below box H.
floor 6. II). Three boxes are kept between box F and A.
a) Only I III). Box A is not kept adjacent to B but kept above box
b) Either II or III H.
c) Only II a) Either II or III
d) Either I or II or III b) Only III
e) None of these c) Either III or II
44) Sam, Teza, Ravi, Shreya, Deep, Jay and Hari sit in d) Only II
a row facing south. Sam sits third from right end. One e) None of these
person sits between Sam and Hari, who neither sits 46) Eight persons – P, Q, R, S, T, U, V and W are sitting
adjacent to Ravi nor sits at end of the row. Three in a circular table facing centre but not necessarily in
persons sit between Ravi and Teza. Who sits the same order. R is facing Q, who doesn’t sits adjacent
immediate left of Shreya? to T. One person sits between Q and W. S and V sits
I). Shreya and Sam don’t sit together.

Page 1244 of 1334


Subscribe The Xpress Video Course & Mock Test Package for Bank & Insurance Exams
If there are any suggestions/ errors in our PDFs Feel Free to contact us via this email: admin@exampundit.in
Ultra Practice Bundle PDF
SBI Clerk/ RBI Assist. Mains – Reasoning
together. Neither S nor V sits adjacent to Q. Who is a) If the data in Statement I alone is sufficient to answer
facing T? (2 Marks) the question, while the data in Statement II alone is not
I).T sits second to left of V and sits facing P. sufficient to answer the question.
II).S and U sit together. b) If the data in Statement II alone is sufficient to answer
III). T and W sit together. the question, while the data in Statement I alone is not
a) Either II or III sufficient to answer the question.
b) Only III c) If the data either in Statement I alone or in Statement II
c) Either I or III alone is sufficient to answer the question.
d) Either I or II d) If the data in both the Statements I and II together are
e) one of these not sufficient to answer the question.
47) Ages of A, B, C, D, E, F and G are different. How e) If the data in both the Statements I and II together are
many persons are younger to A? necessary to answer the question.
I). D is younger to F and B but not C and G. F is eldest 48) P, Q, R, S and T lives on five floor building marked
among all. 1 to 5 from bottom to top but not necessarily in the
II). A is younger to D but not E and G. Only one person same order. Each person likes different colours viz.-
is younger to E. Red, Blue, Green, Black and Yellow. Who lives just
III). Only two persons are older to D. A is younger to F below the one who likes Yellow? (2Marks)
and B but not E. C is elder to E and G but not B and D. B I). S, who likes Black, lives on an even number floor. The
is not the eldest but G is the youngest. F and B are elder one who likes Black lives two floors below P. The one
to D. who likes Red lives three floors above Q. Neither the one
a) Only III who likes Green nor Yellow lives on lowermost floor.
b) Both I and II II). P likes Red and lives on floor 4. One person lives
c) Either I or III between the one who likes Green and R. The one who
d) Either III or Both I and II likes Red lives two floors above S, who doesn’t likes
e) None of these Green. Two persons live between the one who likes Green
Each of the questions below consists of a question and and the one who likes Black.
two statements numbered I and II given below it. You 49) Six friends namely – Mike, Sam, Tom, Jai, Rohit
have to decide whether the data provided in the and Hari are standing in a row according to their
statements are sufficient to answer the question: heights in descending order from left to right not

Page 1245 of 1334


Subscribe The Xpress Video Course & Mock Test Package for Bank & Insurance Exams
If there are any suggestions/ errors in our PDFs Feel Free to contact us via this email: admin@exampundit.in
Ultra Practice Bundle PDF
SBI Clerk/ RBI Assist. Mains – Reasoning
necessary in same order . Height of how many persons box which contains Pen and the box B04, which
is less than Jai? contains Ball. The box B06 is neither kept adjacent to
I). Height of only one person is more than Mike. Height the box which contains Ball nor the box B03. The box
of Jai is more than Rohit but not as much as Tom and Hari. B06 is kept just below the box which contains Book.
Hari is not a tallest person and Rohit is not the shortest How many boxes are kept below the box which
person. contains Bat? (2 Marks)
II). Height of only two persons are more than Hari. Height I). The box B01 is kept at a gap of two boxes from the box
of Rohit is more than Sam but not as much as Jai and B02, which contains Book. The box B05 is kept just below
Mike. Tom is tallest among all. the box which contains Mobile.
Below consists of a question and three statements II). The box which contains Mobile is kept at a gap of two
numbered I, II and III given below it. You have to boxes above the box B02. Three boxes are kept between
decide whether the data provided in the statements are the box which contains Bottle and the box B01.
sufficient to answer the question: III). The box B02 is kept just above the box which
50) Six boxes B01, B02, B03, B04, B05 & B06 are kept contains Bottle. The box B01 doesn’t contain Bat. Two
one above other in the form of stack but not necessarily boxes are kept between the box which contains Bottle and
in the same order. Each box contains different articles the box B05.
viz. Bottle, Book, Ball, Mobile, Pen and Bat but not a) Only II
necessary in the same order. Box B03 is kept at a gap b) Either II or III
of three boxes below the box which contains Ball. The c) Only III
box which contains Pen is neither kept at top nor d) Either I or II or III
adjacent to the box B06.One box is kept between the e) None

Data Sufficiency - Answers with Explanation


1. D • Two persons sit between G and A, who sits
Explanation: adjacent to D.
We have: That means, in case (1a) & case (2) A sits
• D sits third to the left of E. immediate left of D, in case (1b) A sits immediate
• G and E sit together. right of D.
That means, in case (2) G sits immediate left of E, Based on the above-given information we have:
in case (1) G sits immediate right of E.
Page 1246 of 1334
Subscribe The Xpress Video Course & Mock Test Package for Bank & Insurance Exams
If there are any suggestions/ errors in our PDFs Feel Free to contact us via this email: admin@exampundit.in
Ultra Practice Bundle PDF
SBI Clerk/ RBI Assist. Mains – Reasoning
Hence, option D is the correct choice.
2. C
Explanation:
We have:
• B sits fourth from the right end.
From C:
• Three persons sit between B and H, who sits
We have:
immediate right of R.
B sits facing F, who sits immediate left of C.
That means, R sits at left end.
F and G don’t sit together.
• W neither sits adjacent to H nor sits adjacent to B.
Based on the above-given information we have:
• Two persons sit between S and W.
That means, in case (1) W sits second to the right
of H, in case (2) W sits second from the right end.
Based on the above-given information we have:

Cleary, the exact position of H is not known.


Hence, statement C is not sufficient.
From C:
Form D: We have:
We have: Two persons sit between N and G, who doesn’t sit
C sits immediate right of F, who doesn’t sits adjacent to adjacent to W.
G. J sits at one of the ends but not adjacent to G.
B sits adjacent to H, who doesn’t sits adjacent to A. Based on the above-given information we have:
Based on the above-given information we have:

Clearly, S sits second to the right of L. Thus, only


statement C is sufficient to answer.
Hence, option C is the correct choice.
Clearly, B sits immediate right of H. Thus, statement D is
Direction (3-5):
alone sufficient.
Page 1247 of 1334
Subscribe The Xpress Video Course & Mock Test Package for Bank & Insurance Exams
If there are any suggestions/ errors in our PDFs Feel Free to contact us via this email: admin@exampundit.in
Ultra Practice Bundle PDF
SBI Clerk/ RBI Assist. Mains – Reasoning
3. E Clearly, K was born just before H.
4. A Hence, the statement I is alone sufficient.
5. E From II:
Explanation: We have:
3.
From I and II:
We have:

Clearly, in both cases the different person was born just


before H, thus statement II is not sufficient.
5.
From I:
Clearly, S sits immediate right of Q. We have:
Hence, both statements I and II together are sufficient to
answer.
4.
From I:
We have:

Clearly, the exact seminar day of D is not known.


Thus, the statement I is not sufficient alone.
From II:
Page 1248 of 1334
Subscribe The Xpress Video Course & Mock Test Package for Bank & Insurance Exams
If there are any suggestions/ errors in our PDFs Feel Free to contact us via this email: admin@exampundit.in
Ultra Practice Bundle PDF
SBI Clerk/ RBI Assist. Mains – Reasoning
We have: 6.
We have:
Two persons are sitting between P and V, who sits third to
the right of T.
Based on the above given information we have:

Clearly, the exact seminar day of D is not known.


Thus, statement II is not sufficient alone.
Clearly, P sits immediately left of T.
From I and II:
Hence, option C is the correct choice.
7.
From the above given statements we have:

Clearly, U attends just after D.


Hence, both statements I and II together are sufficient.
Direction (6-8):
6. C
7. D Clearly, we can’t find a person who born just after
8. C Murgan.
Explanation: Hence, option D is the correct choice.
Page 1249 of 1334
Subscribe The Xpress Video Course & Mock Test Package for Bank & Insurance Exams
If there are any suggestions/ errors in our PDFs Feel Free to contact us via this email: admin@exampundit.in
Ultra Practice Bundle PDF
SBI Clerk/ RBI Assist. Mains – Reasoning
8.
From statement C we have the following diagram:

Hence, option C is the correct choice.


Direction (9-10): Clearly, we don’t have the exact date of T.
9. D Hence, statement II is not alone sufficient.
10. D From I and II:
Explanation: Since, both statements are independent.
9. Hence, the statement I and II together are not sufficient.
From I: 10.
We have: From I & II:
We have:

Since, the gender of I is not known.


Clearly, we don’t have the exact date of T. Hence, the statement I and II are not sufficient.
Hence, the statement I is not alone sufficient. Direction (11-12):
From II: 11. E
We have: 12. A
Explanation:
11.
Page 1250 of 1334
Subscribe The Xpress Video Course & Mock Test Package for Bank & Insurance Exams
If there are any suggestions/ errors in our PDFs Feel Free to contact us via this email: admin@exampundit.in
Ultra Practice Bundle PDF
SBI Clerk/ RBI Assist. Mains – Reasoning
I) 13. B
F 14. A

G 15. C
16. E
17. E
A
Explanation:
13.
From I:
B

II)

Thus, we can’t say who sits an immediate right of B.


Hence, the statement I is not sufficient.
From II:

Either I or II statement is required to answer the question.


12.
I)

Only I statement is required to answer the question.


Direction (13-17):
Thus, D sits an immediate right of B.
Page 1251 of 1334
Subscribe The Xpress Video Course & Mock Test Package for Bank & Insurance Exams
If there are any suggestions/ errors in our PDFs Feel Free to contact us via this email: admin@exampundit.in
Ultra Practice Bundle PDF
SBI Clerk/ RBI Assist. Mains – Reasoning
Hence, statement II is alone sufficient.
14.
From I:

Thus, A sits immediate left of H.


Hence, the statement I is alone sufficient.
From II:
Hence, the statement I is alone sufficient. We have:
15.
We have:

Thus, A sits immediate left of H.


Hence, statement II is alone sufficient.
From I:
16.
We have:
From I:

Page 1252 of 1334


Subscribe The Xpress Video Course & Mock Test Package for Bank & Insurance Exams
If there are any suggestions/ errors in our PDFs Feel Free to contact us via this email: admin@exampundit.in
Ultra Practice Bundle PDF
SBI Clerk/ RBI Assist. Mains – Reasoning

Thus, we don’t know who was born just after B. Thus, F was born just after B.
Hence, the statement I is not sufficient. Hence, both statements I and II together are sufficient.
From II: 17.
We have: From I& II:
We have:

Thus, we don’t know who was born just after B.


Hence, statement II is not sufficient.
From I and II:
Direction (18-20):
We have:
18. D
19. E
20. D

Page 1253 of 1334


Subscribe The Xpress Video Course & Mock Test Package for Bank & Insurance Exams
If there are any suggestions/ errors in our PDFs Feel Free to contact us via this email: admin@exampundit.in
Ultra Practice Bundle PDF
SBI Clerk/ RBI Assist. Mains – Reasoning
Explanation: Thus, G sits second to the right of E.
18. We have: Hence, option D is the correct choice.
C sits three places away from B. 19.
Person who sits second right of B sits facing H, who sits A sits third from the left end and sits fourth to the left of
third to the right of F. B.
Based on the above given information we have: Two persons are sitting between F and H, who either sits
immediate left of B or immediate right of A.
H sits fourth to the left of E.
Based on the above given information we have:

None of the above statements is sufficient.


Hence, option E is the correct choice.
20.
From D:
The red box is kept three places above box D, which is
We have:
kept just above the Pink box.
The Person facing D sits second to the right of A, who
One box is kept between box E and Blue box.
doesn’t sit adjacent to G.
The color of box E is neither red nor Pink.
B sits immediate left of G.
Based on the above given information we have:
Based on the above given information we have:

From D:
Page 1254 of 1334
Subscribe The Xpress Video Course & Mock Test Package for Bank & Insurance Exams
If there are any suggestions/ errors in our PDFs Feel Free to contact us via this email: admin@exampundit.in
Ultra Practice Bundle PDF
SBI Clerk/ RBI Assist. Mains – Reasoning
The black box is kept exactly between boxes A and C.Box II)
E is not in black color.

Thus, box D is kept just below box B.


Only statement I alone is sufficient
Hence, option D is the correct choice.
22.
Direction (21-24):
Statements I, II and III is sufficient
21. A
23.
22. D
Total Students = Ram’s position from the right end after
23. C
interchanging + Shyam’s position from the left end – 1
24. B
20+17-1 = 36 students.
Explanation:
II) Total Students = Ram’s position + Shyam’s position +
21.
Students between them
16 + 17 + 5 = 38 Students
Either statement I or statement II is sufficient
24.
I)
Stack X Stack Y

2 B F

1 A

Page 1255 of 1334


Subscribe The Xpress Video Course & Mock Test Package for Bank & Insurance Exams
If there are any suggestions/ errors in our PDFs Feel Free to contact us via this email: admin@exampundit.in
Ultra Practice Bundle PDF
SBI Clerk/ RBI Assist. Mains – Reasoning

Stack X Stack Y 26.


From I, II and III it is clear that M is in north-east of N.
3 B F

2 A

II)
Stack X Stack Y

3 E
Direction (27-28):
2 C D 27. A
28. C
1 B
Explanation:
27. I)
Only II statement is required to answer the question.
Direction (25-26):
25. D
26. D
Explanataion:
25.
Sol By using Any of the two statements we can determine
that A lives on 3rd floor.
Floors Persons
5 B
4 D
3 A
2 C
1 E
Page 1256 of 1334
Subscribe The Xpress Video Course & Mock Test Package for Bank & Insurance Exams
If there are any suggestions/ errors in our PDFs Feel Free to contact us via this email: admin@exampundit.in
Ultra Practice Bundle PDF
SBI Clerk/ RBI Assist. Mains – Reasoning
Either Statement I or Statement II is required to answer
the question.

29. C
Explanation
From I:
We have:
• Box P is kept three places above the box S.
That means, in case (1) the box P is kept at top, in case (2)
II)
the box P is kept second from top, in case (3) the box P is
kept third from top.
• One box is kept between the box U and the box T.
• Box R is kept just above the box U.
• Box S and box T are not kept adjacent to each
other.
That means, case (1) & case (2) are not valid.
Based on above given information we have:

Only I statement is required.


28.
I)
Mon Tue Wed Thu Fri Sat Sun

L R O M N P Q

II)
Mon Tue Wed Thu Fri Sat Sun
L M Q

Page 1257 of 1334


Subscribe The Xpress Video Course & Mock Test Package for Bank & Insurance Exams
If there are any suggestions/ errors in our PDFs Feel Free to contact us via this email: admin@exampundit.in
Ultra Practice Bundle PDF
SBI Clerk/ RBI Assist. Mains – Reasoning
Case (1) & case (2) are not valid as box S and T are not Case (1) is not valid as box R is neither kept adjacent to
kept adjacent to each other. the box P nor S.
Thus, box P is kept just above the box T. Thus, box P is kept just above the box T.
Hence, statement I alone is sufficient to answer. Hence, statement II alone is sufficient to answer.
From II: Hence, option C is correct choice.
We have: 30. A
• The box U is kept three places above the box Q. Explanation
• The box Q is not kept at bottom. From I:
That means, in case (1) box U is kept at top, in case (2) We have:
box U is kept second from top. • Q sits third to left of R, who works in BOB.
• Two boxes are kept between S and P, which is kept • One person sits between R and the one who works
just below the box U. in PNB.
• The box R is neither kept adjacent to the box P nor That means, in case (1) Q sits at extreme left end, in case
to box S. (2) Q sits second from left end.
That means, in case (2) box R is kept at top and box S is • The one who works in PNB sits second to right of
kept at bottom, case (1) is not valid. S.
Based on above given information we have: That means, in case (2) S sits at extreme left end, case (1)
is not valid.
• S neither works in RBI nor works in UBI.
• The one who works in UBI sits immediate right of
P.
That means, S works in SBI and P works in PNB.
Based on above given information we have:

Page 1258 of 1334


Subscribe The Xpress Video Course & Mock Test Package for Bank & Insurance Exams
If there are any suggestions/ errors in our PDFs Feel Free to contact us via this email: admin@exampundit.in
Ultra Practice Bundle PDF
SBI Clerk/ RBI Assist. Mains – Reasoning
Case (1) is not valid as the one who works in PNB sits Explanation
second to right of S.
Clearly, P sits immediate right of the one who works in From I:
RBI. • H sits third to right of F, who sits immediate left
Hence, statement I is alone sufficient. of A.
From II: • D neither sits adjacent to H nor sits adjacent to F.
We have: • D is facing E, who sits immediate left of C which
• The one who works in SBI sits third to left of T at means D sits at the corner.
extreme end. • One person sits between C and A, who doesn’t sits
• T neither works in PNB nor works in BOB. adjacent to H.
• One person sits between the one who works in Based on above given information we have:
BOB and the one who works in PNB.
• Two persons sit between Q and R, who doesn’t sits
adjacent to the one who works in SBI.
That means, Q sits second from left end and the one who
works in SBI sits at left end.
Based on above given information we have:

Here, exact position of G is not known.


Hence, statement I is alone not sufficient.
From II:
We have:
Thus, The one who works in PNB sits immediate right of
• B sits at one of the corner third to right of A.
P the above possibility will become invalid.
• One person sits between A and C, who doesn’t sit
Hence, statement II is not sufficient.
adjacent to B.
Hence, option A is correct choice.
That means, C sits second to left of A.
31. E
Page 1259 of 1334
Subscribe The Xpress Video Course & Mock Test Package for Bank & Insurance Exams
If there are any suggestions/ errors in our PDFs Feel Free to contact us via this email: admin@exampundit.in
Ultra Practice Bundle PDF
SBI Clerk/ RBI Assist. Mains – Reasoning
• C sits adjacent to E, who sits facing D. Hence, both statements I and II together are sufficient.
That means, E sits immediate left of C. Hence, option E is correct choice.
• Two persons sit between D and H when counted 32. D
from the left of D. Explanation
That means, H sits immediate left of B. From I:
Based on above given information we have: We have:
• Age of only two persons is more than P.
• S is younger to P and R but not as much as V and
Q.
• U is eldest and age of only one person is less than
T.
Based on above given information we have:
U > R > P > S > V/Q > T > Q/V.
Since, exact age of Q is not known.
Hence, statement I is not sufficient.

Since, exact position of G is not known. From II:

Hence, statement II is not sufficient. We have:

From I and II: • Age of only three persons is less than S.

After combining both statements we have: • Age of only one person is more than R.
• Age of P is more than S and T but not as much as
U and R.
Based on above given information we have:
U > R > P > S > T/V/Q > V/T/Q > Q/V/T.
Since, exact age of Q is not known.
Hence, statement II is not sufficient.
From I and II:
After combining statement I and II we have:
U > R > P > S > V/Q > T > Q/V.
Since, exact age of Q is not known.
Clearly, H sits second to right of G.
Page 1260 of 1334
Subscribe The Xpress Video Course & Mock Test Package for Bank & Insurance Exams
If there are any suggestions/ errors in our PDFs Feel Free to contact us via this email: admin@exampundit.in
Ultra Practice Bundle PDF
SBI Clerk/ RBI Assist. Mains – Reasoning
Hence, statement I and II together is not sufficient. After combining above statements we have:
Hence, option D is correct choice. G>B>A>E>D>C>F
33. E Thus, salary of C is second lowest.
Explanation Hence, statement I and II together are sufficient to answer
From I: this question.
We have: 34. D
• Salary of A is more than E and D but not as much Explanation
as B. From I:
• Salary of only G is more than B. We have:
B > A > E/ D. • T lives on even number floor and lives three floors
G>B>__>__>__>__>__ away from P.
• Salary of only two persons is less than D. • P lives just below Q, who doesn’t lives on topmost
• Salary of C is not lowest. floor.
Based on above given information we have: That means, in case (1) T lives on floor 2 and Q lives on
G > B > __ >__> D >__> __. floor 6, in case (2) T lives on floor marked 4 and Q lives
Since, exact salary of A, E, F & C are not known. on floor 2, in case (3) T lives on floor 6 and Q lives on
Thus, statement I is not sufficient. floor 4.
From II: • R neither lives on adjacent floor of P nor lives
We have: adjacent floor of Q.
• Salary of F is lowest. • U lives four floors above R.
• Salary of E is more than D and C but not as much That means, in case (1) U lives on top floor, in case (3) U
as G and B. lives on floor 5, case (2) is not valid.
• Salary of only one person is more than B. Based on above given information we have:
• Salary of A is not the highest.
Based on above given information we have:
G > B > __ > __ > __> __> F
Since, exact salary of D, A,E & C are not known.
Thus, statement II is not sufficient alone.
From I and II:

Page 1261 of 1334


Subscribe The Xpress Video Course & Mock Test Package for Bank & Insurance Exams
If there are any suggestions/ errors in our PDFs Feel Free to contact us via this email: admin@exampundit.in
Ultra Practice Bundle PDF
SBI Clerk/ RBI Assist. Mains – Reasoning

Since, exact position of S is not known.


Since, exact position of S is not known.
Hence, statement II is not sufficient.
Thus, statement I is not alone sufficient.
From I and II:
From II:
After combining above statements we have:
We have:
• R lives three floors below Q, who lives on even
numbered floor.
That means, in case (1) R lives on floor 1, in case (2) R
lives on floor 3.
• V neither lives on adjacent floor of Q nor R.
• T lives on adjacent floor of V but not on odd
number floor.
• U lives on any floor above P, who lives on odd
Clearly, exact position of S is not known.
number floor.
Thus, statement I and II together are not sufficient.
That means, in case (1) V lives on top floor and P lives on
Hence, option D is correct choice.
floor number 3, in case (2) V lives on bottom most floor
35. A
and P lives on floor 5.
Explanation
Based on above given information we have:
From I:
We have:
• P sits third from left end.
• One person sits between P and R.
Page 1262 of 1334
Subscribe The Xpress Video Course & Mock Test Package for Bank & Insurance Exams
If there are any suggestions/ errors in our PDFs Feel Free to contact us via this email: admin@exampundit.in
Ultra Practice Bundle PDF
SBI Clerk/ RBI Assist. Mains – Reasoning
• Q neither sits adjacent to P nor sits adjacent to R. We have:
• S sits immediate right of Q. • T sits second to left of Q, who doesn’t sits adjacent
• Neither U nor S sits at end of the row. to U.
That means, Q sits at left end and R sits second from right • S neither sits adjacent to Q nor T.
end. • T sits facing R.
Based on above given information we have: That means, S sits second to left of T and P sits immediate
right of T.
Since, Q and U doesn’t sits together, thus U sits immediate
left of T.
Thus, R sits immediate right of U. Based on above given information we have:
Hence, statement I alone is sufficient to answer.
From II:
We have:
• Three persons sit between Q and R, who doesn’t
sits at the end.
• P sits second to left of R, who doesn’t sits adjacent Thus, S sits immediate right of R.
to S. Hence, statement I is alone sufficient.
• One person sits between T and U. From II:
That means, Q sits at left end. We have:
Based on above given information we have: • U sits facing Q, who sits adjacent to P.
• R neither sits adjacent to P nor sits adjacent to U.
In case (1) P sits immediate right of Q, in case (2) P sits
immediate left of Q.
Since, exact position of U is not known. Based on above given information we have:
Thus, statement II alone is not sufficient to answer.
Hence, option A is correct choice.
36. A
Explanation
From I:

Page 1263 of 1334


Subscribe The Xpress Video Course & Mock Test Package for Bank & Insurance Exams
If there are any suggestions/ errors in our PDFs Feel Free to contact us via this email: admin@exampundit.in
Ultra Practice Bundle PDF
SBI Clerk/ RBI Assist. Mains – Reasoning
Since, person immediate right of R is not known. Clearly, we don’t know about R’s son.
Hence, statement II alone is not sufficient to answer. Thus, statement II alone is not sufficient to answer.
Hence, option A is correct choice. From I and II:
37. D Clearly, both statements are independent so, we can’t
Explanation compare both the statements.
From I: Hence, statement I and II together are not sufficient to
We have: answer.
• R is mother of Q, who is only sister of T’s father. Hence, option D is correct choice.
• L is only brother of T and is son of J who is a 38. D
female. T is a granddaughter of R. Explanation
Based on above given information we have: From I:
We have:
• Height of only two persons is more than Q.
• Height of P is more than R and T but not as much
as U and S.
Based on above given information we have:
S/U > U/S > Q > P > R/T > T/R.
Since, exact height of U and S is not known.
Clearly, we don’t know about R’s son.
Thus, statement I alone is not sufficient to answer this
Hence, statement I alone is not sufficient to answer.
question.
From II:
From II:
We have:
We have:
• R is brother of Q’s mother.
• Height of only two persons is less than P.
• H is mother-in-law of L’s brother.
• Height of S is more than R and U but not as much
• J is married to Q and is only son of T.
as Q and T.
Based on above given information we have:
Based on above given information we have:
T/Q > Q/T > S > P > R/U > U/R.
Since, exact height of U and S is not known.

Page 1264 of 1334


Subscribe The Xpress Video Course & Mock Test Package for Bank & Insurance Exams
If there are any suggestions/ errors in our PDFs Feel Free to contact us via this email: admin@exampundit.in
Ultra Practice Bundle PDF
SBI Clerk/ RBI Assist. Mains – Reasoning
Thus, statement II alone is not sufficient to answer this • R sits immediate left of Q, who sits at one of the
question. smaller side.
From I and II: • One person sits between S and U.
Since, both statements are independent, so the statements That means, T sits facing P.
can’t be compared. Based on above given information we have:
Hence, statement I and II together not sufficient to answer.
39. B
Explanation
From I:
We have:
• U sits second to left of S, who sits at smaller side.
• R neither sits adjacent to U nor sits adjacent to S.
• T sits immediate left of R.
That means, R sits facing U.
Based on above given information we have: Clearly, T sits facing P.
Thus, statement II alone is sufficient to answer this
question.
Hence, option B is correct choice.
40. E
Explanation
From I:
We have:
• The one who likes 10 lives three floors below B.
• B, who likes 2, lives on an odd number floor.
Clearly, exact position of P is not known.
That means, B lives on floor marked 5.
Thus, statement I is not sufficient.
• D, who likes 8, lives on an odd number floor just
From II:
below E.
We have:
• One person lives between D and C, who doesn’t
• P sits third to right of R.
lives on adjacent floor of E.

Page 1265 of 1334


Subscribe The Xpress Video Course & Mock Test Package for Bank & Insurance Exams
If there are any suggestions/ errors in our PDFs Feel Free to contact us via this email: admin@exampundit.in
Ultra Practice Bundle PDF
SBI Clerk/ RBI Assist. Mains – Reasoning
That means, C lives on floor marked 1.
Based on above given information we have:

Since, we don’t know who likes 6.


Thus, statement II is not alone sufficient.
From I and II:
Since, we don’t know who likes 6. After combining both statements together, we have:
Thus, statement I is not alone sufficient.
From II:
We have:
• The one who likes 2 lives just above E.
• At least two person lives below E.
That means, in case (1) E lives on floor marked 4, in case
(2) E lives on floor marked 3.
• C does not like 6.
• D lives two floors above C and likes 8.
That means, in case (1) D lives just below floor of E, case Clearly, E likes 6, thus B lives just above the one who likes

(2) is not valid. 6.

Based on above given information, we have: Hence, both statement I and II are together sufficient to
answer this question.
41. E
Explanation
From I:
We have:

Page 1266 of 1334


Subscribe The Xpress Video Course & Mock Test Package for Bank & Insurance Exams
If there are any suggestions/ errors in our PDFs Feel Free to contact us via this email: admin@exampundit.in
Ultra Practice Bundle PDF
SBI Clerk/ RBI Assist. Mains – Reasoning
• Box U is kept third from top and is kept at a gap
of two box from P, that means box P is kept at
bottom.
• Box R is neither kept adjacent to box P nor box U,
that means box R is kept at top.
• Box T is kept at any place below box U but not
exactly below, that means box T is kept just above
box P.
Based on above given information we have:

Clearly, we don’t know exact position of box Q.


Thus, statement II alone is not sufficient to answer.
From I and II:
After combining above given statements we get:

Clearly, we don’t know position of box Q.


Thus, statement I alone is not sufficient to answer.
From II:
We have:
• Only one box is kept below box T, which is kept
at a gap of two from box S, that means box T is
kept second from bottom.
• Box R is kept adjacent to box S and is at a gap of
Clearly, Box T is kept just below box Q.
one box from box U, that means box R is kept at
Thus, both statement I and II together are sufficient to
top. R is placed above U.
answer this question.
Based on above given information we have:
Hence, option E is correct choice.

Page 1267 of 1334


Subscribe The Xpress Video Course & Mock Test Package for Bank & Insurance Exams
If there are any suggestions/ errors in our PDFs Feel Free to contact us via this email: admin@exampundit.in
Ultra Practice Bundle PDF
SBI Clerk/ RBI Assist. Mains – Reasoning
42. D Day --> xm
Explanation Long --> pm
From I: Heavy --> pz
We have: Clearly, we can’t determine exact code of “Life”.
“Book More Life System” ------ “go la zm hq”. Thus, both statement I and II together are not sufficient to
“Half Book Keep System” ------- “hm dj la zm” answer this question.
After decoding above statements we have: 43. E
Book/System --> la/zm (not necessary in same order) Explanation
More/Life --> go/hq (not necessary in same order) We have:
Half/Keep --> dj/hm (not necessary in same order) • D lives on floor marked 3 and lives three floors
Clearly, we can’t determine exact code of “Life”. below C.
Thus, statement I alone is not sufficient. • F, who likes 16, lives just below C.
From II: • Three persons live between the one who likes 16
We have: and 21.
“Long Day More Life” ------ “pm xm hq go”. • Two persons live between the one who likes 21
“Heavy Day Book System” -------- “xm pz zm la”. and B, who doesn’t likes odd number.
After decoding above statements we have: Based on above given information we have:
Day --> xm
Long/More/Life --> pm/hq/go (not necessary in same
order)
Heavy/Book/System --> pz/zm/la (not necessary in same
order)
Clearly, we can’t determine exact code of “Life”.
Thus, statement IIalone is not sufficient.
From I and II:
After combining above statements we have:
Book/System --> la/zm (not necessary in same order)
From I:
More/Life --> go/hq (not necessary in same order)
We have:
Half/Keep --> dj/hm (not necessary in same order)
• D neither likes 7 nor likes 11.
Page 1268 of 1334
Subscribe The Xpress Video Course & Mock Test Package for Bank & Insurance Exams
If there are any suggestions/ errors in our PDFs Feel Free to contact us via this email: admin@exampundit.in
Ultra Practice Bundle PDF
SBI Clerk/ RBI Assist. Mains – Reasoning
• Two person lives between the one who likes 11
and the one who likes 19.
That means, D likes 19.
Based on above given information we have:

Clearly, we don’t know who likes 7.


Thus, statement II is not sufficient.
From III:
We have:
Clearly, D lives just above the one who likes 7. • E, who doesn’t likes prime number, lives two
Thus, statement I alone is sufficient. floors below the one who likes 19.
From II: • The one who likes 11 lives on floor marked 6.
We have: That means, D likes 19 and A likes 7.
• A, who doesn’t likes 21, lives on any floor below Based on above given information we have:
the one who likes 11.
• At least two person lives between E and the one
likes 11.
Based on above given information we have:

Clearly, D lives just above the one who likes 7.

Page 1269 of 1334


Subscribe The Xpress Video Course & Mock Test Package for Bank & Insurance Exams
If there are any suggestions/ errors in our PDFs Feel Free to contact us via this email: admin@exampundit.in
Ultra Practice Bundle PDF
SBI Clerk/ RBI Assist. Mains – Reasoning
Thus, statement III alone is sufficient. Thus, statement II alone is sufficient to answer.
Hence, option E is correct choice. From III:
44. E We have:
Explanation • Neither Hari nor Ravi sits adjacent to Deep.
We have: That means, Deep sits at left end.
• Sam sits third from right end. Based on above given information we have:
• One person sits between Sam and Hari, who
neither sits adjacent to Ravi nor sits at end of the
row.
That means, Hari sits second to the left of Sam. Clearly, exact position of Shreya is not known.
• Three persons sit between Ravi and Teza. Thus, statement III is not sufficient.
That means, Ravi sits immediate right of Sam. Hence, option E is correct choice.
Based on above given information we have: 45. E
Explanation
We have:
• Box B is kept four places below C, which is not
From I:
• Shreya and Sam don’t sit together. So Shreya may kept adjacent to H.
• One box is kept between C and E.
be sits either from left end of the row or from right
• Box E is kept three places below box G. G is not
of the row.
Statement I alone is not sufficient to answer. kept sixth from the bottom.

From II: That means, in case (1) box B is kept third from bottom,

• Jay sits at any position right of Deep but not at the in case (2) box B is kept second from bottom
• At least three boxes are kept between E and D.
end of the row.
That means, Shreya sits at right end and Deep sits at left That means, in case (1) box G is kept at top, in case (2)

end. box G is kept second from top.

Based on above given information we have: Based on above given information we have:

Clearly, Ravi sits immediate left of Shreya.


Page 1270 of 1334
Subscribe The Xpress Video Course & Mock Test Package for Bank & Insurance Exams
If there are any suggestions/ errors in our PDFs Feel Free to contact us via this email: admin@exampundit.in
Ultra Practice Bundle PDF
SBI Clerk/ RBI Assist. Mains – Reasoning

From I: Clearly, we don’t know which box is kept exactly below


We have: box F.
• Box F is kept at any position below box H. Thus, statement II alone is not sufficient.
Clearly, we don’t know which box is kept exactly below From III:
box F. We have:
Thus, statement I alone is not sufficient. • Box A is not kept adjacent to B but kept above box
From II: H.
We have: That means, box A is kept just below box C.
• Three boxes are kept between box F and A. Based on above given information we have:
Based on above given information we have:

Page 1271 of 1334


Subscribe The Xpress Video Course & Mock Test Package for Bank & Insurance Exams
If there are any suggestions/ errors in our PDFs Feel Free to contact us via this email: admin@exampundit.in
Ultra Practice Bundle PDF
SBI Clerk/ RBI Assist. Mains – Reasoning
Clearly, we don’t know which box is kept exactly below
box F.
Thus, statement III alone is not sufficient.
Hence, option E is correct choice.
46. D
Explanation
We have:
Thus, statement I alone is sufficient.
• R sits facing Q, who doesn’t sits adjacent to T.
From II:
• One person sits between Q and W.
• S and U sit together.
• S and V sit together.
That means, in both case U sits facing T.
• Neither S nor V sits adjacent to Q.
Based on above given information we have:
Based on above given information we have:

Clearly, T sits facing U.


From I:
Thus, statement II alone is sufficient.
We have:
From III:
• T sits second to left of V and sits facing P.
We have:
That means, in case (2) P sits facing T.
• T and W sit together.
Based on above given information we have:
Based on above given information we have:

Page 1272 of 1334


Subscribe The Xpress Video Course & Mock Test Package for Bank & Insurance Exams
If there are any suggestions/ errors in our PDFs Feel Free to contact us via this email: admin@exampundit.in
Ultra Practice Bundle PDF
SBI Clerk/ RBI Assist. Mains – Reasoning
• B is not the eldest but G is the youngest.
That means, F is eldest in the group.
Based on above given information we have:
F > B > D > C/A > A/C > E > G
Clearly, exact age of A is not known.
Thus, statement III is not sufficient.
Hence, option E is correct choice.
Clearly, we don’t know who sits facing T.
48. E
Thus, statement III is not sufficient.
Explanation
Hence, option D is correct choice.
From I:
47. E
We have:
Explanation
• S, who likes Black, lives on an even number floor.
We have:
• The one who likes Black lives two floors below P.
From I and II:
That means, S lives on floor marked 2.
• F is eldest among all.
• Neither the one who likes Green nor Yellow lives
• D is younger to F and B but not C and G.
on lowermost floor.
F > B > D > C, G.
• The one who likes Red lives three floors above Q.
• A is younger to D but not E and G.
That means, Q likes Blue and lives on floor marked 1.
D > A > E, G.
Based on above given information we have:
• Only one person is younger to E.
Based on above given information we have:
F > B > D > A/C > C/A > E > G/C.
Clearly, exact age of A is not known.
Thus, statement I and II is not sufficient.
From III:
We have:
• Only two persons are older to D.
• A is younger to F and B but not E.
• C is elder to E and G but not B and D. F and B are Clearly, exact floor of the one who likes Yellow is not
elder to D. known.
Page 1273 of 1334
Subscribe The Xpress Video Course & Mock Test Package for Bank & Insurance Exams
If there are any suggestions/ errors in our PDFs Feel Free to contact us via this email: admin@exampundit.in
Ultra Practice Bundle PDF
SBI Clerk/ RBI Assist. Mains – Reasoning
Thus, statement I alone is not sufficient to answer this
question.
From II:
We have:
• The one who likes Red lives two floors above S,
who doesn’t likes Green.
• P likes Red and lives on floor 4.
• Two persons live between the one who likes Green
and the one who likes Black.
Clearly, S lives just below the one who likes Yellow.
That means, the one who likes Green lives on top floor.
Thus, statement I and II are together sufficient to answer
• One person lives between the one who likes Green
this question.
and R.
49. A
Based on above given information we have:
Explanation
From I:
We have:
• Height of only one person is more than Mike.
• Height of Jai is more than Rohit but not as much
as Tom and Hari.
Tom/ Hari > Jai > Rohit.
• Hari is not a tallest person and Rohit is not the
shortest person.
Clearly, exact floor of the one who likes Yellow is not Based on above given information we have:
known. Tom > Mike > Hari > Jai > Rohit > Sam
Thus, statement II alone is not sufficient to answer this Clearly, height of only two persons is less than Jai.
question. Thus, statement I alone is sufficient to answer this
From I and II: question.
After combining both statement I and II we have: From II:
We have:
• Height of only two persons are more than Hari.

Page 1274 of 1334


Subscribe The Xpress Video Course & Mock Test Package for Bank & Insurance Exams
If there are any suggestions/ errors in our PDFs Feel Free to contact us via this email: admin@exampundit.in
Ultra Practice Bundle PDF
SBI Clerk/ RBI Assist. Mains – Reasoning
• Height of Rohit is more than Sam but not as much which contains Book is kept second from bottom
as Jai and Mike. and case (2) is not valid.
• Tom is tallest among all. Based on above given information we have:
Based on above given information we have:
Tom > Mike/Jai > Hari > Jai/Mike > Rohit > Sam
Clearly, exact height of Jai and Mike is not known.
Thus, statement II alone is not sufficient to answer this
question.
Statement I alone is sufficient to answer this question and
statement II is not necessary to answer this question.
Hence, option A is correct choice.
50. B
Explanation Case (2) is not valid as the box which contains Book is
We have: kept just above the box B06 and Case (3) is not valid as
• Box B03 is kept at a gap of three boxes below the the box B06 is neither kept adjacent to the box which
box which contains Ball. contains Ball nor the box B03.
• One box is kept between the box which contains From I:
Pen and the box B04, which contains Ball. We have:
• The box which contains Pen is neither kept at top • The box B01 is kept at a gap of two boxes from
nor adjacent to the box B06. the box B02, which contains Book, that means the
• The box B06 is neither kept adjacent to the box box B01 is kept second from top.
which contains Ball nor the box B03, that means • The box B05 is kept just below the box which
in case (1) The box which contains Ball is kept at contains Mobile, that means the box which
the top, in case (2) the box which contains Ball is contains Mobile is kept second from top.
kept third from top, in case (3) the box which Based on above given information we have:
contains Ball is kept second from top.
• The box B06 is kept just below the box which
contains Book, that means in case (1) the box

Page 1275 of 1334


Subscribe The Xpress Video Course & Mock Test Package for Bank & Insurance Exams
If there are any suggestions/ errors in our PDFs Feel Free to contact us via this email: admin@exampundit.in
Ultra Practice Bundle PDF
SBI Clerk/ RBI Assist. Mains – Reasoning
Clearly, only two boxes are kept below the box which
contains Bat.
Hence, statement II alone is sufficient.
From III:
We have:
• The box B02 is kept just above the box which
contains Bottle.
• Two boxes are kept between the box which

Clearly, we don’t know exact position of the box which contains Bottle and the box B05,that means the

contains Bat. box which contains Bottle is kept at bottom.

Hence, statement I alone is not sufficient. • The box B01 doesn’t contain Bat, thus the box B01

From II: must contain Mobile.

We have: Based on above given information we have:

• The box which contains Mobile is kept at a gap of


two boxes above the box B02, that means the box
B02 contains Book.
• Three boxes are kept between the box which
contains Bottle and the box B01, that means the
box B01 is kept second from top.
Based on above given information we have:

Clearly, only two boxes are kept below the box which
contains Bat.
Thus, statement III alone is sufficient.
Hence, option B is correct choice.

Page 1276 of 1334


Subscribe The Xpress Video Course & Mock Test Package for Bank & Insurance Exams
If there are any suggestions/ errors in our PDFs Feel Free to contact us via this email: admin@exampundit.in
Ultra Practice Bundle PDF
SBI Clerk/ RBI Assist. Mains – Reasoning
Coding Decoding with Input Output

Direction1-5: Study the following information following a particular pattern in each step. The
carefully and answer the given question: following is an illustration of input and
A word and number arrangement device when rearrangement.
given an input line of words, rearranges them
Statements Step 1 Step 2 Step 3 Step 4

see more be ees 33t s4n4 c2 4on4 %%1 2& %%%


main remo & 1& &2 &&& %&
be & &&&
inma
cold ldco med4 3& %&
particular arparti 10sq10su10d10m &6&&& &&&&&
when and cul 4oxi oe3 &3 2% &% &%
enwh
nda
bat hat input atb 3uc 3ui 5u5oq %2 %2 %& %&
words ath etx5s %3% 4% %%% &%
utinp
dswor

1. What is the step 2 of the given following e. None of these


statement? 2) What is the 3rd word of the 3rd step of the
Statement- blue gone white following statement from right?
a. 44cm o4h4 u5xi5 Statement: - injury even dog girl
b. 3ui 5u5oq etx5s a. 2%&
c. s4n4 c2 4on4 b. 3&&
d. 4oxi oe3 etx5s c. &2&

Page 1277 of 1334


Subscribe The Xpress Video Course & Mock Test Package for Bank & Insurance Exams
If there are any suggestions/ errors in our PDFs Feel Free to contact us via this email: admin@exampundit.in
Ultra Practice Bundle PDF
SBI Clerk/ RBI Assist. Mains – Reasoning
d. &2% e. None of these
e. None of these 5) What is 3rd word of the 2nd step of the
3) What is the 3rd word step 4 of the given following statement?
following statement? Statement: - hat input words
Statement: - particular when and a. sw5r
a. &% b. 5u5oq
b. && c. etx5s
c. %% d. either a or b
d. %& e. None of these
e. None of these Direction6-10: Study the following information
4) What is 2nd word of the 1st step of the carefully and answer the given question:
following statement? A word and number arrangement device when
Statement: - game win month given an input line of words, rearranges them
a. inw following a particular pattern in each step. The
b. nwi following is an illustration of input and
c. win rearrangement.
d. either a or b
Statements Step Step Step Step
1 2 3 4
White gone Wht w8t 2@ @@
bad gn g14 1@ #@
bd b4 1@ #@
Red turns Rd r4 1@ #@
blue trns t18ns 3@ #@
bl b12 1@ #@
Black eyes Blck b12ck 3@ #@
true ys tr y19 1# ##
t18 1@ #@

Page 1278 of 1334


Subscribe The Xpress Video Course & Mock Test Package for Bank & Insurance Exams
If there are any suggestions/ errors in our PDFs Feel Free to contact us via this email: admin@exampundit.in
Ultra Practice Bundle PDF
SBI Clerk/ RBI Assist. Mains – Reasoning
6. What is the step 1 of the given following 9) What is 1st word of the 1st step of the
statement? following statement?
Statement- brown pram gone Statement: - raju wants car
a. Brwn prm gn a. rj
b. bron prm gon b. rau
c. orwn parm gen c. rju
d. Brwn pram gne d. either a or b
e. None of these e. None of these
7) What is the 2nd word of the 4th step of the 10) What is 4th word of the 3rd step of the
following statement? following statement?
Statement: - white elephant blue Statement: - kids playing football daily
a. @@ a. 2@
b. #@ b. 4@
c. ## c. 4#
d. #@ d. 2#
e. None of these e. None of these
8) What is the 3rd word of the step 2 of the Direction 11-15: Study the following
given following statement? information carefully and answer the given
Statement: - notebook turns yellow question:
a. y12lw A word and number arrangement device when
b. yl12w given an input line of words, rearranges them
c. yll23 following a particular pattern in each step. The
d. 25llw following is an illustration of input and
e. None of these rearrangement.
Statements Step 1 Step 2 Step Step
3 4
Eat healthy Fbs F21 1& &&
food gfbksgx gfbks31 5& &7
eppc ep19 2& $&

Page 1279 of 1334


Subscribe The Xpress Video Course & Mock Test Package for Bank & Insurance Exams
If there are any suggestions/ errors in our PDFs Feel Free to contact us via this email: admin@exampundit.in
Ultra Practice Bundle PDF
SBI Clerk/ RBI Assist. Mains – Reasoning
Play strong Okbx Ok26 2$ $&
daily rsqpmf rsqp19 4& $&
cbjkx cbj35 3& &&
Love each Kpuf Kp27 gb9 2& $&
other fbbg psg23 2& $&
psgfq 3& &&

11. What is the step 3 of the given following d. gk13


statement? e. None of these
Statement- square are circle 14) What is 2nd word of the 4th step of the
a. 4& 1& 4& following statement?
b. 4$ 1& 4& Statement: - Only drive
c. 4$ 1& 4$ a. &$
d. 4& 1$ 4& b. $$
e. None of these c. &&
12) What is the 1st word of the 1st step of the d. either a or b
following statement? e. None of these
Statement: - learn again 15) What is 1st word of the 2nd step of the
a. Kfbqm following statement?
b. Kfbrm Statement: - party hard
c. Kfbqn a. Obq43
d. Kebqm b. Ocq43
e. None of these c. Obr43
13) What is the 2nd word of the step 2 of the d. Ocr43
given following statement? e. None of these
Statement: - gone high Directions 16-20: Study the following information
a. Fp19 carefully and answer the questions given below.
b. gj13 In a certain language,
c. gj12

Page 1280 of 1334


Subscribe The Xpress Video Course & Mock Test Package for Bank & Insurance Exams
If there are any suggestions/ errors in our PDFs Feel Free to contact us via this email: admin@exampundit.in
Ultra Practice Bundle PDF
SBI Clerk/ RBI Assist. Mains – Reasoning
‘CLIPS REFINE APPRECIATE MASSAGE’ is a. 19I
coded as ‘13#FI 8##S 16I 4###EC’ b. 5#TT
‘EFFORTS RECRUITMENT MIND SOULFUL’ c. 7##EM
is coded as ‘7##L 2###I 9IN 14#O’ d. 8DU
‘STUDIO LOTION RETRIBUTE REQUIRED’ e. None of these
is coded as ‘4UD 13###I 14##UI 2#TI’ 20) EFFORTS is coded as?
‘MODULE MOVEMENT ANCIENT MATTER’ a. 7##L
is coded as ‘19I 5#TT 7##EM 8DU’ b. 2###I
16) What is the code for ‘MOISTURISER’? c. 9IN
a. 13#U d. 14#O
b. 5###U e. None of these
c. 8##U Directions 21-25: Study the following
d. 5###TU information carefully and answer the
e. None of these questions given below.
17) ‘ROTATING’ is coded as? In a certain language,
a. 9#AT “Some parts are tough” is coded as “O#5, O&8,
b. 9##T A%19, R@5”,
c. 11#AT “Search all the persons” is coded as “E#8, H&5,
d. 11##AT E%19, L@12”,
e. None of these “Some truths are personal” is coded as “R@5,
18) Which of the following word is coded as O#5, R&19, E%12”,
‘1##M’? “Always prepare the sandwich” is coded as
a. SUPERMARKET “H&5, A#8, L@19, R%5,”
b. STUPIDITY 21) What is the code for “TALENT”?
c. SUMMARY a. A&20
d. FAILURE b. S&4
e. None of these c. A@4
d. S@20
19) MOVEMENT is coded as? e. None of these

Page 1281 of 1334


Subscribe The Xpress Video Course & Mock Test Package for Bank & Insurance Exams
If there are any suggestions/ errors in our PDFs Feel Free to contact us via this email: admin@exampundit.in
Ultra Practice Bundle PDF
SBI Clerk/ RBI Assist. Mains – Reasoning
22) Which of the following statements is true? When any two symbols used together then one of
a. KEPT THERE is coded as K%12, H&6 them represent hour hand and another represent
b. PALACE is coded as A%5 minute hand in a clock.
c. SKEPTICAL is coded as K#5 For e.g
d. HAPPY ALRIGHT is A@12, A@20 D ~ is represents 6:10 AM in clock.
e. None of these ^ F is represents 7:50 AM in clock.
23) What is the code of the sentence “ALONE Note - Consider all time at AM unless stated.
PRAYER STRUGGLE”? 26) Sanjay starts walking at + F and goes to
a. R%18, E%19, T@5 park and come back to home at D^. How
b. L@5, P%18, S#5 much time Sanjay took in this period?
c. L@5, R%18, T#5 a. 112 min
d. T#5, R@18, L@5 b. 113 min
e. None of these c. 115 min
24) What is code for “personal”? d. 110 min
a. R@5 e. 105 min
b. O#5 27) Renu start listening song at F + and listens
c. R&19 until E ^. How much time she thinks she spend
d. E%12 in listening songs?
e. None of these a. 2 hour 15 minutes
25) Which letter is coded as the code for b. 2 hour 18 minutes
R&19? c. 3 hour 14 minutes
a. Some d. 3 hour 17 minutes
b. truths e. None of these
c. are 28) A Fashion show is scheduled at ^ F the
d. personal Manager is required to reach at least 90 mins
Direction 26-30: Study the information early so that it looks all the arrangements and
carefully answers the questions given below. schedule everything at the show. At what time
In a coded language D, E, F, +, ^, ~ are should he reach?
represented by 6, 12, 10, 4, 7 and 2 respectively. a. F^

Page 1282 of 1334


Subscribe The Xpress Video Course & Mock Test Package for Bank & Insurance Exams
If there are any suggestions/ errors in our PDFs Feel Free to contact us via this email: admin@exampundit.in
Ultra Practice Bundle PDF
SBI Clerk/ RBI Assist. Mains – Reasoning
b. D+ “Public tackle sincere here” is coded as “18@21,
c. +D 14$17, 3#0, __(1)__”
d. ~E “Silence not corner hopeful” is coded as “12$15,
e. Data inadequate 16&19, 18&21, 20&23”
29) A washing machine takes 100 minutes to “Run mobile __(2)__ situation” is coded as
be Wash and Dry clothes completely at what “18#21, 20$23, 2&5, 14%17”
time should Sneha start the machine so that it 31) Which among the following code comes in
is wash and Dry clothes at the time when she place of (1)?
wants the cloth to wear and she wants the a. 3%6
cloth to wear at +D? b. 2%5
a. D~ c. 6@9
b. ^+ d. 5$8
c. F+ e. 4$7
d. DE 32) Which among the following word comes in
e. ~F place of (2)?
30) If the Program start at ~^ but dancer a. since
reaches 120 minutes late. At what time does b. timely
she reach? c. horticulture
a. ^~ d. garden
b. +F e. covers.
c. +^ 33) Which among the following is correctly
d. D+ matched?
e. E~ a. Picnic – 3&0
Directions 31-35: Answer the following b. Hungama – 17%14
questions based on the information given c. Lover – 22&25
below: d. Pointer – 10&13
In a code language, e. Quite – 9&12
“Centre solution jump migrate” is coded as 34) Which of the following is the code for
“12&15, 7$4, 13%10, 14@17” “hopeful”?

Page 1283 of 1334


Subscribe The Xpress Video Course & Mock Test Package for Bank & Insurance Exams
If there are any suggestions/ errors in our PDFs Feel Free to contact us via this email: admin@exampundit.in
Ultra Practice Bundle PDF
SBI Clerk/ RBI Assist. Mains – Reasoning
a. 12$15 e. None of these
b. 16&19 36-40) Direction: In each of the question given
c. 18&21 below, a group of digits/letter is given followed
d. 20&23 by four combinations of symbols numbered
e. None of these (1), (2), (3) and (4). You have to find out which
35) Which of the following is the code for of the four combinations correctly represents
“migrate”? the group of digits/letters based on the symbol
a. 12&15 codes and the conditions given below. If none
b. 7$4 of the four combinations represents the group
c. 13%10 of digits correctly, give (5) i.e., none of these as
d. 14@17 the answer.
Digit 3 8 9 7 4 6 1 2 5
Symbo # % @ $ * € © µ £
l

Conditions for coding the group elements: Note: If two conditions follows at the same time,
(i) If the first digit is even and the last digit is apply condition in consecutive order i,e condition
divisible by 3, then both are to be coded as X. iv is followed after applying condition I or ii or iii
(ii) If the first, as well as the last digit, is odd, 36) What is the code for “29548”?
then both are to be coded by the code of the first a. @ % µ * $
digit. b. µ µ £ * %.
(iii) If the first digit is odd and the last digit is c. * © € $ #
even number, then the code of the first and last d. $ @ % µ *
digit is to be interchanged. e. None of these
(iv) If even number is followed by perfect square, 37) What is the code for “18537”?
then the code of the perfect square is same as the a. © € $ # $
code of that even number. (1 is counted as perfect b. € $ # * %
square) c. © € $ # *
d. © % £ # ©

Page 1284 of 1334


Subscribe The Xpress Video Course & Mock Test Package for Bank & Insurance Exams
If there are any suggestions/ errors in our PDFs Feel Free to contact us via this email: admin@exampundit.in
Ultra Practice Bundle PDF
SBI Clerk/ RBI Assist. Mains – Reasoning
e. None of these Based on this coded language, answer the
38) What is the code for “41359”? following questions.
a. X © # £ @ 41) Which of the following is the sum of
b. * © µ € % AABBAA and ABABAB?
c. ©© # £ X a. BAAAAB
d. * # % € X b. BAABAB
e. None of these c. BAAABB
39) What is the code for “61348”? d. BBAAAB
a. $ * % € $ e. None of them
b. € € # * %. 42) What is the resultant if BAAAAA is
c. @ µ € © % divided by BAAA?
d. $ * # % € a. BA
e. None of these b. BAA
40) What is the code for “74523”? c. BB
a. $*£ µ $. d. BBA
b. %*£ µ $. e. BBB
c. $*£ % $. 43) Which of the following is the average of
d. $*£ %% BBBB and BBBBB?
e. None of these a. 22
41-45) Directions: In a certain number system b. 18
there are only two notations to represent c. 25
numbers: A and B. 0 is represented by A and 1 d. 23
by B. The subsequent numbers are e. None of these
represented in the following manner: 44) What is the cube of 20% of BBBB?
2 is represented as BA, a. BBBAB
3 is represented as BB, b. BBBBA
4 is represented as BAA, c. BBBAA
5 is represented as BAB and so on. d. BBABB
e. None of them.

Page 1285 of 1334


Subscribe The Xpress Video Course & Mock Test Package for Bank & Insurance Exams
If there are any suggestions/ errors in our PDFs Feel Free to contact us via this email: admin@exampundit.in
Ultra Practice Bundle PDF
SBI Clerk/ RBI Assist. Mains – Reasoning
45) Which of the following is a multiple of 47) What is the code for “CLEARED
BABB? EXAM”?
a. BBABA a. E4&E X2#N
b. BBBAA b. E4&D A2%N
c. BABAB c. G5&E X3#N
d. BBBBA d. E4&F X4#O
e. BABBA e. None on these
46-50) Directions: Answer the following 48) What is the code for ‘AUGUST’?
questions based on the information given a. S3%U
below: b. U6$R
In a code language, c. U3%U
“PREACH ENLARGE BOARD TOUCHING” is d. S5&U
coded as “O5*H, R3$E, R4%I, G4&F” e. None of these.
“OFFICE THICK KINGDOM SAVER” is coded 49) What is the code for “KINGDOM”?
as “E3$S, C4$L, O5&N, F3%F” a. E3$S
“HOSTILE DONE STITCH PAID” is coded as b. C4$L
“L4&F, O2#F, A2#E, T5%I” c. O5&N
“EMOTION APART PASTE SULPHAT” is d. F3%F
coded as “A5&U, T3$F, O3&O, R3$U” e. None of these
46) What would be the word for ‘N4&U’? 50) What is the code for “COLD CITY”?
a. PROMINENT a. I3#Z O3#E
b. EMINENT b. I3@Z O3@E
c. UBUNTU c. O3&Z I3&E
d. PRESENT d. O3#Z I3@E
e. None of these. e. None of these

Coding Decoding with Input Output – Answer and Explanation


1. A
2. C
3. A
Page 1286 of 1334
Subscribe The Xpress Video Course & Mock Test Package for Bank & Insurance Exams
If there are any suggestions/ errors in our PDFs Feel Free to contact us via this email: admin@exampundit.in
Ultra Practice Bundle PDF
SBI Clerk/ RBI Assist. Mains – Reasoning
4. A
5. C
Solution 1
The pattern followed here is,
In step 1 →
Last two alphabets of every word are placed in front.
Example: see, “ee” will come in front of the word and all will become “ees”.
Step 2 →
Consonants are replaced by the next letter of English alphabetical series.
Vowels are replaced by the number of letters present in that word.
Example: more, m → n, r → s, e→ 4, o→ 4
More- s4n4
Step 3 →
In place of alphabets, number of alphabets present in that word are taken.
In place of odd number → %
In place of even number → &
Example: s4n4
4→ &, n, s→ 2, 4→ &
Step 4: 2&&
Symbols are kept same, but numbers are replaced.
Odd number → %
Even number → &
Sentence: blue gone white
Applying conditions:
Statements Step 1 Step 2 Step 3 Step 4
blue gone Uebl 44cm &&2 &&&
white nego o4h4 2&& &&&
tewhi u5xi5 3%% %%%

Page 1287 of 1334


Subscribe The Xpress Video Course & Mock Test Package for Bank & Insurance Exams
If there are any suggestions/ errors in our PDFs Feel Free to contact us via this email: admin@exampundit.in
Ultra Practice Bundle PDF
SBI Clerk/ RBI Assist. Mains – Reasoning
Hence in second step blue gone white is coded as 44cm o4h4 u5xi5
Solution 2
Applying conditions:
Statements Step 1 Step 2 Step 3 Step 4
Ryinju %&&&
Injury even S66ok6 4o4w 3he 3&&&
ehev ogd &&&
dog girl smh4 &2& %2 3&
rlgi %& %&

Hence &2& is correct answer


Solution 3
Applying conditions:
Statements Step 1 Step 2 Step 3 Step 4
arparticul &6&&&
particular
enwh nda 10sq10su10d10m &3 &&&&&
when and
4oxi oe3 2% &% &%

Hence &% is correct answer


Solution 4
Applying conditions:
Statements Step 1 Step 2 Step 3 Step 4
mega
game win n4h4 3ox
inw 2@@ #2 4# @@@ @@ @#
month uin5o
thmon

Hence inw is correct answer


Solution 5
Applying conditions:
Statements Step 1 Step 2 Step 3 Step 4

Page 1288 of 1334


Subscribe The Xpress Video Course & Mock Test Package for Bank & Insurance Exams
If there are any suggestions/ errors in our PDFs Feel Free to contact us via this email: admin@exampundit.in
Ultra Practice Bundle PDF
SBI Clerk/ RBI Assist. Mains – Reasoning
ath utinp 3ui 5u5oq %2 %&
bat hat
dswor etx5s %3% %%%
input
4% &%

Hence etx5s is correct answer


Solution 6-10
6. A
7. A
8. A
9. A
10. A
The pattern followed here is,
In step 1 →
Vowels are eliminated from the word and only consonants are written.
Example: White, will become wht as I and e are vowels and they are eliminated
Step 2 →
The second letter of the changed is changed to numerical position
Example: wht, h numerical position is 8 so “wht” is coded as “w8t” and rest letters will be kept same
Step 3 →
In place of alphabets, number of alphabets present in that word is taken.
In place of odd number → #
In place of even number → @
Example: w8t
8→ @, w, t→ 2,
W8t→2@
Step 4: 2@
Symbols are kept same, but numbers are replaced.
Odd number → #
Even number → @

Page 1289 of 1334


Subscribe The Xpress Video Course & Mock Test Package for Bank & Insurance Exams
If there are any suggestions/ errors in our PDFs Feel Free to contact us via this email: admin@exampundit.in
Ultra Practice Bundle PDF
SBI Clerk/ RBI Assist. Mains – Reasoning
Sentence: brown pram gone
Applying conditions:
Statements Step 1 Step 2 Step 3 Step 4
brown Brwn B18wn 3@ 2@ #@ @@
pram gone prm gn p18m g14 1@ #@

Hence in first step brown pram gone is coded as Brwn prm gn


Solution 7
Applying conditions:
Statements Step 1 Step 2 Step 3 Step 4
white W8t
Wht 2@ 4@ @@ @@
elephant l16hnt
lphnt bl 1@ #@
blue b12

Hence @@ is correct answer


Solution 8
Applying conditions:
Statements Step 1 Step 2 Step 3 Step 4
notebook Ntbk trns N20bk 3@ 3@ #@ #@
turns yllw t18ns 3@ #@
yellow y12lw

Hence y12lw is correct answer

Solution 9
Applying conditions:
Statements Step 1 Step 2 Step 3 Step 4
raju wants Rj wnts 1@ 3@ #@ #@
R10 w14ts c18
car cr @ #@

Page 1290 of 1334


Subscribe The Xpress Video Course & Mock Test Package for Bank & Insurance Exams
If there are any suggestions/ errors in our PDFs Feel Free to contact us via this email: admin@exampundit.in
Ultra Practice Bundle PDF
SBI Clerk/ RBI Assist. Mains – Reasoning

Hence Rj is correct answer


Solution 10
Applying conditions:
Statements Step 1 Step 2 Step 3 Step 4
kids Kds K4s p12yng 2@ 4@ @@
playing plyng f20bll d12y 4@ 2@ @@
football ftbll dly @@
daily @@

Hence 2@ is correct answer


Solution 11-15
11. A
12. A
13. B
14. C
15. A

The pattern followed here is,


In step 1 →
Vowels will be +1
Consonants will be -1
Example: healthy, will become gfbksgx
Step 2 →
The last two letters of the word are changed to sum of their numerical position
Example: gfbksgx, “g and x” numerical position is 7 and 24 so 7+24=31 so gfbksgx is coded as “gfbks31”
and rest letters will be kept same
Step 3 →
In place of alphabets, number of alphabets present in that word is taken.

Page 1291 of 1334


Subscribe The Xpress Video Course & Mock Test Package for Bank & Insurance Exams
If there are any suggestions/ errors in our PDFs Feel Free to contact us via this email: admin@exampundit.in
Ultra Practice Bundle PDF
SBI Clerk/ RBI Assist. Mains – Reasoning
In place of odd number → &
In place of even number → $
Example: gfbks31
31→ &,
g, f, b, k, s→ 5,
gfbks31→ 5&
Step 4: 5&
Symbols are kept same, but numbers are replaced.
Odd number → &
Even number → $
Sentence: square are circle
Applying conditions:
Statements Step 1 Step 2 Step 3 Step 4
square are Rpvbqf Rpvb23 4& 1& $& &&
circle bqf b23 4& $&
bjqbkf bjqb17

Hence in 3rd step square are circle is coded as 4& 1& 4&
Solution 12
Applying conditions:
Statements Step 1 Step 2 Step 3 Step 4
learn Kfbqm Kfb30
3$ 3& &$ &&
again bfbjm bfb23

Hence Kfbqm is correct answer


Solution 13
Applying conditions:
Statements Step 1 Step 2 Step 3 Step 4

Page 1292 of 1334


Subscribe The Xpress Video Course & Mock Test Package for Bank & Insurance Exams
If there are any suggestions/ errors in our PDFs Feel Free to contact us via this email: admin@exampundit.in
Ultra Practice Bundle PDF
SBI Clerk/ RBI Assist. Mains – Reasoning
Fpmf Fp19 2& 2& $& $&
gone high
gjfg gj13

Hence gj13 is correct answer


Solution 14
Applying conditions:
Statements Step 1 Step 2 Step 3 Step 4
Pmkx
Only drive Pm35 cqj27 2& 3& $& &&
cqjuf

Hence && is correct answer


Solution 15
Applying conditions:
Statements Step 1 Step 2 Step 3 Step 4
Obqsx Obq43 gb20 3& 2$ && $$
party hard
gbqc

Hence Obq43 is correct answer


Solution 16-20
16. B
17. C
18. A
19. C
20. D
The coding language used to code the given sentences should be decoded in three steps.
Step I: The first element of the code is the difference in the alphabetic positions of first and the last alphabets
of the word.
Step II: The second element of the code is the symbol on the basis of the number of pairs of same letters in the
words.

Page 1293 of 1334


Subscribe The Xpress Video Course & Mock Test Package for Bank & Insurance Exams
If there are any suggestions/ errors in our PDFs Feel Free to contact us via this email: admin@exampundit.in
Ultra Practice Bundle PDF
SBI Clerk/ RBI Assist. Mains – Reasoning
For 1 pair = #
For 2 pairs = ##
For 3 pairs = ###
Step III: The third element of the code is the middle alphabet/alphabets of the word.
For example,
APPRECIATE is coded as ‘4###EC’
Difference of A and E = 5 – 1= 4
Three pairs (AA, EE, PP) = ###
EC are the middle alphabets.
Solution 21-25
21. A
22. B
23. C
24. D
25. B
From the above statements, we can see that each statement contains words whose first alphabets are A, P, T
and S. Also, each code contains symbols among @, %, &, #. So, we conclude that the word which starts from
A has ‘@’ symbol, P has ‘%’ symbol, T has ‘&’ symbol and S has ‘#’ symbol. Second alphabet of the word
from the left is written as first, then symbol and then the position of last alphabet of the words.
For example, first letter of the word ‘SEARCH’ is S, so we would use # symbol. Second letter of the word is
E. Position of the last letter i.e. H, in the word according to alphabetical series is 8. So, ‘SEARCH’ is coded as
‘E#8’.
Solution 26-30
26. E
27. A
28. B
29. E
30. C
Symbol / Letter D E F + ^ ~

Page 1294 of 1334


Subscribe The Xpress Video Course & Mock Test Package for Bank & Insurance Exams
If there are any suggestions/ errors in our PDFs Feel Free to contact us via this email: admin@exampundit.in
Ultra Practice Bundle PDF
SBI Clerk/ RBI Assist. Mains – Reasoning
Code 6 12 10 4 7 2

Sanjay starts walking at + F means 4:50 AM.


He reached back home at D ^ means 6:35 AM.
Hence, Time taken by Sanjay in this period is 6:35 - 4:50 = 105 minutes.
Solution 27
Symbol / Letter D E F + ^ ~
Code 6 12 10 4 7 2

Renu starts listening song at F + means 10:20 AM


She listens until it is E ^ means 12:35 PM
Hence, Renu spends time in listening songs is 12:35 PM - 10:20 AM = 2 hours 15 minutes.
Solution 28
Symbol / Letter D E F + ^ ~
Code 6 12 10 4 7 2

Scheduled time of fashion show ^ F means 7:50 AM


90 mins early means 6:20 AM i.e. D+.
Hence, the manger should reach at D+.
Solution 29
Symbol / Letter D E F + ^ ~
Code 6 12 10 4 7 2

She wants the cloth to wear at +D means 04:30 AM.


04:30 AM - 100 min = 2:50 AM i.e. ~F.
Hence, Sneha starts the washing machine at ~F.
Solution 30
Symbol / Letter D E F + ^ ~
Code 6 12 10 4 7 2

Page 1295 of 1334


Subscribe The Xpress Video Course & Mock Test Package for Bank & Insurance Exams
If there are any suggestions/ errors in our PDFs Feel Free to contact us via this email: admin@exampundit.in
Ultra Practice Bundle PDF
SBI Clerk/ RBI Assist. Mains – Reasoning

The Program starts at ~^ means 2:35 AM.


Dancer reaches 120 minutes late means; 2:35 + 120 = 04:35 AM i.e. +^.
Hence, worker reaches at +^.
Solution 31-35
31. B
32. D
33. C
34. B
35. B
In each code, then number before the symbol is the value of the 3rd letter (from the left end) of the word in the
alphabetical series.
The symbol in the code is as per the vowel which is 2nd letter from the left end in each word:
a. For vowel “A”, the symbol used is #
b. For vowel “E”, the symbol used is @
c. For vowel “I”, the symbol used is $
d. For vowel “O”, the symbol used is &
e. For vowel “U”, the symbol used is %
If the number representing the 3rd alphabet in alphabetical series from left end is an even number, then that
even number is added by 3 and the obtained number is the number after the symbol.
If the number representing the 3rd alphabet in alphabetical series from left end is an odd number, then the
difference of odd number and 3 and the obtained number is the number after the symbol.
Example:
‘CENTRE’
3rd letter (from the left end) of the word is N, its value in alphabetical series is 14, so 14 is the number before
the symbol.
2nd letter from the left end of the word is E, so the symbol used is @.
As the number representing 3rd alphabet from left end is 14 (an even number), so the number after the symbol
is (14 + 3)=17.

Page 1296 of 1334


Subscribe The Xpress Video Course & Mock Test Package for Bank & Insurance Exams
If there are any suggestions/ errors in our PDFs Feel Free to contact us via this email: admin@exampundit.in
Ultra Practice Bundle PDF
SBI Clerk/ RBI Assist. Mains – Reasoning
Therefore, the code for ‘CENTRE’ is ‘14@17’.
Solution 36-40
36. B
37. D
38. C
39. B
40. A

Digit 3 8 9 7 4 6 1 2 5
Symbo # % @ $ * € © µ £
l

Conditions for coding the group elements:


If even number is followed by perfect square, then the code of the perfect square is same as the code of that
even number.
so the required code for 29548 is µ µ £ * %.

Solution 37
Digit 3 8 9 7 4 6 1 2 5
Symbo # % @ $ * € © µ £
l
Conditions for coding the group elements:
(ii). If the first as well as the last digit is odd, then both are to be coded by the code of the first digit.
By using condition (ii), the code for 18537 is © % £ # ©.

Solution 38
Digit 3 8 9 7 4 6 1 2 5
Symbo # % @ $ * € © µ £
l

Page 1297 of 1334


Subscribe The Xpress Video Course & Mock Test Package for Bank & Insurance Exams
If there are any suggestions/ errors in our PDFs Feel Free to contact us via this email: admin@exampundit.in
Ultra Practice Bundle PDF
SBI Clerk/ RBI Assist. Mains – Reasoning

Conditions for coding the group elements:


(i). If the first digit is even and the last digit is divisible by 3, then both are to be coded as X.
(iv) If even number is followed by perfect square, then the code of the perfect square is same as the code of
that even number.
By using condition (i) and (iv), the code for 41359 is ©© # £ X
Solution 39
Digit 3 8 9 7 4 6 1 2 5
Symbo # % @ $ * € © µ £
l

Conditions for coding the group elements:


(iv) If even number is followed by perfect square, then the code of the perfect square is same as the code of
that even number.
Clearly, the code for 61348 will be € € # * %.
Solution 40
Digit 3 8 9 7 4 6 1 2 5
Symbo # % @ $ * € © µ £
l

Conditions for coding the group elements:


(ii). If the first as well as the last digit is odd, then both are to be coded by the code of the first digit.
By using condition (ii), the code for 74523 is $*£ µ $.
Solution 41-45
41. A
42. B
43. D
44. D
45. E

Page 1298 of 1334


Subscribe The Xpress Video Course & Mock Test Package for Bank & Insurance Exams
If there are any suggestions/ errors in our PDFs Feel Free to contact us via this email: admin@exampundit.in
Ultra Practice Bundle PDF
SBI Clerk/ RBI Assist. Mains – Reasoning
Logic:
So AABBAA → 001100
So, the equivalent decimal of 001100 is
(0 ×20 ) + (0 ×21 ) + (1 ×22 ) + (1 ×23 ) + (0 ×24 ) + (0 ×25 )
=0+0+4+8+0+0
= 12
And ABABAB → 010101
So, the equivalent of 010101 is
(1 ×20 ) + (0 ×21 ) + (1 ×22 ) + (0 ×23 ) + (1 ×24 ) + (0 ×25 )
= 1 + 0 + 4 + 0 + 16 + 0
= 21.
The sum is 12 + 21 = 33
And the binary equivalent of 33 is
33 = 32 + 1 → 100001.
And by the notations, it is BAAAAB.
Hence, BAAAAB is the correct answer.

Solution 42
Logic:
BAAAAA → 100000
The decimal equivalent of 100000 is
(0 ×20 ) + (0 ×21 ) + (0 ×22 ) + (0 ×23 ) + (0 ×24 ) + (1 ×25 )
= 0 + 0 + 0 + 0 + 0 + 32
= 32
And BAAA → 1000
The decimal equivalent of 1000 is
(0 ×20 ) + (0 ×21 ) + (0 ×22 ) + (1 ×23 )
=8
=0+0+0+8

Page 1299 of 1334


Subscribe The Xpress Video Course & Mock Test Package for Bank & Insurance Exams
If there are any suggestions/ errors in our PDFs Feel Free to contact us via this email: admin@exampundit.in
Ultra Practice Bundle PDF
SBI Clerk/ RBI Assist. Mains – Reasoning
=8
So, 32/8 = 4.
And the binary equivalent of 4 is 4 + 0 + 0 → 100 → BAA.
Hence, BAA is the correct answer
Solution 43
Logic:
BBBB → 1111 which is the binary equivalent of

(1 ×20 ) + (1 ×21 ) + (1 ×22 ) + (1 ×23 )=


1 + 2 + 4 +8 = 15.
BBBBB → 11111 which is the binary equivalent of
(1 ×20 ) + (1 ×21 ) + (1 ×22 ) + (1 ×23 ) + (1 ×24 )
1 + 2 + 4 + 8 + 16 = 31
15 + 31 = 46
46/2 = 23
Hence, 23 is the correct answer.

Solution 44
Logic:
BBBB → 1111 which is the binary equivalent of
(1 ×20 ) + (1 ×21 ) + (1 ×22 ) + (1 ×23 )=
1 + 2 + 4 +8 = 15.
20% of 15 is 3 and cube of 3 is 27.
Binary equivalent of 27 will be 16 + 8 + 2 + 1.

So, 11011 → BBABB.


Hence BBABB is the correct answer.

Solution 45

Page 1300 of 1334


Subscribe The Xpress Video Course & Mock Test Package for Bank & Insurance Exams
If there are any suggestions/ errors in our PDFs Feel Free to contact us via this email: admin@exampundit.in
Ultra Practice Bundle PDF
SBI Clerk/ RBI Assist. Mains – Reasoning
Logic:
BABB → 1011 is the binary equivalent of
(1 ×20 ) + (1 ×21 ) + (0 ×22 ) + (1 ×23 )= 11
1) BBABA → 11010 is the binary equivalent of
(0 ×20 ) + (1 ×21 ) + (0 ×22 ) + (1 ×23 ) + (1 ×24 ) =
0 + 2 + 0 + 8 + 16 = 26
26 is not a multiple of 11.
2) BBBAA → 11100 is the binary equivalent of
(0 ×20 ) + (0×21 ) + (1 ×22 ) + (1 ×23 ) + (1 ×24 )
0 + 0 + 4 + 8 + 16 = 28
26 is not a multiple of 11.
3) BABAB → 10101 is the binary equivalent of
(1 ×20 ) + (0×21 ) + (1 ×22 ) + (0 ×23 ) + (1 ×24 )
1 + 0 + 4 + 0 + 16 = 21
26 is not a multiple of 11.
4) BBBBA → 11110 is the binary equivalent of
(0 ×20 ) + (1×21 ) + (1 ×22 ) + (1 ×23 ) + (1 ×24 )
0 + 2 + 4 + 8 + 16 = 30
26 is not a multiple of 11.
5) BABBA → 10110 is the binary equivalent of
(0 ×20 ) + (1×21 ) + (1 ×22 ) + (0 ×23 ) + (1 ×24 )
0 + 2 + 4 + 0 + 16 = 22
Hence, 22 is a multiple of 11.
Hence, BABBA is the correct answer.

Solution 46-50
46. B
47. A
48. C

Page 1301 of 1334


Subscribe The Xpress Video Course & Mock Test Package for Bank & Insurance Exams
If there are any suggestions/ errors in our PDFs Feel Free to contact us via this email: admin@exampundit.in
Ultra Practice Bundle PDF
SBI Clerk/ RBI Assist. Mains – Reasoning
49. C
50. A

The first letter in the code is as per following rules:


a. If the number of letters in the word is even, the 2nd letter from the left of the word is written.
b. If the number of letters in the word is odd, the 2nd letter from the right of the word is written.
The digit in the code is the number of consonants in the word.
The symbol in the code is as per the number letters in the word:
a. If number of letters is 3, the symbol used is @.
b. If number of letters is 4, the symbol used is #.
c. If number of letters is 5, the symbol used is $.
d. If number of letters is 6, the symbol used is %.
e. If number of letters is 7, the symbol used is &.
f. If number of letters is 8, the symbol used is *.
The last letter in the code is the immediate succeeding letter (in alphabetical series) of the last letter of the
word.
Example:
‘PREACH’
As the number of letters in the word is 4, so the first letter in its code is 2nd letter of the word i.e. R
Number of consonants in the word is 4.
Number of letters in the word is 6, so the symbol used is %.
Last letter of the word is H, so the last letter in the code is immediate succeeding letter of H i.e. I.
Therefore, the code for ‘PREACH’ is ‘R4%I’

Coding Decoding New Pattern 2


Directions (1-5) Study the following information to “Prime minister is Modi” is written as “%@ #@ $&
answer the following questions. @%”
In a certain code, “Adithyanath known as Yogi” is written as “&% %&
“Yogi became chief minister” is written as “@# %@ #% &$ @#”
%#”
Page 1302 of 1334
Subscribe The Xpress Video Course & Mock Test Package for Bank & Insurance Exams
If there are any suggestions/ errors in our PDFs Feel Free to contact us via this email: admin@exampundit.in
Ultra Practice Bundle PDF
SBI Clerk/ RBI Assist. Mains – Reasoning
“Modi is chief of BJP” is written as “%# @% #@ @& b. %@
&@” c. $&
Where codes are group of 2 symbols. d. Can’t be determine
1) How “Adithyanath is Yogi” possible will be coded? e. Either %@ or #@
a. %@ %@ &$ Directions (6-10): Study the following information
b. &% @% @# carefully and answer the questions given below.
c. &$ %@ @# In coded language
d. %@ &@ &$ “tradition festival iconic” is coded as – ‘8X 9J 6XJ’
e. None of these “aesthetic recreate vibe” is coded as – ‘8E 9VJ 4W’
2) What is code for word “Prime”? “creative emerging shine” is coded as – ‘8NO 5K 8C’
a. #@ 6) What can be the code of ‘during autumn’?
b. %@ a. 6M 7FN
c. $& b. 6M 6FN
d. Can’t be determine c. 6K 6FN
e. Either %@ or #@ d. 6K 6EM
3) What is code for word “Becomes”? e. None of these
a. #% 7) What can be the code of ‘Impulse Response’?
b. %@ a. 7NT 7F
c. $& b. 8T 7NR
d. Can’t be determine c. 8F 7MT
e. Either %@ or #@ d. 7NT 8F
4) “%# @% #@” will be code of? e. None of these
a. Chief of BJP 8) What can be the code of ‘Vibrant Ocean’?
b. Modi is BJP a. 5XB 7J
c. Modi is chief b. 5XB 7K
d. Can’t be determine c. 5XZ 7M
e. None of these d. 5YB 7K
5) What is code for word “BJP”? e. None of these
a. #@ 9) What can be the code of ‘Ideal Journey’?

Page 1303 of 1334


Subscribe The Xpress Video Course & Mock Test Package for Bank & Insurance Exams
If there are any suggestions/ errors in our PDFs Feel Free to contact us via this email: admin@exampundit.in
Ultra Practice Bundle PDF
SBI Clerk/ RBI Assist. Mains – Reasoning
a. 5WC 7T a. M#19
b. 5FB 7T b. M#18
c. 5WC 7V c. M@18
d. 5FB 7V d. N#18
e. 5WB 7T e. None of these
10) What can be the code of ‘Enough Rise’? 13) What may be the possible code for ‘ONWARD’ in
a. 6NF 4F the given code language?
b. 6MH 4F a. I#12
c. 6MF 4D b. M#23
d. 6MH 4H c. I@12
e. None of these d. I@15
Directions(11-15): Study the following and answer the e. None of these
following questions: 14) What is the code for ‘RETURNED’ in the given
In a certain code language code language?
“START HER FOOD IN” is coded as “I#8 V#19 L@21 a. R@19
R@18”. b. V@9
“THE HUMAN MAKE PRODUCTION” is coded as c. I#5
“S#7 Z#19 P@14 L@11”. d. X#8
“HAIR RETURNED VICTORY SILENCE” is coded as e. None of these
“R@19 V@9 I#5 X#8”. 15) What is the code for ‘PRODUCTION’ in the given
11) What is the possible code for ‘HANDSOME’ in the code language?
given code language? a. S#7
a. N@19 b. Z#19
b. M#8 c. P@14
c. N#19 d. L@11
d. M@8 e. None of these
e. None of these Directions(16-20): Study the following information
12) What may be the possible code for ‘EXCLUDING’ and answer the given below questions.
in the given code language? In a certain code language

Page 1304 of 1334


Subscribe The Xpress Video Course & Mock Test Package for Bank & Insurance Exams
If there are any suggestions/ errors in our PDFs Feel Free to contact us via this email: admin@exampundit.in
Ultra Practice Bundle PDF
SBI Clerk/ RBI Assist. Mains – Reasoning
‘garden tangle soaps paper’ is written as ‘@E6 &R5 %N6 b. %D4
#S5’. c. @Y6
‘great teeth solution paste’ is written as ‘#N8 @H5 %T5 d. &K4
&E5’. e. None of these
‘super pink gold theory’ is written as ‘#R5 %D4 @Y6 20) Which of the following is code for “‘glory”?
&K4’. a. &M4
‘glory soup poem tongue’ is written as ‘&M4 #P4 %Y5 b. #P4
@E6’. c. %Y5
16) Which of the following is code for “pink solution”? d. @E6
a. &K4, #N8 e. None of these
b. &P4, &N6 Directions (21-25): Study the following information
c. #N8, %K4 and answer the given below questions.
d. %T5, #R5 In a certain code language
e. Can’t be determined ‘Pollution is root of everything’ is written as ‘G&10 S^2
17) In the given code language, what does the code F)2 T*4 N?9’.
“%T5” means? ‘Ramesh have everything he wants’ is written as ‘H%6
a. Teeth G&10 E#4 E#2 S!5 ’.
b. Theory ‘This tree have long root’ is written as ‘E~4 S@4 E#4 T*4
c. Tongue G|4’.
d. Great ‘Ramesh have short hair’ is written as ‘E#4 T$5 H%6
e. Paste R#4’.
18) Which of the following is code for “tongue soaps”? 21) What is the code for “hair”?
a. %E6, #S5 a. E#4
b. @E6, &S5 b. E^4
c. @E6, #S5 c. R#4
d. #E6, @S5 d. R@4
e. @F, #T5 e. R!4
19) Which of the following is code for “gold”? 22) What is the code for “everything”?
a. #R5 a. G&10

Page 1305 of 1334


Subscribe The Xpress Video Course & Mock Test Package for Bank & Insurance Exams
If there are any suggestions/ errors in our PDFs Feel Free to contact us via this email: admin@exampundit.in
Ultra Practice Bundle PDF
SBI Clerk/ RBI Assist. Mains – Reasoning
b. G%10 people have become crazy’ is written as ‘5yB 4eG 6eO
c. E#10 6eA’‘
d. E&10 more public health services’ is written as ‘6hG 8sR 4eL
e. N@10 6cO’
23) In the given code language, what does the code 26) Which of the following is code for “more floods
“E~4” means? occur”?
a. Have a. 4Le 6Se 5Rn
b. This b. 4eL 6sG 5rN
c. Tree c. 4eL 6sE 5rN
d. Hair d. 4rX 6eA 5yD
e. Can’t be determined e. Can’t be determined
24) What is the code for “‘Ramesh”? 27) In the given code language, what does the code
a. E#4 “3eS 9yM 8sR” means?
b. T$5 a. the public services
c. H%6 b. every necessary services
d. R#4 c. the necessary services
e. None of these d. more necessary services
25) What is the code for “Pollution”? e. Can’t be determined
a. G&10 28) In the given code language, what does the code
b. S^2 “5yD 6eO” means?
c. F)2 a. Public occur
d. T*4 b. Crazy people
e. N?9 c. Every public
Directions (26-30): Refer to the data below and answer d. Every people
the questions that follow. e. Can’t be determined
In a certain code language, 29) Which of the following is code for “necessary
‘lack the necessary medical’ is written as ‘4kK 7lL 3eS health year”?
9yM’‘ a. 9yM 6hG 4rZ
floods occur every year’ is written as ‘4rX 5rN 5yD 6sE’‘ b. 9yN 6hG 4rZ

Page 1306 of 1334


Subscribe The Xpress Video Course & Mock Test Package for Bank & Insurance Exams
If there are any suggestions/ errors in our PDFs Feel Free to contact us via this email: admin@exampundit.in
Ultra Practice Bundle PDF
SBI Clerk/ RBI Assist. Mains – Reasoning
c. 9yM 6hI 4rX e. M23Z
d. 9yM 6hG 4rX 33) What is the code for ‘Announced?
e. Can’t be determined a. E21O
30) Which of the following is code for “occur”? b. E21M
a. 4rX c. D21M
b.5rN d. B20M
c. 5yD e. None of these
d. 6sE 34) What is the code for ‘Celebrate ‘?
e. None of these a. E21O
Directions (31-35): Study the following information b. E21M
carefully and answer the given questions. c. D20H
In a certain code language d. B25T .
“Lynch Called Songs” is coded as “S25Z B19O C12M”. e. E20M
“Rhythm Twenty Identical” is coded as “N25I E20F 35) What is the code for ‘Songs?
B25F” a. S25Z
“System Together Celebrate” is coded as “D20H B25T b. B19O
E20M”. c. C12M
d. B20M
31) What is the code for ‘Simplify? e. None of these
a. D21L Directions (36-40): Study the following information
b. C25L carefully and answer the given questions.
c. C25N In a certain code language: -
d. B22L “case draft lock” is written as ‘T@X G#EG D@O’
e. None of these “serious blow white” is written as ‘H#TV P@Y V#SU’
32) What is the code for ‘Hymn? “pride kite null” is written as ‘V#CE U@P M@M’
a. N25Z “green very soon” is written as ‘M#DF S@E P@H’
b. Z25M 36) In the given coding language, which of the
c. X23H following will be the code for “announce”?
d. M25Z a. D@V

Page 1307 of 1334


Subscribe The Xpress Video Course & Mock Test Package for Bank & Insurance Exams
If there are any suggestions/ errors in our PDFs Feel Free to contact us via this email: admin@exampundit.in
Ultra Practice Bundle PDF
SBI Clerk/ RBI Assist. Mains – Reasoning
b. V#BD c. V#SU
c. D#BV d. D@Z
d. D@Z e. V@XZ
e. V@XZ Directions (41-45): Refer to the data below and answer
37) In the given coding language, which of the the questions that follow.
following will be the code for “drone”? ‘winter happy feeling’ is coded as ‘ I13% E11@ O22# ’
a. M@W ‘summer heat mountain is coded as ‘ I14% A26% U6& ’
b. M#VW ‘raining queueing clown’ is coded as ‘ O18# U22$ E12@
c. V@M ‘
d. V#MO 41) What is the code for ‘Maximize’ in the given code
e. V@O language?
38) Which of the following words could be coded as a. E22#
“M#VX”? b. O3$
a. Magnet c. U22#
b. Sword d. U3&
c. Known e. None of these
d. Gone 42) What may be the possible code for ‘Veracity’ in the
e. None of these given code language?
39) In the given coding language, which of the a. U9#
following will be the code for “soon”? b. O5$
a. D@V c. E9#
b. V#BD d. A2&
c. M#DF e. None of these
d. S@E 43) What may be the possible code for ‘Naval army’
e. P@H in the given code language?
40) In the given coding language, which of the a. E14% A13&
following will be the code for “blow”? b. E5% A14@
a. H#TV c. O5% E13@
b. P@Y d. A14@ I13$

Page 1308 of 1334


Subscribe The Xpress Video Course & Mock Test Package for Bank & Insurance Exams
If there are any suggestions/ errors in our PDFs Feel Free to contact us via this email: admin@exampundit.in
Ultra Practice Bundle PDF
SBI Clerk/ RBI Assist. Mains – Reasoning
e. None of these e. OJ
44) What is the code for ‘College days' in the given 47) What is the code for "audio"?
code language? a. QF
a. I12% E2@ b. PB
b. E15@ A2& c. TJ
c. O15# A2@ d. TT
d. A2# O15$ e. NZ
e. None of these 48) What word does the code FD denote?
45) What is the code for ‘clown’ in the given code a. take
language? b. creep
a. E22# c. cake
b. O3$ d. grass
c. O18# e. key
d. U22$ 49) What is the code for "Mango"?
e. E12@ a. QF
Directions (46-50): Refer to the data below and answer b. UG
the questions that follow. c. BF
In a certain code language, d. TJ
“Sun sets in the west” is coded as “UX FU OJ TT OT”. e. PN
“Vanilla is a flavour” is coded as “SG B TJ BW”. 50) What is the code for "Sun"?
“Mango is a fruit” is coded as “UG B TJ PN”. a. UX
46) What is the code for "fruit"? b. FU
a. UG c. OJ
b. TJ d. TT
c. PN e. OT
d. B

Coding Decoding New Pattern 2 – Answer and Explanation


Solution(1-5):

Page 1309 of 1334


Subscribe The Xpress Video Course & Mock Test Package for Bank & Insurance Exams
If there are any suggestions/ errors in our PDFs Feel Free to contact us via this email: admin@exampundit.in
Ultra Practice Bundle PDF
SBI Clerk/ RBI Assist. Mains – Reasoning

1. E 8. B
2. C 9. E
3. A 10. B
4. C Solution(11-15):
5. D Here, we are applying following concept: -
Solution(6-10):
Let us understand the logic behind the given coding
decoding
Case-1: If the first letter of the code is consonant-
a. Second last letter of the word changed to second
preceding letter of the opposite letter.
b. Number – Total number of letters in the word. 11. A
For Ex- FESTIVAL -8X 12. E
Case-2: If the first letter of the code is vowel. 13. C
a. Second letter of the word changed to opposite letter of 14. B
the same. 15. D
b. Second last letter of the word changed to next letter of Solution(16-20):
the same. 1. 1st element is symbol that represents first letter of word.
c. Number – Total number of letters in the word. 2. 2nd element is letter that represent last letter of word.
For Ex- iconic- 6XJ 3. 3rd element is number that represents number of letter
6. C in word.
7. D Ex: &K4 for “pink”,

Page 1310 of 1334


Subscribe The Xpress Video Course & Mock Test Package for Bank & Insurance Exams
If there are any suggestions/ errors in our PDFs Feel Free to contact us via this email: admin@exampundit.in
Ultra Practice Bundle PDF
SBI Clerk/ RBI Assist. Mains – Reasoning
K is the last letter in the word pink. 25. E
4 is number of letters in word. Solution(26-30):
& is the symbol that represent first letter of word “pink” 1) The first element is a number which represents the
which is “p”. number of letter in that word.
#N8 for “solution”, 2) The second element is the letter which represents the
N is the last letter in the word plan. last letter of word.
8 is number of letters in word. 3) The third element is a letter which represents before
# is the symbol that represent first letter of word letter in the capital form of the first letter of the word.
“solution” which is “s” Ex: 9yM for “necessary”.
16. A 9 is number of letters in word.
17. D “y” is the last letter of the word.
18. C M is the before letter in the capital form of the first letter
19. B i.e. “n” of the word.
20. C 26. C
Solution(21-25): 27. C
1) 1st element is letter that represents last letter of word. 28. D
2) 3nd element is number that represents number of letter 29. D
in word. 30. B
3) 2rd element is the symbol that represents the first letter Solution(31-35):
of the word. For the digit of the code – Number in the code will be the
Ex: G&10 for “everything”. place value of the highest place value of letter present in
G is the last letter in the word plan. the word.
10 is number of letters in word. For the first letter of the code-
& is the symbol that represent first letter of word Case-I
“everything” which is “e”. If the given word has some vowels, then the code will be
21. C according to the given order:
22. A No. of vowels Code
23. C in the word
24. C 1 B

Page 1311 of 1334


Subscribe The Xpress Video Course & Mock Test Package for Bank & Insurance Exams
If there are any suggestions/ errors in our PDFs Feel Free to contact us via this email: admin@exampundit.in
Ultra Practice Bundle PDF
SBI Clerk/ RBI Assist. Mains – Reasoning
2 C
3 D
4 E

For ex- Arrive -In this word three vowels letters ‘a, e and
i’. So the first letter of code for Arrive is ‘D ’.
For the last letter of the code – The immediate succeeding
letter of the third letter present in the word according to
the English alphabet.
Case II: 36. D
If the given word has no vowel letter, then first letter of 37. D
the code will be coded as opposite letter of the last letter 38. C
of the word. 39. E
For the last letter of the code – The immediate succeeding 40. B
letter of the second letter present in the word according to Solution(41-45):
the English alphabet. Symbol: Symbols in the code are used according to
31. C number of vowels present in the word i.e.
32. D 1 vowel- @
33. A 2 Vowels - %
34. E 3 Vowels - #
35. B 4 vowels -&
Solution(36-40): 5 vowels -$
Vowels: Vowels in the code are used according to number
of letters in the word
4 letters- A
5 letters- E
6 letters- I
7 letters- O
8 letters- U

Page 1312 of 1334


Subscribe The Xpress Video Course & Mock Test Package for Bank & Insurance Exams
If there are any suggestions/ errors in our PDFs Feel Free to contact us via this email: admin@exampundit.in
Ultra Practice Bundle PDF
SBI Clerk/ RBI Assist. Mains – Reasoning
Number: Numbers in the code are used according to place 4. The whole code is in the reverse order of the sentence.
value of the reverse letter of the 3rd letter of the word.
41. D
42. A
43. B
44. C
45. E
46. A
Solution(46-50):
47. B
1. The first letter of the code denotes the alphabet that
48. C
comes after the last letter of the word.
49. E
2. The last letter of the code denotes the alphabet that
50. E
comes after the first letter of the word.
3. The code words arranged in the reverse order of the
sentence.

Coded Direction
Direction 1-5: Read the following information A % B means A is to the south direction of B at
carefully and answer the given questions. a distance of either 4m or 9m
A $ B means A is to the north of B at a distance A # B means A is to the east direction of B at a
of either 4m or 9m distance of either 3m or 12m
A & B means A is to the west direction of B at a A $# B means A is to the north-east of B
distance of either 3m or 12m

Page 1313 of 1334


Subscribe The Xpress Video Course & Mock Test Package for Bank & Insurance Exams
If there are any suggestions/ errors in our PDFs Feel Free to contact us via this email: admin@exampundit.in
Ultra Practice Bundle PDF
SBI Clerk/ RBI Assist. Mains – Reasoning
A %# B means A is to the south-east direction of 5) If C & J, then what is the total distance
B between J and D. (It is given that AF=CJ)
Statement a. 5m
B$A, C&B, D%C, D$&A, D&H&G, B$H, b. 15m
A&F, (CB>AF) c. 4m
1) In which direction is C with respect to F? d. 12m
a. North e. None of these
b. East Directions 6-10: Study the information below
c. North-East and answer the given questions.
d. North-West A%254m B – A is 250m east of B.
e. None of these A&116mB – A is 126m west of B.
2) What is the distance between D and G? A?96mB – A is 90m south of B.
a. 10 m A@220mB – A is 225m north of B.
b. 28 m Point M is & -4m point N, which is @3m point
c. 9m O, which is %16m point P, which is ?14m
d. 15 m point Q, which is %10m point R, which is
e. 14 m ?12m point S, which is &-4m point T.
3) What is the total distance between F and G? 6) What is the shortest distance between point
a. 5m R and point T (approx)?
b. 15m a. Less than 15m
c. 4m b. Between 15m and 17m
d. 12m c. Between 17m and 19m
e. None of these d. More than 19m
4) In which direction is B with respect to D? e. None of these
a. North 7) What is the shortest distance between point
b. East M and point P?
c. North-East a. 7m
d. North-West b. 8m
e. None of these c. 10m

Page 1314 of 1334


Subscribe The Xpress Video Course & Mock Test Package for Bank & Insurance Exams
If there are any suggestions/ errors in our PDFs Feel Free to contact us via this email: admin@exampundit.in
Ultra Practice Bundle PDF
SBI Clerk/ RBI Assist. Mains – Reasoning
d. 12m X @ P (25m), W # V (18m), R % S (18m), Q
e. None of these % P (22m), T # U (19m), Q & S (20m), W & U
8) In which direction is point S with respect to (28m), T @ R (25m)
point O? 11) If Z % X (33m) then what is sum of the
a. North shortest distance between S and T and the
b. North-East shortest distance between Z and U?
c. South-East a. 20m
d. North-West b. 25m
e. None of these c. 29m
9) What is the distance between point R and d. 32m
point N? e. None of these
a. 7m 12) If R # K (19m), which among the following
b. 8m statements is true?
c. 18m a. X # A (33m), K @ A (4m)
d. 12m b. B @ V (16m), K # B (18m)
e. None of these c. D & X (18m), K % D (20m)
10) In which direction is point P with respect d. F % Q (24m), F & K (19m)
to point N? e. None is true.
a. North 13) If P # Q (25m), S & T (19m), Q @ R (31m)
b. North-East and S % R (17m) then what is the shortest
c. South-West distance between P and T?
d. North-West a. 24m
e. None of these b. 18m
Directions 11-15: Study the information below c. 20m
and answer the questions. d. 22m
M & N (89m) – M is 76m south of N. e. None of these
M % N (52m) – M is 39m east of N. 14) What is the direction of S with respect to
M @ N (110m) – M is 97m north of N. P?
M # N (58m) – M is 45m west of N. a. North

Page 1315 of 1334


Subscribe The Xpress Video Course & Mock Test Package for Bank & Insurance Exams
If there are any suggestions/ errors in our PDFs Feel Free to contact us via this email: admin@exampundit.in
Ultra Practice Bundle PDF
SBI Clerk/ RBI Assist. Mains – Reasoning
b. North-East 16) If N#M, N@O $ K, RV > NO and the point
c. South-West M is exactly to the east of U and also to the
d. North-West South of T. Then what is the shortest distance
e. None of these between Point S and Point O?
15) What is the total distance between S and a. √65
T? b. √137
a. 14m c. Cannot be determined
b. 18m d. Either (A) or (B)
c. 13m e. √70
d. 22m 17) In which direction T with respect to V?
e. None of these a. North
16-20) In certain coding language, the b. North-East
directions are coded as per below conditions. c. South-West
A@B means – A is North of B d. North-West
A%B means – A is South of B e. None of these
A#B means – A is East of B 18) If R%B, RB<QT and RB= QU, than what
A $ B means – A is West of B is the total distance between QT and QU?
@ and $ means the distance between the two a. 14m
points is either 4m or 8m. b. 10m
% and # means the distance between the two c. 6m
points is either 3m or 7m. d. either a or b
AB > CD Means the distance between point A e. None of these
and B is greater than that of point C and D. 19) If SW> WP, than what is the total distance
Example: A @ B means A is north of B and the between RU and SW?
distance between A and B is either 4m or 8m and a. 8m
so on. b. 12m
Condition: c. 16 m
T#Q, U%Q, R@V, S#V, R $ U, S@W, P#W, d. 11m
TQ > WP, VS > RU e. None of these

Page 1316 of 1334


Subscribe The Xpress Video Course & Mock Test Package for Bank & Insurance Exams
If there are any suggestions/ errors in our PDFs Feel Free to contact us via this email: admin@exampundit.in
Ultra Practice Bundle PDF
SBI Clerk/ RBI Assist. Mains – Reasoning
20) In which direction R with respect to P? a. 12 m
a. North b. 13 m
b. North-East c. 6 m
c. South-West d. 4 m
d. North-West e. 10 m
e. None of these 23) B @ T * F % G * R, then T is in which
21-25) Direction 1-5: Read the following direction with respect to R?
information carefully and answer the given a. South
questions. b. North-West
A * B means A is to the left of B at a distance of c. West
5m. d. North-East
A # B means A is to the south direction of B at a e. East
distance of 3m. 24) A # F * P @ D # G then find the distance
A @ B means A is to the right of B at a distance between A and D?
of 2m. a. 2 m
A % B means A is to the north direction of B at b. 3 m
a distance of 4m. c. 3√2 m
In each of the following questions initially, all d. 4 m
persons are facing north. e. None of these
21) Q @ Y % R # L @ K, then in which 25) B @ T * F % G * R, then what is the
direction is K with respect to R? distance between B and G?
a. North a. 3m
b. East b. 4m
c. Right c. 5m
d. North-West d. 2m
e. None of these e. None of these
22) Z % Y, F * Z, Y % G, D * F then find the Direction 26-30: Read the following
minimum distance between G and D information carefully and answer the given
(approx)? questions.

Page 1317 of 1334


Subscribe The Xpress Video Course & Mock Test Package for Bank & Insurance Exams
If there are any suggestions/ errors in our PDFs Feel Free to contact us via this email: admin@exampundit.in
Ultra Practice Bundle PDF
SBI Clerk/ RBI Assist. Mains – Reasoning
A @ B means A is to the north of B at a distance 28) What is the total distance between E and
of either 7m or 12m G?
A # B means A is to the west direction of B at a a. 12 m
distance of either 5m or 14m b. 17 m
A $ B means A is to the south direction of B at a c. 19 m
distance of either 7m or 12m d. 21 m
A & B means A is to the east direction of B at a e. None of these
distance of either 5m or 14m 29) In which direction is G with respect to D?
A @& B means A is to the north-east of B a. North
A $& B means A is to the south-east direction of b. East
B c. North-East
Statement :A@B, B#C, C$D@&A, d. North-West
D#E@F@&C, F@G$&C, G&H$C e. South east
26) In which direction is B with respect to G? 30) What is the total distance between D and
a. North H?
b. East a. 12 m
c. North-East b. 17 m
d. North-West c. 19 m
e. None of these d. 21 m
27) If distance between B and C is same as the e. None of these
distance between D and E and this distance is 31-35) Directions: Study the information
less than the distance between A and B then below and answer the questions.
what is the total distance between BC and DE? Y # Z (129m) – Y is 137m west of Z.
a. 10 m Y & Z (111m) – Y is 123m north of Z.
b. 28 m Y $ Z (106m) – Y is 99m east of Z.
c. Either a or b Y @ Z (97m) – Y is 87m south of Z.
d. 5 m A & P (11m), P # C (9m), C @ B (56m), T $ E
e. 14 m (33m), D @ R (33m), B # Q (22m), S $ R (27m),
E & S (21m), B @ T (20m)

Page 1318 of 1334


Subscribe The Xpress Video Course & Mock Test Package for Bank & Insurance Exams
If there are any suggestions/ errors in our PDFs Feel Free to contact us via this email: admin@exampundit.in
Ultra Practice Bundle PDF
SBI Clerk/ RBI Assist. Mains – Reasoning
31) If K @ B (33m), which among the b. East
following statement is true? c. North-East
a. S # K (26m) d. North-West
b. K $ A (24m) e. South West
c. K @ C (33m) Directions 36-40: Study the information below
d. K @ S (42m) and answer the following questions:
e. None is true. P @6m Q – P is 3m east of Q.
32) What is the shortest distance between P *8mQ – P is 5m west of Q.
point P and point D? P &7mQ – P is 4m south of Q.
a. 23m P$10mQ – P is 7m north of Q.
b. Between 24m and 29m R is & 6m of Q. S is $7m of T. V is $10m of U.
c. 29m U is *6m of T. S is *7m of R. V is *6m of W. Q
d. More than 29m is @5m of P.
e. None of these 36) What is the shortest distance between W
33) In which direction R with respect to Q? and T?
a. North a. 5m
b. East b. 6m
c. North-East c. 8m
d. North-West d. 7m
e. South West e. None of these
34) If M&R (21), than distance between M and 37) What is the total distance between V and
R is equal to distance between? Q?
a. R and D a. 5m
b. E and S b. 6m
c. A and P c. 7m
d. B and C d. 8m
e. None of these e. 4m
35) In which direction B with respect to P?
a. North 38) In which direction is Q with respect to U?

Page 1319 of 1334


Subscribe The Xpress Video Course & Mock Test Package for Bank & Insurance Exams
If there are any suggestions/ errors in our PDFs Feel Free to contact us via this email: admin@exampundit.in
Ultra Practice Bundle PDF
SBI Clerk/ RBI Assist. Mains – Reasoning
a. North 41) If Point K is #7m of point T then which of
b. East the following is the position of K with respect
c. North-East to F?
d. North-West a. @, 24m
e. South West b. &, 10 m
39) If R$7Mm, than what is the area of square c. #, 15 m
STMR? d. $, 10m
a. 36m e. None of these
b. 9m 42) Point B is in which direction from point V?
c. 4m a. #
d. 16m b. @$
e. None of these c. #$
40) In which direction is V with respect to T? d. @&
a. North e. #&
b. East 43) Point C is in which direction from point
c. North-East G?
d. North-West a. #
e. South West b. @
41-45) Study the information below and c. $
answer the following questions d. &
P#Q - P is in the south direction of Q. e. #&
P@Q - P is in the north direction of Q. 44) What is the distance between point G and
P&Q - P is in the east direction of Q. point Q?
P$Q - P is in the west direction of Q. a. 6m
P£QS- P is the mid-point of QS vertically. b. 5m
G is #5m of B. T is &20m of G. Point T is c. 8m
@14m of V. C is $20m of V. Q is @7m of C. F d. 7m
is &10m Q. U£GT. e. 10m

Page 1320 of 1334


Subscribe The Xpress Video Course & Mock Test Package for Bank & Insurance Exams
If there are any suggestions/ errors in our PDFs Feel Free to contact us via this email: admin@exampundit.in
Ultra Practice Bundle PDF
SBI Clerk/ RBI Assist. Mains – Reasoning
45) What is the distance between point B and 47) If there is a landmark X at 4m south to J,
point U? then if a bike moves from F to G, then G to X,
a. 5m then X to I, what is the total distance covered
b. 5√5m by the bike?
c. 3√5m a. 24m
d. 7m b. 29m
e. None of these c. 32m
46-50) Study the information below and d. 30m
answer the following questions e. None of the above
P % Q – P is north of Q. 48) If Vivek moves 17m towards west from H,
P # Q – P is south of Q. then how far he is from A?
P @ Q – P is east of Q. a. 10m
P$ Q – P is west of Q. b. 7m
P * QR – P is midpoint of vertical straight line c. 8m
QR. d. 5m
P€ QR – P is midpoint of horizontal straight line e. None of these
QR. 49) In which direction is D with respect to J?
A is 6m%B. C*AB. D is5m@C. E is9m#D. F a. North
is6m@E.G is4m%F. H€GI.J is4m#H. G b. East
is12m$I. c. North-East
46) A car moves from B to C, then C to D and d. North-West
then D to J, what is the total distance covered e. South West
by the car? 50) If Rahul moves from point G to point F
a. 19m and then to point E, what is total distance
b. 23m covered by Rahul?
c. 20m a. 4m
d. 28m b. 8m
e. None of these c. 10m
d. 12m

Page 1321 of 1334


Subscribe The Xpress Video Course & Mock Test Package for Bank & Insurance Exams
If there are any suggestions/ errors in our PDFs Feel Free to contact us via this email: admin@exampundit.in
Ultra Practice Bundle PDF
SBI Clerk/ RBI Assist. Mains – Reasoning
e. None of these

Coded Direction – Answer and Explanation


Solution 1-5
B$A, C&B, D%C, D$&A, D&H&G, B$H,
A&F, (CB>AF)
1. d. North-West
2. d. 15 m
3. a. 5m
4. c. North-East
5. a. 5m
1) B$A means B is either 4 or 9m north of A.
2) C&B means C is either 3 or 12m west of B. Solution 5
3) D%C means D is either 4 or 9m south of C. AF=CJ means CJ =3m also we know CD=4m
But it is also given that D$&A that means D is Hence from Pythagoras theorem
north-west of A. JD=5m
If we apply the above condition than distance Solution 6-10
between D and C must be 4m and Distance
between B and A must be 9m.
4) B$H means B is either 4 or 9m north of H but
as only point A is 9m south of B. Both point H
and A can’t be at same point.
So Point H is 4m south of B or between B and A.
5) A&F means A is either 3 or 12m west of F. 6. a. Less than 15m
6) CB>AF means distance between CB is greater 7. c. 10m
than AF so CB= 12m and AF=3m 8. d. North-West
9. c. 18m
10. c. South-West

Solution 11
Page 1322 of 1334
Subscribe The Xpress Video Course & Mock Test Package for Bank & Insurance Exams
If there are any suggestions/ errors in our PDFs Feel Free to contact us via this email: admin@exampundit.in
Ultra Practice Bundle PDF
SBI Clerk/ RBI Assist. Mains – Reasoning
M & N (89m) – M is 76m south of N, here (89 – As given in the code,
76) = 13m X @ P (25m), W # V (18m), R % S (18m), Q %
M % N (52m) – M is 39m east of N, here (52 – P (22m), T # U (19m), Q & S (20m), W & U
39) = 13m (28m), T @ R (25m)
M @ N (110m) – M is 97m north of N, here (110 The direction diagram for the above code is given
– 97) = 13m. below:
M # N (58m) – M is 45m west of N, here (58 –
45) = 13m.
The actual distance between M and N is 13m less
than the distance given in the code.
As given in the code,
X @ P (25m), W # V (18m), R % S (18m), Q %
P (22m), T # U (19m), Q & S (20m), W & U
(28m), T @ R (25m)
As Z % X (33m), so Z is 20m east of X, thus, the As B @ V (16m), K # B (18m), so B is 3m north
shortest distance between U and Z is 7m. of V and K is 5m west of B.
Hence, option b is correct

Solution 13

As P # Q (25m), S & T (19m), Q @ R (31m) and


S % R (17m), so the direction diagram can be:

The shortest distance between S and T


= √ [(122 ) + (52 )] m
= 13m.
Sum = (13 + 7) = 20m
Hence, option a is correct
Solution 12

Page 1323 of 1334


Subscribe The Xpress Video Course & Mock Test Package for Bank & Insurance Exams
If there are any suggestions/ errors in our PDFs Feel Free to contact us via this email: admin@exampundit.in
Ultra Practice Bundle PDF
SBI Clerk/ RBI Assist. Mains – Reasoning

Hence in triangle SRT, SR= 5m, RT= 12m


And 𝐒𝐓 𝟐 =𝟏𝟐𝟐 + 𝟓𝟐 = √144+25= √𝟏𝟔𝟗 = 13m
The shortest distance between P and T Solution 16
= √[(16)2 + (12)2]m
= 20m
Hence, option c is correct
Solution 14
b. North-East

Hence applying Pythagoras theorem,


Case-1
SO= √72 + 42 = √65
Case-2
SO = √112 + 42 = √137
Solution 15
Hence option D is correct

Solution 17
b. North-East

Page 1324 of 1334


Subscribe The Xpress Video Course & Mock Test Package for Bank & Insurance Exams
If there are any suggestions/ errors in our PDFs Feel Free to contact us via this email: admin@exampundit.in
Ultra Practice Bundle PDF
SBI Clerk/ RBI Assist. Mains – Reasoning

Solution 20
d. North-West
Solution 18
Hence, R%B is R is south of B either 3 or 7m
RB<QT means RB distance is less than QT
which is 7m
Hence, RB=3m and RB=QU=3m Hence,
QT+QU =10m

Solution 21:
Q 2m right of Y who is 4m north of R who is 3m
south of L who is 2 m right of K

Solution 19
Hence distance between SW is either 4 or 8m and
distance between RU is 4m.
But WP=4m so SW becomes 8m
Hence, RU+SW=12m

Page 1325 of 1334


Subscribe The Xpress Video Course & Mock Test Package for Bank & Insurance Exams
If there are any suggestions/ errors in our PDFs Feel Free to contact us via this email: admin@exampundit.in
Ultra Practice Bundle PDF
SBI Clerk/ RBI Assist. Mains – Reasoning
Hence K is North West direction with respect to
R.

Solution 22:

Distance between AF=3m and FD= 3m


By Pythagoras theorem,
AD2 = AF 2 + FD2
AD2 = 32 + 32 = 9 + 9 = 18
AD = 3√2
Hence, option C is correct.
Solution 25

GZ = 8m and DZ = 10m.
Therefore, GD =√102 +82 =√=13(approx)
Hence, the minimum distance between G and D
is 13m.
Solution 23.
Distance between BF=3m and FG =4 m
By Pythagoras theorem,
BG2 = BF 2 + FG2
BG2 = 32 + 42
BG = √9+16
BG =√25 = 5m
Hence, option C is correct
Hence, T is in the North-West direction with
Solution 26
respect to R.
d. North-West
Solution 24

Page 1326 of 1334


Subscribe The Xpress Video Course & Mock Test Package for Bank & Insurance Exams
If there are any suggestions/ errors in our PDFs Feel Free to contact us via this email: admin@exampundit.in
Ultra Practice Bundle PDF
SBI Clerk/ RBI Assist. Mains – Reasoning
c. 19 m

Solution 27
If we apply this condition
Distance between B and C is same as the distance
between D and E and this distance is less than the
Solution 29
distance between A and B,
e. South east
Then, DE=BC=5m
Hence DE+BC=10m

Solution 30
As DH is parallel to EG and EG= 19m
Hence DH= 19m
Solution 28
Page 1327 of 1334
Subscribe The Xpress Video Course & Mock Test Package for Bank & Insurance Exams
If there are any suggestions/ errors in our PDFs Feel Free to contact us via this email: admin@exampundit.in
Ultra Practice Bundle PDF
SBI Clerk/ RBI Assist. Mains – Reasoning

Solution 32

Solution 31
As given in the code,
Y # Z (129m) – Y is 137m west of Z, actually Y
is (129 + 8) = 137m west of Z. Hence we can see distance between P and D is

Y & Z (111m) – Y is 123m north of Z, actually 29m

Y is (111 + 12) = 123m north of Z. Solution 33

Y $ Z (106m) – Y is 99m east of Z, actually Y is e. South West

(106 – 7) = 99m east of Z.


Y @ Z (97m) – Y is 87m south of Z, actually T
is (97 – 10) = 87m south of Z.

Solution 34

K $ A (24m) means K is 17m east of A.


Page 1328 of 1334
Subscribe The Xpress Video Course & Mock Test Package for Bank & Insurance Exams
If there are any suggestions/ errors in our PDFs Feel Free to contact us via this email: admin@exampundit.in
Ultra Practice Bundle PDF
SBI Clerk/ RBI Assist. Mains – Reasoning
M&R 21 means M is north of R and distance
between them is 21+12= 33m which is equal to
distance between E and S

Solution 35 The distance between point W and point T is 7m.


c. North-East

Solution 37

Solution 36
R is & 6m of Q means R is 3m south of Q
S is $7m of T means S is 4m north of T
V is $10m of U means V is 7m north of U
U is *6m of T means U is 3m west of T Hence distance between V and Q is 7m
S is *7m of R means S is 4m west of R Solution 38
V is *6m of W means V is 3m west of W c. North-East
Q is @5m of P means Q is 2 m east of P
Page 1329 of 1334
Subscribe The Xpress Video Course & Mock Test Package for Bank & Insurance Exams
If there are any suggestions/ errors in our PDFs Feel Free to contact us via this email: admin@exampundit.in
Ultra Practice Bundle PDF
SBI Clerk/ RBI Assist. Mains – Reasoning

Solution 39 Solution 41
Hence M will be 4 m south of R G is #5m of B. T is &20m of G. Point T is @14m
of V. C is $20m of V. Q is @7m of C. F is &10m
Q. U£GT.
Point G is 5m south of point B. Point T is 20m
east of point G. Point T is 14m north of point V.
Point C is 20m west of point V. Point Q is 7m
north of point C. Point F is 10m east point Q.
Point U is midpoint of GT.
If Point K is 7m south of point T
2
The area of Square STMR will be 4 = 16m
Solution 40

Hence, K is 10 m east of F, so correct answer is


&, 10 m
Page 1330 of 1334
Subscribe The Xpress Video Course & Mock Test Package for Bank & Insurance Exams
If there are any suggestions/ errors in our PDFs Feel Free to contact us via this email: admin@exampundit.in
Ultra Practice Bundle PDF
SBI Clerk/ RBI Assist. Mains – Reasoning
14m = GQ + 7m
Solution 42 GQ = 7m
b. @$ Solution 45

Solution 43 BG= 5m and GU =10m


By Pythagoras theorem,
BU 2 = BG2 + GU 2
BU 2 = 52 + 102
BU = √25+100= √125
BU = 5√5
Solution 46
Hence, C is south of G, so correct answer is # A is 6m%B. C*AB. D is5m@C. E is9m#D. F
Solution 44 is6m@E.G is4m%F. H€GI.J is4m#H. G
is12m$I.
After decoding the above statements:
A is 6m north to B. C is in the middle of A and
B. D is 5m east to C. D is 9m north to E, which
is 6m west to F, which is 4m south to G, which is
12m west to I and H is in the middle of G and I.
J is 4m south to H.
Hence, QC= 7m and TV =14m and GC = TV The final arrangement is as follows:
So, GC = GQ + QC

Page 1331 of 1334


Subscribe The Xpress Video Course & Mock Test Package for Bank & Insurance Exams
If there are any suggestions/ errors in our PDFs Feel Free to contact us via this email: admin@exampundit.in
Ultra Practice Bundle PDF
SBI Clerk/ RBI Assist. Mains – Reasoning
Solution 48

We can calculate the total distance covered by


the car.
If Vivek moves 17m towards west from H, then
BC+ CD + DJ.
he is 8m far from A.
Distance between D and J can be calculated by
Solution 49
using Pythagoras theorem:
d. North-West
DJ2 = 144 + 81 = 225m
DJ = 15m
Total distance = 3 + 5 + 15 = 23m
Solution 47
FG+ GX + XI
Distance between (G and X) and (X and I) can be
calculated by using Pythagoras theorem:
GX2 = XI2 = 64 + 36 = 100m Solution 50
GX = XI =10m
Total distance = 4 + 10 + 10 = 24m

Hence total distance between GF and EF= 10m

Page 1332 of 1334


Subscribe The Xpress Video Course & Mock Test Package for Bank & Insurance Exams
If there are any suggestions/ errors in our PDFs Feel Free to contact us via this email: admin@exampundit.in
Ultra Practice Bundle PDF
SBI Clerk/ RBI Assist. Mains – Reasoning

Get More Reasoning Practice Questions PDF


Click Here For More SBI Clerk / RBI Assistant Mains Bundle PDF
THE COMPLETE Static GK Capsule for Upcoming Exams
The COMPLETE Static Banking Awareness PDF
Click Here to Join Our What’s App Group & Get Instant Notification on Study
Materials & PDFs
Click Here to Join Our Official Telegram Channel

Page 1333 of 1334


Subscribe The Xpress Video Course & Mock Test Package for Bank & Insurance Exams
If there are any suggestions/ errors in our PDFs Feel Free to contact us via this email: admin@exampundit.in

You might also like